You are on page 1of 0

A 36-year-old man presents to his physician complaining of right scrotal swelling.

He states that the swelling has been present for 1 week. He


initially noticed the swelling shortly after moving furniture for his new living room.
He denies any nausea, vomiting, change in bowel habits,
abdominal pain, or urinary tract symptoms. He has no other significant medical or
surgical history. On examination, he has an enlarged right
hemi-scrotum with a mass that appears to be originating at the level of the
external inguinal ring. With the patient completely relaxed, the
physician is able to reduce the mass by pushing it back through the external
inguinal ring. With the mass reduced, the physician instructs the
patient to perform a Valsalva maneuver, upon which a protrusion is felt at the
external inguinal ring. Once the mass is reduced, the testicle
appears normal in size and consistency.
Question 1 of 4
Which of the following is the most likely diagnosis?
/ A. Hydrocele
/ B. Femoral hernia
/ C. Inguinal hernia
/ D. Testicular cancer
/ E. Varicocele

Explanation - Q: 1.1 Close

The correct answer is C. This patient has a reducible inguinal hernia. A
hernia is a protrusion of a structure, usually intestine, through tissue that
normally contains it. Inguinal hernias are either direct or indirect. Indirect
inguinal hernias occur through the internal inguinal ring in a protrusion of
peritoneum along the spermatic cord in the internal spermatic fascia. Direct
inguinal hernias occur through the floor of the inguinal canal, separate from
the spermatic cord as a result of breakdown of the transversus abdominis
aponeurosis and transversalis fascia. If the mass, i.e., hernia, is easily
returned back to its normal position, then it is called reducible. If the mass is
not reducible, then this is called incarcerated. And, if the mass becomes
incarcerated and develops compromised blood supply, it is termed
strangulated. Strangulated hernias require emergent repair because the
intestinal contents will necrose and cause the patient to become sick. As
long as the mass is reducible, surgical repair can be performed on an
outpatient basis.
A hydrocele (choice A) is a fluid collection contained within the tunica
vaginalis that surrounds the testicle. It presents as a painless swelling of the
scrotum, which transilluminates light when it is placed against the mass. A
hydrocele cannot be reduced.
Femoral hernias (choice B) are more common in women, presenting as
swellings in the upper part of the thigh. The neck of the hernia sac lies at the
femoral ring, below and lateral to the pubic tubercle, distinguishing these
from inguinal hernias, which are above and medial to the tubercle.
A mass caused by testicular cancer (choice D) cannot be reduced either.
Patients will complain of a dull, heavy, aching feeling within the testicle. The
testicle itself is usually enlarged, hard, irregular, and nontender.
A varicocele(choice E) is caused by dilatation of the pampiniform venous
plexus of the spermatic cord. It is typically described as feeling like "a bag of
worms" on physical examination. The dilatation is more easily appreciated
with the patient standing or during the Valsalva maneuver.

A 36-year-old man presents to his physician complaining of right scrotal swelling.
He states that the swelling has been present for 1 week. He
initially noticed the swelling shortly after moving furniture for his new living room.
He denies any nausea, vomiting, change in bowel habits,
abdominal pain, or urinary tract symptoms. He has no other significant medical or
surgical history. On examination, he has an enlarged right
hemi-scrotum with a mass that appears to be originating at the level of the
external inguinal ring. With the patient completely relaxed, the
physician is able to reduce the mass by pushing it back through the external
inguinal ring. With the mass reduced, the physician instructs the
patient to perform a Valsalva maneuver, upon which a protrusion is felt at the
external inguinal ring. Once the mass is reduced, the testicle
appears normal in size and consistency.
Question 2 of 4
Which of the following nerves travels along the spermatic cord within the inguinal
canal and may be damaged during a surgical procedure to
correct this patient's condition?
/ A. IIiohypogastric
/ B. IIioinguinal
/ C. Lateral femoral cutaneous
/ D. Obturator
/ E. Pudendal

Explanation - Q: 1.2 Close

The correct answer is B. Although it is now possible to perform inguinal
hernia repairs laparoscopically, the traditional approach is through an
incision over the inguinal canal. At the time of surgery, the inguinal canal is
opened via sharp dissection through the external oblique aponeurosis (the
anterior wall of the inguinal canal). The ilioinguinal nerve is then encountered
as it runs on the anterior aspect of the spermatic cord. If not properly
identified at time of surgery it is possible to transect the ilioinguinal nerve or
to "trap" it during closure. If the ilioinguinal nerve is transected or entrapped
in closure, the patient will complain of numbness over the nerve's
distribution, i.e., the upper medial aspect of the thigh and the anterior portion
of the scrotum on the affected side.
The iliohypogastric, lateral femoral cutaneous, obturator, and pudendal
nerves are not encountered during traditional inguinal hernia repair.
The iliohypogastric nerve(choice A) is derived from L1 (as is the ilioinguinal
nerve) and runs with the ilioinguinal nerve as they both pierce the
transversus abdominis muscle near the anterior superior iliac spine. They
then pass through the internal and external oblique muscles to supply the
skin of the suprapubic and inguinal regions and the abdominal musculature.
The iliohypogastric sends a lateral branch to the skin of the gluteal region
and then continues on, to pass through the superficial inguinal ring.
The lateral femoral cutaneous nerve (choice C) originates from L2 and L3
and is a direct branch of the lumbar plexus. It enters the thigh deep to the
lateral end of the inguinal ligament near the anterior superior iliac spine and
supplies the skin on the anterior and lateral aspects of the thigh.
The obturator nerve(choice D) is the nerve of the adductor muscles of the
thigh. It arises from the lumbar plexus (L2, L3, L4), enters the pelvis minor,
and then leaves the pelvis via the obturator foramen. The obturator nerve
also sends a small cutaneous branch to the medial aspect of the mid thigh.
The pudendal nerve (choice E) arises from the sacral plexus (S2, S3, S4),
accompanies the internal pudendal artery, and leaves the pelvis between the
piriformis and coccygeus muscles. The nerve hooks around the
sacrospinous ligament to enter the perineum through the lesser sciatic
foramen to supply the muscles of the perineum, including the external anal
sphincter, and then ends as the dorsal nerve of the penis or clitoris. It also
supplies some sensation to the external genitalia
A 36-year-old man presents to his physician complaining of right scrotal swelling.
He states that the swelling has been present for 1 week. He
initially noticed the swelling shortly after moving furniture for his new living room.
He denies any nausea, vomiting, change in bowel habits,
abdominal pain, or urinary tract symptoms. He has no other significant medical or
surgical history. On examination, he has an enlarged right
hemi-scrotum with a mass that appears to be originating at the level of the
external inguinal ring. With the patient completely relaxed, the
physician is able to reduce the mass by pushing it back through the external
inguinal ring. With the mass reduced, the physician instructs the
patient to perform a Valsalva maneuver, upon which a protrusion is felt at the
external inguinal ring. Once the mass is reduced, the testicle
appears normal in size and consistency.
Question 3 of 4
If a segment of terminal ileum becomes strangulated as a consequence of his
condition, it may become infarcted and necrotic due to
occlusion of a branch of which of the following vessels?
/ A. Celiac trunk
/ B. Inferior mesenteric artery
/ C. Middle colic artery
/ D. Right colic artery
/ E. Superior mesenteric artery
Explanation - Q: 1.3 Close

The correct answer is E. The small bowel is a derivative of the midgut and
therefore receives its blood supply from the superior mesenteric artery. This
artery emerges from the aorta 1 cm below the celiac trunk and passes
ventral to the left renal vein to give off 12 to 15 jejunal and ileal arteries. As
these arteries divide, they join with an adjacent branch to form arches. These
arches may then communicate to form an arcade. Straight arteries also
emerge from these arches to supply the bowel. Although there are variants,
the ileal artery is usually a branch of the superior mesenteric artery, which
supplies a branch to the terminal ileum.
The celiac trunk (choice A) provides branches to supply the stomach, liver,
pancreas, and duodenum. It is not responsible for supplying blood to the
terminal ileum.
The inferior mesenteric artery (choice B) provides branches that supply a
limited part of the transverse colon near the splenic flexure and the
descending and sigmoid colon.
The middle colic artery (choice C) is a branch of the superior mesenteric
artery. There are two branches, right and left. The right branch supplies the
right half of the transverse colon and the left branch supplies the left half of
the transverse colon.
The right colic artery (choice D) is also a branch of the superior mesenteric
artery. It is responsible for supplying the hepatic flexure as well as that part
of the ascending colon not supplied by the ileocolic artery.
A 36-year-old man presents to his physician complaining of right scrotal swelling.
He states that the swelling has been present for 1 week. He
initially noticed the swelling shortly after moving furniture for his new living room.
He denies any nausea, vomiting, change in bowel habits,
abdominal pain, or urinary tract symptoms. He has no other significant medical or
surgical history. On examination, he has an enlarged right
hemi-scrotum with a mass that appears to be originating at the level of the
external inguinal ring. With the patient completely relaxed, the
physician is able to reduce the mass by pushing it back through the external
inguinal ring. With the mass reduced, the physician instructs the
patient to perform a Valsalva maneuver, upon which a protrusion is felt at the
external inguinal ring. Once the mass is reduced, the testicle
appears normal in size and consistency.
Question 4 of 4
Which of the following pathological processes might cause the patient's
underlying condition to occur in an infant?
/ A. Defect in the floor of the inguinal canal
/ B. Defect in the internal inguinal ring
/ C. Defect in the linea semilunaris
/ D. Patent processus vaginalis
/ E. Persistent lumen of the tunica vaginalis
Explanation - Q: 1.4 Close

The correct answer is D. The pathologic process that causes hernias is
different in neonates and infants than in an adult. Embryologically, in the
seventh week, the testes begin descending from their location at the 10th
thoracic level into the scrotum. After the eighth week, a peritoneal
evagination called the processus vaginalis forms just anterior to the
gubernaculum. The gubernaculum is a condensation of peritoneum that
attaches superiorly to the gonad and inferiorly to the fascia that is developing
between the external and internal oblique muscles in the region of the
labioscrotal swellings. The processus vaginalis pushes out as a "sock-like"
extension into the transversalis fascia, the internal oblique muscle, and the
external oblique muscle, thus forming the inguinal canal. After the processus
vaginalis has evaginated into the scrotum, the gubernacula shorten and
simply pull the gonads through the canal. Within the first year after birth, the
superior portion of the processus vaginalis is usually obliterated, leaving only
a distal remnant sac, the tunica vaginalis, which lies anterior to the testis.
During infancy, this sac wraps around most of the testis. Its lumen is normally
collapsed, but under pathologic conditions it may fill with serous secretions,
forming a testicular hydrocele (choice E). If the processus vaginalis remains
patent, a connection between the abdominal cavity and scrotal sac will occur.
Loops of intestine may herniate into this processus, resulting in an indirect
inguinal hernia.
In adults, hernias are caused by a protrusion of a structure, usually intestine,
through a tissue that normally contains it. Direct inguinal hernias occur
through the floor of the inguinal canal separate from the spermatic cord
(choice A). Direct inguinal hernias occur because of a breakdown of the
transversus abdominis aponeurosis and transversalis fascia.
An indirect inguinal hernia occurs through the internal inguinal ring in a
protrusion of peritoneum along the spermatic cord in the internal spermatic
fascia. Therefore, a large indirect inguinal hernia will descend into the
scrotum along the spermatic cord (choice B).
Spigelian hernias are rare and occur due to a weakness in the linea
semilunaris (choice C), which is located at the lateral margin of the rectus
sheath.
A 2-year-old child is seen for a welI-child visit in a pediatric clinic. Abdominal
examination demonstrates a palpable, non-tender mass on the
Ieft side of the abdomen. The mother had no idea the mass was present and the
pediatrician did not note the presence of the mass at the
child's 18-month welI-child visit. Physical examination is otherwise unremarkable.
Question 1 of 5
CT examination demonstrates an 8-cm diameter, roughly round, mass involving
the lower pole of the kidney. Which of the following is the
most likely cause of this mass?
/ A. Ewing sarcoma
/ B. Neuroblastoma
/ C. Renal cell carcinoma
/ D. Transitional cell carcinoma
/ E. Wilms tumor

Explanation - Q: 2.1 Close

The correct answer is E. Wilms tumor is an embryonal malignancy of the
kidney that most commonly affects children from birth to age 4, but can affect
children up to about age 15. This tumor is the second most common
extracranial solid tumor in children and tends to form large, round, solitary
masses of the kidney, although bilaterality and multicentricity may also occur.
Wilms tumors usually present with a palpable abdominal mass, with less
common presentations including abdominal pain, hematuria (indicating
invasion of the collecting system), hypertension, fever, nausea, and vomiting.
Ewing sarcoma (choice A) most commonly involves the long bones of older
children and young adolescents.
Neuroblastoma (choice B) may also effect very young children, and typically
arises in the adrenal gland. Extensions into the kidney can occur, but usually
involve the upper pole first.
Renal cell carcinoma (choice C) and transitional cell carcinoma (choice D)
would be much more likely to involve the kidney of an adult.
A 2-year-old child is seen for a welI-child visit in a pediatric clinic. Abdominal
examination demonstrates a palpable, non-tender mass on the
Ieft side of the abdomen. The mother had no idea the mass was present and the
pediatrician did not note the presence of the mass at the
child's 18-month welI-child visit. Physical examination is otherwise unremarkable.
Question 2 of 5
If a CT guided biopsy of the mass were performed, which of the following
histological patterns would be most suggestive of the likely
diagnosis?
/ A. Cords of clear cells with rounded or polygonal shape and abundant clear
cytoplasm
/ B. Invasive papillary lesions with delicate connective tissue stalk covered with
epithelium resembling that lining the bladder
/ C. Small dark cells embedded in a finely fibrillar matrix with formation of
numerous rosettes
/ D. Triphasic pattern with tubule formation, spindle cells, and blastemal
elements
/ E. Uniform sheets of small round cells with scanty cytoplasm

Explanation - Q: 2.2 Close

The correct answer is D. Adequate sampling of most Wilms tumors will
detect the three distinct growth patterns that are most characteristic of this
tumor: less cellular tumor areas containing spindle cells; densely cellular
(blastemal) areas with closely packed small cells with scanty cytoplasm and
darkly blue nuclei; and areas with production of more mature-appearing
epithelium that may form occasional tubules. The tissues present are similar
to those present in the developing kidney, and may also include primitive
glomeruli. Wilms tumors may additionally contain heterologous elements,
such as skeletal muscle, smooth muscle, squamous or mucinous epithelium,
adipose tissue, cartilage, bone, or neural tissue.
Choice A suggests renal cell carcinoma.
Choice B suggests transitional cell carcinoma.
Choice C suggests neuroblastoma.
Choice E suggests Ewing sarcoma; neuroblastoma and some lymphomas
may also sometimes have a similar appearance.







A 2-year-old child is seen for a welI-child visit in a pediatric clinic. Abdominal
examination demonstrates a palpable, non-tender mass on the
Ieft side of the abdomen. The mother had no idea the mass was present and the
pediatrician did not note the presence of the mass at the
child's 18-month welI-child visit. Physical examination is otherwise unremarkable.
Question 4 of 5
Sometimes this child's disease occurs in association with certain congenital
anomalies. Children with which of the following abnormalities at
birth should be monitored for the development of this disease?
/ A. Aniridia
/ B. Coarctation of the aorta
/ C. Cystic hygroma
/ D. Parathyroid hyperplasia
/ E. Simian crease

Explanation - Q: 2.4 Close

The correct answer is A. The WAGR syndrome includes Wilms tumor,
aniridia (lack or defect of the iris), genital anomalies (gonadal dysgenesis,
hypospadias, cryptorchidism, or other genitourinary anomalies), and mental
retardation. Wilms tumor can also occur as part of the Beckwith-Wiedemann
syndrome with hemihypertrophy, and the Drash syndrome with nephropathy
and ambiguous genitalia.
Coarctation of the aorta (choice B) and cystic hygroma (choice C) are
associated with Turner syndrome, which may predispose for later
development of ovarian cancer.
Parathyroid hyperplasia (choice D) can be a component of multiple
endocrine neoplasia (MEN).
Simian crease (choice E) suggests Down syndrome, which is associated
with an increased incidence of leukemia.
A 2-year-old child is seen for a welI-child visit in a pediatric clinic. Abdominal
examination demonstrates a palpable, non-tender mass on the
Ieft side of the abdomen. The mother had no idea the mass was present and the
pediatrician did not note the presence of the mass at the
child's 18-month welI-child visit. Physical examination is otherwise unremarkable.
Question 5 of 5
Abnormalities of which of the following chromosomes have been linked to this
disease?
/ A. 3
/ B. 5
/ C. 11
/ D. 15
/ E. 21

Explanation - Q: 2.5 Close

The correct answer is C. Inactivation of the WT-1 Wilms tumor gene
(located at 11p13, and thought to encode a DNA-binding protein important in
fetal kidney development) has been reported in the tumor cells from many
Wilms tumor cases. Also, the WAGR syndrome involves a deletion of
chromosome 11p13 and the Beckwith-Wiedemann syndrome involves a
rearrangement of chromosome 11p15. The genetic basis of the Drash
syndrome has not been established. The other choices are distracters.


A 67-year-old man with an 18-year history of type 2 diabetes mellitus presents for
a routine physical examination. His temperature is 36.9 C
(98.5 F), his blood pressure is 158/98 mm Hg and his pulse is 82/minute and
regular. On examination, the physician notes a nontender,
pulsatile, mass in the mid-abdomen. A plain abdominal x-ray film with the patient
in the lateral position reveals spotty calcification of a
markedly dilated abdominal aortic walI.
Question 1 of 5
Which of the following is most likely pathogenetically related to this patient's
aortic disease?
/ A. Atherosclerosis
/ B. Cystic medial necrosis
/ C. Kawasaki disease
/ D. Mnckeberg arteriosclerosis
/ E. Syphilis




Explanation - Q: 3.1 Close

The correct answer is A. This patient has an abdominal aortic aneurysm
(AAA); these are most commonly the result of atherosclerosis. Hypertension
and cigarette smoking are also risk factors. AAA may be asymptomatic, or
may be associated with pain. Some aortic aneurysms are palpable as
pulsating masses in the abdomen. Abdominal aortic aneurysms typically
begin below the renal arteries and may extend well into the iliac system
bilaterally. Calcified atherosclerotic plaques on plain films of the abdomen
can be seen in the majority of cases. Rupture of an aortic aneurysm is
usually preceded by excruciating pain in the lower abdomen and back. The
mortality rate for abdominal aortic aneurysm repair performed electively is
about 2-5%, while that performed emergently after rupture of the aneurysm
has begun is 50%. Many patients with atherosclerotic aneurysms also have
severe coronary artery disease, and repair of the coronary arteries may be
needed before the patient is subjected to the risk of aortic aneurysm repair.
Cystic medial necrosis (choice B) is a risk factor for dissecting aneurysms
that typically do not widen the lumen of the aorta.
Kawasaki disease (choice C) can cause small aneurysms of the coronary
arteries.
Mnckeberg's arteriosclerosis (choice D) can cause focal calcification of
(usually medium-sized) arteries, but is not associated with aneurysm
formation.
Syphilis (choice E) is a now rare cause of aortic aneurysms that typically
involve the aortic root and ascending aorta.


A 67-year-old man with an 18-year history of type 2 diabetes mellitus presents for
a routine physical examination. His temperature is 36.9 C
(98.5 F), his blood pressure is 158/98 mm Hg and his pulse is 82/minute and
regular. On examination, the physician notes a nontender,
pulsatile, mass in the mid-abdomen. A plain abdominal x-ray film with the patient
in the lateral position reveals spotty calcification of a
markedly dilated abdominal aortic walI.
Question 2 of 5
Which of the following physiologic observations helps to account for the fact that
75% of the aneurysms of this patient's type are found in the
abdomen and only 25% principally involve the thorax?
/ A. Diastolic pressure is greater in the abdominal aorta in the supine position
/ B. Negative intrathoracic pressure reduces aortic wall tension in the thorax
/ C. The average blood flow in the abdominal aorta is greater than that in the
thoracic aorta
/ D. The average blood pressure in the abdominal aorta is higher than that in the
thoracic aorta
/ E. The average degree of turbulence in the thoracic aorta is higher than that in
the abdominal aorta

Explanation - Q: 3.2 Close

The correct answer is D. Increased blood pressure is a strong risk factor for
atherosclerosis, and humans pay a price for their erect sitting and standing
postures. In these postures, the abdominal aorta experiences the weight of a
column of blood added to the pressure produced by the heart. In the supine
posture, the pressures in the thoracic and abdominal aorta are similar. So, if
an average daily pressure is taken, the abdominal aorta tends to have a
significantly higher pressure than does the thoracic aorta.
Diastolic pressure (choice A) is actually greater in the thoracic aorta
compared to the abdominal aorta in the supine position. However, the
systolic blood pressure is greater in the abdominal aorta.
A negative intrathoracic pressure (choice B) would tend to increase
transmural pressure across the wall of the thoracic aorta, and thereby
increase wall tension and promote the development of aneurysms.
Blood flow (choice C) in the abdominal aorta is less than that in the thoracic
aorta, because some blood leaves the aorta through its thoracic branches.
Higher turbulence (choice E) tends to predispose for atherosclerosis, and
the flow in the abdominal aorta, with its many branches, tends to be more
turbulent than that in the thoracic aorta; this problem is exacerbated as
atherosclerotic disease advances.



A 67-year-old man with an 18-year history of type 2 diabetes mellitus presents for
a routine physical examination. His temperature is 36.9 C
(98.5 F), his blood pressure is 158/98 mm Hg and his pulse is 82/minute and
regular. On examination, the physician notes a nontender,
pulsatile, mass in the mid-abdomen. A plain abdominal x-ray film with the patient
in the lateral position reveals spotty calcification of a
markedly dilated abdominal aortic walI.
Question 3 of 5
The patient is taken to surgery and the abdominal aorta and proximal common
iliac arteries are replaced with a graft. Which of the following
aneurysm diameters is usually considered the threshold above which elective
surgery is recommended, unless contraindicated by other
disease?
/ A. 1 cm
/ B. 2 cm
/ C. 6 cm
/ D. 10 cm
/ E. 15 cm


Explanation - Q: 3.3 Close

The correct answer is C. This recommendation is made because larger
aneurysms have a much greater chance of rupture, and emergency repair
carries a high mortality rate. For aneurysms larger than 5 cm, the risk of
rupture is 5-10% per year

A 67-year-old man with an 18-year history of type 2 diabetes mellitus presents for
a routine physical examination. His temperature is 36.9 C
(98.5 F), his blood pressure is 158/98 mm Hg and his pulse is 82/minute and
regular. On examination, the physician notes a nontender,
pulsatile, mass in the mid-abdomen. A plain abdominal x-ray film with the patient
in the lateral position reveals spotty calcification of a
markedly dilated abdominal aortic walI.
Question 4 of 5
Following surgery, the patient is placed on a low-fat diet to reduce the risk of
continued progression of his atherosclerotic disease. A bile
acid sequestrant is added to interrupt enterohepatic circulation of bile acids.
Which of the following agents was most likely prescribed?
/ A. Atorvastatin
/ B. Cholestyramine
/ C. CIofibrate
/ D. Gemfibrozil
/ E. Lovastatin
Explanation - Q: 3.4 Close

The correct answer is B. Cholestyramine and colestipol are bile acid
sequestrants that bind bile acids in the intestine, thereby interrupting
enterohepatic circulation of bile acids. This has an indirect effect to enhance
LDL clearance and lower lipids in the blood.
Atorvastatin (choice A) and lovastatin (choice E) are lipid-lowering drugs
that competitively inhibit HMG-CoA reductase, an early step in cholesterol
biosynthesis.
Clofibrate (choice C) and gemfibrozil (choice D) are fibric acid derivatives
that may increase the activity of lipoprotein lipase.
A 67-year-old man with an 18-year history of type 2 diabetes mellitus presents for
a routine physical examination. His temperature is 36.9 C
(98.5 F), his blood pressure is 158/98 mm Hg and his pulse is 82/minute and
regular. On examination, the physician notes a nontender,
pulsatile, mass in the mid-abdomen. A plain abdominal x-ray film with the patient
in the lateral position reveals spotty calcification of a
markedly dilated abdominal aortic walI.
Question 5 of 5
The table shows values of vessel radius (r), intraluminal pressure (P), and wall
thickness (w) for both the normal aorta and an aortic aneurysm.



What effect does the aneurysm have on wall stress?
/ A. Wall stress decreases 10 fold
/ B. Wall stress decreases 2.5 fold
/ C. Wall stress decreases 5 fold
/ D. Wall stress increases 10 fold
/ E. Wall stress increases 2.5 fold
/ F. Wall stress increases 5 fold

Explanation - Q: 3.5 Close

The correct answer is E. According to the Law of LaPlace for a cylindrical
structure such as the aorta: wall stress (s) = (P x r)/w. Because P and w are
shown in the table to be unaffected by the aneurysm and because radius is
increased by 2.5 fold at the site of the aneurysm, it is clear that wall stress
has increased by 2.5 fold at the site of the aneurysm. This relationship
between vessel radius and wall tension can explain why the probability of
rupture increases as the aneurysm becomes larger.

A 3-year-old girl is seen in the emergency department with acute abdominal pain.
She has a 5-day history of vomiting and abdominal
distension. She has not passed stool during this time, and during the past day,
has been vomiting bilious materiaI. On physical examination,
she is lethargic, with a firm and tender abdomen, and peritoneal signs are
present. She is immediately referred for laparotomy for suspected
diagnoses of intussusception vs. volvulus. At surgery, approximately 20 cm of
small intestine is found to be markedly distended, and is
resected. The section contains a tightly knotted ball of nematodes that are about
15 to 35 cm in length. The worms have tapered ends without
hooks.
Question 1 of 5
The worms are most likely which of the following?



Explanation - Q: 4.1 Close

The correct answer is B. Ascaris lumbricoides is the only parasitic worm that is
likely to cause intestinal obstruction, and then only if the worm burden is high. The
description given of the worms in the case presentation is typical. All of the other
worms listed in the choices are also nematodes or roundworms.
Ankylostoma duodenale(choice A) is a small (approximately 1 cm) hookworm that
inhabits the small intestine and clings to the mucosa.
Enterobius vermicularis(choice C) is the pinworm. This is an approximately 1 cm
long worm that inhabits the large bowel (and appendix); the female deposits eggs on
the perianal skin.
Strongyloides stercoralis(choice D), or threadworm, is a 2.5 mm worm that lives in
the crypts of the small bowel and may cause chronic infection due to autoinfection. It
is the only nematode capable of increasing its numbers in a host. In
immunocompromised hosts, it can cause life-threatening disseminated infection.
Trichuris trichiura(choice E), or whipworm, is a 3-5 cm worm that lives on the
colorectal mucosa.

A 3-year-old girl is seen in the emergency department with acute abdominal pain.
She has a 5-day history of vomiting and abdominal
distension. She has not passed stool during this time, and during the past day,
has been vomiting bilious materiaI. On physical examination,
she is lethargic, with a firm and tender abdomen, and peritoneal signs are
present. She is immediately referred for laparotomy for suspected
diagnoses of intussusception vs. volvulus. At surgery, approximately 20 cm of
small intestine is found to be markedly distended, and is
resected. The section contains a tightly knotted ball of nematodes that are about
15 to 35 cm in length. The worms have tapered ends without
hooks.
Question 2 of 5
Which of the following best describes what would have been seen under the
microscope if the patient's stool had been analyzed for ova and
parasites?
/ A. Large oval eggs with a lateral spine
/ B. Large round to oval eggs, with a thick mammillated shells
/ C. Lemon-shaped eggs, with bipolar plugs,
/ D. Round eggs and proglottids filled with eggs
/ E. Small larvae

Explanation - Q: 4.2 Close

The correct answer is B. This is the correct description of fertilized Ascaris
eggs. Both fertilized and somewhat more elongated unfertilized eggs may be
seen; fertilized eggs may contain multiple cells under the thick wavy shell.
Choice A describes the eggs of the trematode (fluke) Schistosoma mansoni.
Choice C describes the eggs of Trichuris trichiura.
Choice D describes the eggs of the cestodes (tapeworms) Taenia saginata
and Taenia solium.
Choice E describes the larvae of Strongyloides stercoralis. Strongyloides
stercoralis has a rapid life cycle in its host. It is the only nematode whose
diagnostic form is a larva, not an egg.
A 3-year-old girl is seen in the emergency department with acute abdominal pain.
She has a 5-day history of vomiting and abdominal
distension. She has not passed stool during this time, and during the past day,
has been vomiting bilious materiaI. On physical examination,
she is lethargic, with a firm and tender abdomen, and peritoneal signs are
present. She is immediately referred for laparotomy for suspected
diagnoses of intussusception vs. volvulus. At surgery, approximately 20 cm of
small intestine is found to be markedly distended, and is
resected. The section contains a tightly knotted ball of nematodes that are about
15 to 35 cm in length. The worms have tapered ends without
hooks.
Question 3 of 5
If this child acquired her infection in the United States, in what region of the
nation does she most likely reside?
/ A. Desert Southwest
/ B. Midwest
/ C. Northeast
/ D. Pacific Northwest
/ E. Southeast

Explanation - Q: 4.3 Close

The correct answer is E. It is easy for medical students, residents, and
physicians to neglect parasitic diseases because they assume that the
American population is not likely to have them. However, it is thought that
more than 4 million individuals in the Untied States, most of whom are
immigrants from developing countries, are infected with Ascaris species,
predominately Ascaris lumbricoides. Most of these people have
asymptomatic infections. Ascaris can also be acquired in rural areas of the
Southeastern United States, where it is endemic. Worldwide, 1.4 billion
people are estimated to be infected.
A 3-year-old girl is seen in the emergency department with acute abdominal pain.
She has a 5-day history of vomiting and abdominal
distension. She has not passed stool during this time, and during the past day,
has been vomiting bilious materiaI. On physical examination,
she is lethargic, with a firm and tender abdomen, and peritoneal signs are
present. She is immediately referred for laparotomy for suspected
diagnoses of intussusception vs. volvulus. At surgery, approximately 20 cm of
small intestine is found to be markedly distended, and is
resected. The section contains a tightly knotted ball of nematodes that are about
15 to 35 cm in length. The worms have tapered ends without
hooks.
Question 4 of 5
Which of the following medications would be the most appropriate
pharmacotherapy for this patient?
/ A. Bithionol
/ B. Mebendazole
/ C. Metronidazole
/ D. Niclosamide
/ E. Praziquantel

Explanation - Q: 4.4 Close

The correct answer is B. Mebendazole is a systemically absorbed broad-
spectrum anthelminthic agent effective against Ascaris species, hookworm,
tapeworm, liver fluke, and pinworms.
Bithionol (choice A) is used to treat infections caused by Fasciola hepatica,
a tissue fluke.
Metronidazole (choice C) is used to treat infections caused by anaerobic
organisms, including the intestinal protozoa, Giardia lamblia, and Entamoeba
histolytica.
Niclosamide (choice D) is not available in the United States. It can be used
to treat tapeworm infections caused by T. saginata and D. latum. It is less
expensive than praziquantel.
Praziquantel (choice E) has broad-spectrum activity against most
trematodes and cestodes, with the exception of F. hepatica.




A 3-year-old girl is seen in the emergency department with acute abdominal pain.
She has a 5-day history of vomiting and abdominal
distension. She has not passed stool during this time, and during the past day,
has been vomiting bilious materiaI. On physical examination,
she is lethargic, with a firm and tender abdomen, and peritoneal signs are
present. She is immediately referred for laparotomy for suspected
diagnoses of intussusception vs. volvulus. At surgery, approximately 20 cm of
small intestine is found to be markedly distended, and is
resected. The section contains a tightly knotted ball of nematodes that are about
15 to 35 cm in length. The worms have tapered ends without
hooks.
Question 5 of 5
Part of the life cycle of this patient's parasite is a filarial stage in which larva,
hatched in the duodenum, penetrate the wall of the small
intestine. The passage of migrating larvae most commonly produces
symptomatic disease in which of the following organs?
/ A. Heart
/ B. Liver
/ C. Lungs
/ D. Pancreas
/ E. Stomach

Explanation - Q: 4.5 Close

The correct answer is C. Ascaris infection is acquired by ingestion of the
eggs, usually from contaminated soil. The ingested eggs hatch in the
duodenum to produce larvae, which cross the small intestinal wall to enter
the blood stream. They then pass through the heart to lodge in the lungs.
They leave the lung capillary bed to enter the airspaces. They then ascend
the bronchial tree into the oropharynx, where they are again swallowed. This
time, they return to the small intestine where they develop into mature worms
that can live in the host for up to two years. A gravid female worm may
produce up to 250,000 eggs daily, which are shed in stool. The passage of
the larvae through the lungs often produces cough and/or wheezing, and
may, in severe cases, produce fever, dyspnea, fleeting patchy pulmonary
infiltrates (Loeffler pneumonitis with prominent eosinophilia), and rarely
hemoptysis.
Passage through the heart (choice A) is usually asymptomatic, as the larvae
are small.
A single adult worm, but not usually larval forms, can migrate into the biliary
tree (choice B), leading to biliary colic, cholangitis, or gallstone formation;
obstructive jaundice uncommonly occurs.
The pancreas (choice D) and stomach (choice E) are not common sites for
complications of ascariasis.

A 27-year-old woman goes to an emergency room with severe abdominal pain.
She had previously experienced similar episodes of pain that
Iasted several hours to a few days, but this episode is the most severe. She has
also been experiencing nausea, vomiting, and constipation.
The physician is left with the impression that she is agitated and somewhat
confused, and an accurate history is difficult to elucidate. The
patient is sent for emergency laparotomy, but no pathology is noted at surgery.
Following the unrevealing surgery, an older surgeon
comments that he had once seen a similar case that was actually due to
porphyria.
Question 1 of 5
The porphyrias are biochemical abnormalities in which of the following
pathways?
/ A. GIycogen degradation
/ B. Heme synthesis
/ C. Lipoprotein degradation
/ D. Nucleotide degradation
/ E. Urea cycle


Explanation - Q: 1.1 Close

The correct answer is B. The porphyrias are a group of rare, related
diseases that have in common a block in the heme synthesis pathway. The
block is usually partial rather than complete, and thus many of these patients
have only intermittent symptoms. Most cases of porphyria present with either
a neurovisceral pattern (including both psychiatric symptoms and abdominal
pain) or with photosensitive skin lesions. These two patterns are associated
with different forms of porphyria.
Associate abnormalities of glycogen degradation (choice A) with the
glycogen storage diseases, such as von Gierke disease, Pompe disease,
and Forbes disease.
Associate abnormalities of lipoprotein degradation (choice C) with some
forms of hyperlipoproteinemia (notably Type I).
Associate abnormalities of nucleotide degradation (choice D) with gout and
Lesch-Nyhan syndrome.
Associate abnormalities of the urea cycle (choice E) with congenital
hyperammonemia, citrullinemia, and argininosuccinic acidemia.
A 27-year-old woman goes to an emergency room with severe abdominal pain.
She had previously experienced similar episodes of pain that
Iasted several hours to a few days, but this episode is the most severe. She has
also been experiencing nausea, vomiting, and constipation.
The physician is left with the impression that she is agitated and somewhat
confused, and an accurate history is difficult to elucidate. The
patient is sent for emergency laparotomy, but no pathology is noted at surgery.
Following the unrevealing surgery, an older surgeon
comments that he had once seen a similar case that was actually due to
porphyria.
Question 2 of 5
Following the surgery, the decision is made to screen for the porphyrias that
cause acute neurovisceral symptoms. Which of the following
tests would be most likely to be used?
/ A. Erythrocyte porphyrins
/ B. Total fecal porphyrins
/ C. Total plasma porphyrins
/ D. Total urinary porphyrins
/ E. Urinary porphobilinogen

Explanation - Q: 1.2 Close

The correct answer is E. The acute neurovisceral porphyrias are those that
tend to present with severe abdominal pain, often accompanied by
neuropsychiatric symptoms. The best tests to use for screening of these
diseases are urinary porphobilinogen (PBG, either random or 24 hour) and
urinary delta-aminolevulinic acid (ALA, either random or 24 hour).
Erythrocyte porphyrins (choice A) are used for follow-up in the
photosensitive types of porphyria.
Total fecal porphyrins (choice B) are used for follow-up evaluation after
screening tests for either the photosensitive porphyrias or the acute
neurovisceral porphyrias are positive.
Total plasma porphyrias (choice C) are useful for first line screening of the
photosensitive porphyrias, and are used for further evaluation after screening
in the acute neurovisceral porphyrias.
Total urinary porphyrins (choice D) are used for further evaluation after
screening for acute neurovisceral porphyrias.

A 27-year-old woman goes to an emergency room with severe abdominal pain.
She had previously experienced similar episodes of pain that
Iasted several hours to a few days, but this episode is the most severe. She has
also been experiencing nausea, vomiting, and constipation.
The physician is left with the impression that she is agitated and somewhat
confused, and an accurate history is difficult to elucidate. The
patient is sent for emergency laparotomy, but no pathology is noted at surgery.
Following the unrevealing surgery, an older surgeon
comments that he had once seen a similar case that was actually due to
porphyria.
uestion 3 of 5
Which of the following are the three most common forms of porphyria?
/ A. Acute intermittent porphyria, erythropoietic protoporphyria, and porphyria
cutanea tarda
/ B. Acute intermittent porphyria, hepatoerythropoietic porphyria, and variegate
porphyria
/ C. Congenital erythropoietic porphyria, delta-aminolevulinic acid dehydratase-
deficient porphyria, and hepatoerythropoietic porphyria
/ D. Erythropoietic protoporphyria, hereditary coproporphyria, and porphyria
cutanea tarda
/ E. Hereditary coproporphyria, variegate porphyria, and X-Iinked sideroblastic
anemia


Explanation - Q: 1.3 Close

The correct answer is A. The porphyrias are complex diseases that can
easily appear overwhelming. A very useful point to know (both clinically and
for the USMLE) is that the three most common forms are acute intermittent
porphyria, erythropoietic protoporphyria, and porphyria cutanea tarda. Acute
intermittent porphyria tends to present with acute neurovisceral symptoms.
Erythrocytic protoporphyria tends to present acutely with painful skin lesions.
Porphyria cutanea tarda tends to present with chronic blistering skin lesions.
The other types listed in various choices are also porphyrias, but are less
common.


A 27-year-old woman goes to an emergency room with severe abdominal pain.
She had previously experienced similar episodes of pain that
Iasted several hours to a few days, but this episode is the most severe. She has
also been experiencing nausea, vomiting, and constipation.
The physician is left with the impression that she is agitated and somewhat
confused, and an accurate history is difficult to elucidate. The
patient is sent for emergency laparotomy, but no pathology is noted at surgery.
Following the unrevealing surgery, an older surgeon
comments that he had once seen a similar case that was actually due to
porphyria.
uestion 4 of 5
This patient is found to have increased levels of both delta-aminolevulinic acid
(ALA) and porphobilinogen (PBG) in blood. Follow-up studies
demonstrate low PBG deaminase in erythrocytes. AIso, additional history is
elicited, revealing that the woman had started a very low
carbohydrate diet about one week before being admitted to the hospitaI. Which
of the following is the most likely diagnosis?
/ A. Acute intermittent porphyria
/ B. Congenital erythropoietic porphyria
/ C. Erythropoietic protoporphyria
/ D. Porphyria cutanea tarda
/ E. X-Iinked sideroblastic anemia


Explanation - Q: 1.4 Close

The correct answer is A. These laboratory findings are most consistent with
acute intermittent porphyria, which is due to PBG deaminase deficiency.
Patients usually, but not always, have a deficiency of erythrocyte PBG
deaminases. (Some cases have also been described in which the enzyme
deficiency is limited to liver.) The condition is an autosomal dominant
disorder that typically becomes symptomatic in women after puberty, and
then often only if a precipitating event (dieting, use of certain drugs,
premenstrual) is also present. Symptoms during the attacks can include
abdominal symptoms (pain, nausea, vomiting, constipation, diarrhea,
abdominal distension, ileus), which are thought to be due to the effects of
this condition on visceral nerves. Other symptoms that may be mediated
neurologically include incontinence, urinary retention, tachycardia,
diaphoresis, hypertension, muscle weakness, psychiatric symptoms,
seizures, and rarely, severe paralysis, respiratory insufficiency, and death.
Both intravenous glucose (oral is often inadequate due to poor intestinal
function) and exogenous heme supplementation can suppress the heme
biosynthetic mechanism, and tend to ameliorate the acute attack. Patients
should be cautioned to diet gently, as intense dieting can precipitate attacks.
Congenital erythropoietic porphyria (choice B) is characterized by severe
skin blistering that usually begins after birth, pink to dark-brown urine, normal
ALA and PBG, and increased porphyrins (primarily uroporphyrin I and
coproporphyrin I) in urine, plasma, and erythrocytes.
Erythropoietic protoporphyria (choice C) is characterized by cutaneous
photosensitivity that begins early in life and high protoporphyrin in
erythrocytes and bone marrow.
Porphyria cutanea tarda (choice D) is characterized by photosensitivity with
skin blistering, elevated plasma porphyrins, and elevated urine porphyrins
(mostly uroporphyrin and heptacarboxylporphyrin).
The very rare X-linked sideroblastic anemia (choice E), due to a deficiency
of delta-aminolevulinic acid synthase, can clinically resemble acute
intermittent porphyria, and is characterized by elevated levels of urinary ALA
and coproporphyrin.


A 27-year-old woman goes to an emergency room with severe abdominal pain.
She had previously experienced similar episodes of pain that
Iasted several hours to a few days, but this episode is the most severe. She has
also been experiencing nausea, vomiting, and constipation.
The physician is left with the impression that she is agitated and somewhat
confused, and an accurate history is difficult to elucidate. The
patient is sent for emergency laparotomy, but no pathology is noted at surgery.
Following the unrevealing surgery, an older surgeon
comments that he had once seen a similar case that was actually due to
porphyria.
Question 5 of 5
Which of the following drugs would be most likely to induce an attack of
abdominal pain in this patient?
/ A. Acetaminophen
/ B. Aspirin
/ C. Barbiturate
/ D. GIucocorticoid
/ E. Insulin

Explanation - Q: 1.5 Close

The correct answer is C. Some symptomatic episodes of acute porphyria
(including acute intermittent porphyria, hereditary coproporphyria, variegate
porphyria, and aminolevulinic acid dehydratase porphyria) are triggered by
drug ingestion, and administration of drugs to undiagnosed patients can
cause an acute exacerbation of an ongoing attack of acute porphyria. Drugs
considered unsafe for use in these patients notably include alcohol,
anticonvulsants, barbiturates, many other sedatives, and sulfonamide
antibiotics. Of particular concern are the sedative agents, since it may be
very tempting to give an obviously agitated patient a sedative to allow easier
examination of the patient. Many other drugs are also on the lists of
potentially dangerous drugs in these patients. Once the diagnosis is
established, the patient should be instructed to always inform her/his
physician of her condition, and ask that the safety of drugs prescribed in
patients with porphyria be checked. Many of the drugs that can induce or
exacerbate an attack of porphyria do so by increasing the activity of the
cytochrome P450 system, which indirectly triggers an increase in heme
biosynthesis. The other medications listed in the choices are "safe" in these
patients.





A 47-year-old woman presents to the emergency department with
cramping/colicky abdominal pain. The current episode of pain began
several hours ago, following a fatty meaI. The pain began slowly, and rose in
intensity to a plateau over the course of several hours. The
patient reports that she had had several other episodes of similar pain during the
past several months, with long intervening periods of
freedom from pain. On physical examination, she is noted to have tenderness to
deep palpation in the right upper quadrant of the abdomen
near the rib cage. The patient also reports that she is experiencing shoulder/back
pain at a site she identifies near the right lower scapula, but
no tenderness can be elicited during the back and shoulder examination.
Question 1 of 7
Which of the following organs is the most likely source of this woman's pain?
/ A. Appendix
/ B. Diaphragm
/ C. Esophagus
/ D. Gallbladder
/ E. Stomach

Explanation - Q: 2.1 Close

The correct answer is D. This woman most likely has gallstones.
Cholelithiasis, or the formation of calculi (gallstones) within the gallbladder, is
very common in the United States, with over 500,000 cholecystectomies
being performed yearly. While many cases of gallstone disease are
symptomatic, right upper quadrant pain with referral of the pain to the lower
right scapula should specifically suggest gallbladder disease. The pattern of
episodes of several hours of pain followed by long periods of freedom from
pain is also typical of symptomatic gallstone disease.
The appendix (choice A) would most likely cause lower abdominal pain.
Pain from irritation of the diaphragm (choice B) can cause right upper
quadrant pain and referred pain in the supraclavicular area (rather than the
subscapular pain of biliary colic). The absence of right upper quadrant
tenderness to palpation, and the history of pain after a fatty meal also argue
against this diagnosis.
Esophageal pain (choice C) related to regurgitation of gastric contents
(heartburn) can occur postprandially, but tends to radiate into the neck,
throat, or even face.
Peptic ulcer pain of gastric origin (choice E) is usually described as causing
burning, gnawing, or hunger, and may be relieved by eating.
A 47-year-old woman presents to the emergency department with
cramping/colicky abdominal pain. The current episode of pain began
several hours ago, following a fatty meaI. The pain began slowly, and rose in
intensity to a plateau over the course of several hours. The
patient reports that she had had several other episodes of similar pain during the
past several months, with long intervening periods of
freedom from pain. On physical examination, she is noted to have tenderness to
deep palpation in the right upper quadrant of the abdomen
near the rib cage. The patient also reports that she is experiencing shoulder/back
pain at a site she identifies near the right lower scapula, but
no tenderness can be elicited during the back and shoulder examination.
Question 2 of 7
Which of the following techniques would be most appropriate to demonstrate the
patient's most likely diagnosis?
/ A. Colonoscopy
/ B. CT scan of the abdomen
/ C. Esophagoduodenoscopy
/ D. MRI scan of the abdomen
/ E. UItrasonography
Explanation - Q: 2.2 Close

The correct answer is E. Real-time ultrasonography, with 98% sensitivity
and 95% specificity, is considered the method of choice for diagnosing
possible gallbladder stones.
Colonoscopy (choice A) and esophagoduodenoscopy (choice C) might be
helpful for excluding alternative diagnoses, but would not themselves
establish a diagnosis of gallstone disease.
CT (choice B) and MRI (choice D) scans of the abdomen are expensive
tests whose use is not warranted, since real-time ultrasonography performs
as well or better.

A 47-year-old woman presents to the emergency department with
cramping/colicky abdominal pain. The current episode of pain began
several hours ago, following a fatty meaI. The pain began slowly, and rose in
intensity to a plateau over the course of several hours. The
patient reports that she had had several other episodes of similar pain during the
past several months, with long intervening periods of
freedom from pain. On physical examination, she is noted to have tenderness to
deep palpation in the right upper quadrant of the abdomen
near the rib cage. The patient also reports that she is experiencing shoulder/back
pain at a site she identifies near the right lower scapula, but
no tenderness can be elicited during the back and shoulder examination.
Question 3 of 7
Following appropriate diagnostic studies, the patient is taken to the surgical suite.
During the surgery, the surgeon inserts his fingers from
right to left behind the hepatoduodenal ligament. As he does so, his fingers enter
which of the following?
/ A. Ampulla of Vater
/ B. Common bile duct
/ C. Epiploic foramen
/ D. Greater peritoneal sac
/ E. Portal vein

Explanation - Q: 2.3 Close

The correct answer is C. The space behind the stomach, hepatoduodenal
ligament, and hepatogastric ligament is the omental bursa. This space can
be entered by passing through the epiploic foramen of Winslow, as described
in the question stem.
The common bile duct enters the duodenum through the ampulla of Vater
(choice A).
The hepatoduodenal ligament contains the common bile duct (choice B), the
portal vein (choice E), and the hepatic artery.
The greater peritoneal sac (choice D) lies anterior to the stomach and
hepatoduodenal ligament.

A 47-year-old woman presents to the emergency department with
cramping/colicky abdominal pain. The current episode of pain began
several hours ago, following a fatty meaI. The pain began slowly, and rose in
intensity to a plateau over the course of several hours. The
patient reports that she had had several other episodes of similar pain during the
past several months, with long intervening periods of
freedom from pain. On physical examination, she is noted to have tenderness to
deep palpation in the right upper quadrant of the abdomen
near the rib cage. The patient also reports that she is experiencing shoulder/back
pain at a site she identifies near the right lower scapula, but
no tenderness can be elicited during the back and shoulder examination.
Question 4 of 7
During the cholecystectomy, the surgeon ligates the cystic artery. This is typically
a branch of which of the following?
/ A. Gastroduodenal artery
/ B. Left gastroepiploic artery
/ C. Right gastroepiploic artery
/ D. Right hepatic artery
/ E. Superior pancreaticoduodenal artery
Explanation - Q: 2.4 Close

The correct answer is D. The cystic artery is generally a branch of the right
hepatic artery.
The gastroduodenal artery (choice A) is a branch of the (common) hepatic
artery.
The left gastroepiploic artery (choice B) is a branch of the splenic artery.
The right gastroepiploic artery (choice C) is a branch of the gastroduodenal
artery.
The superior pancreaticoduodenal artery (choice E) is a branch of the
gastroduodenal artery.

A 47-year-old woman presents to the emergency department with
cramping/colicky abdominal pain. The current episode of pain began
several hours ago, following a fatty meaI. The pain began slowly, and rose in
intensity to a plateau over the course of several hours. The
patient reports that she had had several other episodes of similar pain during the
past several months, with long intervening periods of
freedom from pain. On physical examination, she is noted to have tenderness to
deep palpation in the right upper quadrant of the abdomen
near the rib cage. The patient also reports that she is experiencing shoulder/back
pain at a site she identifies near the right lower scapula, but
no tenderness can be elicited during the back and shoulder examination.


Question 5 of 7



Pathologic examination of the specimen removed by the surgeon demonstrates
the presence of numerous yellow stones (shown above).
These are most likely composed primarily of which of the following?
/ A. Bilirubinate
/ B. Calcium phosphate
/ C. Cholesterol
/ D. Cystine
/ E. Struvite

Explanation - Q: 2.5 Close

The correct answer is C. The stones are gallstones, and their yellow color
indicates that they are composed of cholesterol. Cholesterol stones are the
most common form of gallstones. Risk factors include female sex, multiparity,
obesity, increased age (female, fat, forty, and fertile) and North American
Indian race.
Bilirubinate (choice A) gallstones, which are usually associated with
hemolytic anemias, are less common, brown, rather than yellow, and often
faceted.
Calcium phosphate (choice B), cystine (choice D), and struvite (choice E)
composition can be seen in kidney stones.

A 47-year-old woman presents to the emergency department with
cramping/colicky abdominal pain. The current episode of pain began
several hours ago, following a fatty meaI. The pain began slowly, and rose in
intensity to a plateau over the course of several hours. The
patient reports that she had had several other episodes of similar pain during the
past several months, with long intervening periods of
freedom from pain. On physical examination, she is noted to have tenderness to
deep palpation in the right upper quadrant of the abdomen
near the rib cage. The patient also reports that she is experiencing shoulder/back
pain at a site she identifies near the right lower scapula, but
no tenderness can be elicited during the back and shoulder examination.
Question 6 of 7
If this patient had a small stone lodge near the ampulla of Vater, which of the
following complications would be most likely to occur?
/ A. Crohn disease
/ B. Diabetes mellitus
/ C. Pancreatitis
/ D. Peptic ulcer
/ E. Polyarteritis nodosa



Explanation - Q: 2.6 Close

The correct answer is C. A small gallstone obstructing the pancreatic
outflow is a well-known cause of acute pancreatitis. The other conditions
listed are not caused by gallstones.


A 47-year-old woman presents to the emergency department with
cramping/colicky abdominal pain. The current episode of pain began
several hours ago, following a fatty meaI. The pain began slowly, and rose in
intensity to a plateau over the course of several hours. The
patient reports that she had had several other episodes of similar pain during the
past several months, with long intervening periods of
freedom from pain. On physical examination, she is noted to have tenderness to
deep palpation in the right upper quadrant of the abdomen
near the rib cage. The patient also reports that she is experiencing shoulder/back
pain at a site she identifies near the right lower scapula, but
no tenderness can be elicited during the back and shoulder examination.
Question 7 of 7
If this patient had refused surgical treatment, which of the following would be the
most appropriate pharmacotherapy to provide definitive
treatment and thereby relieve associated pain?
/ A. Ampicillin
/ B. CIofibrate
/ C. Meperidine
/ D. Oxycodone
/ E. Ursodiol

Explanation - Q: 2.7 Close

The correct answer is E. The question is asking, "Which of the following will
eradicate a gallstone?" When a gallstone is eliminated the pain will
subsequently be eliminated. This question is NOT asking, "which of the
following is the most appropriate form of pain control?". Ursodiol
(ursodeoxycholic acid) is a hydrophilic bile acid that is used to dissolve small
(< 20 mm), non-calcified, radiolucent cholesterol gallstones in patients with
functioning gallbladders who cannot undergo (or refuse) cholecystectomy.
Analgesics and antibiotics, such as ampicillin (choice A), are administered
when appropriate, but do not help eradicate the stones.
Clofibrate (choice B) is an antihyperlipidemic that is associated with the
development of gallstones. High-risk patients, such as diabetics and the
elderly, should be watched closely.
As a side note, if this question were asking: "which of the following is the
most appropriate form of pain control in this patient", the most appropriate
answer would be meperidine. Meperidine (choice C) is the narcotic of choice
since it causes the least amount of spasm of the sphincter of Oddi. In other
words, meperidine is preferred over oxycodone (choice D).


A 64-year-old man with a history of coronary artery disease (CAD) comes to the
emergency department with the acute onset of severe,
constant, Iower abdominal pain and rectal bleeding. He reports that he previously
has had several episodes of similar, but less severe pain.
About 12 hours after the onset of pain, the patient began passing copious bright
red blood per rectum. He denies nausea, vomiting, sick
contacts, or foreign traveI. Initial physical examination reveals a distressed man,
who is afebrile, but tachypneic, with scant diffuse abdominal
tenderness to palpation. Rectal examination is positive for blood. Laboratory
studies reveal a metabolic acidosis with an elevated serum
Iactate.

Question 1 of 5

Which of the following is the most likely diagnosis?
/ A. Colon carcinoma
/ B. Infectious colitis
/ C. Inflammatory bowel disease
/ D. Ischemic colitis
/ E. Necrotizing enterocolitis

Explanation - Q: 3.1 Close

The correct answer is D. A patient with severe abdominal pain and rectal
bleeding with an unremarkable physical examination is likely suffering from
ischemic colitis. "Pain out-of-proportion to examination" is a classic finding for
ischemic colitis. The previous episodes of less severe pain represent
ischemic angina. An infarction has occurred, as indicated by the rise in
serum lactate secondary to the colon's anaerobic metabolism. The history of
coronary artery disease also suggests this diagnosis, as the atherosclerotic
processes that contribute to his CAD are also likely present in his abdominal
vasculature.
Colon cancer (choice A) would produce less acute symptoms, but
occasionally, colon cancer may present acutely with obstructive symptoms.
Patients may have bleeding and abdominal pain, but the pain is typically
intermittent and accompanied by nausea, vomiting, abdominal distention,
and absence of flatus.
Infectious colitis (choice B) is incorrect. While patients may have bleeding
and abdominal pain, nothing in the history suggests a disease of infectious
origin (no sick contacts or foreign travel). The acute onset also suggests a
vascular event, rather than an infectious one.
Inflammatory bowel disease (IBD) (choice C) is incorrect because while the
patient reports previous episodes, an elderly man with IBD would likely have
a chronic history of abdominal pain and bleeding.
Necrotizing enterocolitis (choice E) affects premature infants and would not
be relevant in this setting.


A 64-year-old man with a history of coronary artery disease (CAD) comes to the
emergency department with the acute onset of severe,
constant, Iower abdominal pain and rectal bleeding. He reports that he previously
has had several episodes of similar, but less severe pain.
About 12 hours after the onset of pain, the patient began passing copious bright
red blood per rectum. He denies nausea, vomiting, sick
contacts, or foreign traveI. Initial physical examination reveals a distressed man,
who is afebrile, but tachypneic, with scant diffuse abdominal
tenderness to palpation. Rectal examination is positive for blood. Laboratory
studies reveal a metabolic acidosis with an elevated serum
Iactate.
Question 2 of 5
The lactate produced from the anaerobic metabolism in the infarcted gut will
likely be which of the following?
/ A. Exhaled as a fruity odor
/ B. Incorporated into glycogen in the liver
/ C. Incorporated into myoglobin in muscle
/ D. Incorporated into urea in the urine
/ E. Secreted by the kidneys unchanged

Explanation - Q: 3.2 Close

The correct answer is B. Lactate is converted into glucose, and then
glycogen in the liver by a process know as the Cori cycle.
Choice A is incorrect, as lactate would not be exhaled. A fruity odor on the
breath would be a sign of ketoacidosis.
While some of the carbon from the lactate may be incorporated into peptides
via Krebs intermediates (e.g., choice C), the vast majority would be left as
carbohydrate.
Urea (choice D) represents a means of eliminating nitrogenous waste.
Choice E is wrong, as the kidneys would retain the lactate, rather than
excreting it.

A 64-year-old man with a history of coronary artery disease (CAD) comes to the
emergency department with the acute onset of severe,
constant, Iower abdominal pain and rectal bleeding. He reports that he previously
has had several episodes of similar, but less severe pain.
About 12 hours after the onset of pain, the patient began passing copious bright
red blood per rectum. He denies nausea, vomiting, sick
contacts, or foreign traveI. Initial physical examination reveals a distressed man,
who is afebrile, but tachypneic, with scant diffuse abdominal
tenderness to palpation. Rectal examination is positive for blood. Laboratory
studies reveal a metabolic acidosis with an elevated serum
Iactate.


Question 3 of 5
If this patient's disease were drug-induced, which of the following agents would
most likely be responsible?
/ A. Acetaminophen
/ B. Amiodarone
/ C. Cocaine
/ D. Dexamethasone
/ E. Nitroglycerin

Explanation - Q: 3.3 Close

The correct answer is C. Cocaine is a sympathomimetic drug that indirectly
acts on both the alpha and beta adrenergic receptors on the vasculature. As
such, cocaine may cause vasospasm in the abdominal vasculature leading
to infarction and ischemic colitis. Similar vasospastic events may occur in the
coronary vasculature, leading to myocardial infarction.
Acetaminophen (choice A) is an analgesic, and would not play a role in
producing ischemic colitis.
Amiodarone (choice B) is an antiarrhythmic, and would not contribute to
ischemic colitis.
Dexamethasone (choice D) is a steroidal anti-inflammatory drug. Not only
would this medication not cause ischemic colitis, it might mask the symptoms
due to its potent anti-inflammatory properties.
Nitroglycerin (choice E) is a venodilator, and would not contribute to
ischemic colitis. As a venodilator, nitroglycerin is used to treat coronary
ischemia by reducing cardiac preload.
A 64-year-old man with a history of coronary artery disease (CAD) comes to the
emergency department with the acute onset of severe,
constant, Iower abdominal pain and rectal bleeding. He reports that he previously
has had several episodes of similar, but less severe pain.
About 12 hours after the onset of pain, the patient began passing copious bright
red blood per rectum. He denies nausea, vomiting, sick
contacts, or foreign traveI. Initial physical examination reveals a distressed man,
who is afebrile, but tachypneic, with scant diffuse abdominal
tenderness to palpation. Rectal examination is positive for blood. Laboratory
studies reveal a metabolic acidosis with an elevated serum
Iactate.
Question 4 of 5
While the patient is in the emergency department, the pain becomes increasingly
severe. Several hours after his initial examination, the
patient becomes febrile and is now exquisitely tender to palpation. He writhes in
pain when the physician jostles the bed. Air is seen under
the diaphragm in an upright chest x-ray film. These new findings suggest which
of the following?
/ A. Abdominal aortic aneurysm
/ B. Bowel obstruction
/ C. Cholecystitis
/ D. Hypovolemia
/ E. Perforation with peritonitis

Explanation - Q: 3.4 Close

The correct answer is E. This patient has experienced a bowel perforation.
Air under the diaphragm in an upright chest film provides definitive evidence
that a hollow viscus has ruptured. Air near the liver on a left lateral decubitus
(patient lays with the left side down) is an alternative study to demonstrate
perforation. Spillage from the perforated bowel has irritated and inflamed the
peritoneum, resulting in peritonitis. Symptoms of peritonitis include extreme,
sharp pain exacerbated by jostling (patients often report that the bumpy ride
to the emergency department caused extreme pain). Patients will be
exquisitely tender to palpation and percussion and may have abdominal
rigidity. Fever typically accompanies peritonitis.
While an abdominal aortic aneurysm or AAA (choice A) presents as acute
abdominal pain, this pain is described as tearing and may radiate to the
back. A pulsatile abdominal mass may be palpated. The air on the chest film
is also inconsistent with AAA.
This patient does not have bowel obstruction (choice B). Signs and
symptoms of bowel obstruction include: nausea, vomiting, intermittent
abdominal pain, hypovolemia, abdominal distention, absence of flatus, and a
"step ladder" bowel pattern on abdominal films.
Cholecystitis (choice C) typically presents as right upper quadrant (RUQ)
pain, fever, and jaundice. Patients usually have a history of colicky RUQ
pain.
While the patient is at risk for hypovolemia (choice D), none of the
symptoms listed typify hypovolemia. Signs and symptoms of mild to
moderate hypovolemia include malaise, dry mouth, thirst, decreased skin
turgor, tachycardia, hypotension, and decreased urine output.
A 64-year-old man with a history of coronary artery disease (CAD) comes to the
emergency department with the acute onset of severe,
constant, Iower abdominal pain and rectal bleeding. He reports that he previously
has had several episodes of similar, but less severe pain.
About 12 hours after the onset of pain, the patient began passing copious bright
red blood per rectum. He denies nausea, vomiting, sick
contacts, or foreign traveI. Initial physical examination reveals a distressed man,
who is afebrile, but tachypneic, with scant diffuse abdominal
tenderness to palpation. Rectal examination is positive for blood. Laboratory
studies reveal a metabolic acidosis with an elevated serum
Iactate.
Question 5 of 5
Upon surgical exploration of the abdomen, the colon is dull and dusky from the
mid transverse colon to the rectum. The patient has occluded
which of the following vessels?
/ A. Celiac trunk
/ B. Cystic artery
/ C. External iliac artery
/ D. Inferior mesenteric artery
/ E. Superior mesenteric artery

Explanation - Q: 3.5 Close

The correct answer is D. The inferior mesenteric artery distributes blood to
the embryologic hindgut. This includes the distal 1/3 of the transverse colon
to the rectum. The rectum is spared because it receives circulation from the
inferior rectal artery (not mesenteric).
The celiac trunk (choice A) supplies the embryologic foregut. The first three
branches include the splenic artery, the left gastric artery, and the common
hepatic artery. This patient has no findings in this distribution.
The cystic artery (choice B) supplies the gall bladder. There are no gall
bladder findings in this case.
The external iliac artery (choice C) gives rise to the vessels of the lower
extremity. Symptoms of occlusion or stenosis might include buttock and thigh
pain exacerbated by walking. Severe stenosis might give patients buttock
and thigh pain, even at rest.
The superior mesenteric artery (choice E) supplies the embryologic hindgut.
This extends from the duodenum to the proximal 2/3 of the transverse colon.


A 45-year-old man goes to an emergency department because he is
experiencing severe abdominal pain, which is radiating straight through
to his back. The pain began several hours after an admitted alcoholic binge, and
has not changed in position, although it has become worse.
Question 1 of 5
Which of the following would be the most likely cause of this type of pain?
/ A. Acute appendicitis
/ B. Acute hepatitis
/ C. Acute pancreatitis
/ D. Chronic hepatitis
/ E. Myocardial infarction

Explanation - Q: 4.1 Close

The correct answer is C. The typical pain described occurs in
approximately 50% of patients with acute pancreatitis. Other patients may
have milder pain or even, uncommonly, pain first felt in the lower abdomen.
The pain of acute appendicitis (choice A) is often felt first as referred pain
near the umbilicus, with tenderness on palpation in the left lower quadrant.
Acute hepatitis (choice B) can cause pain referred to the right shoulder.
Chronic hepatitis (choice D) does not usually cause pain.
Myocardial infarction (choice E) can cause substernal pain and pain
radiating to the left shoulder.

A 45-year-old man goes to an emergency department because he is
experiencing severe abdominal pain, which is radiating straight through
to his back. The pain began several hours after an admitted alcoholic binge, and
has not changed in position, although it has become worse.
Question 2 of 5
In addition to alcohol use, which of the following is a common predisposing factor
for this patient's disease?
/ A. Biliary tract stones
/ B. Duodenal cancer
/ C. Gastric carcinoma
/ D. Kidney stones
/ E. Peptic ulcer


Explanation - Q: 4.2 Close

The correct answer is A. The overwhelmingly most common predisposing
factors for acute pancreatitis are gallstones (more specifically tiny ones that
lodge in the extrahepatic bile duct system) and alcohol abuse.
Rarely, nearby cancers (choices B and C) can occlude the pancreatic duct
system and cause a secondary acute pancreatitis.
Kidney stones (choice D) have no relationship with pancreatitis.
Peptic ulcers (choice E) that erode into the pancreas can uncommonly
secondarily inflame the pancreas.


A 45-year-old man goes to an emergency department because he is
experiencing severe abdominal pain, which is radiating straight through
to his back. The pain began several hours after an admitted alcoholic binge, and
has not changed in position, although it has become worse.
Question 3 of 5
Marked serum elevation of which of the following markers would most strongly
substantiate the likely diagnosis?
/ A. Acid phosphatase
/ B. Amylase
/ C. Aspartate aminotransferase
/ D. AIkaline phosphatase
/ E. Creatinine kinase

Explanation - Q: 4.3 Close

The correct answer is B. The usual markers for pancreatitis are amylase
and lipase. Marked elevation of amylase usually means either pancreatic
disease or salivary gland disease; lipase will be elevated in pancreatic
disease but not salivary gland disease. If you see elevated amylase on a
USMLE question, you should think of pancreatitis or salivary gland disease
(mumps, salivary gland stone). However, you should be aware, for your
general medical knowledge, that modest elevations of amylase can be seen
in a much wider variety of settings (often reflecting either subclinical
pancreatic damage or hemoconcentration of pancreatic enzymes), including
GI obstruction, mesenteric thrombosis and infarction, macroamylasemia (a
genetic condition with abnormal amylase), renal disease, ruptured tubal
pregnancy, lung cancer, acute alcohol ingestion, and following abdominal
surgery.
Associate acid phosphatase (choice A) with diseases involving the prostate
and, to lesser degrees, bone, the heart, platelets, and the liver.
Associate aspartate aminotransferase (choice C) with diseases of the heart,
muscle, liver, pancreas (though not as important for diagnosis as amylase
and lipase), and brain.
Associate alkaline phosphatase (choice D) with diseases of bone, liver, and
to lesser degrees, lung and heart.
Associate creatinine kinase (choice E) with diseases of the heart, muscle,
brain, and the general body (trauma, surgery).
A 45-year-old man goes to an emergency department because he is
experiencing severe abdominal pain, which is radiating straight through
to his back. The pain began several hours after an admitted alcoholic binge, and
has not changed in position, although it has become worse.
Question 4 of 5
The patient has a severe course that requires treatment in an ICU. CIinically, he
appears similar to patients with sepsis, with fever, elevated
white count, hypotension, increased pulse rate, shallow and rapid breathing,
oliguria, and a blunted sensorium, in addition to his pain and
abdominal tenderness. These clinical findings are most likely related to which of
the following?
/ A. Activation of the inflammatory cascade
/ B. AIcohol withdrawal symptoms
/ C. AIIergic reaction to alcohol
/ D. Drug toxicity effect
/ E. Secondary infection with mixed flora gut bacteria
Explanation - Q: 4.4 Close

The correct answer is A. Acute pancreatitis can either be relatively mild, or
a severe condition that may cause death. It is thought that, in severe cases,
leakage of enzyme-containing pancreatic secretions into the tissues/and or
blood stream causes cleavage of precursors, thus strongly activating the
complement and inflammatory cascades. These, in turn, produce abundant
cytokines, which worsen the symptoms. The clinical result is similar to
sepsis, with risk of multi-organ failure and death. The treatment of acute
pancreatitis is primarily supportive, and may include careful attention to fluid
resuscitation, oxygen supplementation, cardiovascular support, dialysis,
management of electrolyte abnormalities, pain control, and total parenteral
nutrition.
Alcohol allergy (choice C) or withdrawal (choice B) do not play any
additional part in most of these symptoms once the pancreatitis has
developed.
Infection (choice E) and drug toxicity (choice D) are also not a necessary
part of the clinical picture, although physicians may worry that the patient's
general clinical status is masking other, potentially more treatable, problems.

A 45-year-old man goes to an emergency department because he is
experiencing severe abdominal pain, which is radiating straight through
to his back. The pain began several hours after an admitted alcoholic binge, and
has not changed in position, although it has become worse.
Question 5 of 5
The patient's condition resolves in about two weeks, but he continues to drink
after leaving the hospitaI. When seen several years later, he
has had a number of similar episodes, and now has chronic severe abdominal
pain. CT scan demonstrates a single, smooth-walled, fluid
filled space in the tail of the pancreas, which can be reached by the radiologist
for CT-guided aspiration with an approach from the back. The
fluid aspirated is yellowish, clear, and acellular. Which of the following is the most
likely diagnosis?
/ A. Pancreatic microcystic adenoma
/ B. Pancreatic mucinous cystadenocarcinoma
/ C. Pancreatic mucinous cystadenoma
/ D. Pancreatic pseudocyst
/ E. Pancreatic solid-cystic tumor

Explanation - Q: 4.5 Close

The correct answer is D. Pancreatic pseudocyst is a fairly common
complication of both acute and chronic pancreatitis, and appears to develop
when trapping of pancreatic digestive juices (containing amylase, lipase, and
proteases) causes a "digestion" of part of the pancreas, leaving a fluid filled
cystic space. The term "pseudocyst", rather than "cyst", is used by purists
because the space does not have an epithelial lining, and is hence not a
"true cyst". Pseudocysts are usually solitary and typically measure 5-10 cm in
diameter. They can be surgically excised (and the surrounding tissue will
typically show evidence of chronic pancreatitis in long-standing cases) or
sometimes, if the anatomy is favorable, drained into adjacent hollow viscera.
Some are medically managed if small.
Most true neoplasms of the pancreas contain (often large numbers of)
smaller, multiple, cysts. These tumors can be benign or malignant, and the
ones with mucus-secreting epithelium (choices B and C) are more common
than those with a serous lining (choices A and E).

A 17-year-old boy is taken to the emergency department because he has
developed severe abdominal pain. The pain began abruptly
several hours previously, and was felt initially in the periumbilical region, but later
shifted to the right lower quadrant. The boy had initially felt
somewhat nauseous, but this has passed. On physical examination, he is noted
to have localized pain on cough and to be running a low-
grade fever.
Question 1 of 5
Examination of the abdomen demonstrates right lower quadrant tenderness at
the junction of the middle and outer thirds of the line joining the
umbilicus to the anterior superior spine of the iliac. This location is known as
which of the following?
/ A. Gubernaculum
/ B. Langer's line
/ C. Linea alba
/ D. McBurney's point
/ E. Tunica albuginea

Explanation - Q: 5.1 Close

The correct answer is D. The point described is McBurney's point, which
overlies the location of the appendix in most individuals.
The gubernaculum (choice A) is the fibrous cord that connects the primordial
testis or ovary to the anterolateral abdominal wall.
Langer's lines (choice B) are the cleavage lines of the skin.
The linea alba (choice C) is a sheet-like aponeurosis that covers the anterior
abdominal wall.
The tunica albuginea (choice E) is a tough fibrous coat that covers the testis.
A 17-year-old boy is taken to the emergency department because he has
developed severe abdominal pain. The pain began abruptly
several hours previously, and was felt initially in the periumbilical region, but later
shifted to the right lower quadrant. The boy had initially felt
somewhat nauseous, but this has passed. On physical examination, he is noted
to have localized pain on cough and to be running a low-
grade fever.
Question 2 of 5
Which of the following is the most likely diagnosis?
/ A. Appendicitis
/ B. Diverticulitis
/ C. Gallstones
/ D. Rectal ulcer
/ E. Renal colic

Explanation - Q: 5.2 Close

The correct answer is A. This patient has a typical presentation for
appendicitis, and the diagnosis is confirmed by the presence of localized
tenderness at McBurney's point.
Diverticulitis (choice B) is usually a disease of middle-aged or older
individuals and most commonly affects the left-lower quadrant.
Symptomatic gallstone disease (choice C) causes pain and tenderness in
the right upper quadrant.
Rectal ulcer (choice D) causes pain with stool movement, but does not
usually produce tenderness identifiable on abdominal examination.
Renal colic (choice E) usually produces flank or lower back pain.

A 17-year-old boy is taken to the emergency department because he has
developed severe abdominal pain. The pain began abruptly
several hours previously, and was felt initially in the periumbilical region, but later
shifted to the right lower quadrant. The boy had initially felt
somewhat nauseous, but this has passed. On physical examination, he is noted
to have localized pain on cough and to be running a low-
grade fever.
Question 3 of 5
The patient also exhibits an increase in pain in the right lower quadrant from the
passive extension of the right hip joint. This finding suggests
that the inflammation also involves which of the following?
/ A. BIadder
/ B. External oblique muscle
/ C. Femur
/ D. IIiopsoas muscle
/ E. Transverse abdominal muscle

Explanation - Q: 5.3 Close

The correct answer is D. This patient has a "positive psoas sign," which is
an increase in pain from passive extension of the right hip joint. This
maneuver stretches the iliopsoas muscle, which lies behind the appendix
and can become secondarily inflamed when the appendiceal inflammation
extends through the serosa. The psoas sign is clinically useful in both
confirming the appendix as the probable origin of the patient's pain, and
indicating that the inflammation is transmural and that the risk of rupture and
peritonitis is increased.
The bladder (choice A) is located more medially, and is usually not affected
by appendicitis.
The external oblique (choice B) and transverse abdominal (choice E)
muscles are in the anterior and lateral abdominal walls, and do not usually
become inflamed with appendicitis.
The femur (choice C) is moved during the extension of the right hip joint, but
is not the source of the pain.

A 17-year-old boy is taken to the emergency department because he has
developed severe abdominal pain. The pain began abruptly
several hours previously, and was felt initially in the periumbilical region, but later
shifted to the right lower quadrant. The boy had initially felt
somewhat nauseous, but this has passed. On physical examination, he is noted
to have localized pain on cough and to be running a low-
grade fever.
Question 4 of 5
The patient is prepared for immediate surgery. Cefotaxime is administered
before, during, and after surgery. The specimen, once removed,
is sent to the laboratory for pathology and bacteriologic culture. A malodorous
pus surrounds the serosa of the surgical specimen, and a
mixed gram-negative flora is cultured. Rapid enzyme tests for beta-Iactamase
production are positive. Which of the following drugs should be
added to the initial cefotaxime regimen?
/ A. Bacitracin
/ B. CIavulanic acid
/ C. CIindamycin
/ D. Isoniazid
/ E. Vancomycin

Explanation - Q: 5.4 Close

The correct answer is B. Clavulanic acid is a beta-lactamase inhibitor,
which when administered with beta lactam agents, irreversibly binds and
inactivates bacterial beta-lactamases, thereby permitting the companion drug
to disrupt bacterial cell wall synthesis. Suspected appendicitis is usually
treated with prompt appendectomy, since delay is associated with increased
risk of potentially life-threatening peritonitis and sepsis.
Bacitracin (choice A) is not correct, since this drug inhibits bacterial cell wall
synthesis by binding to and inhibiting the dephosphorylation of a membrane-
bound lipid pyrophosphate. Gram-negative bacteria are resistant to this
agent, and it would not have a synergistic effect if administered with a third
generation cephalosporin.
Clindamycin (choice C) is not correct, because this drug blocks protein
elongation by binding to the 50S ribosome. Although it is effective against
anaerobic gram-negative bacilli, it would not have a complementary effect
when administered with a third generation cephalosporin.
Isoniazid (choice D) is not correct because it inhibits the synthesis of mycolic
acids for the cell wall of actively dividing Mycobacteria. It would not be
effective in the flora of this patient's gut, nor would it act synergistically with
third generation cephalosporins.
Vancomycin (choice E) is not correct because it disrupts cell wall synthesis
in growing gram-positive bacteria. It would not be effective against the flora
of this patient's gut, nor would it act synergistically with third generation
cephalosporins.
A 17-year-old boy is taken to the emergency department because he has
developed severe abdominal pain. The pain began abruptly
several hours previously, and was felt initially in the periumbilical region, but later
shifted to the right lower quadrant. The boy had initially felt
somewhat nauseous, but this has passed. On physical examination, he is noted
to have localized pain on cough and to be running a low-
grade fever.
Question 5 of 5
The patient's postoperative recovery is uneventfuI, but 10 days after discharge,
he returns to his physician complaining of continuous low-
grade fever. An abscess is drained transrectally, and organisms are
cultured from the pus. Which of the following is an attribute of
this organism that makes it an important abscess former?
/ A. It is an anaerobe
/ B. It is an intracellular pathogen
/ C. Its endotoxin lacks 2,3-ketodeoxyoctonate
/ D. Mycolic acid
/ E. Prodigious capsule

Explanation - Q: 5.5 Close

The correct answer is E. Prevotella (Bacteroides) is a frequent cause of
abscesses in the intestinal tract because it is a normal flora organism and
produces a large capsule, which impedes phagocytosis.
Although the genus is anaerobic (choice A), it is not this attribute which
causes its formation of abscesses.
Prevotella is extracellular, not an intracellular pathogen (choice B).
Although Prevotella does indeed have this type of endotoxin (choice C), the
absence of this molecule decreases the toxicity of the toxin, and does not
contribute to its proclivity toward abscess formation.
Mycobacteria, and not other genera such as Prevotella, are known for their
long-chain fatty acids (mycolic acids; choice D).


The parents of a 13-month-old boy are contacted by a local social agency because the day
care they are using has reported them for
suspected child abuse. The abuse was suspected because the day care personnel thought
that the toddler had "too many bruises." The
parents deny the abuse, and promptly arrange to have their child seen by the chairman of
the pediatrics department in a clinic run by a
university medical center. On physical examination, the child is noted to have multiple
large bruises on his legs, arms, and buttocks. No skin
abrasions are seen, and no "pattern marks" (suggestive of being hit by an object such as a
belt buckle or rod) are seen. A blood smear isunremarkable. PTT is prolonged, and PT
and bleeding times are normaI.

Question 1 of 6

Which of the following is the most likely diagnosis?
/ A. Acute myelogenous leukemia
/ B. Disseminated intravascular coagulation
/ C. Hemophilia
/ D. Thrombocytopenia
/ E. Von Willebrand's disease



Explanation - Q: 1.1 Close

The correct answer is C. This child has hemophilia, which is an X-linked
recessive disorder of clotting. The severity of the symptoms experienced can
vary with the degree of enzymatic block in the production of the affected
clotting factor. Unfortunately, some cases may reach medical attention only
when the child begins to experience large numbers of falls when learning to
walk, and the parents are suspected of child abuse. The severity of the
symptoms can vary markedly from case to case, since even as little as 2-5%
of the normal levels of the missing clotting factor can markedly reduce
symptoms. Symptoms can include bruising, spontaneous bleeding, bleeding
into joints with associated pain and swelling, gastrointestinal and urinary tract
hemorrhage, and prolonged bleeding from cuts, tooth extractions, or surgery.
Acute myelogenous leukemia (choice A) and thrombocytopenia (choice D)
can also produce a tendency to bruise, but would not be associated with
normal peripheral smears.
Disseminated intravascular coagulation (choice B) is usually seen in patients
with other severe diseases, such as sepsis or complicated pregnancy.
Von Willebrand's disease (choice E) tends to cause oozing blood after injury
rather than bruising, and is associated with a long bleeding time.



The parents of a 13-month-old boy are contacted by a local social agency
because the day care they are using has reported them for
suspected child abuse. The abuse was suspected because the day care
personnel thought that the toddler had "too many bruises." The
parents deny the abuse, and promptly arrange to have their child seen by the
chairman of the pediatrics department in a clinic run by a
university medical center. On physical examination, the child is noted to have
multiple large bruises on his legs, arms, and buttocks. No skin
abrasions are seen, and no "pattern marks" (suggestive of being hit by an object
such as a belt buckle or rod) are seen. A blood smear is
unremarkable. PTT is prolonged, and PT and bleeding times are normaI.

Question 2 of 6

Deficiency of which of the following has been linked to this patient's condition?
/ A. Factor VII
/ B. Factor VIII
/ C. Factor X
/ D. Factor XII
/ E. Factor XIII

Explanation - Q: 1.2 Close

The correct answer is B. Hemophilia can be caused by a deficiency of
either factor VIII (classic hemophilia or hemophilia A) or factor IX (Christmas
disease or hemophilia B). The two variations are very similar clinically,
although the appropriate factor must be replaced when treating an acute
bleeding episode. Both of these factors are in the intrinsic coagulation
pathway. The other factors listed in the choices are not involved.


The parents of a 13-month-old boy are contacted by a local social agency
because the day care they are using has reported them for
suspected child abuse. The abuse was suspected because the day care
personnel thought that the toddler had "too many bruises." The
parents deny the abuse, and promptly arrange to have their child seen by the
chairman of the pediatrics department in a clinic run by a
university medical center. On physical examination, the child is noted to have
multiple large bruises on his legs, arms, and buttocks. No skin
abrasions are seen, and no "pattern marks" (suggestive of being hit by an object
such as a belt buckle or rod) are seen. A blood smear is
unremarkable. PTT is prolonged, and PT and bleeding times are normaI.

Question 3 of 6

This child's condition is due to a defective enzyme encoded on which of the
following chromosomes?
/ A. 13
/ B. 18
/ C. 21
/ D. X
/ E. Y


Explanation - Q: 1.3 Close

The correct answer is D. Both the hemophilia A and hemophilia B genes
are located on the X chromosome. While almost all affected individuals are
boys (rare exceptions are girls with a carrier mother and an affected father),
the Y chromosome (choice E) is not involved.
Associate chromosome 13 (choice A) with the trisomy known as Patau
syndrome.
Associate chromosome 18 (choice B) with the trisomy known as Edwards
syndrome.
Associate chromosome 21 (choice C) with the trisomy known as Down
syndrome.

The parents of a 13-month-old boy are contacted by a local social agency
because the day care they are using has reported them for
suspected child abuse. The abuse was suspected because the day care
personnel thought that the toddler had "too many bruises." The
parents deny the abuse, and promptly arrange to have their child seen by the
chairman of the pediatrics department in a clinic run by a
university medical center. On physical examination, the child is noted to have
multiple large bruises on his legs, arms, and buttocks. No skin
abrasions are seen, and no "pattern marks" (suggestive of being hit by an object
such as a belt buckle or rod) are seen. A blood smear is
unremarkable. PTT is prolonged, and PT and bleeding times are normaI.

Question 4 of 6

Which of the following can temporarily increase secretion of the needed factor in
some patients with a milder form of this disease?
/ A. Desmopressin
/ B. Epsilon-aminocaproic acid
/ C. Factor VIII concentrate
/ D. Factor IX concentrate
/ E. Fresh frozen plasma
Explanation - Q: 1.4 Close

The correct answer is A. Desmopressin can temporarily raise factor VIII
levels in a patient with mild classic hemophilia by causing the release of
factor VIII stored in the body. The use of desmopressin before dental work or
small surgical procedures may allow such procedures to be performed
without using factor VIII concentrates.
Epsilon-aminocaproic acid (choice B) is a anti-fibrinolytic agent used to slow
the degradation of blood clots, particularly in patients with hemophilia B.
Fresh frozen plasma (choice E) contains both factor VIII (choice C) and
factor IX (choice D), both of which are also available as concentrates. These
products are used to raise clotting factor levels, usually in patients who are
either actively bleeding or who are expected to be actively bleeding because
of surgery or dental procedures.

The parents of a 13-month-old boy are contacted by a local social agency
because the day care they are using has reported them for
suspected child abuse. The abuse was suspected because the day care
personnel thought that the toddler had "too many bruises." The
parents deny the abuse, and promptly arrange to have their child seen by the
chairman of the pediatrics department in a clinic run by a
university medical center. On physical examination, the child is noted to have
multiple large bruises on his legs, arms, and buttocks. No skin
abrasions are seen, and no "pattern marks" (suggestive of being hit by an object
such as a belt buckle or rod) are seen. A blood smear is
unremarkable. PTT is prolonged, and PT and bleeding times are normaI.

Question 5 of 6

The patient continues to experience persistent bleeding. Which of the following
would be the most appropriate pharmacotherapy?
/ A. Aminocaproic acid
/ B. Aspirin
/ C. Desmopressin
/ D. High-dose IV immune globulin
/ E. Prednisone

Explanation - Q: 1.5 Close

The correct answer is A. Aminocaproic acid is a synthetic hemostatic with a
specific antifibrinolysis action. The medication inhibits plasminogen activator
substance, which is concerned with the destruction of clots. This medication
does not control bleeding caused by a loss of vascular integrity.
Aminocaproic acid is indicated for control of excessive bleeding resulting
from systemic hyperfibrinolysis, well as for prevention of bleeding in
hemophiliacs undergoing a surgical procedure. It is contraindicated for use in
patients with severe renal impairment or active disseminated intravascular
clotting.
Aspirin (choice B) is absolutely contraindicated for use in this patient since it
would result in a more severe bleed.
Desmopressin is a synthetic analog of arginine vasopressin (choice C), the
naturally occurring human antidiuretic hormone (ADH). This agent is
generally used only in milder forms of the disease.
High-dose IV immune globulin (choice D) is most commonly used in the
treatment of ITP (idiopathic thrombocytopenic purpura) to rapidly raise the
platelet count.
Prednisone (choice E) is a glucocorticoid indicated for the treatment of
idiopathic thrombocytopenic purpura (IV dosing only) as well as in the
treatment of a variety of inflammatory conditions. It would be ineffective in
this patient.

The parents of a 13-month-old boy are contacted by a local social agency
because the day care they are using has reported them for
suspected child abuse. The abuse was suspected because the day care
personnel thought that the toddler had "too many bruises." The
parents deny the abuse, and promptly arrange to have their child seen by the
chairman of the pediatrics department in a clinic run by a
university medical center. On physical examination, the child is noted to have
multiple large bruises on his legs, arms, and buttocks. No skin
abrasions are seen, and no "pattern marks" (suggestive of being hit by an object
such as a belt buckle or rod) are seen. A blood smear is
unremarkable. PTT is prolonged, and PT and bleeding times are normaI.

Question 6 of 6

In approximately what year did effective screening of blood products begin to
limit the transmission of HIV and hepatitis B in these patients?
/ A. 1955
/ B. 1965
/ C. 1975
/ D. 1985
/ E. 1995

Explanation - Q: 1.6 Close

The correct answer is D. This is clinically important information, because
hemophiliac children who were diagnosed after 1985 have had very little
chance of developing AIDS or hepatitis as a complication of their disease
(they may still get exposures by other routes, e.g., sexual), while these were
major problems in patients who were treated earlier. Fortunately, recently
developed recombinant DNA factor VIII products are coming into increasing
use, and offer the chance of providing truly adequate hemostatic control
without any increased viral risk.

Following a viral illness, a previously healthy 3-year-old child develops multiple
petechiae and purpura, and is noted to be oozing blood from
the mouth. Prothrombin and activated partial thromboplastin times are within
normal limits. Review of a peripheral smear demonstrates that
platelet counts are markedly decreased, but all the platelets that are present
appear morphologically normaI. The remainder of the blood
smear and additional bone marrow studies are normaI.

Question 1 of 4

Which of the following is the most likely diagnosis?
/ A. Disseminated intravascular coagulation
/ B. Hemophilia A
/ C. Hemophilia B
/ D. Idiopathic thrombocytopenic purpura
/ E. Von Willebrand disease

Explanation - Q: 1.1 Close

The correct answer is D. Idiopathic thrombocytopenic purpura (ITP), also
known as immune thrombocytopenic purpura, now that the immunologic
basis of the disease has been better defined, is a hemorrhagic disorder that
can present in two very different forms in children and adults. Children (peak
ages 2-4 years) tend to present with acute, self-limited thrombocytopenia
that often follows a viral (or immunization) trigger. In contrast, adults (peak
ages 20-50 years) tend to have an insidious development of
thrombocytopenia that may persist for long periods. In both types of ITP, the
hemorrhagic diathesis is the result of a pure thrombocytopenia that does not
affect prothrombin (PT) and activated partial thromboplastin (PTT) times
because the blood clotting factors are present in normal quantities.
In disseminated intravascular coagulation (choice A), platelets are often
decreased as a result of consumption, but PT and PTT are prolonged.
In hemophilia A and B (choices B and C), platelets are present in normal
numbers, PT is normal, and PTT is long.
In von Willebrand disease (choice E), platelet counts are normal, but platelet
function is decreased because of decreased or abnormal von Willebrand
factor.

Following a viral illness, a previously healthy 3-year-old child develops multiple
petechiae and purpura, and is noted to be oozing blood from
the mouth. Prothrombin and activated partial thromboplastin times are within
normal limits. Review of a peripheral smear demonstrates that
platelet counts are markedly decreased, but all the platelets that are present
appear morphologically normaI. The remainder of the blood
smear and additional bone marrow studies are normaI.

Question 2 of 4

Which of the following tests would be most helpful in confirming the probable
diagnosis?
/ A. D-dimer levels
/ B. Factor VIII concentration
/ C. Factor IX concentration
/ D. IgG autoantibodies on platelet surfaces
/ E. Total plasma von Willebrand factor antigen

Explanation - Q: 1.2 Close

The correct answer is D. While ITP is mostly a diagnosis of exclusion, the
presence of IgG autoantibodies on the platelet surface can be an important
confirmatory test result.
High D-dimer levels (choice A) suggest disseminated intravascular
coagulation.
Low factor VIII (choice B) suggests hemophilia A.
Low factor IX (choice C) suggests hemophilia B.
Low or abnormal von Willebrand factor antigen (choice E) suggests von
Willebrand disease.


Following a viral illness, a previously healthy 3-year-old child develops multiple
petechiae and purpura, and is noted to be oozing blood from
the mouth. Prothrombin and activated partial thromboplastin times are within
normal limits. Review of a peripheral smear demonstrates that
platelet counts are markedly decreased, but all the platelets that are present
appear morphologically normaI. The remainder of the blood
smear and additional bone marrow studies are normaI.

Question 3 of 4

Most patients with this condition have antibodies to which of the following?
/ A. PIatelet ADP
/ B. PIatelet membrane cholesterol
/ C. PIatelet membrane glycoproteins
/ D. PIatelet membrane lipids
/ E. PIatelet thromboxane A2

Explanation - Q: 1.3 Close

The correct answer is C. The autoantibodies are usually directed against
platelet membrane glycoproteins.
ADP (choice A) and thromboxane A2 (choice E) are involved in platelet
aggregation.
Cholesterol (choice B) and lipids (choice D) are components of virtually all
mammalian plasma membranes.

Following a viral illness, a previously healthy 3-year-old child develops multiple
petechiae and purpura, and is noted to be oozing blood from
the mouth. Prothrombin and activated partial thromboplastin times are within
normal limits. Review of a peripheral smear demonstrates that
platelet counts are markedly decreased, but all the platelets that are present
appear morphologically normaI. The remainder of the blood
smear and additional bone marrow studies are normaI.

Question 4 of 4

Approximately what percentage of children with this condition will undergo
spontaneous remission?
/ A. Less than 5%
/ B. 15%
/ C. 30%
/ D. 60%
/ E. More than 80%

Explanation - Q: 1.4 Close

The correct answer is E. More than 80% of childhood cases of ITP undergo
spontaneous remission, which frequently occurs within 2 months.
Spontaneous remission is uncommon (2%) in adults, but more than 60% will
eventually recover with treatment. Treatment in children is controversial;
treatment in adults can involve splenectomy, glucocorticoids, and
intravenous IgG. Approximately 1% of children with ITP and 5% of adults
with ITP die from hemorrhagic complications, including intracranial
hemorrhage.

A 23-year-old man is admitted to the hospital through the emergency department
with probable appendicitis. He has been having right lower
quadrant abdominal pain for several days, which has been becoming
increasingly worse. His temperature 39.2 C (102.6 F), blood pressure
is 80/40 mm Hg, pulse is 120/min, and respirations are 35/min. The abdomen is
rigid with guarding. Multiple petechiae and purpura are
present, and the patient is oozing blood from his oral mucosa. According to the
patient's wife, he has not had bleeding problems in the past.

Question 1 of 5

The fact the abdomen is rigid with guarding suggests which of the following?
/ A. Colon cancer
/ B. Diverticulitis
/ C. Liver failure
/ D. Peritonitis
/ E. Shock

Explanation - Q: 2.1 Close

The correct answer is D. The usual reason for a patient to have a rigid
abdomen is that peritonitis is present, and is causing severe pain related to
peritoneal nerve fiber stimulation. The probable cause of the peritonitis is a
ruptured appendix. This patient is also probably in shock (choice E), as
indicated by the hypotension with increased respirations and heart rate, but
this would not cause the abdominal guarding. The other answers are
distracters.

A 23-year-old man is admitted to the hospital through the emergency department
with probable appendicitis. He has been having right lower
quadrant abdominal pain for several days, which has been becoming
increasingly worse. His temperature 39.2 C (102.6 F), blood pressure
is 80/40 mm Hg, pulse is 120/min, and respirations are 35/min. The abdomen is
rigid with guarding. Multiple petechiae and purpura are
present, and the patient is oozing blood from his oral mucosa. According to the
patient's wife, he has not had bleeding problems in the past.

Question 2 of 5

Given this patient's presentation, which of the following is the most likely cause of
his petechiae, purpura, and mucosal blood oozing?
/
A. Disseminated intravascular coagulation
/ B. Hemophilia A
/ C. Idiopathic thrombocytopenic purpura
/ D. Von Willebrand disease
/ E. Wiskott-AIdrich syndrome

Explanation - Q: 2.2 Close

The correct answer is A. You should suspect disseminated intravascular
coagulation (DIC) in patients who are seriously ill with other disease who
then develop evidence of a coagulopathy. In this case, sepsis is the probable
inciting event. DIC is thought to occur in 30-50% of patients with sepsis, and
may, in many cases, be triggered by a reaction to gram-negative or
staphylococcal cell wall material. Other settings in which DIC can be a
complication include major trauma, obstetric complications, acute myelocytic
leukemias, disseminated carcinomas, burns, massive transfusions, acute
hepatic failure, myocardial infarction, and inflammatory conditions (e.g.,
ulcerative colitis, rheumatoid arthritis, Crohn disease, sarcoidosis).
The other conditions listed in the choices would not be expected to be
specifically associated with sepsis, although for medical/legal and safety
reasons, supporting diagnostic tests should be ordered to exclude previously
undiagnosed disease.
Hemophilia (choice B) is a hereditary coagulopathy with X-linked genetics
that typically presents in early childhood with extensive tissue hemorrhages
and hemoarthroses following minor trauma.
Idiopathic thrombocytopenic purpura (choice C) is an acquired autoimmune
disease characterized, in adults, by chronic coagulopathy with petechiae,
purpura, and mucosal bleeding. It is not usually associated with any other
systemic diseases, such as this patient's appendicitis and probable sepsis.
Von Willebrand disease (choice D) is hereditary, with usually autosomal
dominant genetics. It typically causes prolonged menstrual bleeding and
prolonged bleeding from small cuts and surgical procedures. Petechiae and
ecchymoses are less often seen.
Wiskott-Aldrich syndrome (choice E) is a hereditary disease with X-linked
genetics, thrombocytopenia, eczema, and recurrent infection that tends to
present in children with bloody diarrhea.
A 23-year-old man is admitted to the hospital through the emergency department
with probable appendicitis. He has been having right lower
quadrant abdominal pain for several days, which has been becoming
increasingly worse. His temperature 39.2 C (102.6 F), blood pressure
is 80/40 mm Hg, pulse is 120/min, and respirations are 35/min. The abdomen is
rigid with guarding. Multiple petechiae and purpura are
present, and the patient is oozing blood from his oral mucosa. According to the
patient's wife, he has not had bleeding problems in the past.

Question 3 of 5

Which of the following findings on screening laboratory tests would be most
consistent with the probable diagnosis?
/ A. Decreased platelet count, increased prothrombin time, increased activated
partial thromboplastin time
/ B. Increased platelet count, increased prothrombin time, normal activated
partial thromboplastin time
/ C. Increased platelet count, normal prothrombin time, increased activated
partial thromboplastin time
/ D. Normal platelet count, increased prothrombin time, increased activated
partial thromboplastin time
/ E. Normal platelet count, normal prothrombin time, normal activated partial
thromboplastin time

Explanation - Q: 2.3 Close

The correct answer is A. Platelet count is invariably decreased (and usually
obvious on peripheral smear) in DIC, and this is one of the more reliable
screening tests for the condition (although it does not exclude many other
diagnoses). Both prothrombin time and activated partial thromboplastin times
are often prolonged, but you should be aware that they might each be
shortened or normal as well in DIC. Thus, these two commonly performed
tests are actually not as useful as they might appear. The other choices
listed are wrong because the platelet counts are not decreased.

A 23-year-old man is admitted to the hospital through the emergency department
with probable appendicitis. He has been having right lower
quadrant abdominal pain for several days, which has been becoming
increasingly worse. His temperature 39.2 C (102.6 F), blood pressure
is 80/40 mm Hg, pulse is 120/min, and respirations are 35/min. The abdomen is
rigid with guarding. Multiple petechiae and purpura are
present, and the patient is oozing blood from his oral mucosa. According to the
patient's wife, he has not had bleeding problems in the past.
Question 4 of 5
For confirmation of the probable diagnosis, the emergency department physician
orders a D-dimer test. D-dimer is which of the following?
/ A. A clotting factor in the common coagulation pathway
/ B. A clotting factor in the extrinsic coagulation pathway
/ C. A clotting factor in the intrinsic coagulation pathway
/ D. A factor found in the complement pathway
/ E. A fibrin degradation product

Explanation - Q: 2.4 Close

The correct answer is E. A variety of specialized tests have been
developed to help in the diagnosis of disseminated intravascular coagulation.
Unfortunately, many of these are only available in major medical centers or
through reference laboratories, which markedly limits their usefulness in
emergency rooms elsewhere. One of the more available and useful of these
tests is the D-dimer tests, which measures a fibrin degradation product that is
made when a cross-linked clot (indicating the presence of thrombin to form
the clots) is lysed by plasmin. One way to understand disseminated
intravascular coagulation is to think of it as primarily a thrombotic problem,
with excessive clot formation and then lysis, which leads secondarily to a
hemorrhagic problem as platelets and clotting factors are consumed. High D-
dimer levels indicate that clots are forming and then lysing at an unusual rate
in the body, and thus, together with a low platelet count, suggest that DIC is
present. Functionally, DIC can be subdivided into acute and chronic
presentations, which tend to occur in somewhat different settings. In the
acute presentations, the hemorrhagic processes often dominate the clinical
difficulties, while in the chronic presentations, the clotting tendencies often
dominate the picture. The D-dimers are not in themselves factors (e.g.,
substances to be consumed) in either the coagulation or complement
pathways.

A 23-year-old man is admitted to the hospital through the emergency department
with probable appendicitis. He has been having right lower
quadrant abdominal pain for several days, which has been becoming
increasingly worse. His temperature 39.2 C (102.6 F), blood pressure
is 80/40 mm Hg, pulse is 120/min, and respirations are 35/min. The abdomen is
rigid with guarding. Multiple petechiae and purpura are
present, and the patient is oozing blood from his oral mucosa. According to the
patient's wife, he has not had bleeding problems in the past.
Question 5 of 5
Which of the following would be most effective in diminishing the rate at which
clot formation is occurring?
/ A. Aminocaproic acid
/ B. Heparin
/ C. PIatelet transfusion
/ D. Red cell transfusion
/ E. Tranexamic acid

Explanation - Q: 2.5 Close

The correct answer is B. The therapy of disseminated intravascular
coagulation is complex and problematic, and, frankly, not always successful.
The patients are often already critically ill, and the DIC they have developed
often contributes to major organ failure. The basic problem in therapy is how
to treat simultaneous bleeding and clotting tendencies. Paradoxically, while
the bleeding tendency may be appearing to dominate the clinical picture,
treatment with the anticoagulant heparin may actually help the patient by
decreasing the rate at which the clotting factors are being consumed. This
must, of course, be done very carefully, since an "overshoot" may
exacerbate the patient's problems. Blood components including platelet
transfusions (choice C), red cell transfusions (choice D), and cryoprecipitate
are also sometimes used to restore the blood to a closer to normal state. As
a last resort when other measures have failed, some physicians will also try
antifibrinolytic agents such as aminocaproic acid (choice A) and tranexamic
acid (choice E) to try to keep clots from dissolving and exacerbating the
bleeding tendency.


A 7-year-old girl is evaluated by a pediatrician for a possible bleeding disorder.
The girI's father and grandmother have both had life-Iong
histories of easy bruising. On physical examination, the child is noted to have
multiple small bruises over her body, and the mother reports
that she will frequently bleed for most of an hour following a small cut. The child
has also had numerous nosebleeds that were stopped only
when the nose was packed with facial tissue. CBC is normal on all measured
values. The peripheral smear shows the usual number and
morphology of platelets. Template bleeding time is longer than normaI.
Prothrombin time is within the reference range. Activated partial
thromboplastin time is slightly longer than normaI.
Question 1 of 5
Which of the following is the most likely diagnosis?
/ A. Disseminated intravascular coagulation
/ B. Hemophilia A
/ C. Hemophilia B
/ D. Idiopathic thrombocytopenic purpura
/ E. Von Willebrand disease

Explanation - Q: 3.1 Close

The correct answer is E. Von Willebrand disease is a usually autosomal
dominant condition in which there is either a quantitative or qualitative
abnormality of von Willebrand factor (VWF) production. You should also be
aware that the disease can uncommonly be autosomal recessive or acquired
(associated with Wilms tumor, congenital heart disease, systemic lupus
erythematosus, seizures treated with valproic acid, and hypothyroidism). The
presentation and laboratory findings illustrated are typical. The disease is
life-long, and patients may also present with menorrhagia, post-operative
bleeding, postpartum bleeding, or gingival bleeding. Activated partial
thromboplastin time may be either normal or prolonged, depending on the
degree to which the intrinsic blood coagulation pathway has been inhibited
by decreased binding of factor VIII to von Willebrand factor.
Disseminated intravascular coagulation (choice A) and idiopathic
thrombocytopenic purpura (choice D) are both acquired conditions.
Hemophilia A and B (choice B and C) have X-linked genetics.

A 7-year-old girl is evaluated by a pediatrician for a possible bleeding disorder.
The girI's father and grandmother have both had life-Iong
histories of easy bruising. On physical examination, the child is noted to have
multiple small bruises over her body, and the mother reports
that she will frequently bleed for most of an hour following a small cut. The child
has also had numerous nosebleeds that were stopped only
when the nose was packed with facial tissue. CBC is normal on all measured
values. The peripheral smear shows the usual number and
morphology of platelets. Template bleeding time is longer than normaI.
Prothrombin time is within the reference range. Activated partial
thromboplastin time is slightly longer than normaI.
Question 2 of 5
Multimers of all sizes of von Willebrand's factor form complexes in plasma with
which of the following?
/ A. Factor V
/ B. Factor VI
/ C. Factor VIII
/ D. Factor IX
/ E. Factor X
Explanation - Q: 3.2 Close

The correct answer is C. Normal plasma levels of clotting factor VIII are
maintained by complexing with multimers of Von Willebrand factor. This
protects factor VIII from degradation and helps to deliver it to the site of
injury. This is the contribution VWF makes to the intrinsic pathway of
coagulation. The other factors listed do not complex with VWF.


A 7-year-old girl is evaluated by a pediatrician for a possible bleeding disorder.
The girI's father and grandmother have both had life-Iong
histories of easy bruising. On physical examination, the child is noted to have
multiple small bruises over her body, and the mother reports
that she will frequently bleed for most of an hour following a small cut. The child
has also had numerous nosebleeds that were stopped only
when the nose was packed with facial tissue. CBC is normal on all measured
values. The peripheral smear shows the usual number and
morphology of platelets. Template bleeding time is longer than normaI.
Prothrombin time is within the reference range. Activated partial
thromboplastin time is slightly longer than normaI.
Question 3 of 5
Very large von Willebrand's factor multimers contribute most to which of the
following platelet activities?
/ A. Normal adherence of platelets to damaged blood vessels
/ B. Normal budding of platelets from megakaryocytes
/ C. Normal digestion of platelets during clot lysis
/ D. Normal discharge of granules from platelets
/ E. Normal transport of platelets in the blood

Explanation - Q: 3.3 Close

The correct answer is A. The typical platelet life span begins in the bone
marrow when platelets bud off from megakaryocytes, and are released to
float freely in the blood stream. When a damaged blood vessel is
encountered, VWF acts as an adhesive bridge between the platelets and the
damaged subendothelium at the site of vascular injury. The platelets
subsequently contract and release granules to help in formation of the clot.
Eventually, the clot lyses, and macrophages consume the platelet debris.
A 7-year-old girl is evaluated by a pediatrician for a possible bleeding disorder.
The girI's father and grandmother have both had life-Iong
histories of easy bruising. On physical examination, the child is noted to have
multiple small bruises over her body, and the mother reports
that she will frequently bleed for most of an hour following a small cut. The child
has also had numerous nosebleeds that were stopped only
when the nose was packed with facial tissue. CBC is normal on all measured
values. The peripheral smear shows the usual number and
morphology of platelets. Template bleeding time is longer than normaI.
Prothrombin time is within the reference range. Activated partial
thromboplastin time is slightly longer than normaI.
Question 4 of 5
Which of the following is the binding site for the von Willebrand's factor multimers
on the platelets?
/ A. ADP
/ B. Factor VIII
/ C. GIycoprotein 1b
/ D. Thromboxane A2
/ E. Prostaglandin l2

Explanation - Q: 3.4 Close

The correct answer is C. Von Willebrand factor binds on platelets to the
specific receptor glycoprotein 1b when acting as a bridge between the
platelets and the site of vascular injury.
ADP (choice A) is released by damaged erythrocytes and helps VWF to
adhere to the exposed collagen. It is not, however, the binding factor that
connects the VWF to the platelets.
Factor VIII (choice B) does not participate directly in the platelet portion of
blood clot formation.
Thromboxane A
2
(choice D) promotes subendothelial exposure after vascular
injury and induces platelet aggregation.
Prostaglandin I
2
(choice E) inhibits subendothelial exposure after minor
vascular injury.

A 7-year-old girl is evaluated by a pediatrician for a possible bleeding disorder.
The girI's father and grandmother have both had life-Iong
histories of easy bruising. On physical examination, the child is noted to have
multiple small bruises over her body, and the mother reports
that she will frequently bleed for most of an hour following a small cut. The child
has also had numerous nosebleeds that were stopped only
when the nose was packed with facial tissue. CBC is normal on all measured
values. The peripheral smear shows the usual number and
morphology of platelets. Template bleeding time is longer than normaI.
Prothrombin time is within the reference range. Activated partial
thromboplastin time is slightly longer than normaI.
Question 5 of 5
In an individual with one of the milder forms of the disease presented in this
case, which of the following medications could be used to
increase von Willebrand's factor and factor VIII after treatment?
/ A. Aspirin
/ B. Aminocaproic acid
/ C. Desmopressin
/ D. Ibuprofen
/ E. Naproxen


Explanation - Q: 3.5 Close

The correct answer is C. Desmopressin is a synthetic analogue of
antidiuretic hormone that can be administered intravenously, intranasally, or
subcutaneously, and is frequently used as the primary treatment for bleeding
in individuals with mild von Willebrand disease. Its function in this disease is
to cause release of VWF from endothelial storage sites, so its use is
reserved for acute treatment of bleeding symptoms (including home
treatment with the high dose intranasal preparation), rather than as a routine
medication. Factor VIII concentrates that also contain VWF in high molecular
weight form are used in the treatment of patients with the more severe forms
of Von Willebrand disease. With treatment, the majority of patients,
particularly those with milder forms of Von Willebrand disease, have normal
life spans.
Aminocaproic acid (choice B) is an inhibitor of fibrinolysis that is used to
maintain clots in some individuals with von Willebrand disease.
Drugs with platelet inhibiting activities, including aspirin (choice A), ibuprofen
(choice D), and naproxen (choice E), should be avoided in patients with
disorders of hemostasis.



A child is taken to a pediatrician because the mother is concerned that the child
has been losing, rather than gaining weight, and has been
having numerous infections. After a complete physical examination with
screening serum chemistry and blood studies, the physician
suspects that the child has a malignancy.
Question 1 of 5
Which of the following is the most common malignancy of childhood?
/ A. Ewing sarcoma
/ B. Leukemia
/ C. Neuroblastoma
/ D. Retinoblastoma
/ E. Wilms tumor


Explanation - Q: 1.1 Close

The correct answer is B. Leukemia is the most common malignancy of
childhood. The other malignancies listed are also characteristically childhood
diseases, but occur less frequently.
Ewing sarcoma (choice A) can involve bone or soft tissues;
Neuroblastoma (choice C) can involve the adrenal gland or the extra-
adrenal sympathetic chain.
Retinoblastoma (choice D) involves the eye, and Wilms tumor (choice E)
involves the kidney.
A child is taken to a pediatrician because the mother is concerned that the child
has been losing, rather than gaining weight, and has been
having numerous infections. After a complete physical examination with
screening serum chemistry and blood studies, the physician
suspects that the child has a malignancy.
Question 2 of 5
The complete blood count shows anemia, thrombocytopenia, and a white count
of 110,000/mm3. The peripheral smear contains numerous
blasts, which are smalI, intensely blue cells with homogeneous chromatin,
regular nuclear shape, small or absent nucleolus, and scanty blue
cytoplasm. This patient's disease most likely will be classified as which of the
following types?
/ A. L1
/ B. L2
/ C. L3
/ D. M3
/ E. M7

Explanation - Q: 1.2 Close

The correct answer is A. These blasts cells are the lymphoid L1 cells, and
the child has ALL, type L1. L1 is one of the French-American-British
classifications for lymphoblastic leukemia, which also includes L2 and L3
types.
L2 (choice B) cells are large and heterogenous blast class with
heterogeneous chromatin, irregular nuclear shape, and large nucleolus.
L3 (choice C) cells are large and homogeneous cells whose most prominent
feature is cytoplasmic vacuolization that may overlie the nucleus.
A French-American-British classification has also been used for acute
myeloid leukemias, and different types are given different M designations.
M3 (choice D) is acute promyelocytic leukemia, and the cells have abundant
cytoplasm that often contains numerous Auer rods (small rod like structures).
M7 (choice E) is megakaryoblastic leukemia, and the cells are very large
and maybe multinucleated, resembling megakaryocytes.


A child is taken to a pediatrician because the mother is concerned that the child
has been losing, rather than gaining weight, and has been
having numerous infections. After a complete physical examination with
screening serum chemistry and blood studies, the physician
suspects that the child has a malignancy.
Question 3 of 5
Which of the following is the most common form of leukemia in childhood?
/ A. Acute lymphoblastic leukemia
/ B. Acute myeloblastic leukemia
/ C. Chronic lymphocytic leukemia
/ D. Chronic myelogenous leukemia
/ E. Erythroleukemia


Explanation - Q: 1.3 Close

The correct answer is A. Acute lymphoblastic leukemia is the most
common form of leukemia in children, and accounts for nearly 1/3 of all
pediatric cancers.

A child is taken to a pediatrician because the mother is concerned that the child
has been losing, rather than gaining weight, and has been
having numerous infections. After a complete physical examination with
screening serum chemistry and blood studies, the physician
suspects that the child has a malignancy.
Question 4 of 5
What is the peak age of incidence of the most common form of this disease in
children?
/ A. Less than 6 months
/ B. 8 -14 months
/ C. 2-5 years
/ D. 10-12 years
/ E. More than 15 years

Explanation - Q: 1.4 Close

The correct answer is C. While acute lymphoblastic leukemia can occur at
any age, its peak age of incidence is 2 to 5 years.
A child is taken to a pediatrician because the mother is concerned that the child
has been losing, rather than gaining weight, and has been
having numerous infections. After a complete physical examination with
screening serum chemistry and blood studies, the physician
suspects that the child has a malignancy.
Question 5 of 5
What is the cure rate of the most common form of this disease in children?
/ A. Less than 5%
/ B. 10-25%
/ C. 35-50%
/ D. 70-80%
/ E. More than 95%


Explanation - Q: 1.5 Close

The correct answer is D. With modern chemotherapeutic regimens, we
have now achieved a 70-80% cure rate of acute lymphoblastic leukemia. The
medical regimens with which we treat these children may include
corticosteroids (prednisone, dexamethasone), anti-neoplastic agents
(vincristine, asparaginase, daunorubicin, 6-mercaptopurine, cytarabine,
etoposide, cyclophosphamide), and prophylactic antimicrobials
(sulfamethoxazole and trimethoprim, nystatin, clotrimazole, itraconazole).
Radiotherapy is also sometimes used in these patients. Complications of
acute lymphoblastic leukemia or its treatment can include suppression of
normal bone marrow elements (predisposing for anemia, neutropenia, and
thrombocytopenia), tumor lysis syndrome (which produces hyperuricemia
and electrolyte abnormalities and can be prevented or ameliorated with
hydration and allopurinol), neurotoxicity (which may cause a mild mental
retardation and poor emotional control), and secondary malignancies.
A 22-year-old man has been having fatigue and headaches. On physical
examination, the patient is noted to be pale and has a petechial
rash. A complete blood count and peripheral smear show markedly decreased
numbers of platelets, red blood cells, granulocytes,
monocytes, and reticulocytes. The erythrocyte morphology is normaI. Bone
marrow aspiration and biopsy show a marrow with 10% cellularity,
the rest of the marrow being occupied principally by fat cells.
Question 1 of 5
Which of the following is the most likely diagnosis?
/ A. Aplastic anemia
/ B. Folate deficiency anemia
/ C. Iron deficiency anemia
/ D. Sickle cell anemia
/ E. Thalassemia

Explanation - Q: 2.1 Close

The correct answer is A. This patient most likely has aplastic anemia, which
is defined as an anemia that involves all cell lines produced in the marrow.
The underlying problem is an impaired ability of the most primitive stem cells
in the bone marrow (from which all the blood cell lines except lymphocytes
are derived) to reproduce. Patients may present with symptoms related to
anemia (pallor, headache, palpitations, dyspnea, or fatigue),
thrombocytopenia (mucosal bleeding or petechiae), or neutropenia
(infections, mouth and pharyngeal ulcerations). There is a double peak of
incidence, in the early 20's and then in older adulthood.
Folate deficiency (choice B) produces a megaloblastic anemia, and altered
neutrophils with hypersegmentation of the nucleus may be seen, but the
marrow will not be hypocellular.
Iron deficiency anemia (choice C) tends to be microcytic, and does not affect
cell lines other than erythrocytes.
Sickle cell anemia (choice D) has bizarre-shaped erythrocytes and affects
only the erythrocyte line.
Thalassemia (choice E) is characterized by hypochromic erythrocytes that
may have a "target" appearance, and does not affect cell lines other than the
erythrocytic line.

A 22-year-old man has been having fatigue and headaches. On physical
examination, the patient is noted to be pale and has a petechial
rash. A complete blood count and peripheral smear show markedly decreased
numbers of platelets, red blood cells, granulocytes,
monocytes, and reticulocytes. The erythrocyte morphology is normaI. Bone
marrow aspiration and biopsy show a marrow with 10% cellularity,
the rest of the marrow being occupied principally by fat cells.
Question 2 of 5
Which of the following is noted for sometimes causing this patient's condition?
/ A. Asbestos
/ B. Benzene
/ C. Lead
/ D. Mercury
/ E. Methyl alcohol


Explanation - Q: 2.2 Close

The correct answer is B. Approximately 80% of cases of aplastic anemia
are acquired, with the remainder being congenital. Among the acquired
cases, often no inciting cause can be identified, and an autoimmune basis
appears probable. However, in some cases, an inciting cause can be
identified, and in these, it is important to recognize and if possible remove
the cause. Benzene is the chemical most commonly cited as being
associated with development of aplastic anemia.
Associate asbestos (choice A) with pulmonary fibrosis, mesothelioma, and
lung cancer.
Associate lead poisoning (choice C) with a hypochromic, microcytic anemia,
abdominal discomfort, and CNS effects.
Associate mercury poisoning (choice D) with (depending upon the route of
exposure) GI disturbance, pneumonitis, renal failure, and CNS involvement.
Associate methyl alcohol (choice E) with visual system and central nervous
system damage.


A 22-year-old man has been having fatigue and headaches. On physical
examination, the patient is noted to be pale and has a petechial
rash. A complete blood count and peripheral smear show markedly decreased
numbers of platelets, red blood cells, granulocytes,
monocytes, and reticulocytes. The erythrocyte morphology is normaI. Bone
marrow aspiration and biopsy show a marrow with 10% cellularity,
the rest of the marrow being occupied principally by fat cells.
Question 3 of 5
Which of the following medications has been associated with the development of
this patient's condition?
/ A. Amyl nitrate
/ B. Aspirin
/ C. Chloramphenicol
/ D. CIofazimine
/ E. Dapsone


Explanation - Q: 2.3 Close

The correct answer is C. Drugs noted for causing aplastic anemia include
chloramphenicol, phenylbutazone, and gold. Cases triggered by idiosyncratic
reaction to drugs appear not to involve an immune mechanism, but may be a
result of direct toxicity, possibly involving altered detoxification.
Amyl nitrate (choice A) does not have any hematologic side effects.
Aspirin (choice B) can cause leukopenia, thrombocytopenia, and
prolongation of bleeding time, but not aplastic anemia.
Clofazimine (choice D) does not have any hematologic side effects.
Dapsone (choice E) can cause dose-related hemolysis in patients with or
without glucose-6-phosphatase deficiency.

A 22-year-old man has been having fatigue and headaches. On physical
examination, the patient is noted to be pale and has a petechial
rash. A complete blood count and peripheral smear show markedly decreased
numbers of platelets, red blood cells, granulocytes,
monocytes, and reticulocytes. The erythrocyte morphology is normaI. Bone
marrow aspiration and biopsy show a marrow with 10% cellularity,
the rest of the marrow being occupied principally by fat cells.
Question 4 of 5
With supportive care alone, this patient's condition is estimated to have which of
the following mortality rates?
/ A. Less than 5%
/ B. 10-25%
/ C. 30-40%
/ D. 50-60%
/ E. More than 70%


Explanation - Q: 2.4 Close

The correct answer is E. Supportive care consists primarily of appropriate
transfusions to replace life-threateningly low red cell counts or platelet
counts. Supportive care alone is no longer considered adequate therapy,
since aplastic anemia has a greater than 70% mortality rate if that is the only
therapy offered.
A 22-year-old man has been having fatigue and headaches. On physical
examination, the patient is noted to be pale and has a petechial
rash. A complete blood count and peripheral smear show markedly decreased
numbers of platelets, red blood cells, granulocytes,
monocytes, and reticulocytes. The erythrocyte morphology is normaI. Bone
marrow aspiration and biopsy show a marrow with 10% cellularity,
the rest of the marrow being occupied principally by fat cells.
Question 5 of 5
If treated with immunosuppression, the five year survival rate is approximately
which of the following?
/ A. 5%
/ B. 25%
/ C. 50%
/ D. 75%
/ E. 95%

Explanation - Q: 2.5 Close

The correct answer is D. Patients treated with immunosuppression have an
estimated 75% 5-year-survival rate, and those lucky individuals who have
matched sibling donor bone marrow transplant have a more than 90% 5-
year-survival. Colony stimulating cytokines (G-CSF and GM-CSF) are now
commercially available, and can also have great value in stimulating any
residual growth capacity the stem cells have.
A complete blood count (CBC) and a blood smear are taken from a patient who
has been complaining of fatigue. The CBC shows a white
count of 200,000/mm3, with differential count including 70% segmented
neutrophils and 15% band forms. The absolute numbers of basophils
and eosinophils are increased somewhat. Review of the peripheral smear does
not show any blast forms.
Question 1 of 8
Which of the following is the most likely diagnosis?
/ A. Acute lymphocytic leukemia
/ B. Acute myelogenous leukemia
/ C. Chronic lymphocytic leukemia
/ D. Chronic myelogenous leukemia
/ E. Hodgkin disease

Explanation - Q: 3.1 Close

The correct answer is D. This patient most likely has chronic myelogenous
leukemia (CML). This is a cancer of pluripotent marrow stem cells
characterized by a proliferation, predominantly of granulocytes, although
other cell lines (megakaryocytes, monocytes, erythrocytes, and even some T
and B cell lines) may also be increased.
Blast cells would be seen in the peripheral smear in acute myelogenous
leukemia (choice B) and acute lymphocytic leukemia (choice A).
In chronic lymphocytic leukemia (choice C), the numbers of lymphocytes are
increased and the numbers of other blood cells may be decreased.
Hodgkin disease (choice E) may show a modest increase in neutrophils and
eosinophils in the blood, but would not have the dramatic increase seen in
this patient.

A complete blood count (CBC) and a blood smear are taken from a patient who
has been complaining of fatigue. The CBC shows a white
count of 200,000/mm3, with differential count including 70% segmented
neutrophils and 15% band forms. The absolute numbers of basophils
and eosinophils are increased somewhat. Review of the peripheral smear does
not show any blast forms.
Question 2 of 8
Which of the following is the median age for presentation of this disease?
/ A. 1 year
/ B. 5 years
/ C. 25 years
/ D. 45 years
/ E. 75 years

Explanation - Q: 3.2 Close

The correct answer is D. Chronic myelogenous leukemia is primarily a
disease of middle age, with a median age of 45 years. The comparatively
few cases occurring in the elderly are more likely to be aggressive than those
occurring in middle-aged adults.
A complete blood count (CBC) and a blood smear are taken from a patient who
has been complaining of fatigue. The CBC shows a white
count of 200,000/mm3, with differential count including 70% segmented
neutrophils and 15% band forms. The absolute numbers of basophils
and eosinophils are increased somewhat. Review of the peripheral smear does
not show any blast forms.
Question 3 of 8
This patient's disease can be confused in some settings with polycythemia vera.
Which of the following tests would be most helpful in
distinguishing between these two diseases?
/ A. Leukocyte alkaline phosphatase
/ B. Myelin basic protein
/ C. Protein C activity
/ D. ScI-70
/ E. Tartrate resistant acid phosphatase


Explanation - Q: 3.3 Close

The correct answer is A. While to the novice, polycythemia vera and
chronic myelogenous leukemia appear to be obviously different diseases,
both cause an increase in many marrow cell lines, and ambiguous cases do
exist. Leukocyte alkaline phosphatase is a helpful test to perform in these
settings, because it is characteristically low in chronic myelogenous
leukemia, but high in polycythemia vera. Also, polycythemia vera does not
usually have the Philadelphia chromosome.
Myelin basic protein (choice B) is used in the diagnosis of multiple sclerosis.
Protein C (choice C) is involved in the clotting mechanism.
Scl-70 (choice D) is an antibody often present in scleroderma.
Associate tartrate-resistant acid phosphatase (choice E) positivity with hairy
cell leukemia.

A complete blood count (CBC) and a blood smear are taken from a patient who
has been complaining of fatigue. The CBC shows a white
count of 200,000/mm3, with differential count including 70% segmented
neutrophils and 15% band forms. The absolute numbers of basophils
and eosinophils are increased somewhat. Review of the peripheral smear does
not show any blast forms.
Question 4 of 8
Which of the following organs is most commonly significantly enlarged in these
patients?
/ A. Liver
/ B. Pancreas
/ C. Stomach
/ D. Spleen
/ E. Thymus

Explanation - Q: 3.4 Close

The correct answer is D. Splenomegaly is common in patients with chronic
myelogenous leukemia, and in more than half of the patients, is palpable
more than 5 cm below the costal margin at the time of diagnosis.
Splenomegaly may be a very helpful sign on physical examination, since it is
uncommon in reactive leukocytosis. Hepatomegaly (choice A) can also
occur, but is less common. The other answers are distracters.

A complete blood count (CBC) and a blood smear are taken from a patient who
has been complaining of fatigue. The CBC shows a white
count of 200,000/mm3, with differential count including 70% segmented
neutrophils and 15% band forms. The absolute numbers of basophils
and eosinophils are increased somewhat. Review of the peripheral smear does
not show any blast forms.
Question 5 of 8
Which of the following translocations is very common in this patient's disease?
/ A. t(8;14)
/ B. t(8;21)
/ C. t(9;22)
/ D. t(12;21)
/ E. t(15;17)

Explanation - Q: 3.5 Close

The correct answer is C. The characteristic chromosomal translocation of
chronic myelogenous leukemia is t(9;22)(q34;q11). This translocation results
in a distinctive abnormal chromosome sometimes called the Philadelphia
chromosome, and is seen in 90-95% of CML cases. The Philadelphia
translocation is also seen in 20-25% of adults with acute lymphoblastic
leukemia, 2% of adults with acute myeloid leukemia, and 2-10% of childhood
acute lymphoblastic leukemia. Some authors think that these cases may
represent variants of chronic myelogenous leukemia in which the "blast
transformation" (which commonly occurs 3-5 years after diagnosis in
untreated patients with chronic myelogenous leukemia) either develops very
early in the disease, or later, but in a patient in whom the chronic phase had
been missed clinically. The Philadelphia translocation is not usually
encountered in children with acute myeloid leukemia or in adults with chronic
lymphocytic leukemia.
T(8;14) (choice A) is a translocation with unfavorable prognosis found in
some cases of acute lymphoblastic (L3 form) leukemia.
T(8;21) (choice B) is a translocation with favorable prognosis found in some
cases of acute myelogenous leukemia (M2 form).
T(12;21) (choice D) is a translocation with favorable prognosis found in
some cases of acute lymphoblastic (L1 form) leukemia.
T(15;17) (choice E) is a translocation with favorable prognosis found in
some cases of acute myelogenous leukemia (M3 form).
A complete blood count (CBC) and a blood smear are taken from a patient who
has been complaining of fatigue. The CBC shows a white
count of 200,000/mm3, with differential count including 70% segmented
neutrophils and 15% band forms. The absolute numbers of basophils
and eosinophils are increased somewhat. Review of the peripheral smear does
not show any blast forms.
Question 6 of 8
This translocation joins which of the following two genes?
/ A. BCR and ABL
/ B. E2A and PBX1
/ C. myc and lgH
/ D. myc and kappa light chain
/ E. myc and lambda light chain

Explanation - Q: 3.6 Close

The correct answer is A. The t(9;22) Philadelphia translocation links the
Abelson oncogene (c-ABL) (normally on chromosome 9) to the BCR region
of chromosome 22. The resulting gene product is a new, chimeric, protein
called BCR-ABL, which has tyrosine kinase activity. In some manner still not
well understood, this gene product then produces the clinical problems we
diagnosis as CML.
E2A and PBX1 (choice B) describes t(1;19), seen in some cases of pre-B-
cell ALL, which links the E2A gene product (a basic helix-loop-helix
transcription factor) with the homeobox gene PBX1.
The myc oncogene and the immunoglobulin heavy chain IgH (choice C) are
joined in a t(8;14) translocation commonly seen in Burkitt's lymphoma and
type L3 acute lymphoblastic leukemia.
The translocations described in choices D and E also occur with lower
frequency in Burkitt's lymphoma and L3 acute lymphoblastic leukemia; in
these cases, the translocations produced are, respectively, t(2;8) and t(8;22).


A complete blood count (CBC) and a blood smear are taken from a patient who
has been complaining of fatigue. The CBC shows a white
count of 200,000/mm3, with differential count including 70% segmented
neutrophils and 15% band forms. The absolute numbers of basophils
and eosinophils are increased somewhat. Review of the peripheral smear does
not show any blast forms.
Question 7 of 8
Which of the following specifically inhibits the abnormal gene product produced
by the translocation seen in this disease?
/ A. Busulfan
/ B. Hydroxyurea
/ C. Imatinib mesylate
/ D. Interferon alfa-2a
/ E. Interferon alfa-2b

Explanation - Q: 3.7 Close

The correct answer is C. Imatinib mesylate (Gleevec) is an important new
drug that specifically targets the abnormal gene product (a tyrosine kinase)
of the BCR-ABL translocation. This new drug is well absorbed after oral
administration and can be used to treat both the chronic phase and blast
crisis of CML. It also shows promise in potentially being used in autologous
(self-to-self) bone marrow transplant, where it may be possible to clear the
transplant of leukemic cells in vitro with chemotherapy before reinjecting the
transplant. The other agents listed in the choices are also used in the
chemotherapy of CML.
Busulfan (choice A) is an alkylating agent, hydroxyurea (choice B) inhibits
DNA synthesis, and recombinant forms of interferons (choices D and E) can
help induce remissions in CML.

A complete blood count (CBC) and a blood smear are taken from a patient who
has been complaining of fatigue. The CBC shows a white
count of 200,000/mm3, with differential count including 70% segmented
neutrophils and 15% band forms. The absolute numbers of basophils
and eosinophils are increased somewhat. Review of the peripheral smear does
not show any blast forms.
Question 8 of 8
The cure rate with patients who have this condition and undergo bone marrow
transplant is now estimated to be which of the following?
/ A. 5%
/ B. 15%
/ C. 25%
/ D. 50%
/ E. 80%


Explanation - Q: 3.8 Close

The correct answer is D. In the past, the average life expectancy of CML
patients has been 3-5 years, with most patients dying after a blast crisis
develops. The blast crisis may produce either clinical acute lymphoblastic
leukemia or acute myeloid leukemia. This was historically true despite
chemotherapy and any apparent improvements in blood counts with
chemotherapy. Bone marrow transplant is becoming an increasingly
preferred treatment modality in those cases in which a suitable donor can be
found, because it now has a cure rate of approximately 50%. The mortality
rate of this procedure is 10-20% or less if a matched sibling donor is used
and increases to 30-40% if an unrelated donor is used.


A 17-year-old African American with type 1 diabetes spends a weekend camping
with friends. A day after returning, he is brought into the
emergency department because he is in diabetic ketoacidosis. In the emergency
room, he is noted to be jaundiced. The laboratory calls the
emergency department physician because the plasma has a red color.
Question 1 of 5
Which of the following enzymatic deficiencies would most likely cause a
hemolytic episode such as this man is experiencing?
/ A. Acid sphingomyelinase deficiency
/ B. GIucocerebrosidase deficiency
/ C. GIucose-6-phosphate dehydrogenase deficiency
/ D. Ferrochelatase deficiency
/ E. Phytanic acid alpha-oxidase deficiency


Explanation - Q: 4.1 Close

The correct answer is C. Of the enzyme deficiencies listed, only glucose-6-
phosphate dehydrogenase deficiency (G-6-PD) would cause hemolysis. The
cells most likely to be affected are usually the older erythrocytes. Most
patients with this condition have long asymptomatic intervals and only
undergo hemolysis when a metabolic stress occurs. While elementary
discussions of this condition usually stress the ingestion of fava beans or the
taking of medicines (e.g., sulfonamides, antimalarial drugs) as triggers of
hemolysis, you should be aware that in real life in the United States, the most
common triggers are fever, acute viral and bacterial infections, and diabetic
ketoacidosis. Once the diagnosis of G6PD deficiency is suspected, the
actual enzyme activity can be assayed in the laboratory to establish the
diagnosis.
Acid sphingomyelinase deficiency (choice A) produces the lysosomal
storage disease Niemann-Pick disease.
Glucocerebrosidase deficiency (choice B) produces the lipid storage
disease Gaucher disease.
Ferrochelatase deficiency (choice D) produces erythropoietic protoporphyria.
Phytanic acid alpha-oxidase deficiency (choice E) produces the storage
disease Refsum disease.
A 17-year-old African American with type 1 diabetes spends a weekend camping
with friends. A day after returning, he is brought into the
emergency department because he is in diabetic ketoacidosis. In the emergency
room, he is noted to be jaundiced. The laboratory calls the
emergency department physician because the plasma has a red color.
Question 2 of 5
Which of the following is the approximate incidence of this enzymatic defect in
African-American males?
/ A. Less than 0.1%
/ B. 1%
/ C. 10%
/ D. 30%
/ E. 90%


Explanation - Q: 4.2 Close

The correct answer is C. The incidence of glucose-6-phosphate deficiency
in American black males is thought to be about 12%. The condition can also
be found in lower frequency in black females (who either have both X
chromosomes involved or who have one affected X chromosome with the
other inactivated as a Barr body in a high percentage of the body). G6PD
deficiency has a worldwide distribution, but occurs most frequently in tropical
and subtropical areas of the Eastern hemisphere. Populations that have a
significant frequency of G6PD deficiency include Brazilian Blacks, people
from the lowlands of the Mediterranean basin, especially those of Greek or
Sardinian descent, Kurdish Jews and the Southern Chinese. Most affected
individuals are asymptomatic. However, individuals from ethnic backgrounds
with high frequencies of G6PD deficiency should be checked for it if
treatment of an illness requires a medication known to precipitate hemolysis.
G6PD deficiency should be also be considered in the differential diagnosis of
any nonimmune hemolytic anemia.

A 17-year-old African American with type 1 diabetes spends a weekend camping
with friends. A day after returning, he is brought into the
emergency department because he is in diabetic ketoacidosis. In the emergency
room, he is noted to be jaundiced. The laboratory calls the
emergency department physician because the plasma has a red color.
Question 3 of 5
Which of the following is a product of the enzyme for which the patient has a
deficiency?
/ A. FADH2
/ B. NAD+
/ C. NADH
/ D. NADP+
/ E. NADPH

Explanation - Q: 4.3 Close

The correct answer is E. Glucose-6-phosphate dehydrogenase is the
committed step of the hexose monophosphate shunt pathway (pentose
phosphate pathway). The enzymes of this pathway are present in the
cytoplasm of all cells, but are especially prevalent in adipose and liver cells.
The hexose monophosphate shunt pathway makes ribose-5-phosphate for
nucleotide synthesis, interconverts 6 carbon sugars and 5 carbon sugars,
and produces NADPH. NADPH, the reduced form of nicotinamide adenine
dinucleotide phosphate, is used in reductive biosynthetic reactions such as
fatty acid synthesis, and in the regeneration of glutathione by glutathione
reductase. Glutathione has three main roles in a cell: it keeps the protein
sulfhydryl groups of cysteines reduced, it reduces peroxides, and it maintains
hemoglobin in the reduced state.
NADP+ (choice D) is also involved in this reaction, but as a substrate, rather
than a product.
The various forms of NAD (choices B and C) and FAD (choice A) do not
take part.

A 17-year-old African American with type 1 diabetes spends a weekend camping
with friends. A day after returning, he is brought into the
emergency department because he is in diabetic ketoacidosis. In the emergency
room, he is noted to be jaundiced. The laboratory calls the
emergency department physician because the plasma has a red color.
Question 4 of 5
Regeneration of which of the following in a reduced form is impaired in
individuals with this disorder?
/ A. Acetaldehyde
/ B. AcetyI CoA
/ C. Biotin
/ D. GIutathione
/ E. GIycogen


Explanation - Q: 4.4 Close

The correct answer is D. Reduced glutathione is an effective scavenger of
dangerous oxidative metabolites; regeneration of reduced glutathione is
accomplished by glutathione reductase with the aid of NADPH, a product of
the hexose monophosphate shunt pathway. Red blood cells that are deficient
for G6PD, and are exposed to oxidants, can be depleted of their store of
glutathione. The lack of glutathione leads to the oxidation of the sulfhydryl
groups on hemoglobin, causing the formation of methemoglobin and then
denatured globin, which forms insoluble masses called Heinz bodies that are
attached to the RBC membrane. The overall outcome is that the red blood
cells become rigid, making them susceptible to destruction by the
reticuloendothelial system.
The other answers are distracters.

A 17-year-old African American with type 1 diabetes spends a weekend camping
with friends. A day after returning, he is brought into the
emergency department because he is in diabetic ketoacidosis. In the emergency
room, he is noted to be jaundiced. The laboratory calls the
emergency department physician because the plasma has a red color.
Question 5 of 5
In a Nigerian population, the frequency of males with G6PD deficiency is 0.2.
Assuming the population is in Hardy-Weinberg equilibrium at
this locus, what is the frequency of homozygous females?
/ A. 0.02
/ B. 0.04
/ C. 0.2
/ D. 0.32
/ E. 0.45


Explanation - Q: 4.5 Close

The correct answer is B. Glucose 6-phosphate dehydrogenase deficiency
is an X-linked recessive trait. A male is hemizygous for the X chromosome,
and thus has only one copy of each trait located on that chromosome. The
frequency of an X-linked recessive in males is thus equal to the frequency of
the allele in the population. From this, we know that q= 0.2 and p = 0.8. A
woman has two copies of each gene on the X chromosome, so the equation
for Hardy Weinberg equilibrium is the same as for autosomal traits. In this
case, a homozygous recessive female would occur at a frequency of q
2
or
0.04.
Choice A, 0.02, is incorrect. If a person remembered that G6PD was more
frequent in males, but didn't know how to use the equations to get the true
estimate, he or she might guess this answer.
Choice C, 0.2, makes the assumption that the trait is autosomal, and so the
frequency of affected males and affected females are equal.
Choice D, 0.32, assumes that q=0.01, and then calculates the frequency of
carrier females (2pq).
Choice E, 0.45 is a distracter

A follow-up physical examination is performed on a 7-year-old child with known
celiac disease, which has been difficult to controI. A complete
blood count is done as part of the evaluation, which demonstrates a moderate
anemia. Review of the peripheral blood smear shows
numerous enlarged, hypochromic erythrocytes. Some of the neutrophils show
hypersegmentation of the nuclei, with up to 8 Iobes, rather than
the usuaI 3.
Question 1 of 5
Deficiency of which of the following would most likely cause this child's anemia?
/ A. Iron
/ B. Folate
/ C. Vitamin A
/ D. Vitamin C
/ E. Vitamin K

Explanation - Q: 5.1 Close

The correct answer is B. The combination of enlarged, hypochromic
erythrocytes with hypersegmented neutrophils indicates that the patient has
a megaloblastic anemia. Of the causes listed in the choices, only folate
deficiency will produce these changes. Folate deficiency can also be caused
by inadequate intake (elderly, alcoholics, infants), increased requirements
(pregnancy, infancy, high metabolic states), impaired utilization
(methotrexate therapy), or increased loss (hemodialysis).
Iron deficiency (choice A) usually produces a microcytic, hypochromic
anemia.
Vitamin A deficiency (choice C) causes night blindness.
Vitamin C deficiency (choice D) causes scurvy and poor wound healing.
Vitamin K deficiency (choice E) causes a bleeding tendency.

A follow-up physical examination is performed on a 7-year-old child with known
celiac disease, which has been difficult to controI. A complete
blood count is done as part of the evaluation, which demonstrates a moderate
anemia. Review of the peripheral blood smear shows
numerous enlarged, hypochromic erythrocytes. Some of the neutrophils show
hypersegmentation of the nuclei, with up to 8 Iobes, rather than
the usuaI 3.
Question 2 of 5
Deficiency of which the following can also cause this patient's type of anemia?
/ A. Calcium
/ B. Magnesium
/ C. Vitamin B12
/ D. Vitamin D
/ E. Vitamin E

Explanation - Q: 5.2 Close

The correct answer is C. Vitamin B12 deficiency can also cause
megaloblastic anemia.
Calcium and vitamin D deficiencies (choices A and D) can cause
osteoporosis and rickets.
Magnesium deficiency (choice B) is usually seen in settings in which
patients are very ill for other reasons, but appears to cause GI symptoms,
personality change, and tetany.
Vitamin E (choice E) is an antioxidant. Deficiency may produce mild
hemolytic anemia or neurologic (spinocerebellar) disease.

A follow-up physical examination is performed on a 7-year-old child with known
celiac disease, which has been difficult to controI. A complete
blood count is done as part of the evaluation, which demonstrates a moderate
anemia. Review of the peripheral blood smear shows
numerous enlarged, hypochromic erythrocytes. Some of the neutrophils show
hypersegmentation of the nuclei, with up to 8 Iobes, rather than
the usuaI 3.
Question 3 of 5
The substance most likely deficient in this child is normally absorbed primarily at
which of the following sites?
/ A. Colon
/ B. Distal ileum
/ C. Duodenum and jejunum
/ D. Esophagus
/ E. Stomach


Explanation - Q: 5.3 Close

The correct answer is C. Folate is absorbed in the proximal small bowel
and vitamin B12 is absorbed in the ileum (choice B). Thus poorly controlled
celiac disease, which principally affects the proximal small intestine, is more
likely to cause folate deficiency than B12 deficiency.
The esophagus (choice D) and stomach (choice E) are not major absorptive
sites; and the colon (choice A) principally absorbs fluid and electrolytes.
A follow-up physical examination is performed on a 7-year-old child with known
celiac disease, which has been difficult to controI. A complete
blood count is done as part of the evaluation, which demonstrates a moderate
anemia. Review of the peripheral blood smear shows
numerous enlarged, hypochromic erythrocytes. Some of the neutrophils show
hypersegmentation of the nuclei, with up to 8 Iobes, rather than
the usuaI 3.
Question 4 of 5
The two substances that can cause this type of anemia if deficient, are necessary
for synthesis of which of the following?
/ A. AcetyI CoA
/ B. ADP
/ C. ATP
/ D. NADPH
/ E. Tetrahydrofolate (THF)

Explanation - Q: 5.4 Close

The correct answer is E. Vitamin B12 is a cofactor in the activation of folic
acid to tetrahydrofolate (THF). The THF is necessary for DNA base
synthesis, and careful evaluation will demonstrate megaloblastic changes in
all 3 hematopoietic lines. The other answers are distracters.
A follow-up physical examination is performed on a 7-year-old child with known
celiac disease, which has been difficult to controI. A complete
blood count is done as part of the evaluation, which demonstrates a moderate
anemia. Review of the peripheral blood smear shows
numerous enlarged, hypochromic erythrocytes. Some of the neutrophils show
hypersegmentation of the nuclei, with up to 8 Iobes, rather than
the usuaI 3.
Question 5 of 5
Deficiency of vitamin B12 can produce damage to which of the following?
/ A. Cardiovascular system
/ B. Central nervous system
/ C. Female reproductive system
/ D. Hepatobiliary system
/ E. Urinary system

Explanation - Q: 5.5 Close

The correct answer is B. Vitamin B12 functions in transfer of 1-carbon
fragments, and has a role in other pathways besides DNA synthesis. Vitamin
B12 deficiency can also cause demyelination and myelopathy, with resultant
CNS degeneration. The other choices are distracters.


_ .
A 2-year-old African American child is evaluated by a pediatrician because the
mother noticed that the child had been behaving listlessly for
several months. Office hematocrit is 29%. A peripheral blood smear is shown
above.
Question 1 of 6
Which of the following is the most likely diagnosis?
/ A. AIpha thalassemia major
/ B. Beta thalassemia major
/ C. Beta thalassemia minor
/ D. Sickle cell anemia
/ E. Sickle cell trait


Explanation - Q: 6.1 Close

The correct answer is D. This child has sickle cell anemia. This is an
autosomal recessive, chronic hemolytic anemia characterized by sickle- or
crescent-shaped erythrocytes. Almost all cases are individuals with at least
some African ancestry. In African Americans, the prevalence of sickle cell
anemia is about 0.3%.
The various thalassemias (choices A, B, and C) can be excluded because
the peripheral smear findings would include a reference to or a description of
target cells. In questionable cases, or when the possibility of coexisting
disease is present, hemoglobin electrophoresis can distinguish between the
various diseases noted in the choices.
Sickle cell trait (choice E), or the heterozygote and milder form of sickle cell
disease, can be excluded because the patient is frankly anemic with a very
low hematocrit. Sickle cell trait has a prevalence in African Americans of
about 8-13%. These individuals are usually asymptomatic, but may develop
clinical disease in unusual situations (such as with severe infections).


Question 2 of 6
The disease is caused by a nucleotide substitution at position +70 in the gene
region. The mutation also destroys a restriction endonuclease
site from +66 to +72 in the gene. Using a probe that binds just upstream from the
mutation, restriction patterns from his mother and from his
father were obtained and are shown at the left on the diagram below.





Which of the patterns on the right most likely represents that of the child?
/ A. A
/ B. B
/ C. C
/ D. D
/ E. E

Explanation - Q: 6.2 Close

The correct answer is E. The mutation that causes the disease also
destroys a restriction enzyme site. Loss of a restriction site would produce a
longer restriction fragment that would be retained higher on the gel. Thus, in
both parents' patterns, the upper band must be cut from the chromosome
bearing the mutation. The affected child would be homozygous for this
mutation and show only the upper band in his pattern. The band is thicker,
indicating more material. The same size restriction fragment is cut from both
his chromosomes.
Choice A shows a homozygous normal individual.
Choice B shows an individual with a band representing a shorter fragment
not seen in either parent.
Choice C shows an individual with a band representing a longer fragment
not seen in either parent. If the mutation involved a triplet repeat expansion,
a pattern like this might be generated. It is not correct for the mutation
described in the stem.
Choice D shows a carrier.

Question 3 of 6
The prevalence of this disease in African Americans is about 1/625. The boy's
uncle has been tested and knows that he is a carrier of the
mutation involved. If he marries an African American woman, what is the risk
their first child will be affected with this disease?
/ A. 1/2500
/ B. 1/1200
/ C. 1/125
/ D. 1/100
/ E. 1/50


Explanation - Q: 6.3 Close

The correct answer is E. If the prevalence of the disease (q
2
) were 1/625,
then the carrier frequency (2pq) in the same population would be 2(1/25) or
2/25

The risk of having an affected child is then: 2/25 x 1/4 = 2/100 or 1/50. The
risk is the same for each child, regardless of birth order.

Question 4 of 6
Compared to the prevalence in African Americans (1/625), the prevalence of this
disease in Africans may be as high as 1/50. Which of the
following genetic mechanisms contributes to this disparity?
/ A. Founder effect
/ B. Gene flow
/ C. Genetic drift
/ D. Incomplete penetrance
/ E. Increased mutation rate in Africans as compared to African Americans.


Explanation - Q: 6.3b Close

The correct answer is E. If the prevalence of the disease (q
2
) were 1/625,
then the carrier frequency (2pq) in the same population would be 2(1/25) or
2/25

The risk of having an affected child is then: 2/25 x 1/4 = 2/100 or 1/50. The
risk is the same for each child, regardless of birth order.


Question 4 of 6
Compared to the prevalence in African Americans (1/625), the prevalence of this
disease in Africans may be as high as 1/50. Which of the
following genetic mechanisms contributes to this disparity?
/ A. Founder effect
/ B. Gene flow
/ C. Genetic drift
/ D. Incomplete penetrance
/ E. Increased mutation rate in Africans as compared to African Americans.

Explanation - Q: 6.4 Close

The correct answer is B. Gene flow refers to the exchange of genes among
populations. When two populations, previously isolated reproductively, begin
to intermarry, gene frequencies within both populations will change due to
this flow, eventually reaching new equilibria. In either of the populations,
some gene frequencies will increase and some will decrease. Another cause
of the higher prevalence of sickle cell disease in Africans is the continued
selective pressure of malaria, not present in the U.S. Natural selection is not
given as an option in this question.
A founder effect (choice A) refers to the introduction of a mutation into a
rapidly expanding population by an individual who was part of a small
ancestral group. Both populations in this question share the same genetic
origin (Africa) and carry the same mutation. A founder effect would not
account for this disparity in gene frequency.
Genetic drift (choice C) refers to the random fluctuations of gene
frequencies in small populations. Some genes can increase in frequency,
others may decrease, when compared to the gene frequencies seen in larger
populations. Gene frequencies are statistical measurements, and small
populations are more likely to vary from the statistical mean than large
populations. Consider a family in which both parents are carriers of an
autosomal recessive disease-producing gene. Statistics indicate that if the
family has enough children, 25% will be AA, 50% will be Aa, and 25% will be
aa. If the family only has 4 children, there is some random chance that 2 will
be affected (e.g. 50% rather than 25%)
Incomplete penetrance (choice D) refers to the fact that not all individuals
with the disease-producing genotype have the disease (phenotype). The
same mutation should show nearly identical penetrance in two populations,
and would not account for the differences described in this question.
Mutation rates in large populations tend to be the same (compare to choice
E), and do not account for major differences in gene frequencies.
Question 5 of 6
Which of the following viruses most commonly causes aplastic crises in patients
with this disease?
/ A. BK virus
/ B. Human herpes virus 8
/ C. Human papilloma virus 18
/ D. JC virus
/ E. Parvovirus B19


Explanation - Q: 6.5 Close

The correct answer is E. Sickle cell patients are vulnerable to a variety of
potentially life-threatening clinical crises. Important causes of aplastic crises
(also called hematologic crises), in which the marrow fails to produce
adequate numbers of erythrocytes, include parvovirus B19 infection and
folate deficiency. For your personal information, although the question is
usually asked about sickle cell anemia, these two inciting agents can also
induce exacerbation of anemia in individuals with thalassemia and a variety
of other hereditary red cell disorders. Simple transfusions can be used to
treat aplastic crises.
BK virus (choice A) causes renal disease in immunocompromised hosts.
Human herpes virus 8 (choice B) has been implicated in Kaposi sarcoma.
Human papilloma virus 18 (choice C) is an important cause of cervical
dysplasia and cancer.
JC virus (choice D) causes progressive multifocal leukoencephalopathy.

Question 6 of 6
At age 5, the boy and his mother take a trip in a friend's small plane. On landing,
the boy complains of very severe pain involving his bones
and abdomen. Which of the following problems most likely developed?
/ A. Aplastic crisis
/ B. Hematologic crisis
/ C. Infectious crisis
/ D. Osteomyelitis
/ E. Vasoocclusive crisis

Explanation - Q: 6.6 Close

The correct answer is E. This patient probably developed a vasoocclusive
crisis, which occurs when there is an exacerbation of red cell sickling. The
increased sickling leads to obstruction of the microcirculation with sickled
erythrocytes, which in turn produces ischemia with its accompanying severe
pain (analogous to the pain of angina and myocardial infarction). The pain
most frequently involves bones, abdomen, and/or chest. Involvement of the
nervous system can produce focal neurologic signs and symptoms.
Involvement of the chest can produce acute chest syndrome (due to
microvascular occlusion in the pulmonary capillary bed), and is the major
cause of death in children less than 5 years of age who have sickle cell
disease. Precipitating causes for vasooclusive crises include cold weather
(which induces vasospasm), hypoxia (as in this patient who flew in an
unpressurized airplane), infection, dehydration, acidosis, alcohol intoxication,
emotional stress, and pregnancy. Vasoocclusive crises are usually treated
with hydration and analgesia. In severe cases, or if acute chest syndrome
develops, exchange blood transfusion is sometimes used to reduce the
number of circulating sickled cells. Important triggers for aplastic or
hematologic crises (choices A and B) are discussed in the explanation to
the preceding question. Infections (choice C), which can include
osteomyelitis (choice D), are usually related to these patients' functional
asplenia (due to splenic ischemia and infarction) and resultant defective
immunity. Encapsulated organisms (Haemophilus influenzae and
Streptococcus pneumoniae) are a particular vulnerability, but there is also an
increased vulnerability (apparently related to altered serum IgM levels,
impaired opsonization, and poor complement pathway function) to some
other organisms including Mycoplasma pneumoniae, Salmonella
typhimurium, Staphylococcus aureus, and Escherichia coli.

A 62-year-old woman consults her physician because she has been having
headaches, dizziness, and tinnitus. The patient also reports that
her skin seems to be a great deal more itchy than several years ago, and that the
episodes of itchiness are sometimes triggered by a hot
shower. Physical examination demonstrates hypertension, splenomegaly, and a
ruddy complexion seen best in the face, palms, and nail
beds. The patient did not appear to be dehydrated.
Question 1 of 4
The patient has an erythrocyte count of 6.7 million/mm3. Which of the following
studies would be helpful in establishing that an absolute
increase in erythrocyte mass has occurred?
/ A. Free erythrocyte protoporphyrin
/ B. Hematocrit
/ C. Hemoglobin
/ D. Partial thromboplastin time
/ E. Radioactive chromium labeling

Explanation - Q: 7.1 Close

The correct answer is E. The dilution of small numbers of
51
Cr-labelled
erythrocytes in the patient's blood can be used to calculate the patient's red
cell mass. This is important in cases of erythrocytosis, because it allows the
exclusion of hemoconcentration effects (relative erythrocytosis) not related to
a true increase in red cell mass.
Free erythrocyte protoporphyrin (choice A) is used in the evaluation of the
porphyrias.
Hematocrit (choice B) and hemoglobin (choice C) cannot distinguish
between a relative and an absolute erythrocytosis.
Partial thromboplastin time (choice D) is used in the evaluation of clotting
disorders.
A 62-year-old woman consults her physician because she has been having
headaches, dizziness, and tinnitus. The patient also reports that
her skin seems to be a great deal more itchy than several years ago, and that the
episodes of itchiness are sometimes triggered by a hot
shower. Physical examination demonstrates hypertension, splenomegaly, and a
ruddy complexion seen best in the face, palms, and nail
beds. The patient did not appear to be dehydrated.
Question 2 of 4
If the erythrocytosis is related to ectopic production of erythropoietin, which of the
following tumors would be the most likely source of the
erythropoietin?
/ A. Adenocarcinoma of the breast
/ B. Endometrial adenocarcinoma
/ C. Hepatocellular carcinoma
/ D. Renal cell carcinoma
/ E. Serous cystadenoma of the ovary


Explanation - Q: 7.2 Close

The correct answer is D. Erythropoietin is normally produced in the kidneys,
and renal cell carcinoma can produce it ectopically, producing a form of
secondary erythrocytosis. The other answers are distracters.
A 62-year-old woman consults her physician because she has been having
headaches, dizziness, and tinnitus. The patient also reports that
her skin seems to be a great deal more itchy than several years ago, and that the
episodes of itchiness are sometimes triggered by a hot
shower. Physical examination demonstrates hypertension, splenomegaly, and a
ruddy complexion seen best in the face, palms, and nail
beds. The patient did not appear to be dehydrated.
Question 3 of 4
In addition to the increased red cell count, this patient has a leukocytosis of 15 x
103per microliter, with increases in neutrophils, eosinophils,
and basophils. Her platelet count is 450 x 103per microliter. Which of the
following is most likely to be contributing to this woman's pruritus?
/ A. Basophils
/ B. Eosinophils
/ C. Erythrocytes
/ D. Neutrophils
/ E. PIatelets

Explanation - Q: 7.3 Close

The correct answer is A. This patient's disease is closely related to chronic
myelogenous leukemia, and increases in a variety of cell lines are seen,
although the most striking increase is in the erythrocyte line. Basophils and
mast cells are interrelated cell types that can both release histamine and a
variety of vasoactive chemicals and cytokines. An occasionally helpful clinical
clue is that itchiness not related to an obvious rash can be due to increased
numbers of these cell lines.
High eosinophil (choice B) counts might be associated with itchiness, but in
non-allergic diseases, the high mast cell and basophil counts are more likely
to cause symptoms.
Erythrocytes (choice C), neutrophils (choice D), and platelets (choice E) do
not contain granules that would be likely to cause itchiness.

A 62-year-old woman consults her physician because she has been having
headaches, dizziness, and tinnitus. The patient also reports that
her skin seems to be a great deal more itchy than several years ago, and that the
episodes of itchiness are sometimes triggered by a hot
shower. Physical examination demonstrates hypertension, splenomegaly, and a
ruddy complexion seen best in the face, palms, and nail
beds. The patient did not appear to be dehydrated.
Question 4 of 4
If this patient has a normal arteriaI O2 saturation level and low erythropoietin,
which of the following is the most likely diagnosis?
/ A. Chronic lung disease
/ B. Chronic myelogenous leukemia
/ C. Essential thrombocythemia
/ D. Polycythemia vera
/ E. Tumor-associated erythrocytosis


Explanation - Q: 7.4 Close

The correct answer is D. Polycythemia vera is an idiopathic chronic
myeloproliferative disorder that is characterized by an increased red cell
mass. For the diagnosis to be formally made, the patient must have either all
major criteria for diagnosis, or the first two major criteria plus any two minor
criteria. Major criteria include (1) a red cell mass equal to or greater than 36
mL/kg in men and 32 mL/kg in women; (2) an arterial O
2
saturation greater
than 92%; and (3) splenomegaly (related to extramedullary hematopoiesis).
Minor criteria include (1) thrombocytosis greater than 400 x 10
3
/microliter;
leukocytosis greater than 12 x 10
3
/microliter; leukocyte alkaline phosphatase
activity greater than 100 in the absence of fever and infection (this helps
exclude chronic myelogenous leukemia, choice B); and serum B12 greater
than 900 pg/mL or unsaturated B12-binding capacity greater than 2200
pg/mL (B12 abnormality is common in polycythemia vera). This condition is
closely related to both chronic myelogenous leukemia and essential
thrombocythemia (choice C), in that all cell lines produced in the marrow
tend to be increased, although in the case of polycythemia vera, the
predominant increase is in the erythrocyte line. Patients are usually treated
with recurrent phlebotomy (removal of blood as for a donation) to prevent
complications related to hyperviscosity of the blood (venous thrombosis,
stroke, hepatic portal vein thrombosis, angina pectoris, intermittent
claudication). These patients are also vulnerable to bleeding complications
(related to abnormal platelets), peptic ulcer disease (related to high
histamine levels from basophils and mast cells), and pruritus. Some cases of
polycythemia vera eventually "burn out" and the patient becomes anemic and
requires transfusions. Patients are also vulnerable to developing secondary
leukemias as either a direct complication of their disease or as a
complication of therapy.
Chronic lung disease (choice A) severe enough to cause erythrocytosis
would not show a normal O
2
saturation.
Tumor-associated erythrocytosis (choice E) would show a high
erythropoietin.

A 3-year-child is taken to a physician for a routine physical examination. On
questioning about any problems she may have noticed that
concern her, the mother comments that the child does not seem to be feeling
well much of the time. She has noticed that the child is irritable,
sleeps poorly, and is often constipated. Physical examination is unremarkable.
Office hematocrit shows a mild anemia.
Question 1 of 6
The physician considers the possibility that the child has an environmental
illness. Which of the following is the most common environmental
illness of children in the United States?
/ A. Arsenic poisoning
/ B. Bismuth poisoning
/ C. Copper poisoning
/ D. Insecticide poisoning
/ E. Lead poisoning

Explanation - Q: 8.1 Close

The correct answer is E. Lead poisoning is thought to be the most common
environmental illness of children in this country. A high index of suspicion is
warranted, because no pathognomic symptoms exist and the physical
findings are usually negative. Symptoms that can be seen are often vaguely
neurologic (irritability, either sleeplessness or excessive lethargy, poor
appetite, headaches, vague changes in activity level). Abdominal pain with or
without vomiting can also be seen.
Arsenic poisoning (choice A) is less common than lead poisoning in this
country, and is mostly likely to be the result of either ground-water
contaminated by flowing through arsenic bearing rocks, or ingestion of rat
poison.
Bismuth (choice B) is used industrially and is not usually found in
environments to which children are exposed.
Copper (choice C) is relatively nontoxic, although it can contribute to liver
disease in patients with abnormal copper metabolism (e.g., Wilson's
disease).
Insecticide poisoning (choice D) can occur in children with some frequency
since insecticides are common in households, but this is less frequent than
lead poisoning.
A 3-year-child is taken to a physician for a routine physical examination. On
questioning about any problems she may have noticed that
concern her, the mother comments that the child does not seem to be feeling
well much of the time. She has noticed that the child is irritable,
sleeps poorly, and is often constipated. Physical examination is unremarkable.
Office hematocrit shows a mild anemia.
Question 2 of 6
The child's family is poor, and lives in an apartment built in the 1930's and never
substantively remodeled. Rodents and insects are common
in the building, as are pesticides used to try to eliminate them. If the child does
have the most common environmental illness in US children,
which of the following would be the most likely source of the poison?
/ A. Crumbling sidewalk fragments
/ B. OId metal doorknob
/ C. OId paint chips
/ D. Rat poison
/ E. Roach poison


Explanation - Q: 8.2 Close

The correct answer is C. In the past, the interior paint used on houses often
contained lead, which provided a very white color to the paint (to which other
colors could be added to produce vibrant paints). These paints are no longer
in use, but there is enough very old housing still present in this country that
lead poisoning remains a significant problem. The old paint often chips off
and the chips may be ingested by toddlers. The affected children have often
been bottle-fed for protracted periods and have developed some degree of
pica (ingestion of non-food substances). Lead was also commonly used in
plumbing pipes in early houses as well. In black, non-Hispanic children living
in homes built before 1946, the prevalence of lead poisoning is almost 22%.
The rates drop to 13.7% in those living in homes built from 1946 to 1973, and
then to 3.4% in homes built after 1973. Rates of lead poisoning are lower
among Mexican-American children (13%, 2.3% and 1.6%, respectively) and
white, non-Hispanic children (5.6%, 1.4%, and 1.5%, respectively). Lead
poisoning from paints also is occasionally a problem when decorative dishes
are used to contain acid foods (the classic example is an imported Mexican
pitcher used for orange juice) that leak the lead. Also, exposures can come
from some folk remedies and from adults working in lead-related
occupations.
The crumbling sidewalk fragments (choice A) and the old metal doorknob
(choice B) are probably non-toxic.
Rat poison (choice D) can contain arsenic and roach poison (choice E) can
contain organophosphate insecticides.
A 3-year-child is taken to a physician for a routine physical examination. On
questioning about any problems she may have noticed that
concern her, the mother comments that the child does not seem to be feeling
well much of the time. She has noticed that the child is irritable,
sleeps poorly, and is often constipated. Physical examination is unremarkable.
Office hematocrit shows a mild anemia.
Question 3 of 6
Which of the following is traditionally associated with this child's disease, but is
actually now uncommonly seen?
/ A. Aphthous ulcer on inside of cheek
/ B. Dark line on the gums
/ C. Fissures on the tongue
/ D. Hairy leukoplakia of the tongue
/ E. Pyogenic granuloma of gums


Explanation - Q: 8.3 Close

The correct answer is B. The "lead-line," which is due to lead deposition
along the gums, is traditionally associated with lead poisoning, but has
become uncommon because we are picking up most cases of lead poisoning
earlier, when the body load of lead is smaller. The physician should look for it
in suspected cases of lead poisoning, even though he or she probably won't
find it.
Aphthous ulcers (choice A), commonly called "canker sores," are idiopathic
benign oral ulcers that can occur with emotional distress, or unrelated to any
obvious inciting causes.
Fissures on the tongue (choice C) can be seen with a variety of nutritional
deficiencies.
Hairy leukoplakia (choice D) is a distinctive oral lesion (fuzzy white tongue)
seen in some AIDS patients.
Pyrogenic granuloma (choice E) produces a ball-like red mass lesion, and
can occur sporadically or in association with pregnancy.
A 3-year-child is taken to a physician for a routine physical examination. On
questioning about any problems she may have noticed that
concern her, the mother comments that the child does not seem to be feeling
well much of the time. She has noticed that the child is irritable,
sleeps poorly, and is often constipated. Physical examination is unremarkable.
Office hematocrit shows a mild anemia.
Question 4 of 6
This child's disease often produces an anemia that on peripheral smear most
closely resembles which of the following?
/ A. Folate deficiency
/ B. Hereditary spherocytosis
/ C. Iron deficiency
/ D. Sickle cell anemia
/ E. Vitamin B12 deficiency


Explanation - Q: 8.4 Close

The correct answer is C. The anemia of lead toxicity is a microcytic,
hypochromic anemia that closely resembles that of iron deficiency on the
peripheral blood smear. While the gold standard for diagnosing lead
poisoning is the whole blood lead level, the free erythrocyte protoporphyrin
level is often used to demonstrate the degree of biological abnormalities that
exists. Lead interferes with ferrochelatase, the enzyme helps to incorporate
iron into the protoporphyrin molecule in hemoglobin synthesis. This is why
the anemia of lead poisoning clinically resembles that of iron deficiency - the
iron is present, but the body cannot use it.
Folate deficiency (choice A) and vitamin B12 deficiency (choice E) both
produce macrocytic anemia.
In hereditary spherocytosis (choice B), all of the erythrocytes are small
spheres that lack the biconcave shape typical of normal erythrocyte.
In sickle cell anemia (choice D), even when the patient is not in a sickling
crisis, at least a few markedly deformed sickle cells will be seen.

Question 5 of 6


A peripheral blood smear is examined (above). Which of the following diagnostic
features is present in the smear?
/ A. Acanthocytes
/ B. Auer rods
/ C. Basophilic stippling
/ D. Hairy cells
/ E. Sickle cells


Explanation - Q: 8.5 Close

The correct answer is C. Basophilic stippling causes round, dark-blue
granules in red blood cells on smears stained with brilliant cresyl blue (the
granules are more red-purple on Wright/Giemsa stains). The granules are
precipitated ribosomes and mitochondria, and are usually caused by an
impairment of maturation in the bone marrow. Causes include lead
poisoning, exposure to some drugs, severe burns, anemias, and septicemia.
Acanthocytes (choice A) are "star-like" red cells with irregularly spaced
projections that can be found in abetalipoproteinemia and some liver
diseases.
Auer rods (choice B) are elongated, bluish-red rods composed of fused
lysosomal granules. They are found in the cytoplasm of cells in the
myelocytic and monoblastic lines in patients with acute myelogenous
leukemia.
Hairy cells (choice D) are seen only in hairy cell leukemia and are white cells
with fine, irregular pseudopods ("hairs") and immature nuclear features.
Sickle cells (choice E) are seen in sickle cell anemia, not lead poisoning.

A 3-year-child is taken to a physician for a routine physical examination. On
questioning about any problems she may have noticed that
concern her, the mother comments that the child does not seem to be feeling
well much of the time. She has noticed that the child is irritable,
sleeps poorly, and is often constipated. Physical examination is unremarkable.
Office hematocrit shows a mild anemia.
Question 6 of 6
Which of the following is an oral drug that can be used to treat this disorder in
children?
/ A. Amantadine
/ B. Succimer
/ C. Sulfasalazine
/ D. Tolbutamide
/ E. Tubocurarine

Explanation - Q: 8.6 Close

The correct answer is B. Lead poisoning is treated with lead chelating
agents to increase urinary secretion. Succimer is the only one of these
agents which has been approved for oral use in children; D-penicillamine is
also commonly used, but has never been formally approved for this use by
the FDA. Other chelating agents that are sometimes used in severe acute
lead poisoning include dimercaprol (IM) and calcium disodium edetate (IM or
IV).
Amantadine (choice A) is an antiviral agent active against influenza A virus.
Sulfasalazine (choice C) is used in rheumatoid arthritis.
Tolbutamide (choice D) is a sulfonylurea used as an oral hypoglycemic
agent.
Tubocurarine (choice E) is a skeletal muscle relaxant.


A 55-year-old man consults a physician because he has been experiencing
chronic weakness and fatigue. Physical examination is notable
for the presence of petechiae and a massively enlarged spleen. A peripheral
blood smear was sent, which on review demonstrates anemia,
thrombocytopenia, modest neutropenia, and the presence of small numbers of
unusual white blood cells. These blood cells are small cells
with a moderate amount of cytoplasm that are covered with distinctive
cytoplasmic projections that appear to be long microvilli.
Question 1 of 5
Which of the following is the most likely diagnosis?
/ A. Chronic myelogenous leukemia
/ B. Erythroleukemia
/ C. Hairy cell leukemia
/ D. Hodgkin's lymphoma
/ E. Metastatic oat cell carcinoma

Explanation - Q: 9.1 Close

The correct answer is C. Hairy cell leukemia is a chronic leukemia that
often has an indolent course. It can present with weakness and fatigue due
to anemia, bleeding due to thrombocytopenia, or fever and infections due to
neutropenia. Almost every patient has splenomegaly, which is massive in 4/5
of cases. The male to female ratio is 5:1 and the patients are usually middle
aged with a median age of 52. The other conditions listed would not have
"hairy cells" in the peripheral blood.

A 55-year-old man consults a physician because he has been experiencing
chronic weakness and fatigue. Physical examination is notable
for the presence of petechiae and a massively enlarged spleen. A peripheral
blood smear was sent, which on review demonstrates anemia,
thrombocytopenia, modest neutropenia, and the presence of small numbers of
unusual white blood cells. These blood cells are small cells
with a moderate amount of cytoplasm that are covered with distinctive
cytoplasmic projections that appear to be long microvilli.
Question 2 of 5
Which of the following tests could confirm this diagnosis?
/ A. 5-hydroxyindoleacetic acid
/ B. Free erythrocyte protoporphyrin
/ C. Homovanillic acid
/ D. Myelin basic protein
/ E. Tartrate-resistant acid phosphatase


Explanation - Q: 9.2 Close

The correct answer is E. Strong positivity for tartrate-resistant acid
phosphatase (TRAP) is a classic finding in hairy cell leukemia cells, and you
may see an item about this on a USMLE question. However, you should also
be aware that this test is no longer available at many centers.
5-hydroxyindoleacetic acid (choice A) is a serotonin derivative used in
diagnosing carcinoid tumors.
Free erythrocyte protoporphyrin (choice B) is used in the evaluation of
porphyrias.
Homovanillic acid (choice C) is a urinary catecholamine derivative that is
used in diagnosis of pheochromocytomas.
Myelin basic protein (choice D) is used in diagnosing multiple sclerosis.
A 55-year-old man consults a physician because he has been experiencing
chronic weakness and fatigue. Physical examination is notable
for the presence of petechiae and a massively enlarged spleen. A peripheral
blood smear was sent, which on review demonstrates anemia,
thrombocytopenia, modest neutropenia, and the presence of small numbers of
unusual white blood cells. These blood cells are small cells
with a moderate amount of cytoplasm that are covered with distinctive
cytoplasmic projections that appear to be long microvilli.
Question 3 of 5
Which of the following immunologic markers is considered to have high
sensitivity and specificity for this patient's disease?
/ A. BIy-7
/ B. CD3
/ C. CD19
/ D. CD20
/ E. CD21



Explanation - Q: 9.3 Close

The correct answer is A. Immunophenotyping of peripheral blood has
mostly replaced the (less sensitive) TRAP test for diagnosis, and is thought
to be capable of making the correct diagnosis in this setting in over 90% of
cases, even when only very small numbers of circulating leukemia cells are
present. Monoclonal Bly-7 is both sensitive and specific for hairy cell
leukemia cells.
These cells also express CD19 (choice C) and CD20 (choice D), but these
are common pan-B cell antigens and are not specific for hairy cell leukemia.
They do not express the late B cell antigen CD21 (choice E) or the T-cell
antigen CD3 (choice B). A last useful tip is that they also stain for CD22 at a
higher intensity than normal B cells.
A 55-year-old man consults a physician because he has been experiencing
chronic weakness and fatigue. Physical examination is notable
for the presence of petechiae and a massively enlarged spleen. A peripheral
blood smear was sent, which on review demonstrates anemia,
thrombocytopenia, modest neutropenia, and the presence of small numbers of
unusual white blood cells. These blood cells are small cells
with a moderate amount of cytoplasm that are covered with distinctive
cytoplasmic projections that appear to be long microvilli.
Question 4 of 5
The cells described are abnormal cells in which of the following cell lines?
/ A. B-Iymphocytes
/ B. Eosinophils
/ C. Macrophages
/ D. Neutrophils
/ E. T-Iymphocytes

Explanation - Q: 9.4 Close

The correct answer is A. Hairy cell leukemia cells are an abnormal type of
mature B lymphocytes. Unlike many leukemias, these cells are often present
in circulating blood in small numbers, although involvement of the spleen is
more prominent. Bone marrow aspiration is often unsuccessful due to a dry
tap; core biopsy will show the tumor cells with abundant cytoplasm infiltrating
a fine fibrillar network. The hairy cell leukemia cells are sensitive to purine
analogues (cladribine) and interferons. Granulocyte colony stimulating factor
(filgrastim) is used to reduce the neutropenia that may produce life-
threatening susceptibility to infections (gram-negative rods, atypical
mycobacteria, disseminated fungus, and Pneumocystis). Patients often
experience long-term remissions with chemotherapy, and late relapses (after
5-10 years) often respond a second time to the same chemotherapy. The
other choices listed are distracters.

A 55-year-old man consults a physician because he has been experiencing
chronic weakness and fatigue. Physical examination is notable
for the presence of petechiae and a massively enlarged spleen. A peripheral
blood smear was sent, which on review demonstrates anemia,
thrombocytopenia, modest neutropenia, and the presence of small numbers of
unusual white blood cells. These blood cells are small cells
with a moderate amount of cytoplasm that are covered with distinctive
cytoplasmic projections that appear to be long microvilli.
Question 5 of 5
Which of the following is the most appropriate pharmacotherapy?
/ A. CIadribine
/ B. Cyclophosphamide
/ C. FIudarabine
/ D. Methotrexate
/ E. 6-Mercaptopurine


Explanation - Q: 9.5 Close

The correct answer is A. The treatment of hairy cell leukemia (HCL) has
dramatically improved with the development of newer, more effective
antineoplastic agents, such as cladribine. Cladribine is considered to be the
treatment of choice for this condition. This is a relatively nontoxic drug that
provides benefit in approximately 95% of all cases and a complete remission
in more than 80% of these cases. In general, response rates are long-lasting,
with few patients relapsing in the first several years. Interferon and
splenectomy are rarely used today to treat this condition. With respect to the
pharmacology of this agent, cladribine is indicated for the treatment of active
HCL as defined by clinically significant anemia, neutropenia,
thrombocytopenia, or disease related symptoms. Non-labeled uses include
treatment of advanced cutaneous T-cell lymphoma, chronic lymphocytic
leukemia, non-Hodgkin lymphomas, acute myeloid leukemia, and
autoimmune hemolytic anemia.
Cyclophosphamide (choice B) is an antineoplastic commonly used in the
treatment of a variety of malignant diseases, such as Hodgkin disease as
well as treatment of several types of leukemia. This medication is an
alkylating agent related to the nitrogen mustards. It suppresses cell-mediated
and humoral immunity as to interfere with "recall responses", causes
lymphopenia of B and T cells, and reduces serum immunoglobulin levels.
Fludarabine (choice C) is an antimetabolite antineoplastic agent indicated
for the treatment of chronic lymphocytic leukemia (CLL). Although not
approved for hairy cell leukemia, this agent has been used with some
success in the treatment of this condition. Fludarabine is specifically a
fluorinated nucleotide analog of the antiviral agent, vidarabine. This agent
appears act by inhibiting DNA polymerase alpha, ribonucleotide reductase,
and DNA primase, thus inhibiting DNA synthesis.
Methotrexate (choice D) is an agent that competitively inhibits dihydrofolic
acid reductase and is also used in antineoplastic therapeutic treatment
regimens, such as ALL, cancers of the head, neck, and breast, usually in
combination with other therapies. It is also used to treat severe, active,
classic or definite rheumatoid arthritis in adults who have an insufficient
response with conventional therapies, and for severe, recalcitrant, disabling
psoriasis.
6-Mercaptopurine (choice E), otherwise known as mercaptopurine or 6-MP,
is a purine analog antimetabolite indicated for the treatment of remission
induction and maintenance therapy of acute lymphocytic leukemia (ALL) as
well as treatment of acute myelogenous (and acute myelomonocytic)
leukemia.

A college student is brought to the student health center at the urging of his
roommate. He has been missing class because he needs to
check the room lock many times before he can leave. Once he starts to ride his
bicycle to class, he frequently returns several times to lock the
door. He repeats this ritual every morning and often when he leaves the house.
He misses his appointments and his academic performance
suffers. His hands are chafed.
Question 1 of 4
Which of the following is the most likely diagnosis?
/ A. Generalized anxiety disorder
/ B. Obsessive compulsive disorder
/ C. Panic disorder
/ D. Paranoid personality disorder
/ E. Posttraumatic stress disorder

Explanation - Q: 1.1 Close

The correct answer is B. Patients with obsessive compulsive disorder
(OCD) suffer from obsessive thoughts and compulsive behaviors that impair
everyday function. Obsessions are defined as recurrent and persistent
thoughts, impulses, or images that are intrusive, inappropriate, and cause
anxiety and distress. Patients realize that these thoughts and images are a
product of their own mind and they will attempt to suppress them. In addition
to obsessions, patients experience compulsions. Compulsions are repetitive
behaviors or mental acts that a person performs in accordance with an
obsession. The behaviors are aimed at reducing distress or preventing some
dreaded event or situation. Locking doors is a common compulsion, and thus
this patient meets the diagnostic criteria for obsessive compulsive disorder.
Generalized anxiety disorder (choice A) is characterized by excessive
anxiety and apprehensive expectation for a period greater than 6 months.
Patients experience anxiety, cognitive vigilance, autonomic hyperactivity,
motor tension, irritability, and poor concentration. Compulsions are not a part
of this disorder.
Panic disorder (choice C) is characterized by episodes of panic. Patients
have a discrete period of intense fear with tachycardia, palpitations,
sweating, trembling, shortness of breath, chest pain and tightening,
abdominal discomfort, fear of dying, and paresthesias.
Paranoid personality disorder (choice D) is characterized by enduring
patterns of personality characterized by mistrust and suspiciousness of
people.
Posttraumatic stress disorder (choice E) is an anxiety disorder that develops
around a traumatic event. Symptoms revolve around the event and include
reexperiencing of the trauma, psychic numbing, and increased autonomic
arousal.
A college student is brought to the student health center at the urging of his
roommate. He has been missing class because he needs to
check the room lock many times before he can leave. Once he starts to ride his
bicycle to class, he frequently returns several times to lock the
door. He repeats this ritual every morning and often when he leaves the house.
He misses his appointments and his academic performance
suffers. His hands are chafed.
Question 2 of 4
The patient's incessant door locking is an example of which of the following?
/ A. Compulsion
/ B. Delusion
/ C. Magical thinking
/ D. Obsession
/ E. Paranoid ideation

Explanation - Q: 1.2 Close

The correct answer is A. Compulsions are repetitive behaviors or mental
acts that patients perform in accordance with an obsession (choice D). It is
important to realize that obsessions are the mental processes and that
compulsions are the actions or behaviors.
Delusions (choice B) are fixed false beliefs that are not culturally accepted.
(People that believe in Santa Claus are not deluded.)
Magical thinking (choice C) is a mental process, not a behavior, like door
locking. Patients with magical thinking believe they have special, "magical"
capacities that others do not have. Thus door locking does not exemplify
magical thinking.
Door locking does not exemplify paranoid ideation (choice E). However,
paranoid ideation, or the patient's belief that others are out to harm him/her,
may provide the source of anxiety to drive the compulsion.
A college student is brought to the student health center at the urging of his
roommate. He has been missing class because he needs to
check the room lock many times before he can leave. Once he starts to ride his
bicycle to class, he frequently returns several times to lock the
door. He repeats this ritual every morning and often when he leaves the house.
He misses his appointments and his academic performance
suffers. His hands are chafed.
Question 3 of 4
Which of the following is the most appropriate pharmacotherapy for this patient?
/ A. CIozapine
/ B. Desipramine
/ C. FIuoxetine
/ D. Haloperidol
/ E. Lorazepam


Explanation - Q: 1.3 Close

The correct answer is C. The effectiveness of selective serotonin reuptake
inhibitors (SSRIs) in treating OCD has contributed significant indirect
evidence to the role of the serotonergic system in the pathophysiology of
OCD. It is hypothesized that dysregulation of this neurotransmitter could
contribute to the repetitive obsessions and ritualistic behaviors. This
hypothesis is also supported by the relative ineffectiveness of noradrenergic
antidepressants, such as desipramine.
Clozapine (choice A) is an atypical antipsychotic, and would not play a role
in the treatment of OCD unless psychotic features were noted.
Desipramine (choice B) is a noradrenergic antidepressant, and as noted
above, has no effect on OCD symptoms.
Haloperidol (choice D) is an antipsychotic drug that has no effect on OCD
symptoms unless psychotic features were noted.
Lorazepam (choice E) is a benzodiazepine that is used to treat acute
agitation. It would not be used to treat OCD.

A college student is brought to the student health center at the urging of his
roommate. He has been missing class because he needs to
check the room lock many times before he can leave. Once he starts to ride his
bicycle to class, he frequently returns several times to lock th
door. He repeats this ritual every morning and often when he leaves the house.
He misses his appointments and his academic performance
suffers. His hands are chafed.
Question 4 of 4
According to psychoanalytic theory, this patient's disorder develops when
defense mechanisms fail to contain the patient's anxiety. One
defense mechanism employed in this patient's constellation of symptoms is
reaction formation. Which of the following is an example of
reaction formation?
/ A. A man ignores the fact that his spouse is cheating on him and they invest in
a house together
/ B. A man in the intensive care unit becomes infantile and unruly
/ C. A man in the intensive care unit tells his nurse, "You that are the best nurse
l have ever seen-unlike those horrible nurses yesterday who
made me wait for pain medicine. Those nurses were horrible and l never want to
see them again."
/ D. A man who is extremely angry with his spouse treats her gently and kindly
/ E. A promiscuous man accuses his spouse of being unfaithful to him

Explanation - Q: 1.4 Close

The correct answer is D. Obsessional patients often show the defense
mechanism of reaction formation. Reaction formation is when affects are
transformed into their opposites and ambivalence is resolved in the opposite
manner from which it arises. This man resolves his anger with his wife by
creating the opposite affect.
Choice A exemplifies the defense mechanism known as denial. Denial is the
invalidation of an unpleasant or unwanted piece of information. He denies
that his marriage is compromised, and continues investing in it.
Choice B exemplifies regression. When regression is employed, patients
return to an earlier level of functioning. This patient's infantile behavior
represents regression. Regression is often seen in medically ill patients.
Choice C exemplifies splitting. In splitting, aspects of mental content are kept
separate. The man has overidealized those who met his needs, and
devalued those who frustrate him. This defense is often seen in patients with
borderline personality disorder.
Choice E exemplifies projection. In projection, a person rids him/herself of
unacceptable thoughts by attributing them to others. While this rids the
affected individual of the unwanted affect, he/she then lives in a world of
others who harbor the unacceptable material. This is often seen in paranoid
patients.


A 32-year-old married lawyer presents to the emergency department with a
complaint of "having a heart attack." He explains he was "doing
nothing particular" at home about 45 minutes ago when he began having chest
pain with shortness of breath and nausea. His symptoms
peaked within ten minutes, and he "knew this was the big one." His wife noted he
was "shaking and sweaty." His wife immediately brought
him to the hospitaI. He has no significant past medical history, takes no
medications, and denies substance use. His family medical history is
significant for a paternal grandfather that "died of a massive heart attack" at age
56. Physical examination reveals an anxious diaphoretic
man taking short, shallow breaths. Vital signs, cardiac auscultation, ECG, and
cardiac enzymes are completely normaI. The patient has been
to the emergency department 5 times in the last 6 weeks and apologizes for "the
million dollar workup," but explains "every time it happens l
just know l am doomed to die."
Question 1 of 6
Which of the following is the most likely diagnosis?
/ A. Agoraphobia
/ B. Generalized anxiety disorder
/ C. Malingering
/ D. Panic disorder
/ E. Phobia


Explanation - Q: 2.1 Close

NONE AVAILABLE

A 32-year-old married lawyer presents to the emergency department with a
complaint of "having a heart attack." He explains he was "doing
nothing particular" at home about 45 minutes ago when he began having chest
pain with shortness of breath and nausea. His symptoms
peaked within ten minutes, and he "knew this was the big one." His wife noted he
was "shaking and sweaty." His wife immediately brought
him to the hospitaI. He has no significant past medical history, takes no
medications, and denies substance use. His family medical history is
significant for a paternal grandfather that "died of a massive heart attack" at age
56. Physical examination reveals an anxious diaphoretic
man taking short, shallow breaths. Vital signs, cardiac auscultation, ECG, and
cardiac enzymes are completely normaI. The patient has been
to the emergency department 5 times in the last 6 weeks and apologizes for "the
million dollar workup," but explains "every time it happens l
just know l am doomed to die."
Question 2 of 6
Which of the following criteria would serve to exclude the most likely diagnosis?
/ A. Panic attacks beginning during sleep
/ B. Panic attacks beginning while driving
/ C. Panic attacks occurring at work and home
/ D. Panic attacks occurring "out of the blue"
/ E. Panic attacks occurring with caffeine intake

Explanation - Q: 2.2 Close

The correct answer is E. Panic attacks resulting from substances
(especially stimulants) or general medical conditions are not considered
panic disorder. Myocardial infarction, hypothyroidism, and carcinoid
syndrome should be ruled out.
Panic attacks may begin while driving, or awaken patients from sleep
(choices A and B).
Panic attacks may occur anywhere, including work and home (choice C).
Panic attacks ONLY occurring with the trigger of being in open spaces are
diagnostic of agoraphobia (a phobia).
In panic disorder, panic attacks are unprecipitated ("out of the blue"; choice
D). Panic attacks triggered by a feared event or object are seen in phobias.
A 32-year-old married lawyer presents to the emergency department with a
complaint of "having a heart attack." He explains he was "doing
nothing particular" at home about 45 minutes ago when he began having chest
pain with shortness of breath and nausea. His symptoms
peaked within ten minutes, and he "knew this was the big one." His wife noted he
was "shaking and sweaty." His wife immediately brought
him to the hospitaI. He has no significant past medical history, takes no
medications, and denies substance use. His family medical history is
significant for a paternal grandfather that "died of a massive heart attack" at age
56. Physical examination reveals an anxious diaphoretic
man taking short, shallow breaths. Vital signs, cardiac auscultation, ECG, and
cardiac enzymes are completely normaI. The patient has been
to the emergency department 5 times in the last 6 weeks and apologizes for "the
million dollar workup," but explains "every time it happens l
just know l am doomed to die."
uestion 3 of 6
Which of the following would be the most appropriate pharmacotherapy?
/ A. CIozapine
/ B. Disulfiram
/ C. FIuoxetine
/ D. Lithium
/ E. Risperidone


Explanation - Q: 2.3 Close

The correct answer is C. The selective serotonin reuptake inhibitors
(SSRIs) are considered first line treatment for panic disorder. Buspirone and
the benzodiazepines are also used.
Clozapine (choice A) is an atypical antipsychotic reserved for treatment of
refractory schizophrenia, due to the risk of agranulocytosis and myocarditis.
Disulfiram (choice B) is used as an adjunct treatment to maintain sobriety.
Patients ingesting alcohol while taking this medicine become ill, due to
accumulation of acetaldehyde.
Lithium (choice D) is a first line treatment for bipolar disorder, and can be
used to augment antidepressant medicines.
Risperidone (choice D) is an atypical antipsychotic medication with
prominent D2 blockade. It is used to treat psychotic conditions, such as
schizophrenia
A 32-year-old married lawyer presents to the emergency department with a
complaint of "having a heart attack." He explains he was "doing
nothing particular" at home about 45 minutes ago when he began having chest
pain with shortness of breath and nausea. His symptoms
peaked within ten minutes, and he "knew this was the big one." His wife noted he
was "shaking and sweaty." His wife immediately brought
him to the hospitaI. He has no significant past medical history, takes no
medications, and denies substance use. His family medical history is
significant for a paternal grandfather that "died of a massive heart attack" at age
56. Physical examination reveals an anxious diaphoretic
man taking short, shallow breaths. Vital signs, cardiac auscultation, ECG, and
cardiac enzymes are completely normaI. The patient has been
to the emergency department 5 times in the last 6 weeks and apologizes for "the
million dollar workup," but explains "every time it happens l
just know l am doomed to die."
Question 4 of 6
Years later, the patient develops a "fear of flying" and is started in once-weekly
therapy for systematic desensitization. Which of the following
forms of psychotherapy is systematic desensitization?
/ A. Behavioral psychotherapy
/ B. Cognitive psychotherapy
/ C. Family psychotherapy
/ D. Group psychotherapy
/ E. Psychoanalytic psychotherapy


Explanation - Q: 2.4 Close

The correct answer is A. Behavioral therapies are based on the learning
theory (operant and classical conditioning). If the anxiety is uncoupled from
the situation, the avoidant behavior will decrease. In systematic
desensitization, the patient constructs a hierarchy of images, and gradually
works to tolerate imagining the most fearful situation. Behavioral
psychotherapy explains behavior as being shaped by reward or punishment,
unlike cognitive psychotherapy which posits behavior as secondary to the
way a person thinks.
Cognitive psychotherapy (choice B) is based on the premise that behavior
can be changed by challenging errors in thinking (cognitive distortions).
"Homework" is used to ascertain the underlying (negative) assumptions.
Family psychotherapy (choice C) is based on the theory that a family is a
system striving to maintain homeostasis, which leads to behaviors.
Group psychotherapy (choice D) is based on many theories, and techniques
include identification and universalization.
Psychoanalytic psychotherapy (choice E) is an intensive type of therapy,
usually 4-5 times per week; the goal is for the patient to develop insight into
unconscious conflicts, and become more aware of the underlying causes of
behavior.
Also, it is not uncommon for a patient to develop more than one anxiety
disorder (panic disorder and a phobia).
A 32-year-old married lawyer presents to the emergency department with a
complaint of "having a heart attack." He explains he was "doing
nothing particular" at home about 45 minutes ago when he began having chest
pain with shortness of breath and nausea. His symptoms
peaked within ten minutes, and he "knew this was the big one." His wife noted he
was "shaking and sweaty." His wife immediately brought
him to the hospitaI. He has no significant past medical history, takes no
medications, and denies substance use. His family medical history is
significant for a paternal grandfather that "died of a massive heart attack" at age
56. Physical examination reveals an anxious diaphoretic
man taking short, shallow breaths. Vital signs, cardiac auscultation, ECG, and
cardiac enzymes are completely normaI. The patient has been
to the emergency department 5 times in the last 6 weeks and apologizes for "the
million dollar workup," but explains "every time it happens l
just know l am doomed to die."
Question 5 of 6
A full cardiac workup of this patient is most likely to reveal which of the following?
/ A. Coronary vasospasm
/ B. Ebstein's anomaly
/ C. Mitral valve prolapse
/ D. Myocarditis
/ E. QTc prolongation

Explanation - Q: 2.5 Close

The correct answer is C. As many as 50% of patients with panic disorder
also have mitral valve prolapse. A beta blocker may alleviate some
symptoms.
Coronary vasospasm (choice A) occurs with cocaine use and usually
presents with the signs and symptoms of a myocardial infarction (with a
positive urine toxicology screen for cocaine).
Ebstein's anomaly (choice B) is a rare, albeit notorious, congenital defect
associated with prenatal exposure to lithium.
Myocarditis (choice D) has occurred with clozapine, which received a "black
box" warning in the PDR in 2002.
QTc prolongation (choice E) can occur with antipsychotic medicines and
tricyclic antidepressants. Torsades may develop.
A 32-year-old married lawyer presents to the emergency department with a
complaint of "having a heart attack." He explains he was "doing
nothing particular" at home about 45 minutes ago when he began having chest
pain with shortness of breath and nausea. His symptoms
peaked within ten minutes, and he "knew this was the big one." His wife noted he
was "shaking and sweaty." His wife immediately brought
him to the hospitaI. He has no significant past medical history, takes no
medications, and denies substance use. His family medical history is
significant for a paternal grandfather that "died of a massive heart attack" at age
56. Physical examination reveals an anxious diaphoretic
man taking short, shallow breaths. Vital signs, cardiac auscultation, ECG, and
cardiac enzymes are completely normaI. The patient has been
to the emergency department 5 times in the last 6 weeks and apologizes for "the
million dollar workup," but explains "every time it happens l
just know l am doomed to die."
Question 6 of 6
The patient returns to the emergency department in a "bizarre state." Friends
report he has been "Iaughing like a loon at all the wrong things,"
"Iooking over his shoulder in public places," and repeatedly questioning their
motives. Examination reveals tachycardia, scleral injection, and
a dry cough. Use of which of the following substances is most likely to explain
this patient's symptoms?
/ A. AIprazolam
/ B. Caffeine
/ C. Cannabis
/ D. Methamphetamine
/ E. Pseudoephedrine

Explanation - Q: 2.6 Close

The correct answer is C. Inappropriate laughter, paranoia, and tachycardia,
scleral injection, and a dry cough are all associated with cannabis use.
Alprazolam (choice A) may cause sedation, slurred speech, disinhibition and
ataxia. Patients seem "drunken."
Caffeine use (choice B) may initially present with panic attacks, but when
carefully questioned, the patient will report caffeine intake (coffee, tea,
chocolate, cocoa, over-the-counter cold medications) and usually develops
headaches (during withdrawal) from caffeine. Caffeine intoxicated patients
could develop paranoia and tachycardia, but cannabis use explains all the
listed symptoms.
Methamphetamine (choice D) and pseudoephedrine (choice E) are
stimulants. Patients may initially present with panic attacks, and can develop
paranoia and tachycardia, but the additional history of inappropriate laughter
and scleral injection indicates cannabis use. Amphetamine abuse mimics
symptoms of schizophrenia.

A 30-year-old man is brought to the emergency department by police, who
arrested him because he was in the parking lot of a local malI,
yelling "I am a golden god" as he stepped in front of moving cars. When
questioned about his identity, he talks incessantly in a rapid fashion
and threatens to "unleash God's wrath on those who do not submit." He reports
that he has not slept in a week, and does not need sleep. He
has spent the last week preparing for a secret government mission that only the
president knows about. When asked if he ever hears God
talking to him, he says, "Of course not, I am God!" Police report that he has had
several prior arrests for reckless driving and lewd acts. On
examination, he is disheveled and malodorous. It is nearly impossible to get any
further history, as he rapidly paces about and mutters, "I
must go, I must go" over and over again.
Question 1 of 7
Which of the following is the most likely preliminary diagnosis?
/ A. Antisocial personality disorder
/ B. Bipolar disorder
/ C. Borderline personality disorder
/ D. Post traumatic stress disorder
/ E. Schizophrenia

Explanation - Q: 3.1 Close

The correct answer is B. This patient is displaying many of the diagnostic
criteria for bipolar disorder. He has had a distinct period of abnormally and
persistently elevated mood lasting at least one week. In addition, his
thoughts are grandiose ("I am a golden god"), he has a decreased need for
sleep, and he has pressured and excessive speech. His arrest history
indicates possible previous sexual indiscretions, which are a hallmark of
bipolar disorder (excessive involvement in pleasurable activities that have a
high potential for painful consequences). His pacing indicates psychomotor
agitation, also a hallmark of bipolar disorder. This patient probably has a
history of severe depressive episodes, but the occurrence of a single manic
episode allows the diagnosis of bipolar disorder to be made.
Antisocial personality disorder (choice A) refers to a long-standing pattern of
socially irresponsible behavior that reflects a disregard for the rights of
others. These individuals were formerly called psychopaths, and generally
lack a conscience. Many people with this disorder engage in unlawful acts.
The most pervasive characteristic is a lack of remorse for the harm they
cause others.
Borderline personality disorder (choice C) refers to a lifelong pattern of
unstable affect and self-image with erratic behavior. Borderlines have
interpersonal relationships that are intense, but very unstable ("love-hate"
relationships). They are prone to self-mutilation and the defense mechanism
of splitting. But they have a life of chaos, not a sense of grandeur and power,
so the diagnosis does not fit this case.
Posttraumatic stress disorder (choice D) is an anxiety disorder that develops
surrounding a traumatic event. Symptoms revolve around the event and
include reexperiencing of the trauma, avoidance of associated stimuli,
psychic numbing, and increased autonomic arousal.
While the psychotic symptoms of mania and schizophrenia (choice E) may,
at times, be difficult to distinguish, the grandiose content of this man's
symptoms suggest that he is having a manic episode. Note that his
comments that God does not talk to him can be taken as a denial of auditory
hallucinations.
A 30-year-old man is brought to the emergency department by police, who
arrested him because he was in the parking lot of a local malI,
yelling "I am a golden god" as he stepped in front of moving cars. When
questioned about his identity, he talks incessantly in a rapid fashion
and threatens to "unleash God's wrath on those who do not submit." He reports
that he has not slept in a week, and does not need sleep. He
has spent the last week preparing for a secret government mission that only the
president knows about. When asked if he ever hears God
talking to him, he says, "Of course not, I am God!" Police report that he has had
several prior arrests for reckless driving and lewd acts. On
examination, he is disheveled and malodorous. It is nearly impossible to get any
further history, as he rapidly paces about and mutters, "I
must go, I must go" over and over again.
Question 2 of 7
Before a definitive diagnosis can be made, which of the following should be
performed?
/ A. Cranial nerve exam
/ B. CT scan of the head
/ C. EIectroencephalogram
/ D. Magnetic resonance imaging of the head
/ E. Toxicological screen

Explanation - Q: 3.2 Close

The correct answer is E. Intoxication with a number of agents such as
amphetamines, cocaine, or other sympathomimetics can mimic mania seen
in bipolar disorder. Other possibilities to consider are antidepressant
medications, thyroid hormone replacements, hyperthyroidism, and other
neurologic conditions. It is often difficult to distinguish the mania of bipolar
disorder from the mania of substance abuse. Often the two are comorbid
conditions as patients "treat" their condition. If a manic episode is substance-
induced, it cannot contribute to a diagnosis of bipolar disorder.
Cranial nerve exam (choice A) cannot be performed on an uncooperative
patient and would likely not contribute to reaching a diagnosis in this patient.
CT scan of the head (choice B) would not likely aid in the diagnosis of this
patient unless something in the patient's medical history suggests an organic
cause (e.g., frontal neoplasm).
An electroencephalogram (choice C) would not contribute to the diagnosis of
this patient as there is no evidence of seizures.
MRI (choice D) would not contribute to the diagnosis for the same reasons a
CT scan would not.

A 30-year-old man is brought to the emergency department by police, who
arrested him because he was in the parking lot of a local malI,
yelling "I am a golden god" as he stepped in front of moving cars. When
questioned about his identity, he talks incessantly in a rapid fashion
and threatens to "unleash God's wrath on those who do not submit." He reports
that he has not slept in a week, and does not need sleep. He
has spent the last week preparing for a secret government mission that only the
president knows about. When asked if he ever hears God
talking to him, he says, "Of course not, I am God!" Police report that he has had
several prior arrests for reckless driving and lewd acts. On
examination, he is disheveled and malodorous. It is nearly impossible to get any
further history, as he rapidly paces about and mutters, "I
must go, I must go" over and over again.
Question 3 of 7
This patient is started on lithium to stabilize his mood. BIood levels of lithium
need to be closely monitored as therapeutic levels are close to
toxic levels. The ratio of toxic dose to therapeutic dose is known as which of the
following?
/ A. Fractional elimination constant
/ B. Half life
/ C. Loading dose
/ D. Therapeutic index
/ E. Volume of distribution

Explanation - Q: 3.3 Close

The correct answer is D. The therapeutic index of a drug is the ratio of the
toxic dose to therapeutic dose. For a drug with a small therapeutic index,
care must be taken not to overdose the patient. Small, stepwise increases in
dosing can aid in finding the therapeutic dose without experiencing toxicity.
A 30-year-old man is brought to the emergency department by police, who
arrested him because he was in the parking lot of a local malI,
yelling "I am a golden god" as he stepped in front of moving cars. When
questioned about his identity, he talks incessantly in a rapid fashion
and threatens to "unleash God's wrath on those who do not submit." He reports
that he has not slept in a week, and does not need sleep. He
has spent the last week preparing for a secret government mission that only the
president knows about. When asked if he ever hears God
talking to him, he says, "Of course not, I am God!" Police report that he has had
several prior arrests for reckless driving and lewd acts. On
examination, he is disheveled and malodorous. It is nearly impossible to get any
further history, as he rapidly paces about and mutters, "I
must go, I must go" over and over again.
Question 4 of 7
Which of the following is a known adverse effect of lithium?
/ A. Agranulocytosis
/ B. AItered judgement
/ C. Aplastic anemia
/ D. Hypothyroidism
/ E. Male infertility


Explanation - Q: 3.4 Close

The correct answer is D. Patients on lithium chronically may develop
iatrogenic hypothyroidism. (Approx. 5% of patients taking the drug > 18
months.) Lithium exerts this effect by interfering with the synthesis and
release of thyroid hormone. Thus any patient experiencing prolonged
depressive symptoms while taking lithium needs to have his/her thyroid
assessed.
Agranulocytosis (choice A) is commonly reported for patients taking the
atypical antipsychotic drug clozapine or carbamazepine, which is sometimes
employed as a second-line treatment for bipolar disorder.
Choice B is incorrect. Neurological side effects of lithium include tremor,
choreoathetosis, ataxia, motor hyperactivity, dysarthria, and aphasia.
Lithium is not known to cause aplastic anemia (choice C). Aplastic anemia is
a serious potential adverse effect of the mood stabilizer carbamazepine.
Carbamazepine would present an alternative to lithium in this patient, but a
periodic blood count must be performed to assess for aplastic anemia.
Lithium is not known to affect fertility in males (choice E) or females.
However, lithium has strong teratogenic effects (Ebstein's anomaly of the
tricuspid valve) and should NOT be given to any woman who is pregnant or
even thinking about becoming pregnant.
A 30-year-old man is brought to the emergency department by police, who
arrested him because he was in the parking lot of a local malI,
yelling "I am a golden god" as he stepped in front of moving cars. When
questioned about his identity, he talks incessantly in a rapid fashion
and threatens to "unleash God's wrath on those who do not submit." He reports
that he has not slept in a week, and does not need sleep. He
has spent the last week preparing for a secret government mission that only the
president knows about. When asked if he ever hears God
talking to him, he says, "Of course not, I am God!" Police report that he has had
several prior arrests for reckless driving and lewd acts. On
examination, he is disheveled and malodorous. It is nearly impossible to get any
further history, as he rapidly paces about and mutters, "I
must go, I must go" over and over again.
Question 4 of 7
Which of the following is a known adverse effect of lithium?
/ A. Agranulocytosis
/ B. AItered judgement
/ C. Aplastic anemia
/ D. Hypothyroidism
/ E. Male infertility

Explanation - Q: 3.5 Close

The correct answer is B. Benzodiazepines are frequently used to treat
acute agitation and can help manage acute mania until lithium can exert its
effects. Benzodiazepines potentiate the inhibitory effect on the CNS neurons
by binding to GABA receptors and increasing the frequency of the opening of
chloride channels in response to GABA stimulation. The net effect is CNS
depression and reduction in the patient's agitation.
Barbiturates bind the GABA receptor and prolong the duration of opening of
chloride channels in response to GABA (choice A). This acts to suppress the
CNS.
Lorazepam has no effect on dopamine receptors. Many antipsychotic
medications act by inhibiting dopamine receptors (choice C).
Lorazepam does not effect the release of epinephrine from the adrenal
medulla (choice D).
As noted above, lorazepam has no effect on dopamine receptors (choice E).

- - -
A 30-year-old man is brought to the emergency department by police, who
arrested him because he was in the parking lot of a local malI,
yelling "I am a golden god" as he stepped in front of moving cars. When
questioned about his identity, he talks incessantly in a rapid fashion
and threatens to "unleash God's wrath on those who do not submit." He reports
that he has not slept in a week, and does not need sleep. He
has spent the last week preparing for a secret government mission that only the
president knows about. When asked if he ever hears God
talking to him, he says, "Of course not, I am God!" Police report that he has had
several prior arrests for reckless driving and lewd acts. On
examination, he is disheveled and malodorous. It is nearly impossible to get any
further history, as he rapidly paces about and mutters, "I
must go, I must go" over and over again.
Question 6 of 7
The patient later reaches a steady state level of lithium that produces toxic side
effects. If he decides to discontinue his medication, how long
would it take for his lithium blood levels to reach 25% of his original steady state
levels assuming a half life of 22 hours for lithium?
/ A. 11 hours
/ B. 22 hours
/ C. 33 hours
/ D. 44 hours
/ E. 55 hours
/ F. 66 hours
/ G. 77 hours
/ H. 88 hours


Explanation - Q: 3.6 Close

The correct answer is D. Lithium, like most drugs follows first-order kinetics,
which means a constant percent of the drug is eliminated per unit time. His
drug levels will decrease by 50% every half-life. Therefore, they will be 50%
of original levels after one half-life, 25% after two half-lives, 12.5% after three
half-lives, etc. Two half-lives is 22 x 2 = 44 hours.

A 30-year-old man is brought to the emergency department by police, who
arrested him because he was in the parking lot of a local malI,
yelling "I am a golden god" as he stepped in front of moving cars. When
questioned about his identity, he talks incessantly in a rapid fashion
and threatens to "unleash God's wrath on those who do not submit." He reports
that he has not slept in a week, and does not need sleep. He
has spent the last week preparing for a secret government mission that only the
president knows about. When asked if he ever hears God
talking to him, he says, "Of course not, I am God!" Police report that he has had
several prior arrests for reckless driving and lewd acts. On
examination, he is disheveled and malodorous. It is nearly impossible to get any
further history, as he rapidly paces about and mutters, "I
must go, I must go" over and over again.
Question 7 of 7
A screening test is developed for assessing vulnerability to developing this
condition. A sample of 10,000 people between the ages of 18 to
24 is recruited from the general population and given the screening test. Of this
sample, 200 individuals are identified as likely to develop the
condition. The sample is tracked over the next twenty years. Forty of the original
sample, although none of those selected by the test, are lost
to follow-up. A total of 100 people from the sample eventually were diagnosed
with this condition, of which 90 were correctly identified by the
screening test. Based on this study, the positive predictive value of the screening
test is best estimated as which of the following?
/ A. 45%
/ B. 60%
/ C. 75%
/ D. 90%
/ E. 100%

Explanation - Q: 3.7 Close

The correct answer is A. Positive predictive value assesses the proportion
of those identified as having the condition that actually end up developing the
condition. In this case, 90 of the original 200 identified by the test developed
the disorder. (90/200 = 45%). Note the test has a sensitivity (ability to detect
disease) of 90%. (90 of the 100 people who actually developed the condition
were correctly identified by the screening test)

A 25-year-old man presents to the emergency department with multiple
lacerations to both wrists. He says he has a history of "every
diagnosis in the book, Doc." The patient reports several recent stressors,
including being fired from his job after "Iosing it" with a "rotten
customer," financial problems, and a fight with his girlfriend three hours ago.
When asked about suicidal ideation, the patient responds with
"would you want this life?" With permission, the doctor speaks with the man's
psychiatrist. His psychiatrist describes a long-standing pattern
of unstable relationships, career changes, and extreme mood swings with erratic
sleep patterns. The patient has taken multiple overdoses in
the past, usually when his psychiatrist is out of town. After the doctor listens
several minutes to the patient empathetically, the man responds
with multiple compliments, proclaiming the doctor is "the best doctor ever." An
hour later the doctor is summoned by the nurses to again see
the patient. The man is sullen and angry "You don't even care what happens to
me. You never did; none of you do, and you just let me sit here
forever. What kind of lousy doctor are you? You're an embarrassment to your
profession."
Question 1 of 6
Which of the following is the most likely diagnosis?
/ A. Antisocial personality disorder
/ B. Borderline personality disorder
/ C. Histrionic personality disorder
/ D. Narcissistic personality disorder
/ E. Paranoid personality disorder

Explanation - Q: 4.1 Close

The correct answer is B. Borderline personality disorder is characterized by
a pattern of instability in 1) relationships (fights with girlfriend and customers),
2) self image (career changes may be one), 3) affect (mood swings), AND
marked impulsivity (multiple overdoses). Another clue is the frantic efforts to
avoid abandonment (overdoses precede separation from doctor, wrist
slashing after break up with girlfriend). Persons with borderline personality
are prone to rages and complain of chronic feelings of emptiness.
Antisocial personality disorder (choice A) is characterized by a pervasive
pattern of disregard for the rights of others.
Histrionic personality disorder (choice C) is characterized by a pervasive
pattern of excessive emotionality and attention-seeking. These individuals
are usually the "life of the party," and may be associated with "creating a
scene." They are not self-destructive, like persons with borderline personality
disorder.
Narcissistic personality disorder (choice D) is characterized by a pervasive
pattern of grandiosity, need for admiration, and lack of empathy. They exhibit
a stable self-image and are not self-destructive like persons with borderline
personality disorder.
Paranoid personality disorder (choice E) is characterized by a pervasive
pattern of distrust and suspiciousness.
A 25-year-old man presents to the emergency department with multiple
lacerations to both wrists. He says he has a history of "every
diagnosis in the book, Doc." The patient reports several recent stressors,
including being fired from his job after "Iosing it" with a "rotten
customer," financial problems, and a fight with his girlfriend three hours ago.
When asked about suicidal ideation, the patient responds with
"would you want this life?" With permission, the doctor speaks with the man's
psychiatrist. His psychiatrist describes a long-standing pattern
of unstable relationships, career changes, and extreme mood swings with erratic
sleep patterns. The patient has taken multiple overdoses in
the past, usually when his psychiatrist is out of town. After the doctor listens
several minutes to the patient empathetically, the man responds
with multiple compliments, proclaiming the doctor is "the best doctor ever." An
hour later the doctor is summoned by the nurses to again see
the patient. The man is sullen and angry "You don't even care what happens to
me. You never did; none of you do, and you just let me sit here
forever. What kind of lousy doctor are you? You're an embarrassment to your
profession."
uestion 2 of 6
The doctor repeatedly reassures the patient that he deserves the best care and
his doctors are working on it. The patient refuses to accept
the reassurance, and begins yelling loudly. The doctor screams at the patient to
be quiet. Later, he says " I Iet him have it, because what he
REALLY needed was some tough love for once." The doctor is using which of the
following defense mechanisms?
/ A. Denial
/ B. Isolation
/ C. Rationalization
/ D. Sublimation
/ E. Suppression


Explanation - Q: 4.2 Close

The correct answer is C. Rationalization, which is providing a logical reason
(what the patient needs) for a behavior, is usually employed to avoid being
blamed (e.g., for unprofessional behavior).
Denial (choice A) is usually used to avoid awareness of a painful reality, and
is often seen in patients given the news of a fatal illness, or in patients
confronted about substance abuse.
Isolation (choice B) splits the thought from the feeling, and can be seen
when doctors discuss "interesting cases" or "severe pathology" in completely
intellectual terms.
Sublimation (choice D) is replacing an unacceptable wish with a more
acceptable one. In this example, the doctor might have "taken out his
aggression" in the exercise room.
Suppression (choice E) is consciously deciding to remove an idea or feeling
from awareness. "I'm not going to think about that."
A 25-year-old man presents to the emergency department with multiple
lacerations to both wrists. He says he has a history of "every
diagnosis in the book, Doc." The patient reports several recent stressors,
including being fired from his job after "Iosing it" with a "rotten
customer," financial problems, and a fight with his girlfriend three hours ago.
When asked about suicidal ideation, the patient responds with
"would you want this life?" With permission, the doctor speaks with the man's
psychiatrist. His psychiatrist describes a long-standing pattern
of unstable relationships, career changes, and extreme mood swings with erratic
sleep patterns. The patient has taken multiple overdoses in
the past, usually when his psychiatrist is out of town. After the doctor listens
several minutes to the patient empathetically, the man responds
with multiple compliments, proclaiming the doctor is "the best doctor ever." An
hour later the doctor is summoned by the nurses to again see
the patient. The man is sullen and angry "You don't even care what happens to
me. You never did; none of you do, and you just let me sit here
forever. What kind of lousy doctor are you? You're an embarrassment to your
profession."
Question 3 of 6
Which coping mechanism is illustrated by the statement, "You are the best doctor
ever, the rest of the staff is cold and heartless."?
/ A. Denial
/ B. Projection
/ C. Repression
/ D. Splitting
/ E. Suppression

Explanation - Q: 4.3 Close

The correct answer is D. Splitting is a primitive defense that oversimplifies
all relationships into "good" and "bad." The self, others, and situations are
completely polarized into one category or the other. Persons with borderline
personality disorder have difficulty tolerating ambivalence (concurrent
positive and negative feelings).
Denial (choice A) is the outright rejection of information. "I do not have
cancer."
Projection (choice B) is attributing one's traits/feelings to another person.
"You're mad at me." When, in fact, I am mad at you.
Repression (choice C) is unconscious exclusion of thoughts/feelings, "What
anger? I was never angry."
Suppression (choice E) is active exclusion of thoughts/feelings from
consciousness, "I'm not going to think about that right now."
A 25-year-old man presents to the emergency department with multiple
lacerations to both wrists. He says he has a history of "every
diagnosis in the book, Doc." The patient reports several recent stressors,
including being fired from his job after "Iosing it" with a "rotten
customer," financial problems, and a fight with his girlfriend three hours ago.
When asked about suicidal ideation, the patient responds with
"would you want this life?" With permission, the doctor speaks with the man's
psychiatrist. His psychiatrist describes a long-standing pattern
of unstable relationships, career changes, and extreme mood swings with erratic
sleep patterns. The patient has taken multiple overdoses in
the past, usually when his psychiatrist is out of town. After the doctor listens
several minutes to the patient empathetically, the man responds
with multiple compliments, proclaiming the doctor is "the best doctor ever." An
hour later the doctor is summoned by the nurses to again see
the patient. The man is sullen and angry "You don't even care what happens to
me. You never did; none of you do, and you just let me sit here
forever. What kind of lousy doctor are you? You're an embarrassment to your
profession."
Question 4 of 6
This patient is started on trazodone to help with sleep. Common side effects of
trazodone include which of the following?
/ A. Drowsiness, dizziness, fatigue, and fatal liver failure
/ B. Drowsiness, dizziness, hypertension, and nervousness
/ C. Drowsiness, dizziness, hypotension, and priapism
/ D. Drowsiness, dizziness, nervousness, and seizures
/ E. Drowsiness, nervousness, GI distress and sexual dysfunction

Explanation - Q: 4.4 Close

The correct answer is C. This question illustrates the fact that many
antidepressant drugs have similar side effects, but often have a certain
particular side effect worth knowing. The clue to choice C (trazodone) is
priapism, a painful sustained erection. It is a medical emergency!
The clue to choice A (nefazodone) is fatal liver failure; this drug now has a
"black box" warning in the PDR.
The clue to choice B (venlafaxine) is hypertension, specifically diastolic
hypertension.
The clue to choice D (bupropion) is seizures. DO NOT give this medicine to
patients at risk for seizures (e.g., metabolic derangement, head injury).
The clues to choice E (any and all SSRIs) are GI distress and sexual
dysfunction, which are very troublesome side effects. Inquire about sexual
dysfunction in all patients taking SSRIs.


A 25-year-old man presents to the emergency department with multiple
lacerations to both wrists. He says he has a history of "every
diagnosis in the book, Doc." The patient reports several recent stressors,
including being fired from his job after "Iosing it" with a "rotten
customer," financial problems, and a fight with his girlfriend three hours ago.
When asked about suicidal ideation, the patient responds with
"would you want this life?" With permission, the doctor speaks with the man's
psychiatrist. His psychiatrist describes a long-standing pattern
of unstable relationships, career changes, and extreme mood swings with erratic
sleep patterns. The patient has taken multiple overdoses in
the past, usually when his psychiatrist is out of town. After the doctor listens
several minutes to the patient empathetically, the man responds
with multiple compliments, proclaiming the doctor is "the best doctor ever." An
hour later the doctor is summoned by the nurses to again see
the patient. The man is sullen and angry "You don't even care what happens to
me. You never did; none of you do, and you just let me sit here
forever. What kind of lousy doctor are you? You're an embarrassment to your
profession."
Question 5 of 6
The doctor calls the insurance company to authorize admission for "mood
stabilization." When asked, the patient says he does not want to
harm himself at this point. The insurance company denies authorization for an
inpatient admission. The doctor determines the patient's
presentation is too despondent and hopeless to be safe. The most appropriate
intervention is for the doctor to call the insurance company
back and do which of the following?
/ A. Say "The patient is a danger to himself in my opinion, get your supervisor on
the phone now."
/ B. Say "The patient is a danger to himself in my opinion, Iet me explain my
reasoning."
/ C. Say "The patient says he is actively suicidal now.", even though he did not
/ D. Tell the patient to say he is suicidaI, then say "The patient says he is
actively suicidal now."
/ E. "This is unacceptable. Get your supervisor on the phone now, or you will be
exposing yourself to legal liability."

Explanation - Q: 4.5 Close

The correct answer is B. The FIRST intervention is for the doctor to calmly
explain the reasoning involved in the assessment.
Choice A is an unnecessary beginning to the conversation; calmly asking for
the supervisor later (if needed) would be the appropriate response.
Choice C is lying. It's illegal, for one. Contacting the attending, the board, the
administrator, etc., may help the doctor get the patient's needs met through
honest means.
Choice D is illegal and unethical, and the doctor will ultimately suffer for
modeling to the patient "we can make deals."
Choice E is a threat and implies coercion, at the very least it could damage
the doctor's professional reputation.
A 25-year-old man presents to the emergency department with multiple
lacerations to both wrists. He says he has a history of "every
diagnosis in the book, Doc." The patient reports several recent stressors,
including being fired from his job after "Iosing it" with a "rotten
customer," financial problems, and a fight with his girlfriend three hours ago.
When asked about suicidal ideation, the patient responds with
"would you want this life?" With permission, the doctor speaks with the man's
psychiatrist. His psychiatrist describes a long-standing pattern
of unstable relationships, career changes, and extreme mood swings with erratic
sleep patterns. The patient has taken multiple overdoses in
the past, usually when his psychiatrist is out of town. After the doctor listens
several minutes to the patient empathetically, the man responds
with multiple compliments, proclaiming the doctor is "the best doctor ever." An
hour later the doctor is summoned by the nurses to again see
the patient. The man is sullen and angry "You don't even care what happens to
me. You never did; none of you do, and you just let me sit here
forever. What kind of lousy doctor are you? You're an embarrassment to your
profession."
Question 6 of 6
The nurse calls to tell the doctor that the patient has taken an overdose in the
emergency department. When the doctor arrives, the patient
has slurred speech and is sedated. Within five minutes the patient is sleeping, his
respiratory rate is 10/min and he responds minimally to
painful stimuli. The family tells the doctor that the man takes clonazepam at
home for nocturnal myoclonus. The most appropriate
pharmacologic intervention is to administer which of the following?
/ A. Dextrose
/ B. FIumazenil
/ C. Naloxone
/ D. Naltrexone
/ E. Thiamine


Explanation - Q: 4.6 Close

The correct answer is B. The history is consistent with respiratory
depression secondary to benzodiazepine overdose. Flumazenil is a
benzodiazepine receptor antagonist.
Dextrose (choice A) treats hypoglycemia.
Naloxone (choice C), an opioid antagonist, reverses opioid overdose.
Thiamine, naloxone, and dextrose are often given to patients who are "found
down" and present to the emergency department in a coma without any
history.
Naltrexone (choice D) blocks the effects of opiates via opioid antagonism
(people cannot "get high"). It is used to prevent relapse in a previously opioid
dependent individual.
Thiamine (choice E) prevents Wernicke-Korsakoff syndrome.

A 69-year-old white man visits a physician in the outpatient clinic for the first time.
Over the past 5 months, he reports increasing lethargy,
weight loss, and crying "for no reason." The patient had always been an
optimistic person, but today he feels "detached from everything." He
describes his mood "as if there was a pane of glass between me and the rest of
the world and l don't think I'm going to make it. My family and
friends are like cardboard cutouts." His speech is slow and methodicaI,
punctuated by frequent sighs. The patient has also lost interest in
watching movies, which had been his favorite pastime. He also admits that his
drinking has become a problem over the past few weeks, and
he currently consumes a bottle of white wine every evening. He recently lost his
job and is currently filing for divorce, which would end a
seventeen-year marriage. His son has attention-deficit disorder, for which he is
prescribed amphetamine. The patient was hospitalized for
major depression three years ago. The patient's previous doctor had started him
on a medication, but he is unable to recall the name or
anything about it except that he is not supposed to eat cheese, aged meats, or
chocolate while taking it. On physical examination, the patient
appears emaciated. His abdomen is very distended, with hepatomegaly 3 cm
below the right costal margin. He also has a slight tremor. His
gait is normaI. His latest calcium level is 10.3 mg/dL.
Question 1 of 5
Which of the following is the most likely diagnosis at this time?
/ A. Acute stress disorder
/ B. Adjustment disorder
/ C. AIcohol abuse
/ D. Bipolar disorder
/ E. Hypercalcemia
/ F. Unipolar disorder


Explanation - Q: 5.1 Close

The correct answer is F. The patient's symptoms of weight loss, lethargy,
tearfulness, hopelessness ("I'm not going to make it"), and depersonalization
("world through a pane of glass; family and friends are like cardboard
cutouts") for at least one month meet the criteria for unipolar disorder (major
depression). Even more specific are his loss of interest in favorite activities,
as well as the feelings of sadness. The symptoms have been going on for at
least two weeks and represent a change from previous functioning, which
defines depression. Three stressors in his life are job loss, the upcoming
divorce. and his son's attention-deficit disorder. The lifetime rate of major
depression in men in the US is about 10%.
Acute stress disorder (choice A) is the consequence of the experience of a
traumatic event outside the realm of normal human experience. Neither
divorce nor job loss fits this criterion. Symptoms of acute stress disorder
(ASD) must also include reexperiencing the event as dreams, recollections of
flashback, and avoidance of associated stimuli, along with diffuse other
symptoms such as irritability, sleep disruption, and difficulty concentrating.
Adjustment disorder (choice B) is a dysfunctional change in behavior within
three months of an identifiable stressor. The dysfunction can only last 6
months after the stressor has ended. An adjustment disorder cannot be a
grief response AND the diagnosis only applies if no other Axis I diagnosis
can be used. Given the patient's current symptoms and previous treatment
history, the criteria for this diagnosis are not met.
Alcohol abuse (choice C) is probably occurring as a coping mechanism for
the depression, and is unlikely to be a specific cause of the depressive
symptoms. Ascites, hepatomegaly, tremor, macrocytosis (raised mean
corpuscular volume, and increased liver enzymes are suggestive of chronic
alcohol use.
Bipolar disorder (choice D) can have a similar presentation to unipolar
disorder when in the depressive phase. The differential is based on being
able to identify one or more manic episodes in the patient's past. Because
none are presented here, there is no evidence for this diagnosis.
Hypercalcemia (choice E) may present with symptoms of confusion,
polyuria, polydipsia, and abdominal pain. This patient has none of these
symptoms and, in addition, his calcium levels are actually normal.

A 69-year-old white man visits a physician in the outpatient clinic for the first time.
Over the past 5 months, he reports increasing lethargy,
weight loss, and crying "for no reason." The patient had always been an
optimistic person, but today he feels "detached from everything." He
describes his mood "as if there was a pane of glass between me and the rest of
the world and l don't think I'm going to make it. My family and
friends are like cardboard cutouts." His speech is slow and methodicaI,
punctuated by frequent sighs. The patient has also lost interest in
watching movies, which had been his favorite pastime. He also admits that his
drinking has become a problem over the past few weeks, and
he currently consumes a bottle of white wine every evening. He recently lost his
job and is currently filing for divorce, which would end a
seventeen-year marriage. His son has attention-deficit disorder, for which he is
prescribed amphetamine. The patient was hospitalized for
major depression three years ago. The patient's previous doctor had started him
on a medication, but he is unable to recall the name or
anything about it except that he is not supposed to eat cheese, aged meats, or
chocolate while taking it. On physical examination, the patient
appears emaciated. His abdomen is very distended, with hepatomegaly 3 cm
below the right costal margin. He also has a slight tremor. His
gait is normaI. His latest calcium level is 10.3 mg/dL.
Question 2 of 5
Which of the following medications is the patient most likely currently taking?
/ A. Amitriptyline
/ B. Chlorpromazine
/ C. FIuoxetine
/ D. Lorazepam
/ E. Phenelzine

Explanation - Q: 5.2 Close

The correct answer is E. Monoamine oxidase inhibitors (MAOIs) include
phenelzine and tranylcypromine. MAOIs have the potential for severe side
effects when taken with sympathomimetic medications or tyramine-
containing foods (e.g., cheeses, red wines, beers, meats, fruits, beans, liver,
yeast extracts) because they can cause a hypertensive crisis. MAOIs can
also cause orthostatic hypotension, nausea, insomnia, and sexual
dysfunction.
Tricyclic antidepressants include amitriptyline (choice A) and nortriptyline.
They take anywhere from 2 to 6 weeks to take effect. They have
anticholinergic side-effects including dry mouth, blurred vision, constipation,
ileus, urinary retention, and even delirium.
Neuroleptics such as chlorpromazine (choice B) reduce psychotic symptoms
that result from a number of illnesses, including schizophrenia, bipolar
disorder, major depressive disorder with psychotic features, psychosis
secondary to stimulant drugs, and organic psychoses from Alzheimer
disease or Huntington disease.
Selective serotonin reuptake inhibitors (SSRIs) include fluoxetine (choice C).
They have a reduced side-effect profile and are effective in depression
treatment. The risk of overdose is low. Their main pharmacologic effect is to
block the presynaptic serotonin uptake site. Increasing the availability of
serotonin in the synaptic cleft is thought to improve depressive symptoms.
A benzodiazepine such as lorazepam (choice D) is primarily used for anxiety
rather than depression. Adverse reactions include sedation, dizziness,
weakness, unsteady gait, headache, and sleep disturbance.

A 69-year-old white man visits a physician in the outpatient clinic for the first time.
Over the past 5 months, he reports increasing lethargy,
weight loss, and crying "for no reason." The patient had always been an
optimistic person, but today he feels "detached from everything." He
describes his mood "as if there was a pane of glass between me and the rest of
the world and l don't think I'm going to make it. My family and
friends are like cardboard cutouts." His speech is slow and methodicaI,
punctuated by frequent sighs. The patient has also lost interest in
watching movies, which had been his favorite pastime. He also admits that his
drinking has become a problem over the past few weeks, and
he currently consumes a bottle of white wine every evening. He recently lost his
job and is currently filing for divorce, which would end a
seventeen-year marriage. His son has attention-deficit disorder, for which he is
prescribed amphetamine. The patient was hospitalized for
major depression three years ago. The patient's previous doctor had started him
on a medication, but he is unable to recall the name or
anything about it except that he is not supposed to eat cheese, aged meats, or
chocolate while taking it. On physical examination, the patient
appears emaciated. His abdomen is very distended, with hepatomegaly 3 cm
below the right costal margin. He also has a slight tremor. His
gait is normaI. His latest calcium level is 10.3 mg/dL.
Question 3 of 5
To gather more details about the patient's current state of mind, which of the
following questions would be most appropriate for the physician
to ask at this time?
/ A. " Are you taking your antidepressant medications as your previous doctor
instructed?"
/ B. " Have you ever made plans to kill yourself?"
/ C. " Is there any history of depression in your family?"
/ D. " On a depression scale of one to ten, how depressed do think you feel right
now?"
/ E. " On average, how much alcohol do you drink in one day?"

Explanation - Q: 5.3 Close

The correct answer is B. At this stage, it is crucial to ask a direct question
about suicidal ideation, particularly because this is the first time the physician
is meeting the patient. In addition, the patient is a white male over the age of
50 who has already been hospitalized once for major unipolar depression,
which places him at a greater risk for suicide. The suicide rate among people
who have been hospitalized at least once for unipolar depression has been
estimated to be 15%. A physician is the last person patients have seen
before taking their own life in 10% of suicides. Asking this question as part of
the mental status examination also gives information on how severe the
depression is. If the patient has a plan about how they will take their own life,
then this suggests that they are more likely to follow through with their plan.
Compliance with medication (choice A) is an important cause of depression
relapse. It has been estimated that fewer than 10% of people suffering from
unipolar major depression are likely to be receiving a full therapeutic dose of
medication. More than 20% of patients fail to fill the first prescription they
receive for major depression, and the majority of patients who do begin
treatment discontinue the medication within 14 weeks, usually due to side
effects.
Depression in the family (choice C) is a risk factor for recurrent depression.
Pursuing this line of questioning is useful for the long-term management of
depression but does not address any major life-threatening issues. Other risk
factors for recurrent depression include a history of multiple episodes
(patients with 3 or more prior episodes have at least a 90% recurrence rate),
depression associated with dysthymia, onset after age 60, long duration of
individual episodes, poor symptom control during therapy, comorbid anxiety
disorder, or substance abuse.
Objective analysis of depression such as a depression score of 1 to 10
(choice D) is also useful in the long term care of patients with depression. In
doing so, the physician is able to objectively document the effectiveness of
the patient's antidepressant therapy. Another useful method is counting the
number of improvements in the quality of the patient's life, e.g., more family
interaction, being able to go to church, returning back to work. For this to be
useful, it is important to get an idea of where the patient is at in the initial
visit. It does not, however, take precedence over asking about suicidal
ideation.
Alcoholic intake (choice E) is an important question to ask since the patient
appears to have signs of alcoholism (hepatomegaly, increasing abdominal
girth as an indication of ascites, weight loss, and tremor). However, it does
not necessarily have to be addressed on the initial visit. Suicidal risk is a
more ominous life-threatening event in this instance. Accurate answers can
be derived when asking the patient to describe their drinking habits during
the course of the day rather than asking about alcohol intake as a single
numerical quantity.

A 69-year-old white man visits a physician in the outpatient clinic for the first time.
Over the past 5 months, he reports increasing lethargy,
weight loss, and crying "for no reason." The patient had always been an
optimistic person, but today he feels "detached from everything." He
describes his mood "as if there was a pane of glass between me and the rest of
the world and l don't think I'm going to make it. My family and
friends are like cardboard cutouts." His speech is slow and methodicaI,
punctuated by frequent sighs. The patient has also lost interest in
watching movies, which had been his favorite pastime. He also admits that his
drinking has become a problem over the past few weeks, and
he currently consumes a bottle of white wine every evening. He recently lost his
job and is currently filing for divorce, which would end a
seventeen-year marriage. His son has attention-deficit disorder, for which he is
prescribed amphetamine. The patient was hospitalized for
major depression three years ago. The patient's previous doctor had started him
on a medication, but he is unable to recall the name or
anything about it except that he is not supposed to eat cheese, aged meats, or
chocolate while taking it. On physical examination, the patient
appears emaciated. His abdomen is very distended, with hepatomegaly 3 cm
below the right costal margin. He also has a slight tremor. His
gait is normaI. His latest calcium level is 10.3 mg/dL.
Question 4 of 5
The physician decides to discontinue his current medication and prescribes
sertraline instead. Sertraline directly affects which of the
following neurotransmitters?
/ A. Acetylcholine
/ B. Dopamine
/ C. Epinephrine
/ D. Norepinephrine
/ E. Serotonin


Explanation - Q: 5.4 Close

The correct answer is E. Sertraline specifically blocks the reuptake of
serotonin into the presynaptic axon terminal. This enhances serotonin
activation and brings about a cascade of events ultimately resulting in a
reduced sensitivity of presynaptic autoreceptors for serotonin and reduced
serotonin synthesis. The most common adverse reactions to the SSRIs are
gastrointestinal (especially nausea), neuropsychiatric (particularly headache
and tremor), and changes in sexual functioning. SSRIs also treat anxious
depression, dysthymia, and atypical depression.
Acetylcholine (choice A) has been cited as the main neurotransmitter
involved in Alzheimer dementia. Drugs that utilize this concept are tacrine
and donepezil, which work in the brain as cholinesterase inhibitors at the
neuronal synapse. By inhibiting the cholinesterase enzyme, they increase
the level of acetylcholine and aid in maintaining mental function, although
they do not stop the degeneration of cholinergic cells. Sertraline has no
effect on the levels of acetylcholine.
Dopamine (choice B) is found in both small and large neuronal pathways in
the CNS. The latter include the nigrostriatal pathway involved, in the etiology
of Parkinson disease and mesolimbic/mesocortical pathways, implicated in
psychosis. L-dopa, which is converted to dopamine, is the primary
replacement therapy in Parkinson disease. Dopamine antagonists are used
in the treatment of psychosis. Dopaminergic input to the chemoreceptor
trigger zone is the basis of the use of the agonist apomorphine as an emetic
to treat poisoning, and the use of antagonists as antiemetics. Hypothalamic
dopaminergic neurons inhibit prolactin secretion and lead to the use of
agonists in inhibiting lactation. There are a number of drugs that interact
presynaptically with dopamine terminals including reserpine, amphetamine,
MAO inhibitors, and cocaine.
Epinephrine (choice C) is a neurotransmitter, and a hormone. It stimulates
alpha1-, alpha2-, beta1-, and beta2-adrenergic receptors in a dose-related
fashion. It is the initial drug of choice for treating bronchoconstriction and
hypotension resulting from anaphylaxis as well as all forms of cardiac arrest.
It is useful in managing reactive airway disease, but beta-adrenergic agents
are often used initially because of their convenience and oral inhalation
route. Epinephrine is not a neurotransmitter specifically affected by any
antidepressants currently available.
Norepinephrine (choice D), like serotonin, is a neurotransmitter that may
induce depression if depleted. There are several antidepressants that
increase norepinephrine levels:


A 69-year-old white man visits a physician in the outpatient clinic for the first time.
Over the past 5 months, he reports increasing lethargy,
weight loss, and crying "for no reason." The patient had always been an
optimistic person, but today he feels "detached from everything." He
describes his mood "as if there was a pane of glass between me and the rest of
the world and l don't think I'm going to make it. My family and
friends are like cardboard cutouts." His speech is slow and methodicaI,
punctuated by frequent sighs. The patient has also lost interest in
watching movies, which had been his favorite pastime. He also admits that his
drinking has become a problem over the past few weeks, and
he currently consumes a bottle of white wine every evening. He recently lost his
job and is currently filing for divorce, which would end a
seventeen-year marriage. His son has attention-deficit disorder, for which he is
prescribed amphetamine. The patient was hospitalized for
major depression three years ago. The patient's previous doctor had started him
on a medication, but he is unable to recall the name or
anything about it except that he is not supposed to eat cheese, aged meats, or
chocolate while taking it. On physical examination, the patient
appears emaciated. His abdomen is very distended, with hepatomegaly 3 cm
below the right costal margin. He also has a slight tremor. His
gait is normaI. His latest calcium level is 10.3 mg/dL.
Question 5 of 5
Should this particular patient commit suicide, the likeliest method of suicide is
which of the following?
/ A. A drug overdose
/ B. By running his car in a closed garage for an extended period of time
/ C. Cutting his wrists
/ D. Hanging himself
/ E. With a firearm


Explanation - Q: 5.5 Close

The correct answer is E. Firearms were the commonest method of suicide
used by persons aged 65 years or older representing 71% of suicides. In
1998, firearms were the most common method of suicide by both males and
females, accounting for 78% of male and 35% of female suicides in this age
group.
Overdose with liquids, pills or gas (choice A), 17%, and suffocation (choice
D),11%, represents the other two most common methods of suicide used by
persons aged 65 years or older. Elderly patients make fewer attempts per
completed suicide, have a higher-male-to-female ratio than other groups,
have often visited a health-care provider before their suicide, and have more
physical illnesses. It is estimated that 20% of elderly (over 65 years) persons
who commit suicide visited a physician within 24 hours of their act, 41%
visited within a week of their suicide and 75% have been seen by a physician
within one month of their suicide. The suicide rate of white males rises
dramatically after age 65 and accounts for the majority of all suicides in the
elderly.
In 2000, the death rate by car exhaust fumes (choice B) was less than 1 per
100 000 per year.
Between the age group of 65 to 70 years, the mortality rate from suicide by a
cut or stab (choice C) was 0.32 deaths per 100 000 per year.


A 62-year-old man is seen in an emergency department because of severe pain
in his lower back, Ieft buttock and the posterolateral aspect
of his left leg. The pain began acutely after he lifted his 8-year-old granddaughter
into the back of a truck, and he describes it as "shooting" in
nature. When examined by the physician, he has curled into a fetal position,
which he says relieves the pain. Attempts to extend the man's
spine cause a marked exacerbation of the pain.

Question 1 of 5

On physical examination, the physician notes that spasm is occurring in the large
muscles on the posterior aspect of the thigh. Which of the
following muscles is most likely involved?
/ A. Adductor longus
/ B. Biceps femoris
/ C. Gastrocnemius
/ D. Sartorius
/ E. Soleus

Explanation - Q: 1.1 Close

The correct answer is B. The biceps femoris is the only muscle listed that is
in the posterior compartment of the thigh.
The adductor longus (choice A) and the sartorius (choice D) are in the
anterior compartment of the thigh.
The gastrocnemius (choice C) and the soleus (choice E) are in the posterior
compartment of the leg below the knee.


A 62-year-old man is seen in an emergency department because of severe pain
in his lower back, Ieft buttock and the posterolateral aspect
of his left leg. The pain began acutely after he lifted his 8-year-old granddaughter
into the back of a truck, and he describes it as "shooting" in
nature. When examined by the physician, he has curled into a fetal position,
which he says relieves the pain. Attempts to extend the man's
spine cause a marked exacerbation of the pain.
Question 2 of 5
More detailed examination demonstrates foot drop with weakness of the anterior
tibiaI, posterior tibiaI, and peroneal muscles. Sensory loss
is demonstrated over the anterior shin and dorsal foot. These findings suggest a
radiculopathy at which of the following cord levels?
/ A. C-7
/ B. L-5
/ C. S-3
/ D. T-9
/ E. T-12

Explanation - Q: 1.2 Close

The correct answer is B. The findings are typical for involvement at the L-5
level. Most ruptured disks involve either the lumbar area (L-5 and S-1 most
common) or the cervical area (C-6 and C-7 most commonly). Compression
by a disk of the cervical cord can produce a spastic paresis of the lower
limbs; compression of the cauda equina can cause urine retention or
incontinence.
C-7 (choice A) supplies the back of the shoulders, down the back of the
arm, past the elbow, to the back of the second and third fingers.
S-3 (choice C) supplies a small area on the buttocks.
T-9 (choice D) supplies a band on the abdomen and back that passes above
the umbilicus.
T-12 (choice E) supplies a similar band that passes below the umbilicus.


A 62-year-old man is seen in an emergency department because of severe pain
in his lower back, Ieft buttock and the posterolateral aspect
of his left leg. The pain began acutely after he lifted his 8-year-old granddaughter
into the back of a truck, and he describes it as "shooting" in
nature. When examined by the physician, he has curled into a fetal position,
which he says relieves the pain. Attempts to extend the man's
spine cause a marked exacerbation of the pain.

Question 3 of 5

PIain spinal x-ray films show disk space narrowing in the involved area of the
spine, and herniation with protrusion of the nucleus pulposus of
one disk is demonstrated by CT scan. The normal location of the disk is between
which of the following?
/ A. The body of the superior vertebrae and the body of the inferior vertebrae
/ B. The inferior articular process of the superior vertebrae and the superior
articular process of the inferior vertebrae
/ C. The pedicle of the superior vertebrae and the pedicle of the inferior
vertebrae
/ D. The superior articular process of the superior vertebrae and the inferior
articular process of the inferior vertebrae
/ E. The vertebral canal of the superior vertebrae and the vertebral canal of the
inferior vertebrae

Explanation - Q: 1.3 Close

The correct answer is A. The disk lies between the bodies of the vertebrae,
cushioning the bones to allow for smooth motion. When too much pressure is
placed on the spine, a disk can bulge or rupture (herniated disk). The pain of
disk herniation may begin abruptly after an obvious inciting event (as in this
case) or may be insidious in onset. In many patients, the pain is worsened by
Valsalva maneuver, coughing, laughing, or straining at stool. Patients without
symptoms of spinal cord compression, cauda equina compression, or severe
neurologic deficits related to peripheral nerve root involvement are often
treated conservatively, since up to 95% recover without surgery. Surgical
repair is used in symptomatic patients, or those with more severe or
persistent cases.
The superior and inferior articular processes (choices B and D) form the
surfaces for the joints between the posterior aspects of the vertebrae.
The vertebral canal (choice E) contains the spinal cord, and the pedicle
(choice C) contributes to its posterior margin.

A 62-year-old man is seen in an emergency department because of severe pain
in his lower back, Ieft buttock and the posterolateral aspect
of his left leg. The pain began acutely after he lifted his 8-year-old granddaughter
into the back of a truck, and he describes it as "shooting" in
nature. When examined by the physician, he has curled into a fetal position,
which he says relieves the pain. Attempts to extend the man's
spine cause a marked exacerbation of the pain.

Question 4 of 5

The nucleus pulposus is composed of which of the following?
/ A. Cancellous bone
/ B. Compact bone
/ C. EIastic fibrocartilage
/ D. Hyaline cartilage
/ E. White fibrocartilage

Explanation - Q: 1.4 Close

The correct answer is E. The disk contains a soft, centrally located nucleus
pulposus, surrounded by the protective shell of the annulus fibrosis. The
nucleus pulposus is composed of white fibrocartilage.
The disk contains no bone (choices A and B).
Elastic fibrocartilage (choice C) is found in the larynx, epiglottis, and ear.
The disk has a thin outer layer of hyaline cartilage (choice D), but this does
not involve the nucleus pulposus.
A 62-year-old man is seen in an emergency department because of severe pain
in his lower back, Ieft buttock and the posterolateral aspect
of his left leg. The pain began acutely after he lifted his 8-year-old granddaughter
into the back of a truck, and he describes it as "shooting" in
nature. When examined by the physician, he has curled into a fetal position,
which he says relieves the pain. Attempts to extend the man's
spine cause a marked exacerbation of the pain.

Question 5 of 5

A principal component of the intervertebral disk is collagen. Cross-Iinking of
collagen to form a more stable structure requires the use of a
vitamin C-dependent enzyme that can hydroxylate which of the following amino
acids in procollagen?
/ A. Asparagine
/ B. Cysteine
/ C. Proline
/ D. Serine
/ E. Valine

Explanation - Q: 1.5 Close

The correct answer is C. The amino acids proline and lysine in collagen are
hydroxylated by two vitamin C-dependent enzymes, prolyl- and lysyl
hydroxylase, respectively. Hydroxyproline plays an essential role in stable
triple helix formation. Cysteine residues form disulfide bonds also important
for a stable collagen structure, but hydroxylation plays no role in this process.
The other amino acids listed are not hydroxylated in the process of collagen
cross-linking.

A 40-year-old man consults a physician because of chronic lumbar discomfort.
On physical examination, Iarge tender bilateral masses are
felt deep in his lower back, below his rib cage, and to either side of the vertebral
column. Urinalysis demonstrates mild proteinuria and
microscopic hematuria without red cell casts. CT scan demonstrates massive
bilateral enlargement of the kidneys with near total
replacement of the renal cortex by roughly spherical cysts of varying sizes.
Question 1 of 5
Which of the following is the most likely diagnosis?
/ A. Autosomal dominant polycystic kidney disease
/ B. Autosomal recessive polycystic kidney disease
/ C. Cystic renal dysplasia
/ D. Medullary sponge kidney
/ E. Nephronophthisis-uremic medullary cystic disease complex

Explanation - Q: 2.1 Close

The correct answer is A. This patient most likely has the autosomal
dominant (adult) form of polycystic kidney disease. This relatively common
condition causes massive bilateral enlargement of the kidneys by mid-
adulthood, and affects approximately 1 in 1000 individuals. It may present as
an incidental finding on ultrasound (even in children) or with symptoms
related to the effects of the cysts (lumbar discomfort, hematuria, urinary tract
infection) or uremia. The cysts arise from dilatation of tubules throughout the
nephron, are characteristically spherical, unless compressed by adjacent
cysts, and are separated by residual functioning renal tissue.
In contrast to the autosomal dominant form of polycystic kidney disease, the
autosomal recessive form (choice B) tends to present in childhood, causes
less massive renal enlargement, and tends to have a combination of small
spherical cysts and elongated dilated channels that occur radially at right
angles to the cortical surface.
Cystic renal dysplasia (choice C) is an often unilateral developmental
malformation of the kidney that is characterized by a fairly small number of
large cysts with intervening tissue that may contain abnormal tissues such as
cartilage and undifferentiated mesenchyme.
Medullary sponge kidney (choice D) is characterized by multiple cystic
dilations confined to the renal medulla, rather than cortex.
The nephronophthisis-uremic medullary cystic disease complex (choice E) is
a group of progressive renal disorders that are usually identified in childhood
and present with medullary cysts with tubular atrophy and fibrosis in the
cortex.
A 40-year-old man consults a physician because of chronic lumbar discomfort.
On physical examination, Iarge tender bilateral masses are
felt deep in his lower back, below his rib cage, and to either side of the vertebral
column. Urinalysis demonstrates mild proteinuria and
microscopic hematuria without red cell casts. CT scan demonstrates massive
bilateral enlargement of the kidneys with near total
replacement of the renal cortex by roughly spherical cysts of varying sizes.
Question 2 of 5
Which of the following sites would be most likely to additionally contain cysts in
this patient's condition?
/ A. Adrenal gland
/ B. Cerebral cortex
/ C. Liver
/ D. Ovary
/ E. Thyroid

Explanation - Q: 2.2 Close

The correct answer is C. In addition to the massive cystic replacement of
the kidneys, patients with autosomal dominant polycystic disease also have
a tendency to form smaller numbers of cysts at other sites. Approximately
40% of these patients have one or more (usually asymptomatic) hepatic
cysts; less frequently, cysts are seen in the spleen, pancreas, and lungs. The
sites listed in the other choices do not have a notable predilection for
developing cysts.
A 40-year-old man consults a physician because of chronic lumbar discomfort.
On physical examination, Iarge tender bilateral masses are
felt deep in his lower back, below his rib cage, and to either side of the vertebral
column. Urinalysis demonstrates mild proteinuria and
microscopic hematuria without red cell casts. CT scan demonstrates massive
bilateral enlargement of the kidneys with near total
replacement of the renal cortex by roughly spherical cysts of varying sizes.
Question 3 of 5
There is a specific link between this patient's disease and which of the following
conditions?
/ A. Berry aneurysm
/ B. Lymphedema
/ C. Polyarteritis nodosa
/ D. Temporal arteritis
/ E. Thromboangiitis obliterans

Explanation - Q: 2.3 Close

The correct answer is A. Up to 1/3 of patients with adult polycystic kidney
disease may have berry aneurysms (roughly spherical dilatations) of the
arteries of the circle of Willis at the base of the brain. These may bleed
intracranially, with significant risk of death. It is thought that the same defect
(possibly an abnormally weak basement membrane) that predisposes for the
organ cysts also predisposes for berry aneurysm formation. Other
cardiovascular anomalies that these patients may have include aortic root
dilation and mitral valve prolapse.
Lymphedema (choice B) is an accumulation of excess lymphatic fluid, which
may occur in a congenital form or secondary to malignant or other disease
that occludes lymphatic channels.
Polyarteritis nodosa (choice C) is probably an autoimmune disease, with
focal inflammation of medium sized muscular arteries. It can be seen
idiopathically, or following infection or medication use.
Temporal arteritis (choice D) is a chronic inflammatory disease of large
blood vessels, particularly those of the head. Temporal arteritis is often
associated with polymyalgia rheumatica.
Thromboangiitis obliterans (choice E) is a vascular disease of small and
medium-sized arteries and veins that is most commonly associated with
smoking.

A 40-year-old man consults a physician because of chronic lumbar discomfort.
On physical examination, Iarge tender bilateral masses are
felt deep in his lower back, below his rib cage, and to either side of the vertebral
column. Urinalysis demonstrates mild proteinuria and
microscopic hematuria without red cell casts. CT scan demonstrates massive
bilateral enlargement of the kidneys with near total
replacement of the renal cortex by roughly spherical cysts of varying sizes.

Question 4 of 5

5 years after the diagnosis is established, the patient develops persistent
hypertension secondary to his renal disease. The hypertension is
treated with lisinopriI. This agent acts at which of the following steps?
/ A. CIeavage of angiotensin l
/ B. Increase of sodium and fluid retention
/ C. Induction of aldosterone secretion
/ D. Secretion of angiotensin substrate
/ E. Secretion of rennin

Explanation - Q: 2.4 Close

The correct answer is A. Hypertension is very common among patients with
autosomal dominant polycystic disease, and apparently develops, in large
part, because the cysts impair blood flow to enough renal glomeruli to trigger
the angiotensin-renin system. ACE inhibitors (e.g., captopril, lisinopril,
enalapril, benazepril, fosinopril, moexipril, perindopril, quinapril, ramipril, and
trandolapril) are often effective anti-hypertensive agents in these patients
because they interrupt the stimulation of this pathway. More specifically,
ACE-inhibitors block the conversion of angiotensin I to angiotensin II, a step
that is normally performed by angiotensin converting enzyme (ACE) as blood
circulates through the lung. The renin-angiotensin system is activated when
reduced blood pressure or distal tubular sodium triggers the secretion of
renin (choice E) by the juxtaglomerular cells of the kidney. The circulating
renin cleaves angiotensin substrate (secreted by the liver, choice D) to
angiotensin I. Following cleavage of angiotensin I to angiotensin II by ACE,
angiotensin II triggers both vasoconstriction and aldosterone secretion
(choice C), which then causes increased sodium and fluid retention (choice
B).
A 40-year-old man consults a physician because of chronic lumbar discomfort.
On physical examination, Iarge tender bilateral masses are
felt deep in his lower back, below his rib cage, and to either side of the vertebral
column. Urinalysis demonstrates mild proteinuria and
microscopic hematuria without red cell casts. CT scan demonstrates massive
bilateral enlargement of the kidneys with near total
replacement of the renal cortex by roughly spherical cysts of varying sizes.
Question 5 of 5
Fifteen years after diagnosis, the patient develops chronic renal failure. This
would be indicated most directly by rising serum levels of which
of the following?
/ A. Amylase
/ B. Bilirubin
/ C. Creatinine
/ D. Potassium
/ E. Sodium

Explanation - Q: 2.5 Close

The correct answer is C. Chronic renal failure is common among older
patients with autosomal dominant polycystic kidney disease, and if it
becomes sufficiently severe, may be treated with hemodialysis or kidney
transplant. Renal failure is usually monitored clinically with serum levels of
creatinine and urea.
Amylase (choice A) is a marker for pancreatic damage.
Bilirubin (choice B) is a marker for biliary tree damage and hemolysis.
Serum potassium (choice D) may be either normal or moderately elevated in
renal failure, but can be altered by many conditions and is a less useful
monitoring tool than creatinine.
Serum sodium (choice E) can be normal or reduced in renal failure, but can
also be altered by many other conditions, and so is a less useful monitoring
tool than creatinine.
A 20-year-old man presents to his family physician with a 1-year history of low
back pain and stiffness that is worse in the morning. He reports
that the pain is improved with activity and a warm shower. He now pinpoints most
of his symptoms to his sacroiliac joints. Physical
examination reveals decreased chest expansion on inspiration. X-ray films reveal
bilateral symmetric sacroiliac erosion and bony fusion
(bamboo spine) of part of the lumbar spine.
Question 1 of 5
Which of the following is the most likely diagnosis?
/ A. Ankylosing spondylitis
/ B. Enteropathic arthritis
/ C. Juvenile arthritis
/ D. Psoriatic arthritis
/ E. Reiter syndrome

Explanation - Q: 3.1 Close

The correct answer is A. This patient most likely has ankylosing spondylitis,
an inflammatory arthropathy of the vertebral column. This disorder is most
common in young men, and its peak incidence is at 20 years of age. Greater
than 90% of patients are positive for HLA-B27. Patients usually complain of
pain that is helped with activity and/or a warm shower. There is classically
bilateral sacroiliac involvement.
Enteropathic arthritis (choice B) is seen in patients with inflammatory bowel
diseases such as ulcerative colitis and Crohn disease.
Juvenile arthritis (choice C) is a general term for chronic inflammatory
arthritis in children.
Psoriatic arthritis (choice D) is seen in 7% of patients with psoriasis. HLA-
B27 is positive in many patients with psoriatic arthritis affecting the spine.
Reiter syndrome (choice E) is a triad of seronegative polyarthritis,
conjunctivitis and nonspecific urethritis. It is most commonly seen in men,
and is associated with venereal disease exposure or bacillary dysentery.
Approximately 90% of patients are HLA-B27 positive.
A 20-year-old man presents to his family physician with a 1-year history of low
back pain and stiffness that is worse in the morning. He reports
that the pain is improved with activity and a warm shower. He now pinpoints most
of his symptoms to his sacroiliac joints. Physical
examination reveals decreased chest expansion on inspiration. X-ray films reveal
bilateral symmetric sacroiliac erosion and bony fusion
(bamboo spine) of part of the lumbar spine.
Question 2 of 5
Which of the following genetic markers is most commonly associated with this
condition?
/ A. HLA-B5
/ B. HLA-B8
/ C. HLA-B18
/ D. HLA-B27
/ E. HLA-DR5
Explanation - Q: 3.2 Close

The correct answer is D. Over 90% of patients with ankylosing spondylitis
are HLA-B27 positive. HLA-B27 is also found in patients with Reiter
syndrome and psoriatic arthritis.
HLA-B5 (choice A) is associated with congenital adrenal hyperplasia.
HLA-B8 (choice B) is associated with autoimmune adrenalitis, celiac
disease, Graves disease, and Hashimoto thyroiditis.
HLA-B18 (choice C) is associated with Hodgkin disease.
HLA-DR5 (choice E) is associated with Hashimoto thyroiditis and juvenile
arthritis.

A 20-year-old man presents to his family physician with a 1-year history of low
back pain and stiffness that is worse in the morning. He reports
that the pain is improved with activity and a warm shower. He now pinpoints most
of his symptoms to his sacroiliac joints. Physical
examination reveals decreased chest expansion on inspiration. X-ray films reveal
bilateral symmetric sacroiliac erosion and bony fusion
(bamboo spine) of part of the lumbar spine.
Question 3 of 5
What cardiac condition is most commonly associated with this patient's
diagnosis?
/ A. Aortic regurgitation
/ B. Aortic stenosis
/ C. Ischemia
/ D. Mitral regurgitation
/ E. Mitral stenosis

Explanation - Q: 3.3 Close

The correct answer is A. Approximately 10% of patients with ankylosing
spondylitis develop a characteristic aortic valve lesion. The aortic ring
becomes dilated and the valve leaflets become scarred. Inflammatory lesions
develop on the valve ring, resulting in aortic regurgitation.
The other diagnoses are possible, but are not the most common form of
cardiac involvement in these patients.
A 20-year-old man presents to his family physician with a 1-year history of low
back pain and stiffness that is worse in the morning. He reports
that the pain is improved with activity and a warm shower. He now pinpoints most
of his symptoms to his sacroiliac joints. Physical
examination reveals decreased chest expansion on inspiration. X-ray films reveal
bilateral symmetric sacroiliac erosion and bony fusion
(bamboo spine) of part of the lumbar spine.
Question 4 of 5
The patient later falls, and then complains of low back pain. X-ray films reveal a
fracture of the lumbar spine. On physical examination, he has
Iow back tenderness and is unable to extend his great toe on the right. Which of
the following nerve roots was most likely injured?
/ A. L2
/ B. L3
/ C. L4
/ D. L5
/ E. S1
Explanation - Q: 3.4 Close

The correct answer is D. This patient is unable to extend his great toe,
which indicates impairment of the action of the long toe extensors,
specifically the extensor hallucis longus (EHL). The EHL is innervated by the
L5 nerve root.
The L2 nerve root (choice A) innervates the hip flexors, for example the
iliopsoas.
The L3 nerve root (choice B) innervates the knee extensors, such as the
quadriceps.
The L4 nerve root (choice C) innervates the ankle dorsiflexors, such as the
tibialis anterior.
The S1 nerve root (choice E) innervates the ankle plantar flexors, such as
the gastrocnemius and soleus.
A 20-year-old man presents to his family physician with a 1-year history of low
back pain and stiffness that is worse in the morning. He reports
that the pain is improved with activity and a warm shower. He now pinpoints most
of his symptoms to his sacroiliac joints. Physical
examination reveals decreased chest expansion on inspiration. X-ray films reveal
bilateral symmetric sacroiliac erosion and bony fusion
(bamboo spine) of part of the lumbar spine.
Question 5 of 5
A 24-year-old man is seen later that day in the same clinic with complaints of
dysuria and pain in his knee and ankle joints. On examination,
he has a mild conjunctivitis bilaterally. Which of the following genetic markers is
most commonly associated with this patient's disease?
/ A. HLA-B5
/ B. HLA-B8
/ C. HLA-B18
/ D. HLA-B27
/ E. HLA-DR5

Explanation - Q: 3.5 Close

The correct answer is D. The patient most likely has Reiter syndrome,
another spondyloarthropathy that affects young men. Reiter syndrome is
characterized classically by the triad of seronegative polyarthritis,
conjunctivitis and urethritis. HLA-B27 is found in 60-85% of patients with
Reiter syndrome and is also found in patients with psoriatic arthritis.
HLA-B5 (choice A) is associated with congenital adrenal hyperplasia.
HLA-B18 (choice C) is associated with Hodgkin disease.
HLA-B8 (choice B) is associated with autoimmune adrenalitis, celiac
disease, Graves disease and Hashimoto thyroiditis.
HLA-DR5 (choice E) is associated with Hashimoto thyroiditis and juvenile
arthritis.















A 67-year-old woman is brought to the hospital complaining of severe back pain
after falling out of a chair. A Iateral film of the spine is
obtained; the T6-T8 region is shown above. Laboratory studies demonstrate a
normocytic, normochromic anemia with a normal white blood
cell count, but a markedly elevated erythrocyte sedimentation rate (ESR). A
peripheral blood smear demonstrates rouleaux formation.
Question 1 of 5
Rouleaux formation suggests which of the following?
/ A. High calcium content in serum
/ B. High globulin content in serum
/ C. High sodium content in serum
/ D. Low potassium content in serum
/ E. Low urea content in serum

Explanation - Q: 4.1 Close

The correct answer is B. Rouleaux formation refers to the stacking of
erythrocytes (distinguished from agglutination) seen when there is a relative
or absolute hypergammaglobulinemia. It can be seen when there is excess
globulin production, as in multiple myeloma and Waldenstrm's
macroglobulinemia, and also in liver disease, when there is a loss of enough
albumin to leave a relative hypergammaglobulinemia. The increased
rouleaux formation in this patient also led to a markedly elevated erythrocyte
sedimentation rate (ESR), since the rouleaux settle out of blood much faster
than do individual erythrocytes. Rouleaux formation is unrelated to the
electrolyte or urea content of the serum.
Hypercalcemia (choice A) is a frequent finding in multiple myeloma due to
the numerous osteolytic bone lesions, and the resulting release of calcium
into the blood, but it does not contribute to rouleaux formation.
Sodium (choice C) and potassium (choice D) concentrations in the serum
do not affect the ESR or rouleaux formation.
Urea levels are often elevated, rather than decreased (choice E), in this
condition, secondary to glomerulopathy or tubulointerstitial nephritis.


Question 2 of 5

Serum electrophoresis demonstrates a spike of protein in the gamma zone.
When the electrophoresed proteins are then reacted with
specific antisera, the product is characterized. Which of the following antisera are
most likely to positively react with the excess proteins
being produced?
/ A. Anti-alpha and anti-gamma isotypes
/ B. Anti-epsilon and anti-kappa isotypes
/ C. Anti-gamma and anti-Iambda isotypes
/ D. Anti-kappa and anti-Iambda isotypes
/ E. Anti-mu and anti-delta isotypes

Explanation - Q: 4.2 Close

The correct answer is C. The most common antibody produced in multiple
myeloma is IgG, and the most common light chain produced is lambda light
chain (normal immunoglobulin light chains may be of the kappa or lambda
types). Therefore, this product would react with antisera prepared against the
gamma isotype (heavy chain constant domain determinants) and the lambda
isotype (light chain constant domain determinants).
In multiple myeloma, the antibodies are monoclonal. Thus, it is possible for
anti-alpha OR anti-gamma isotype antisera to react with the product, but not
both (choice A).
Although it is possible for a monoclonal plasma cell myeloma to produce IgE
heavy chains and kappa light chains simultaneously (choice B), it is not the
most likely type of antibody to be produced.
A plasma cell myeloma cannot produce two light chains simultaneously, e.g.,
kappa and lambda isotypes (choice D).
Mu and delta isotypes (choice E) are both heavy chain isotypes, and only
one would be produced by a monoclonal plasma cell myeloma.




Question 3 of 5



Biopsy of one of the lytic bone lesions reveals nodules with the microscopic
appearance shown above. Which of the following cellular
markers would most likely characterize these cells?
/
A. CD5 and surface immunoglobulin M
/ B. CD10 and surface immunoglobulin M
/ C. CD38 and cytoplasmic immunoglobulin
/ D. TdT and CD1
/ E. TdT and CD19

Explanation - Q: 4.3 Close

The correct answer is C. The photomicrograph shows a typical area of
myeloma. The typical appearance of a myeloma is that of sheet-like masses
of cells resembling normal plasma cells, intermixed with other cytologic
variants such as plasmablasts and multinucleated cells. The markers that
identify plasma cells in tissues are surface CD38 molecules and cytoplasmic
immunoglobulin.
CD5 and surface immunoglobulin M (choice A) are the most likely markers
to be found in the malignant cells of chronic lymphocytic leukemia.
CD10 and surface immunoglobulin M (choice B) describes the most likely
markers found in the malignant cells of Burkitt lymphoma.
TdT and CD1 (choice D) are the most likely markers to be found in the
neoplastic cells of acute lymphoblastic leukemia of the T cell type.
Remember that TdT is a marker only found on the most primitive of
lymphocyte precursors. It would never be found in a fully differentiated cell
tumor such as multiple myeloma.
TdT and CD19 (choice E) are the most likely markers to be found in the
neoplastic cells of acute lymphoblastic leukemia of the B cell type.


Question 4 of 5
The presence of Bence-Jones protein in the urine of this patient would put him at
risk for development of disease that is most closely related
to which of the following?
/ A. AIzheimer disease
/ B. Familial amyloidotic neuropathy
/ C. Hemodialysis-associated amyloidosis
/ D. Primary amyloidosis
/ E. Reactive systemic amyloidosis

Explanation - Q: 4.4 Close

The correct answer is D. Multiple myeloma is usually accompanied by the
presence of free immunoglobulin light chains (Bence Jones protein) in the
serum and urine. These proteins eventually build up to toxic levels and
precipitate in the extracellular spaces as amyloid fibrils (AL type). This is
considered a primary amyloidopathy, since its development is not secondary
to an infectious/inflammatory process.
Alzheimer disease (choice A) is the most common non-infectious
amyloidopathy in the U.S. Amyloid in this disorder is formed of amyloid beta
protein cleaved from amyloid precursor protein, and thus is not closely
related to the proteinaceous fibrils of AL (amyloid of light chain
immunoglobulin molecules).
In familial amyloidotic neuropathy (choice B), the amyloid material is
composed of transthyretin.
In hemodialysis-associated amyloidosis (choice C), the precursor to the
amyloid fibril is beta-2 microglobulin.
Reactive systemic amyloidosis (choice E) is a secondary amyloidosis
resulting from chronic inflammatory conditions. The fibril precursor in this
case would be SAA (serum amyloid A protein).

Question 5 of 5
Which of the following conditions is likely to accompany this disease?
/ A. Decreased resistance to
/ B. Decreased resistance to lnfluenza A virus
/ C. Decreased resistance to
/ D. Decreased serum concentrations of C1, C4, and C2
/ E. Increased incidence of bacteremia

Explanation - Q: 4.5 Close

The correct answer is C. Patients with multiple myeloma generally suffer
from a decrease of humoral immune responses, although their cell-mediated
responses remain relatively normal. Of the options, the protective response
to Streptococcus pneumoniae is the one that is most strongly mediated by
antibodies, primarily of the IgA type.
Resistance to fungi (e.g., Candida albicans, choice A) and parasites is
primarily through cell-mediated mechanisms.
Resistance to intracellular pathogens such as the enveloped influenza A
virus (choice B) is primarily maintained by cell-mediated mechanisms.
C1, C4, and C2 (choice D) are components of the classical complement
cascade that are used up when complement is activated by complexes of
antigen and antibody (IgG and IgM). Since the myeloma protein reflects
proliferation of a single clone, it is unlikely that the myeloma antibody would
ever encounter the antigenic determinant that fits its idiotype and activate
complement.
Recurrent Neisseria bacteremia (choice E) usually results from deficiencies
in the late components of the complement cascade (C5-8), which are
genetic, and not present in multiple myeloma.
A 32 year-old woman is brought to the emergency department by her husband.
He states that his wife suddenly developed severe right back
pain approximately three hours ago. The pain is sharp in nature, radiates towards
her groin, and comes and goes. She vomits when she
arrives at the emergency department. She has no fevers or chills, and states that
she has had no prior similar episodes. She complains of
urinary urgency and frequency, but denies dysuria or hematuria. Her last bowel
movement was yesterday and normaI. She does not
remember the date of her last menses. The patient states that when the pain is
most severe, she is unable to catch her breath. As the
physician enters the room to evaluate the patient, he observes that she is tossing
and turning, and unable to get comfortable. On examination,
her temperature is 37.8 C (100.1 F) with otherwise normal vital signs. Heart and
lung examination are within normal limits. Abdominal
examination reveals decreased bowel sounds diffusely with mild right-sided
abdominal pain. There are no peritoneal signs. There is severe
costovertebral angle tenderness on the right side only.
Question 1 of 6
These symptoms are most consistent with which of the following?
/ A. Acute appendicitis
/ B. Acute cholecystitis
/ C. Biliary colic
/ D. Ectopic pregnancy
/ E. Renal colic

Explanation - Q: 5.1 Close

The correct answer is E. This patient is exhibiting a classic picture of renal
colic. This term describes the pain that is caused by the stretching of the
collecting system or ureter, and/or distention of the renal capsule. Urinary
obstruction (usually from a calcification) is the main mechanism responsible
for renal colic. As the stone moves down the ureter, local pain is referred to
the distribution of the ilioinguinal nerve and the genital branch of the
genitofemoral nerve, thus explaining the pain she is feeling in her groin.
Severity and location of the pain can vary from patient to patient due to stone
size, location, degree of obstruction, and acuteness of obstruction.
In contrast to renal colic, patients with acute appendicitis (choice A) have
peritoneal irritation and therefore prefer to remain perfectly still. Movement
irritates the peritoneal inflammation, making their pain worse. Abdominal
examination will reveal peritoneal signs.
Acute cholecystitis (choice B) has an onset of pain that is similar to biliary
colic (see below), however, the pain persists and may be unremitting for
several days. Progression of inflammation causes gallbladder distention,
leading to inflammation of the peritoneum; therefore the patient is most
comfortable lying still.
Biliary colic (choice C) is due to the presence of an impacted stone in the
cystic duct, or Hartmann pouch, or from passage of a stone through the duct.
It is characterized by a rapid increase in intensity of pain, a plateau of
discomfort that lasts for several hours, and a gradual decrease in intensity.
Pain is felt in the right upper quadrant or midepigastrium. This colic usually
occurs after meals. There are no associated urinary complaints.
Patients with an ectopic pregnancy (choice D) have lower abdominal pain
and irregular or absent menses. They do not complaint of back/flank pain.
On examination, there is discrete tenderness over the lower abdomen.



A 32 year-old woman is brought to the emergency department by her husband.
He states that his wife suddenly developed severe right back
pain approximately three hours ago. The pain is sharp in nature, radiates towards
her groin, and comes and goes. She vomits when she
arrives at the emergency department. She has no fevers or chills, and states that
she has had no prior similar episodes. She complains of
urinary urgency and frequency, but denies dysuria or hematuria. Her last bowel
movement was yesterday and normaI. She does not
remember the date of her last menses. The patient states that when the pain is
most severe, she is unable to catch her breath. As the
physician enters the room to evaluate the patient, he observes that she is tossing
and turning, and unable to get comfortable. On examination,
her temperature is 37.8 C (100.1 F) with otherwise normal vital signs. Heart and
lung examination are within normal limits. Abdominal
examination reveals decreased bowel sounds diffusely with mild right-sided
abdominal pain. There are no peritoneal signs. There is severe
costovertebral angle tenderness on the right side only.
Question 2 of 6
If the source of this patient's pain is found to be obstructive, chemical analysis of
the obstruction would most likely reveal which of the
following?
/ A. Bilirubin
/ B. Calcium oxalate
/ C. Cholesterol
/ D. Cystine
/ E. Uric acid

Explanation - Q: 5.2 Close

The correct answer is B. The source of pain is a renal calcification. There
are a variety of components that make up a renal stone, however, in the
United States, calcium oxalate is the most common type of kidney stone.
Bilirubin (choice A) is present in pigmented gallstones. These are the most
common types of gallbladder calculi worldwide. They are characterized by
their high concentration of bilirubin and their low cholesterol content.
Cholesterol gallstones (choice C) are the most common form of gallbladder
calculi in the United States. Neither cholesterol nor bilirubin are present in
renal stones.
Cystine stones (choice D) are relatively rare. They form in acidic urine and
are secondary to an inborn error of metabolism (cystinuria) resulting in
abnormal intestinal mucosal absorption and renal tubular absorption of
dibasic amino acids including cystine, ornithine, lysine, and arginine.
Uric acid stones (choice E) comprise less than 5% of all urinary calculi.
There is a higher incidence in patients with gout or myeloproliferative
diseases, and those treated for malignant conditions with cytotoxic drugs.
Acidic pH of the urine is necessary for their formation.
Question 3 of 6
If the patient instead was found to have a struvite stone, which of the following
would be a likely associated finding?
/ A. Acidic pH of the urine
/ B. E. coli
/ C. EIevated uric acid levels
/ D. Low urinary ammonium levels
/ E. Urinary tract infection
Explanation - Q: 5.3 Close

The correct answer is E. Struvite stones are composed of magnesium
ammonium phosphate (MAP). They are found most commonly in women,
and may recur rapidly. They are associated with infection with urea-splitting
organisms such as Proteus, Pseudomonas, Providencia, Klebsiella,
Staphylococci, and Mycoplasma. The high ammonium concentration derived
from the urea-splitting organisms results in an alkaline urinary pH. The
urinary pH of a patient with a MAP stone ranges from 6.8 to 8.3 and is rarely
below 7.0 (compare with choice A). The normal urinary pH is 5.85.
E. coli(choice B) is a frequent cause of urinary tract infections. However, it is
not a urea-splitting organism and therefore is not associated with struvite
stones.
Elevated uric acid levels (choice C) are associated with uric acid stones.
Uric acid stones compromise less than 5% of all urinary calculi.
As previously stated, the ammonium levels of patients with struvite stones is
elevated, not decreased (choice D).

Question 4 of 6
Ketorolac is given intravenously to the patient. To which of the following drug
classes does this agent belong?
/ A. Antibiotic
/ B. Antidiuretic
/ C. Nonsteroidal anti-inflammatory drug
/ D. Opioid
/ E. Steroid

Explanation - Q: 5.4 Close

The correct answer is C. Ketorolac (Toradol) is a nonsteroidal anti-
inflammatory agent (NSAID) and is the only NSAID approved for parenteral
use in the US. Ketorolac can dramatically improve the pain of renal colic and
can be administered either IM or IV. In severe cases, ketorolac can be used
in combination with opioid analgesics. Ketorolac has minimal side effects,
however, it can increase prothrombin time when used in conjunction with
anticoagulants, increase phenytoin levels and methotrexate toxicity, and is
potentiated by probenecid. This agent is contraindicated in patients with
renal failure, peptic ulcer disease, or recent GI bleeding.
The use of antibiotics (choice A) in patients with kidney stones is
controversial. Antibiotic use is justified if there is evidence of a UTI. Whereas
only 3% of patients treated for renal colic have been shown to develop new
UTIs, an infection can seriously complicate the patient's clinical outcome.
The antidiuretic (choice B) desmopressin (DDAVP), a synthetic analogue of
vasopressin, has been shown to substantially reduce the pain of patients
with renal colic. Its mechanism of action is thought to be reduction of
intraureteral pressure, but it may also relax ureteral and renal pelvic
musculature.
Opioids (choice D) are an important part of acute renal colic therapy.
Morphine, meperidine, and butorphanol are the agents most commonly used.
Prednisone, an oral corticosteroid (choice E), is a potent anti-inflammatory
agent that can reduce local edema in the ureter.
Question 5 of 6
The patient continues to have nausea and vomiting. Which of the following is the
most appropriate pharmacotherapy?
/ A. Diphenoxylate
/ B. Lansoprazole
/ C. Loperamide
/ D. Metoclopramide
/ E. Sucralfate

Explanation - Q: 5.5 Close

The correct answer is D. Patients with acute renal colic frequently have
nausea and vomiting, therefore antiemetics frequently have a role in the
treatment of such patients. The antiemetic effect of metoclopramide results
from its ability to block central dopamine receptors. Clinical trials have
additionally shown metoclopramide to be effective in relief of the pain of renal
colic. Other antiemetic medications are also used, such as promethazine,
hydroxyzine, and prochlorperazine.
Diphenoxylate (choice A) and loperamide (choice C) are antidiarrheal
agents.
Lansoprazole (choice B) decreases gastric acid production by irreversibly
inhibiting the H+/K+ ATPase located in the luminal membrane of parietal
cells. This agent, and similar agents such as omeprazole, are used to treat
Zollinger-Ellison syndrome and gastroesophageal reflux disorder (GERD).
Sucralfate (choice E), or aluminum sucrose sulfate, polymerizes in the acid
environment of the stomach. The polymer then forms a protective coating
over ulcer beds, thus accelerating the healing of peptic ulcers and reducing
their rate of recurrence.

Question 6 of 6

For this stone to pass spontaneously from this patient, it must travel the entire
length of the ureter. Which of the following correctly describes
the anatomical position of the right ureter?
/ A. It crosses underneath the common iliac artery
/ B. It lies medial to the inferior vena cava
/ C. It sits on the quadratus lumborum muscle
/ D. It travels between the inferior vena cava and the descending abdominal
aorta
/ E. The right gonadal vessels cross over the ureter

Explanation - Q: 5.6 Close

The correct answer is E. The gonadal vessels roughly parallel the ureter
through much of its retroperitoneal extent. These vessels obliquely cross
over the ureter from medial to lateral before entering the renal pelvis.
Normally, the gonadal arteries are branches of the abdominal aorta. The
right gonadal vein drains directly into the vena cava while the left gonadal
vein drains into the left renal vein.
At no point does either ureter go underneath the iliac vessels (choice A).
The ureter crosses above the iliac vessels at approximately the bifurcation of
the internal and external iliac arteries.
The inferior vena cava and abdominal aorta are both retroperitoneal
structures. The aorta runs slightly on the patient's left side and the inferior
vena cava runs slightly on the patient's right side. Along its path, the right
ureter remains lateral to the inferior vena cava (choices B and D). Rarely,
the ureter may cross underneath the vena cava in its superior location. This
is called a retrocaval ureter.
In its retroperitoneal location, the ureter (on either side) is related posteriorly
to the psoas muscle. The quadratus lumborum (choice C) runs lateral to the
psoas muscle and is not related to the ureter.


A 23-year-old woman complains to her physician of chronic flatulence, abdominal
cramping, and being bloated. She delivered a healthy baby
3 months previously, and she states that her symptoms began several months
into her pregnancy. She had expected them to disappear after
delivery, but they did not. On further questioning, the woman reports that she has
been having alternating diarrhea and constipation. Her stool
appears dark and oily to her, and floats in the toilet bowI. She feels hungry all of
the time, and has been losing weight despite eating a great
deaI.
Question 1 of 5
This woman's symptoms are most suggestive of which of the following?
/ A. Biliary tract disease
/ B. Cancer of the gastrointestinal tract
/ C. Granulomatous infectious disease
/ D. Malabsorption
/ E. Psychiatric disease

Explanation - Q: 1.1 Close

NONE AVAILABLE

Question 2 of 5
The woman's physician initially suggests that the patient avoid milk products, but
her symptoms fail to improve. This finding is consistent with
which of the following conditions?
/ A. Adenomatous polyps
/ B. Appendicitis
/ C. Celiac disease
/ D. Cystic fibrosis
/ E. UIcerative colitis

Explanation - Q: 1.2 Close

The correct answer is C. Flatulence, bloating, and abdominal cramping
often reflect bacterial gas production as a result of utilization of unabsorbed
nutrients by the bacteria within the gut. On a practical basis, lactose
intolerance is the most common cause of malabsorption with flatulence, and
a trial of avoidance of lactose-containing milk products was reasonable. If
this fails to correct the problem, then more serious gastrointestinal (usually
small intestinal) disease must be considered. Of the list given in the choices,
only celiac disease and cystic fibrosis (choice D) commonly cause
malabsorption, and cystic fibrosis almost always presents in childhood.
Adenomatous polyps (choice A) are usually asymptomatic, but may cause
bleeding per rectum.
Appendicitis (choice B) usually presents with fever and right lower quadrant
abdominal pain.
Ulcerative colitis (choice E) can cause alternating diarrhea and constipation,
but would not usually cause significant malabsorption, since the small bowel
is not significantly involved.
Question 3 of 5
Which of the following is usually considered to be the most accurate way of
establishing this patient's likely diagnosis?
/ A. BIood culture
/ B. Jejunal biopsy
/ C. Nasal biopsy
/ D. Rectal biopsy
/ E. Stool for ova and parasites

Explanation - Q: 1.3 Close

The correct answer is B. Jejunal biopsy demonstrating flattened mucosal
villi is considered the gold standard for demonstrating celiac disease. The
patient should have active disease (e.g., be on a normal diet and
symptomatic) at the time of biopsy in order for the diagnosis to be
established. Some clinicians also simply do a trial of gluten-free diet, to avoid
subjecting the patient to biopsy. You should also be aware that endomysial
antibody (EMA) titers in serum are relatively sensitive and specific for celiac
disease, and have been proposed (although still not widely accepted) as a
screening tool for celiac disease.
The disease does not appear to have an infectious basis, and so blood
culture (choice A) and stool examination for ova and parasites (choice E)
would not be helpful.
Celiac disease is usually confined to the small intestine, so nasal (choice C)
and rectal (choice D) biopsies would not be helpful.
Question 4 of 5
Which of the following is an example of a food that the patient should avoid?
/ A. Apple
/ B. Bread
/ C. Carrot
/ D. Lettuce
/ E. Steak
Explanation - Q: 1.4 Close

The correct answer is B. Celiac disease appears to be the result of an
immunologically mediated toxic reaction to gluten that occurs in genetically
susceptible individuals. The gluten proteins can be found in all forms of
wheat (including durum, semolina, spelt, kamut, einkorn, and faro), as well as
related grains including barley, triticale, rye, and possibly, oats. Breads are
consequently an obvious food to avoid, but patients should be warned that
the essentially complete avoidance of these grain products may require that
they prepare most of their own foods, since a surprising variety of
commercially prepared foods contain small amounts of flour or other grain
products. Even products like vinegars, alcohols, and pharmaceuticals may
have gluten-containing additives. Celiac disease often presents in childhood,
but some individuals (such as this patient) remain either asymptomatic or
undiagnosed until some event in adulthood (such as pregnancy, child birth,
surgery, viral infection, or severe emotional stress) causes an acute
exacerbation of their disease. Consequently, the absence of a life-long
history of problems should not be used to exclude the possibility of celiac
disease in a symptomatic patient.
Fresh fruits (choice A), vegetables (choices C and D), and meats (choice
E) prepared at home without additives can be eaten safely.

Question 5 of 5
Several years later, the patient cheats on her dietary restrictions and develops a
blistering, intensely itchy skin rash that has a symmetrical
distribution and is found on her elbows, knees, and buttocks. This is most likely
which of the following?
/ A. Dermatitis herpetiformis
/ B. Herpes simplex
/ C. Herpes zoster
/ D. Molluscum contagiosum
/ E. Psoriasis
Explanation - Q: 1.5 Close

The correct answer is A. Dermatitis herpetiformis is a skin condition known
to be associated with celiac disease, and, interestingly, may also be the
presenting complaint for celiac disease. The diagnosis can be confirmed by
skin biopsy with demonstration of IgA in the biopsy tissues. More than 85%
of patients with dermatitis herpetiformis also have (obviously symptomatic or
not) gluten sensitivity, and removal of gluten from the diet may improve the
skin disease. Untreated or poorly treated celiac disease is also associated
with anemia (due to iron, folate, or vitamin B12 deficiency), osteoporosis,
vitamin K deficiency with risk of hemorrhage, nervous system disorders
secondary to nutrient deficiencies, pancreatic insufficiency, and intestinal
lymphomas. Other disorders, in addition to dermatitis herpetiformis with
which celiac disease is associated, include diabetes mellitus, thyroid
disease, lupus, IgA nephropathy, primary biliary cirrhosis, and less commonly
chronic active hepatitis, scleroderma, myasthenia gravis, Addison disease,
rheumatoid arthritis, Sjgren syndrome, and Down syndrome.
Herpes simplex (choice B) and herpes zoster (choice C) are infectious
causes of vesicular skin disease not specifically associated with celiac
disease.
Molluscum contagiosum (choice D) is due to a viral infection that induces the
formation of raised papules with central craters ("small volcanoes").
Psoriasis (choice E) is a predominately autoimmune skin condition
characterized by scaly patch formation.


During a routine examination of a 2-year-old child, the mother mentions to the
pediatrician that the child has been bloated and has passed
Iarge amounts of gas for the last several months. On further questioning, the
mother also reports that the child has been having intermittent,
but frequent episodes of watery malodorous diarrhea with abdominal cramps and
sometimes fever, which began shortly after the child was
moved to a new day care center. Other family members have also had similar
symptoms, but of shorter duration, which have since resolved.
Physical examination of the child is remarkable only for hyperactive bowel
sounds.
Question 1 of 5
Which of the following is the most likely pathogen?


Explanation - Q: 2.1 Close

The correct answer is D. All of the protozoa listed in the choices are
intestinal parasites that can cause chronic diarrhea. Giardia lamblia is the
most frequent cause of protozoal diarrhea in North America. The case history
illustrates the typical course in children who acquire the infection in a day
care setting and then go on to develop chronic infection. Giardia lamblia is an
interesting protozoal parasite whose life cycle alternates between trophozoite
and cyst stages. While most bacterial infections require inoculating doses of
hundreds to thousands of organisms or more to produce disease, giardiasis
requires ingestion of only as few as 10 or less organisms in the cyst form
(research studies have shown infection after ingestion of a single cyst). In
practice, this means that the infection is very highly communicable in a day
care or home setting, since it is difficult to remove all of the cysts from one's
hands after handling a contaminated diaper. The use of gloves and taking
care to keep children from playing with their diapers can be helpful in
controlling the spread of the disease. Giardiasis can also be acquired
through ingestion of contaminated water (it will resist chlorination of less than
8 hours but can be killed with boiling or removed with micropore filtration),
and so can also be seen among hikers and as traveler's diarrhea. The
reservoirs appear to be beavers, bears, dog, cats, and humans.


Question 2 of 5

A thorough stool exam for ova and parasites is most likely to reveal which of the
following?
/ A. Acid fast oocyst that are 5 microns in diameter
/ B. Acid fast oocysts that are 10 microns in diameter
/ C. Oval cysts with up to 4 nuclei
/ D. Round cysts containing no more than 4 nuclei
/ E. Round cysts, including some with 5 to 8 nuclei

Explanation - Q: 2.2 Close

The correct answer is C. The cysts of Giardia lamblia are oval, have
prominent cell walls, and up to 4 nuclei. A diagnosis of giardiasis can be
made by finding either the trophozoites and/or the cysts in a stool sample.
However, organisms are only found in 50% of cases if only one stool sample
is used. If three stool samples obtained on three different days are used, the
detection rate is approximately 90%. If stools are negative, Giardia can be
detected in duodenal material that can be sampled using the Enterotest. A
gelatin capsule that is attached to a nylon string is swallowed. After 4 to 6
hours, the string is removed and examined under the microscope for
trophozoites.
Choice A describes the oocysts of Cryptosporidium parvum, a common
cause of mild diarrheal illness. It causes incurable, protracted diarrhea in
AIDS patients. The diagnosis of cryptosporidiosis is made by finding oocysts
in the feces using a modified acid fast or auramine stain.
Choice B describes the oocysts of Cyclospora cayetanensis, a protozoa that
has recently been identified as a cause of diarrheal illness. In 1995 and
1996, it caused major outbreaks in the United States that were later traced to
contaminated raspberries from Guatemala. Diagnosis of cyclosporiasis can
be made by finding the acid fast oocyst in stool. Its oocyst is much larger
than that of Cryptosporidium parvum, and it is important to differentiate
between them since Cyclospora can be treated with trimethoprim-
sulfamethoxazole and other antibiotics, whereas there is no effective
treatment for Cryptosporidium parvum.
Choice D describes the cysts of Entamoeba histolytica. The diagnosis of
intestinal infection with Entamoeba histolytica can be made by finding either
trophozoites or cysts in the stool. The cysts of Entamoeba histolytica are
round, have refractile walls and contain up to four nuclei.
Choice E describes the cysts of Entamoeba coli, a nonpathogenic
protozoan. Its cysts can be distinguished from the cysts of Entamoeba
histolytica because they contain 5 to 8 nuclei.
Question 3 of 5
How long, after exposure to this organism, do most people who develop clinical
disease exhibit symptoms?
/ A. 1-2 days
/ B. 3-5 days
/ C. 1-3 weeks
/ D. 4-6 weeks
/ E. 2-3 months

Explanation - Q: 2.3 Close

The correct answer is C. Most patients who develop clinical disease
become symptomatic 1-3 weeks after cyst ingestion. Many patients have
asymptomatic infections, and it appears that some degree of immunity is
often acquired after exposure, since clinical infection is more common in
children than in adults after known exposures. Many patients who develop
clinical illness have disease that lasts only 1 to 2 weeks, but chronic cases
also occur, and are an important source for spread of the infection.
Symptoms can include diarrhea, abdominal cramps, pale and greasy stools,
fatigue, bloating, and weight loss. The mechanism by which the organisms
produce disease is still poorly understood, but they are known to colonize
(sometimes very heavily) the surface of the small intestine without invasion
into the mucosa. Some of the symptoms may be due to an acquired (and
reversible) lactase deficiency related to malfunction of the intestinal
epithelium.
Question 4 of 5
Which of the following drugs would be most effective in the treatment of this
child?
/ A. Diloxanide furoate
/ B. Iodoquinol
/ C. Metronidazole
/ D. Paromycin
/ E. Trimethoprim-sulfamethoxazole

Explanation - Q: 2.4 Close

The correct answer is C. It is important to establish an accurate diagnosis
in protozoal intestinal infections, because different organisms have markedly
different drug sensitivities and no "shot-gun" therapy is likely to be effective.
Metronidazole (Flagyl) is effective against Giardia (and also Entamoeba); it is
not officially licensed for this use in the United States but is commonly used
anyway. Alternative agents include furazolidone (less effective than
metronidazole) and oral quinacrine (no longer available in the United States
because of severe side effects).
Diloxanide furoate (choice A) and iodoquinol (choice B) and are sometimes
used in the treatment of amebiasis.
Paromycin (choice D) can be used to treat cryptosporidiosis.
Trimethoprim-sulfamethoxazole (choice E) can be used to treat isosporiasis
and cyclosporiasis.
Question 5 of 5
This disease would most likely contribute to the patient's death if which of the
following diseases were also present?
/ A. AIDS
/ B. Crohn disease
/ C. Goodpasture's syndrome
/ D. Measles
/ E. Minimal change disease
Explanation - Q: 2.5 Close

The correct answer is A. Patients who have had rectal contact during sex
with an infected partner may easily acquire giardiasis. If these patients also
have AIDS, a particularly overwhelming infection may occur, with essentially
"wall-to-wall" colonization of the intestinal mucosal surface with Giardia. In
these severe cases, the malabsorption that is produced may be sufficiently
severe to induce malnutrition, which will exacerbate the AIDS patient's
already high vulnerability to other infectious disease. The diseases listed in
the other choices are distracters that have no particular link to giardiasis.










A 62-year-old man complains to his physician that he has noticed a marked
increase in the amount of flatulence he experiences. He has also
been experiencing intermittent, but increasing abdominal pain, and has had a 20
pound weight loss in the last month. On physical
examination, he seems to be slightly jaundiced. AIkaline phosphatase and
bilirubin are increased. CT scan demonstrates a mass in the head
of the pancreas.
Question 1 of 4
Approximately how many Americans in the United States are diagnosed with this
disorder each year?
/ A. 280
/ B. 2,800
/ C. 28,000
/ D. 280,000
/ E. 2,800,000

Explanation - Q: 3.1 Close

The correct answer is C. Approximately 28,000 patients in the United
States are diagnosed with pancreatic cancer yearly, and nearly that number
also die of it yearly. Pancreatic cancer is a deadly disease, with one of the
highest mortalities of any cancer, many patients dying within one year of
diagnosis. The underlying problem is that pancreatic cancer tends to not
produce symptoms until after metastases to nearby critical structures (lymph
nodes, liver, celiac plexus, superior mesenteric vessels, Ligament of Treitz,
portal vein) have occurred. At least some pancreatic cancers appear to have
a genetic basis. The National Familial Pancreas Cancer Registry now has
over 250 families with two or more members with pancreatic cancer. Also,
the second familial breast cancer gene, BRCA2 (in both men and women),
appears to be important, and may account for the increased incidence of
pancreatic cancer seen in Ashkenazi Jews. Other rare syndromes that may
have pancreatic cancer as a component include Peutz-Jeghers syndrome,
familial melanoma, hereditary colon cancer (the form without polyps), and
hereditary pancreatitis.
Question 2 of 4
Which of the following is an important risk factor for developing this disorder?
/ A. Aflatoxin exposure
/ B. AIcohol use
/ C. Caffeine ingestion
/ D. Cigarette smoking
/ E. Hepatitis B infection

Explanation - Q: 3.2 Close

The correct answer is D. Cigarette smoking appears to be a very important
risk factor for pancreatic carcinoma, but interestingly, neither alcohol use
(choice B) nor caffeine ingestion (choice C) have been implicated. Other
risk factors include older age, race (more common in African-Americans and
some Jewish groups than Caucasians), gender (men more than women, but
may just reflect smoking rates), chronic pancreatitis, diabetes mellitus,
gastric resection, and diet (bad: meats, cholesterol, fried foods, nitrosamines;
good: fruit, vegetables).
Exposure to the fungal product aflatoxin (choice A) and hepatitis B (choice
E) are risk factors for liver, but not pancreatic, cancers.
Question 3 of 4
In patients with this man's condition with intractable pain referred to the back, the
network of nerves around the aorta is sometimes blocked
with alcohoI. This network is known as which of the following?
/ A. Celiac plexus
/ B. Choroid plexus
/ C. Esophageal plexus
/ D. Hepatic plexus
/ E. Mesenteric plexus
Explanation - Q: 3.3 Close

The correct answer is A. The celiac plexus contains the nerves around the
aorta that may be stimulated by either pressure or direct involvement by
tumor, and so produce pain.
The choroid plexus (choice B) is a vascular plexus of the eye, and is also a
vascular plexus extending into the ventricles.
The esophageal plexus (choice C) refers to the nerve plexus around the
esophagus.
The hepatic plexus (choice D) is the division of the celiac plexus that
accompanies the hepatic artery and portal vein to the liver.
There are actually two mesenteric plexuses (choice E), which accompany
the superior and inferior mesenteric arteries.

Question 4 of 4
Which of the following surgical methods is used most often to treat patients with
small tumors of the head of the pancreas with no evidence of
metastasis?
/ A. Billroth l procedure
/ B. Billroth ll procedure
/ C. Roux-en-Y gastric bypass
/ D. Vertical banded gastroplasty
/ E. Whipple procedure
Explanation - Q: 3.4 Close

The correct answer is E. While you do not need to know a great deal about
surgery for the Step 1 USMLE examination, a few operations may be
mentioned. Among these is the Whipple procedure, also known as a
pancreaticoduodenectomy. In this surgical procedure, the duodenum,
proximal pancreas, gallbladder, and sometimes, the distal stomach are
resected. The remaining pancreas, biliary tree, and stomach are then
reattached to the small intestine. The Whipple procedure is only performed in
the relatively small percentage of patients who may have resectable disease.
Tumors of the tail of the pancreas might, in theory, be more easily resected,
but this area tends to not produce any signs or symptoms until late in the
disease, after metastasis has occurred. Pancreatic cancers that are not
resectable can be treated with chemotherapy and radiation therapy.
Immunotherapy using a vaccine based on the patient's own cancer cells is
also being tried experimentally, and shows promise. An important thing to
remember when treating these patients (and other poor prognosis patients)
is that long-term survivors have occurred, and that even if death eventually
intervenes, a survival of several years may be very much worthwhile to both
the patient and his family (particularly if young children are indirectly
involved).
The Billroth I and II procedures (choices A and B ) are used to treat gastric
carcinomas and intractable peptic ulcers of the duodenum and stomach.
While you could argue that the gastric resection part of the Whipple
procedure resembles a Billroth II operation, you should pick the Whipple
procedure if both answers are available, because it is much more specific for
pancreatic carcinoma.
The Roux-en-Y gastric bypass (choice C) and vertical banded gastroplasty
(choice D) are used to treat obesity by reducing the effective stomach
volume.
A 14-year-old girl asks her pediatrician if there is anything she can do about the
large amounts of gas she passes during class at schooI. She
states that her flatulence is worse in the afternoon, and she often has diarrhea
when she goes home from schooI. Her symptoms generally
subside by morning, and she thinks she is better on weekends. She has been
having these problems for four or five years, but has the
impression that they are worse now than when she was younger.
Question 1 of 5
The physician suspects food intolerance. Statistically, intolerance to which of the
following would be most likely in this patient's age group and
with her history?
/ A. Fructose
/ B. Galactose
/ C. Lactose
/ D. Maltose
/ E. Sucrose

Explanation - Q: 4.1 Close

The correct answer is C. Lactose intolerance is the most common form of
intolerance to sugars. Some authors argue that it is often not even really a
"disease," but rather a natural consequence of the maturation of the small
intestine, with resulting reduction in the levels of mucosal lactase.
Caucasians tend to have a lower incidence of symptomatic problems than
many other races (e.g., Blacks, Asians and Native Americans). Impaired
absorption of the other sugars listed in the choices can also occur, but is
rare.
Question 2 of 5
A consulting dietician identifies a major food group that should be avoided by the
young patient and further cautions that avoiding these foods
may put her at risk for a secondary dietary deficiency. If the patient removes the
offending food from her diet, she is at greatest risk of
developing a deficiency of which of the following?
/ A. Calcium
/ B. Chloride
/ C. Iron
/ D. Potassium
/ E. Sodium

Explanation - Q: 4.2 Close

The correct answer is A. Lactose is found predominately in milk and milk
products. Aged cheeses and butter have lower concentrations of lactose and
may be tolerated by many individuals. Complete removal of milk products
from the diet puts the individual at greatest risk for developing calcium
deficiency, because milk products are a major source of calcium. Three easy
ways to prevent the deficiency are to encourage the individual to drink fruit
juice products supplemented with calcium, to use lactose-free milk products,
and to chew calcium-containing antacid tablets. Lactase enzyme is also
available in forms that can be added to milk (usually hours before drinking) or
taken as tablets before eating a lactose-containing meal.
Table salt is the usual source of sodium and chloride (choices B and E).
Meats are the most important dietary source of iron (choice C).
Dietary deficiencies of potassium (choice D) are usually only seen if severe
restriction of all fruits and vegetables is present or if the patient is taking a K-
losing diuretic.
Question 3 of 5
This patient's diarrhea would be most accurately described as which of the
following
/ A. Diarrhea related to increased intestinal motility
/ B. Diarrhea related to short gut syndrome
/ C. Exudative diarrhea
/ D. Osmotic diarrhea
/ E. Secretory diarrhea

Explanation - Q: 4.3 Close

The correct answer is D. While there is some overlap in mechanisms in
some cases of diarrhea, it can be helpful to conceptualize diarrhea into
categories based on the mechanism accounting for the diarrhea. In this case,
the lactose sugar that is retained in the gut markedly increases the
osmolarity of the gut contents, which provides a physiologic force that tends
to hold fluid in the gut.
Irritable bowel syndrome is a good example of a type of diarrhea due to
increased intestinal motility (choice A).
The diarrhea seen in infants after recovery from necrotizing enterocolitis is a
good example of diarrhea related to a short gut (choice B).
The diarrhea seen in patients with invasive amebiasis is an example of an
exudative diarrhea (choice C).
The toxin-mediated diarrhea seen in cholera is a good example of a
secretory diarrhea (choice E).

Question 4 of 5
The undigested carbohydrate passing into the colon will induce enzymes for its
metabolism in resident E. Coli. The mechanism that induces
gene expression for these enzymes most directly involves the carbohydrate
/ A. binding to a repressor protein
/ B. binding to an activator protein
/ C. binding to an enhancer element associated with the gene region
/ D. decreasing cAMP within the E. Coli
/ E. increasing cAMP within the E.Coli

Explanation - Q: 4.4 Close

The correct answer is A. The enzymes required to metabolize lactose are
encoded by the lactose operon in E. coli. Lactose induces gene expression
by binding to a repressor protein and interfering with repressor binding to the
operator region of the DNA. Lack of repression contributes to inducing
expression of the genes. E. coli carries out lactose fermentation, producing
hydrogen gas that may be exhaled and detected in the breath of individuals
who are lactose intolerant.
Lactose has no direct effect on the cAMP concentration (choices D and E).
cAMP concentrations are more directly controlled by the extracellular
concentration of glucose. In response to a low extracellular glucose
concentration, cAMP increases, binds to an activator protein (choice B) that
in turn binds to a CAP region (similar to a eukaryotic enhancer) (choice C) in
the DNA. This mechanism does not directly involve lactose.


Question 5 of 5



The result of a large population study to determine the adult distribution of
deficiency of the intestinal enzyme that would normally metabolize
the nutrient in question is shown in the graph above. Given this data, if one
wanted to determine the heritability of this deficiency, the most
appropriate test would be which of the following?
/ A. A concordance study in twins
/ B. Calculation of the Hardy-Weinberg distribution
/ C. Calculation of the linkage dysequilibrium
/ D. Log of the odds (LOD) score calculation
/ E. Spectral karyotype

Explanation - Q: 4.5 Close

The correct answer is A. The graph shows a distribution characteristic of a
multifactorial trait. If one wanted to determine the contribution of genes
(heritability) versus non-genetic factors (diet, environment, etc.), a
concordance study in monozygotic (MZ) versus dizygotic (DZ) twins would
be appropriate. Heritability is calculated as (C
MZ
- C
DZ
)/(1- C
DZ
). It is not
necessary to memorize the calculation but rather to understand the concept
of heritability and concordance studies in twins.
The Hardy-Weinberg equation (choice B) is most commonly used to
calculate carrier frequencies in recessive disease. It can also be used to
calculate the disease prevalence from the carrier frequency. Although it can
be applied to dominant genetic diseases, there is no carrier status in these
conditions and the equation is not typically used in these cases.
Linkage dysequilibrium exists if specific combinations of alleles at two loci
are seen together more often than expected by chance. Calculation of
linkage dysequilibrium (choice C) is most commonly used in mapping genes.
A LOD score calculation (choice D) is used to identify linkage between two
genetic loci. A common usage is in establishing linkage between a disease
phenotype and a known chromosomal marker in an effort to map the gene
involved.
A spectral karyotype (choice E) is used to assess cytogenetic abnormalities
such as trisomies, deletions, and translocations.


A 41-year-old woman comes to the physician 3 weeks after a vaginal delivery,
complaining of a lump in her right breast. She states that over
the past few days, she has noticed increasing redness in the area and that the
site is painful and feels somewhat firm to her. She also
complains of fever and chills. She is concerned because she is currently breast-
feeding her child. She has no medical or surgical history. She
uses acetaminophen occasionally for headaches and is allergic to sulfa drugs.
Her temperature is 37.8 C (100.1 F), blood pressure is
110/70 mm Hg, pulse is 98/minute, and respirations are 12/minute. The right
breast has an area of erythema and edema that is tender with
no fluctuance.
Question 1 of 4
Which of the following is the most likely diagnosis?
/ A. Breast carcinoma
/ B. Eczema
/ C. Fibroadenoma
/ D. Mastitis
/ E. Trauma

Explanation - Q: 1.1 Close

The correct answer is D. Mastitis represents a parenchymatous infection of
the mammary glands. It is most often seen in postpartum women who are
breast-feeding, and the symptoms typically appear 3-4 weeks postpartum.
Most women with mastitis complain of pain in the breast with an area of
redness and "hardness." Women also often have fever and chills, myalgias
and arthralgias, and tachycardia. Examination shows erythema, edema, and
tenderness.
Patients with breast carcinoma (choice A) often present with an
asymptomatic mass. While it is possible for patients with breast cancer to
have erythema and edema of the breast and tenderness on exam, this
presentation in a postpartum, breast-feeding woman is most consistent with
mastitis.
Eczema (choice B) does not present with a lump, as this patient has. It is a
superficial disorder involving the top part (epidermis and upper dermis) of the
skin only. It is also characterized by scale with erythema and not associated
with fever, chills, and systemic symptoms like this patient has.
Fibroadenoma (choice C) represents a proliferation of fibrous tissue in the
breast. It is the most common tumor in young women. Patients with
fibroadenoma typically present with complaints of a breast lump. The mass is
usually small, unilateral, firm, and freely mobile. Patients with fibroadenoma
do not usually present with erythema, edema, pain in the breast, and
systemic signs of infection.
Trauma (choice E) to the breast can lead to a condition called fat necrosis.
Patients with breast trauma usually complain of a firm area of the breast,
sometimes mobile, and occasionally with induration. However, patients with
fat necrosis from breast trauma usually recall a prior incident of trauma.
Question 2 of 4
Histological examination of diagnostic tissue from this patient would reveal which
of the following?
/ A. A Iocalized area of acute inflammation
/ B. Cellular, fibroblastic stroma enclosing cystic spaces lined by epithelium
/ C. Edema fluid within the intercellular spaces of the epidermis
/ D. Hemorrhage enclosed within the tissue
/ E. Strands of infiltrating tumor cells

Explanation - Q: 1.2 Close

The correct answer is A. This patient has a presentation that is most
consistent with mastitis, which represents an acute localized infection.
Histologic evaluation would, therefore, reveal a localized area of acute
inflammation with edema and neutrophil emigration.
Cellular, fibroblastic stroma enclosing cystic spaces lined by epithelium
(choice B) would be the histology of a fibroadenoma and not mastitis.
Edema fluid within the intercellular spaces of the epidermis (choice C) is the
histologic finding in eczema.
Hemorrhage enclosed within a tissue (choice D) describes a hematoma and
would be the expected histologic finding in a patient with trauma to the breast
with a resultant hematoma. This patient's presentation is more consistent
with mastitis than trauma.
Strands of infiltrating tumor cells (choice E) would be the expected finding in
a patient with certain types of carcinoma of the breast. This is not the
histologic finding in mastitis.

Question 3 of 4
Which of the following is most likely responsible for this pathologic process?



Explanation - Q: 1.3 Close

The correct answer is E. Staphylococcus aureus is a catalase-positive,
coagulase-positive, and beta-hemolytic organism that is the most common
cause of mastitis. The source of the organism is almost always from the
nursing infant's oropharynx. Enterotoxin F, or Toxic Shock Syndrome Toxin,
has been reported to cause toxic shock syndrome in some patients with
mastitis caused by Staphylococcus aureus.
Blunt force injury (choice A) to the breast might be expected to cause a
hematoma or fat necrosis. This patient has a presentation that is consistent
with mastitis, and not traumatic injury to the breast.
Chlamydia trachomatis(choice B) is an obligate intracellular organism. It is
most commonly found in the genital tract and is associated with cervicitis and
pelvic inflammatory disease in women, urethritis in men, and pneumonia and
conjunctivitis in newborns. It is not normally associated with mastitis.
Hormonal exposure (choice C) is not considered causative of mastitis. There
is some evidence that hormonal exposure may contribute to the development
of breast cancer. This patient, however, has a presentation more consistent
with mastitis than breast cancer.
Neisseria gonorrhoeae(choice D) is a gram-negative coccus that can cause
cervicitis, pelvic inflammatory disease, arthritis, pharyngitis, and urethritis. It
can also cause neonatal conjunctivitis. It is not commonly associated with
mastitis.

Question 4 of 4
The patient is started on dicloxacillin. This medication works via which of the
following mechanisms?
/ A. BIocking cell wall synthesis
/ B. Inhibition of bacterial dihydrofolate reductase
/ C. Inhibition of bacteriaI DNA gyrase
/ D. Inhibition of protein synthesis
/ E. Inhibition of resorption of sodium and chloride

Explanation - Q: 1.4 Close

The correct answer is A. Dicloxacillin belongs to the general class of
penicillin antibiotics. Penicillins interfere with bacterial cell wall synthesis by
binding to bacterial penicillin binding proteins, resulting in eventual bacterial
cell lysis. Bacterial resistance to penicillins results when bacterial beta-
lactamases disrupt the beta-lactam ring contained within these antibiotics.
Dicloxacillin (like methicillin and nafcillin) is synthesized to be resistant to
beta-lactamases. However, resistance to these antibiotics is increasing as
well. If a patient with mastitis does not respond to dicloxacillin, bacterial
resistance should be suspected and vancomycin should be used.
Inhibition of bacterial dihydrofolate reductase (choice B) is the mechanism of
action of trimethoprim and pyrimethamine.
Inhibition of bacterial DNA gyrase (choice C) is the mechanism of action of
the fluoroquinolones and quinolones.
Inhibition of protein synthesis (choice D) is the mechanism of action of the
lincosamines (clindamycin, lincomycin). These drugs bind the 50S subunit of
ribosomes to inhibit the bacterial protein synthesis.
Inhibition of resorption of sodium and chloride (choice E) is the mechanism
of furosemide (a loop diuretic).

A 37-year-old woman undergoes a routine breast examination. During the breast
examination, the physician is aware that the skin of the
breast moves together with the underlying breast tissue, rather than being
obviously separate from it.
Question 1 of 6
The breast tissue is normally attached to the overlying skin via which of the
following?
/ A. Cooper's ligaments
/ B. Cruciate ligaments
/ C. Falciform ligament
/ D. Poupart's ligaments
/ E. Rhomboid ligaments

Explanation - Q: 2.1 Close

The correct answer is A. The suspensory ligaments of Cooper are fibrous
condensations of connective tissue stroma that attach the mammary gland to
the dermis of the overlying skin. These are particularly prominent in the
superior aspect of the breast, and help to support the breast tissue.
The cruciate ligaments(choice B) are in the knee.
The falciform ligament (choice C) attaches the peritoneum to the liver.
Poupart's ligament (choice D) is an alternative name for the inguinal
ligament.
The rhomboid ligament (choice E) is another name for the costoclavicular
ligament.
Question 2 of 6
Careful examination of the central depressed area of the nipple demonstrates
multiple small openings. These openings are from which of the
following?
/ A. Areola
/ B. Lactiferous ducts
/ C. Montgomery's glands
/ D. Sweat glands
/ E. Terminal ductules

Explanation - Q: 2.2 Close

The correct answer is B. The lactiferous ducts open into the nipples. The
area of each of these ducts immediately below the nipple is usually dilated,
forming a lactiferous sinus, which can store a droplet of milk that helps to
initiate the baby's sucking reflex during nursing.
The areola (choice A) is the ring of darkly pigmented skin around the nipple.
Montgomery's glands (choice C) are modified eccrine glands (described
incorrectly by some authors as sebaceous glands) that provide oil and
moisture for the skin of the nipple and areola. They open into the areola in
small tubercles rather than the nipple.
Sweat glands (choice D) are common in the skin of the breast generally, but
are too small to be able to seen by the unaided eye.
The terminal ductules (choice E) of the breast system are at the deep end of
the duct system of the breasts, and receive milk from the lobular tissue.
Question 3 of 6
The physician identifies a palpable mass in one breast, and the patient is
scheduled for a "Iumpectomy." The reason that "Iumpectomies" for
breast lumps can be safely performed in some patients is that the breast is
divided into multiple lobes, each of which contains a separate
duct system with connecting lobules. The normal breast usually contains how
many lobes?
/ A. 2 to 3
/ B. 5 to 7
/ C. 10 to 12
/ D. 15 to 25
/ E. 30 to 50

Explanation - Q: 2.3 Close

The correct answer is D. The normal breast contains 15 to 25 lobes. Each
lobe can be thought of as having an "inverted tree" composed of a "trunk"
made of the lactiferous duct, "branches" made of smaller ducts that feed into
the lactiferous duct, and "leaves" made of lobular tissue. Depending on the
clinical setting, surgeons will also sometimes excise the duct system under
the nipple along with the lump in the breast.

Question 4 of 6
The surgical specimen is sent fresh from the surgical suite to the laboratory for
frozen section examination. Before cutting into the specimen,
the pathologist makes a careful gross examination, which demonstrates that
most of the specimen has replacement of the normally fatty
breast tissue with strands of dense, white, firm tissue. In a few areas, roughly
spherical lesions up to 3-cm diameter with a bluish hue to them
are seen. Palpation of these areas produces a fluctuant sensation. Which of the
following is the most likely diagnosis based on the gross
evaluation?
/ A. Breast cancer
/ B. Changes of pregnancy
/ C. Fibroadenoma
/ D. Fibrocystic disease
/ E. Mastitis

Explanation - Q: 2.4 Close

The correct answer is D. While a careful pathologist will wait for appropriate
frozen or permanent tissue histologic examination, the description given in
the question stem is most suggestive of fibrocystic disease. The fibrous part
of the lesion forms the dense, white tissue strands, while the blue lesions are
what are called "blue-domed cysts. The blue color comes from the presence
of darkly colored fluid (which usually means old hemorrhage) within the cyst.
The interior cyst wall is usually smooth. These blue-domed cysts are a
favorite of examiners because they produce a distinctive gross picture and
should be specifically associated with fibrocystic disease, but you should be
aware that in real life they do not occur with anything near the frequency of
fibrocystic disease without obvious large cysts grossly (but many smaller
cysts on microscopic examination). Fibrocystic disease of the breast is a very
common lesion, and a frequent source of palpable lumps in the breast, which
may require further evaluation through either excisional biopsy (as in this
case), Tru-cut needle biopsy (producing a thin core about 1 cm long), or
needle aspiration (producing fluid or cells for cytology).
While a diagnosis of breast cancer should obviously be confirmed
microscopically before telling the clinician, invasive breast cancer (choice A)
can be suspected when there is a hard, white, nodular area of the breast that
has irregular (but often reasonably well-defined - in contrast to the fibrotic
areas of fibrocystic disease) boundaries.
The changes of pregnancy (choice B) cannot be reliably picked up on gross
examination of the breast.
Fibroadenoma (choice C), like breast cancer, tends to produce a well-
defined mass lesion, but it typically has smooth borders and may have a
slightly gray and slightly mucoid (e.g., shiny or oily appearing) surface on
cross-section.
Long-standing mastitis (choice E) can cause breast fibrosis, but will not
cause blue-domed cyst formation. More acutely, mastitis can produce
abscesses (which appear white rather than blue).
Question 5 of 6
Frozen section examination demonstrates fibrosis and cystic spaces. AIso seen
are areas of compressed glands with a lobular orientation.
The glands are lined by a single layer of epithelial cells with oval nuclei and
regular arrangement. No true invasion of glands into the adjacent
stroma is seen. This patient probably has which of the following?
/ A. Atypical ductal hyperplasia
/ B. Ductal carcinoma in situ
/ C. Lobular carcinoma in situ
/ D. Sclerosing adenosis
/ E. Usual ductal hyperplasia

Explanation - Q: 2.5 Close

The correct answer is D. The lesion described is sclerosing adenosis. The
tip-offs in the description are the references to compressed glands and
lobular orientation. Sclerosing adenosis is a common component of
fibrocystic disease, and occurs when fibrosis distorts the normal lobular
architecture. The result can be some fairly bizarre, but completely benign,
compressed glands that may mimic carcinoma on both frozen and
permanent sections. Low power examination is often helpful, as this
emphasizes the lobular character of the lesion.
Lesions actually involving the epithelium of the duct system typically have
multiple layers of cells and range from usual ductal hyperplasia (choice E,
with low risk of invasive carcinoma and characterized by the presence of
both myoepithelial cells and epithelial cells within the duct) through atypical
ductal hyperplasia (choice A, with medium risk of invasive carcinoma and
characterized by ductal carcinoma in situ-like features only involving portions
of a duct) to ductal carcinoma in situ (choice B, with relatively high risk of
invasive carcinoma and characterized by clearly abnormal features such as
loss of myoepithelial cells and formation of cribriform patterns involving
complete cross-sections of ducts).
Lobular carcinoma in situ (choice C) typically produces lobular units whose
lumina are completely filled with epithelial cells.
In practice, while you may be asked to distinguish classic examples of the
different lesions mentioned in the choices on examinations, you should be
aware that this whole area can be very problematic in real-life microscopic
examinations of breast tissue, and one piece of breast tissue sent to different
experts in breast pathology may be returned with a variety of diagnoses.


Question 6 of 6
Which of the following breast lesions is considered to have the greatest potential
for eventual progression to a malignant lesion?
/ A. Apocrine metaplasia
/ B. BIue dome cyst
/ C. Epithelial hyperplasia
/ D. Fat necrosis
/ E. Fibrosis


Explanation - Q: 2.6 Close

The correct answer is C. While fibrocystic disease may have many
components, including cyst formation (choice B), apocrine metaplasia
(choice A, a benign alteration of cyst epithelium to resemble that of apocrine
sweat glands), sclerosing adenosis, and fibrosis (choice E), only the
epithelial hyperplasia (usual, atypical, or carcinoma in situ) is thought to
indicate significant premalignant (or malignant, for carcinoma in situ)
potential. For this reason, most pathologists pay particular attention to the
epithelial lining of the ducts and lobules when evaluating breast biopsy
specimens with fibrocystic disease. Fibrocystic breasts without any evidence
of epithelial changes do not appear to have any significant increased risk of
progression to breast cancer. (You should, however, be aware that a
fibrocystic breast may make both breast palpation and mammography more
difficult and make it more likely to miss a small lesion.)
Fat necrosis (choice D) is seen after breast trauma, and has no significant
malignant potential.


A 47-year-old man presents to his physician because he has noticed that his
right breast is noticeably larger than his left breast. Breast
examination demonstrates that the right breast is diffusely enlarged. The breast
tissue is freely mobile and without distinct masses. The
patient reports that the breast has been slowly enlarging over a period of several
years.
Question 1 of 7
Which of the following is the most likely diagnosis?
/ A. Ductal carcinoma in situ
/ B. Fibroadenoma
/ C. Gynecomastia
/ D. Invasive ductal carcinoma
/ E. Lobular carcinoma in situ

Explanation - Q: 3.1 Close

The correct answer is C. The most common cause for male breast
evaluation in the United States is gynecomastia, or benign breast
enlargement. Gynecomastia may involve one or both breasts. Male breasts
can also develop ductal carcinoma in situ (choice A), invasive ductal
carcinoma (choice D), and fibroadenoma (choice B), but these lesions are
much less common than gynecomastia.
The glands in male breasts do not have lobular tissue, so lobular carcinoma
in situ (choice E) does not occur.

Question 2 of 7
AIso noted on physical examination is the presence of multiple spider angiomata.
These are most closely associated with disease of which
of the following organs?
/ A. Liver
/ B. Prostate
/ C. Stomach
/ D. Testes
/ E. Thyroid

Explanation - Q: 3.2 Close

The correct answer is A. Spider angiomas are small vascular lesions with
small blood vessels radiating from a central point. They are specifically
associated with liver disease, particularly due to alcohol abuse. They are not
specifically associated with diseases of the other organs listed in the choices.


Question 3 of 7

The presence of the spider angiomas should specifically trigger a question to the
patient about his history of use of which of the following?
/ A. AIcohol
/ B. Caffeine
/ C. Cigarettes
/ D. Heroin
/ E. Marijuana

Explanation - Q: 3.3 Close

The correct answer is A. Spider angiomas are most often associated with
liver disease related to alcohol abuse. Abuse of the other items listed in the
choices does not predispose for spider angioma formation.

Question 4 of 7
Which of the following is the most likely mechanism causing a relative excess of
hormone leading to the breast enlargement in this patient?
/ A. Decreased production of testosterone secondary to primary hypogonadism
/ B. Drug that inhibits testosterone synthesis
/ C. Drug with estrogen-Iike activity
/ D. Increased peripheral conversion of androgens to estrogens
/ E. Increased production of estrogen by a cancer

Explanation - Q: 3.4 Close

The correct answer is D. Peripheral conversion of androgens (testosterone
and androstenedione) to estrogens occurs mainly in adipose tissue, muscle,
and skin. In patients with chronic liver disease, malnutrition, and
hyperthyroidism, this peripheral conversion is increased, and may be
associated with feminization (seen as changes in hair distribution, body fat
distribution, and breast size).
Conditions that cause primary or secondary hypogonadism (choice A) can
cause gynecomastia by the mechanisms of decreased production and/or
action of testosterone. These conditions can include Klinefelter syndrome,
congenital anorchia, testicular trauma or torsion, viral orchitis (e.g., mumps),
pituitary tumors, and renal failure.
Drugs that can cause gynecomastia by inhibiting testosterone synthesis
(choice B) or action include ketoconazole, metronidazole, cisplatin,
spironolactone, and cimetidine.
Drugs that can cause gynecomastia because of their estrogen-like activity
(choice C) include diethylstilbestrol, digitalis, and estrogen-containing foods
and cosmetics.
Gynecomastia can also be seen as a consequence of increased estrogen
production by some tumors (choice E), including testicular tumors and
cancers secreting ectopic hCG (from lung, kidney, GI tract, and extragonadal
germ cell tumors). Gynecomastia can also occur as a normal physiologic
variant, particularly during puberty and in older men.

Question 5 of 7
Which of the following enzymes converts androgens to estrogens?
/ A. ALA synthase
/ B. Aromatase
/ C. Branching enzyme
/ D. MethylmalonyI CoA mutase
/ E. PRPP synthetase

Explanation - Q: 3.5 Close

The correct answer is B. The enzyme aromatase is found in adipose tissue
(and hence is increased in fat people), muscle, and skin. It acts on
androgens to form estrogens by adding double bonds to make a benzene-
like ring. It is this enzyme whose activity is relatively increased in liver
disease.
ALA synthase (choice A) is an important early enzyme in heme synthesis.
Branching enzyme (choice C) is used in glycogen formation.
Methylmalonyl CoA mutase (choice D) is involved in the propionic acid
pathway leading to synthesis of succinyl CoA.
PRPP synthetase (choice E) occurs in purine synthesis.






Question 6 of 7

Which of the following is a genetic syndrome associated with this patient's
condition and a 10- to 20-fold increased incidence of breast
cancer?
/ A. Cushing syndrome
/ B. Down syndrome
/ C. Hashimoto disease
/ D. KIinefelter syndrome
/ E. Turner syndrome

Explanation - Q: 3.6 Close

The correct answer is D. The genetic condition Klinefelter syndrome
(47,XXY) is fairly unique among the causes of gynecomastia in that it is
accompanied by an increased incidence of male breast cancer. In most
types of patients with gynecomastia, the incidence of breast cancer is not
increased, and there are no adverse medical complications of the
gynecomastia (although social and psychological problems may occur).
Gynecomastia is usually only treated (with surgery) in patients in whom the
breast either continues to enlarge or is personally troubling to the patient.
Gynecomastia may occur in Cushing syndrome (choice A) and
hyperthyroidism related to Hashimoto disease (choice C), but these are not
genetic diseases.
Gynecomastia is not usually a feature of Down syndrome (choice B), or
Turner syndrome (choice E).
Question 7 of 7
The normal male breast differs from the normal female breast in which of the
following ways?
/ A. Adipose tissue is absent
/ B. Dermis is absent
/ C. Ductal tissue is absent
/ D. Lobular tissue is absent
/ E. Muscle is absent

Explanation - Q: 3.7 Close

The correct answer is D. The normal male breast (and the male breast with
gynecomastia), unlike the normal female breast, lacks lobular tissue. Both
male and female breasts contain ductal tissue (choice C), adipose tissue
(choice A), dermis (choice B), and small amounts of smooth muscle tissue
(choice E).


A 53-year-old woman consults a physician after discovering a mass in her breast.
Physical examination demonstrates a 1.5-cm diameter,
firm mass lesion in the upper, outer quadrant of her right breast. The mass is
surgically removed and pathologic examination of tissue
obtained at surgery reveals invasive breast cancer.

Question 1 of 5

What percentage of breast masses are discovered by the patient, rather than by
mammography or physician examination of the breasts?
/ A. 5%
/ B. 20%
/ C. 50%
/ D. 80%
/ E. 95%

Explanation - Q: 1.1 Close

The correct answer is D. Breast cancer accounts for the greatest number of
new cancer cases in women each year. Mammography is the screening
method used to detect subclinical breast cancer-the stage at which breast
cancer is least likely to have spread, but about 80% of breast masses are
discovered initially by the patient, which it is why it is important to continue to
stress breast self-examination.


Question 2 of 5

While about 75% of the lymphatic fluid from the breast drains first to the axilla,
most of the remaining lymphatic fluid drains first to which of the
following groups of lymph nodes?
/ A. Anterior internal thoracic nodes
/ B. Internal inferior thoracic nodes
/ C. Lateral intercostal nodes
/ D. Superior mediastinal nodes
/ E. Tracheobronchial nodes

Explanation - Q: 1.2 Close

The correct answer is A. The anterior internal thoracic nodes, also known
as the internal mammary nodes, are a pair of chained lymph nodes running
superiorly to inferiorly along the chest wall near both sides of the sternum.
They are inaccessible for surgical removal during mastectomy, but may
contain metastatic breast cancer. Rarely, the lateral intercostal nodes
(choice C) may contain metastatic breast cancer.
The inferior internal thoracic nodes (choice B) drain the liver and diaphragm.
The superior mediastinal nodes (choice D) drain the trachea, esophagus,
and heart.
The tracheobronchial nodes (choice E) drain the lung.
Question 3 of 5
Which of the following will most likely be identified at pathologic examination of
this woman's breast mass?
/ A. Ductal carcinoma
/ B. Lobular carcinoma
/ C. Medullary carcinoma
/ D. Paget disease
/ E. Tubular carcinoma

Explanation - Q: 1.2 Close

The correct answer is A. The anterior internal thoracic nodes, also known
as the internal mammary nodes, are a pair of chained lymph nodes running
superiorly to inferiorly along the chest wall near both sides of the sternum.
They are inaccessible for surgical removal during mastectomy, but may
contain metastatic breast cancer. Rarely, the lateral intercostal nodes
(choice C) may contain metastatic breast cancer.
The inferior internal thoracic nodes (choice B) drain the liver and diaphragm.
The superior mediastinal nodes (choice D) drain the trachea, esophagus,
and heart.
The tracheobronchial nodes (choice E) drain the lung.
Question 3 of 5
Which of the following will most likely be identified at pathologic examination of
this woman's breast mass?
/ A. Ductal carcinoma
/ B. Lobular carcinoma
/ C. Medullary carcinoma
/ D. Paget disease
/ E. Tubular carcinoma

Explanation - Q: 1.3 Close

The correct answer is A. There is a cumulative risk in women of developing
breast cancer of 1 in 8 by age 95; 1/3 to 1/2 of the patients die of the
disease. Breast cancer may occur in a ductal or a lobular pattern. Invasive
ductal carcinoma, not otherwise specified, is the most common histological
type of invasive breast cancer. Risk factors for breast cancer include positive
family history, early menarche, late menopause, late first pregnancy, and
history of in situ or invasive breast cancer. Women who develop breast
cancer before age 35 tend to have more aggressive disease. Two breast
cancer genes, BRCA1 and BRCA2, have been identified; 5% of women with
breast cancer carry one or the other of these genes. Male breast cancer is
much less common than female, but has a high mortality rate
Lobular carcinoma (choice B) is the second most common type of breast
cancer. It may occur bilaterally in some patients.
Medullary carcinoma (choice C) and tubular carcinoma (choice E) are
histological variants of ductal carcinoma with statistically better prognoses
than ductal carcinoma, not otherwise specified.
Paget disease (choice D) is the presence of individual adenocarcinoma cells
within the skin overlying a breast cancer.

Question 4 of 5
Immunohistochemical examination of paraffin-embedded sections through the
tumor demonstrates that it stains for estrogen receptors (ER)
and progesterone receptors (PR). This finding specifically suggests that the
tumor may respond to which of the following drugs?
/ A. 5-FIuorouracil
/ B. Cyclophosphamide
/ C. Doxorubicin
/ D. Methotrexate
/ E. Tamoxifen

Explanation - Q: 1.4 Close

The correct answer is E. Treatment with adjuvant tamoxifen for 5 years in
ER positive tumors can reduce the risk of death by 25% in both pre- and
postmenopausal women with or without axial lymph node involvement.
Breast cancers that express the Her-2/neu receptor may respond to a new
drug, trastuzumab, which is a monoclonal antibody directed against the
receptor. The other drugs listed are all used for breast cancer chemotherapy,
but work because they are cytotoxic rather than because they affect the
hormonal response of the cancer.


Question 5 of 5

The patient's physician suggests that she immediately begin chemotherapy
treatment. The patient dismisses his suggestion and says, "I do
not need any medicine, all l need is bedrest." This statement is most consistent
with which of the following responses to illness?
/ A. Acceptance
/ B. Anger
/ C. Bargaining
/ D. Denial
/ E. Grieving

Explanation - Q: 1.5 Close

The correct answer is D. Denial is a coping mechanism to defend against
overwhelming anxiety. Pathologic and extreme denial can interfere with
accurate diagnosis, impede treatment, and consequently perpetuate the
disease state. Denial is common in the early stages of dealing with a terminal
illness and is not necessarily pathologic. Less extreme forms of denial may
even serve the patient in positive ways. "I'm as strong now as I was when I
was 20 and I'm gonna make it." Notably, the stages of grieving over the loss
of a loved one (bereavement) are very similar to grieving over the loss of
one's health (stages of dying).
Acceptance (choice A) is a realistic perspective concerning the
consequences of illness. "Coming to terms" with the illness restores
emotional equilibrium and patients appear to return to their baseline
personality and emotional functioning.
Anger (choice B) is often directed at fate, God, themselves, their caretakers,
and their families and, if taken to the extreme, may result in isolation from
much needed support.
Bargaining (choice C) entails promises to buy additional time.
Grieving (choice E) is a process of changing affective states over time and
includes five stages as described by Elisabeth Kubler-Ross (denial, anger,
bargaining, depression, and acceptance). Denial, anger, anxiety, depression,
and dependence can all be abnormal responses to illness (when extreme).


A 23-year-old woman comes to the physician because of a lump in her right
breast. She states that she first noted the lump about a year ago
and that it has seemed to enlarge over the past year. She notes some occasional
tenderness in the area, usually at the same time during her
menstrual cycle. She has no medical problems. She had an appendectomy at the
age of 18. She takes no medications and is allergic to
penicillin. Examination of the breast demonstrates a freely mobile, smoothly
contoured, discrete mass in the upper outer quadrant of the
breast. UItrasonography demonstrates a smooth mass with circumscribed
margins and homogeneous echo pattern, consistent with a solid
Iesion.

Question 1 of 4

Which of the following is the most likely diagnosis?
/ A. Breast abscess
/ B. Fibroadenoma
/ C. Fibrocystic breast changes
/ D. Mastitis
/ E. Pregnancy

Explanation - Q: 2.1 Close

The correct answer is B. Fibroadenomas are the most common breast
lesions found in women under 25 years of age. Fibroadenomas are the
second most common cause of benign breast lesions (second only to
fibrocystic changes) in women of all ages. Patients with a fibroadenoma
typically present complaining of a palpable lump, often with some gradual
growth. There may be some occasional cyclic tenderness. Management is
with biopsy or close observation. If the lesion is palpable, increasing in size,
or psychologically disturbing, biopsy should be performed. If the woman is
less than 25 years of age with small fibroadenomas that appear "classic" by
imaging, then expectant management with careful continued observation can
be considered.
A breast abscess (choice A) can also present as a lump in the breast.
However, an abscess represents a localized collection of pus resulting from
an infection. Therefore, patients with a breast abscess will often have
erythema, edema, pain, and tenderness around the area of the mass. Such
patients may also have systemic signs of infection, including fever and
tachycardia. This patient has no evidence of infection.
Fibrocystic breast changes (choice C) are the most common, benign
condition of the breast. They can be present in young women, become more
common as a woman approaches the menopause, and often regress during
and after the menopause. The most common symptoms are pain and
tenderness, and the masses are usually bilateral. Mammography and
ultrasound of the breast often reveal the fibrocystic changes.
Mastitis (choice D) is an infection of the breast. It can occur in any woman,
but most often occurs in lactating women during the postpartum period.
Patients with mastitis will often present with tenderness and erythema of the
breast along with fever. Treatment is with antibiotics.
Pregnancy (choice E) is associated with a number of changes in the breast,
especially as the breast prepares for lactation. Fibroadenomas may grow
rapidly during pregnancy, but the primary diagnosis, and hence the best
answer, is still fibroadenoma

Question 2 of 4

Histological examination of diagnostic tissue from this patient would reveal which
of the following?
/ A. A classic cribriform pattern with neoplastic epithelial cells
/ B. Cystically dilated ducts plus stromal fibrosis
/ C. Irregular steatocytes and intervening necrotic material and inflammatory
cells
/ D. Lobular hypertrophy
/ E. Proliferating ducts and stromal cells

Explanation - Q: 2.2 Close

The correct answer is E. Fibroadenomas have a typical microscopic
appearance. The predominant feature is the fibroblastic stroma. This is a
delicate, cellular, fibroblastic stroma resembling intralobular stroma. Within
this fibroblastic stroma are seen proliferating ducts. These ducts are usually
compressed and are lined by benign-appearing epithelium. If the margin
includes surrounding tissue beyond the fibroadenoma, compressed breast
connective tissue forming a "capsule" to the mass may be seen as well.
A classic cribriform pattern with neoplastic epithelial cells (choice A) is what
would be revealed by histological examination of an intraductal carcinoma of
the breast. The epithelium in a fibroadenoma is benign-appearing.
Histologic examination of a biopsy specimen from a patient with fibrocystic
breast changes would demonstrate cystically dilated ducts plus stromal
fibrosis (choice B).
Irregular steatocytes and intervening necrotic material and inflammatory cells
(choice C) describes the findings on pathologic evaluation of a biopsy
specimen from a patient with fat necrosis. Fat necrosis is most commonly
caused by trauma, but can also occur after surgery or radiation therapy.
Lobular hypertrophy (choice D) is seen in pregnant women. This lobular
hypertrophy occurs during the pregnancy to allow for lactation in the
postpartum period.

Question 3 of 4
If the histologic examination revealed similar findings as in this patient, but
demonstrated increased cellularity, an elevated mitotic rate,
stromal overgrowth, and infiltrative borders, then which of the following is the
most likely diagnosis?
/ A. Fat necrosis
/ B. Fibrocystic changes
/ C. Mastitis
/ D. Normal breast tissue
/ E. Phyllodes tumor

Explanation - Q: 2.2 Close

The correct answer is E. Fibroadenomas have a typical microscopic
appearance. The predominant feature is the fibroblastic stroma. This is a
delicate, cellular, fibroblastic stroma resembling intralobular stroma. Within
this fibroblastic stroma are seen proliferating ducts. These ducts are usually
compressed and are lined by benign-appearing epithelium. If the margin
includes surrounding tissue beyond the fibroadenoma, compressed breast
connective tissue forming a "capsule" to the mass may be seen as well.
A classic cribriform pattern with neoplastic epithelial cells (choice A) is what
would be revealed by histological examination of an intraductal carcinoma of
the breast. The epithelium in a fibroadenoma is benign-appearing.
Histologic examination of a biopsy specimen from a patient with fibrocystic
breast changes would demonstrate cystically dilated ducts plus stromal
fibrosis (choice B).
Irregular steatocytes and intervening necrotic material and inflammatory cells
(choice C) describes the findings on pathologic evaluation of a biopsy
specimen from a patient with fat necrosis. Fat necrosis is most commonly
caused by trauma, but can also occur after surgery or radiation therapy.
Lobular hypertrophy (choice D) is seen in pregnant women. This lobular
hypertrophy occurs during the pregnancy to allow for lactation in the
postpartum period.

Question 3 of 4
If the histologic examination revealed similar findings as in this patient, but
demonstrated increased cellularity, an elevated mitotic rate,
stromal overgrowth, and infiltrative borders, then which of the following is the
most likely diagnosis?
/ A. Fat necrosis
/ B. Fibrocystic changes
/ C. Mastitis
/ D. Normal breast tissue
/ E. Phyllodes tumor

Explanation - Q: 2.3 Close

The correct answer is E. Phyllodes tumors are similar to fibroadenomas in
that they arise from intralobular stroma. Furthermore, on pathologic
evaluation, low-grade phyllodes tumors can resemble fibroadenomas.
However, there are important differences. First, most phyllodes tumors
present in the sixth decade, whereas fibroadenomas most commonly present
in young women. Also, while most phyllodes tumors are low-grade tumors
that only rarely metastasize, some are aggressive high-grade lesions that
commonly recur locally and do metastasize hematogenously. These
aggressive lesions are often called cystosarcoma phyllodes. Some phyllodes
tumors are small, while others may be large enough to involve virtually the
entire breast. Grossly, these tumors often have leaf-like projections off of
them. On histologic evaluation, the keys to distinguishing between
fibroadenoma and phyllodes tumor are the increased cellularity, enhanced
mitotic rate, stromal overgrowth, nuclear pleomorphism, and infiltrative
borders that are seen in phyllodes tumors and are absent in fibroadenomas.
Fat necrosis (choice A) demonstrates necrotic fat cells that are surrounded
by lipid-filled macrophages and an infiltration of neutrophils. It does not
resemble a fibroadenoma.
Fibrocystic changes (choice B) are characterized by cysts, and do not
closely resemble fibroadenomas, as low-grade phyllodes tumors do.
Mastitis (choice C) is an infection of the breast, usually by Staphylococcus
aureus. It is characterized by acute inflammation and does not resemble
fibroadenoma.
Normal breast tissue (choice D) is not characterized by a pattern similar to
fibroadenoma, except with increased cellularity, elevated mitotic rate, stromal
overgrowth, and infiltrative borders. These are characteristics of phyllodes
tumors.

Question 4 of 4
If the patient were instead found to have an aggressive breast carcinoma with a
poor prognosis, the Kubler-Ross model predicts that she will
go through which of the following stages?
/ A. Acceptance, anger, ambivalence, deniaI, depression
/ B. BIues, depression, psychosis, treatment, resolution
/ C. DeniaI, anger, bargaining, depression, acceptance
/ D. DeniaI, anger, psychosis, homicide, suicide
/ E. Realization, infantilization, socialization, condemnation

Explanation - Q: 2.4 Close

The correct answer is C. Elisabeth Kubler-Ross is the psychiatrist who
authored the groundbreaking "On Death and Dying" in 1969. She was born in
Switzerland, but moved to the United States in the 1950s, where she worked
with dying patients. Her model identified five stages that occur when
individuals are confronted with death: 1) Denial, 2) Anger, 3) Bargaining, 4)
Depression, and 5) Acceptance. While she originally described this process
as it relates to facing death, others have used these 5 steps to describe
reaction to grief or loss. While these stages are useful to understand and
contemplate the experience of grief or dying, it is important to recognize that
not all people will go through these steps. Those who do go through each of
these steps may also not go through them in the "order" described by the
Kubler-Ross model. No model is perfect for explaining the intricacies of every
different human being's response to grief or death. The Kubler-Ross model
does provide a nice framework and starting point for understanding these
difficult issues.
None of the other choices correctly describes the stages proposed by
Kubler-Ross.


A 45-year-old woman presents to her primary care physician complaining of
fatigue, weight gain, and shortness of breath. She has always
been an active athlete, but in the past 2 weeks, has found it impossible to jog for
more than a few minutes, after which she feels tired and
winded. She feels like her appetite is normal or has even declined, but she
notices that she has gained 15 pounds and her pants and shoes
no longer fit welI. She has very little energy, and is sleeping poorly, with
occasional difficulty breathing at night. She denies any pain, fever, or
chills. Review of her chart reveals an up-to-date health screening including a
normal baseline mammogram, a normaI Pap smear in the last
year, and total cholesterol of 165 mg/dL two years ago. On physical examination,
she appears comfortable, has a temperature of 36.8 C
(98.2. F), blood pressure of 135/68 mm Hg, pulse of 90/min, and respiratory rate
of 24/min. She appears fatigued but not in acute distress,
and her skin appears normaI. Expiratory wheezes are heard at the bases of both
lungs. Her heart has a normaI-sounding S1 and S2, with a
II/IV soft holosystolic murmur heard best at the apex of the heart. Her abdomen
is modestly distended, and her ankles are edematous. A
chest x-ray film reveals cardiomegaly as well as increased vascular markings in
the lung beds and bilateral small pleural effusions.
Laboratory studies show:
Question 1 of 5
Which of the following is the most likely diagnosis?
/ A. Acute leukemia
/ B. Cardiomyopathy
/ C. Fibromyalgia
/ D. Hypothyroidism
/ E. Major depressive disorder


Explanation - Q: 1.1 Close

The correct answer is B. This woman has many of the classic symptoms of
heart failure, with symptoms of both poor forward cardiac output (fatigue,
poor appetite) and of vascular congestion in both the right and left atria
(edema, abdominal distension that may be ascites, cardiomegaly, pulmonary
vascular congestion and effusions seen on chest x-ray, dyspnea with
exertion, and paroxysmal nocturnal dyspnea.)
Acute leukemia (choice A) is a potential cause of fatigue, poor energy, and
poor nutritional status (which can cause edema and pleural effusion). Usually
some abnormality will be apparent, most commonly pancytopenia, due to
replacement of bone marrow with leukemic cells; the leukocyte count may be
elevated due to the presence of leukemic cells in the peripheral blood. They
often present with bleeding or infectious complications of pancytopenia.
Anemia could potentially cause a murmur due to elevated cardiac output, but
an acute leukemia would not typically cause cardiomegaly or pulmonary
edema.
Fibromyalgia (choice C) is a potential cause of fatigue, poor energy, and
poor sleep, especially in women ages 25-45: its principal sign, however, is
diffuse musculoskeletal pain and stiffness, with characteristic tender trigger
points. It is not consistent with this patient's chest x-ray abnormalities or
cardiac and lung findings.
Based on examination, this patient could certainly have hypothyroidism
(choice D). Symptoms are usually insidious in onset and include fatigue,
poor appetite with weight gain, poor sleep and possibly, obstructive sleep
apnea. Patients often complain of constipation, cold intolerance, stiffness
and muscle cramping, as well as decreased intellectual activity. Severe
hypothyroidism can result in cardiomegaly, pericardial effusion, and
symptoms of cardiac failure. The skin often appears dry, rough, and doughy
in texture. The normal TSH, however, makes hypothyroidism in this patient
very unlikely: The TSH is nearly always elevated, as most hypothyroidism is
primary, which means the pituitary is secreting maximal TSH in an attempt to
stimulate a hypofunctional thyroid gland. Rarely, TSH may be normal or
depressed (even undetectable) in pituitary or hypothalamic failure. To rule
this out, one might test first for T4 and T3 levels. Normal levels of these, in
conjunction with the normal TSH, would rule out hypothyroidism as a cause
of this clinical presentation.
Major depression (choice E) should always be in the differential for a patient
who presents with disturbances in sleep, appetite, and energy, and can also
result in weight loss or gain. These "vegetative signs" of depression may be
the presenting abnormality in a depressed patient who does not note a mood
disturbance themselves. One should also ask about depressed mood,
anhedonia (loss of interest in or inability to take pleasure in activities the
person normally enjoys), an inability to concentrate and carry on usual
intellectual activities, feelings of worthlessness or guilt, and suicidal ideation.
Depression cannot, however, on its own, produce the physical findings this
patient has, which taken together, are worrisome for some physiologic
abnormality.
Question 2 of 5
Which of the following is the most likely cause of the patient's murmur?
/ A. Aortic insufficiency
/ B. Aortic stenosis
/ C. High-output flow murmur
/ D. Mitral regurgitation
/ E. Mitral stenosis
/ F. Pulmonic insufficiency
/ G. Pulmonic stenosis
/ H. Tricuspid regurgitation
/ I. Tricuspid stenosis


Explanation - Q: 1.2 Close

The correct answer is D. Mitral regurgitation is characterized by a
holosystolic murmur heard best at the apex, often with a blowing sound,
which may radiate to the axilla.
The murmur of aortic insufficiency (choice A) is a decrescendo diastolic
murmur. Remember that the aortic valve is open during systole; a systolic
murmur, then, cannot represent regurgitant aortic flow due to an improperly
closed valve.
Aortic stenosis (choice B) does produce a systolic murmur caused by
turbulent flow across a narrowed aortic valve during systole. This murmur is
usually a crescendo-decrescendo murmur, often with a harsh quality, and is
characteristically heard best at the base of the heart; it may radiate to the
carotids as well.
High-output states (choice C) can cause a similar soft systolic murmur to
that described here. However, this patient's history is most consistent with
cardiac failure, which is a low-output state.
Mitral stenosis (choice E) causes a murmur due to turbulent low-velocity flow
during diastolic filling of the left ventricle through a narrowed mitral orifice.
This results in a soft diastolic murmur heard best at the apex. Remember that
the mitral valve is closed during systole, therefore, an abnormal mitral sound
in systole must be the sound of abnormal regurgitant flow through a closed
valve.
The right-sided murmurs are less common, similar in quality, and usually less
loud than the left-sided murmurs (given that pressures on the right are
usually lower):
Pulmonic insufficiency (choice F), when audible, therefore causes a soft
diastolic murmur at the right upper sternal border.
Pulmonic stenosis (choice G) causes a crescendo-decrescendo systolic
murmur also heard at the base of the heart.
Tricuspid regurgitation (choice H) causes a holosystolic murmur at the left
lower or right lower sternal border.
Tricuspid stenosis (choice I) when audible, is a diastolic murmur heard best
at the same location
Question 3 of 5
BIood in the pulmonary veins is at the same pressure (during all phases of the
cardiac cycle) as blood in which of the following?
/ A. Aorta
/ B. Left atrium
/ C. Left ventricle
/ D. Right atrium
/ E. Right ventricle

Explanation - Q: 1.3 Close

The correct answer is B. The pressures in two chambers, which are not
separated by a closed valve, will be equal. The pulmonary vein empties into
the left atrium, and no valve separates the two chambers, therefore the
pressures are equal in all phases of the cardiac cycle. This patient's
pulmonary vascular congestion is likely due to elevated pulmonary venous
pressure, which is, in turn, likely due to elevated left atrial pressures.
Pressures in the aorta (choice A) will be higher than pressures in the
pulmonary veins during the cardiac cycle.
The left ventricle (choice C) is separated from the left atrium and the
pulmonary veins by the mitral valve. The pulmonary veins and the left atrium
are at the same pressure as the left ventricle during diastole, when the mitral
valve is open. With complete mitral insufficiency, the pulmonary veins are
completely exposed to left ventricular pressures during systole, resulting in
severe pulmonary edema.
The right atrium (choice D) is not in communication with the pulmonary
veins, being separated from them by, in sequence, the tricuspid valve, the
right ventricle, the pulmonic valve, the pulmonary arterial system, and the
pulmonary capillary bed.
The right ventricle (choice E), during systole, is at the same pressure as the
pulmonary artery, not the pulmonary veins. During diastole, the pulmonary
arterial pressure exceeds right ventricular pressure, and the valve is closed.


Question 4 of 5
To improve her shortness of breath, the patient is given furosemide. What is the
molecular mechanism and site of action of this drug?
/ A. ADH antagonism of in the collecting ducts
/ B. AIdosterone antagonism in the distal tubule
/ C. BIockade of sodium reabsorption in the proximal tubule
/ D. BIockade of sodium transport in the distal tubule
/ E. Inhibition of carbonic anhydrase in the proximal tubule
/ F. Inhibition of sodium-potassium-chloride cotransport in the loop of Henle

Explanation - Q: 1.4 Close

The correct answer is F. The Na-K-2Cl cotransporter in the loop of Henle
operates via an ATP-dependent sodium-potassium exchange pump in the
cell that creates a gradient for sodium diffusion from the urine space into the
cell. This maintains the sodium concentration gradient of the renal medulla.
Furosemide is the most commonly used loop diuretic; it acts by blocking the
action of the cotransporter in the thick ascending limb of the loop of Henle.
ADH antagonism (choice A) is not an important diuretic drug mechanism,
however, certain drugs, most notably lithium, inhibit ADH's action, resulting in
nephrogenic diabetes insipidus.
Aldosterone promotes the reabsorption of sodium in the late distal tubule and
collecting system and promotes the excretion of potassium. Aldosterone
receptor antagonism (choice B) is the mechanism of action of potassium-
sparing diuretics such as spironolactone.
Sodium reabsorption in the proximal tubule (choice C) is a largely passive
process, which is coupled to the transport of organic solutes and anions and
also to chloride transport, via both transcellular and paracellular
mechanisms.
The thiazide diuretics work primarily by blocking sodium transport in the early
portion of the distal tubule (choice D).
Acetazolamide inhibits carbonic anhydrase (choice E), preventing the
luminal transformation of bicarbonate into CO
2
, which diffuses back into the
cell. Inhibition of this enzyme increases both bicarbonate and sodium
concentrations in the urine, resulting in high urine pH and metabolic acidosis.
Question 5 of 5
What important physiologic effect will starting this patient on an angiotensin-
converting-enzyme inhibitor achieve?
/ A. Decrease in arteriolar resistance, resulting in less resistance to forward
cardiac output
/ B. Decrease in cardiac filling pressures, resulting in less pulmonary congestion
/ C. Increase in arteriolar resistance, resulting in improved blood pressure
/ D. Increase in left-ventricular end-diastolic volume, improving stroke volume
via Starling forces
/ E. Increase in myocardial contractility, resulting in improved stroke volume
/ F. Stabilization of myocardial membranes, resulting in reduced risk of
arrhythmia
Explanation - Q: 1.5 Close

The correct answer is A. In cardiac failure, the juxtaglomerular apparatus
releases renin in response to low blood pressure or low flow states. Renin
cleaves angiotensinogen into angiotensin I, which is then cleaved by
angiotensin-converting enzyme (ACE) into angiotensin II. Angiotensin II is a
potent vasoconstrictor and increases blood pressure. This, however,
increases the resistance against which the heart must pump, thereby
reducing cardiac output. By reducing angiotensin II activity, systemic
vascular resistance (normally high in cardiac failure, in an attempt to maintain
blood pressure in the presence of low flow) is reduced, permitting the heart
to eject more volume against a lower aortic pressure. This is often described
as "afterload reduction" and is the mainstay of therapy in congestive heart
failure. Paradoxically, blood pressure may not change: the reduced
resistance, by permitting increased flow, may result in no net change in
pressure. This is most easily understood as a physiologic manifestation of
Ohm's law: V = IR. In electricity, this law means that voltage is equal to
current times resistance. Blood pressure is analogous to voltage, cardiac
output to current flow, and the resistance in this case is the resistance of the
systemic vasculature.
Reduction of cardiac filling pressures (choice B) or "preload," is also an
important aspect of the treatment of heart failure. In heart failure, the heart
operates at high filling pressures and high left ventricular end-diastolic
volume (LVEDV) because both aldosterone and ADH promote the retention
of fluid in response to low forward flow and decreased effective circulating
volume. The result is vascular congestion in the pulmonary veins due to the
high LV diastolic pressure, resulting in symptomatic pulmonary edema. By
reducing this preload, congestive symptoms can be relieved, and LVEDV
can be reduced without significant loss of stroke volume. ACE inhibitors,
however, do not reduce preload: drugs that do this are nitrates (which act as
venodilators) and diuretics.
Increasing arteriolar resistance (choice C) in heart failure increases the
"afterload" against which the heart must eject and does not improve cardiac
output.
Increasing LVEDV (choice D) is usually helpful in hypovolemia or other
states in which inadequate volume is available to the heart, thereby limiting
cardiac output. This happens in the portion of the Starling curve at low
LVEDV, where an increase in LVEDV results in a large increase in stroke
volume. Patients in symptomatic heart failure like this patient operate at very
high LVEDV and benefit from its reduction.
Increasing myocardial contractility (choice E) is beneficial in heart failure,
and is the mechanism of action of inotropic drugs. This is not a mechanism of
ACE inhibitors.
Prevention of arrhythmia (choice F) is also important in heart failure, as the
dilated heart is vulnerable to both atrial and ventricular arrhythmias. This is
not a direct action of ACE inhibitors, however.

A 23-year-old man presents to the urgent care clinic complaining of severe throat
pain, fever, chills, and diffuse joint pains. He first developed
symptoms two weeks ago and was evaluated by another physician at the same
clinic. A throat culture was done, and the patient was given a
prescription for antibiotics that he did not filI. He now returns with a worsening of
his symptoms. He has since developed severe joint pain and
swelling, which first affected his right wrist, then spread to both knees, and now
has also affected his left ankle. He also complains of
moderate to severe chest discomfort and shortness of breath. His temperature is
38.7 C (101.6 F), blood pressure is 118/86 mm Hg, pulse
is 104/min, and respirations are 20/min. There is an exudate on his oropharynx
and bilateral anterior cervical lymphadenopathy. On lung
examination, there are bibasilar crackles, and the cardiac examination reveals
tachycardia, but a normal rhythm and no murmurs or rubs.
Examination of his joints reveals synovitis in his right wrist, Ieft ankle, and both
knees. A c `. ... . ... 1 .. . .
;.. 1 `....`.``-..--.....`..```..`-.-`.--.`
Question 1 of 5
:
Which of the following is the most likely cause of this patient's cardiac findings?
/ A. Acute myocardial infarction
/ B. Aortic dissection
/ C. Mitral regurgitation
/ D. Myocarditis
/ E. Wolff-Parkinson-White (WPW) syndrome

Explanation - Q: 2.1 Close

The correct answer is D. The patient has myocarditis, which is an
inflammation of the cardiac muscle. It is most commonly the result of an
infectious process. Signs and symptoms can range from an asymptomatic
state to arrhythmias, heart failure, and death. The patient often has an
antecedent infection, and in this case, he had an exudative pharyngitis.
Acute myocardial infarction (choice A) usually presents with severe
squeezing left-sided chest pain that can radiate down the left arm. Patients
are generally middle-aged, and can have risk factors for cardiac disease
such as hypertension, diabetes, hypercholesterolemia, or a history of
tobacco use. The electrocardiogram can vary from nonspecific T wave
changes to ST segment elevation.
Aortic dissection (choice B) would present as sudden onset of severe chest
pain, which often radiates to the back. Patients can have hypotension,
depending on the severity of the dissection, or hypertension, which is often a
predisposing factor. Patients can also have unequal pulses in their
extremities, if the dissection affects one of the major arteries branching off
the aortic arch.
Mitral regurgitation (choice C) is a result of mitral valve insufficiency, in
which there is a regurgitant flow of blood across the mitral valve, from the left
ventricle, into the left atrium, during systole. It is often due to rheumatic heart
disease, but can also result from mitral valve prolapse, or papillary muscle
rupture. Physical examination should reveal a holosystolic murmur, heard
best at the apex.
Wolff-Parkinson-White (WPW) syndrome (choice E) is a ventricular
preexcitation syndrome associated with an atrioventricular bypass track.
Patients often have paroxysmal tachycardias, and an electrocardiogram will
often reveal a shortened PR interval, a delta wave, and a wide QRS
complex.
Question 2 of 5
What underlying condition can explain the patient's upper respiratory as well as
cardiac and joint signs and symptoms?
/ A. Acute rheumatic fever
/ B. Budd-Chiari syndrome
/ C. Ebstein's anomaly
/ D. Sjgren syndrome
/ E. Takayasu arteritis

Explanation - Q: 2.2 Close

The correct answer is A. Acute rheumatic fever is an inflammatory disorder
that affects multiple systems. There are five major criteria for rheumatic
fever: carditis, migratory polyarthritis, subcutaneous nodules, Sydenham
chorea, and erythema marginatum. There are also minor criteria: fever,
arthralgia, elevated acute phase reactants, and a prolonged PR interval.
Budd-Chiari syndrome (choice B) is an occlusion of the major hepatic veins,
which leads to congestive liver disease. Patients often have abdominal pain,
jaundice, and hepatomegaly.
Ebstein's anomaly (choice C) is due to an anomalous attachment of the
tricuspid leaflets, and results in downward displacement of the tricuspid valve
into the right ventricle. This results in tricuspid regurgitation. Symptoms can
vary from cyanosis to arrhythmias.
Sjgren syndrome (choice D) is an autoimmune disorder characterized by
inflammatory changes in glands, producing dry eyes and dry mouth.
Takayasu arteritis (choice E) is a vasculitis syndrome that affects medium to
large arteries, in particular, the aortic arch and its branches. It is also known
as "pulseless disease" because patients have weak or absent pulses in their
upper extremities. It primarily affects young Asian females.

Question 3 of 5
Which of the following test results would help confirm the most likely diagnosis?
/ A. EIevated antinuclear antibody
/ B. Low anti-deoxyribonuclease B titer
/ C. Low anti-hyaluronidase titer
/ D. Low anti-streptolysin O titer
/ E. Throat culture positive for group A streptococci

Explanation - Q: 2.3 Close

The correct answer is E. To meet criteria for the diagnosis of rheumatic
fever, patients must have either two major, or 1 major and 2 minor criteria,
plus evidence of an antecedent streptococcal infection. A throat culture
positive for group A streptococci would fulfill the criteria in the presence of
myocarditis and migratory polyarthritis.
Elevated antinuclear antibody (choice A) is not associated with rheumatic
fever. In the appropriate clinical setting, it is helpful in the diagnosis of
rheumatologic disorders such as systemic lupus erythematous.
Anti-streptolysin O (choice D), anti-deoxyribonuclease B (choice B), and
anti-hyaluronidase (choice C) are all streptococcal antibody tests. In the
setting of rheumatic fever associated with a recent group A streptococcal
infection, the titers for these antibody tests would be elevated (in the
absence of infection, they may actually be undetectable). A significant titer of
any of these antibody tests would meet criteria for the documentation of an
antecedent streptococcal infection.
Question 4 of 5
A biopsy of the affected cardiac tissue would most likely show which of the
following?
/ A. Angiosarcoma
/ B. Aschoff body
/ C. Atheromas
/ D. Hyperplastic arteriolosclerosis
/ E. Libman-Sacks lesions
Explanation - Q: 2.4 Close

The correct answer is B. The Aschoff body is the classic lesion of
rheumatic fever. It is an area of focal interstitial myocardial inflammation. It is
characterized by large cells, known as Anitschkow myocytes, and Aschoff
cells, which are multinucleated giant cells.
Angiosarcoma (choice A), a rare malignant tumor affecting the vascular
tissue, can occur in the skin, breast, liver, or musculoskeletal system.
Atheromas (choice C) are fibrous plaques within the intima of arteries. They
are a finding of atherosclerosis.
Hyperplastic arteriolosclerosis (choice D) is characterized by concentric,
laminated thickening of arteriolar walls. It often occurs in the kidneys, and
may lead to malignant nephrosclerosis.
Libman-Sacks lesions (choice E) are small vegetations that occur on
valvular heart tissue. They can occur on either side of the valve, and are
associated with endocarditis in systemic lupus erythematous.
uest|on 0
The patient continues to deteriorate, he develops worsening heart failure, and
requires transfer to the intensive care unit for use of an
inotropic agent to increase his cardiac output. Which of the following agents
would most likely be used?
/ A. Benazepril
/ B. Diltiazem
/ C. Dobutamine
/ D. Metoprolol
/ E. Phenylephrine


Explanation - Q: 2.5 Close

The correct answer is C. Dobutamine is a positive inotropic agent used in
severe cases of heart failure that require inotropic support.
Benazepril (choice A) is an angiotensin converting enzyme inhibitor.
Medications in this class can be used in heart failure to decrease afterload,
but they do not have any direct affect on cardiac tissue.
Diltiazem (choice B) and metoprolol (choice D) are both negative inotropic
agents. When used in the setting of acute heart failure, the patient's course
can worsen, although beta blockers such as metoprolol and carvedilol (mixed
alpha and beta blocker) are sometimes cautiously used in some patients with
CHF.
Phenylephrine (choice E) is an alpha-receptor agonist. It causes
vasoconstriction, and is used in severe cases of hypotension.

A 78-year-old man had been previously active, but found that his health was
declining. Over a four-month period, his ability to perform even
very minimal exercise, such as walking around his yard, declined precipitously.
The family took him from doctor to doctor, none of whom were
initially able to figure out what was wrong with him. Because of the patient's age,
most of the physicians that the family consulted were
unwilling to do much other than to listen to the family's story and then run a few
screening tests. In some ways, he acted as if he were in
congestive heart failure, but he initially had no evidence of fluid overload and his
lungs were clear. The cardiac profile on chest X-ray was
slightly enlarged. His ECG studies were interpreted as within the normal range
for his age. Angiography studies showed no evidence of
significant coronary artery occlusion. Pulmonary function studies were
unrevealing.
Question 1 of 6
Following a Thanksgiving meaI, the patient's condition worsened markedly over
the next few hours, and he was taken to an emergency
department. At that point, the patient was in obvious, severe, congestive heart
failure with evidence of fluid overload and pulmonary edema.
Intravenous furosemide was started, which over the next few hours markedly
improved his clinical condition. Furosemide is classified as
which of the following?
/ A. Carbonic anhydrase inhibitor
/ B. Loop diuretic
/ C. Osmotic diuretic
/ D. Potassium-sparing diuretic
/ E. Thiazide diuretic

Explanation - Q: 3.1 Close

The correct answer is B. Large, salty, holiday meals are notorious for
setting off (potentially fatal) exacerbations of what might have been
previously mild congestive failure. There are a number of drugs with diuretic
activity that can increase the amount of urine that is produced.
Pharmacologists subclassify these drugs based on the mechanisms by
which they act. Furosemide is a diuretic that is commonly used in the hospital
setting in intravenous form to rapidly reduce the degree of fluid overload
present in a patient in severe congestive heart failure. This diuretic acts by
inhibiting the Na/K/2Cl cotransporter on the luminal membrane of the thick
ascending portion of the loop of Henle. It is consequently classified as a loop
diuretic, as is ethacrynic acid, which has a similar mechanism of action.
Carbonic anhydrase inhibitors (choice A), such as acetazolamide and
dorzolamide, act on the proximal convoluted tubule to reduce Na
+
resorption
secondary to an inhibition of CO
2
formation with resulting decreased
intracellular bicarbonate and H
+
levels.
Osmotic diuretics (choice C), such as mannitol, inhibit water reabsorption
throughout the nephron.
Potassium-sparing diuretics (choice D), such as spironolactone, amiloride,
and triamterene, act at the level of the collecting tubules and ducts by acting
as aldosterone receptor antagonists.
Thiazide diuretics (choice E), such as hydrochlorothiazide, indapamide, and
metolazone, inhibit the Na/Cl cotransporter in the distal convoluted tubule.
Question 2 of 6
The patient is seen the following morning by a cardiologist. The cardiologist does
a very careful physical examination. He notes that the heart
sounds appear distant. He then has the patient lie at an angle of 30 to 45
degrees, and does a careful examination of the right jugular pulse,
which he finds very worrisome. The pulse is both very elevated and shows
dramatic x and y descents. Further, he notes that the venous
distention paradoxically increases during inspiration. This last finding is
sometimes called which of the following?
/ A. Chvostek's sign
/ B. Corrigan's sign
/ C. Homans' sign
/ D. KussmauI's sign
/ E. Murphy's sign


Explanation - Q: 3.2 Close

The correct answer is D. The sign described is Kussmaul's sign. The act of
inflating the lungs during inspiration lowers the pressure in the chest while
increasing that in the abdomen, drawing blood from the abdomen into the
chest (and increased abdominal pressure helps to directly drive blood toward
the chest). If the right atrium cannot fill, then the jugular venous pressure
rises paradoxically (not so much from blood flow from the head as from the
abdomen, because the inferior vena cava and superior vena cava are
functionally connected through the right atrium). Kussmaul's sign is seen in
patients who have non-compliant right ventricles. It can also be seen in
patients with severe ascites (which increases the intra-abdominal pressure).
This case illustrates the importance of considering the jugular venous pulse
as well as the arterial pulse, since the cardiologist was able to find a number
of significant findings pertaining to the jugular venous pulse, which other
physicians had missed. The jugular venous pressure can be used at the
bedside to estimate the right atrial filling pressure. The jugular venous
pressure is estimated by measuring the height of the visible venous pulse
above the sternal angle, and then adding 5 cm (corresponding to how far
below the sternum the right atrium is located). The jugular venous waveform
has an A wave, which is followed by an X descent, then a V wave, and finally
a Y descent. The A wave (first rise in pressure) reflects the right atrial
contraction, while the X-descent reflects right atrial diastole, and then early
right ventricular systole. The V wave is the second major positive wave, and
reflects continued venous inflow into the right atrium in opposition to a closed
mitral valve. The following Y-descent is the negative deflection that occurs
when the tricuspid valve opens in early diastole.
Chvostek's sign (choice A) is seen in tetany, and is a facial muscle spasm
occurring when the facial nerve is tapped anterior to the external auditory
meatus.
Corrigan's sign (choice B), which suggests aortic regurgitation, is a full, hard
arterial pulse, which is followed by a sudden collapse.
Homans' sign (choice C) is pain at the back of the knee or calf when the
ankle is dorsiflexed, and suggests venous thrombosis of the leg.
Murphy's sign (choice E) is pain on palpation of the right subcostal area
during inspiration, and is frequently seen in acute cholecystitis.
Question 3 of 6
This patient most likely has which of the following?
/ A. Acute myocarditis
/ B. Congestive cardiomyopathy
/ C. OId left ventricle myocardial infarction
/ D. Recent left ventricle myocardial infarction
/ E. Restrictive cardiomyopathy
Explanation - Q: 3.3 Close

The correct answer is E. The "distant" heart sounds and jugular venous
pulse findings both suggest that this patient has restrictive cardiomyopathy
that is limiting the heart's ability to fill during diastole and is also impairing
ventricular contraction. Other findings that may be encountered on physical
examination in patients with restrictive cardiomyopathy include S3 and/or S4
heart sounds, occasional mitral or tricuspid regurgitation murmurs, and, if the
patient is in secondary congestive failure, peripheral edema and pulmonary
rales. Restrictive cardiomyopathy is relatively rare and the findings on
physical examination are subtle, and consequently this patient's history of
missed diagnosis is unfortunately not all that uncommon. Underlying causes
of restrictive cardiomyopathy include endomyocardial fibrosis, Loeffler
eosinophilic endomyocardial disease, hemochromatosis, amyloidosis,
sarcoidosis, scleroderma, carcinoid heart disease, and glycogen storage
disease. Patients typically present at an advanced stage of the disease, and
may have symptoms of angina, shortness of breath, peripheral edema, and
ascites with abdominal discomfort (related to pooling of blood in the liver and
other abdominal organs). Once the diagnosis is suspected,
echocardiography typically demonstrates normal to symmetrically thickened
heart chamber walls with rapid early-diastolic filling and slow late-diastolic
filling (the cardiac chambers are acting more or less like poorly distensible
plastic bags). Cardiac catheterization will more or less repeat the
observations seen in the analysis of the jugular venous pulse, typically
showing elevated ventricular end-diastolic pressure, normal to slightly
decreased ejection fraction, and prominent x and y descents.
Acute myocarditis (choice A) can cause congestive cardiomyopathy (choice
B), but the heart is usually larger and the constrictive findings seen in this
case would not be present.
While recent and old myocardial infarctions affecting the right ventricle may
produce similar jugular venous findings to those seen in this case, left
ventricular infarction (choices C and D) would not impair right ventricular
filling and contraction.

Question 4 of 6
An endomyocardial biopsy is performed, which demonstrates eosinophilic
acellular deposits within the myocardial biopsy. When recut,
histological sections are stained with Congo red and viewed under polarized
light, and the deposits appear bright green. These deposits are
most likely to be composed of which of the following?
/ A. Amyloid
/ B. Fibrin
/ C. Hemosiderin
/ D. Melanin
/ E. Uric acid


Explanation - Q: 3.4 Close

The correct answer is A. Amyloid deposits are suspected when
hematoxylin and eosin-stained histological sections show extracellular
eosinophilic deposits. The presence of amyloidosis is confirmed when the
characteristic "apple-green birefringence" on Congo red stain is
demonstrated.
Fibrin deposits (choice B) are also red on hematoxylin and eosin stain, but
show no fluorescence with Congo red stain.
Hemosiderin (choice C) causes yellow brown deposits; melanin (choice D)
causes brown-black deposits; and uric acid (choice E) causes yellow
crystalline deposits.
Question 5 of 6
Which of the following features of proteins is most likely responsible for the bright
green appearance of the Congo red-stained materiaI?
/ A. Beta pleated sheet configuration
/ B. Calcium binding
/ C. Iron containing heme moiety
/ D. Multiple alpha helices
/ E. Presence of multiple subunits

Explanation - Q: 3.5 Close

The correct answer A. It was originally assumed by biochemists that
amyloid was always composed of the same material. It came as something
of a shock when antibody techniques were developed that demonstrated that
the antigenicity of amyloid in different clinical settings varied markedly. The
common feature these proteins shared that accounted for both the affinity for
Congo red and their characteristic regular fibrillar structure on electron
microscopy turned out to be that the proteins all have a beta pleated sheet
tertiary (secondary according to some biochemical purists) configuration,
best demonstrated by X-ray diffraction.
Selective or non-selective binding to calcium (choice B) is common in
proteins.
Heme moieties containing iron (choice C) are a part of myoglobin and
hemoglobin.
Alpha helices (choice D) are a common secondary structure in proteins, but
do not contribute to the protein forming amyloid.
The presence of multiple subunits (choice E) is also common in proteins, but
does not contribute to a protein forming amyloid.
Question 6 of 6
Which of the following would most likely be found in the Congo red-stained
extracellular deposits with the bright green appearance under
polarized light?
/ A. Amyloid AA
/ B. Beta-2-microglobulin
/ C. Beta protein precursor
/ D. Immunoglobulin light chains
/ E. Transthyretin

Explanation - Q: 3.6 Close

The correct answer is D. Amyloidosis occurs in a large variety of forms.
Primary amyloidosis is one of the more common forms of systemic
amyloidosis, and can affect a variety of organs, including the heart, kidney,
peripheral nerve, gastrointestinal tract, and respiratory tract. In primary
amyloidosis, the amyloid is composed of immunoglobulin light chains, and
the disease is now interpreted as a plasma cell disorder closely related to
multiple myeloma. This interpretation is clinically significant, as it has led to
modern treatments of primary amyloidosis (which formerly had a dismal
prognosis) with the chemotherapies designed for multiple myeloma. The
treatments are affective only if the disease is recognized and passed to the
appropriate specialists as early as possible in the clinical course.
Amyloid AA (choice A) is seen in inflammation-associated amyloidosis and
familial Mediterranean fever.
Beta-2-microglobulin (choice B) comprises the amyloid of dialysis-
associated amyloidosis.
Beta protein precursor (choice C) comprises the amyloid seen in the brains
of patients with Alzheimer's disease and Down syndrome.
Transthyretin (choice E) comprises the amyloid seen in familial amyloidosis
and senile cardiac amyloidosis.

A 40-year-old man presents to the emergency department complaining of severe
shortness of breath. The breathlessness has been
worsening over the past few years, and the patient reports growing tachypneic
with mild exertion, and sometimes even at night. On
examination, he has generalized edema, jugular venous distention, and hepatic
distention. Cardiac examination shows a right ventricular
heave, a right-sided S3, and S4 with a pulmonary ejection click. A chest x-ray
film shows cardiomegaly and widening of the hilar vessels,
including the pulmonary arteries. An electrocardiogram shows talI, peaked P
waves in leads lI, III, and aVF, right axis deviation, and right
ventricular hypertrophy.
Question 1 of 5
Which of the following is the most likely diagnosis?
/ A. Cor pulmonale
/ B. Hypertrophic cardiomyopathy
/ C. Left ventricular failure
/ D. Myocardial infarction
/ E. Pulmonary embolus (acute)
Explanation - Q: 4.1 Close

The correct answer is A. This patient has cor pulmonale, which is defined
as enlargement of the right ventricle secondary to diseases of the lung,
thorax, or pulmonary circulation. In this case, it is chronic, given the duration
of the patient's symptoms and the presence of many clinical sequelae of the
condition: edema, jugular venous distention, hepatic distention, and right
ventricular heave. The electrocardiogram also supports the diagnosis of
enlargement of the right ventricle showing right axis deviation due to the
increase in the mass of the right heart. Evidence of right atrial enlargement is
also present, i.e., the tall peaked P waves in leads II, III, and aVF (P
pulmonale).
Hypertrophic cardiomyopathy (choice B) is an anomaly in which the
myocardium hypertrophies. The fibers are erratic and conduction
abnormalities and outflow obstruction may result. Typically, this disorder
presents in the second decade of life, and will manifest as dysrhythmia
and/or shortness of breath. In addition, a right axis deviation would be
inconsistent with this cardiomyopathy because the left ventricle
hypertrophies, as well as the right. Thus, this diagnosis is unlikely.
Left ventricular failure (choice C) often accompanies right ventricular failure,
but in this case, the right-sided symptoms, such as systemic edema, jugular
venous distention, and hepatic congestion, are more pronounced. Left-sided
failure shows engorgement of the entire pulmonary tree in conjunction with
pulmonary edema.
Myocardial infarction (choice D) is unlikely. The ECG findings are not
consistent with the pattern typically seen in MI. In addition, this patient does
not suffer from the symptoms of myocardial infarction, such as chest pain,
pressure, jaw numbness, and diaphoresis.
Pulmonary embolus (choice E) may cause acute right heart strain and
failure, but this patient has a chronic condition. Chronic emboli may produce
increased resistance in the pulmonary tree and a picture similar to this.



Question 2 of 5
Pulmonary hypertension is suspected in the patient, and a Swan-Ganz catheter
is placed. Which of the following denotes the correct
anatomic sequence of vessels that would be traversed by the catheter if it was
introduced into the left subclavian vein?
/ A. Left subclavian vein, Ieft brachiocephalic vein, superior vena cava, right
atrium, right ventricle, pulmonary artery
/ B. Left subclavian vein, Ieft common carotid, superior vena cava, right atrium,
right ventricle, pulmonary artery
/ C. Left subclavian vein, Ieft jugular vein, Ieft atrium, Ieft ventricle, aorta
/ D. Left subclavian vein, Ieft jugular vein, superior vena cava, right atrium, right
ventricle, pulmonary artery
/ E. Left subclavian vein, superior vena cava, right atrium, right ventricle,
pulmonary artery

Explanation - Q: 4.2 Close

The correct answer is A. The correct sequence for a catheter inserted into
the left subclavian vein is as follows: left subclavian vein, left brachiocephalic
vein, superior vena cava, right atrium, right ventricle, pulmonary artery. With
this catheter in place, a variety of cardiac parameters can be measured,
including pressures in the pulmonary artery. Thus, this catheter can aid in
establishing the diagnosis of pulmonary hypertension.



Calcium channel blockers can be used in this setting to decrease pulmonary
vascular resistance. Which of the following is the calcium
channel blocker that will have the most predominant effect on vascular smooth
muscle?
/ A. Diltiazem
/ B. Hydrochlorothiazide
/ C. Nifedipine
/ D. Pseudoephedrine
/ E. Verapamil

Explanation - Q: 4.3 Close

The correct answer is C. The calcium channel blockers vary in the
propensity to affect vascular smooth muscle versus their effect on cardiac
muscle. Thus, in this case, it is important to select an agent that has
maximum ability to relax the smooth muscle in the pulmonary vessels. The
effect on smooth muscle is as follows: nifedipine>diltiazem (choice A)
>verapamil (choice E). The effect on cardiac muscle is as follows:
verapamil>diltiazem>nifedipine. Thus nifedipine is the agent of choice.
Hydrochlorothiazide (choice B) is a diuretic, and thus would have no effect
on the vascular smooth muscle.
Pseudoephedrine (choice D) is an alpha agonist, and therefore would cause
vasoconstriction.

Question 4 of 5
Which of the following physiologic stimuli will result in decreased pulmonary
vascular resistance?
/ A. Decreased cardiac output
/ B. Increased cardiac output
/ C. Low O2 tension
/ D. Lung volumes near residual volume (RV)
/ E. Lung volumes near total lung capacity (TLC)


Explanation - Q: 4.4 Close

The correct answer is B. A unique feature of the pulmonary circulation is
that it maintains itself as a low-pressure system. Many of the mechanisms
that control pulmonary vasculature differ from those of the systemic
circulation. One of these features is that pulmonary vasculature resistance is
decreased in response to increased cardiac output. This is accomplished
through distention of open capillaries and the recruitment of collapsed
capillaries. Thus, the resistance in the pulmonary tree decreases in response
to increased right ventricular output. In the pathologic state of pulmonary
hypertension, in which the resistance is elevated and the ventricle fails, this
decreased cardiac output (choice A) may compound the problem and trigger
increased resistance in spite of the primary elevation.
Low O
2
tension (choice C) in the pulmonary vessels initiates
vasoconstriction. In the systemic circulation, low O
2
tension initiates
vasodilation.
Lung volume also affects pulmonary vascular resistance. The curve of lung
volume versus pulmonary vascular resistance is U-shaped. This effect is due
to the fact alveolar and extra-alveolar vessels act as resistors in series
(additive), and these vessels have little intrinsic support. Thus, resistance in
these vessels is affected by pleural pressures. At low lung volumes (choice
D), the alveolar vessels are open, but extra-alveolar vessels are
compressed. At high lung volumes (choice E), the alveolar vessels are
compressed by distended alveoli, but the extra-alveolar vessels become
distended due to the increase in transmural pressure. Thus a U-shaped
curve describes this relationship.
Question 5 of 5
Some of the examination findings indicate hepatic congestion. Which of the
following terms is commonly used to identify the macroscopic
pattern of red, depressed hepatic nodules with pale periphery that accompanies
the chronic hepatic congestion seen in this condition?
/ A. Centrilobular hemorrhage
/ B. Cirrhosis
/ C. Fatty change
/ D. Nutmeg liver
/ E. Piecemeal necrosis


Explanation - Q: 4.5 Close

The correct answer is D. Chronic passive congestion of the liver leads to a
macroscopic pattern known as nutmeg liver. This is due to the congestion of
blood in the centrilobular region (dark) with hypoxia and fatty change in the
more peripheral hepatocytes. When viewed macroscopically, this pattern
resembles that seen in a cross section of a nutmeg, hence the name.
In this condition, centrilobular hemorrhage (choice A) usually only occurs in
severe acute ischemia. This patient has a chronic condition, and thus most
likely will have nutmeg liver instead.
Cirrhosis (choice B) of the liver may result from chronic damage caused by
chronic congestion. It however produces a scarred, whitish, shrunken liver,
and not the pattern seen here.
Fatty liver (choice C) would produce a large, smooth yellow liver and would
not resemble the pattern seen here.
Piecemeal necrosis (choice E) is a microscopic finding of scattered
hepatocellular necrosis. This diagnosis cannot be made macroscopically.


A 45-year-old man presents to the emergency department complaining of chest
pain, which began twenty minutes before while he was filling
up his car with gasoline. He describes the pain as substernaI, intense, dulI, and
squeezing. It does not change with respiration. He also
complains that he is nauseated. He has never experienced anything like this
before. His temperature is 37.5 C (99.5 F), blood pressure is
124/76 mm Hg, pulse is 80/min, respiratory rate is 22/min, and oxygen saturation
is 95% on room air. On physical examination, he is
diaphoretic. His lungs are clear, his heart rate is regular, and he has a normaI S1
and S2 without murmur, rub, or gallop. The examiner
estimates that his jugular venous pressure is elevated to the angle of the jaw. His
abdomen is nontender, with normal bowel sounds. An
electrocardiogram is performed, which reveals sinus rhythm, normal axis, normal
intervals, and ST elevation in leads lI, III, and aVF. A chest x-
ray film reveals no apparent cardiac or pulmonary abnormalities.

Question 1 of 7

Which of the following is the most likely diagnosis?
/ A. Acute myocardial infarction
/ B. Aortic dissection
/ C. Gastroesophageal reflux
/ D. Pericarditis
/ E. Pulmonary embolus

Explanation - Q: 1.1 Close

The correct answer is A. The differential diagnosis of chest pain is broad
and includes all the answers on this list, all of which can present, as in this
patient, with a relatively normal examination of the heart and lungs. Acute
myocardial infarction is the only one of these findings that is associated with
ST segment elevation in an anatomical distribution (in this case, the "inferior"
leads). His elevated jugular venous pressure is also a clue to abnormal
cardiac function; however, this might be present in severe constrictive
pericarditis or pulmonary embolism as well.
Aortic dissection (choice B) often presents with chest pain or pain radiating
to the back. It is not, however, typically associated with ST segment changes
on the EKG, unless the dissection extends proximally into the ostia of the
coronary arteries, obstructing flow, and resulting in secondary acute
myocardial infarction, in which case a patient could present like this.
However, this presentation would be an uncommon presentation of a
relatively uncommon disease. The risk for aortic dissection is increased with
long-standing essential hypertension, other peripheral vascular disease,
hyperlipidemia, and advanced age, as well as connective tissue disorders
such as Marfan syndrome or Ehlers-Danlos syndrome.
Patients with gastroesophageal reflux (choice C) may complain of intense
substernal chest pain that is difficult to distinguish from the pain of
myocardial infarction. However, the ST elevations on the EKG suggest
transmural ischemia of the myocardium and do not occur with isolated
gastroesophageal reflux. Do not let relatively normal vital signs and
gastrointestinal symptoms such as nausea fool you!
Pericarditis (choice D), an inflammatory disease of the pericardium, presents
with chest pain and is often associated with ST elevation on the EKG, as well
as PR interval depression. However, the ST elevation usually involves
multiple leads of the EKG, and is not in a strictly anatomic distribution.
Pericarditis is often associated with a friction rub.
Pulmonary thromboembolism (choice E) can also present with chest pain. A
patient who has had a large PE, however, typically will not have normal vital
signs, and is likely to have tachycardia, tachypnea, and possibly hypoxemia.
Hemodynamically significant pulmonary embolism can present with elevated
JVP due to right heart strain. The patient might have ST depression in the
inferior leads, but will not have ST elevation. The pain associated with
pulmonary embolus is generally pleuritic in nature, that is, lateralizing, and
changing with inspiration. Risk factors for pulmonary embolism include
hypercoagulable states, immobilization, and vascular injury (Virchow's triad).
Question 2 of 7
What is the pathophysiologic process most likely to be responsible for this
patient's presentation?
/ A. Atherosclerotic plaque rupture resulting in thrombus formation
/ B. Buildup of atherosclerotic stenosis to produce high-grade obstruction of the
artery
/ C. Dissection of the vessel
/ D. Embolization of blood clot, air, or foreign material
/ E. Myocardial hypertrophy resulting in vessel narrowing


Explanation - Q: 1.2 Close

The correct answer is A. Acute coronary syndromes are thought to be the
result of rupture of a pre-existing atherosclerotic plaque, often one that is not
producing high-grade stenosis. When the endothelial surface covering the
lipid core of an unstable plaque ruptures, a platelet plug forms and the
clotting cascade is activated, rapidly propagating thrombus formation and
suddenly occluding the vessel. This results in transmural ischemia, which
becomes infarction, should the clot fail to recanalize quickly.
Atherosclerotic stenosis (choice B) is an important pathologic component of
coronary artery disease. However, high-grade coronary stenoses most
frequently cause stable angina, that is, chest pain and myocardial ischemia
induced when an increase in myocardial oxygen demand exceeds the fixed
supply that can be obtained through a severely stenotic vessel. As vessels
progressively narrow over time, they produce ischemia, but not sudden
infarction. Often distal to a narrowed vessel, collaterals will form from less
diseased vessels, compensating for the reduced flow.
Coronary artery dissection (choice C) is a rare phenomenon that can
produce transmural ischemia and infarction, but most commonly occurs in
the setting of instrumentation of the vessel.
Embolization (choice D) is a relatively rare phenomenon in the coronary
circulation under normal circumstances. Foreign material can only enter the
left side of the heart via right-to-left intracardiac shunt, pulmonary vein
interruption, or surgical opening of the left heart. Thrombus that forms in the
left atrium (usually the result of low-flow states such as mitral stenosis or
atrial fibrillation) can also embolize; patients with atrial fibrillation are
anticoagulated to reduce the risk of cerebral embolization of left atrial clot.
Myocardial hypertrophy (choice E) can restrict subendocardial oxygen
supply by creating high capillary pressure relative to arteriolar pressure. This
does not occur in large coronary vessels, but can produce subendocardial
ischemia in disease states that are associated with hypertrophic
myocardium, such as aortic stenosis, long-standing essential hypertension,
and idiopathic hypertrophic subaortic stenosis (IHSS).
Question 3 of 7
Which of the following vessels is most likely to be diseased in this patient?
/ A. Coronary sinus
/ B. Left anterior descending coronary artery
/ C. Left circumflex coronary artery
/ D. Left main coronary artery
/ E. Right coronary artery


Explanation - Q: 1.3 Close

The correct answer is E. Acute myocardial infarction is usually due to
obstruction of one coronary vessel. The right coronary artery exits the right
sinus of Valsalva of the aorta and gives off branches to the right ventricle, the
SA nodal artery (in 70% of patients), the AV nodal artery, and, in the 85% of
patients whose circulations are said to be "right-dominant," the posterior
descending artery, which supplies the inferior wall of the RV and the LV as
well as the posterior 1/3 of the interventricular septum. This patient has EKG
signs of an inferior myocardial infarction, with ST elevation in the inferior
leads, II, III, and aVF. He also has a physical sign of right ventricular
dysfunction (elevated jugular venous pressure).
The coronary sinus (choice A) is the principal vein draining the left ventricle
and runs alongside the circumflex artery in the posterior AV groove. It is not
a common site for atherosclerotic disease or for obstruction.
The left anterior descending artery (choice B) supplies the anterior and
anteroseptal portions of the left ventricle. Obstruction would produce ST
elevation in the anterior (V2-V6) and occasionally the lateral (I, aVF) leads of
the EKG, with possible "reciprocal" ST depression in the inferior leads.
The left circumflex artery (choice C) supplies the lateral wall of the left
ventricle. 85% of patients have a "right-dominant" coronary anatomy. That is,
the right coronary artery gives off the posterior descending artery (PDA). In
the "left-dominant" remaining 15%, the PDA comes off the circumflex.
Isolated inferior EKG lead changes are, therefore, most likely to be due to
RCA obstruction; circumflex obstruction typically produces EKG lead
changes in the lateral (I, aVL, V5, V6) leads.
The left main coronary artery (choice D) exits the aorta at the left sinus of
Valsalva and divides into the left anterior descending and left circumflex
arteries. Obstruction of the left main makes the entire left ventricle ischemic,
often resulting in cardiogenic shock. This would produce ST segment
elevation in leads I, aVL, and V2-V6.


Question 4 of 7

This patient is given aspirin in the emergency department. Decreased production
of which of the following mediators is responsible for the
beneficial effects of aspirin in this disorder?
/ A. cAMP
/ B. PIatelet glycoprotein lIB/IIIA
/ C. Prostacyclin
/ D. Thromboxane A2
/ E. Ubiquinone (coenzyme Q)

Explanation - Q: 1.4 Close

The correct answer is D. Aspirin irreversibly inhibits the enzyme
cyclooxygenase, which produces all the prostaglandin mediators from
arachidonic acid. Cyclooxygenase in platelets produces thromboxane A2,
which is a potent promoter of platelet aggregation and vasoconstriction. By
blocking this, aspirin irreversibly inhibits platelet function, preventing
aggregation at the site of plaque rupture. Platelets, having no nuclei, are
unable to synthesize more cyclooxygenase, and therefore thromboxane
production is inhibited for the life of the platelet, approximately 10 days.
Cyclic AMP (choice A) is an intracellular small molecule responsible for
multiple signal transduction pathways. In cardiac myocytes, it activates
protein kinases responsible for the phosphorylation of calcium channels,
promoting entry of calcium into the cell. cAMP is broken down by
phosphodiesterase, the inhibition of which is responsible for the beneficial
effects of inotropic phosphodiesterase inhibitors such as milrinone.
Platelet surface glycoprotein IIb/IIIA (choice B) binds fibrinogen and von
Willebrand factor, promoting aggregation and clot formation. It is inhibited by
drugs like eptifibatide and tirofiban, which are used in acute coronary
syndromes to further inhibit platelet aggregation and thrombus formation.
Prostacyclin (choice C) is produced by cyclooxygenase in endothelial cells,
where it promotes vasodilation and inhibits platelet aggregation. Prostacyclin
should therefore be a beneficial mediator. Aspirin inhibits prostacyclin
formation, however, endothelial cells can produce more cyclooxygenase and
are able to continue to synthesize adequate amounts of prostacyclin.
Ubiquinone (choice E), or coenzyme Q, is a naturally occurring coenzyme
that plays a vital role in the mitochondrial electron transport chain. Studies
have shown an association between decreased levels of coenzyme Q and
heart disease, so inhibition of ubiquinone production would not likely be
beneficial.

Question 5 of 7
EIevation of which of the following serum proteins is the most specific
biochemical marker for this patient's condition?
/ A. AIanine aminotransferase
/ B. Creatine phosphokinase
/ C. Lactate dehydrogenase
/ D. Transferrin
/ E. Troponin


Explanation - Q: 1.5 Close

The correct answer is E. Troponins (in isoforms troponin C, troponin I, and
troponin T) are required for actin-myosin cross linking in cardiac muscle.
Small elevations in serum troponin levels are currently the most sensitive
clinical serum markers for myocardial injury, elevating within 3-12 hours of
infarction. Levels remain elevated for 5-14 days.
Alanine aminotransferase (choice A) occurs in both cardiac muscle and in
the liver, and has been used in the past as a marker of cardiac injury.
However, currently, its more common clinical use is as a marker of
hepatocyte injury. Its time course of elevation in MI is intermediate between
CPK and LDH (see below).
Creatine phosphokinase (choice B) has been the mainstay of diagnosis of
myocardial injury for many years. CPK has several isoforms, of which the MB
isoform is specific for cardiac muscle. The fraction of the total CPK that is the
MB isoform has been used to differentiate myocardial injury from other injury
processes elevating CPK. CPK is also elevated with skeletal muscle and with
brain injury, but neither of these tissues contains significant amounts of MB
isoform. In MI, CPK levels usually rise within 8 to 24 hours and return to
normal after 48 to 72 hours.
Lactate dehydrogenase (choice C) is, like alanine aminotransferase, an
enzyme that is released both with cardiac injury and with hepatocellular
injury. It can also be elevated in hemolysis and with some neoplasms. In MI,
it generally rises within 12 hours and peaks after 24-48 hours, remaining
elevated for 10-14 days. These properties made LDH, prior to the
introduction of troponin assays, the test used to detect MI occurring more
than a day previously.
Transferrin (choice D) is a plasma protein responsible for the uptake of iron
after absorption in the small intestine, and is responsible for iron-binding
capacity in the blood. It is measured (usually as "total iron-binding capacity")
in the differential diagnosis of the anemias.
Question 6 of 7
Three days after hospital admission, the patient suddenly develops shortness of
breath and becomes hypotensive. His heart rate is 100/min,
with a normaI PR and QRS intervaI. His blood pressure is 75/50 mm Hg. His
respiratory rate is 38/min and his oxygen saturation on 2 Iiters
via nasal cannula drops to 60%. A chest x-ray reveals bilateral fluffy infiltrates in
the lung fields. Which of the following complications of his
condition has most likely occurred?
/ A. Dilation of the left ventricle
/ B. Dressler syndrome
/ C. Rupture of the left ventricular free wall
/ D. Rupture of the posteromedial papillary muscle
/ E. Ventricular tachycardia

Explanation - Q: 1.6 Close

The correct answer is D. This patient is suddenly in cardiogenic shock with
severe pulmonary edema. This could be the result of arrhythmia, cardiac
tamponade, or left ventricular valvular dysfunction. He had an inferior MI,
which is most likely due to thrombosis of the right coronary artery. The
posteromedial papillary muscle is supplied by the RCA alone in most patients
and is prone to rupture in inferoposterior MI. Rupture leads to acute and
severe mitral valve dysfunction, resulting in pulmonary edema and poor
forward ejection.
Dilation of the left ventricle (choice A) typically occurs after extensive
damage occurs, which would have appeared on this patient's acute EKG as
ST elevation in the anterior leads. Dilation can result in mitral regurgitation,
but typically of an insidious onset.
Dressler syndrome (choice B) is a late complication of MI that may occur
weeks to months later, characterized by symptoms of pericarditis including
pleuritic chest pain, fever, friction rub, and elevated white blood cell count.
Patients can also develop early postinfarction pericarditis in the days to
weeks following MI with friction rub and pericardial effusion. This is seldom
associated with cardiac tamponade.
Rupture of the LV free wall (choice C) is a complication more likely to occur
with more extensive damage to the LV than is produced in an inferior MI
such as this patient had. However, LV free wall rupture would produce
cardiac tamponade, which could produce this clinical picture.
Ventricular tachycardia (VT) (choice E) is a complication of MI (though risk is
highest early in the course of infarction) and could also produce this clinical
picture. However, it is excluded by the EKG, which reveals an atrial-
ventricular conducted rhythm (VT displays no P waves) and a narrow-
complex QRS (VT typically has a QRS much greater than 0.12 s).
Question 7 of 7
The patient is taken emergently to the operating room. During surgery, a sample
of affected myocardial tissue is sent to the pathology
Iaboratory for examination. Which of the following would be the likely pathologic
finding(s)?
/ A. Acellular fibrosis
/ B. Monocyte infiltration, absent nuclei and striations
/ C. Myocyte disarray
/ D. Myocyte edema, hemorrhage, and dense neutrophil infiltration
/ E. Wavy myofibers with eosinophilic contraction bands

Explanation - Q: 1.7 Close

The correct answer is B. Three days after infarction, coagulation necrosis is
complete, with complete loss of cellular structure (hence the high risk of
mechanical complications such as rupture) and infiltration of monocytes to
phagocytize debris.
Acellular fibrosis (choice A) replaces necrosis after many weeks when
debris is removed and fibroblasts have invaded the dead tissue and replaced
it with collagen.
Myocyte disarray (choice C) is associated with hypertrophic subaortic
stenosis (IHSS), rather than myocardial infarction.
Myocyte edema, with hemorrhage and neutrophil infiltration (choice D)
occurs within 4-12 hours after infarction.
Wavy myofibers and contraction bands (choice E) are the first light
microscopic pathologic changes to occur after MI, and appear within 1-3
hours after infarction.


A 52-year-old man presents to the emergency department because of severe
chest pain. The excruciating pain began abruptly, 30 minutes
previously, and feels to the patient as if something were "ripping." When asked to
point to where the pain is worst, the patient points to the
precordium. The man additionally reports that the pain seems to be changing in
position slowly.
Question 1 of 6
Which of the following is most likely causing the patient's severe pain?
/ A. Aortic dissection
/ B. Atherosclerotic aortic aneurysm
/ C. Esophageal reflux
/ D. Myocardial infarction
/ E. Peptic ulcer

Explanation - Q: 2.1 Close

The correct answer is A. This patient has a classic presentation of aortic
dissection. Any time a patient in excruciating chest pain describes the pain
as "tearing" or "ripping," you should strongly consider the diagnosis of aortic
dissection. The pain may move with time as the dissection progresses. Aortic
dissection is a highly lethal condition that may lead to aortic rupture, most
often into the pericardial cavity or left pleural space. The two most common
sites of origin of the dissection are in the proximal aorta within 5 cm of the
aortic valve and in the descending thoracic aorta just distal to the origin of
the left subclavian artery. CT scan with contrast is often used to confirm the
diagnosis suspected clinically. Therapy is promptly initiated with medications
that lower the blood pressure to try to prevent extension of the dissection.
Surgery is usually then performed in patients in which the dissection begins
in the proximal aorta near the aortic root; medical therapy alone can
sometimes be used for those with distal aortic dissection that has not
compromised blood flow to limbs or organs.
Atherosclerotic aortic aneurysm (choice B) more commonly involves the
abdominal aorta, and, when rupturing, may produce excruciating pain that is
usually referred to the lower abdomen and back.
The pain of esophageal reflux (choice C) is rarely excruciating, and usually
does not produce a ripping or tearing sensation.
Myocardial infarction (choice D) can produce severe precordial chest pain,
but the pain usually does not move with time and does not have a tearing or
ripping character.
Peptic ulcer pain (choice E) may be severe and referred to the chest, but
patients are more likely to use terms like "burning" than ripping or tearing,
and the pain does not slowly change position.
Question 2 of 6
Extension of this patient's disease process would be most likely to produce which
of the following?
/ A. Aortic insufficiency
/ B. Aortic stenosis
/ C. Mitral insufficiency
/ D. Mitral stenosis
/ E. Tricuspid stenosis

Explanation - Q: 2.2 Close

The correct answer is A. Dissecting aneurysms tend to start near the root
of the aorta, and aortic insufficiency is a common complication. This can be
very helpful in the initial evaluation of the patient, since up to 2/3 of the
patients with proximal aortic dissection demonstrate, on auscultation, the
characteristic murmur of aortic insufficiency, which is a pandiastolic
decrescendo murmur that is loudest over the sternum and left lower sternal
border. Aortic stenosis (choice B) usually does not occur.
Involvement of the mitral (choices C and D) and tricuspid valves (choice E)
would be very rare, and probably only seen if the aortic dissection interrupted
the orifices of the coronary arteries and induced a secondary myocardial
infarction.






Question 3 of 6
If enzyme chemistries were sent, which would be the most likely results?
/ A. Decreased AST, elevated CK, decreased LDH
/ B. EIevated AST, elevated CK, normal to decreased LDH
/ C. EIevated AST, normaI CK, normaI LDH
/ D. NormaI AST, elevated CK, elevated LDH
/ E. NormaI AST, normaI CK, normal to elevated LDH
Explanation - Q: 2.3 Close

The correct answer is E. Unless aortic dissection secondarily causes a
myocardial infarction secondary to occlusion of the coronary arteries,
aspartate aminotransferase (AST) and creatine kinase (CK) levels should be
normal. Lactic dehydrogenase (LDH) levels may be normal, or elevated if
some hemolysis is occurring within the area of dissection.
In general, AST can be elevated (choices B and C) in a variety of cardiac
diseases (e.g., myocardial infarction, heart failure, myocarditis, pericarditis),
muscle damage (e.g., myositis, muscular dystrophy, trauma), and damage to
liver, pancreas, kidney, or brain. AST is decreased (choice A) in pyridoxine
(vitamin B6) deficiency and in the terminal stages of liver disease. In general,
CK can be elevated (choices A, B, and D) in disease or damage involving
heart, muscle, or brain. Decreased CK has no medical significance. Lactic
dehydrogenase (LDH) can be elevated (choice D) in myocardial infarction,
pulmonary infarct, hemolytic and pernicious anemia, hematologic
malignancies, and disease of liver, kidney, or brain. Decreases in LDH
(choices A and B) are not medically significant.

Question 4 of 6
If surgery is necessary to repair this problem, the surgeon will be required to
understand the anatomic relationship of the aorta to the
surrounding structures. Which of the following most accurately describes the
descending portion of the thoracic aorta?
/ A. It descends on the right side of the thoracic vertebrae
/ B. It flattens the posterior aspect of the trachea
/ C. It is to the left of the esophagus at the hiatus
/ D. It is to the left of the thoracic duct at the T10 Ievel
/ E. It is to the right of the inferior vena cava

Explanation - Q: 2.4 Close

The correct answer is D. The thoracic duct is the main lymphatic duct and it
lies on the bodies of the inferior seven thoracic vertebrae. It conveys most of
the lymph of the body to the venous system. It passes superiorly from the
cisterna chyli (the expanded inferior end of the thoracic duct) through the
aortic hiatus in the diaphragm. The thoracic duct ascends in the posterior
mediastinum, on the right of the thoracic aorta and to the left of the azygos
vein. At the level of T4, T5, or T6, the thoracic duct crosses to the left,
posterior to the esophagus and ascends to the superior mediastinum. The
thoracic duct empties into the venous system near the union of the left
internal jugular and subclavian veins.
As a continuation of the aortic arch, the descending aorta begins on the left
side of the inferior border of the body of the T4 vertebra and descends in the
posterior mediastinum on the left sides of T5 to T12 (choice A).
The trachea travels in the superior mediastinum and does not have direct
contact with the descending thoracic aorta. The trachea is kept patent by a
series of C-shaped tracheal cartilages. The posterior aspect is flat where it is
applied to the esophagus, not the aorta (choice B).
At the level of the esophageal hiatus (choice C), the esophagus lies anterior
to the descending thoracic aorta.
The inferior vena cava (choice E) is located to the right of the abdominal
aorta, not the thoracic aorta. The IVC returns blood from the lower limbs,
most of the abdominal wall, and the abdominopelvic viscera. This vessel
begins anterior to L5 vertebra by union of the common iliac veins. It then
ascends on the right psoas major muscle to the right of the median plane
and aorta. It passes through the vena caval foramen in the diaphragm at the
level of T8 to enter the right atrium.

Question 5 of 6
Which of the following would be most likely to be seen on pathological
examination of a specimen removed from this patient at surgery?
/ A. Bacterial vegetations
/ B. Cystic medial degeneration
/ C. Multiple small granulomas
/ D. Parasitic organisms
/ E. Polyarteritis nodosa

Explanation - Q: 2.5 Close

The correct answer is B. Cystic medial degeneration is a disruption and
fragmentation of the elastic tissue in aortic media, with formation of areas
devoid of elastin. These changes weaken the aortic wall, predispose for
dissection, and are seen in the majority of cases of aortic dissection.
Bacterial vegetations (choice A) are a feature of endocarditis.
Multiple small granulomas (choice C) can be seen in temporal arteritis and
Takayasu arteritis.
Parasitic organisms (choice D) do not usually affect the aorta; the organisms
of trichinosis and Chagas disease can affect the heart.
Polyarteritis nodosa (choice E) is a focal inflammation that usually involves
smaller blood vessels than the aorta.
Question 6 of 6
Which of the following conditions has been associated with this patient's
disease?
/ A. Cushing syndrome
/ B. Dandy-Walker syndrome
/ C. Kawasaki syndrome
/ D. Marfan syndrome
/ E. Tourette syndrome

Explanation - Q: 2.6 Close

The correct answer is D. Marfan syndrome is an autosomal dominant
connective tissue disease characterized by skeletal changes (e.g., tall
stature, long limbs, long fingers, lax joints), a tendency to develop
dislocations of the lens of the eye, and a tendency to develop aortic
dissection secondary to prominent cystic medial degeneration in the aortic
media. A similar condition, Ehlers-Danlos syndrome, also predisposes for
dissecting aneurysm. Other predisposing factors include congenital
cardiovascular abnormalities (e.g., coarctation of the aorta, patent ductus
arteriosus, bicuspid aortic valve) that increase the turbulence of blood flow in
the aorta, atherosclerosis, and trauma (including iatrogenic trauma during
arterial catheterization and cardiovascular surgical procedures). The other
conditions listed in the choices are unrelated to aortic dissection.
Cushing syndrome (choice A) is a characteristic pattern of physical changes
(truncal obesity, moon face, buffalo hump), biochemical/hormonal changes
(hypertension, altered carbohydrate and protein metabolism, amenorrhea),
and sometimes psychiatric disturbances that are seen in patients with
increased levels of adrenocortical hormones.
Dandy-Walker syndrome (choice B) is a congenital abnormality of the
cerebellum and fourth ventricle.
Kawasaki syndrome (choice C) is a febrile childhood disease that
predisposes for the formation of tiny aneurysms of the coronary arteries.
Tourette syndrome (choice E) is a motor and vocal tic disorder.




A 35-year-old man with no significant past medical history presents to clinic with
a six week history of worsening chest discomfort and pain.
He describes the pain as a substernal burning sensation that occasionally wakes
him up at night and is often worse after he eats. He
sometimes notices a sour taste in his mouth when he wakes up in the morning.
He has no dysphagia or odynophagia. The pain is unrelated
to exertion, and he jogs 3 miles every other day without difficulty or chest pain.
His vital signs and physical examination are normaI.
Question 1 of 6
Which of the following is the most likely diagnosis?
/ A. Acute viral pericarditis
/ B. Aortic dissection
/ C. Candida esophagitis
/ D. Gastroesophageal reflux disease (GERD)
/ E. Stable angina

Explanation - Q: 3.1 Close

The correct answer is D. A patient who presents with chest discomfort that
is burning in nature, and worsened after eating without symptoms of
dysphagia or odynophagia, most likely has gastroesophageal reflux disease
(GERD). GERD occurs when there is reflux of gastric contents into the
esophagus. This may occur with or without inflammation. It is often caused
by inappropriate relaxation of the lower esophageal sphincter. Certain foods
such as peppermint, caffeine, and high-fat and spicy foods are often
associated with GERD.
Acute viral pericarditis (choice A) would present with more severe and
sudden onset of chest pain that is relieved with leaning forward or sitting up.
Acute viral pericarditis is often associated with a prodrome and usually
presents with a fever. Occasionally, a pericardial friction rub can be heard on
exam.
Aortic dissection (choice B) would also present as sudden onset of severe
chest pain, which often radiates to the back. Patients can have hypotension,
depending on the severity of the dissection. Patients can also have unequal
pulses in their extremities if the dissection affects one of the major arteries
branching off the aortic arch.
Candida esophagitis (choice C) would present with dysphagia and
odynophagia. Patients also have oral thrush, and generally are
immunocompromised. These patients usually have a fever.
Stable angina (choice E) should present with typical chest pain that is
worsened after exertion. The fact that this patient can jog 3 miles without
difficulty goes against stable angina. Furthermore, he is young and does not
have any risk factors for cardiac disease such as hypertension, diabetes, or
hypercholesterolemia.
Question 2 of 6
Which of the following tests would be most likely to confirm the probable
diagnosis?
/ A. 24-hour ambulatory esophageal luminal pH monitoring
/ B. Cardiac angiogram
/ C. Chest radiograph
/ D. Exercise treadmill test
/ E. Serologic blood tests for H. Pylori infection

Explanation - Q: 3.2 Close

The correct answer is A. Twenty-four hour ambulatory esophageal luminal
pH monitoring is one of the most sensitive tests for GERD. In most cases,
the disease is diagnosed clinically by history, but pH monitoring would help
confirm the diagnosis.
Cardiac angiograms (choice B) are used to evaluate the coronary arteries
for signs of blockage, to evaluate heart function, or to evaluate cardiac valve
function.
A chest radiograph (choice C) can be used to evaluate the structures in the
thorax, but will not help confirm the diagnosis because GERD patients
generally have normal chest radiographs.
An exercise treadmill test (choice D) is used to evaluate patients who are
believed to have underlying coronary heart disease or to rule out heart
disease.
Serologic blood testing for H. pylori infection (choice E) only documents the
presence of a current infection or the history of an H. pylori infection. A past
or present infection does not confirm a diagnosis, because GERD can occur
in the setting with or without H. pylori. Furthermore, the role of H. pylori in
GERD is still unclear.
Question 3 of 6
The patient is treated with cimetidine, which completely relieves his symptoms.
Which of the following is the mechanism of action of this
medication?
/ A. Beta-1 adrenergic blockade
/ B. Histamine H2 receptor blockade
/ C. Inhibition of cell wall synthesis
/ D. Inhibition of cyclooxygenase
/ E. Smooth muscle relaxation
Explanation - Q: 3.3 Close

The correct answer is B. Cimetidine and other histamine H2 receptor
blockers such as ranitidine block the action of histamine on H2 receptors,
resulting in a decrease in gastric acid production, thus decreasing the
symptoms of GERD.
Beta-1 adrenergic blockade (choice A) (e.g., atenolol, metoprolol) is used to
lower blood pressure, which is not related to GERD.
Inhibition of cell wall synthesis (choice C) (e.g., amoxicillin) is a mechanism
that is used by many antibiotics. GERD can be associated with the presence
of H. pylori, but the treatment of H. pylori with antibiotics in GERD patients
remains controversial, and its benefit remains questionable.
Inhibition of cyclooxygenase (choice D) (e.g., ibuprofen, naproxen) does not
play a role in the treatment of GERD. GERD may or may not be associated
with inflammation of the esophagus, but anti-inflammatory agents may
actually worsen symptoms.
Smooth muscle relaxation (choice E) (e.g., nitroglycerin) does play a role in
the relief of esophageal spasm, but this patient does not complain of
dysphagia or odynophagia.
Question 4 of 6

The physician cautions the patient about cimetidine because of which of the
following potential side effects?
/ A. CNS depression
/ B. Hypertensive crisis
/ C. Inhibition of hepatic metabolism
/ D. Masking symptoms of hypoglycemia
/ E. Ototoxicity

Explanation - Q: 3.4 Close

The correct answer is C. Many drugs can lead to clinically significant drug
interactions via inhibition of the hepatic drug-metabolizing enzymes,
particularly the cytochrome P450 isozymes. This can lead to unwanted
elevations of plasma drug levels. Cimetidine is a classic example of one of
these drugs. Other examples include erythromycin, ketoconazole,
sulfonamides, quinidine, and disulfiram.
Benzodiazepines and barbiturates are examples of drugs that can cause
CNS depression (choice A).
MAO inhibitors prior to the ingestion of tyramine-containing foods can cause
a hypertensive crisis (choice B).
Beta blockers can mask the symptoms of hypoglycemia (choice D).
Aminoglycosides can produce ototoxicity (choice E). The risk of ototoxicity
may be further increased if the patient is also taking loop diuretics.

Question 5 of 6

The patient's symptoms are initially controlled on cimetidine. After 10 years, he
develops refractory symptoms, and the physician places him
on a proton pump inhibitor. Which of the following medications was most likely
prescribed?
/ A. Lansoprazole
/ B. Loperamide
/ C. Metoclopramide
/ D. Ondansetron
/ E. Ranitidine

Explanation - Q: 3.5 Close

The correct answer is A. Lansoprazole is a proton pump inhibitor and acts
directly to inhibit the gastric parietal cell hydrogen-potassium ATPase. It can
be used as the initial treatment for GERD, or for refractory cases.
Loperamide (choice B) is an anti-diarrheal agent, which inhibits peristalsis.
Using it in this setting may worsen the symptoms of GERD.
Metoclopramide (choice C) stimulates upper gastrointestinal motility. It can
be used in refractory cases of GERD, but it is not a proton pump inhibitor.
Ondansetron (choice D) is an antiemetic and acts by selectively
antagonizing serotonin 5-HT
3
receptors. It is primarily used is severe cases
of nausea, such as in patients receiving chemotherapy for cancer treatment.
Ranitidine (choice E) is also an histamine H2 receptor blocker, like
cimetidine. Some patients who do not respond to one histamine H2 receptor
blocker, may respond to another, but ranitidine blocks the action of histamine
on H2 receptors, resulting in a decrease in gastric acid production. It is not a
proton pump inhibitor.

Question 6 of 6
Histologic examination of the affected tissue shows Barrett's esophagus. This is
most correctly described as which of the following?
/ A. Adenocarcinoma
/ B. Esophageal stricture
/ C. H.Pylori infection
/ D. Localized outpouching of the esophageal wall
/ E. Metaplasia of the squamous epithelium

Explanation - Q: 3.6 Close

The correct answer is E. Patients who have long-standing GERD are at risk
for development of Barrett's esophagus, which is the replacement of the
normal esophageal squamous epithelium with columnar epithelium
(metaplasia). This is a premalignant lesion that needs to be monitored
regularly for the development of adenocarcinoma.
Adenocarcinoma (choice A) is a malignant lesion that can result from
cellular metaplasia, but Barrett's esophagus is the premalignant lesion that
occurs before the development of adenocarcinoma of the esophagus.
Esophageal strictures (choice B) can occur in patients with long-standing
GERD, but the presence of a stricture does not mean that there is cellular
dysplasia, or Barrett's esophagus.
H. pylori infection (choice C) can occur in the setting of GERD, but it is not
synonymous with Barrett's esophagus.
An esophageal diverticulum is a localized outpouching of the esophageal
wall (choice D). This is unrelated to Barrett's esophagus.


A 45-year-old man presents with a 3-day history of persistent, severe chest pain.
Prior to this, he had flu-Iike symptoms for 2 weeks, including
fever, cough, myalgias, and arthralgias. His pain is worse when he takes a deep
breath and is improved when he sits up. On physical
examination, he is febrile, and his pulse is 110/min. His oxygen saturation is
normaI, and his breath sounds are equal and clear to
auscultation over all lung fields. There is a scratching and scraping, high-pitched
sound on auscultation of the heart over the left third
intercostal space, which is increased when the patient is sitting forward.







Question 1 of 4

Which of the following is the most likely diagnosis?
/ A. Acute pericarditis
/ B. Aortic dissection
/ C. Pneumonia
/ D. Pulmonary embolus
/ E. Tension pneumothorax

Explanation - Q: 4.1 Close

The correct answer is A. This patient has symptoms that are typical of
inflammation of the pericardial sac. In addition, the sound that is heard over
his heart is a pericardial friction rub. Acute pericarditis is often associated
with viral syndromes, connective tissue diseases, renal failure, myocardial
infarction, and tumor invasion of the pericardium.
Aortic dissection (choice B) will also present with severe chest pain, but it is
tearing in quality, and is not positional, nor pleuritic in nature.
Pneumonia (choice C) may present similarly, but auscultation of the lung
fields should reveal abnormal breath sounds, and a pericardial friction rub
should not be heard.
Pulmonary embolus (choice D) can present with chest pain that is worse
with deep breaths. However, it is not positional in nature, and it is not
associated with a pericardial friction rub. Depending on the size of the
embolus, the oxygen saturation may be abnormal.
Tension pneumothorax (choice E) is not associated with flu-like symptoms
nor a pericardial friction rub. Furthermore, there should be an absence of
breath sounds over the affected part of the lung.
Question 2 of 4
Which of the following would help confirm the diagnosis?
/ A. Angiogram showing a clot in one of the coronary arteries
/ B. Chest radiograph showing multiple emphysematous bullae
/ C. CT scan of the chest showing a widened mediastinum
/ D. EIectrocardiogram showing diffuse ST elevation
/ E. Endoscopy revealing esophageal varices


Explanation - Q: 4.2 Close

The correct answer is D. Acute pericarditis often presents with diffuse ST
elevation on an electrocardiogram.
An angiogram showing a clot in one of the coronary arteries (choice A)
would be seen in a patient having an acute myocardial infarction.
A chest radiograph showing multiple emphysematous bullae (choice B)
would be seen in a chronic smoker, and would be a possible cause of
spontaneous pneumothorax.
CT scan of the chest showing a widened mediastinum (choice C) would be
seen in a patient with an acute aortic dissection.
Endoscopy revealing esophageal varices (choice E) is seen in patients with
chronic liver disease or portal hypertension.

Question 3 of 4
Other than an antecedent viral syndrome, which of the following conditions can
predispose a patient to this problem?
/ A. AIcohol abuse
/ B. Liver failure
/ C. Peptic ulcer disease
/ D. Recent total hip replacement
/ E. Renal failure

Explanation - Q: 4.3 Close

The correct answer is E. Patients with renal failure or uremia can often
present with a fibrinous or serofibrinous pericarditis.
Alcohol abuse (choice A) and liver failure (choice B) do not necessarily
predispose a patient to pericarditis, nor are they associated with pericarditis.
Peptic ulcer disease (choice C) may cause epigastric pain that can be
confused with chest pain, and patients may have recurrent bleeding, but this
disorder is not associated with pericarditis.
A recent total hip replacement (choice D) predisposes patients to the
development of a deep venous thrombosis, which can embolize and cause a
pulmonary embolus. This is not associated with pericarditis.

Question 4 of 4
The patient is treated with a nonsteroidal anti-inflammatory agent. Which of the
following was prescribed?
/ A. AIIopurinol
/ B. Gemfibrozil
/ C. Indomethacin
/ D. Labetalol
/ E. Methocarbamol

Explanation - Q: 4.4 Close

The correct answer is C. Indomethacin or any of the other nonsteroidal
anti-inflammatory agents can be used in the treatment of acute pericarditis.
Allopurinol (choice A) is used for the treatment of gout. It inhibits xanthine
oxidase, which decreases the production of uric acid.
Gemfibrozil (choice B) is a lipid lowering agent used in patients with
hypercholesterolemia.
Labetalol (choice D) is a beta blocker and is used in hypertensive
emergencies.
Methocarbamol (choice E) is a muscle relaxant, which helps to relieve pain
associated with muscle spasms.


A 62-year-old white man is brought to the emergency department after
experiencing twenty minutes of sudden onset, crushing, substernal
chest pain radiating to his neck and left shoulder. The pain started when the
patient was lifting a heavy trolley. He denies shortness of breath,
back pain, or loss of consciousness. In the emergency department, he is given
sublingual nitroglycerin tablets, which relieve the pain almost
immediately. The patient has a history of hypertension, drinks beer several times
a week, smokes one pack of cigarettes per day, and does
not exercise. He is not on any medications. On physical examination, he is a
diaphoretic patient with a temperature of 36.6 C (98 F), blood
pressure of 150/80 mm Hg, pulse of 84/minute, and respirations of 16/minute.
His heart and breath sounds are normaI, his abdomen is soft
and non-tender, and no bruits were heard. A chest radiograph is normaI. An
electrocardiogram confirms T-wave inversion in leads V2 and
V3, with an otherwise normal sinus rhythm. The patient is started on atenoloI, a
nitroglycerin paste patch, and aspirin. He is admitted into the
hospitaI, where he remains pain free. Troponin levels taken every eight hours for
the next 24 hours are all within normal limits.

Question 1 of 5

Which of the following is the most likely diagnosis?
/ A. Angina
/ B. Gastroesophageal reflux disease
/ C. Myocardial infarction
/ D. Pulmonary embolism
/ E. Thoracic aortic aneurysm rupture

Explanation - Q: 5.1 Close

The correct answer is A. The patient has four independent cardiac risk
factors (male sex, smoker, hypertensive, inactivity). He presented with typical
cardiac chest pain on exertion, accompanied by diaphoresis, which was
relieved with nitroglycerin. This is typical of angina. T-wave inversion is also
consistent with ischemia. However, this is unlikely to be a myocardial
infarction (choice C) because there were no raised ST segments on the
electrocardiogram and his cardiac enzymes (e.g., troponin) were not
elevated. He would probably require a cardiac stress test to see if the EKG
changes could be reproduced in a controlled stressful environment (e.g.,
treadmill). On this visit, his symptoms are better described as being anginal
with ischemic EKG findings on the anterior cardiac wall.
Gastroesophageal reflux disease (choice B) would present with a
longitudinal burning sensation in the epigastric area, usually after the
consumption of large meals, caffeinated products, exercising, or lying flat
after eating. It is treated with H2-blockers and proton pump inhibitors, along
with eating smaller meals and the avoidance of caffeine. It is an important
part of the differential diagnosis of chest pain.
The patient has risks for thromboembolism, including smoking and inactivity,
which could lead to pulmonary embolism. Pulmonary embolism (choice D)
would be suggested if the patient had a sudden onset of shortness of breath
and chest pain. In addition, on examination, one would look for tachycardia,
tachypnea, and diaphoresis. The work-up includes D-dimers, a lower leg
Doppler, prothrombin time (PT), partial thromboplastin time (PTT), and a
ventilation/perfusion scan. Treatment would be anticoagulation with heparin,
and then eventually with Coumadin.
Rupture of a thoracic aortic aneurysm (choice E) would present as severe
chest pain radiating to the back. The patient would typically be elderly. The
blood pressure may drop and there will sometimes be a discrepancy in arm-
to-arm blood pressure readings.
Question 2 of 5
Which of the following vessels is most likely affected in this patient?
/ A. Circumflex coronary artery
/ B. Left anterior descending artery
/ C. Marginal artery
/ D. Posterior descending artery
/ E. Right coronary artery

Explanation - Q: 5.2 Close

The correct answer is B. The electrocardiogram revealed inverted T-waves
(suggestive of ischemia) in leads V2 and V3. These leads correspond mainly
to the anterior wall of the left ventricle, where the ischemia is likely to have
taken place. The left anterior descending artery is a terminal branch of the
left coronary artery (the other branch being the circumflex artery). It passes
posterior to the pulmonary artery, then anteriorly between this vessel and the
left auricle to reach the anterior longitudinal sulcus. Since it supplies blood to
the anterior walls of both ventricles, ischemia in this location would manifest
as T-wave inversion in leads V1 to V4.
The circumflex artery (choice A) supplies the lateral wall of the heart (atrium
and ventricle). Ischemia with this artery would manifest as T-wave inversion
of leads I, aVL, and leads V5 and V6. The circumflex artery branches off the
left coronary artery and follows the left part of the coronary sulcus giving
branches to the left atrium and ventricle.
The marginal artery (choice C) is a branch of the right coronary artery, and it
supplies branches to both surfaces of the right ventricle. Disease in this
vessel is not associated with specific EKG changes.
Inverted T-waves in leads II, III, and aVF would correspond mainly to
ischemia of the inferior wall of the left ventricle. The main artery supply to this
area is the right coronary artery (choice E). It arises from the anterior aortic
sinus, then passes between the conus arteriosus and the right auricle, and
then runs in the right portion of the coronary sulcus. From there it continues
on the diaphragmatic surface of the heart from the right to left, as far as the
posterior longitudinal sulcus. It eventually arrives at the apex of the heart in
the form of the posterior descending artery (choice D).



Question 3 of 5

Which of the following is most likely responsible for this patient's symptoms?
/ A. Atheromatous plaque rupture
/ B. Hyaline arteriosclerosis
/ C. Hyperplastic arteriosclerosis
/ D. Thrombus embolization
/ E. Vasospasm of the coronary vessels.

Explanation - Q: 5.3 Close

The correct answer is A. Several hypotheses have been formulated about
atheromatous plaque formation. One hypothesis involves endothelial wall
damage, resulting in monocyte and platelet adhesion and migration of
monocytes into the intima from the lumen and smooth muscle media.
Another hypothesis involves hyperlipidemia causing an increased rate of LDL
(low density lipoprotein) penetration into the artery wall, causing endothelial
injury and the promotion of foam cells. Once the atheromatous plaque is
formed, any rupture (e.g., with vasospasm) will expose subendothelial
collagen, promoting platelet adherence, and the eventual aggregation of
platelets. This may cause a total occlusion of the coronary vessel (as in the
case of an infarction) or a transient subocclusion (that leads to angina due to
ischemia).
Arteriosclerosis typically produces less acute effects than the occlusion seen
with atheromatous plaque formation. Hyaline arteriosclerosis (choice B) is
microangiopathy seen in hypertensive and diabetic patients. It is due to
leakage of plasma components across the vascular endothelium and
increasing extracellular matrix production by smooth muscle walls. This
eventually narrows the arteriolar lumina causing reduced blood flow for that
organ e.g., the kidney.
Hyperplastic arteriosclerosis (choice C) is usually seen in "malignant" or
severe hypertension and is characterized by laminated thickenings of the
wall of the lumina. The most commonly affected sites are the kidney,
gallbladder, pancreas, and intestines.
An embolus is a detached intravascular solid, liquid, or gaseous mass
carried by the blood to a site distal to the point of origin. Thrombus
embolization (choice D) represents 99% of all emboli. An example is a
pulmonary (thrombo)embolism. Although atherosclerosis may provide a site
for thrombus formation and embolization, this is not the most common
mechanism for angina.
Vasospasm (choice E) may cause angina more rarely; this is called
Prinzmetal's angina. There may be essentially no major atheromatous
changes in the coronary vessels. Prinzmetal's angina is often treated with
calcium channel blockers.

Question 4 of 5
The cardiac enzyme marker troponin is normally found predominantly in which of
the following sites?
/ A. Cell membranes of myocardial cells
/ B. Endoplasmic reticulum
/ C. Mitochondria
/ D. Myofibrils
/ E. Nuclei
/ F. Ribosomes

Explanation - Q: 5.4 Close

The correct answer is D. Troponins are tightly bound structural proteins that
regulate the calcium-mediated interaction of actin and myosin in striated
muscle (myofibrils). Troponin release would indicate cell death, such as that
seen in infarction. The rise of this marker after cardiac injury parallels that of
creatine kinase (CK) (another cardiac enzyme marker). However, in contrast
to CK, baseline levels of troponin are undetectable in normal volunteers.
Neither troponin nor creatine kinase are found in cell membranes (choice A)
of the myocardium.
Creatine kinase is a cardiac marker that resides in the cytoplasm and
mitochondria (choice C). During cardiac injury, destruction of the cell walls
and sarcolemma of the myocardial cells causes rapid release of the CK into
the bloodstream. It is also released in noninfarction cardiac injuries (e.g.,
blunt chest trauma).
The nucleus (choice E), ribosomes (choice F), and endoplasmic reticulum
(choice B) are sites of transcription, translation and post-translational
modification of troponin. Troponin is then incorporated into the myofibril
structure via covalent attachment.

Question 5 of 5

Which of the following is the mechanism of action of nitroglycerin?
/ A. Increases stroke-work index of the heart
/ B. Maintains coronary perfusion despite fluctuations in blood pressure
/ C. Relaxes bronchial smooth muscle
/ D. Relaxes vascular smooth muscle
/ E. Unblocks coronary vessels by fibrinolysis

Explanation - Q: 5.5 Close

The correct answer is D. Relaxation of vascular smooth muscle is the
principal pharmacologic action of nitroglycerin. Nitroglycerin (glyceryl
trinitrate) is a therapeutically active member of the nitrate group of drugs.
Nitrates improve oxygen supply to the myocardium (dilation of the coronary
artery, reduction of the end-diastolic pressure in the left ventricle) and
reduces myocardial oxygen requirements (dilation of the peripheral arteries,
lowering of the systemic blood pressure). Reduction of venous return is a
particular benefit (reduction of preload) in the event of heart failure. In
smooth muscle cells, nitrates increase the intracellular concentration of nitric
oxide (NO). NO stimulates guanylate cyclase, forming cGMP, which probably
dephosphorylates the phosphorylated form of myosin light chain, leading to
smooth muscle relaxation.
Myocardial oxygen consumption or demand (as measured by the pressure-
rate product, tension-time index, and stroke-work index, choice A) is
decreased by both the arterial and venous effects of nitroglycerin, and a
more favorable supply-demand ratio can be achieved. Therapeutic doses of
intravenous nitroglycerin reduce systolic, diastolic, and mean arterial blood
pressures.
Effective coronary perfusion pressure (choice B) is usually maintained, but
can be compromised if blood pressure falls excessively or increased heart
rate decreases diastolic filling time.
The bronchial smooth muscle is unaffected by nitrates. Bronchodilation
(choice C) can be achieved by stimulation of beta-2 adrenergic receptors.
Inhalation of beta-agonists, such as albuterol, can help reduce wheezing
during asthmatic episodes.
Reperfusion of coronary vessels through fibrinolysis (choice E) is the action
of fibrinolytic agents like alteplase or streptokinase. They are used primarily
in myocardial infarctions. If the agent is administered done quickly enough,
the infarct size is reduced and left ventricular function is preserved, thereby
reducing mortality.


A 19-year-old college student is referred to a family medicine physician for an
abnormal heart sound heard on a routine school physical
examination. On questioning, he denies chest pain or shortness of breath, but
says he does occasionally experience palpitations. He is
adopted and does not know his family history. On physical examination, he is
201 cm (79 inches) talI, weighs 73 kg (160 Ib), and has long
arms and legs. On examination, there is a mid-systolic click, heard best at the
apex. The click also occurs earlier when the patient stands or
performs a Valsalva maneuver.
Question 1 of 6
Which of the following cardiac abnormalities does this patient most likely have?
/ A. Aortic regurgitation
/ B. Aortic stenosis
/ C. Mitral stenosis
/ D. Mitral valve prolapse
/ E. Tricuspid regurgitation

Explanation - Q: 6.1 Close

The correct answer is D. Mitral valve prolapse is a common valvular
abnormality that can be benign and asymptomatic, but it can also cause
palpitations, chest pain, and shortness of breath. It is due to stretching of the
posterior mitral valve leaflets, resulting in prolapse of the valve. This change
produces the classic midsystolic click, that is sometimes is followed by a
murmur.
Aortic regurgitation (choice A) would result in a blowing, diastolic murmur
heard best at the left second intercostal space.
Aortic stenosis (choice B) would result in a systolic ejection murmur heard
best at the right second intercostal space.
Mitral stenosis (choice C) produces a low-pitched diastolic murmur heard
best over the apex. It is often preceded by an opening snap.
Tricuspid regurgitation (choice E) produces a blowing systolic murmur heard
best at the left lower sternal border.
Question 2 of 6
An echocardiogram is performed. Which of the following would likely be observed
during the study?
/ A. Ballooning of the aortic valve into the ventricle during diastole
/ B. Ballooning of the mitral valve into the atrium during diastole
/ C. Rupture of the aortic valve
/ D. Rupture of the tricuspid valve
/ E. Stenotic mitral valve

Explanation - Q: 6.2 Close

The correct answer is B. Ballooning of the mitral valve into the atrium
during diastole is diagnostic of mitral valve prolapse.
Ballooning of the aortic valve into the ventricle during diastole (choice A)
does not occur in mitral valve prolapse. In addition, this would not result in a
mid-systolic click.
Rupture of the aortic valve (choice C) could result in severe hemodynamic
compromise, especially if it were acute, due to a large regurgitant flow of
blood. A harsh diastolic murmur would be heard on examination.
Rupture of the tricuspid valve (choice D) would cause a blowing systolic
murmur heard over the left lower sternal border.
A stenotic mitral valve (choice E) is not associated with mitral valve
prolapse. In mitral stenosis, the valve is thick and stiff. There is a low-pitched
diastolic murmur, which is often preceded by an opening snap.

Question 3 of 6
Which of the following genetic disorders can be associated with this abnormality?
/ A. KIinefelter syndrome
/ B. Marfan syndrome
/ C. Osler-Weber-Rendu syndrome
/ D. Tay-Sachs disease
/ E. Wilson disease

Explanation - Q: 6.3 Close

The correct answer is B. Marfan syndrome is a connective tissue defect
due to a deficiency in fibrillin. This results in defects in skeletal, visual, and
cardiovascular structures, such as mitral valve prolapse and/or aortic
aneurysms.
Klinefelter syndrome (choice A) is chromosomal disorder, most often
characterized by the karyotype 47,XXY. It results in hypogonadism, and in
some cases, mild mental retardation.
Osler-Weber-Rendu syndrome (choice C) is also known as hereditary
hemorrhagic telangiectasia. It is characterized by telangiectasias of the skin
and mucous membranes and hemorrhage at these sites.
Tay-Sachs disease (choice D) is a lysosomal storage disease caused by a
deficiency of hexosaminidase A. It results in central nervous system
degeneration, mental retardation, motor deterioration, and blindness.
Wilson disease (choice E) is a disorder of copper metabolism. It is
characterized by decreased serum ceruloplasmin and results in
accumulation of copper in the liver, kidney, brain, and cornea.

Question 4 of 6
This patient is diagnosed with a genetic disorder associated with the valvular
abnormality. Which of the following modes of inheritance does
this disease follow?
/ A. Autosomal dominant inheritance
/ B. Autosomal recessive inheritance
/ C. Mitochondrial inheritance
/ D. X-Iinked dominant inheritance
/ E. X-Iinked recessive inheritance

Explanation - Q: 6.4 Close

The correct answer is A. Marfan syndrome is inherited in an autosomal
dominant manner. Usually, in diseases with this pattern of inheritance, one
heterozygous parent carries a gene with the phenotypic expression of the
disorder, and the other parent is normal.
Autosomal recessive inheritance (choice B) usually occurs when both
parents are heterozygotes and do not phenotypically manifest the disorder.
Cystic fibrosis is an example of an autosomal recessive disease.
Mitochondrial inheritance (choice C) is due to the maternal transmission of
mitochondrial genes. Mitochondrial myopathies are inherited in this pattern.
X-linked dominant inheritance (choice D) is a rarity. The heterozygous
female and the hemizygous male will manifest the phenotype.
X-linked recessive inheritance (choice E) is most often the result of a
heterozygous female parent and a genotypically and phenotypically normal
male parent. Male children who inherit the affected X chromosome will
manifest the disorder, but the female children will only be carriers.
Question 5 of 6
Over the next year the patient develops chest pain, shortness of breath, and
progressive fatigue. On auscultation, he has a midsystolic click,
which is now followed by a high-pitched, blowing systolic murmur. Which of the
following is the most likely cause of this new development?
/ A. Aortic aneurysm
/ B. Aortic stenosis
/ C. Mitral regurgitation
/ D. Mitral stenosis
/ E. Myocardial infarction

Explanation - Q: 6.5 Close

The correct answer is C. In severe cases of mitral valve prolapse, patients
can develop mitral regurgitation, which may require surgical treatment if the
symptoms are severe.
Aortic aneurysm (choice A) can occur in Marfan disease, but it is not
associated with mitral prolapse or a systolic murmur.
Aortic stenosis (choice B) causes a systolic ejection murmur heard best at
the right second intercostal space, which often radiates to the carotid
arteries. It is not associated with extra heart sounds.
Mitral stenosis (choice D) is not a complication of mitral valve prolapse.
Furthermore, it causes a diastolic murmur, which can be preceded by an
opening snap.
Myocardial infarction (choice E) is not associated with mitral valve prolapse.
Question 6 of 6
The patient is treated with propanoloI. What is the mechanism of action of this
medication?
/ A. AIpha-1 selective adrenergic blockade
/ B. Beta-1 selective adrenergic blockade
/ C. Calcium channel blockade
/ D. Nonselective beta adrenergic blockade
/ E. Nonselective alpha adrenergic blockade

Explanation - Q: 6.6 Close

The correct answer is D. Propanolol, as well as timolol, pindolol, nadolol,
and labetalol, all exhibit nonselective beta-adrenergic blockade. They are
generally used in the treatment of hypertension, angina, and arrhythmias.
Alpha-1 selective adrenergic blockade (choice A) is a mechanism used by
prazosin, terazosin, and doxazosin. These agents can be used to treat
hypertension or urinary retention due to benign prostatic hypertrophy.
Beta-1 selective adrenergic blockade (choice B) is a mechanism used by
metoprolol, atenolol, and esmolol. Unlike the nonselective beta-adrenergic
blockers, these medications selectively block beta 1 receptors over beta 2
receptors.
Calcium channel blockade (choice C) is a mechanism used by nifedipine,
diltiazem, and verapamil. They can also be used to treat hypertension and
angina. Verapamil and diltiazem can also be effective in the treatment of
arrhythmias.
Nonselective alpha-adrenergic blockade (choice E) is a mechanism used by
phenoxybenzamine and phentolamine. They can be used in the treatment of
pheochromocytoma.


A 21-year-old woman presents to an urgent care clinic with complaints of sharp,
knife-Iike chest pain and shortness of breath for the past 4
days. She states that the pain is worse with inspiration, coughing, and
movement. However, it is not related to position. She also complains
of fever, chills, and a dry cough. She has a history of some type of rheumatologic
disease, but she does not know which one. She was
previously seen by a rheumatologist, but was lost to follow up, and stopped
taking her medications 6 months ago. Her physical examination is
notable for a temperature of 37.8 C (100 F), tachycardia, decreased breath
sounds at both lung bases, and a friction rub. There is no pinpoint
chest wall tenderness.
Question 1 of 6
Which of the following is the most likely cause of this patient's chest pain?
/ A. Angina pectoris
/ B. Anxiety
/ C. Esophageal spasm
/ D. PIeuritis
/ E. Rib fracture

Explanation - Q: 7.1 Close

The correct answer is D. Pleuritis is inflammation of the parietal pleura of
the lung, which can often result in a friction rub heard on auscultation.
Patients can also have pleural effusions. Pleuritis is associated with many
different disease processes, including collagen vascular diseases and
infections.
Angina pectoris (choice A) is chest pain related to underlying coronary artery
disease. The pain is usually substernal, worse with exertion, and relieved by
rest. It is not associated with fever or chills, nor is it pleuritic in nature.
Anxiety (choice B) can often be a cause of chest pain. It is a clinical
diagnosis and should be associated with other symptoms of anxiety. It is not
pleuritic in nature, and is not associated with a cough, fever, or chills.
Esophageal spasm (choice C) is due to uncoordinated contractions of the
esophagus. It is generally associated with dysphagia.
Rib fracture (choice E) should result in well-localized chest pain. There is
pinpoint chest wall tenderness, and bone crepitus can be noted on palpation.
Question 2 of 6
On a chest radiograph, the patient is noted to have bilateral pleural effusions.
Evaluation of the fluid would most likely reveal which of the
following?
/ A. Exudative effusion
/ B. Low pleural fluid LDH
/ C. PIeural fluid to serum lactate dehydrogenase (LDH) ratio <0.6
/ D. PIeural fluid to serum protein ratio <0.5
/ E. Transudative effusion

Explanation - Q: 7.2 Close

The correct answer is A. This patient has an exudative pleural effusion
associated with an underlying rheumatologic condition.
Pleural fluid to serum protein ratio <0.5 (choice D), pleural fluid to serum
lactate dehydrogenase (LDH) ratio <0.6 (choice C), and low pleural fluid
LDH (choice B) are all diagnostic of a transudative effusion.
A transudative effusion (choice E) occurs when systemic factors that affect
the formation of pleural fluid are altered. It is associated with heart failure,
cirrhosis, and nephrotic syndrome.
Question 3 of 6
The patient also gives a history of symmetric joint pain and swelling as well as a
malar rash and photosensitivity. Which of the following is the
most likely diagnosis?
/ A. Ankylosing spondylitis
/ B. Polyarteritis nodosa
/ C. Psoriatic arthritis
/ D. Rheumatoid arthritis
/ E. Systemic lupus erythematous (SLE)

Explanation - Q: 7.3 Close

The correct answer is E. Systemic lupus erythematous (SLE) is an
autoimmune disorder that can affect multiple organ systems, especially
joints, serous membranes, lungs, skin, and kidneys. It is characterized by the
presence of pathogenic autoantibodies and immune complexes, which cause
cellular and tissue damage.
Ankylosing spondylitis (choice A) is a chronic disease in which inflammatory
changes and new bone formation occur at the attachment of tendons and
ligaments to bone. It often affects the spine and sacroiliac joints, and may
result in rigidity and fixation of the spine.
Polyarteritis nodosa (choice B) is an immune complex vasculitis, which
involves small and medium sized arteries. It can affect any organ system,
and predominantly occurs in men.
Psoriatic arthritis (choice C) is an arthritis associated with psoriasis. It can
present in many different ways, including a monoarthritis or a polyarthritis,
which can be symmetrical or asymmetrical.
Rheumatoid arthritis (choice D) is a chronic inflammatory disorder that
primarily affects the joints. It can produce extra-articular manifestations
including arteritis, scleritis, and pleural involvement, but it is not associated
with a malar rash or photosensitivity.
Question 4 of 6
Evaluation of the malar rash would most likely reveal which of the following?
/ A. Coral red fluorescence under Wood's lamp
/ B. Hyperplasia of all components of the epidermis
/ C. IgG autoantibodies directed against the epidermal basement membrane
/ D. Immune complex deposits at the dermaI-epidermal junction
/ E. Proliferation of the epidermis and parakeratosis


Explanation - Q: 7.4 Close

The correct answer is D. Immune complex deposits at the dermal-
epidermal junction are classically associated with the malar rash of SLE.
Coral red fluorescence under Wood's lamp (choice A) evaluation occurs in
erythrasma, which is a chronic bacterial infection caused by
Corynebacterium minutissimum.
Hyperplasia of all components of the epidermis (choice B) occurs in lichen
simplex chronicus, which is a well-circumscribed area of lichenification due to
repeated physical trauma such as scratching or rubbing.
IgG autoantibodies directed against the epidermal basement membrane
(choice C) cause a disease known as bullous pemphigoid. It is an
autoimmune disorder, which presents as chronic bullous eruptions.
Proliferation of the epidermis and parakeratosis (choice E) are associated
with psoriasis, which is a chronic inflammatory process characterized by
erythematous papules and plaques with silvery scaling.


Question 5 of 6
Which of the following laboratory tests would help confirm the diagnosis of the
underlying rheumatologic disorder?
/ A. Antibodies to single-stranded DNA
/ B. Antibodies to Sm antigen
/ C. EIevated C reactive protein
/ D. EIevated sedimentation rate
/ E. Rheumatoid factor

Explanation - Q: 7.5 Close

The correct answer is B. Antibodies to Sm antigen as well as antibodies to
double-stranded DNA are highly specific for systemic lupus erythematous.
Antibodies to single-stranded DNA (choice A) can be found in drug drug-
induced lupus, but not in idiopathic SLE.
Elevated C reactive protein (choice C) and elevated sedimentation rate
(choice D) often occur in SLE, especially when it is active. However, these
tests are nonspecific and can be elevated whenever there is inflammation or
infection. They do not confirm the diagnosis of SLE.
Rheumatoid factor (choice E) is often positive in patients with rheumatoid
arthritis. It is not associated with SLE.
Question 6 of 6
The patient is treated empirically with levofloxacin for a possible pneumonia.
Which of the following is the mechanism of action of this drug?
/ A. Inhibition of cell wall synthesis
/ B. Inhibition of DNA gyrase and topoisomerase IV
/ C. Inhibition of ergosterol synthesis
/ D. Inhibition of protein synthesis by blocking translocation
/ E. Inhibition of squalene epoxidase

Explanation - Q: 7.6 Close

The correct answer is B. Levofloxacin is a bactericidal antibiotic that inhibits
DNA gyrase and topoisomerase IV. It belongs to the group of antibiotics
known as fluoroquinolones, which also includes ciprofloxacin and
norfloxacin.
Penicillin is an example of an antibiotic that inhibits cell wall synthesis
(choice A).
Fluconazole is an example of an antifungal agent that inhibits ergosterol
synthesis (choice C).
Macrolides inhibit protein synthesis by blocking translocation (choice D). The
macrolides include erythromycin, clarithromycin, and azithromycin.
Inhibition of squalene epoxidase (choice E) results in decreased cell
membrane ergosterol synthesis. This is a mechanism of action used by
antifungal agents, including terbinafine.

A 55-year-old man consults a physician because he has become increasingly
constipated over the last 2 months. This has occurred despite
attempts to increase fiber in his diet and to markedly increase his physical
activity. Over-the-counter laxatives have been mostly ineffective. On
further questioning, the man reports that he has twice seen fresh blood on his
stools, which he attributed to hemorrhoids. He has also noticed a
sense of fullness and discomfort in his lower left abdominal quadrant, but
attributed these findings to his constipation. Digital examination is
negative. Colonoscopy demonstrates an area of stricture with surface ulceration
10 cm above the anal verge. The distal colon is surgically
resected. Pathologic examination demonstrates a napkin-ring lesion with marked
thickening of the mucosal layer of the rectum. On cross-
section, the lesion is seen to extend through the rectal muscle and into, but not
through, the adjacent serosal adipose tissue
Question 1 of 6
Which of the following is the most likely diagnosis?
/ A. Adenocarcinoma
/ B. Hyperplastic polyp
/ C. Squamous cell carcinoma
/ D. Tubular adenoma
/ E. Villous adenoma

Question 2 of 6
Further evaluation fails to demonstrate the patient's disease in either lymph
nodes or distant sites. This patient's disease would be staged as
which of the following?
/ A. Duke's classification A
/ B. Duke's classification B1
/ C. Duke's classification B2
/ D. Duke's classification C1
/ E. Duke's classification C2
/ F. Duke's classification D

Explanation - Q: 1.2 Close

The correct answer is C. The Duke's classification is commonly used for
staging of colorectal carcinoma. This patient has stage B2, characterized by
invasion through the muscularis propria and without lymph node involvement
or distant metastases. Duke's classification B2 has a 55-65% 5-year survival
rate.
Duke's classification A (choice A) has tumor limited to the mucosa with no
metastatic disease, and has a 5-year survival of more than 90%.
Duke's classification B1 (choice B) has invasion into the muscularis propria,
but no lymph node or distant metastases; this stage has a 5-year survival of
70-85%.
Duke's classification C1 (choice D) has invasion into the muscularis propria
and lymph node but not distant metastatic disease; the 5-year survival is 45-
55%.
Duke's classification C2 (choice E) has invasion through the muscularis
propria and lymph node but not distant metastases; the 5-year survival is 20-
30%.
Duke's classification D (choice F) is given whenever distant metastatic
disease is present, and has a 5-year survival of less than 1%.
Question 3 of 6
Which of the following lesions would yield the poorest prognosis in this patient?
/ A. Hyperplastic polyp
/ B. Juvenile polyp
/ C. Peutz-Jeghers polyp
/ D. Tubular adenoma
/ E. Villous adenoma

Explanation - Q: 1.3 Close

The correct answer is E. The adenomatous polyps, which are considered to
have some potential for undergoing malignant transformation, include both
villous adenomas and tubular adenomas (choice D). Villous adenomas tend
to be larger and have more severe atypia than tubular adenomas, and
consequently have a greater risk of progression to frank carcinoma.
Hyperplastic polyps (choice A), juvenile polyps (choice B), and Peutz-
Jeghers polyps (choice C) are not considered to be premalignant lesions.
Question 4 of 6
One of the acquired genetic changes that can occur during the progression of
this disease is the loss of the p53 site on chromosome 17. The
gene product of this site is thought to function as which of the following?
/ A. Apoptosis regulator
/ B. DNA repair regulator
/ C. Growth factor
/ D. Growth factor receptor
/ E. Signal transduction inhibitor

Explanation - Q: 1.4 Close

The correct answer is A. The biochemistry of tumor cells has been an
active topic of research over the last few decades, and many molecular
changes have been elucidated. p53 acts as a cell cycle and apoptosis
(programmed cell death) regulator, and a loss of p53 activity has been linked
to progression of a late adenomatous polyp to adenocarcinoma of the colon.
BRCA-1 and BRCA-2, which are important in breast cancer, are examples of
gene products that act as DNA repair regulators (choice B).
Examples of growth factors (choice C) important in tumor growth include
PDGF (platelet-derived growth factor) and FGF (fibroblast growth factor).
An example of a growth factor receptor (choice D) important in tumor growth
is the EGF (epidermal growth factor) receptor.
The APC gene product, discussed in the previous question, acts as an
inhibitor of signal transduction (choice E).
Question 5 of 6
If the patient was unable to tolerate surgery, the treatment of choice would be
which of the following?
/ A. Cisplatin
/ B. Cyclophosphamide
/ C. FIuorouracil
/ D. Methotrexate
/ E. Sulfasalazine

Explanation - Q: 1.5 Close

The correct answer is C. In the treatment of colorectal cancer, surgical
resection is the treatment of choice for virtually all patients who have
resectable lesions and can tolerate general anesthesia. Regional lymph node
dissection is generally recommended to determine staging, which
subsequently guides decisions for pharmacological therapy. Adjuvant
chemotherapy generally includes treatment with fluorouracil and either
levamisole or leucovorin. This therapy has been shown to reduce mortality
by about one-third. Fluorouracil, or 5-FU, is an antimetabolite that blocks the
methylation reaction of deoxyuridylic acid to thymidylic acid. This interferes
with DNA synthesis, and to a lesser extent, RNA synthesis. 5-FU is indicated
for the palliative management of colorectal, breast, stomach, and pancreas
carcinoma. This agent is commonly associated with the development of
nausea, vomiting, alopecia, severe ulceration of the oral and GI mucosa, as
well as dermopathy (erythematous desquamation of the palms and soles).
Cisplatin (choice A) is an antineoplastic agent that exerts its antineoplastic
activity by binding to DNA, producing intrastrand cross-links, and forming
DNA adducts. Cross-linkage of DNA strands blocks the synthesis of DNA,
RNA, and protein. This agent is commonly used to treat solid tumors, such
as metastatic testicular carcinoma, ovarian carcinoma, as well as bladder
carcinoma. Aside from immunosuppression, the most common side effect of
this agent is severe persistent emesis.
Cyclophosphamide (choice B) is an antineoplastic commonly used to treat a
variety of malignant diseases, such as Hodgkin disease, as well as several
types of leukemia. It is associated with the development of hemorrhagic
cystitis in approximately 7 -12% of all patients receiving this medication. This
medication is an alkylating agent related to the nitrogen mustards.
Methotrexate (choice D) competitively inhibits dihydrofolic acid reductase. It
is indicated for the treatment of severe, active, rheumatoid arthritis in adults
who have an insufficient response with conventional therapies. It is also used
in antineoplastic therapeutic regimens, such as those for ALL, cancers of the
head, neck, and breast. It is contraindicated in pregnancy, nursing mothers
and hepatic insufficiency, as well as in patients with AIDS, blood dyscrasias,
bone marrow hypoplasia, leukopenia, and severe anemia.
Sulfasalazine (choice E) is a competitive antagonist of para-aminobenzoic
acid (PABA). It is indicated for the treatment of ulcerative colitis. Non-labeled
indications include use in patients with rheumatoid arthritis, Crohn disease,
and ankylosing spondylitis. Warnings and precautions include use in patients
with porphyria, since it can precipitate an attack; the drug causes
phototoxicity as well as the hematologic problems.
Question 6 of 6
A close relative of the patient is diagnosed with a familial form of colon cancer, in
which cancer develops in the setting of thousands of
adenomatous polyps in the boweI. A gene at which of the following chromosomal
locations has been linked to this cancer?
/ A. 3p25
/ B. 5q12
/ C. 11p13
/ D. 13q14
/ E. 22q

Explanation - Q: 1.6 Close

The correct answer is B. Acquired and inherited genetic changes seen in
colon cancer include loss of APC on chromosome 5q, activation of k-ras on
12q, loss of DCC on 18q, and loss of p53 on 17p. Colon cancers presenting
at less than 40 years of age are more likely to have a familial component.
3p25 (choice A) is the location of the VHL gene, which is important in von
Hippel-Lindau disease, renal cell carcinoma, and pheochromocytoma.
11p13 (choice C) is the location of the WT1 gene, which has been linked to
Wilms tumor.
13q14 (choice D) is the location of the Rb gene, which is important in
retinoblastoma, osteosarcoma, and to lesser degrees, cancers of the breast,
lung, prostate, and bladder.
22q (choice E) is the location of the NF2 gene, which has been linked to
neurofibromatosis type 2, central schwannomas, and meningiomas.


A 26-year-old woman gives birth, at term, to a 3200 g (7 Ib 1 oz) baby boy.
Physical examination of the neonate is unremarkable. However, the
baby still has not passed stool by the third day of life, and develops intestinal
obstruction with vomiting. The small amount of stool that is
eventually passed consists of particularly thick, tenacious meconium.

Question 1 of 8
The baby undergoes a laboratory test in which a small electrical current is
passed through his skin and the sweat produced by this stimulus is
collected. Which of the following would be an expected result in the sample
obtained?
/ A. Decreased Mg2+
/ B. Decreased Na+
/ C. Decreased phosphate
/ D. Increased Mg2+
/ E. Increased Na+
/ F. Increased phosphate

Explanation - Q: 2.1 Close

The correct answer is E. The underlying biochemical defect involves the
cystic fibrosis transmembrane conductance regulator (CFTR) protein, which
is coded for by a gene on the long arm of chromosome 7. The CFTR protein
forms a channel across the plasmalemma that regulates the flux of negative
ions, e.g., chloride and bicarbonate. Alterations in transport of these ions
secondarily impair water and sodium ion transport, leading to very viscous
mucous secretions and abnormal sweat electrolytes. If asked, some mothers
will report that their babies taste "salty" when they kiss them. Once cystic
fibrosis is suspected, the diagnosis can be confirmed with measurements of
sodium secretion in the sweat. Sweat production can be stimulated with the
drug pilocarpine, which is driven into the skin with a small electrical current
(iontophoresis). Sweat sodium levels must be elevated above 70 mmol/L on
two separate occasions for the diagnosis of cystic fibrosis to be made.
Although the actual ion transported by the channel is chloride, sodium ion,
which is more easily measured, will follow the chloride ion and can be
monitored in the sweat.
Mg
2+
(choices A and D) and phosphate (choices C and F) are not altered by
the abnormal CFTR protein.
Question 2 of 8
The underlying biochemical defect in the neonate's disease most likely involves
which of the following?
/ A. Amino acid transporter
/ B. Enzyme in glycolysis
/ C. Enzyme in tyrosine metabolism
/ D. Hemoglobin
/ E. Transmembrane ion channel

Explanation - Q: 2.2 Close

The correct answer is E. The neonate likely has cystic fibrosis. The earliest
and most distinctive presentation of this disease, which is seen in 10-15% of
cases, is meconium ileus, due to tenacious pancreatic secretions that stick to
the walls of the intestine, and cannot be passed through. Other children
present at a somewhat older age with failure to gain weight despite good
food intake. Cystic fibrosis is the most common genetic recessive disease in
the Caucasian population. It is a multisystem disease that can cause
problems through life. The disease is characterized by thick, tenacious bodily
secretions that eventually can lead to the destruction of the exocrine
pancreas and lungs. The metabolic defect involves a transmembrane
conductance regulator (CFTR) protein that regulates the transport of
negative ions such as chloride and bicarbonate. The other choices are
distracters unrelated to this case.


Question 3 of 8





In order to confirm the diagnosis, genetic testing was done on the patient (III-1)
and other available family members referenced in the pedigree
shown above. DNA from each individual was amplified using a polymerase chain
reaction (PCR) and tested using a panel of allele-specific
oligonucleotide (ASO) probes for the more common mutations in the gene
associated with this disease. A probe for the normal allele was
included next to each probe for a disease-producing mutation. The results
appear below.




Which of the following conclusions can be made about the baby?
/ A. He is a carrier of the DF508 allele
/ B. He is a carrier of the W1282X allele
/ C. He is a compound heterozygote for two disease-producing alleles
/ D. He is a homozygote for the DF508 mutation
/ E. He is a homozygote for the normal allele


Explanation - Q: 2.3 Close

The correct answer at C. The proband (III-1) is a compound heterozygote
for two disease-producing alleles (DF508 from the mother, and W1282X from
the father). Both mutations, associated with cystic fibrosis, occur in the CFTR
gene at 7q31. A compound heterozygote is an individual carrying two
different disease-causing mutations at a locus. In recessive diseases, such
as cystic fibrosis, compound heterozygotes are almost always affected.
DF508 (deletion of phenylalanine at position 508 in the CFTR protein) is the
most common mutation associated with cystic fibrosis in individuals of
European ancestry. An exception (do not memorize for the USMLE) is
Ashkenazi Jews in whom the W1282X (a nonsense mutation in which the
codon for tryptophan at position 1282 in the CFTR protein is converted to a
stop codon) is more common. The resultant CFTR protein ends with the
amino acid at position 1281. Both the CFTR proteins encoded by the DF508
allele and the W1282X allele are non-functional.
Choices A and B are not correct since the proband is an affected individual
and would not be best described as a carrier.
Choices D and E specify homozygous individuals in which both alleles of the
gene are identical. This interpretation is inconsistent with the blot shown in
which the proband's amplified DNA reacts with two different probes, making
him a heterozygote.

Question 4 of 8




The patient's uncle (II-4) and his wife (II-5) are concerned that their expected
child (III-2) may be affected with the disease but are not
immediately available for genetic testing. The geneticist knows that for this
disease, the carrier frequency in the general population is 1 in 25.
Based on this, what is the risk that the expected child (III-2) will be affected?
/ A. 1/25
/ B. 1/50
/ C. 1/100
/ D. 1/200
/ E. 1/625

Explanation - Q: 2.4 Close

The correct answer is D. Because individual I-1 is homozygous for the
normal allele (from the dot blot) and individual I-2 is a carrier of the W1282X
mutation (from the dot blot), the uncle (II-4) has a 1/2 risk for carrying a
mutation, and a 1/2 chance of passing it along if he does. Therefore, from
this father, the risk to the expected child (III-2) is 1/2 x 1/2 = 1/4.
The mother (II-5), an unrelated individual from the general population has a
1/25 chance of carrying a mutation, and a 1/2 chance of passing it along if
she does. The risk from the mother is 1/25 x 1/2 = 1/50.
The risk of these parents having an affected child is 1/4 x 1/50 = 1/200.
Question 5 of 8
The infant survives the neonatal period. Later, as the infant is started on solid
foods, he develops malabsorption and steatorrhea and it
becomes necessary to replace the pancreatic enzymes that are not being
released into the gut because of the viscosity of the pancreatic
secretions. Which of the following should be replaced?
/ A. Intrinsic factor
/ B. Lactase
/ C. Lipase
/ D. Maltase
/ E. Sucrase

Explanation - Q: 2.5 Close

The correct answer is C. The therapy of cystic fibrosis, while still far from
perfect, is one of the great success stories of modern medicine. Formerly,
most patients died in early childhood. Damage to the pancreas by tenacious
secretions entrapped in the ducts eventually leads to a deficient production
of pancreatic enzymes including amylase, lipase, and protease. The use of
(extremely expensive - up to $1000/month) oral pancreatic enzyme
(amylase+lipase+protease) replacement therapy throughout life means that
these children are no longer dying of malnutrition.
Intrinsic factor (choice A) is deficient in pernicious anemia.
Lactase (choice B), maltase (choice D), and sucrase (choice E) are
enzymes on the intestinal mucosa that help to process sugars, and can be
genetically deficient.


Question 6 of 8

The child has problems throughout childhood with recurrent pneumonia. At age
eight, he develops a particularly serious case of pneumonia,
which is characterized by production of sputum with an unusual bluish-green
coloration. This finding should specifically suggest which of the
following microorganisms as the cause of the pneumonia?



Explanation - Q: 2.6 Close

The correct answer is C. Pseudomonas aeruginosa is an important cause
of pneumonia in these children and can be suspected clinically because the
organism produces a blue-green pigment that may stain sputum. Therapy
often requires intravenous antibiotics, as the organism is both very tissue-
destructive and, often, very antibiotic-resistant.
The other organisms listed (choices A, B, D, and E) can also cause
pneumonia in cystic fibrosis patients, but would not produce the unusual
sputum discoloration.



Question 7 of 8

By age 15, a chest x-ray film shows visible bronchial shadows out to the lung
periphery. This finding suggests which of the following?
/ A. Asthma
/ B. Bronchiectasis
/ C. Chronic bronchitis
/ D. Emphysema
/ E. Pneumonia

Explanation - Q: 2.7 Close

The correct answer is B. Repeated episodes of bronchopneumonia can
cause permanent bronchial dilation known as bronchiectasis, which is visible
radiologically. Physiotherapy with postural therapy supplemented by
percussion of the chest is now used routinely as a prophylactic measure to
remove the tenacious bronchial secretions that predispose for pneumonia.
Pulmonary infections are treated aggressively. The result of these
improvements in management is that many of these children are now
surviving to their late teens or early twenties. By that point, the lung damage,
which typically includes bronchiectasis, has usually been sufficient to also
cause right heart damage, leading to cor pulmonale. Combined heart-lung
transplants are now being attempted in some of these young people, and
offer the potential of another decade of survival.
Asthma (choice A) and chronic bronchitis (choice C) cannot be diagnosed
radiologically.
Emphysema (choice D) will produce overly dark lung fields (lucencies) on x-
ray films.
Pneumonia (choice E) is associated with infiltrates that are visible as areas
of increased density on chest x-ray films.
*** Kleptomania FAQ is at http://www.structurise.com/kleptomania/faq.htm ***

Question 8 of 8

At age 19, the patient develops persistent severe ankle edema. This finding
suggests which of the following?
/ A. Cor pulmonale
/ B. Endocarditis
/ C. Myocardial infarction
/ D. Pericarditis
/ E. Pulmonary valvular stenosis

Explanation - Q: 2.8 Close

The correct answer is A. By young adulthood, the lung damage caused by
cystic fibrosis has usually been sufficient to also cause right heart damage,
leading to cor pulmonale. The other cardiac lesions listed (choices B, C, D,
and E) are not seen with increased frequency in these patients.

A 3-month-old child is taken to the emergency department because his bowel
movements have completely stopped, and he has developed a
fever and an expanded abdomen. The baby has a history of a several day delay
before passing meconium after birth. Since that time, the
baby had been willing to take some nourishment, but fed poorly and seemed to
be constipated all of the time. A sweat chloride test performed
at 6 weeks of age was normaI.
Question 1 of 5
Which of the following is the most likely diagnosis?
/ A. Cystic fibrosis
/ B. Diaphragmatic hernia
/ C. Esophageal atresia
/ D. Hirschsprung disease
/ E. Hypertrophic pyloric stenosis

Explanation - Q: 3.1 Close

The correct answer is D. This child most likely has Hirschsprung disease. In
this condition, a congenital failure of adequate innervation of the distal bowel
causes a functional obstruction secondary to failed peristalsis.
Characteristically, the involved bowel segment is narrowed, and there is a
dilatation proximal to the obstruction in the "normal" bowel. Depending upon
the length of the bowel segment that is involved, affected individuals may be
diagnosed in infancy (delayed passage of meconium with normal anal tone
and empty rectal ampulla), during the first year of life (chronic constipation,
large fecal mass on abdominal palpation), or even at later ages, including
adulthood (particularly in mild cases) with chronic constipation. Most
individuals with Hirschsprung disease have normal life spans, although in
rare cases when very large segments of bowel are involved, patients may
suffer from chronic malnutrition, with all of its complications.
Cystic fibrosis (choice A) is one cause of delayed passage of meconium, but
the results of the sweat chloride test argue against this diagnosis.
Diaphragmatic hernia (choice B) is a neonatal emergency requiring
immediate surgery because loops of bowel have herniated through the
diaphragm and are compressing the lungs, thereby preventing the lungs from
inflating.
Esophageal atresia (choice C) and hypertrophic pyloric stenosis (choice E)
typically cause food regurgitation and vomiting.

Question 2 of 5

A fulI-thickness biopsy of an involved area of the gastrointestinal react would
most likely show a lack of which of the following cell types?
/ A. Columnar epithelial cells
/ B. Endothelial cells
/ C. Fibrocytes
/ D. Ganglion cells
/ E. Smooth muscle cells



Explanation - Q: 3.2 Close

The correct answer is D. The histologic basis of Hirschsprung disease is a
lack of ganglion cells in both Meissner's submucosal plexus and Auerbach's
myenteric plexus. This lack of ganglion cells may be accompanied by
hypertrophied nerve bundles with a high acetylcholinesterase content. The
origins of the ganglion cell deficit in neuronal cells is still not completely
worked out, but may involve a failure of the cells to migrate during
development from the proximal bowel to the distal bowel. Uninvolved
portions of bowel have a normal distribution of ganglion cells and nerve
fibers. The other cell types listed are found in normal numbers.
Columnar epithelial cells (choice A) line the large intestinal lumen.
Endothelial cells (choice B) line blood vessels.
Fibrocytes (choice C) are found in the submucosa and between muscle
layers.
Smooth muscle cells (choice E) are found in the muscular wall of the gut.

Question 3 of 5
Which of the following structures is invariably involved in this patient's condition?
/ A. Appendix
/ B. Ascending colon
/ C. Cecum
/ D. IIeum
/ E. Rectum

Explanation - Q: 3.3 Close

The correct answer is E. The rectum is invariably involved in Hirschsprung
disease, and the absence of ganglion cells may also be present proximally to
varying degrees. The defective innervation is usually for all ganglion cells
past a point that varies from individual to individual, and no skip lesions are
seen. Involvement of the other structures listed in the choices can occur, but
is not invariable.

Question 4 of 5

This child is at particular risk of developing which of the following complications?
/ A. Amebiasis
/ B. Crohn disease
/ C. Diverticulitis
/ D. Toxic enterocolitis
/ E. UIcerative colitis

Explanation - Q: 3.4 Close

The correct answer is D. Toxic enterocolitis, or toxic megacolon, is a feared
and potentially fatal complication of Hirschsprung disease. The condition can
develop when chronic obstipation induces bacterial overgrowth. Production
of bacterial toxins then induces a fulminant water loss into the bowel lumen,
or if the colon is not completely blocked by stool, a profuse watery diarrhea.
The other conditions listed are not associated with Hirschsprung disease.
Amebiasis (choice A) is a gastrointestinal infection with amebae (usually
Entamoeba histolytica) that is typically acquired by drinking contaminated
water, and may be asymptomatic, or cause a dysentery-like illness.
Crohn disease (choice B) and ulcerative colitis (choice E) are chronic
inflammatory bowel diseases that appear to have an autoimmune
component, and these diseases may be associated with other autoimmune
diseases.
Diverticulitis (choice C) typically affects older individuals and is most strongly
associated with a low fiber diet.




Question 5 of 5
The child is treated with a diverting colostomy that is pulled through a transverse
left lower quadrant abdominal incision. Which of the following
muscles would be most likely to be affected by the incision?
/ A. IIiacus
/ B. Psoas
/ C. Quadratus lumborum
/ D. Rectus abdominis
/ E. Transversus abdominis

Explanation - Q: 3.5 Close

The correct answer is D. Of the muscles listed, only the rectus abdominis,
which is the largest of the abdominal muscles, would be likely to be
damaged. The rectus abdominis muscle is a long, straight muscle that is
oriented longitudinally, arises from the pubic bone and symphysis pubis, and
inserts in the anterior surface of the costal cartilages and xiphoid process.
The muscle is wide superiorly and narrower inferiorly, which helps to protect
it from damage during the diverting colostomy procedure. A diverting
colostomy is commonly used in (tiny) newborn infants, and then later
switched to a "pull-through" colostomy (through the anus) when the child is
larger.
The iliacus (choice A), the closely related psoas (choice B), and quadratus
lumborum (choice C) are all muscles of the posterior abdominal wall.
The transversus abdominis (choice E) runs horizontally in the upper half of
the abdominal wall.

A patient complains to his physician of chronic constipation. On the instruction of
his physician, the patient increases his dietary fiber, but his
constipation persists. The physician refers the patient to a gastroenterologist.
Colonoscopy reveals over fifty 2-3 mm openings into the bowel
mucosa. No bleeding is seen, and the intervening mucosa appears normaI.
Question 1 of 5
Which of the following is the most likely diagnosis?
/ A. Amebiasis
/ B. Crohn disease
/ C. Diverticulosis
/ D. Pseudomembranous colitis
/ E. UIcerative colitis

Explanation - Q: 4.1 Close

The correct answer is C. The appearance described is typical for
diverticulosis. The openings seen are into diverticula, which are small,
acquired outpouchings of the mucosa through the muscle of the bowel.
Milder cases of diverticulosis are usually asymptomatic; patients with
extensive diverticula may experience mild cramps, bloating, or constipation.
In amebiasis (choice A), the patient would have much more severe
symptoms and bleeding would probably be present.
In Crohn disease (choice B), pseudomembranous colitis (choice D), and
ulcerative colitis (choice E), the mucosa would appear obviously diseased.

Question 2 of 5
When population studies are done, which of the following sites has the largest
number of the lesions described in the question stem?
/ A. Cecum
/ B. Descending colon
/ C. Rectum
/ D. Sigmoid colon
/ E. Transverse colon

Explanation - Q: 4.2 Close

The correct answer is D. While diverticula can be found throughout the
colon (choices A, B, C, and E), most people have a predominance of
diverticula in the left side of the colon, and the sigmoid colon is the most
frequent site of involvement on the left side.

Question 3 of 5
The patient would be most likely to be which of the following ages?
/ A. 15 years old
/ B. 20 years old
/ C. 30 years old
/ D. 40 years old
/ E. 65 years old

Explanation - Q: 4.3 Close

The correct answer is E. Diverticulosis is primarily a disease of the elderly,
and the more extensive the disease, the older the individuals tend to be.
Diverticulosis is rare below the age of 40, occurs in about half of Americans
between the ages of 60 and 80, and is nearly universal in individuals older
than 80 years.

Question 4 of 5
Which of the following would be most effective in preventing this patient's
disorder?
/ A. High fat diet
/ B. High fiber diet
/ C. High meat diet
/ D. High sugar diet
/ E. High vitamin C diet

Explanation - Q: 4.4 Close

The correct answer is B. A high fiber diet tends to soften stool, and reduces
the chance of fecal impaction within a diverticulum, with resultant increased
risk of developing diverticulitis (inflamed diverticula). It is also thought that a
high fiber diet may reduce the rate at which diverticula develop, although
evidence for this is not completely conclusive.
The typical American diet with high fat (choice A), high meat (choice C),
and high sugar (choice D) is thought to predispose for diverticulosis.
While vitamin C (choice E) is good for general health, it does not, per se,
have a role in preventing diverticulosis or its complications.

Question 5 of 5
If an inflammatory/infectious complication did occur because the patient failed to
modify his diet, this would most likely clinically resemble
which of the following diseases?
/ A. Acute hepatitis
/ B. Appendicitis
/ C. Chronic hepatitis
/ D. Peptic ulcer disease
/ E. Renal calculus

Explanation - Q: 4.5 Close

The correct answer is B. Diverticula are similar structures to the appendix,
and acute diverticulitis can share many clinical features with acute
appendicitis, including abdominal pain, lower abdominal tenderness (usually
left-sided), fever, nausea, vomiting, chills, and cramping. In some older
patients, these symptoms may be blunted, and the patient may only appear
to be fairly severely ill without obvious localizing signs. Diverticulitis should
be considered a serious complication, since death can occur if the inflamed
diverticula rupture and seed the abdomen with fecal bacteria, leading to
peritonitis, and potentially, sepsis. Diverticular disease can also evolve to a
chronic diverticulitis that predisposes for stricture, abscess formation, fistula
formation, and intestinal obstruction. Severe cases may require resection of
much of the colon to prevent further complications from developing.
Diverticula can also be a significant source of GI bleeding, which can be
severe because the bleeding is often from a small artery that has been
distorted by a diverticulum and then ruptures.
Jaundice or other symptoms that would suggest hepatic involvement
(choices A and C) are not part of the clinical picture of diverticulitis.
The gnawing pain of peptic ulcer (choice D) and the sharp, intense pain of
renal calculus (choice E) are different from the diffuse abdominal pain with
focal tenderness seen in diverticulitis.



A 32-year-old woman consults a physician because she has been having diffuse
abdominal pain accompanied by chronic constipation with
narrow caliber, hard stools that are painful to defecate. She has tried laxatives,
but they do not seem to work. Sometimes the hard stools
alternate with periods of diarrhea with small volumes of loose stooI, with
evacuation preceded by urgency. At times, her stool is coated with
white mucus. She also has problems with bloating and gas. The patient is sent to
a gastroenterologist, who performs a variety of diagnostic
tests including a lactose tolerance test, barium enema, and colonoscopy. There
are no abnormal results on these studies.
Question 1 of 5
Which of the following is the most likely diagnosis?
/ A. Crohn disease
/ B. Diverticulitis
/ C. Irritable bowel syndrome
/ D. UIcerative colitis
/ E. Viral gastroenteritis
Explanation - Q: 5.1 Close

The correct answer is C. Irritable bowel syndrome is an often chronic
functional bowel disorder that may cause either constipation or diarrhea,
usually accompanied by abdominal pain. Patients vary in whether
constipation or diarrhea is the predominant symptom. In most patients, the
symptoms start before age 35, and there is a female predominance, at least
in those patients seeking medical care. Characteristically, the usual
screening diagnostic tests for gastrointestinal disease are unrevealing.
Crohn disease (choice A), diverticulitis (choice B), and ulcerative colitis
(choice D) would all produce visible lesions on barium enema or
colonoscopy.
Viral gastroenteritis (choice E) rarely lasts longer than one or two weeks.

Question 2 of 5
Population studies have estimated the prevalence of this patient's condition in the
United States to be which of the following?
/ A. Less than 1%
/ B. 2-6%
/ C. 10-20%
/ D. 30-50%
/ E. 75-90%

Explanation - Q: 5.1 Close

The correct answer is C. Irritable bowel syndrome is an often chronic
functional bowel disorder that may cause either constipation or diarrhea,
usually accompanied by abdominal pain. Patients vary in whether
constipation or diarrhea is the predominant symptom. In most patients, the
symptoms start before age 35, and there is a female predominance, at least
in those patients seeking medical care. Characteristically, the usual
screening diagnostic tests for gastrointestinal disease are unrevealing.
Crohn disease (choice A), diverticulitis (choice B), and ulcerative colitis
(choice D) would all produce visible lesions on barium enema or
colonoscopy.
Viral gastroenteritis (choice E) rarely lasts longer than one or two weeks.

Question 2 of 5
Population studies have estimated the prevalence of this patient's condition in the
United States to be which of the following?
/ A. Less than 1%
/ B. 2-6%
/ C. 10-20%
/ D. 30-50%
/ E. 75-90%

Explanation - Q: 5.2 Close

The correct answer is C. The incidence (new diagnosis rate) of irritable
bowel syndrome is 1-2% a year, but because the condition is so often long-
standing, the prevalence seen in screening population studies is much
higher, at 10-20%. Many of these patients only rarely or never seek medical
attention for their bowel complaints. 20-50% of gastrointestinal referrals are
for what is eventually diagnosed as irritable bowel syndrome.

Question 3 of 5
Which of the following findings would suggest an alternative diagnosis in this
patient?
/ A. Abnormal stool frequency
/ B. BIood in the stool
/ C. Feeling of abdominal distension
/ D. Passage of mucus
/ E. Straining at stool

Explanation - Q: 5.3 Close

The correct answer is B. One of the problems with the medical handling of
irritable bowel syndrome is that nongastroenterologists sometimes jump to
the diagnosis as an explanation for the patient's symptoms when actually
other, either more treatable or more serious, disease is present. It is
therefore important to be aware of the types of features that make the
diagnosis of irritable bowel syndrome less likely. These include blood in the
stool, pain that awakens or interferes with sleep, diarrhea that awakens or
interferes with sleep, weight loss, fever, and an abnormal physical
examination. The features listed in the other choices are common in irritable
bowel syndrome.

Question 4 of 5
While formerly, many physicians regarded this disease as a nearly purely
psychopathologic phenomenon, evidence now suggests that which
of the following physiologic abnormalities are often present in these individuals?
/ A. Decreased visceral pain sensation
/ B. Generalized smooth muscle hyporesponsiveness
/ C. Longer intervals between small intestinal migratory motor complexes
/ D. Narrowed dermatomal distributions of referred pain
/ E. Variations in colonic slow wave frequency


Explanation - Q: 5.4 Close

The correct answer is E. Like achalasia, which was formerly thought to be a
nearly purely psychologic problem, but is now accepted as having a clear
physiologic basis, the evidence supporting the presence of altered
physiology in irritable bowel syndrome is becoming increasingly compelling.
Current thinking suggests that irritable bowel syndrome may be best
visualized as a combination of altered GI motility, visceral hyperalgesia, and
psychopathology. With respect to altered GI motility, the colon in patients
with irritable bowel syndrome is now known to have variations in slow wave
frequency. Additionally, there is a blunted, late response of the colonic spike
potential following eating. These colonic alterations tend to be more marked
in patients prone to diarrhea. The transit time of a meal through the small
bowel tends to be increased in patients with predominately constipation, and
decreased in those with predominately diarrhea. The intervals between
migratory motor complexes are also often shorter (rather than longer as in
choice C) than in asymptomatic individuals. It is thought that all of these
alterations may be related to a generalized smooth muscle
hyperresponsiveness (not hyporesponsiveness as in choice B), that also
often affects the bladder and produces symptoms including frequency,
urgency, nocturia, and hyperresponsiveness to methacholine challenge. In
addition to the altered smooth muscle response, visceral hyperalgesia (rather
than decreased pain as in choice A) is also frequently present. This can be
demonstrated by studies in which balloons are inflated in the colon and small
intestine, and produce pain at lower volumes than in controls. These patients
also frequently report widened (rather than narrowed as in choice D)
dermatomal distributions of referred pain. It is thought that there may be an
increased sensitization of the intestinal nociceptive pathways.
The role of psychiatric disturbance in these patients is unclear. On the one
hand, psychological disturbances among patients seen in clinic are strikingly
frequent (up to 77% of patients). However, on the other hand, the large
numbers of individuals identified in screening studies who have irritable
bowel symptoms, but do not aggressively seek medical attention for them,
appear to be, on average, no different from the general population. It may be
that the emotional disturbances experienced by severely psychiatrically
disturbed individuals tend to exacerbate the irritable bowel symptoms, and
also that these individuals often tend to have lowered pain thresholds and
lowered ability to ignore discomfort than other individuals. Treatment of
irritable bowel disease remains primarily supportive (good doctor-patient
relationship, psychiatric help if needed, avoidance of any identified triggers,
caffeine avoidance, increasing fiber in diet) at the moment. This may change
rapidly in the future as the recent increases in understanding the underlying
pathophysiology are suggesting a variety of experimental pharmacologic
approaches including peripheral opiate narcotic antagonists to modulate
visceral nociception without central effect, serotonin agonists and
somatostatin agonists to stimulate colonic transit, muscarinic receptor
antagonists to inhibit intestinal motility, cholecystokinin receptor antagonists
to stimulate intestinal mobility, and gut-selective calcium channel blockers to
decrease postprandial motility.

Question 5 of 5

This patient is initially prescribed a course of dietary interventions and patient
education, however this approach is largely unsuccessfuI. The
most appropriate next step in the therapy of this patient would be which of the
following?
/ A. Hyoscyamine
/ B. Metronidazole
/ C. Prednisone
/ D. Sulfasalazine
/ E. Surgery

Explanation - Q: 5.5 Close

The correct answer is A. Dietary therapy is effective in almost 2/3 of
patients with irritable bowel syndrome. Pharmacological therapy is often
reserved for patients with refractive/severe signs and symptoms of this
condition. The antispasmodic agents are the most commonly used agents in
the treatment of this condition. They can alleviate postprandial abdominal
pain in these patients when administered 30-60 minutes before a meal.
Commonly used agents are dicyclomine and hyoscyamine. Antidiarrheals
and opioids may be useful in patients with frequent loose stools. Furthermore
they may be used prophylactically when diarrhea is anticipated, or would be
inconvenient. Psychotropic agents may also be used in patients with chronic,
unremitting abdominal pain, since some symptoms may be due to psychiatric
disturbances, and hence, may respond to antidepressants such as
desipramine or sertraline.
Metronidazole (choice B) is an antibiotic that is considered by many to be
the treatment of choice for antibiotic-induced colitis, since it is less expensive
than vancomycin and does not encourage the emergence of vancomycin-
resistant bacteria.
Corticosteroids such as prednisone (choice C) are commonly used in the
treatment of ulcerative colitis, but not irritable bowel syndrome.
Sulfasalazine (choice D) is a competitive antagonist of PABA. It is indicated
for the treatment of ulcerative colitis. Unlabeled indications include the
treatment of rheumatoid arthritis, Crohn disease, collagenous colitis and
ankylosing spondylitis.
Surgery (choice E) may be used in the treatment of severe ulcerative colitis
unresponsive to traditional therapies. However, it is not used in the treatment
of irritable bowel syndrome.



A 50-year-old man consults a physician because he has developed a chronic,
non-productive cough and is experiencing a reduced ability to
do strenuous work. His symptoms have developed insidiously. On questioning,
he states that he is a smoker and has also worked as a
contractor for all of his adult life. Physical examination is notable for the presence
of repetitive end-inspiratory basal crackles and finger
clubbing. A chest x-ray film shows diffusely distributed, small irregular opacities
that are most prominent in the lower lung zones. Localized
areas of pleural thickening are also noted. No large masses are seen.

Question 1 of 4

The chest x-ray film is most consistent with which of the following?
/ A. Emphysema
/ B. Interstitial disease
/ C. Lobar pneumonia
/ D. Lung cancer
/ E. PIeural effusion

Explanation - Q: 1.1 Close

The correct answer is B. Diffusely distributed small irregular opacities
suggest the presence of interstitial lung disease.
Emphysema (choice A) would produce unusually dark lung fields.
Lobar pneumonia (choice C) would produce a "white out" of one or more
lung lobes.
Lung cancer (choice D), if large, would produce a mass lesion (often
involving a bronchus), or, if very small, might not be recognized on chest x-
ray.
Pleural effusion (choice E) would cause a whitened area due to fluid below
the lung

Question 2 of 4

The patient is sent for spirometry for further evaluation. FEV1 and FVC are both
shown to be about 60% of the expected values, and the ratio
of FEV1/FVC is 90%. These findings are most consistent with which of the
following?
/ A. Asthma
/ B. Bronchiectasis
/ C. Chronic bronchitis
/ D. Emphysema
/ E. Restrictive lung disease

Explanation - Q: 1.2 Close

The correct answer is E. Spirometry is commonly used to subdivide non-
tumorous lung diseases into diseases that are predominately obstructive in
nature and those that are predominately restrictive. The process of
spirometry involves controlled breathing in and out while airflow is plotted
against volume to obtain a continuous loop. FVC (forced vital capacity) is the
maximum volume of air that can be forcibly and rapidly exhaled following a
maximum inspiration. FEV1 (forced expiratory volume in the first second) is
the volume of air expelled in the first second of a forced expiration starting
from full inspiration. Restrictive lung disease usually shows a reduction in
both FVC and FEV1, and the ratio of FEV1/FVC is greater than 80%. If these
readings are obtained in a patient, further studies to measure lung volumes
are usually performed to confirm the finding.
Asthma (choice A), bronchiectasis (choice B), chronic bronchitis (choice
C), and emphysema (choice D) all tend to produce obstructive patterns, with
FEV1 disproportionately decreased when compared to FVC.


Question 3 of 4

The patient's work history is most suggestive of exposure to which of the
following?
/ A. Asbestos
/ B. Beryllium
/ C. Coal
/ D. Kaolin
/ E. Silica

Explanation - Q: 1.3 Close

The correct answer is A. Working history often offers helpful clues about
possible toxic exposures that may have contributed to lung disease.
Asbestos was formerly a common constituent of insulating material in
buildings because of both its insulating properties and its fire-resistant
properties. People working on old buildings, particularly when removing the
old insulation, are consequently vulnerable to high exposures unless they
take care to minimize exposure with respirators. The risk to individuals living
and working in old buildings is usually markedly less, since the asbestos is
typically found behind walls. Individuals who work in asbestos mines
(principally in Canada, South Africa, and the former USSR) may also have
high exposures.
Beryllium (choice B) is used in the nuclear industry and in x-ray tubes, and
was formerly used in ceramics, metallic alloys, and fluorescent lights.
Most significant coal (choice C) exposures occur in coal miners.
Kaolin (choice D) is a component of clay dust.
Silica (choice E) is found in sand and glass, and significant exposures can
be seen in individuals working in environments where small particles of these
materials may become aerosolized.

Question 4 of 4

In addition to predisposing for pulmonary fibrosis and bronchogenic carcinoma,
this patient's disease is associated which of the following?
/ A. Basal cell carcinoma
/ B. Germ cell tumor
/ C. Hemangioma
/ D. Mesothelioma
/ E. PIeomorphic adenoma


Explanation - Q: 1.4 Close

The correct answer is D. Asbestos exposure is also linked to late (often
after 20 years) development of the rare tumor, malignant mesothelioma. The
amphibole forms of asbestos appear to be much more likely to induce
mesothelioma than does chrysotile, and some authors have even speculated
that the rare cases of mesothelioma in persons with predominant exposure
to the serpentine form of asbestos may have been actually related to trace
exposures to the amphibole forms. In contrast, all types of asbestos can
cause pleural plaques, pulmonary fibrosis, and lung cancer. None of the
other lesions listed in the choices have ties to asbestos.
Basal cell carcinoma (choice A) is a form of skin cancer.
Germ cell tumors (choice B) usually occur in testes and ovary.
Hemangioma (choice C) is a benign tumor of blood vessels.
Pleomorphic adenoma (choice E) is a salivary gland tumor.






A 70-year-old man is seen by his family practice physician during a routine office
visit. The man complains of not feeling well for the last three
months. Further questioning reveals that the patient has a chronic, unproductive
cough that he attributes to an old smoking history. Physical
examination is notable for a 15-pound weight loss since the last office visit three
months previously. A multinodular infiltrate is seen in the lung
field behind and above the right clavicle.

Question 1 of 5

The patient is injected intradermally with PPD. 3 days after the injection, there is
a 13-mm diameter area of induration at the injection site. This
reaction is an example of which of the following types of immune response?
/ A. Type l hypersensitivity
/ B. Type ll hypersensitivity, cytotoxic subtype
/ C. Type ll hypersensitivity, noncytotoxic subtype
/ D. Type lll hypersensitivity
/ E. Type IV hypersensitivity

Explanation - Q: 2.1 Close

The correct answer is E. This patient has a positive PPD test, as indicated
by an area of induration greater than or equal to 10 mm. PPD is a purified
protein derivative of tuberculin, so this finding indicates that the patient has
tuberculosis. The PPD reaction is an example of type IV hypersensitivity,
also known as delayed-type hypersensitivity.
Type I hypersensitivity (choice A) is immediate hypersensitivity, and can be
seen for example, in hay fever and atopic dermatitis.
Cytotoxic type II hypersensitivity (choice B) can be seen in autoimmune
hemolytic anemia and transfusion reactions.
Noncytotoxic type II hypersensitivity (choice C) can be seen in myasthenia
gravis and Graves disease.
Type III hypersensitivity (choice D) is due to immune complex deposition,
and can be seen in systemic lupus erythematosus and rheumatoid arthritis.
Question 2 of 5
Which of the following principally mediates this form of hypersensitivity?
/ A. Cytotoxic T cells, TH1 cells, and macrophages
/ B. IgE, basophils, and mast cells
/ C. IgG and complement
/ D. IgG, IgM, neutrophils, and macrophages
/ E. IgG, IgM, neutrophils, macrophages, and natural killer cells

Explanation - Q: 2.2 Close

The correct answer is A. Cytotoxic T lymphocytes, T helper 1 lymphocytes,
and macrophages mediate the delayed-type hypersensitivity reaction, which
develops in response to viral, fungal, and intracellular bacterial antigens.
Antibodies do not play a significant role in this type of response.
IgE, basophils, and mast cells (choice B) are involved in the production of
type I hypersensitivity.
IgG and complement (choice C) is a mechanism of toxicity in type II
cytotoxic hypersensitivity.
IgG, IgM, neutrophils, and macrophages (choice D) may be involved in types
II or III hypersensitivities.
IgG, IgM, neutrophils, macrophages, and natural killers (choice E) may be
involved in types II or III hypersensitivities.

Question 3 of 5
The strongest definitive identification of the pathogen responsible for this
patient's disease would be provided by a positive result on which of
the following biochemical tests?
/ A. Arylsulfatase
/ B. Heat-stable catalase
/ C. Niacin
/ D. Nitrate reductase
/ E. Urease

Explanation - Q: 2.3 Close

The correct answer is C. Mycobacterium tuberculosis is the only member of
the genus Mycobacterium that is a producer of niacin.
Mycobacterium fortuitum and M. chelonae produce arylsulfatase (choice A),
but M. tuberculosis does not.
Heat-stable catalase (choice B) is produced by Mycobacterium kansasii, M.
avium intracellulare, and M. fortuitum, but not by M. tuberculosis.
Mycobacterium tuberculosis, M. kansasii, and M. fortuitum produce nitrate
reductase (choice D), however niacin is more definitive for the identification
of M. tuberculosis.



Question 4 of 5
The mass lesion in the patient's lung is evaluated with fiberoptic bronchoscopy
with transbronchial biopsy. Sputum collected after the
procedure demonstrates acid-fast bacteria with a "beaded" appearance. Giant
cells found in the biopsy material would be likely to express
which of the following membrane markers?
/ A. CD4
/ B. CD8
/ C. CD14
/ D. CD16
/ E. CD19

Explanation - Q: 2.4 Close

The correct answer is C. CD14, the endotoxin receptor, is a standard
marker for macrophages and cells of their lineage. The giant cells in a TB
granuloma are histiocytes, which are modified macrophages, so they would
bear macrophage cell markers.
CD4 (choice A) is a cell marker for helper T lymphocytes that would be
found in the halo of lymphocytes surrounding the Langerhans giant cells, but
would not be on the giant cells themselves.
CD8 (choice B) is a cell marker for cytotoxic T lymphocytes that would be
present in the halo of lymphocytes surrounding the giant cells, but would not
be on the giant cells themselves.
CD16 (choice D) is a cell marker for NK cells, which would not be on the
giant cells in a TB granuloma.
CD19 (choice E) is a cell marker for B-lymphocytes, which are not likely to
be present in a TB granuloma, a reaction mediated exclusively by cell-
mediated immunity.


Question 5 of 5

The principal drug recommended for treatment of this patient's disease targets
which of the following molecules?
/ A. Arabinogalactan
/ B. Dihydrofolate reductase
/ C. Dihydropteroate synthetase
/ D. Mycolic acid
/ E. Peptidoglycan


Explanation - Q: 2.5 Close

The correct answer is D. The mainstay of therapy for tuberculosis is
treatment with drugs such as isoniazid, rifampin, ethambutol, ethionamide,
pyrazinamide, and streptomycin. Both isoniazid and ethionamide target the
mycolic acid molecules in the mycobacterial cell wall. A single drug, usually
isoniazid, can be used for chemoprophylaxis and when clinical disease is
absent. Multiple drugs are usually used in other settings. Mycobacterium
tuberculosis resistant to all first-line drugs is being seen with increasing
frequency among AIDS patients.
Arabinogalactan (choice A) is the molecule targeted by ethambutol, which is
a drug added to the anti-mycobacterial regimen only when drug-resistant
strains are implicated.
Dihydrofolate reductase (choice B) is the molecule targeted by trimethoprim,
which is not the principal drug recommended for treatment of tuberculosis.
Dihydropteroate synthetase (choice C) is the molecule targeted by dapsone,
which is the drug of choice for Mycobacterium leprae, not M. tuberculosis.
Peptidoglycan (choice E) is the molecule targeted by the penicillins,
vancomycin, and cycloserine.

A 60-year-old man presents to the emergency department complaining of
shortness of breath, cough, and copious sputum production. He
states that he has been coughing for years, and has had increased sputum
production for several months each year. On examination, he is
obese, afebrile, cyanotic, and in acute distress. Coarse rales are auscultated
bilaterally at the lung bases. He smokes two packs of cigarettes
a day and has a seventy-five pack-year smoking history. A chest x-ray film
appears normaI, except for slightly enlarged lung fields.

Question 1 of 5

Which of the following is the most likely diagnosis?
/ A. Chronic bronchitis
/ B. Emphysema
/ C. Myocardial infarction
/ D. Pneumonia
/ E. Pulmonary embolus

The correct answer is A. This patient has findings classic for the "blue bloater"
of chronic bronchitis. Patients with chronic bronchitis have excessive
tracheobronchial mucus production sufficient to cause cough with expectoration
for at least three months of the year for more than two consecutive years. "Blue
bloaters" are named for their obese body habitus, copious sputum production,
and cyanotic episodes. This condition may occur initially without airway
obstruction, but eventually, most patients progress to obstructive disease.
Patients with emphysema (choice B) represent another form of COPD. They are
known as "pink puffers" because they do not become cyanotic until they
decompensate. They display a thin body habitus and belabored breathing. This
patient is not consistent with the pink puffer of emphysema.
This patient does not have the classic findings for myocardial infarction (choice
C), which include: chest pressure or pain, shortness of breath, and/or pain that
radiates to the jaw or left arm.
It is unlikely that this patient has pneumonia (choice D). Patients with pneumonia
have cough with purulent sputum production, but they are usually febrile and
have chest x-ray opacities.
While the diagnosis of pulmonary embolus (choice E) is elusive due to its varied
presentations, it is unlikely that this patient has a pulmonary embolus. Findings
for PE include pleuritic chest pain, shortness of breath, hemoptysis, and a history
of calf pain indicative of deep vein thrombosis.
Question 2 of 5
As this patient waits in the emergency department, his condition begins to
deteriorate. He turns increasingly blue and an arterial blood gas is
drawn. His PO2 is 45 mm Hg, which under normal conditions means that his
hemoglobin would be 75% saturated. Which of the following
mechanisms could cause a hemoglobin saturation of less than 75% at this pO2?
/ A. Decreased 2,3-DPG Ievels
/ B. Decreased hemoglobin
/ C. Decrease in body temperature
/ D. Decreased PCO2
/ E. Decreased serum pH

Explanation - Q: 3.2 Close

The correct answer is E. A decrease in pH (an increase in H
+

concentration) decreases the affinity of hemoglobin for O
2
. This facilitates
unloading of oxygen from hemoglobin to the tissues.
Choices A, C, and D increase the affinity of hemoglobin for O
2
, and thus
cause the Hb to retain the O
2
.
Choice B is a distracter.

Question 3 of 5
Why must care be exercised when administering O2 to this patient?
/ A. Administering O2 washes out alveolar CO2 and inhibits respiration
/ B. Chronic hypoxia alters the blood-brain barrier such that CO2 cannot diffuse
into the medullary apneustic center
/ C. Chronic hypoxia induces atrophy in the dorsal respiratory group in the
medulla
/ D. Increased PO2 worsens CO2 retention by decreasing respiratory drive
/ E. O2 is acutely toxic to the chronically hypoxic alveolar epithelium

Explanation - Q: 3.3 Close

The correct answer is D. In a patient with normal respiratory function, the
PCO
2
and pH of cerebrospinal fluid drives the respiratory center in the
medulla oblongata. As serum PCO
2
rises, increased CO
2
diffuses across the
blood-brain barrier. When this CO
2
is buffered with the high HCO
3
-
of
cerebral spinal fluid, the concentration of H
+
rises accordingly. The
chemosensitive cells in the medulla's respiratory center respond to this
localized decrease in pH by stimulating ventilation. The pH in the CSF
returns to nearly normal, more quickly than the renal compensation of the
arterial pH, which takes 2-3 days. Patients with chronic CO
2
retention will
have an abnormally low ventilation for their PCO
2
because the pH of their
CSF is nearly normal. Therefore, in patients with chronic CO
2
retention,
arterial hypoxemia becomes their primary ventilatory stimulus. When the
patient is given supplemental O
2
, the PO
2
rises, and the hypoxic stimulation
disappears, and respiration can become markedly depressed. The PCO
2
-
dependent respiratory drive does not revert immediately, and thus the patient
hypoventilates and retains CO
2
, which may precipitate coma, stupor, or
death.
Administering O
2
at high flow rates (choice A) may wash out alveolar CO
2
,
but this is not the mechanism for hypoventilation of the chronically hypoxic
patient.
Chronic hypoxia does not alter the diffusing capacity of the BBB (choice B).
Chronic hypoxia does not cause the respiratory center to atrophy (choice C).
Chronically high O
2
concentrations can damage the alveolar epithelium, but
in the acute setting, it does not alter its diffusion capabilities (choice E).


Question 4 of 5
The mucus seen in this patient is derived from which of the following cell types?
/ A. AIveolar macrophages
/ B. Goblet cells
/ C. Neuroendocrine cells
/ D. Type l pneumocyte
/ E. Type ll pneumocyte

Explanation - Q: 3.4 Close

The correct answer is B. In chronic bronchitis, goblet cell hyperplasia is
seen in the airways. This hyperplasia, caused by chronic irritation (usually by
tobacco smoke), results in increased mucus secretion and formation of
mucus plugs. These mucus plugs obstruct the airways, and are responsible
for part of the obstructive component of chronic bronchitis. Some degree of
loss of elastic recoil of the airways is also seen and further adds to the
obstructive component of the disease.
Alveolar macrophages (choice A) do not secrete mucus. They have a
phagocytic function.
Neuroendocrine cells (choice C) are present in the respiratory tract. They
are the cell of origin for oat cell carcinomas, and the neuroendocrine
capabilities of these cells become evident with the paraneoplastic syndromes
seen in this high-grade carcinoma.
Type I pneumocytes (choice D) comprise the majority of alveolar epithelium.
They provide the majority of the surface area on which gas exchange occurs.
Type II pneumocytes (choice E) secrete the alveolar surfactant that allows
alveoli of different diameters to inflate at the same pressure.

Question 5 of 5
Which of the following spirometry profiles would most likely be seen in this
patient?
/ A. Decreased TLC, decreased FEV1
/ B. Decreased TLC, decreased RV
/ C. Decreased TLC, increased FEV1
/ D. EIevated TLC, decreased FEV1
/ E. NormaI TLC, decreased FEV1

Explanation - Q: 3.5 Close

The correct answer is E. Patients with chronic bronchitis tend to have
normal TLC and decreased FEV1. Patients with chronic bronchitis do not
typically have the increased TLC that their counterparts with emphysema
have. They may however, have a modestly increased residual volume due to
air trapping distal to mucus plugs. Similarly these mucus plugs obstruct the
airways, creating the obstructive component of chronic bronchitis. Hence a
decreased FEV1 is noted.
Choices A and B are not typical of any common pulmonary disorder.
Choice C is consistent with restrictive lung disease. The increased
fibroelastic elements in the lung parenchyma decrease the lung capacity,
while allowing the airways to remain open at increasingly lower pressures.
Thus, a decreased TLC is seen with an increased FEV1.
Choice D is consistent with emphysema. These patients have destruction of
the fibroelastic elements of the lung. As a result, the TLC increases. With
decreased elastic tissue, the airways collapse at higher airway pressures,
and a decreased FEV1 is seen.


A 72-year-old woman presents to the emergency department with cough, fever,
and shortness of breath. The woman lives alone at home, but
spends part of each day shopping and riding public buses. Approximately 4 days
previously she had developed an upper respiratory infection.
Approximately 2 days ago, she abruptly became much more ilI, and her
symptoms started worsening, beginning with a single, Iong, shaking
chilI. Since that time, she has had fever, pain with breathing, cough, and
dyspnea. She decided to come to the emergency department when
her temperature at home was 103.4 F. In the emergency department, her
temperature is 39.9 C (103.8 F), blood pressure is 90/50 mm Hg,
pulse is 120/min, and respirations are 30/min. No breath sounds are heard over
her lower left lung field, but they can be heard at other sites.

Question 1 of 7

A chest x-ray film would be most likely to demonstrate which of the following:
/ A. A single roughly ovoid white area
/ B. Complete whitening over one lobe of her lungs
/ C. Marked dilation and elongation of bronchial spaces
/ D. Multiple small spotty areas of white found primarily near the bronchi
/ E. No obvious radiologic changes

Explanation - Q: 4.1 Close

The correct answer is B. This patient has lobar pneumonia, which is seen
radiologically as a complete whitening of one or more lobes of the lungs.
Choice A describes the appearance of a mass lesion of the lung, such as
tumor or tuberculosis
Choice C describes the radiologic changes accompanying bronchiectasis.
Choice D describes the radiologic appearance of bronchopneumonia, rather
than lobar pneumonia.
While very early in lobar pneumonia, no obvious radiologic changes may be
seen (choice E), this patient's lack of breath sounds over the lower left lung
field indicates that her pneumonia is past this very early stage.

Question 2 of 7
Gram's stain of a smear from a sputum sample demonstrates gram-positive
lancet-shaped diplococci in short chains. Which of the following
would most likely be identified after culturing?


Explanation - Q: 4.2 Close

The correct answer is E. Streptococcus pneumoniae is the most commonly
identified causative organism for bacterial pneumonia. Up to two-thirds of
bacteremic community-acquired pneumonias are due to this organism. 5-
25% of healthy individuals carry S. pneumoniae in their pharynx. The classic
description of S. pneumoniae on Gram's stain is that given in the question
stem. S. pneumoniae can be verified by the Quellung reaction, or
counterimmunoelectrophoresis to determine serotypes of isolated strains or
for case detection using sputum specimens (there are more than 80 distinct
serotypes based on studies of capsular antigens).
Chlamydia pneumoniae(choice A) is not identified on the basis of Gram's
stain, but rather, can be seen with Giemsa or immunofluorescence.
Haemophilus influenzae(choice B) and Legionella pneumophila(choice C)
are gram-negative rods.
Staphylococcus aureus(choice D) is also a gram-positive coccus, but the
classic description of this organism typically includes a reference to "grape-
like clusters."



Question 3 of 7
Which of the following is thought to contribute to the ability of gram-positive
organisms to retain the Gram's stain during the decolorization
process?
/ A. Large periplasmic space
/ B. Presence of capsule
/ C. Presence of outer membrane
/ D. Presence of pili
/ E. Thick peptidoglycan layer
Explanation - Q: 4.3 Close

The correct answer is E. The peptidoglycan layer in the cell wall of gram-
positive organisms is much thicker than that in gram-negative organisms,
and is thought to contribute to the gram-positive staining reaction.
A large periplasmic space (choice A) and an outer membrane (choice C)
are characteristics of gram-negative cell walls.
Capsules (choice B) can be seen in both gram-positive and gram-negative
species.
Pili (choice D) are small hair-like structures that are most often seen in
gram-negative organisms.


Question 4 of 7
Which of the following is a characteristic feature on culture of this patient's
organism?
/ A. AIpha-hemolytic colonies inhibited by optochin on blood agar and lysed by
bile
/ B. Beta-hemolytic colonies that are bacitracin-resistant on blood agar
/ C. Beta-hemolytic colonies that are inhibited by bacitracin on blood agar
/ D. Catalase-negative organisms that hydrolyze esculin in 40% bile and 6.5%
NaCI
/ E. Catalase-positive, coagulase-positive organisms that cause beta-hemolytic,
yellow colonies on blood agar

Explanation - Q: 4.4 Close

The correct answer is A. These findings are used to identify Streptococcus
pneumoniae. Optochin sensitivity is used to differentiate the viridans
streptococci (resistant) from S. pneumoniae (sensitive). Another test used to
identify S. pneumoniae is the Quellung reaction. The only medically
important optochin-sensitive organism that gives a positive Quellung reaction
is S. pneumoniae.
Choice B is seen with Streptococcus agalactiae.
Choice C is seen with Streptococcus pyogenes.
Choice D is seen with Enterococcus faecalis.
Choice E is seen with Staphylococcus aureus.
Question 5 of 7
At this stage of the disease (two days following onset of first symptoms), and
before treatment is started, what are the expected
histopathologic findings in the affected lung parenchyma?
/ A. Advanced organization with macrophages and fibroblasts
/ B. Interstitial lymphomonocytic inflammatory infiltration
/ C. Intra-alveolar purulent exudate admixed with erythrocytes
/ D. Resolution of inflammatory exudate with semifluid debris
/ E. Vascular congestion without intra-alveolar exudation

Explanation - Q: 4.5 Close

The correct answer is C. Acute lobar pneumonia has classically been
subdivided into four pathologic stages. The initial stage of congestion(choice
E) lasts less than 24 hours and is characterized by engorgement of vessels,
with intra-alveolar fluid containing few cells, but often, numerous bacteria.
Over the following few days, an exuberant intra-alveolar exudate of red cells,
neutrophils, and fibrin develops, heralding the stage of red hepatization
(choice C; so-called because the lung resembles liver at this stage). During
the last half of the first week of illness, the red cells begin to break down, but
a fibrinous exudate remains in the alveoli; this is the stage of gray
hepatization. If death does not supervene, resolution (choice D) occurs in
the second week in untreated cases, with digestion of the exudate to leave
semifluid debris that are phagocytized, or coughed up. In some cases, the
exudate, rather than resolving in this manner, undergoes further organization
(choice A).
An interstitial lymphomonocytic infiltrate (choice B) is characteristic of
primary atypical pneumonia, caused by Mycoplasma or viruses.

Question 6 of 7
The patient's infection is treated with parenteral penicillin, to which she promptly
responds. This drug acts by which of the following
mechanisms?
/ A. Inhibits bacterial protein synthesis
/ B. Inhibits growth of cell walls
/ C. Interferes with bacteriaI DNA synthesis
/ D. Interferes with folate metabolism
/ E. Punches holes in cell membranes
Explanation - Q: 4.6 Close

The correct answer is B. The penicillins and cephalosporins both inhibit cell
wall synthesis. Penicillin G is the preferred antibiotic for sensitive strains;
25% of strains are resistant and can be treated with cephalosporins,
erythromycin, and clindamycin. Pneumococcal pneumonia can be prevented
in a number of cases; the pneumococcal vaccine contains 23 specific
polysaccharide antigens found in 85-90% of the serious pneumococcal
infections.
Inhibitors of bacterial protein synthesis (choice A) include aminoglycosides,
chloramphenicol, macrolides, and tetracycline.
Inhibitors of nucleic acid synthesis (choice C) include fluoroquinolones and
rifampin.
Inhibitors of folic acid synthesis (choice D) include sulfonamides,
trimethoprim, and pyrimethamine.
Agents that disrupt cell membranes (choice E) include azole and polyene
antifungal agents.

Question 7 of 7
A few minutes after the patient receives the antibiotic therapy, she develops an
adverse reaction characterized by an itchy skin eruption and
acute respiratory distress. This reaction is most likely attributable to which of the
following mechanisms?
/ A. Antibody-mediated cellular dysfunction
/ B. Complement-dependent reaction
/ C. Delayed-type hypersensitivity
/ D. IgE-mediated mast cell degranulation
/ E. Immune-complex deposition

Explanation - Q: 4.7 Close

The correct answer is D. This patient has had an acute allergic reaction to
the antibiotic, which can be further classified as an immediate
hypersensitivity reaction. These reactions are mediated in a way similar to
hay fever, with preformed IgE binding to the antibiotic antigen, and then the
antigen-antibody complex triggering degranulation of mast cells with release
of histamine and other active substances. These substances then cause
both the itchiness of the skin and the bronchospasm that caused the
respiratory distress.
An example of a disease caused by antibody-mediated cellular dysfunction
(choice A) is Graves disease, in which antibodies to the TSH receptor cause
a non-physiologic chronic stimulation of thyroid epithelial cells.
An example of a complement-dependent reaction (choice B) is hemolytic
disease of the newborn.
An example of delayed-type hypersensitivity (choice C) is the tuberculin
reaction.
An example of an immune complex deposition disease (choice E) is the
vasculitis seen in systemic lupus erythematosus.


A 14-year-old girl receives a bone marrow transplant as part of her treatment for
acute lymphoblastic lymphoma. During the period of profound
immunosuppression before the marrow engrafts, she develops nonproductive
cough, fever, mild hemoptysis, and pleuritic chest pain. A plain
chest x-ray film shows a pleuraI-based wedge-shaped lesion with focal
cavitation. Open chest lung biopsy reveals necrosis and hemorrhage.
Septate fungal forms with dichotomous 45-degree branching are seen in the
necrotic areas and involving the walls of several blood vessels.
When the fungus is cultured, it is found to be a monomorphic fungus.

Question 1 of 6

Which of the following is a monomorphic filamentous fungus?


Explanation - Q: 5.1 Close

The correct answer is A. The fungi that cause "deep infections" in humans
are subdivided into the dimorphic forms (which, depending upon
temperature, can be either yeast forms or hyphal forms) and the
monomorphic forms (which grow in the same general form at different
temperatures). Of the fungi listed, only Aspergillus is monomorphic.
Aspergillus is a common saprophytic mold found on decaying material in the
environment throughout the world. It can cause human diseases, including
allergic bronchopulmonary aspergillosis (which is essentially an allergic
reaction to inhaled Aspergillus conidia or spores), fungus ball (in which the
Aspergillus grows without invading in a preexisting cavitary lesion of the
lung), invasive aspergillosis (including pneumonia, meningitis, and other
systemic infections), and cellulitis. Invasive aspergillosis is most often seen in
severely immunocompromised patients with severe neutropenia, notably
including those with a history of transplantation, chronic granulomatous
disease, and leukemia. This patient's presentation is typical for invasive
pulmonary aspergillosis. The other fungi listed (choices B, C, D, and E) are
all dimorphic.

Question 2 of 6
Two commonly encountered invasive fungi are Aspergillus and Candidia. In
tissue specimens, which of the following features is often helpful in
distinguishing these organisms?

/ A. Aspergillus has both hyphae and pseudohyphae
/ B. Aspergillus has budding yeasts
/ C. Aspergillus has germ tubes
/ D. Aspergillus shows dichotomous branching
/ E. Aspergillus shows generally obtuse angles of branching

Explanation - Q: 5.2 Close

The correct answer is D. Aspergillus has hyphae, but not pseudohyphae,
budding yeasts, or germ tubes. The branching is dichotomous and at an
acute (often about 45 degrees) angle (opposite of choice E).
Characteristics to look for with Candida include both pseudohyphae and true
hyphae (choice A), budding yeasts (choice B), and occasionally germ tubes
(choice C, better seen with some culture methods).


Question 3 of 6

Involvement by this patient's infection of which of the following is considered to
have the worst prognosis?
/ A. Brain
/ B. Lung
/ C. Middle ear
/ D. Sinus
/ E. Skin

Explanation - Q: 5.3 Close

The correct answer is A. Cerebral aspergillosis is a feared complication of
other forms of aspergillosis because most patients die despite appropriate
antifungal therapy. Of the other sites listed in the question choices, true
invasive infection of the lung (choice B), as opposed to a fungus ball, is
considered the most serious site of infection, though less so than infection of
the brain.
Middle ear and sinus involvement (choices C and D) become clinically
worrisome when dissemination or extension to the brain occurs.
Skin involvement (choice E) is worrisome if dissemination occurs.

Question 4 of 6
Which of the following is the most appropriate pharmacotherapy for this patient?
/ A. Amphotericin B
/ B. CIotrimazole
/ C. FIuconazole
/ D. FIucytosine
/ E. Griseofulvin

Explanation - Q: 5.4 Close

The correct answer is A. Amphotericin B is the mainstay of therapy of
invasive aspergillosis. This medication has a wide fungicidal spectrum and
remains either the drug of choice or the co-drug of choice for severe
infections caused by Aspergillus, Candida, Cryptococcus, Histoplasma,
Mucor, and Sporothrix. Alternative antifungal agents that are not the
preferred therapy but do have some activity against aspergillus include
itraconazole, flucytosine (choice D), and voriconazole.
Clotrimazole (choice B) is used topically for candidal and dermatophytic
infections.
Fluconazole (choice C) is the drug of choice for esophageal and invasive
candidiasis and coccidioidomycosis, and is used for prophylaxis and
suppression in cryptococcal meningitis.
Flucytosine (choice D) is also synergistic with amphotericin B in candidiasis
and cryptococcosis.
Griseofulvin (choice E) is given orally, but is active only against
dermatophytes.

Question 5 of 6

Which of the following best describes the mechanism of action of the most
appropriate medication for this patient's disease?
/ A. Forms pores in fungal membranes
/ B. Inhibits the demethylation of lanosterol
/ C. Inhibits squalene epoxidase
/ D. Inhibits thymidylate synthase
/ E. Interferes with the synthesis of ergosterol

Explanation - Q: 5.5 Close

The correct answer is A. The polyene anti-fungal amphotericin B is an
amphoteric compound, with both polar and non-polar structural elements,
that interacts with ergosterol in fungal membranes to form artificial "pores,"
which disrupt membrane permeability. Resistant fungal strains are those that
appear to have low ergosterol content in their cell membranes. Amphotericin
B is given by slow IV infusion and penetrates poorly into the central nervous
system (intrathecal injections can be used). It has a half-life of more than 2
weeks and is removed by both metabolism and renal elimination.
Antifungal agents in the azole class (including ketoconazole, fluconazole,
and itraconazole) interfere with the synthesis of ergosterol (choice E) by
inhibiting the P450-dependent 14-alpha-demethylation of its precursor
molecule, lanosterol (choice B).
Flucytosine is activated by fungal cytosine deaminase to 5-fluorouracil (5-
FU), which can be incorporated into fungal RNA and can be also used to
form 5-fluorodeoxyuridine monophosphate, which in turn inhibits thymidylate
synthase (choice D) and the synthesis of thymine.
The anti-dermatophytic drug terbinafine inhibits squalene epoxidase (choice
C) and thus decreases ergosterol synthesis (choice E).
Question 6 of 6

Toxicity to which of the following organs is most likely to limit the administration
of the most appropriate medication for this patient's disease?
/ A. Brain
/ B. Heart
/ C. Kidney
/ D. Liver
/ E. Lung







Explanation - Q: 5.6 Close

The correct answer is C. Nephrotoxicity is most likely to be dose-limiting,
and may force amphotericin treatment to end prematurely. This is an
important problem, since we have so few drugs with wide spectrum activity
against invasive fungal organisms. An amphotericin B lipid complex (ABLC,
Abelcet) that may be less nephrotoxic in those patients who cannot tolerate
conventional amphotericin is available. Amphotericin B also causes infusion-
related reactions including fever, chills, nausea, vomiting, headache,
generalized malaise, hypotension, and arrhythmias.
Major problems encountered less commonly than renal toxicity include
anaphylaxis, generalized pain, bone marrow toxicity, rash, a variety of
cardiac problems including cardiac arrest (choice B), liver failure (choice D),
pulmonary edema (choice E), and convulsions (choice A).


A 45-year-old man is admitted to the hospital for the evaluation of diplopia,
weakness of his lower extremities, and gait difficulties. During the
interview, the patient reveals that has been forcing himself to vomit after almost
every meal over the last 6 weeks. He denies associated eye
pain and discomfort, headache, or dysphagia. He has never experienced such
symptoms in the past. Examination of the patient demonstrates
slight disorientation, vertical nystagmus worse on downgaze, diffuse weakness of
the lower extremities, bilateral dysmetria, and hypothermia.
Laboratory analyses reveal mild dehydration and hypokalemia.

Question 1 of 5

Which of the following is the most likely diagnosis?
/ A. Anticonvulsant intoxication
/ B. Benedict syndrome
/ C. Delirium tremens
/ D. Wernicke encephalopathy
/ E. Wernicke-Korsakoff syndrome
Explanation - Q: 1.1 Close

The correct answer is D. Wernicke encephalopathy is caused by a
nutritional deficiency of thiamine. Classic symptoms and signs include
"Wernicke's triad": acute mental confusion, ataxia, and ophthalmoplegia,
although not all the patients present with all of these. A common
misconception about Wernicke encephalopathy is that it is seen exclusively
in alcoholics. Prolonged vomiting and malnutrition, eating disorders, hunger
strikes, prolonged intravenous feeding, and malabsorption syndrome can
also be a potential cause of thiamine deficiency. Mental confusion is
characterized by impaired awareness, spatial disorientation, and inability to
concentrate. Ocular abnormalities are the hallmarks of this disease.
Horizontal or vertical nystagmus and paralysis of lateral rectus muscles are
common. The wide-based ataxic gait results from cerebellar dysfunction,
either alone or in combination with vestibular dysfunction.
Lateral nystagmus, ataxia, and drowsiness, and a history of drug abuse may
be associated with phenytoin intoxication (choice A).
Benedict syndrome (choice B) is a brain stem syndrome of crossed
oculomotor paralysis, with tremor and hemiparesis.
Tremors, hallucinations, severe agitation, and seizures are the features of
delirium tremens (choice C).
If persistent memory and learning deficits are present in addition to
Wernicke's triad, the complex disorder is termed Wernicke-Korsakoff
syndrome (choice E).


Question 2 of 5
The patient's status has improved after an administration of a vitamin. This
vitamin plays a central role in the metabolism of which of the
following compounds?
/ A. Amino acids
/ B. Carbohydrates
/ C. Cholesterol
/ D. DNA
/ E. Fatty acids

Explanation - Q: 1.2 Close

The correct answer is B. Thiamine (vitamin B
1
) is converted intracellularly
to its active form, thiamine pyrophosphate, which is an essential cofactor in
intermediate carbohydrate metabolism. Thiamine acts as a coenzyme for
enzymes involved in the tricarboxylic acid cycle, which plays a critical role in
the production of energy from food, and the pentose-phosphate pathway,
which produces an important intermediate, ribose-5-phosphate, required for
the synthesis of ATP, GTP, DNA, RNA, and NADPH. The brain is particularly
vulnerable to thiamine deficiency because it relies exclusively on
carbohydrates for immediate energy demands. The selective vulnerability of
certain structures accounts for the specific clinical manifestations of
Wernicke encephalopathy.
The active form of vitamin B6, pyridoxal phosphate, is a coenzyme in several
types of reactions, mostly involving amino acid metabolism (choice A).
Pantothenic acid is a component of coenzyme A (CoA), and thus is essential
in the synthesis of cholesterol (choice C).
The synthesis of DNA (choice D) from its precursors is dependent on folate
coenzymes.
Biotin is important in the synthesis of fatty acids (choice E).


Question 3 of 5

The physiologically active form of this vitamin acts as a coenzyme for which of
the following enzymes?
/ A. GIycogen phosphorylase
/ B. L-methylmalonyI-CoA mutase
/ C. Methionine synthase
/ D. Pyruvate carboxylase
/ E. Pyruvate dehydrogenase
Explanation - Q: 1.3 Close

The correct answer is E. Thiamine pyrophosphate (TPP), the
physiologically active form of thiamine, is required for the activity of pyruvate
dehydrogenase (PDH). PDH catalyzes oxidative decarboxylation of pyruvate
to acetyl-CoA, regulating entry into the citric acid cycle for metabolites
leaving glycolysis. The PDH complex is comprised of 3 separate enzymes:
pyruvate dehydrogenase (decarboxylase), dihydrolipoamide transacetylase
and dihydrolipoamide dehydrogenase. Besides TPP, the complex also
requires CoA, NAD, FAD, and lipoic acid as coenzymes. The mechanism of
TPP action includes formation of a carbanion that can attack the electron-
deficient keto carbon of pyruvate. In addition, TPP functions as a coenzyme
for alpha-ketoglutarate dehydrogenase and transketolase.
Pyridoxal phosphate serves as a coenzyme for glycogen phosphorylase
(choice A).
Vitamin B12 acts as a cofactor of L-methylmalonyl-CoA mutase (choice B),
which catalyzes conversion of L-methylmalonyl-CoA to succinyl-CoA.
Methionine syntase (choice C) activation leads to synthesis of methionine
from homocysteine with folic acid as a coenzyme.
Biotin is required for the activation of pyruvate carboxylase (choice D), a
crtical enzyme in the pathway of gluconeogenesis.
Question 4 of 5
Which of the following would be the most appropriate initial therapy for this
patient?
/ A. Chlordiazepoxide
/ B. Cyanocobalamin
/ C. GIucose
/ D. Niacin
/ E. Thiamine

Explanation - Q: 1.4 Close

The correct answer is E. 50-100 mg of thiamine IV should be administered
promptly. Objective of thiamine therapy is to replenish vitamin stores. Thus,
50 mg IM qd of thiamine should be given until the patient resumes a normal
diet. Magnesium is also required for normal function of thiamine
pyrophosphate, so correction of magnesium deficiency should be the final
step in treating this patient, after thiamine replenishment is well underway.
10-30 mEq of magnesium per day and 100-200 mEq of potassium (or more,
depending of the severity of hypokalemia) would be necessary to achieve
optimum metabolic balance. Patient will need IV glucose solutions, but
thiamine administration should be begun prior to treatment with IV glucose.
Administration of glucose prior to thiamine will increase the metabolic needs
for thiamine, and hence will drastically worsen the thiamine deficiency.
Correction of dehydration and metabolic alkalosis, if any, would represent
additional steps in therapy of this patient.
Chlordiazepoxide (choice A) is a benzodiazepine that would be indicated for
treatment of delirium tremens, rather than Wernicke syndrome.
Pernicious anemia is caused by impaired absorption of cyanocobalamin
(choice B) because of the lack of intrinsic factor.
Glucose (choice C) infusion before thiamine administration may precipitate
Wernicke disease.
Pellagra is the systemic disease resulting from niacin deficiency (choice D).
Question 5 of 5
Pathologic examination of this patient's brain would most likely reveal damage to
which of the following structures?
/ A. Caudate nucleus and putamen
/ B. Midline brain structures
/ C. Motoneurons in the anterior horns and cranial motor nuclei
/ D. Posterior columns and posterior roots
/ E. Substantia nigra

Explanation - Q: 1.5 Close

The correct answer is B. Midline brain structures show significant damage
in patients with Wernicke encephalopathy. The medial thalamic and
hypothalamic structures, periaqueductal and oculomotor regions of the
midbrain, and the superior cerebellar vermis are commonly damaged in this
disease. Wernicke originally detected punctate hemorrhages in the gray
matter surrounding the third and fourth ventricles and aqueduct of Sylvius.
The lesions in the oculomotor areas do not show significant neuronal
destruction, which may be the explanation for the rapid improvement of the
symptoms with thiamine repletion. Pathologic findings in the superior vermis
involve degeneration of all layers of the cerebellar cortex.
Gross atrophy of the caudate nucleus and putamen (choice A), with
selective neuronal loss and astrogliosis is seen in Huntington disease.
Amyotrophic lateral sclerosis is a disorder of the anterior horn cells of the
spinal cord and the cranial motor nuclei (choice C).
Demyelination involving the posterior columns and inflammatory changes
with fibrosis of posterior roots (choice D) are typical findings in tabes
dorsalis.
Loss of pigmented dopaminergic neurons in the substantia nigra (choice E)
and the presence of Lewy bodies is characteristic of Parkinson disease.
A 57-year-old man is evaluated because of progressive memory problems and
language disturbance. Several months ago, he started noticing
increasing difficulty with driving, turning left instead of right and getting lost in his
own neighborhood. His family states that he would repeatedly
ask the same question and started forgetting easily, particularly recent events.
They also noted waxing and waning difficulties with speech, an
ataxic gait, uncontrollable trembling, and a tendency to doze off easily. There is
no family history of psychiatric illness, epilepsy, or neurological
or neurodegenerative disorders.
Question 1 of 6
Which of the following is the most likely diagnosis?
/ A. Creutzfeldt-Jakob disease (CJD)
/ B. Fatal familial insomnia
/ C. Gerstmann-Straussler-Scheinker syndrome
/ D. Kuru
/ E. Variant Creutzfeldt-Jakob disease (vCJD)

Explanation - Q: 2.1 Close

The correct answer is A. CJD is a degenerative disorder of the central
nervous system that is caused by accumulation of abnormally folded protein
(PrPsc) particles termed prions. Normal prion protein is termed PrPc
(cellular), while an abnormal, pathogenic isoform of the prion protein is
designated PrPsc. This accumulation may be initiated by germline mutations
in the PRNP gene (familial), by abnormal prion proteins from infected
sources (mostly, iatrogenic), or somatic mutation or spontaneous conversion
of PrPc to PrPsc (sporadic form). The sporadic form is the prevalent form,
accounting for more than 85% of human prion disease cases. It usually
presents in late middle age (50-75 yr), with rapidly progressive dementia,
ataxia, dysarthria, myoclonic fasciculations, somnolence, and eventually
death, usually following pneumonia, within a year of onset. MRI typically
shows bilateral areas of increased intensity, predominantly in the caudate
and putamen. On light microscopy, the pathologic hallmarks of CJD are
spongiform degeneration, astrogliosis and the lack of an inflammatory
response. Spongiform changes occur in the putamen, caudate nucleus,
cerebral cortex, thalamus, and cerebellum. The amyloid plaques that are
seen in about 10% of cases are histologically different from those seen in
scrapie or Kuru. There is no known effective therapy for treating or
preventing CJD, with the exception of the prevention of iatrogenic cases.
Fatal familial insomnia (choice B) is an autosomal dominant disease,
characterized by progressive insomnia, dysautonomia, and motor signs. The
mutation of the PrP gene occurs at codons 178 and 129 that code for
methionine. The pathology shows severe selective atrophy of the thalamus.
Gerstmann-Straussler-Scheinker syndrome (GSS, choice C) typically occurs
in the 4th-5th decade. It is characterized by cerebellar ataxia, and less
commonly, dementia. Pathological examination reveals widespread plaque
formation.
Kuru (choice D) is an epidemic transmissible spongiform encephalopathy
(TSE) that occurred in certain Fore tribes in Papua, New Guinea. Cerebellar
ataxia, shivering-like tremor, dysarthria, and death usually within the year are
the features of this disease. It was a result of a ritualistic consumption of the
brains of dead relatives (form of cannibalism). It was prevalent in women and
children, since adult men seldom ate ritually prepared brains. Since the
authorities banned this endocannibalism, the disease is slowly disappearing.
Variant CJD (choice E) differs from classic CJD in several ways. The
patients are younger (16-41 yr), the presenting features are often behavioral
changes, ataxia, and peripheral sensory disturbances (dementia develops
later). The time between the onset of symptoms and death is longer, and the
EEG pattern is not characteristic for CJD. Pathologically, characteristic
features are "florid" plaques, with the central core of PrP amyloid surrounded
by vacuoles in a pattern resembling a flower.
Question 2 of 6
Which of the following electroencephalographic (EEG) findings would most likely
be recorded in this patient?
/ A. AIpha rhythm
/ B. Paroxysmal diffuse outbursts of high voltage, fast rhythm with many spikes
/ C. Periodic sharp waves
/ D. Polyphasic sharp and slow complexes
/ E. Theta rhythm, K-complexes

Explanation - Q: 2.2 Close

The correct answer is C. The EEG in Creutzfeldt-Jakob disease is
characterized initially by biphasic or triphasic discharges, approximately 1-2
seconds apart. They are present during wakefulness and disappear during
sleep. Sometimes, a normal EEG pattern may be seen in these patients at
this stage. As the disease advances, the pattern becomes generalized and
synchronous with periodic, stereotypic 200-400 msec sharp waves that
occurs at intervals of 0.5-1.0 second, representing the EEG hallmarks of this
disease. The sharp waves typically react to external stimuli. Certain drugs,
e.g., barbiturates and benzodiazepines, may temporarily eliminate periodic
patterns.
An EEG rhythm with the frequency of 8-13 Hz and low amplitude (below
50V), that is most prominent over the parieto-occipital cortex with the eyes
closed is called alpha rhythm (choice A). It is indicative of relaxed
wakefulness in most normal individuals.
A characteristic fast rhythm with many spikes and paroxysmal diffuse
outbursts of high voltage (choice B) is seen in generalized tonic clonic
seizures (grand mal seizures).
Polyphasic, repetitive, sharp and slow complexes (choice D) of high voltage
that recur every 5-15 seconds are characteristic of subacute sclerosing
panencephalitis (SSPE).
Theta activity, sleep spindles and K complexes (choice E) are characteristic
of stage 2 of slow wave sleep.


Question 3 of 6
Which of the following would most likely inactivate the causative agent of this
disease?
/ A. Ethylene oxide
/ B. Formalin
/ C. 1M sodium hydroxide
/ D. Proteinase K
/ E. UItraviolet irradiation

Explanation - Q: 2.3 Close

The correct answer is C. Appropriate decontamination procedures minimize
the risk of CJD transmission. Since prions are resistant to most commonly
used routine procedures, it is necessary to use specific methods to inactivate
them. Prions resist inactivation by nucleases, treatment with psoralens,
divalent cations, acids, hydroxylamine, boiling, and metal ion chelators. The
methods of choice include 1M NaOH, 4.0 M guanidinium hydrochloride, or
sodium hypochlorite (2% free chlorine concentration). However, the latter two
can damage fabric and metal, and irritate the airways of laboratory workers.
Therefore, sodium hydroxide (for at least 1 hour contact time) has been
recommended. Using this method, over 99% of infectivity will be eliminated.
Because paraformaldehyde vaporization does not affect prion titers,
decontamination of biosafety cabinets must be performed using 1M NaOH
followed by 1N HCl, and rinsing with water.
Low temperature sterilization with ethylene oxide (choice A) will not
eliminate prions. Therefore, for surgical instruments or other items requiring
sterilization, autoclaving at 132 C for 4.5 hours is recommended.
Prions are not inactivated by formalin (choice B). This is very important for
pathological examinations or autopsies in which formalin-fixed tissues are
processed.
The normal prion protein (PrPc) isoform is sensitive to proteinase K (choice
D). In contrast, proteinase K removes only small number of amino acids from
the abnormal isoform, PRPsc.
Prions are resistant to inactivation by UV-irradiation at 254 nm (choice E).


Question 4 of 6
Which of the following agents is the most likely cause of this disease?
/ A. BoPrPsc
/ B. GSS prion
/ C. HuPrPsc
/ D. Kuru prion
/ E. OvPrPsc

Explanation - Q: 2.4 Close

The correct answer is C. Prions are proteinaceous infectious particles that
lack nucleic acid. They are responsible for the pathogenesis of TSE
(transmissible spongiform encephalopathy). The causative agent for CJD is
designated as HuPrPsc. Several features distinguish prions from viruses:
they can exist in multiple molecular forms, they are non-immunogenic, they
do not possess nucleic acid, and they consist of a single protein. Prions are
abnormally folded proteins, which infect their normally folded counterparts in
the brain, setting off a domino-like effect in which the normal proteins refold
into an abnormal form. This influence is continued even after the cell
undergoes division and new protein is made. Therefore, prions confer a
heritable phenotype that is based on a change in protein conformation alone
and has no underlying alteration in the DNA. Sometimes, "rogue" prions are
produced by genetic mutations. A normally folded, cellular PrPc form occurs
in the brains of all mammals so far studied. The gene for PrPc, which in
humans is called PRNP, is located on the short arm of chromosome 20, and
encodes a 35 kDa glycoprotein with two glycosylation sites. The amino acid
sequences of PrPc and PrPsc should be the same, because they are coded
by the same gene. PrPc is expressed on the cell membrane of neurons
attached by the glycoinositol phospholipid anchor. It is believed to be
involved in synaptic function, and it has been suggested that it can protect
the brain from dementia and other degenerative changes associated with
aging. The secondary structure of PrPc consists of three alpha helices and a
single beta-sheet. For unknown reasons, this secondary structure adopts an
alternative conformation (PrPsc) that is rich in beta-sheet structure. This
change in secondary structure leads to alterations in the physiochemical
properties, which results in greater resistance to protease action, more firm
membrane-anchoring, and formation of amyloid plaques. Once formed,
molecules in the misfolded conformation have a tendency to stimulate the
formation of additional PrPsc by post-translational recruitment of PrPc.
BoPrPsc (choice A) is involved in the pathogenesis of BSE (bovine
spongiform encephalopathy). The tridimensional folds of BoPrPsc and its
human counterpart are, in fact, practically identical. Differences occur in the
electrostatic surface charge distribution of the globular region.
Point mutations at a number of different codons of the prion protein gene are
associated with GSS disease (choice B). As a result of the mutations, a
substitution at the corresponding residues of the prion protein occurs.
Amyloidosis coexists with severe spongiform degeneration in patients with
the codon 102 mutations, and with neurofibrillary degeneration in patients
with mutations at codons 145, 198, and 217.
The Kuru prion (choice D) is transmissible in brain or eye tissue to primates,
including humans. Kuru prion is thought to be spread either by ingestion of
brain tissue or by self-inoculation into sores or conjunctiva. The agent is not
passed maternally or by mother's milk.
OvPrPsc (choice E) is a causative agent of scrapie, a degenerative brain
disease of sheep. The "sc" superscript is initially derived from the term
scrapie because scrapie is the prototypic prion disease.

Question 5 of 6
Which of the following would most likely transmit this disease?
/ A. BIood
/ B. Direct contact
/ C. Direct inoculation
/ D. Droplets
/ E. Semen

Explanation - Q: 2.5 Close

The correct answer is C. In contrast to viral diseases, the human prion
diseases are not communicable, but they are transmissible. Research has
determined that prions remain infectious in the environment for years. Prions
can be transmitted to other species, chimpanzees, and mice by inoculation of
infected brain tissue. When prions are introduced peripherally or through the
eye, the infection travels via the lymphoreticular system before it goes to the
central nervous system. Transmission requires direct inoculation past the
natural defensive barriers. In medical institutions, the transmission of CJD
has been reported when infectious material has been directly introduced into
patients via transplanted tissue (corneal transplants and dura mater grafts),
cadaver-derived growth hormone and gonadotropin, and contaminated
surgical instruments (iatrogenic CJD). During the last several years, new
cases of so-called new variant CJD (vCJD) were noted in young people in
Britain. Since substantial evidence has suggested that prions can only
accumulate at high levels in the brain, spinal cord, and retina, it is postulated
that these patients were infected by eating food contaminated with the
infective agent. As a result, specific types of bovine offal (particularly
homogenates of pooled bovine brains, used as binders for hamburgers and
sausages) has been banned for use in food for human consumption.
However, ingestion (other than cannibalism) is an inefficient route of
transmission of prions. Also, there is no direct evidence that the prion
responsible for bovine spongiform encephalopathy ("mad cow disease") is
infectious in humans. Therefore, infection of humans from eating the BSE
prion is unlikely, but remains a possibility.
The current understanding is that there is no conclusive evidence that
infectious prions circulate in blood or plasma (choice A). Therefore, the risk
of transmission of CJD among humans from the use of blood or biological
products of human origin remains theoretical.
The disease is not transmitted by direct contact (choice B). Even living with
someone who has CJD is not a risk factor for developing the disease.
There is no evidence of aerosol transmission (choice D) of prions from one
human to another.
No reports are available that semen (choice E) can transmit CJD.


Question 6 of 6
Two years later, the patient dies and is autopsied. Microscopic examination of
sections from the brain would most likely reveal which of the
following findings?
/ A. Bizarre, enlarged astrocytes
/ B. Loss of substantia nigra neurons
/ C. Macrophages with vacuoles containing metachromatic material
/ D. Spongiform degeneration
/ E. Whorls composed of meningeal cells

Explanation - Q: 2.6 Close

The correct answer is D. On light microscopy, the pathologic hallmarks of
CJD are spongiform degeneration, astrogliosis, and the lack of an
inflammatory response. Spongiform changes occur in putamen, caudate
nucleus, cerebral cortex, thalamus, and cerebellum. The amyloid plaques
that are seen in about 10% of cases are histologically different from those
seen in scrapie or Kuru.
Bizarre, enlarged astrocytes (choice A) suggest anaplastic astrocytoma or
glioblastoma multiforme.
Loss of substantia nigra neurons (choice B) suggests Parkinson disease.
Macrophages with metachromatic material (choice C) suggest
metachromatic leukodystrophy.
Whorls of meningeal cells (choice E) suggest meningioma.


A 68-year-old woman presents to her primary care physician complaining of
clumsiness and urinary incontinence. The daughter tells the
physician that her mother "walks oddly" and has been falling with increasing
frequency. She says that her mother's symptoms began a year or
two before, and gradually became worse, starting with a changing gait, followed
by urinary urgency and incontinence. More recently, she says
her mother has had difficulty remembering things and has "ruined her credit
rating" because she forgets to pay her bills. Her medical history
includes gall bladder surgery 10 years ago and hysterectomy 15 years ago for
abnormal bleeding. Family history is negative for strokes,
aneurysms, or intracranial bleeds. She quit smoking fifteen years ago, and does
not drink alcohoI. On physical examination, the physician
notes that the patient has an ataxic gait. She does not have a tremor, and on
neurologic examination, she is found to have normal strength and
muscle tone. MRI reveals enlargement of the ventricles without cortical atrophy.
Question 1 of 5
Which of the following is the most likely diagnosis?
/ A. AIzheimer disease
/ B. Migraine headaches
/ C. NormaI-pressure hydrocephalus
/ D. Parkinson disease
/ E. Seizures
/ F. Stroke

Explanation - Q: 3.1 Close

The correct answer is C. This patient has the classic triad of normal
pressure hydrocephalus, consisting of urinary incontinence, mental status
changes, and gait disturbance. NPH is a disease usually found in older
adults. Most cases are idiopathic, and are due to impaired cerebrospinal fluid
absorption. Subarachnoid hemorrhage or meningitis are risk factors for the
future development of NPH. The CT makes the diagnosis likely because of
the enlarged ventricles.
Alzheimer disease (choice A) is the most common cause of dementia.
However, a gait disturbance is not a typical early symptom, and cortical and
hippocampal atrophy are often visualized on CT.
Migraine headaches (choice B), seizures (choice E) and strokes (choice F)
are episodic disturbances. Seizures and migraine headaches do not usually
cause a decline, while the decline of multiple strokes is stepwise.
Parkinson disease (choice D) would most likely present with rigidity and a
pill-rolling tremor. Normal pressure hydrocephalus can present with a broad-
based, shuffling, bradykinetic gait similar to Parkinson disease, although in
this case, the patient's gait appears ataxic. MRI scans often show cortical
atrophy

Question 2 of 5
The most pertinent part of the neurological examination in this case would be
which of the following?
/ A. Deep tendon reflexes
/ B. Gait evaluation
/ C. Looking for nystagmus
/ D. Pinprick sensation
/ E. Testing of cranial nerve l

Explanation - Q: 3.2 Close

The correct answer is B. The classic triad of clinical symptoms of normal
pressure hydrocephalus includes abnormal gait, urinary incontinence, and
dementia. Therefore, evaluation of gait would be important with this disorder.
The gait disturbance and urinary dysfunction is thought to be the result of
distortion of the corona radiata (white matter that carries descending and
ascending cortical projections) resulting from the distended ventricles.
Deep tendon reflexes (choice A) may be useful to distinguish an upper
motor neuron from a lower motor neuron lesion.
Nystagmus (choice C) on end gaze is present in many normal individuals.
There are pathological types of nystagmus associated with vestibular or
brain stem lesions.
Pinprick sensation (choice D) should be evaluated particularly carefully if
you suspect a peripheral neuropathy, peripheral sensory nerve lesion, or
central lesion to the spinal-thalamic-cortical system.
Cranial nerve I (choice E) should be carefully tested if the patient reports
any smell abnormality, or if damage along the olfactory pathway is
suspected.


Question 3 of 5
The patient has a lumbar puncture and does better for about two days. The
mechanism underlying this improvement is which of the following?
/ A. BIood products have been removed from the cerebrospinal fluid
/ B. Brainstem pressure has been relieved
/ C. Cervical cord pressure has been relieved
/ D. PIacebo effect
/ E. Stretching of frontal lobe tracts is decreased

Explanation - Q: 3.3 Close

The correct answer is E. Normal-pressure hydrocephalus exerts its effects
by stretching frontal lobe tracts. When spinal fluid is removed, this tension
may be temporarily alleviated. If the patient responds with an improvement in
gait, mental status, and/or a reduction in bladder hyperactivity, he or she may
be a good candidate for shunt surgery.
Subarachnoid hemorrhage is a common antecedent to normal-pressure
hydrocephalus, presumably through inflammation of the arachnoid
granulations. However there is no such history in this case and even if there
were, there would be no utility to doing a lumbar puncture for such a reason
(choice A).
Pressure on the brainstem (choice B) would probably have caused cranial
nerve deficits or changes in consciousness, while cervical cord pressure
(choice C) would have probably caused problems in both upper and lower
extremities. Additionally, increased pressure over the cervical cord and/or
brainstem are both relative contraindications for lumbar puncture.
There may be a minimal placebo effect (choice D), but choice E is clearly a
better answer.

Question 4 of 5
Which of the following central nervous system areas are involved in the control of
urination?
/ A. Frontal lobes, medulla, and pons
/ B. Frontal lobes, pons, and spinal cord
/ C. Medulla, pons, and temporal lobes
/ D. Pons, medulla, and spinal cord
/ E. Spinal cord, occipital lobes, and pons

Explanation - Q: 3.4 Close

The correct answer is B. There are urinary control centers located in the
frontal lobes, pons, and sacral spinal cord. The exact way that these urinary
control centers work in humans is not well understood. Simply put, there is a
cortical area (probably in the medial aspect of the primary motor cortex) that
controls the pontine micturition center. Activation of the pontine micturition
center leads to a coordinated detrusor contraction and sphincter relaxation
via thoracolumbar sympathetic, sacral parasympathetic, and sacral somatic
centers that are involved regulation of the bladder. In normal pressure
hydrocephalus, it is thought that stretching and eventual death of frontal lobe
fibers results in dysfunction of the frontal lobe urinary center.
Urinary control fibers pass through the medulla (choices A, C, and D), but
there is not a control center located there.
The temporal and occipital lobes are not known to play a role in urination
control (choices C and E).

Question 5 of 5
The cerebrospinal fluid production rate in a normal adult is approximately which
of the following?
/ A. 5 mL/day
/ B. 50 mL/day
/ C. 500 mL/day
/ D. 5000 mL/day
/ E. 50000 mL/day

Explanation - Q: 3.5 Close

The correct answer is C. CSF formation is at the rate of about 500 mL per
day. The primary site for production is the choroid plexus. It is absorbed in
the arachnoid granulations. The pathology of normal-pressure hydrocephalus
is thought to be more related to inadequate absorption, rather than excessive
production.







A 56-year-old man is brought to the psychiatrist with a three-year history of
progressive speech difficulties associated with altered social
behavior. The family states that he became aggressive, talked inappropriately to
strangers, and showed insensitivity, contrary to his past
consideration to others. He has had problems with speech production, using
some phrases repeatedly, and showing a decreased vocabulary.
He started to eat a great deaI, and has developed obsessional cravings for
sweets. As a result, the patient has gained 40 pounds over the
past year. His neighbors caught him stealing things from their back yard. The
patient repeats the examiner's words and imitates the
examiner's gestures. Testing reveals difficulties in naming common objects or
pictures.
Question 1 of 4
Which of the following is the most likely diagnosis?
/ A. AIzheimer disease
/ B. Huntington disease
/ C. KIuver-Bucy syndrome
/ D. Pick disease
/ E. Progressive supranuclear palsy


Explanation - Q: 4.1 Close

The correct answer is D. Pick disease is a rare form of neurodegenerative
disorder characterized by a distinct progressive dementing process.
Presenile onset (under 65 years old), initial personality change, progressive
language dysfunction, hyperorality (overeating with obsessional craving for
certain types of food), and disinhibition are the key features of this disease.
Compared to Alzheimer disease, memory loss and impairment of intellect
occur at later stages of the disease. The symptoms of Pick disease occur
because the frontal and temporal lobes are affected. Circumscribed ('knife-
like") lobar atrophy is the hallmark of Pick disease, and sometimes can be so
severe that the postmortem brain weight can be as low as 800 g. The
atrophy affects the anterior temporal and frontal lobes, the orbital frontal
lobe, and the medial temporal lobe, but spares the posterior part of the
superior temporal gyrus and the pre- and postcentral gyri. These lobes are
important for language skills, impulse control, energy and enthusiasm,
problem solving, and maintaining socially appropriate behavior. In most
cases of Pick disease, the cause cannot yet be determined. However, there
is a strong genetic component in certain families. A mutation on chromosome
17 has been identified, which leads to production of an abnormal tau protein.
Key features of Alzheimer disease (choice A) are progressive dementia,
diffuse cortical atrophy, senile plaques, and neurofibrillary tangles.
Huntington disease (choice B) is characterized by chorea, athetoid
movements, dementia, and is associated with the atrophy of the caudate,
putamen and frontal cortex.
Originally described in primates with bilateral temporal lobe lesions
(amygdala), Kluver-Bucy syndrome (choice C) refers to hypersexuality,
gluttony, and an obsession to touch and seize any objects in the visual field.
Progressive supranuclear palsy (choice E) belongs to the tauopathy group
of diseases, together with Pick disease and corticobasal degeneration. It is a
neurodegenerative disorder characterized by supranuclear ophthalmoplegia,
pseudobulbar palsy, parkinsonism, axial dystonia, and dementia at the end-
stage of the disease.

Question 2 of 4
Imaging studies are ordered for this patient. Which of the following will most likely
be found?
/ A. Bitemporal lobar atrophy
/ B. Circumscribed cerebral atrophy (frontal and temporaI)
/ C. Diffuse cortical atrophy
/ D. Enlarged lateral ventricles
/ E. T2 hyperdensities

Explanation - Q: 4.2 Close

The correct answer is B. CT and MRI are useful to identify cortical
shrinkage and low density of white matter in the involved frontal and
temporal lobes. Brain SPECT imaging shows greatly decreased, or absent
perfusion to the affected lobes.
Bitemporal lobar atrophy (choice A), particularly involving the amygdala, is
seen on imaging studies in cases of the rare Kluver-Bucy syndrome.
Brain MRI images in patients with Alzheimer disease show diffuse
cerebral/cortical atrophy (choice C).
Enlarged, dilated lateral ventricles (choice D) are seen on head CT scan in
patients with Huntington disease.
T2 hyperdensities (choice E) on brain MRI are typical findings in patients
with multiple sclerosis.

Question 3 of 4
Which of the following would a brain biopsy most likely show?
/ A. "Ballooned" cells and argyrophilic bodies
/ B. Lewy bodies
/ C. Sclerosis in plaques
/ D. Senile plaques
/ E. Spongiform degeneration

Explanation - Q: 4.3 Close

The correct answer is A. Microscopically, in Pick disease, there is a marked
loss of cortical neurons with gliosis. Neuronal loss and degeneration are
usually maximal in the limbic system, including the hippocampus, entorhinal
cortex, and amygdala. The argyrophilic intraneuronal inclusions (Pick bodies)
are the hallmark of classic Pick disease. They are usually homogenous,
smooth-edged, and intensely argyrophilic. Ultrastructurally, Pick bodies are
composed of bundles of disorganized straight filaments that may be mixed
with coiled fibrils; the filaments and fibrils are made from aggregated tau
proteins. Pick bodies are negative for alpha-synuclein, which differentiates
them from cortical Lewy bodies. In contrast to Alzheimer disease, dendritic
branches of neurons containing Pick bodies are not labeled with anti-tau
antibodies. Pick bodies are most numerous in limbic and paralimbic cortices
and the ventral temporal lobe. In addition to Pick bodies, the cerebral cortex
may contain neurons with distended cytoplasm called "ballooned cells" or
Pick cells. Characteristically, Pick cells are seen in the nonpyramidal cells of
layers 2, 3 and 6 of the cerebral cortex.
Spherical inclusions (Lewy bodies, choice B) in melanin-depleted neurons of
substantia nigra are found in Parkinson disease.
Perivascular infiltration of monocytes and lymphocytes appearing as
indurated areas in pathologic specimens (sclerosis in plaques, choice C) is
seen in multiple sclerosis.
Typical pathologic features of Alzheimer disease are senile plaques (choice
D), composed of beta amyloid protein.
Spongiform degeneration (choice E) is a characteristic pathologic finding in
Creutzfeldt-Jakob disease.

Question 4 of 4
Abnormal amounts of which of the following proteins would most likely be found
in this patient?
/ A. Amyloid beta protein
/ B. Apolipoprotein E
/ C. Myelin basic protein
/ D. Ubiquitin
/ E. Tau
Explanation - Q: 4.4 Close

The correct answer is E. Tau proteins belong to the microtubule-associated
proteins (MAP) family. In humans, they are expressed in brain, mainly in
neurons, but also in glial cells, particularly in pathological conditions. It is
possible to detect tau mRNA also in muscles, heart, kidney, and other
tissues. The human tau gene is located on the long arm of chromosome 17.
Alternative mRNA splicing results in 6 mature isoforms, ranging from 352 to
441 amino acids. In normal brains, the ratio of shorter forms of tau to the
longer ones is 1:1. In Pick disease, the ration is abnormal, i.e., there are
more shorter forms of tau than the long ones. Tau proteins can undergo
nonenzymatic glycosylation and can be phosphorylated at multiple sites.
They are believed to promote normal microtubule assembly and stability, and
are involved in axonal transport. The biochemical analysis in all studied
cases of Pick disease reveals a major 55 and 64 kD tau doublet. This
characteristic electrophoretic pattern of pathological tau in this disease is well
correlated with the presence of Pick bodies. The remaining options are not
affected in Pick disease.
Amyloid beta protein (choice A) is a characteristic finding in Alzheimer
disease.
ApoE (choice B) protein transports cholesterol and other fatty molecules in
blood. ApoE is an integral part of the metabolic pathway of acetylcholine, and
appears to protect neurons by acting as an antioxidant.
Myelin basic protein (choice C) is one of the components of myelin and a
marker of its presence. Its level in CSF is increased in patients with active
demyelination disorders.
Ubiquitin (choice D) is a small, heat-stable stress protein that is found only in
eukaryotic cells. This protein can be found in Pick disease, but its presence
is not as consistent and specific as tau.




A 72-year-old man has impaired concentration and an inability to recollect names
and appointments. These memory problems have become
increasingly worse over a period of months and begin to interfere with his social
and financial activities. Subsequently, the patient becomes
depressed, and his wife persuades him to see a physician. A neurologic
examination confirms the presence of moderately severe short-term
memory loss associated with disturbances in language, such as difficulty in
naming familiar objects and verbal comprehension. MRI of the
head reveals diffuse cerebral atrophy without focal lesions or tumors.
Question 1 of 6
Which of the following is the most likely diagnosis?
/ A. AIzheimer disease
/ B. Creutzfeldt-Jakob disease
/ C. Huntington disease
/ D. Parkinson disease
/ E. Pick disease

Explanation - Q: 5.1 Close

The correct answer is A. This patient has Alzheimer disease (AD), the most
common cause of dementia in the Western world. His symptoms are typical
of early AD and the diffuse cerebral atrophy is also characteristic; MRI scans
may later show hippocampal atrophy. The common form of this disease
typically affects people over age 60. Approximately 4 million people in the
United States have AD, translating to a staggering annual health care cost of
over $80 billion. AD can be divided in to clinical stages, although the disease
progression varies with the individual. In the early stage of AD, patients
experience recent memory loss, personality changes, language difficulties
(especially word finding), emotional lability, and diminished judgement. Some
patients are aware of their difficulties, causing frustration and anxiety; others
are seemingly unaware of their symptoms. Mild to moderate depression is
common in early stages. Surprisingly, sociability may not be affected at this
early stage. The intermediate stage is characterized by a worsening memory,
both recent and remote. Various apraxias are common; patients may have
difficulties with sequential motor tasks, leading to difficulties with dressing,
eating, bathing, and toilet functions. Their sense of time and place are lost,
leading to a host of behavioral problems, including wandering and becoming
lost. Sometimes they can be quite aggressive, agitated, and uncooperative,
which can alternate with being socially withdrawn and passive. They can still
ambulate, but are at risk for accidents resulting from their confusion. In end-
stage AD, patients are unable to walk or perform tasks of daily living and
their recent and remote memory is gone. Death generally results from
malnutrition, secondary infections, and heart disease. The typical duration of
the disease is 8 to 10 years.
Creutzfeldt-Jacob disease (choice B) is a degenerative central nervous
system disease caused by infectious proteins known as prions. Although
these patients suffer from dementia, CJD is associated with myoclonus,
ataxia, and rigidity, and symptoms that progress so rapidly, that death
generally occurs within a year of onset. Patients often do not have brain
abnormalities on gross examination.
Huntington disease (choice C) is an autosomal dominant degenerative brain
disease characterized by chorea and behavioral disturbances. Onset of
symptoms generally occurs in the 40s or 50s. Memory is often not impaired
until late in the disease.
Parkinson disease (choice D) is a slowly progressing, degenerative brain
disease, which is characterized by resting tremor, rigidity, bradykinesia, and
disturbances of gait and posture. AD patients with advanced disease may
appear parkinsonian due to a shuffling gait and generalized muscle rigidity,
but they rarely have the resting tremor. Although Parkinson patients can
have dementia, AD's cognitive deficits far outweigh motor deficits.
Pick disease (choice E) can sometimes be difficult to distinguish from AD
clinically, but imaging studies show only temporal and frontal lobe atrophy. In
early Pick disease, behavioral changes are often more marked than memory
loss.

Question 2 of 6
If the patient undergoes a biopsy of the frontal cortex in the nondominant
hemisphere, which of the following findings would most likely support
the probable diagnosis?
/ A. Neuron loss without specific inclusions
/ B. Round argentophilic cytoplasmic inclusions
/ C. Round eosinophilic cytoplasmic inclusions
/ D. Senile plaques and neurofibrillary tangles
/ E. Spongiform change with mild neuron loss and gliosis

Explanation - Q: 5.2 Close

The correct answer is D. The three most important microscopic findings in
AD are neuritic or senile plaques, cytoplasmic neurofibrillary tangles (NFTs),
and amyloid angiopathy. The plaques are collections of twisted and dilated,
silver-staining, neuritic processes, which surround a central core. The
primary component of the core is A amyloid (derived from amyloid
precursor protein, APP). The core also contains other substances such as
Apo E, proteoglycans, and 1-antichymotrypsin. Degenerating neurons,
macrophages, and microglia surround the plaque core. NFTs, present in
neuronal cytoplasm, are bundles of filaments that displace or surround the
nucleus. They appear primarily as paired helical filaments ultrastructurally, a
major component of which is the abnormally phosphorylated tau protein.
Although NFTs are characteristic of AD, they do occur in other neurologic
disease.
Neuron loss without specific inclusions (choice A) occurs in various
disorders, such as very rare forms of fronto-temporal dementia.
Round argentophilic cytoplasmic inclusions (choice B) are found in Pick
disease. This diagnosis is ruled out by the "diffuse" nature of the of cerebral
atrophy in this case. In Pick disease, atrophy is circumscribed to the frontal
and anterior temporal lobes.
Round eosinophilic cytoplasmic inclusions (choice C) are called Lewy
bodies. Lewy bodies are characteristic of dementia with Lewy bodies (diffuse
Lewy body disease), and are also seen in patients with Parkinson disease.
Spongiform change with mild neuron loss and gliosis (choice E)
characterizes Creutzfeldt-Jakob disease.
Question 3 of 6
The cholinergic deficit thought to contribute to the learning and memory
dysfunction in this disease is associated with degeneration of which of
the following?
/ A. Large neurons in the basal nucleus of Meynert
/ B. Pigmented neurons in the locus coeruleus
/ C. Pigmented neurons in the substantia nigra
/ D. Pyramidal neurons of the hippocampus
/ E. Small neurons in the caudate nucleus

Explanation - Q: 5.3 Close

The correct answer is A. The most severe pathology seen on autopsy are
in the nucleus basalis of Meynert, the hippocampus, and temporal cortex.
The nucleus basalis of Meynert contains large cholinergic neurons that
project to the cerebral cortex, basolateral amygdala, basal ganglia, and
thalamus. The reduction in acetylcholine (ACh) that occurs in AD is thought
to be due to degeneration of this nucleus; this loss of ACh is thought to
underlie the memory and learning dysfunction of AD. Current treatment for
AD involves increasing levels of ACh.
Loss of pigmented neurons in the locus coeruleus (choice B) occurs in AD,
resulting in decreased central norepinephrine levels, however this is not
thought to be involved in the memory deficits.
Loss of pigmented dopaminergic neurons in the substantia nigra (choice C)
is characteristic of Parkinson disease.
Pyramidal neurons of the hippocampus (choice D) do degenerate in AD, but
are not cholinergic. The cholinergic deficit in the hippocampus is secondary
to the degeneration of neurons in the septum that project to the hippocampus
(analogous to the degeneration of the basal nucleus of Meynert, which
results in the neocortical cholinergic deficit).
Small neurons in the caudate nucleus (choice E) degenerate in Huntington
disease. These neurons are primarily GABAergic.
Question 4 of 6
Which of the following drugs can be used to treat this condition?
/ A. Diazepam
/ B. Haloperidol
/ C. L-Dopa
/ D. Scopolamine
/ E. Tacrine

Explanation - Q: 5.4 Close

The correct answer is E. Cholinesterase inhibitors are currently the only
agents approved by the FDA for the treatment of AD. These presumably
increase acetylcholine levels in the brain. Four cholinesterase inhibitors that
have been approved are: tacrine, donepezil hydrochloride, rivastigmine, and
galantamine.
Diazepam (choice A) is a benzodiazepine, used for sedation.
Haloperidol (choice B) is a dopamine antagonist, used as an antipsychotic.
L-Dopa (choice C), the precursor to dopamine, is used to replenish
dopamine in Parkinson disease.
Scopolamine (choice E) is an anticholinergic, and would be expected to
exacerbate this patient's symptoms.

Question 5 of 6
Which of the following syndromes would most likely produce clinical and
neuropathological features similar to those in this patient?
/ A. Cri-du-chat syndrome
/ B. DiGeorge syndrome
/ C. Down syndrome
/ D. KIinefelter syndrome
/ E. Turner syndrome

Explanation - Q: 5.5 Close

The correct answer is C. Almost all patients with Down syndrome, if they
survive into their forties, show clinical and neuropathological changes similar
to those seen in Alzheimer disease. Down syndrome is caused by trisomy
21. This chromosome is the locus of the gene for amyloid precursor protein
(APP). APP is therefore overproduced in these patients, which leads to
overproduction of beta-amyloid (Abeta), and to the typical features of
Alzheimer disease. Studies showed that the predominant type of Abeta in
Down syndrome is Abeta42, which is basically the same finding as in
Alzheimer disease.
Cri-du-chat syndrome (choice A) is caused by a deletion in chromosome 5p.
DiGeorge syndrome (choice B) is caused by a chromosome 22q deletion.
Klinefelter syndrome (choice D) is characterized by a 47,XXY (and variants
with additional X and Y chromosomes) karyotype.
Turner syndrome (choice E) involves a loss of X-chromosomal material,
resulting in a (45,XO) karyotype.

Question 6 of 6
Impairment of short-term memory in this patient suggests involvement of which of
the following brain regions?
/ A. Cerebral cortex, basal ganglia, and cerebellum
/ B. Hippocampus
/ C. Left inferior parietal cortex
/ D. Medial geniculate nucleus
/ E. Right parietal lobe

Explanation - Q: 5.6 Close

The correct answer is B. The hippocampus is a structure located in the
medial temporal lobe. It can be divided into five different areas: the dentate
gyrus at the tip of the hippocampus; CA3 (cornu ammonis) and CA1 with the
small CA2 between them; the subiculum at the base of the hippocampus,
and entorhinal area, which is a part of parahippocampal gyrus. The tract
called the perforant path is responsible for the input of the hippocampus. The
dentate neurons send axons that are called mossy fibers, to the CA3 area.
CA3 sends axons called Schaffer collaterals to CA1, which then sends fibers
to the subiculum, which represents the output of the hippocampus. From
here, the information can flow directly to the hypothalamus and mamillary
bodies, or back to entorhinal cortex and farther to sensory cortex. Although
AD affects the entire brain, the CA1 region of the hippocampus can be
severely affected early in the course of the disease. When the hippocampal
region is damaged, the major symptom is anterograde amnesia, the loss of
ability to store new declarative memories.
Nondeclarative forms of memory (skill learning, classical conditioning) are
based on the function of the cerebral cortex, basal ganglia and cerebellum
(choice A).
Dyscalculia suggests involvement of the left inferior parietal cortex (choice
C).
The medial geniculate nucleus (choice D) is a relay center for audition.
Involvement of the right parietal lobe could explain constructional apraxia
seen in this patient (choice E).


A 52-year-old woman consults a physician because she has been having
increasingly frequent episodes of abdominal cramps and diarrhea.
These episodes are accompanied by an uncomfortable flushing of her skin. She
thinks they are sometimes precipitated by eating, alcohoI, or
emotional distress, but she has also recently been having episodes that had no
obvious trigger.
Question 1 of 5
Her physician suspects that she may have a hormone-secreting tumor. Ectopic
secretion of which of the following substances would be most
Iikely to cause diarrhea?
/ A. Gastrin
/ B. GIucagon
/ C. Histamine
/ D. Insulin
/ E. Serotonin

Explanation - Q: 1.1 Close

The correct answer is E. Excess serotonin can act on smooth muscle to
produce diarrhea, colic, and malabsorption.
Excess gastrin secretion (choice A) can cause peptic ulceration secondary
to stimulation of gastric glands.
Excess glucagon secretion (choice B) can cause rash and impaired glucose
tolerance.
Excess histamine secretion (choice C), as well as excess bradykinin
secretion, can coexist with excess serotonin secretion, and can contribute to
the flushing seen in this patient.
Excess insulin secretion (choice D) can cause hypoglycemia.

Question 2 of 5
Which of the following laboratory tests would be most appropriate to test for
excess secretion of the hormone causing the diarrhea?
/ A. Dexamethasone test
/ B. Hemoglobin A1C
/ C. Urinary 5-hydroxyindoleacetic acid
/ D. Urinary aminolevulinic acid
/ E. Urinary porphobilinogen

Explanation - Q: 1.2 Close

The correct answer is C. 5-hydroxyindoleacetic acid is the major urinary
metabolite of serotonin.
The dexamethasone test (choice A) is used to screen for Cushing
syndrome.
Hemoglobin A1c (choice B) is a marker for long-term glycemic control.
Urinary porphobilinogen (choice E) and urinary aminolevulinic acid (choice
D) are used in the diagnosis of the porphyrias.


Question 3 of 5
CT scan demonstrates several probable metastatic tumors in the patient's liver.
These are most likely which of the following histologic types?
/ A. Burkitt lymphoma
/ B. Carcinoid tumor
/ C. Hepatocellular carcinoma
/ D. Kaposi sarcoma
/ E. Squamous cell carcinoma

Explanation - Q: 1.3 Close

The correct answer is B. The patient's clinical presentation is typical for
carcinoid syndrome, which is a clinical manifestation of hormone-secreting
carcinoid tumors. It is usually seen in the presence of metastatic disease
involving the liver. None of the other tumors listed would be expected to
secrete hormones.


Question 4 of 5
The metastases most likely originated from which of the following?
/ A. Central nervous system
/ B. Gastrointestinal tract
/ C. Reproductive tract
/ D. Respiratory tract
/ E. Urinary tract

Explanation - Q: 1.4 Close

The correct answer is B. Approximately 90% of carcinoid tumors occur in
the gastrointestinal tract, with the most common site in the gastrointestinal
tract being the small intestine (39%, data from www.carcinoid.org web site).
Involvement of the appendix (26%) and rectum (15%) is also fairly common.
Other sites include colon (5-7%), stomach (2-4%), pancreas (2-3%), liver
(less than 1%), bronchial system (10%, choice D), and rarely gonads
(choice C), gallbladder and bile ducts, urinary bladder and kidneys (choice
E), prostate, breast, and thymus. The central nervous system (choice A)
does not appear to be a significant site of origin.
Question 5 of 5
These tumors are thought to be derived from which of the following cell lines?
/ A. Endothelial cells
/ B. Enterochromaffin cells
/ C. Fibroblasts
/ D. Lymphocytes
/ E. Smooth muscle cells

Explanation - Q: 1.5 Close

The correct answer is B. Carcinoid tumors are thought to be derived from
the glandular endocrine hormone-producing cells, known as
enterochromaffin cells, which are widely distributed through the body. They
occupy a borderline category between benign and malignant, because only a
fairly small percentage of them eventually metastasize, and pathologists are
unable, histologically, to tell which ones will or will not. Most of the malignant
ones are derived from the small intestine (1/5 metastasize and 1/3 of those
cause carcinoid syndrome), and they tend to produce the clinical symptoms
of carcinoid syndrome illustrated in the case history only when they have
metastasized to the liver. The tumors probably secrete hormones prior to
metastasis; the absence of carcinoid syndrome in these patients is attributed
to the fact that the venous drainage of the gut passes through the liver via
the portal system, clearing the blood of excess vasoactive substances. Also,
some of the tumors do not secrete clinically significant amounts of hormones.
Serotonin, histamine, and bradykinin are the most common hormones
secreted that produce symptoms. Carcinoid tumors are found incidentally in
approximately 0.5% of appendectomy specimens; appendiceal carcinoids
only rarely metastasize. The tumors may also be a component of the multiple
endocrine neoplasia (MEN) syndromes. Tumors that are identified
incidentally before metastasis can be cured surgically. No effective
chemotherapy or radiotherapy is available for metastatic disease, but the
tumors grow so slowly that 10-15 year survival times are not unusual. The
somatostatin analog octreotide can help control symptoms.
Endothelial cells (choice A) give rise to vascular tumors such as Kaposi
sarcoma and angiosarcoma.
Fibroblasts (choice C) can give rise to a variety of fibromas and
fibrosarcomas.
Lymphocytes (choice D) can give rise to leukemias and lymphomas.
Smooth muscle cells (choice E) can give rise to leiomyomas and
leiomyosarcomas.

A 32-year-old woman goes to an emergency department because she has
developed severe, watery diarrhea. On questioning, she reports
that three days previously, she spent the weekend along the Gulf of Mexico, and
ate at a raw oyster bar. About 36 hours later, she developed
vomiting and an abrupt, painless, watery diarrhea. The volume of diarrhea has
been copious, and she has subsequently developed intense
thirst, oliguria, muscle cramps, and weakness. At the time of being seen in the
emergency department, she is noted to have marked loss of
tissue turgor, sunken eyes, and wrinkling of the skin of her fingers. Laboratory
studies demonstrate hemoconcentration and severe metabolic
acidosis with potassium depletion.
Question 1 of 5
Which of the following is the most likely cause of the woman's problems?
/ A. Amoeba infection
/ B. Bacterial infection
/ C. Food poisoning
/ D. Irritable bowel syndrome
/ E. Viral infection

Explanation - Q: 2.1 Close

The correct answer is B. This patient is severely ill and dehydrated. Severe
watery diarrhea that persists is most likely to be due to a bacterial infection.
Amoeba infection (choice A) usually produces a bloody diarrhea that does
not have the watery characteristic of cholera.
Food poisoning (choice C) usually begins within the first eight hours after
ingestion of contaminated food, and is often over within 24 hours.
Irritable bowel syndrome (choice D) is usually a chronic, but relatively mild,
problem.
Viral infection (choice E) can cause gastroenteritis, but it is not usually as
debilitating as this patient's disease.


Question 2 of 5
An organism cultured from the patient's stool is subsequently characterized as an
oxidase-positive, gram-negative curved rod with polar
flagella that shows "shooting star" mobility and grows best on TCBS medium.
Which of the following is the most likely pathogen?





Explanation - Q: 2.2 Close

The correct answer is E. The features noted are those of Vibrio cholerae,
the causative organism of the severe diarrheal disease, cholera. The
diarrhea is the result of the action of the cholera enterotoxin, which acts by
ADP-ribosylating adenylate cyclase, leading to increased cyclic AMP with
secondary increased efflux of Cl
-
and H
2
O. World-wide, cholera is usually
spread by fecally contaminated water, but in areas such as Europe and the
United States, where the general level of hygiene, sewage control, and water
supply is good, the occasional cases seen are often the result of
contaminated food. Some of the shellfish from the Gulf of Mexico naturally
contain the organism, and ingestion of these shellfish in raw or poorly cooked
form has caused cholera.
Escherichia(choice A) is oxidase-negative.
Pseudomonas(choice B) is an oxidase-positive, gram-negative rod that
characteristically produces the blue pigment, pyocyanin.
Salmonella(choice C) is a motile, gram-negative rod that is a non-lactose
fermenter that produces H
2
S.
Shigella(choice D) are non-motile gram-negative rods.



Explanation - Q: 2.3 Close

The correct answer is A. Vibrio cholera serogroup O1 is the major cause of
epidemic cholera, and up until 1992 was the only cause. Since then, the
O139 serogroup (choice E) has been an important pathogen in India and
Bangladesh, but has not yet become important outside Asia. No other
serotypes are known to cause epidemic cholera.

Question 4 of 5
In the United States, approximately what percentage of patients who acquire this
infection subsequently die?
/ A. 1% or less
/ B. 5-10%
/ C. 20-30%
/ D. 50-70%
/ E. 90% or more

Explanation - Q: 2.4 Close

The correct answer is A. The mortality of cholera varies markedly
depending upon the general health of the individuals who become infected
and the availability of adequate medical care. In extreme situations, such as
in refugee camps with little sanitation, little food, and no available medical
facilities, case fatality rates of 25-50% are encountered. In Europe, the
Americas, and more recently Asia, case fatality rates in most situations are
kept at 1% or less. A major reason for the decreased fatality rates in most
parts of the world is that the World Health Organization has been
encouraging endemic areas to pre-plan for epidemics and have available
large numbers of previously prepared packages of oral rehydration salts.
Question 5 of 5
The patient responds with IV fluids, and is very much better within 24 hours.
While this woman did not require antibiotic therapy, which of the
following antibiotics is used for this disease?
/ A. First generation cephalosporin
/ B. Metronidazole
/ C. Penicillin
/ D. Tetracycline
/ E. Third generation cephalosporin

Explanation - Q: 2.5 Close

The correct answer is D. Most patients with cholera in this country who
have disease severe enough to cause significant dehydration are rehydrated
initially with IV fluids, and are then switched to oral fluids several hours later.
In countries with more likelihood of having an epidemic and poorer medical
availability, packages of oral rehydration salts to which water is added are
commonly (and very effectively) used to treat cholera. Antibiotics are often
not employed in milder cases. In severe cases, antibiotic therapy can reduce
the volume and duration of the diarrhea experienced by the patient. The
usual antibiotic of choice is tetracycline, although some resistant strains are
emerging. Useful alternative drugs include ciprofloxacin, erythromycin,
doxycycline, and furazolidone. No vaccine against cholera is presently
available in the United States, although two vaccines (which are not
completely effective) are available elsewhere in the world. The other agents
listed in the choices are not used to treat cholera.



A 25-year-old woman is hospitalized for multiple injuries to her arms and legs
from an automobile accident. She is treated with several
intravenous broad-spectrum antibiotics because of significant concern that her
wounds are becoming infected with a mixed flora of organisms.
She responds over the next several days to the antibiotics with an initial
decrease in fever. However, on the fourth day after her accident, she
develops severe diarrhea with fever, vomiting, cramping abdominal pain,
tenesmus, abdominal distension, and fluid losses severe enough to
require IV fluids.
Question 1 of 5
Proctoscopic examination demonstrates discrete yellow plaques up to 2 cm
diameter which are scattered over the colonic mucosa. Which of
the following would most likely be demonstrated on colonic biopsy?
/ A. Acid fast bacteria
/ B. Neoplastic polyps
/ C. Parasitic eggs
/ D. Pseudomembrane formation
/ E. Small granulomas

Explanation - Q: 3.1 Close

The correct answer is D. This patient's history and proctoscopic
examination are typical of severe pseudomembranous colitis (also
sometimes called antibiotic associated colitis). Patients with milder disease
may have only diarrhea. On pathologic examination, pseudomembranes
composed of fibrin, neutrophils, necrotic material, and bacteria are seen
overlying partially disrupted glands with prominent submucosal edema.
Acid fast bacteria (choice A) would suggest tuberculosis or atypical
mycobacterial infection.
Neoplastic polyps (choice B) are seen as isolated findings or as part of
familial polyposis syndromes.
Pseudomembranous colitis is not related to parasitic infection (choice C).
Small granulomas (choice E) could suggest either Crohn disease or
tuberculosis.


Explanation - Q: 3.2 Close

The correct answer is B. The clostridia are gram-positive, anaerobic, spore-
forming rods that can produce a variety of nasty diseases. This patient's
disease, pseudomembranous colitis, is due to bacterial gut overgrowth with
Clostridium difficile, usually in the aftermath of broad-spectrum antibiotic
therapy. Both children and adults may become infected. Probably in most
cases, the patients already had small numbers of the organisms in their guts
before antibiotic therapy, although isolated examples of transmission within
wards have also been documented.
Clostridium botulinum(choice A) causes botulism.
Clostridium perfringens(choice C) and Clostridium septicum(choice D)
cause gas gangrene.
Clostridium tetani(choice E) causes tetanus.
Question 3 of 5
Which of the following tests would be most helpful in confirming the diagnosis?
/ A. CT scan
/ B. MRI scan
/ C. "Scotch tape" test
/ D. Specific toxin in stool
/ E. Stool for ova and parasites

Explanation - Q: 3.3 Close

The correct answer is D. Clostridium difficile produces toxins A and B,
which can now be rapidly identified in stool samples. This is the best
confirmatory test following proctoscopy, as the results will be back much
faster than the pathology report on a tissue sample. The toxins cause
necrosis of the superficial gut mucosa that, in turn, leads to
pseudomembrane formation. True infection of the gut wall by the bacteria
does not occur. (The bacteria like to live in and eat the necrotic material and
fibrin of the pseudomembrane.)
CT scan (choice A) and MRI scan (choice B) are expensive tests that will
not contribute to the diagnosis.
The "Scotch tape" test (choice C) involves using tape to collect pinworm
eggs from the perirectal skin.
Clostridium difficile is a bacterium rather than a larger parasite, so stool for
ova and parasites (choice E) would not be helpful.


Question 4 of 5
This patient should not undergo barium contrast studies because of the
increased risk in her disease of which of the following
/ A. AIIergic reaction
/ B. Gut perforation
/ C. Predisposition for cancerous transformation
/ D. Secondary appendicitis
/ E. Trapping of dye in diverticula



Explanation - Q: 3.4 Close

The correct answer is B. In addition to the immediate risks related to
hypotension, dehydration, and electrolyte imbalance in these sometimes
critically ill patients, complications that can incur include colonic perforation
(which may be induced by barium contrast studies) and toxic megacolon.
Pseudomembranous colitis does not alter allergic reactions (choice A),
predispose for appendicitis (choice D) or cancer (choice C), or induce
formation of diverticula (choice E).

Question 5 of 5
Which of the following drugs is most likely to be effective in this case?
/ A. 3rd generation cephalosporin
/ B. Amoxicillin
/ C. Ampicillin
/ D. CIindamycin
/ E. Vancomycin

Explanation - Q: 3.5 Close

The correct answer is E. Clostridium difficile is resistant to most antibiotics,
which is why it tends to cause a bowel bacterial overgrowth when broad-
spectrum antibiotics are used. The two antibiotics to which it is usually
sensitive, and that are consequently most often used to treat
pseudomembranous colitis, are vancomycin and metronidazole. The other
agents listed in the choices are frequently reported causes of
pseudomembranous colitis.

A 17-year-old boy is seen in an emergency department with severe acute
abdominal symptoms that suggest appendicitis. The patient had a
similar episode previously that led to an appendectomy, but the appendix was
free of inflammation at pathologic examination. On questioning,
the boy reports having had intermittent diarrhea with moderate chronic abdominal
pain for several years, which he had been afraid to tell
anyone about after having had a "normaI" appendix removed. On physical
examination, the boy is noted to be thin, with short stature, and have
a palpable fullness in the right lower quadrant of his abdomen. UItrasound
examination of the abdomen shows some thickening of bowel
mesentery, but no distinct masses. Colonoscopy demonstrates sharply
demarcated segments of diseased bowel with patchy mucosal ulcers
separated by adjacent normal bowel involving both the distal ileum and right side
of the colon.

Question 1 of 6
Which of the following is the most likely diagnosis?
/ A. Colon cancer
/ B. Crohn disease
/ C. Hirschsprung disease
/ D. Pseudomembranous colitis
/ E. UIcerative colitis

Explanation - Q: 4.1 Close

The correct answer is B. This patient most likely has Crohn disease, as
indicated by his chronic abdominal complaints and the distal small intestinal
and colonic ulceration with skip lesions (normal bowel separating involved
areas). In addition to the distal small intestine and colon, Crohn disease can
affect the perianal area, entire small intestine, stomach, and esophagus.
Crohn disease may present with chronic diarrhea with systemic complaints,
acute abdomen, or extraintestinal manifestations. The peak incidence for
onset of symptoms occurs between 14 to 24 years. Crohn disease is thought
to be due to a genetic predisposition that leads to an autoimmune reaction in
the intestine that may be triggered in response to an environmental, dietary,
or infectious agent. About 1 in 6 people with Crohn disease have a relative
with Crohn disease, or, less commonly, ulcerative colitis.
Colon cancer (choice A) would be highly unlikely in a 17-year-old, and
usually causes a single mass lesion or stricture.
Hirschsprung disease (choice C) causes megacolon in young children.
Pseudomembranous colitis (choice D) is due to overgrowth of Clostridium
difficile and is usually seen following treatment with broad-spectrum
antibiotics.
Ulcerative colitis (choice E) usually extends from the rectum proximally and
does not have skip lesions.

Question 2 of 6
Which of the following findings on right colon mucosal biopsy would be most
suggestive of the patient's likely diagnosis?
/ A. Absence of nerve cell bodies in submucosa
/ B. Bacteria-Iaden pseudomembrane
/ C. Crypt abscesses
/ D. Neoplastic epithelial cells
/ E. Small granulomas



Explanation - Q: 4.2 Close

The correct answer is E. Microscopic features of Crohn disease include
transmural inflammation, small granulomas (most characteristic, but only
present in about 50% of cases), and variable degrees of inflammation.
Absence of nerve cell bodies in submucosa (choice A) suggests
Hirschsprung disease.
A bacteria-laden pseudomembrane (choice B) suggests
pseudomembranous colitis;
Crypt abscesses (choice C) suggest ulcerative colitis.
Neoplastic epithelial cells (choice D) suggest colon cancer or neoplastic
polyps.

Question 3 of 6
The patient is treated initially with corticosteroids, and then these are tapered and
he is switched to a maintenance therapy with sulfasalazine.
Sulfasalazine is an unusual medication that combines which of the following?
/ A. A histamine derivative and a beta blocker
/ B. A Iipoxygenase inhibitor and a penicillin derivative
/ C. A mast cell degranulation inhibitor and an anticoagulant
/ D. A salicylate derivative and a sulfonamide derivative
/ E. A serotonin antagonist and a proton pump inhibitor

Explanation - Q: 4.3 Close

The correct answer is D. Acute exacerbations of Crohn disease can be
treated with corticosteroids, but then the patient is usually switched to
maintenance therapy with immunomodulating drugs, such as azathioprine
and 6-mercaptopurine, or sulfasalazine or related drugs. Sulfasalazine has a
sulfapyrine component, which is a sulfonamide derivative and a 5-
aminosalicyclic acid (5-ASA) component, which is a salicylate derivative.
Most of the drug activity appears to be related to the 5-ASA component, and
most of the toxicity is related to the sulfapyrine, so alternatives to
sulfasalazine, such as mesalamine, are being developed that preserve the 5-
ASA activity without retaining the sulfapyrine component. The other answers
are distracters.

Question 4 of 6
Several years later, the patient develops recurrent urinary tract infections with
mixed flora bacteria isolated from the urine. This pattern
suggests that which of the following may have developed?
/ A. BIadder stone
/ B. Fistula
/ C. Kidney stone
/ D. Systemic immunosuppression
/ E. Urethral strictures

Explanation - Q: 4.4 Close

The correct answer is B. The patient has probably developed a fistula
between the intestine and the bladder. Fistulas are a relatively common
complication of Crohn disease. Surgery is used to correct recurrent intestinal
obstruction and intractable fistulas, but may be unsatisfactory because of
disease recurrence.
The other choices do not commonly complicate Crohn disease and would not
specifically predispose for mixed flora infections.
Question 5 of 6
At a still later date, the patient develops chronic lower back pain and is
diagnosed with ankylosing spondylitis. Which HLA type has been
associated with this extracolonic manifestation?
/ A. HLA-B27
/ B. HLA-B35
/ C. HLA-Cw6
/ D. HLA-DR3
/ E. HLA-DR5

Explanation - Q: 4.5 Close

The correct answer is A. Ankylosing spondylitis is one of the seronegative
spondyloarthropathies. It should be suspected in any young person
complaining of chronic lower back pain and can be confirmed by radiographs
or CT scans of sacroiliac joints. The disease usually progresses to involve
the whole vertebral column, producing ankylosis and respiratory failure
secondary to restrictive lung disease. HLA-B27 positivity has been
associated with Crohn disease patients who develop extracolonic
manifestations including ankylosing spondylitis, sacroiliitis, uveitis, and
primary sclerosing cholangitis.
You should associate HLA-B35 (choice B) with vitiligo, duodenal ulcer, and
subacute thyroiditis.
You should associate HLA-Cw6 (choice C) with psoriasis.
You should associate HLA-DR3 (choice D) with celiac disease, Goodpasture
syndrome, type I diabetes mellitus, and systemic lupus erythematosus.
You should associate HLA-DR5 (choice E) with juvenile rheumatoid arthritis
and pernicious anemia.

Question 6 of 6
Examination of the patient's legs reveals necrotic ulcers with ragged bluish-red
overhanging edges together with areas containing plaques
with pustules. Which of the following is the most likely diagnosis?
/ A. Eruptive xanthomata
/ B. Lupus vulgaris
/ C. Raynaud's phenomenon
/ D. Psoriasis
/ E. Pyoderma gangrenosum

Explanation - Q: 4.6 Close

The correct answer is E. Pyoderma gangrenosum can precede the onset of
chronic inflammatory bowel disease. It is treated by systemic and topical
corticosteroids. It may also be associated with myeloproliferative disorders
and rheumatoid arthritis.
Xanthomata (choice A) may be located on the tendons on the back of the
hands, the Achilles tendon and patellar tendon, buttocks, and back. They are
usually yellow papules up to 5 mm in diameter. They suggest familial
hypercholesterolemia.
Lupus vulgaris (choice B) is a progressive form of cutaneous tuberculosis
occurring in a person with a moderate or high degree of immunity. The two
types are the plaque form (a tiny reddish-brown, flat plaque that extends
gradually) or the ulcerative form (scarring and ulceration over the areas of
necrosis).
Raynaud's phenomenon (choice C) presents as cold and cyanotic digits
along with atrophy of the finger pulp and, in severe cases, gangrene. It is an
exaggerated physiological response wherein the fingers turn white, and then
when rewarmed, will turn blue, and then red as part of rebound hyperemia. It
may be associated with underlying connective tissue diseases such as
systemic sclerosis.
Psoriasis (choice D) presents as a thick plaque-like scale that is usually
silver or salmon-pink in color. It is usually well defined on the extensor
surfaces of elbows or knees. It is often associated with asymmetrical
arthropathy mainly involving the terminal interphalangeal joints.


Several individuals develop bloody diarrhea after eating hamburgers at a fast
food chain. Testing of stool samples from these patients
demonstrates the presence of verotoxin.

Question 1 of 5
Follow-up cultures from the patients and from food samples taken from the
restaurant would most likely demonstrate which of the following?





Explanation - Q: 5.1 Close

The correct answer is B. Escherichia coli is usually a normal commensal in
the gut, but strains producing gastrointestinal disease do occur. In this case,
this patient has enterohemorrhagic E. coli (EHEC), as evidenced by the
bloody diarrhea. These strains are also sometimes known as verotoxin-
producing E. coli (VTEC), and the most common serotype is O157:H7. The
bacteria can be acquired in poorly cooked beef, unpasteurized milk, and
water or a variety of foods contaminated with cow manure (e.g., alfalfa
sprouts). It is thought that mild disease may be common, but is usually not
specifically diagnosed. In its more severe form, severe cramping
accompanies a diarrhea that is initially watery but becomes grossly bloody.
Vomiting also occurs, albeit less commonly. High fever does not usually
occur, although low fever may. The disease is usually self limited, typically
ending within 8 days.
EaggEC (choice A) causes persistent diarrhea with vomiting and low-grade
fever in the developing world.
EIEC (choice C) causes watery diarrhea with fever and abdominal pain that
sometimes progresses to dysentery.
EPEC (choice D) causes watery diarrhea in babies.
ETEC (choice E) causes traveler's diarrhea.



Explanation - Q: 5.2 Close

The correct answer is D. Verotoxins are plasmid-associated toxins that are
most similar to the Shiga-like toxins 1 and 2, produced by Shigella. All of
these toxins inhibit protein synthesis in the small intestine by damaging the
60S ribosome.
Clostridium botulinum(choice A) produces a toxin that causes paralysis.
Clostridium tetani(choice B) produces a toxin that causes spastic paralysis.
Pseudomonas(choice C) produces exotoxin A, which ADP ribosylates EF-2,
thereby inhibiting protein synthesis (similar to diphtheria toxin).
Vibrio cholerae(choice E) produces a toxin that causes diarrhea by ADP-
ribosylating adenylate cyclase, causing rising cAMP levels.



Question 3 of 5
One of the younger children who had the diarrhea develops a severe
complication. Which of the following complications most likely occurred?
/ A. Congestive heart failure
/ B. Disseminated intravascular coagulation
/ C. Hemolytic uremic syndrome
/ D. Hepatic abscess
/ E. Polycystic kidney disease



Explanation - Q: 5.3 Close

The correct answer is C. Up to 15% (depending on the particular outbreak)
of young children who have E. coli O157:H7 develop hemolytic uremic
syndrome (HUS), which is characterized by hemolytic anemia,
thrombocytopenia, and renal failure. The other answers are distracters


Question 4 of 5
One of the elderly individuals who had the diarrhea develops a severe
complication. Which of the following is the most likely complication?
/ A. Aplastic anemia
/ B. Myelodysplastic syndrome
/ C. Non-Hodgkin lymphoma
/ D. Thrombotic thrombocytopenic purpura
/ E. UIcerative colitis

Explanation - Q: 5.4 Close

The correct answer is D. Thrombotic thrombocytopenic purpura (TTP),
despite its very different name, appears to be very closely related to
hemolytic uremic syndrome, and both can complicate E. coli O157:H7
infection. Some authors think the two conditions are actually the same
(sometimes called TTP-HUS), with varying degrees of renal failure and the
addition of fever and neurologic symptoms commonly in TTP. Both young
children and the elderly are particularly vulnerable to developing severe
complications of E. coli O157:H7 infection, but young children tend to be
diagnosed with HUS while the elderly tend to be diagnosed with TTP. These
are the only two potentially fatal complications of this infection that are seen
with any significant frequency. The other choices are distracters.

Question 5 of 5
Introduction of which of the following treatments has markedly altered the course
of the most serious complications of this condition in adults
and children?
/ A. Bone marrow transplant
/ B. PIasma exchange
/ C. Renal transplant
/ D. Thyroxine supplementation
/ E. Total parenteral nutrition

Explanation - Q: 5.5 Close

The correct answer is B. The pathophysiology of TTP-HUS and its
relationship to the enterohemorrhagic E. coli strains are still unclear. (Cases
may also be related to pregnancy, drugs, bone marrow transplantation,
metastatic cancer, or may occur idiopathically.) Formerly, TTP-HUS was
considered to be almost uniformly fatal, except in epidemic disease in young
children, but the recovery rate is now 85% when plasma exchange is
continued daily until evidence of disease activity has subsided. Patients
should be monitored for relapses, which may recur over a period of many
years. The other therapies listed are not usually used in these diseases.


An 11-month-old girl who attends a day care center develops vomiting and
severe, watery diarrhea. The child is taken to an emergency
department on the third day of the illness, because the mother is concerned that
she has not been able to hold any food or liquids down without
vomiting. Physical examination demonstrates an obviously ill child with listless
behavior, Ioss of skin turgor, dry mucous membranes, and
weight 3 pounds less than 1 week previously. Her blood pressure is 64/40 mm
Hg.


Question 1 of 5
Which of the following is the most common cause of viral gastroenteritis in
children under the age of three?
/ A. Astrovirus
/ B. Calicivirus
/ C. Enteric adenovirus
/ D. Norwalk virus
/ E. Rotavirus

Explanation - Q: 6.1 Close

The correct answer is E. The most common cause of severe dehydrating
diarrhea in infants and young children is Group A rotavirus. The peak
incidence of infection occurs from 3 to 15 months. After age three, severe
diarrhea is rare, but mild disease can occur. This virus accounts for about
half of the cases of diarrhea requiring hospitalization in children in the United
States. Rotaviruses are in the Reoviridae family and have a genome
consisting of 11 double-stranded RNA segments. The other agents listed can
also cause viral gastroenteritis in children and/or adults.

Question 2 of 5
In the United States, this virus tends to cause illness during which of the following
periods?
/ A. Predominately fall months
/ B. Predominately spring months
/ C. Predominately summer months
/ D. Predominately winter months
/ E. Year-Iong with no seasonal preference


Explanation - Q: 6.2 Close

The correct answer is D. In temperate climates, rotavirus has a winter
seasonal pattern. In the United States, epidemics occur from November to
April. In tropical climates, disease caused by rotavirus occurs year round.

Question 3 of 5
The virus is predominately spread by which of the following routes?
/ A. Contaminated blood
/ B. Contaminated food
/ C. Contaminated water sources
/ D. Direct fecaI-oral route
/ E. Inhaled aerosols
Explanation - Q: 6.3 Close

The correct answer is D. Rotavirus is a notoriously contagious virus, and
the infective dose is thought to be as small as 10-100 infectious viral
particles. While the initial yearly source of the virus may be from
contaminated estuary waters (choice C), the vast majority of cases are
spread from person to person via direct contact with stool on diapers, objects
such as toys, or fingers. While not always necessary, the presence of
rotavirus can be confirmed by using enzyme immunoassay on stool
specimens. Day care centers are notorious sources of minor epidemics of
the virus, and day care workers should be encouraged to be very careful.
Reasonable precautions include removal of kids with diarrhea from the day
care environment, prompt diaper changing before the children have a chance
to touch the stool, and routine use of fresh disposable gloves during
diapering.
Contaminated blood (choice A) and inhaled aerosols (choice E) are not
usual routes of infection.
Contaminated food (choice B), typically contaminated by food handlers who
have recently changed an infected child's diapers, is an uncommon source of
infection.


Question 4 of 5
This child's poor skin turgor, dry mucus membranes, significant weight loss, and
low blood pressure all suggest which of the following?
/ A. Dehydration
/ B. Disseminated intravascular coagulation
/ C. IIeus
/ D. Sepsis
/ E. Starvation

Explanation - Q: 6.4 Close

The correct answer is A. These findings strongly suggest that the child is
severely dehydrated and in need of intravenous fluid replacement.
Approximately 500,000 children are seen in emergency room and outpatient
clinic visits for rotavirus infection yearly, and of these, about 50,000
hospitalizations (usually to correct dehydration) and 20 deaths (usually
caused by failure to correct the dehydration in a timely fashion) occur yearly
in the United States. World-wide, rotavirus infection is thought to cause
around 600,000 deaths, primarily in young children. This child with severe
dehydration should at least be given IV fluids, and might require
hospitalization.
Disseminated intravascular coagulation (choice B) would cause multiple
petechiae and purpura, and does not usually complicate rotavirus infection.
Ileus (choice C) is a paralyzed bowel, which is the opposite of the problem
seen in gastroenteritis.
Sepsis (choice D) is not usually diagnosed in viral infections, but is seen in
severe bacterial infections, often with bacteremia.
Three or four days of not eating is not long enough to induce starvation
(choice E).

Question 5 of 5
In 1989, the FDA Iicensed a live attenuated vaccine against the causative agent
of this disease for use in infants. It was later withdrawn for
which of the following reasons?
/ A. Data suggested a link between the vaccine and intussusception in some
infants during the first 1-2 weeks following vaccination.
/ B. It was questioned if there was association between the vaccine and the
development of autism.
/ C. Seizures occurred in a significant number of recipients.
/ D. The vaccine was not widely used because it was too expensive.
/ E. The vaccine was thought to be associated with the development of arthritis
in recipients.

Explanation - Q: 6.5 Close

The correct answer is A. The live attenuated rotavirus vaccine was a virus
mixture of reassorted strains that were primarily animal viruses except for
one human-virus gene segment. The vaccine had a 49-68% efficacy against
any diarrhea due to rotavirus and a 61-100% efficacy against severe
disease. However, during the first 11 months of use, post-licensure studies
identified intussusception, a form of intestinal obstruction in which a segment
of the bowel prolapses into a more distal segment, as an uncommon but
potentially life-threatening side effect. Follow up studies estimated that 1
additional case of intussusception occurred for every 5,000- 10,000
vaccinees. The vaccine has currently been withdrawn from use in the US.

A 45-year-old man presents to a physician because of severe chronic diarrhea
accompanied by a 18 kg (40 Ib) weight loss. The diarrhea
began several years ago, and has become steadily worse. It is often
accompanied by excessive flatulence, and the man notes that his stools
usually float. The man additionally has felt generally poorly and sometimes has
experienced joint symptoms. Intestinal biopsy demonstrates
Iarge numbers of foamy macrophages distending the lamina propria. A periodic
acid-Schiff (PAS) stain shows granules within the
macrophages.
Question 1 of 5
Which of the following is the most likely diagnosis?
/ A. Celiac sprue
/ B. Intestinal lymphangiectasia
/ C. Lactase deficiency
/ D. Tropical sprue
/ E. Whipple disease

Explanation - Q: 7.1 Close

The correct answer is E. This patient has Whipple disease, which is a rare
illness characterized by diarrhea, often severe malabsorption, and weight
loss. The condition most commonly affects men aged 30 to 60. The disease
may present either abruptly or insidiously. If it develops insidiously, non-
gastrointestinal manifestations may bring the patient to medical attention.
These may include pleuritic pain, pleural effusion, anemia, joint problems,
cardiac problems, neuropsychiatric problems, eye problems, or hepatic
dysfunction.
All of the other diseases listed can also cause chronic diarrhea.
Celiac sprue (choice A) shows villous atrophy on biopsy, and is related to
gluten sensitivity.
Intestinal lymphangiectasia (choice B) shows dilated lymphatic channels on
biopsy.
Lactase deficiency (choice C) is due to an enzymatic abnormality and is
characterized by a histologically normal intestinal epithelium.
Tropical sprue (choice D) usually shows moderate broadening and
shortening of villi, and an often intense inflammatory reaction.



Explanation - Q: 7.2 Close

The correct answer is D. Electron microscopy of the intestinal mucosa
shows that the PAS-positive granules in macrophages in Whipple disease
are actually bacterial forms. The causative organism is Tropheryma whippelii,
and, if necessary, its DNA can be identified with PCR technology. This is
necessary because attempts to culture the organism have not, to date, been
successful. At the moment, this test is available only through reference
laboratories. In patients who have extraintestinal manifestations,
macrophages containing the bacteria may be found in extraintestinal sites.
Bartonella(choice A) species can cause cat scratch disease, trench fever,
and disseminated infections.
Borrelia(choice B) causes Lyme disease.
Francisella(choice C) causes tularemia.
Vibrio(choice E) causes cholera and other diarrheal illnesses.



Explanation - Q: 7.3 Close

The correct answer is D. When a patient complains of excessive flatulence,
the first step is to ask them to describe the amount and frequency of the
flatulence, since some people with normal intestinal gas production have
unrealistic expectations about the possibility of completely eliminating
flatulence. In cases in which excessive flatulence really is present, you
should think of gastrointestinal disorders that can cause malabsorption, since
many cases of excessive flatulence are related to bacterial digestion of
unabsorbed nutrients (notably carbohydrates) with resultant gas production
as a by-product. These disorders may include a wide variety of underlying
conditions including pancreatic disease, liver disease, genetic enzymatic
abnormalities of the intestinal mucosa, and acquired intestinal tract disease.
Colonic obstruction (choice A), small intestinal obstruction (choice E), and
GI hypomotility (choice C) can also cause gas to be retained in the bowel
and be visible on abdominal x-ray films, but do not, by themselves, increase
gas production.
GI hypermotility (choice B) often accompanies acute diarrheal illness, but
the excessive flatulence that may also be present in that setting is secondary
to malabsorption rather than the hypermotility.

Question 4 of 5
BIood studies on this patient demonstrate a microcytic hypochromic anemia. The
nutrient whose deficiency would most likely account for the
patient's anemia is primarily absorbed in which of the following sites?
/ A. Colon
/ B. Duodenum
/ C. Esophagus
/ D. IIeum
/ E. Stomach

Explanation - Q: 7.4 Close

The correct answer is B. Microcytic, hypochromic anemia suggests iron
deficiency. Patients with Whipple disease frequently develop anemia related
to poor duodenal absorption of either folate (producing a macrocytic anemia)
or iron. Other substances absorbed in the duodenum include water, calcium,
fats, sugars, proteins, many vitamins, magnesium, and sodium.
The colon (choice A) absorbs water and electrolytes.
The esophagus (choice C) does not absorb nutrients.
The ileum (choice D) can absorb bile salts, vitamin B12, and chloride.
The stomach (choice E) can absorb water and alcohol.

Question 5 of 5

This patient's illness is most effectively treated with which of the following?
/ A. Antibiotics
/ B. Chemotherapy
/ C. Radiation therapy
/ D. Steroid therapy
/ E. Surgery

Explanation - Q: 7.5 Close

The correct answer is A. The symptoms of Whipple disease often
dramatically improve after antibiotic therapy, although the therapy must often
be continued for prolonged intervals to prevent relapse and the microscopic
intestinal changes may not resolve for 2 years. Many antibiotics are effective,
including chloramphenicol, tetracycline, ampicillin, penicillin, and
trimethoprim-sulfamethoxazole. The other therapies listed in the choices are
not usually employed with Whipple disease.










Within a two-day period, a pediatrician in a rural community sees sixteen children
between the ages of 2 and 6 with gastroenteritis. The illness
began with a fever and abdominal pain. The diarrhea was initially watery, but in
some patients, subsequently became mucoid and bloody.
Three days before the onset of illness, all of the children had attended a picnic at
a city park where they played in a wading pool later found to
have an inadequate level of chlorination. Stool cultures performed on these
children show a nonmotile gram-negative rod that does not ferment
Iactose.


Question 1 of 4

Which of the following organisms is the most likely cause of the illnesses?



Explanation - Q: 8.1 Close

The correct answer is D. Shigella is a common cause of bacterial diarrhea
worldwide. It is an obligate human pathogen. In children, Shigella
gastroenteritis typically presents with diarrhea, abdominal cramps, and fever.
The stools are characteristically small in volume, bloody, and mucoid.
Shigella species are members of the family Enterobacteriaceae. They are
nonmotile and nonlactose fermenters. There are 4 serogroups of Shigella: S.
dysenteriae, S flexneri, S. boydii and S. sonnei. The severity of the disease
varies with the infecting serogroup. S. sonnei causes mild disease. S.
dysenteriae and S. flexneri commonly cause dysentery. In the United States,
S. sonnei and S. flexneri infections are most common, and most cases of
shigellosis occur in daycare centers or residential institutions. Infections
caused by S. dysenteriae are usually imported from Mexico or Central
America.
Campylobacter jejuni(choice A) is a leading cause of acute diarrhea
worldwide. Campylobacter gastroenteritis is essentially a food-borne
disease, although infection also can be acquired through direct contact with
animals or their products. Campylobacter jejuni inhabits the intestinal tracts
of a wide range of animals, especially poultry. The most common clinical
features in children are diarrhea, fever, abdominal pain, and vomiting. Blood
in the stools is present in more than one-half of the children with
Campylobacter gastroenteritis. Campylobacter needs to be cultured on
specific selective media, and incubated under microaerophilic and
capnophilic conditions. They have a distinctive appearance on Gram's stain.
They are gram-negative, seagull-shaped organisms. Microscopic
appearance as well as testing for oxidase and catalase production are all
that is needed to confirm a diagnosis.
Enterohemorrhagic E. coli (EHEC)(choice B) was first noted as a pathogen
in the early 1980s. It was found to be the cause of an outbreak of bloody
diarrhea acquired after eating hamburgers at a fast food chain. EHEC strains
colonize cattle, and the transmission of EHEC is most commonly associated
with the consumption of undercooked hamburger meat. This is probably
because grinding distributes the bacteria throughout the meat rather than
leaving it on the surface where it could easily be killed by cooking. However,
EHEC outbreaks have also been associated with contaminated lettuce,
sprouts, and apple cider. E. coli is a member of the Enterobacteriaceae. It is
motile and ferments lactose. The enterohemorrhagic E. coli causing the early
outbreaks belonged to serogroup O157:H7, which could be distinguished
from other E. coli by its inability to ferment sorbitol. However, in the last few
years, major outbreaks of non-O157 EHEC disease have occurred in several
countries.
Salmonella enteritidis(choice C) is the number one cause of food-borne
illness in the United States, and is responsible for more deaths than any
other food-borne pathogen. Salmonella enteritidis has many animal
reservoirs. Typical sources of human infection are poultry, and especially
eggs, since the bacteria can be passed transovarially from chickens to intact
eggs. Salmonella is a member of the Enterobacteriaceae. It is motile and a
nonlactose fermenter.
Yersinia enterocolitica(choice E) can cause both outbreaks and sporadic
cases of gastroenteritis. Bloody diarrhea, fever, and vomiting are more
common in children than adults. It is also a member of the family
Enterobacteriaceae. It is motile and a nonlactose fermenter.
Question 2 of 4
Which of the following is the infecting dose of the organism needed to cause this
disease?
/ A. 10 to 100 organisms
/ B. 104-106 organisms
/ C. 105-108organisms
/ D. 108organisms
/ E. 109 organisms

Explanation - Q: 8.2 Close

The correct answer is A. Data on the number of organisms required for
clinical illness was obtained from studies on human volunteers and
outbreaks in which the source of infection could be quantitatively cultured.
Shigella is not as susceptible to acid as many other bacterial pathogens.
Because of this, Shigella can survive passage through the stomach, so only
a few organisms (10 to 100 organisms) are needed to cause disease. Since
the amplification of bacterial numbers in contaminated food or water is not
essential to achieve an infectious dose, Shigella can be transmitted directly
from person-to-person, as well as from contaminated food and water.
10
4
to 10
6
organisms (choice B) is the infectious dose for Campylobacter
jejuni.
10
5
-10
8
organisms (choice C) is the infectious dose for Salmonella
enteritidis.
10
8
organisms (choice D) is the infectious dose for enteropathogenic E. coli.
10
9
organisms (choice E) is the infectious dose for Yersinia enterocolitica.


Explanation - Q: 8.3 Close

The correct answer is B. Shiga toxin and Shiga-like toxin are two
component cytotoxins that inhibit cellular protein synthesis by enzymatically
cleaving 28S rRNA, which prevents EF-dependent binding of tRNA to
ribosomes. Shiga toxin is found in Shigella dysenteriae. Shiga-like toxin
differs from Shiga toxin by one amino acid, and is found in other species of
Shigella and in enterohemorrhagic E. coli (EHEC), where it is carried on a
lysogenic phage. All Shigella also require a large virulence plasmid, which
contains genes that are required for invasion, and intra- and intercellular
spread.
Cytolethal distending toxin (choice A) is a single-stranded DNAase made by
Campylobacter jejuni.
A heat stable enterotoxin that causes vomiting (choice C) describes the
enterotoxin of Staphylococcus aureus. It acts on neural receptors in the
upper GI tract leading to stimulation of vomiting center.
A plasmid-encoded enterotoxin that stimulates adenylate cyclase (choice D)
describes LT toxin of enteropathogenic E. coli. LT toxin is 80% homologous
with cholera toxin and has the same mechanism of action. The B subunits
bind to a ganglioside on the cell membrane and the A subunit is then
internalized, where it catalyzes the ADP-ribosylation of Gs protein, destroying
the mechanism to turn off the stimulation of adenyl cyclase. This results in
increased levels of cAMP, leading to an increase in prostaglandin E and the
secretion of salt and water.
A tyrosine phosphatase induced by low calcium concentrations (choice E)
describes the YopH gene product of Yersinia. This tyrosine phosphatase is
broad-spectrum and dephosphorylates all tyrosine-phosphorylated host
proteins, subverting phagocytosis.


Question 4 of 4

AIthough infection with this pathogen is generally self-Iimited, several of the
children require treatment with trimethoprim-sulfamethoxazole. This
medication works via which of the following mechanisms?

/ A. Inhibits DNA gyrase
/ B. Inhibits folate synthesis
/ C. Inhibits protein synthesis by binding to the 30s ribosomal subunit
/ D. Inhibits protein synthesis by binding to the 50s subunit
/ E. Inhibits RNA polymerase

Explanation - Q: 8.4 Close

The correct answer is B. Both trimethoprim and sulfamethoxazole inhibit
microbial folate synthesis. When combined, they have a synergistic effect.
Inhibiting DNA gyrase (choice A) is the mechanism of action of the
fluoroquinolones, such as ciprofloxacin and levofloxacin.
Inhibiting protein synthesis by binding to the 30s ribosomal subunit (choice
C) is the mechanism of action of the tetracyclines such as doxycycline and
minocycline.
Inhibiting protein synthesis by binding to the 50s subunit (choice D) is the
mechanism of action of the macrolides, which include erythromycin and the
long acting macrolides, azithromycin and clarithromycin.
Inhibiting RNA polymerase (choice E) is the mechanism of action of the
rifamycins, rifampin, and rifabutin.




A 32 year-old successful female lawyer in a ball gown is dropped off at the
emergency room. She complains of an unusual sensation in her
chest and recently had a nose bleed. She is irritable and talks about "Iiberals,"
trying to shut her office down because she "stands for what is
right." She explains being dropped off at the emergency room as a conspiracy
against her. She reports that tonight's party was to celebrate
her promotion to full partner. She throws her supper tray across the room and
says "Oh please, you don't think l would be so stupid as to eat
that, I know what you are up to." When interviewed, she refuses to speak to the
doctor who enters and then lunges at the "jealous hag trying to
frame her." The patient refuses vital signs but is noted to have mydriasis.
Question 1 of 6
Which of the following is the most appropriate pharmacotherapy?
/ A. Lorazepam
/ B. Methylphenidate
/ C. Midazolam
/ D. Sertraline
/ E. Zolpidem

Explanation - Q: 1.1 Close

The correct answer is A. Lorazepam is often used for treating agitation in
the emergency room due to its quick absorption. Onset of action is within
about thirty minutes.
Methylphenidate (choice B) is a stimulant used to treat attention deficit
disorder. Stimulants would worsen agitation and psychosis.
Midazolam (choice C) is a rapid acting parenteral benzodiazepine used in
anesthesiology. It is not indicated for psychiatry disorders.
Sertraline (choice D) is a SSRI, used to treat depression and anxiety
disorders. Treatment of agitation is the primary goal at this point.
Zolpidem (choice E) is an imidazopyridine that binds the GABA-
benzodiazepine receptor complex. It is used to induce sleep.
Question 2 of 6
This patient is at risk for which of the following complications?
/ A. Anxiety and amotivational syndrome
/ B. Ataxia and respiratory depression
/ C. Bone marrow depression and immunosuppression
/ D. Brain damage and peripheral neuropathies
/ E. Intracranial hemorrhage and myocardial infarction



Explanation - Q: 1.2 Close

The correct answer is E. The patient's symptoms are consistent with
cocaine intoxication. Pupillary dilation, hypertension, tachycardia,
hallucinations, aggressive behavior and paranoia can all be seen with
amphetamine or cocaine use. Acutely, cocaine ingestion can produce
intracranial hemorrhage or myocardial infarction in certain individuals. Long-
term snorting can lead to nosebleeds.
Anxiety and amotivational syndrome (choice A) is commonly associated with
long term marijuana use.
Ataxia and respiratory depression(choice B) is associated with alcohol,
barbiturate or benzodiazepine use.
Brain damage and peripheral neuropathies (choice D) and bone marrow
depression and immunosuppression (choice C) are complications of inhalant
(glues, solvents and cleaners) abuse.


Question 3 of 6
If, in addition to the above history, the patient is also pregnant, which of the
following complications might occur?
/ A. CIeft palate abnormalities
/ B. Congenital hypothyroidism
/ C. Ebstein's anomaly
/ D. Neural tube defects
/ E. PIacental abruption

Explanation - Q: 1.3 Close

The correct answer is E. Maternal cocaine use is associated with placental
abruption. Drug-induced hypertension may play a role.
Cleft palate abnormalities (choice A) are associated with carbamazepine.
Ebstein's anomaly (choice C) and congenital hypothyroidism (choice B) are
side effects of lithium.
Neural tube defects (choice D) are associated with valproic acid.

Question 4 of 6
Which of the following most accurately describes the mechanism responsible for
this patient's presentation?
/ A. BIockade of dopamine receptors
/ B. BIockade of NMDA receptors
/ C. Increased affinity of binding to GABA receptors
/ D. Increased release of monoamines in the synaptic cleft
/ E. Inhibition of monoamine reuptake in the synaptic cleft


Explanation - Q: 1.4 Close

The correct answer is E. Cocaine inhibits the reuptake of norepinephrine,
dopamine, and serotonin (monamines) from the synaptic cleft.
Blockade of dopamine receptors (choice A) is the mechanism of action
responsible for the efficacy and some side effects of the antipsychotic
medications.
Blockade of NMDA receptors (choice B) is the mechanism of action of PCP.
Increased affinity of binding to GABA receptors (choice C) is a mechanism
of both benzodiazepines and barbiturates, in addition to potentiation of the
chloride channels.
Increased release of monamines (choice D) is the mechanism of action of
amphetamines. Amphetamine intoxication would be less likely than cocaine
intoxication in this successful professional woman. The recent history of a
nose bleed also tends to suggest cocaine abuse.
Question 5 of 6
The patient responds very little to the first medication, but begins pacing and
ranting at the nurses. When a police officer appears, she attacks
her when she hallucinates and hears a voice saying the officer is "really working
for the other side." Which of the following is the most
appropriate pharmacotherapy?
/ A. Amitriptyline
/ B. CIonazepam
/ C. Cyproheptadine
/ D. Haloperidol
/ E. Propanolol

Explanation - Q: 1.4 Close

The correct answer is E. Cocaine inhibits the reuptake of norepinephrine,
dopamine, and serotonin (monamines) from the synaptic cleft.
Blockade of dopamine receptors (choice A) is the mechanism of action
responsible for the efficacy and some side effects of the antipsychotic
medications.
Blockade of NMDA receptors (choice B) is the mechanism of action of PCP.
Increased affinity of binding to GABA receptors (choice C) is a mechanism
of both benzodiazepines and barbiturates, in addition to potentiation of the
chloride channels.
Increased release of monamines (choice D) is the mechanism of action of
amphetamines. Amphetamine intoxication would be less likely than cocaine
intoxication in this successful professional woman. The recent history of a
nose bleed also tends to suggest cocaine abuse.


Question 5 of 6
The patient responds very little to the first medication, but begins pacing and
ranting at the nurses. When a police officer appears, she attacks
her when she hallucinates and hears a voice saying the officer is "really working
for the other side." Which of the following is the most
appropriate pharmacotherapy?
/ A. Amitriptyline
/ B. CIonazepam
/ C. Cyproheptadine
/ D. Haloperidol
/ E. Propanolol

Explanation - Q: 1.5 Close

The correct answer is D. Haloperidol is often used in the emergency setting
and the ICU to treat psychotic aggression due to its rapid onset of action and
multiple formulations (oral, IV, IM).
Amitriptyline (choice A) is a tricyclic antidepressant with marked
anticholinergic properties. It would not be appropriate for this patient.
Clonazepam (choice B) does not have a rapid onset of action and is not
used in this setting. More rapidly acting benzodiazepines can be used in the
emergency setting. This patient had assaulted someone due to her psychotic
state and the psychosis needs treatment.
Cyproheptadine (choice C) has been used to target the sexual side effects
induced by treatment with SSRIs.
Propranolol (choice E) is a nonspecific beta-blocker. It is useful for "stage
fright", but would not be effective to treat this patient's psychotic aggression.
Question 6 of 6
The patient refuses the medication and staff overhears her say she plans to "get
rid of the imposter, once and for alI," to "make the world a
safe place again." The patient escapes from the emergency department. The
husband usually returns from working the night shift at this time of
day. In compliance with the Tarasoff-II Supreme Court Decision, the doctor is
expected to do which of the following?
/ A. Chase the patient
/ B. Dispatch the police to warn the husband
/ C. Document the findings
/ D. Leave a message for the husband
/ E. Take out papers to have the patient involuntarily committed

Explanation - Q: 1.6 Close

The correct answer is B. The Tarasoff-II Supreme Court decision indicates
a duty to protect potential victims (not just warn them, as found in the original
Tarasoff ruling). The treating physician "bears a duty to exercise reasonable
care to protect the foreseeable victim of that danger."
Chasing a patient (choice A) is dangerous, inappropriate, and outside the
bounds of the safe practice of medicine.
Documenting the findings (choice C) is crucial, but it does not meet the
requirements of the Tarasoff rulings.
Leaving a message for the husband (choice D) does not meet the
requirements of the Tarasoff rulings.
Taking out papers to have the patient involuntarily committed (choice E) is a
crucial step, and the patient is a danger to others, but it may take several
hours for the papers to be signed, before the police can look for the patient.
Tarasoff indicates that the foreseeable victim should receive a warning if
he/she is in danger.


A 29-year-old man is brought via EMS to the emergency department. Bystanders
report the patient was staggering and mumbling, and then
collapsed. The paramedic recognizes the patient, and believes the patient to be a
homeless drug abuser. Past medical history and allergies
are unknown. BIood pressure is 130/75 mm Hg, pulse is 60/min, and respirations
are 8/min. The patient is comatose, with eyes closed, and
does not withdraw to pain. He does not smell of alcohoI. He has multiple tattoos
and needle tracks on both arms. Pupils are pinpoint and
sclerae are not jaundiced. Respirations are shallow. Abdominal examination is
unremarkable.
Question 1 of 4
Which of the following is the most appropriate pharmacotherapy?
/ A. Deferoxamine
/ B. FIumazenil
/ C. N-Acetylcysteine
/ D. Naloxone
/ E. Naltrexone


Explanation - Q: 2.1 Close

The correct answer is D. Hypoventilation, pupillary miosis, and a comatose
state is highly suggestive of an opioid overdose. The opioid in question is
probably heroin, given the needle tracks on his arms, Naloxone, an opioid
antagonist, is used to treat the acute opioid overdose.
Deferoxamine (choice A) is used to treat iron overload.
Flumazenil (choice B) is used to treat benzodiazepine overdose.
N-Acetylcysteine (choice C) is used to treat acetaminophen overdose. This
might also be appropriate had the patient ingested an excess of an
acetaminophen-opioid combination (e.g., hydrocodone and acetaminophen).
Naltrexone (choice E) is used to prevent relapse in previously opiate-
dependent individuals.



Question 2 of 4

Which of the following is frequently present in patients with this condition?
/ A. Aseptic necrosis of the hip
/ B. Diabetes insipidus
/ C. Hepatitis C
/ D. Intermittent porphyria
/ E. Subacute bacterial peritonitis

Explanation - Q: 2.2 Close

The correct answer is C. Hepatitis B, C, and HIV are all associated with IV
drug use. This patient has needle marks on his arms and his presentation is
consistent with opioid overdose.
Aseptic necrosis of the hip (choice A) is associated with long term steroid
treatment.
Diabetes insipidus (choice B) is associated with lithium.
Intermittent porphyria (choice D) can be precipitated by barbiturate intake in
vulnerable persons.
Subacute bacterial peritonitis (choice E) is associated with ascites and other
signs of cirrhotic liver disease. This patient does not smell of alcohol, and the
examination did not reveal ascites, caput medusa, edema, or bulging flanks.

Question 3 of 4
The patient is admitted to the hospitaI. An arterial blood gas is most likely to
show which of the following?
/ A. Metabolic acidosis
/ B. Metabolic alkalosis
/ C. Marked anion gap
/ D. Respiratory acidosis
/ E. Respiratory alkalosis

Explanation - Q: 2.2 Close

The correct answer is C. Hepatitis B, C, and HIV are all associated with IV
drug use. This patient has needle marks on his arms and his presentation is
consistent with opioid overdose.
Aseptic necrosis of the hip (choice A) is associated with long term steroid
treatment.
Diabetes insipidus (choice B) is associated with lithium.
Intermittent porphyria (choice D) can be precipitated by barbiturate intake in
vulnerable persons.
Subacute bacterial peritonitis (choice E) is associated with ascites and other
signs of cirrhotic liver disease. This patient does not smell of alcohol, and the
examination did not reveal ascites, caput medusa, edema, or bulging flanks.

Question 3 of 4
The patient is admitted to the hospitaI. An arterial blood gas is most likely to
show which of the following?
/ A. Metabolic acidosis
/ B. Metabolic alkalosis
/ C. Marked anion gap
/ D. Respiratory acidosis
/ E. Respiratory alkalosis


Explanation - Q: 2.3 Close

The correct answer is D. Opioids are central CNS depressants, and are
associated with hypoventilation and CO
2
retention, which results in
respiratory acidosis.
Metabolic acidosis (choice A) is associated with substrates such as ketones,
uric acid, salicylates, methyl alcohol, ethylene glycol, and lactic acid.
Metabolic alkalosis (choice B) is associated with loss of acid (vomiting) or
excess bicarbonate or alkali.
A marked anion gap (choice C) is seen in diabetic ketoacidosis, among
other causes.
Respiratory alkalosis (choice E) is associated with hyperventilation and loss
of CO
2
.


Question 4 of 4
Which of the following treatments can be used for long-term maintenance after
this patient is discharged from the hospitaI?
/ A. Disulfiram
/ B. FIumazenil
/ C. Naloxone
/ D. Naltrexone
/ E. Polyethylene glycol

Explanation - Q: 2.4 Close

The correct answer is D. Naltrexone, an opioid antagonist with a longer
half-life than naloxone, is used to prevent relapse in previously opiate-
dependent individuals. It is most successful in the highly motivated patient.
Naltrexone can also be used in combination with clonidine for rapid
detoxification. Clonidine, an alpha 2 agonist, decreases some of the
autonomic symptoms during withdrawal. Later, the patient can alternatively
be maintained on opioid maintenance agonists, such as methadone, levo-
alpha-acetylmethadol (LAAM), or buprenorphine.
Disulfiram (choice A) is used to prevent relapse with alcohol. Persons who
drink alcohol become very ill due to inhibition of aldehyde dehydrogenase.
Flumazenil (choice B) is used to treat benzodiazepine overdose.
Naloxone (choice C) is used to treat acute opioid overdose.
Polyethylene glycol (choice E) is a laxative, which produces whole bowel
irrigation. This may be indicated for patients with massive oral overdoses of
opioids, or in "body packers" (people who smuggle heroin, often in balloons,
in their GI tract) if one of the packages ruptures.


A 46-year-old homeless man is brought to the emergency department after the
EMS receive a call to evaluate a "man down" on the street. No
one had witnessed the event, but the paramedic recognizes the patient as a
chronic substance abuser with a history of alcohol and drug use.
Past medical history is significant for cirrhosis. On examination, the patient is
somnolent, but arousable and is not confused. He smells of
alcohoI, is mildly jaundiced and says "Ieave me alone, Iet me go, I just fell
asleep." His blood pressure is 140/86 mm/Hg, pulse is 78/min, and
respirations are 15/min. The patient refuses laboratory tests and states "I'd rather
die than go back in the hospitaI." The staff knows the patient
has changed his mind in the past and allows the patient to sleep it off for a few
hours. The patient awakens, has a fight with the nurse and
twenty minutes later is found extremely difficult to arouse with an empty bottle of
clonazepam on the floor.


Question 1 of 5

Which of the following is the most appropriate pharmacotherapy?
/ A. Acetylcysteine
/ B. Deferoxamine
/ C. FIumazenil
/ D. Naloxone
/ E. Naltrexone


Explanation - Q: 3.1 Close

The correct answer is C. Flumazenil is used to reverse benzodiazepine
(e.g., clonazepam) overdose. This agent competitively inhibits the action of
benzodiazepines at the benzodiazepine recognition site on the
GABA/benzodiazepine receptor complex. Importantly, flumazenil does not
antagonize the effects of other CNS drugs affecting GABAergic neurons
including ethanol, barbiturates, opioids, or general anesthetics.
Acetylcysteine (choice A) is used in acetaminophen overdose.
Deferoxamine (choice B) is used in iron overdose.
Naloxone (choice D) is used in opioid overdose.
Naltrexone (choice E) is used to promote abstinence from opioid abuse.


Question 2 of 5

The patient recovers from the event in the emergency department and is
admitted to the ICU to evaluate possible gastric bleeding. When
asked about a history of drug and alcohol use, the patient gives a long history of
"taking everything l could get my hands on." Which of the
following substances is associated with a fatal withdrawal syndrome?
/ A. CIozapine
/ B. CIonazepam
/ C. Cocaine
/ D. Heroin
/ E. Pentobarbital

Explanation - Q: 3.2 Close

The correct answer is E. Barbiturate withdrawal (similar to alcohol
withdrawal) can lead to delirium, convulsions, and death. Pentobarbital is
given in a "challenge" or test dose to estimate the amount of barbiturate
needed to adequately treat withdrawal.
Clozapine (choice A) is an antipsychotic medicine reserved for refractory
schizophrenia; seizures may develop if doses are missed.
Clonazepam (choice B) is a benzodiazepine with a long half-life. Withdrawal
is associated with anxiety and insomnia.
Cocaine (choice C) can be fatal in overdose, but not withdrawal. Withdrawal
symptoms usually include tiredness, craving for cocaine, lack of energy, and
depressed mood (sometimes with suicidality).
Heroin (choice D) can be fatal in overdose, but not withdrawal. Opioid
detoxification can be treated symptomatically (clonidine, phenergan,
hyoscyamine, ibuprofen, or cyclobenzaprine). Patients in opioid withdrawal
have extreme discomfort from nausea, diarrhea, and myalgia often
accompanied by depressed mood. Withdrawal is extremely unpleasant, but
generally not fatal.

Question 3 of 5

The patient gives a history of a code team revival after taking an intentional
overdose on "vodka, sleeping pills, and pain-killers." Which of the
following physiologic processes is usually responsible for a fatal overdose on
these substances?
/ A. Agranulocytosis
/ B. Hypertensive crisis
/ C. QTc prolongation
/ D. Respiratory depression
/ E. Stevens-Johnson syndrome

Explanation - Q: 3.3 Close

The correct answer is D. Respiratory depression is a property shared by
alcohol, benzodiazepines, barbiturates, opiates, and drugs like chloral
hydrate (an older sedative-hypnotic). The effects of these different drugs are
additive, making them even more dangerous.
Agranulocytosis (choice A) is a rare (about 1%) side effect associated with
clozapine treatment. Undetected infections can be fatal. Patients taking
clozapine receive frequent complete blood counts.
Hypertensive crisis (choice B) is associated with tyramine ingestion
concurrent with monoamine oxidase inhibitor treatment. It can be fatal.
QTc prolongation (choice C) is associated with several older antipsychotic
medications (thioridazine, droperidol) and especially with tricyclic
antidepressants. QTc prolongation is associated with the development of
Torsades de Pointes and cardiac arrest.
Stevens-Johnson syndrome (choice E) is associated with carbamazepine,
an anticonvulsant used to treat bipolar disorder. Stevens-Johnson syndrome
has also been reported to be associated with some of the newer
anticonvulsant medications as well.
Question 4 of 5

After four days in the hospitaI, the patient develops elevated blood pressure,
tachycardia, severe tremor, and states that "the last time l
stopped drinking l started seeing things and had seizures." Which of the following
is appropriate pharmacotherapy in this patient?
/ A. Diazepam
/ B. CIorazepate
/ C. FIurazepam
/ D. Halazepam
/ E. Lorazepam

Explanation - Q: 3.4 Close

The correct answer is E. The patient's report is classic for alcohol
withdrawal with impending delirium tremens, and benzodiazepines are
indicated. Lorazepam has a shorter half-life than the other drugs listed and
no active metabolites. It is metabolized only by phase II (conjugation) in the
liver. Benzodiazepines metabolized by phase II only are better tolerated in
patients with impaired liver function (elderly patients, patients with cirrhosis).
In summary, lorazepam is the drug of choice when accumulation is a
concern, because lorazepam has a short half-life, no active metabolites, and
is biotransformed only by phase II metabolism. The remaining drugs
(choices A, B, C, and D) all undergo phase I metabolism also. Temazepam
and oxazepam are examples of other benzodiazepines that are metabolized
by phase II only


Question 5 of 5
The patient recovers and returns to the hospital after sustaining an unwitnessed
head injury. Initial workup reveals no changes in mental status
or findings on CT of the head, but the patient develops a seizure disorder and is
started on carbamazepine, and phenobarbital is added six
weeks later. The patient is at risk for which of the following?
/ A. Decreased carbamazepine levels
/ B. Gingival hyperplasia
/ C. Hypernatremia
/ D. Increased phenobarbital levels
/ E. Vitamin D toxicity

Explanation - Q: 3.5 Close

The correct answer is A. Carbamazepine levels decrease over time by
p450 autoinduction, and also the addition of phenobarbital (one of the most
potent of enzyme-inducers) will decrease the levels of other anticonvulsants
the patient is concurrently taking.
Gingival hyperplasia (choice B) is associated with phenytoin administration.
Hypernatremia (choice C) is not associated with these medicines, but
carbamazepine and the newer anticonvulsants are associated with hypo-
osmolar hyponatremia.
Carbamazepine does not inhibit P450 (actually, it induces P450) and
therefore would not increase levels of phenobarbital (choice D).
Vitamin D (choice E) can be depleted by carbamazepine, but vitamin D is
not increased (or toxic). Carbamazepine can also decrease folate levels.





A 25-year-old woman consults a physician because she has developed severe
dysphagia and constipation that has led to a recent weight loss
of 15 pounds. She is referred to a gastroenterologist, to whom she reports that
she has a great deal of trouble swallowing solid foods, but not
Iiquids, and has also been having nearly constant gastroesophageal reflux
symptoms. She has awakened during the night several times
coughing and feeling as if she had aspirated acid stomach contents.
Esophagogastroduodenoscopy and colonoscopy are performed, and
demonstrate massive dilatation of the esophagus and colon.
Question 1 of 5
Which of the following parasitic diseases would most likely produce
megaesophagus and megacolon?
/ A. Chagas disease
/ B. Cysticercosis
/ C. Hydatid disease
/ D. Malaria
/ E. Threadworm infection

Explanation - Q: 1.1 Close

The correct answer is A. Megaesophagus and megacolon are unusual
pathologic changes, and should bring Chagas disease to mind, which is also
known as American trypanosomiasis. Megacolon, but not megaesophagus,
can also be due to Hirschsprung disease, but this is usually diagnosed in
infants and young children and is due to abnormal neural development in the
gut.
Cysticercosis (choice B) can cause cyst formation in the brain and other
organs.
Hydatid disease (choice C) can cause cysts in the liver and other organs.
Malaria (choice D) can be complicated by fever, chills, anemia,
splenomegaly, and in severe cases, renal failure, coma, thrombocytopenia,
and respiratory distress syndrome.
Threadworm infection (choice E) can cause skin rashes, eosinophilia,
abdominal pain, and, in severe cases, massive GI bleeding, severe
malnutrition, and severe pulmonary symptoms
Question 2 of 5
The infecting species is which of the following?
/ A. FIuke
/ B. Nematode
/ C. PIasmodia
/ D. Tapeworm
/ E. Trypanosome

Explanation - Q: 1.2 Close

The correct answer is E. Chagas disease is caused by infection with
Trypanosoma cruzi, which is an intracellular protozoan parasite that takes a
leishmanial form within muscle cells.
Examples of fluke (choice A) or trematode diseases include schistosomiasis
(blood flukes), paragoniasis (lung fluke), and various liver flukes (e.g.,
clonorchiasis, fascioliasis, fasciolopsiasis).
Examples of nematode (choice B), or round worm diseases, include the
various intestinal nematodes (e.g., ascariasis, trichuriasis, ancylostomiasis,
strongyloidiasis), tissue nematodes (e.g., toxocariasis, trichinosis,
dracunculosis), and the filarial nematodes (e.g., elephantiasis,
onchocerciasis).
Plasmodia (choice C) cause malaria.
Tapeworm infections (choice D) include intestinal infections
(diphyllobothriasis, Taenia infections), cysticercosis (due to Taenia solium),
and Echinococcus infections.


Question 3 of 5
On further questioning, the woman reports having lived for a year in Bolivia as a
teenager. At one point, she had had an automobile accident
and had been transfused with blood at a local hospital there. The person whose
blood she received had probably acquired the infection via a
bite by which of the following?
/ A. FIea
/ B. Mosquito
/ C. Reduviid bug
/ D. Tick
/ E. Tsetse fly

Explanation - Q: 1.3 Close

The correct answer is C. Reduviid bugs are insects that live in poorly
constructed adobe housing and feed on human or animal blood.
Trypanosomes they ingest during their feedings multiply in the insect gut and
are deposited in fecal material near the wound when the insect bites another
individual. Contamination through the wound leads to invasion into, and
reproduction within, host cells. Release of the trypanosomes back into the
blood stream makes them available for transmission either via another bug
bite or via blood transfusion. Transmission of Chagas disease via blood
transfusion has also become an important mode of transmission in South
America. Santa Cruz, Bolivia is thought to have a prevalence of infected
blood in blood banks of 53%, which is much higher than the rate of hepatitis
or HIV infection. In other parts of South America, the infection rate is much
lower, due primarily to a massive attempt by the World Health Organization
to eliminate both the reduviid bug (through better housing and insecticide
use) and screen for Chagas disease in blood products in endemic regions.
These attempts have been most successful in eradicating or nearly
eradicating the infection in Chile, Uruguay, and Brazil. Transmission of
Chagas disease via blood transfusions has not been a significant problem in
the United States, primarily because one of the questions asked before a
blood donation is taken is whether or not a person has traveled outside the
United States during the previous year. While Chagas disease can persist for
decades, the period in which significant numbers of parasites are in the
blood is usually only during the comparatively short (and often
asymptomatic) acute phase of the illness.
Flea-borne illnesses (choice A) to remember include bubonic plague and
typhus.
Associate mosquito bites (choice B) with malaria.
Associate tick bites (choice D) with Lyme disease, babesiosis, and Rocky
Mountain spotted fever.
Associate Tsetse fly bites (choice E) with African trypanosomiasis (sleeping
sickness).


Question 4 of 5
While this woman's gastrointestinal tract disease brought her to medical
attention, involvement of which of the following is the principal source
of morbidity and mortality in this condition?
/ A. Heart
/ B. Kidneys
/ C. Liver
/ D. Lungs
/ E. Pancreas

Explanation - Q: 1.4 Close

The correct answer is A. Chagas disease is conventionally divided into
three stages. The short acute period may be asymptomatic or may be
characterized by fever, swelling of lymph glands, hepatosplenomegaly, and
local inflammation at the site of the bite. This is followed by an asymptomatic
period, which may last years to the rest of the patient's life. In one-third of
patients, clinically evident disease becomes apparent 10-20 years after
infection. 27% of infected patients develop cardiac symptoms (which may
cause flaccid cardiomyopathy, cardiac aneurysm formation, or sudden
death); 6% develop digestive system damage with megaviscera (thought to
be related to neural involvement rather than muscle involvement); and 3%
develop neural involvement. You should also be aware that coexisting AIDS
infection may predispose for an unusually rapid and severe course of
Chagas disease.
The kidneys (choice B), liver (choice C), lungs (choice D), and pancreas
(choice E) are not particular targets in this disease.


Question 5 of 5
While no drug therapy is useful in the therapy of chronic cases, treatment can be
used in newly diagnosed acute cases. Which of the following
medications would be most useful in treating the acute infection?
/ A. Chloroquine
/ B. Doxycycline
/ C. Nifurtimox
/ D. Primaquine
/ E. Pyrimethamine-sulfadoxine

Explanation - Q: 1.5 Close

The correct answer is C. The diagnosis in suspected acute cases can be
established with review of thin or thick blood smears. (The diagnosis in latent
and chronic cases usually requires special blood culture techniques or PCR-
amplified detection of parasite DNA in blood.) The only effective drugs are
nifurtimox and benznidazole, which are given in courses up to 4 months
because the organisms are difficult to eradicate. The other choices listed are
drugs used in malaria prophylaxis or treatment.


A 65-year-old man presents to his physician because he has been having
increasing difficulty swallowing over the past 2 months. He is still
able to swallow liquids, but swallowing solid food now causes severe pain and a
sense of fullness behind his sternum. He has lost 18 pounds
since his swallowing difficulties began. The patient is referred to a
gastroenterologist, who demonstrates a mass lesion of the distal
esophagus, which on biopsy is shown to contain cancer.



Question 1 of 6

Which of the following is most important in separating the esophagus from the
larynx, and must consequently be carefully passed behind
during endoscopy?
/ A. Arytenoids
/ B. Cricoid cartilage
/ C. Epiglottis
/ D. Pharynx
/ E. Vocal cords

Explanation - Q: 2.1 Close

The correct answer is C. Endoscopists are very careful when guiding the
endoscope past the epiglottis, which is a pear-shaped portion of elastic
cartilage that can be moved during swallowing to close the larynx, preventing
swallowed material from eventually entering the lungs.
The arytenoids (choice A) are the site of the attachment of the vocal cords
(choice E) within the larynx.
The cricoid cartilage (choice B) is in the more distal portion of the larynx.
The pharynx (choice D) is shared by the respiratory and gastrointestinal
tracts.



Question 2 of 6

Which of the following nerves provides the efferent impulses necessary for the
esophageal actions that occur during swallowing?
/ A. GIossopharyngeal
/ B. Hypoglossal
/ C. Spinal accessory
/ D. Trigeminal
/ E. Vagus

Explanation - Q: 2.2 Close

The correct answer is E. The vagus nerve supplies the efferent input into
the esophagus that is necessary for swallowing.
The glossopharyngeal nerve (choice A) provides taste and sensation on the
palate, but the only muscle it supplies is the stylopharyngeus.
The hypoglossal nerve (choice B) moves the tongue during the initiation of
swallowing, but does not innervate the esophagus.
The spinal accessory nerve (choice C) plays no role in swallowing. This
nerve mediates head and shoulder movement and innervates laryngeal
muscles.
The trigeminal nerve (choice D) provides general sensation to the mouth
and motor innervation to the muscles of mastication.


Question 3 of 6
Which of the following approximately represents the proportion of different
esophageal cancer types now being observed in the United States?
/ A. 1/10 adenocarcinoma and 9/10 squamous cell carcinoma
/ B. 1/3 adenocarcinoma and 2/3 squamous cell carcinoma
/ C. 1/2 adenocarcinoma and 1/2 squamous cell carcinoma
/ D. 2/3 adenocarcinoma and 1/3 squamous cell carcinoma
/ E. 9/10 adenocarcinoma and 1/10 squamous cell carcinoma

Explanation - Q: 2.3 Close

The correct answer is C. More recent statistics indicate that the incidence
of adenocarcinoma and squamous cell carcinoma of the esophagus are now
roughly equal. Formerly, approximately 2/3 of the esophageal cancers were
squamous in origin (choice B). Adenocarcinoma of the esophagus is often
found in the distal esophagus.

Question 4 of 6
Precancerous metaplasia of the esophageal epithelium gives rise to a mucosa
resembling which of the following?
/ A. Mesothelium
/ B. Respiratory epithelium
/ C. Small intestine
/ D. Squamous epithelium
/ E. Stomach

Explanation - Q: 2.4 Close

The correct answer is C. This is an indirect question about Barrett's
esophagus, which is an important precursor of adenocarcinoma of the
esophagus. While Barrett's esophagus was initially defined to be either
gastric-type or intestinal-type metaplasia of the esophagus, more recent
studies have shown that the actual problem lesion is more likely to be
intestinal metaplasia (diagnosed when isolated goblet cells are seen in the
epithelium) rather than gastric metaplasia (choice E).
Metaplasia to mesothelium (choice A) or ciliated respiratory epithelium
(choice B) does not usually occur in the esophagus.
The normal epithelium of most of the esophagus is squamous (choice D).


Question 5 of 6
Frequent use of which of the following has recently been found to probably have
a protective effect against development of esophageal
cancer?
/ A. Acetaminophen
/ B. AIcohol
/ C. Aspirin
/ D. Cigarettes
/ E. Codeine

Explanation - Q: 2.5 Close

The correct answer is C. An interesting new research observation that may
be exploited in the future is that the incidence of esophageal cancer appears
to be much lower in people who use aspirin frequently.
Cigarettes (choice D) and alcohol use (choice B) have been implicated as
risk factors for esophageal cancer.
Acetaminophen (choice A) and codeine (choice E) have no known effects
on the incidence of esophageal cancer.


Question 6 of 6
Currently, esophageal cancer has which of the following long-term survival rates?
/ A. Less than 5%
/ B. 30%
/ C. 50%
/ D. 70%
/ E. More than 95%

Explanation - Q: 2.6 Close

The correct answer is A. Esophageal cancer is one of the very bad
cancers, presently with poor long-term survival. The underlying problem is
that the esophagus is only about 3 mm thick, and both metastatic disease
and direct spread (often unresectable) to mediastinal structures is common.
Active research is presently being undertaken to modify this prognosis by
using chemotherapy and radiation therapy prior to surgery, but these
modalities have not yet come into widespread use.


A 30-year-old man consults a physician because he has been having increasing
difficulty swallowing both solids and liquids. Physical
examination of the patient is noncontributory. Barium swallow studies show a
mostly dilated esophagus with slow passage of barium into the
stomach. The very distal part of the esophagus appears narrowed into a "bird's
beak." Esophageal manometry shows incomplete relaxation of
the lower esophageal sphincter in response to swallowing, high resting lower
esophageal pressure, and absent esophageal peristalsis.

Question 1 of 5

The manometry and barium swallow studies most strongly support which of the
following diagnoses?
/ A. Achalasia
/ B. Adenocarcinoma
/ C. Barrett esophagus
/ D. Squamous cell carcinoma
/ E. Systemic sclerosis

Explanation - Q: 3.1 Close

The correct answer is A. The most likely diagnosis is achalasia. This
condition is a neurogenic esophageal disorder that can occur at any age, but
frequently is diagnosed when individuals are between the ages of 20 and 40.
Characteristically, the swallowing difficulties involve both solid food and
liquids. The manometry findings illustrated are typical; the barium swallow
findings may be as illustrated or may instead show diffuse esophageal
dilation without the "bird's beak" near the lower esophageal sphincter.
Larger cancers of the esophagus (choices B and D) would be more likely to
cause either a mass or an ulceration, which would be visible on barium
swallow.
Very small cancers and Barrett's esophagus (choice C) would require
esophagogastroduodenoscopy with biopsy for diagnosis, and would be
unlikely to cause dysphagia.
Systemic sclerosis (choice E) can involve the esophagus as well, however,
physical examination would usually show obvious skin involvement.

Question 2 of 5
This patient's condition is most likely due to which of the following?
/ A. Acid reflux
/ B. Cancerous destruction
/ C. Candida infection
/ D. Fibrosis of the esophageal wall
/ E. Lack of ganglion cells


Explanation - Q: 3.2 Close

The correct answer is E. Individuals who have achalasia have been found
to have a deficiency of inhibitory ganglion cells within the esophageal wall.
This lack causes an imbalance in excitatory and inhibitory neurotransmission,
with the result that the lower esophageal sphincter tends to have a higher-
than-normal muscle tone and relaxes only with difficulty.
Acid reflux (choice A) can cause esophageal irritation, ulceration, and also
predisposes for Barrett's metaplasia with subsequent risk of adenocarcinoma
of the esophagus.
Cancerous destruction (choice B) would produce a mass, or area of stricture
or ulceration.
Candida infection (choice C) of the esophagus resembles thrush of the
mouth, and causes a usually superficial infection.
Fibrosis of the esophageal wall (choice D) can be the consequence of
ulceration (due to reflux or ingestion of harsh chemicals such as lye) or
systemic sclerosis.


Question 3 of 5
Which of the following regulators would most likely inhibit the lower esophageal
sphincter in normal individuals?
/ A. Acetylcholine and substance P
/ B. Substance P and nitric oxide
/ C. Substance P only
/ D. Vasoactive intestinal polypeptide and acetylcholine
/ E. Vasoactive intestinal polypeptide and nitric oxide

Explanation - Q: 3.3 Close

The correct answer is E. Physiologically important inhibitors of the lower
esophageal sphincter include nitric oxide and vasoactive intestinal
polypeptide. Physiologically important substances that stimulate the lower
esophageal sphincter include acetylcholine and substance P.


Question 4 of 5
Which of the following medications is used to directly relax the lower esophageal
sphincter?
/ A. Diphenoxylate
/ B. Famotidine
/ C. Granisetron
/ D. Isosorbide dinitrate
/ E. Metoclopramide

Explanation - Q: 3.4 Close

The correct answer is D. Commonly used medications to relax the lower
esophageal sphincter in patients with achalasia include nitrates such as
isosorbide dinitrate (remember that nitric oxide physiologically inhibits the
lower esophageal sphincter) and calcium channel blockers such as
nifedipine (which inhibit calcium flow into the smooth muscle of the lower
esophageal sphincter, thereby inhibiting contraction.) For patients in whom
medical therapy fails, other options include paralysis of the lower esophageal
sphincter with intrasphincteric injection of botulinum toxin, pneumatic
dilatation, and a Heller myotomy (which interrupts the muscles of the lower
esophageal sphincter).
Diphenoxylate (choice A) is a opiate antidiarrheal, and would not be useful
for achalasia.
Famotidine (choice B) is an H2 antagonist that would be useful in reducing
stomach acidity, but would not directly affect lower esophageal pressure.
Other drugs in this class include cimetidine, ranitidine, and nizatidine.
Granisetron (choice C) is a 5HT3 antagonist and is used to prevent nausea
and vomiting in patients who receive chemotherapy, and after general
anesthesia. Other members of this drug class include ondansetron and
dolasetron.
Metoclopramide (choice E) stimulates gastric motility in patients with
gastroparesis and is also a antiemetic agent.


Question 5 of 5
Worldwide, which of the following parasitic diseases is most likely to produce a
disorder that clinically resembles this patient's condition?
/ A. Ascariasis
/ B. African sleeping sickness
/ C. Chagas disease
/ D. Cysticercosis
/ E. Malaria

Explanation - Q: 3.5 Close

The correct answer is C. Chagas disease, which is found in South and
Central America and is due to infection with Trypanosoma cruzi, can involve
the heart, colon, and esophagus. The esophageal involvement clinically
closely resembles achalasia.
The adult worms of ascariasis (choice A), or roundworm infection, live
principally in the intestine, and can obstruct the intestine or a bile duct; the
larvae can migrate to the liver, heart, and lungs.
African sleeping sickness (choice B), caused by Trypanosoma brucei and
Trypanosoma gambiense, causes lymphadenopathy, rash, and CNS
involvement.
Cysticercosis (choice D), due to the larval form of the pork tapeworm Taenia
solium, can involve subcutaneous tissue, muscle, viscera (but not specifically
the esophagus), and, most seriously, the CNS.
Malaria (choice E), due to various Plasmodium species, involves the blood,
liver, kidney, spleen, and brain, but does not have a specific predilection for
the esophagus.


A 35-year-old woman consults a physician because she has been having trouble
swallowing. She also often experiences chronic heartburn.
The physician performs a screening physical examination, and notices that the
skin of her hands appears tight and shiny. On specific
questioning, she reports having often experienced color changes in her hands
from white to blue to red.
Question 1 of 5
Which of the following is the most likely cause of the patient's difficulties with
swallowing?
/ A. Achalasia
/ B. Adenocarcinoma
/ C. Chagas disease
/ D. Scleroderma
/ E. Squamous carcinoma

Explanation - Q: 4.1 Close

The correct answer is D. The tip-off is the reference to the patient's skin
changes that are typical for scleroderma, also known as systemic sclerosis.
Scleroderma is a disease that may be either predominately limited to the skin
or involve many body systems, including the musculoskeletal system,
gastrointestinal tract (with esophageal involvement most often symptomatic),
cardiorespiratory system, and renal system. Esophageal dysfunction is a
common complication of scleroderma. In most patients, the skin changes are
obvious, even if the patient has not been previously diagnosed. Rarely, the
skin changes may be noticed at an earlier stage, in which the skin of the
hands appears puffy and edematous, but not scarred.
Achalasia (choice A) and Chagas disease (choice C) are also important
causes of dysphagia, but in achalasia, the physical examination is usually
normal, and in Chagas disease, you should be able to elicit a history of
possible exposure in Central or South America.
Reflux esophagitis with risk of progression to Barrett esophagus and
adenocarcinoma (choice B) can occur in scleroderma, but it is unlikely, at
this early stage in the patient's disease, that she already has
adenocarcinoma.
The risk of squamous cell carcinoma (choice E) is not increased in this
patient.


Question 2 of 5
The color changes described on the patient's hand are most likely due to which
of the following?
/ A. Arteriolar spasm
/ B. BIood clots at sites of vascular injury
/ C. Large artery spasm
/ D. PIatelet clots
/ E. Stasis blood clots

Explanation - Q: 4.2 Close

The correct answer is A. The color changes described are typical for
Raynaud's phenomenon, which occurs because of changes in perfusion due
to arteriolar spasm. Raynaud's phenomenon is common in scleroderma,
largely because the subintimal hyperplasia of small vessels characteristic of
scleroderma can reduce the luminal diameter by more than 75%. Some
authors argue that the vascular changes seen in scleroderma are actually
the insult that triggers the subsequent development of fibrosis.
Blood clots at sites of vascular injury (choice B) are typical in clots that form
in arteries on surfaces such as a fractured atherosclerotic clot.
Larger artery spasm (choice C) of vessels damaged by atherosclerosis is
thought to contribute to some myocardial infarctions, but is not thought to be
an important pathologic mechanism in scleroderma.
Platelet clots (choice D) can be seen in some diseases in which platelet
function is abnormal, such as essential thrombocytopenia.
Stasis blood clots (choice E) commonly are found in venous thromboses.




Question 3 of 5
Additional findings on physical examination include noting that the skin changes
are limited to areas distal to the elbow and knee, the
presence of calcified nodules on the extensor surfaces of the forearms, and the
presence of telangiectasias on the forearms. This suggests
that this patient has which of the following?
/ A. Bauer syndrome
/ B. Charcot syndrome
/ C. CREST syndrome
/ D. Crigler-Najjar syndrome
/ E. Dandy-Walker syndrome

Explanation - Q: 4.3 Close

The correct answer is C. These findings, together with esophageal
dysfunction and Raynaud's phenomenon (both of which this patient has), are
called the CREST syndrome, also known as limited cutaneous scleroderma.
This form of scleroderma has a better long-term prognosis than when the
skin changes also involve the trunk (diffuse scleroderma) and more internal
organs are additionally involved.
Bauer syndrome (choice A) is aortitis and aortic endocarditis as a
complication of rheumatoid arthritis.
Charcot syndrome (choice B) is intermittent claudication.
Crigler-Najjar syndrome (choice D) is a severe familial liver disease.
Dandy-Walker syndrome (choice E) is a malformation of the central nervous
syndrome.




Question 4 of 5
If this woman's involved skin were biopsied, which of the following would most
likely be seen?
/ A. CIeft separating the dermis and subcutaneous tissues
/ B. Epithelial cell hyperplasia
/ C. Marked dermal fibrosis
/ D. Narrowing of the basal lamina of small capillaries
/ E. Thickening of rete pegs


Explanation - Q: 4.4 Close

The correct answer is C. In scleroderma, early changes (at the point at
which the hands appear swollen, rather than with tight, thick skin) show
edema with perivascular infiltrates of CD4+ T cells. At this stage, the
collagen fibers are swollen and beginning to degenerate. The smaller
vessels may show basal lamina thickening (not narrowing as in choice D)
and endothelial (not epithelial as in choice B) cell damage and proliferation.
With time, the characteristic marked dermal fibrosis develops, which tends to
both narrow (not thicken as in choice E) the rete pegs and attach the dermis
tightly (compare with cleft formation as in choice A) to subcutaneous tissues.


Question 5 of 5
More than 90% of the patients with the limited cutaneous form of this disorder
make which of the following autoantibodies?
/ A. Anti-centromere
/ B. Anti-DNA topoisomerase l
/ C. Anti-double-stranded DNA
/ D. Anti-Golgi
/ E. Anti-ScI-70

Explanation - Q: 4.5 Close

The correct answer is A. All forms of scleroderma are thought to have a
strong autoimmune component, and glucocorticoids and azathioprine are
used to suppress the inflammatory complications of scleroderma. (Other
drugs that can be used in therapy include penicillamine, which inhibits
collagen cross-linking, NSAIDS for pain, and ACE inhibitors to protect the
kidney if hypertension or renal damage occurs.) The anti-centromere
antibody is quite specific for CREST syndrome (96% of cases), and is only
seen in a minority of patients with diffuse scleroderma (mainly those with
Raynaud's phenomenon) and rarely in systemic lupus erythematosus and
mixed connective tissue disease.
Anti-DNA topoisomerase I (choice B), also called anti-Scl-70 (choice E)
occurs commonly (64-75%) in diffuse scleroderma, but only rarely in CREST
syndrome.
Anti-double-stranded DNA (choice C) is fairly specific for systemic lupus
erythematosus, although it only occurs in 50-60% of lupus cases.
Anti-Golgi antibodies (choice D) are seen most often in systemic lupus
erythematosus and Sjgren syndrome.

A 40-year-old woman with welI-controlled diabetes comes to the urgent care
clinic complaining of ear pain and low-grade fever for the past
five weeks. She reports that there has also been copious greenish drainage on
her pillow each morning. Her pain has been worsening, which
brought her to the clinic. On examination, she is exquisitely tender when her
external ear is manipulated. A Iimited examination of the ear canal
shows red, inflamed skin with copious greenish exudate. There is no tenderness
over the mastoid process.
Question 1 of 3
Which of the following is the most likely diagnosis?
/ A. Acute otitis media
/ B. Chronic otitis media
/ C. Dental infection
/ D. Mastoiditis
/ E. Otitis externa

Explanation - Q: 1.1 Close

The correct answer is E. Patients with diabetes, or patients who sustain
damage to the external ear canal (usually with swabs) are susceptible to
otitis externa. This patient has many of the findings: ear pain, pruritus, and
discharge. On examination, the ear canal is erythematous and edematous.
Tenderness to manipulation of the external ear is also indicative of otitis
externa, and is not seen in otitis media.
Acute otitis media (choice A) presents with fever and ear pain. The external
ear canal is unaffected in otitis media, and there is no pain with manipulation
of the external ear.
Chronic otitis media (choice B) is inflammation of the middle ear and/or the
mastoid. It may present with otorrhea, but inflammation of the ear canal and
tenderness to manipulation of the external ear canal are not seen. Thus,
otitis externa is a better diagnosis.
Dental infections (choice C) may occasionally present as ear pain, but this is
unlikely.
Mastoiditis (choice D) is characterized by swelling, erythema and tenderness
over the mastoid. Diagnosis is confirmed by cloudiness and loss of margins
of the mastoid on radiograph. None of these findings are present here.






Question 2 of 3
This patient is started on oral antibiotics, but her infection progresses. She
continues to have pain and discharge, but now she has a facial
droop on the ipsilateral face. Which of the following cranial nerves is most likely
affected to create this symptom?
/ A. Cranial nerve l
/ B. Cranial nerve VII
/ C. Cranial nerve VIII
/ D. Cranial nerve XI
/ E. Cranial nerve XII

Explanation - Q: 1.2 Close

The correct answer is B. Cranial nerve VII, the facial nerve, innervates the
muscles of facial expression. Lesions of this nerve produce a facial droop. In
diabetics (even if the blood sugar is under tight control), there is a propensity
to develop a severe form of otitis externa known as malignant otitis externa.
In this condition, the infection in the external ear canal erodes into adjacent
soft tissues, mastoid, and temporal bone. Eventually the infection spreads
across the base of the skull. Cranial nerves are occasionally affected as are
the meninges. Treatment is surgical debridement with appropriate
intravenous antibiotics. Pseudomonas aeruginosa is often implicated in such
cases.

Question 3 of 3
Computed tomography scans of the head show extensive infection involving the
mastoid, temporal bone, and skull base. PIans for surgical
debridement are made. Until cultures are taken at the time of operation, empiric
antibiotic therapy should be directed toward which of the
following organisms?


Explanation - Q: 1.3 Close

The correct answer is C. When a diabetic patient shows symptoms of
malignant otitis externa, the causative organism is Pseudomonas 95% of the
time. Empiric therapy directed at eradicating this organism should be started
immediately. Alterations to therapy can be made when cultures and
sensitivity are obtained at time of operation.
Enterococcus(choice A), E. coli(choice B), and Strep. pneumoniae(choice
E), would be atypical for malignant otitis externa.
Staph. epidermidis(choice D) occasionally causes this condition, but
uncommonly (<5%). Thus, empiric treatment for this organism is
inappropriate.


A three-year-old boy is brought to urgent care complaining of three days of fever,
fussiness, and ear pain. The patient's mother also reveals
that the child has had a two week history of rhinorrhea and cough that resolved
several days ago. On physical examination, the tympanic
membrane is erythematous, opaque, and the ossicles are not seen. No pain is
elicited by manipulating the tragus.
Question 1 of 3
Which of the following is the most likely diagnosis?
/ A. Acute otitis media
/ B. Acute rhinoviral infection
/ C. Otitis externa
/ D. Otitis media with effusion
/ E. Mastoiditis

Explanation - Q: 2.1 Close

The correct answer is A. Acute otitis media is characterized, in children, by
fever, fussiness, and ear pain. Often a viral syndrome is found prior to onset
of ear symptoms. On examination, the tympanic membrane is erythematous,
opaque, and lacks mobility with insufflation. This child has the classic
findings.
Acute rhinoviral infection (choice B) is incorrect because it is too specific.
While many ear infections are viral, the type of virus cannot be definitively
elucidated on clinical history alone.
Otitis externa (choice C) is an infection of the external ear. It presents in a
fashion similar to otitis media, but the ear is exquisitely tender when the
external ear is manipulated. The fact that the tragus can be manipulated
without discomfort eliminates this diagnosis.
Otitis media with effusion (OME; choice D) is fluid in the middle ear without
infection and thus does not present as fever and pain. Usually this condition
occurs as a consequence of acute otitis media and presents as "plugged
ears," hearing loss, vertigo, and clumsiness. On examination, fluid is seen
behind the tympanic membrane, and decreased tympanic membrane
mobility is demonstrated on pneumatic otoscopy and tympanogram. If this
condition persists, hearing loss may develop.
Mastoiditis (choice E) is a complication of acute otitis media. It is
characterized by swelling, erythema and tenderness over the mastoid
process. Diagnosis is confirmed by cloudiness and loss of margins of the
mastoid on radiograph


Explanation - Q: 2.2 Close

The correct answer is E. Streptococcus pneumoniae is an alpha hemolytic,
gram-positive coccus that is the most commonly isolated pathogen in the
setting of acute otitis media in the pediatric setting.
Haemophilus influenzae(choice A) is a gram-negative coccobacillus
commonly found in otitis media. Its grows on chocolate agar and is not alpha
hemolytic.
Moraxella catarrhalis(choice B) can cause acute otitis media, but is not the
most common cause, nor is it alpha hemolytic.
Pseudomonas aeruginosa(choice C) is a common pathogen in otitis externa.
It is a gram-negative rod that grows on MacConkey's agar.
Staphylococcus aureus(choice D) can rarely cause otitis media and tends to
be found in chronic suppurative otitis media. It is beta hemolytic.










Question 3 of 3
Which of the following would be the most appropriate pharmacotherapy?
/ A. Ceftriaxone
/ B. Gentamicin
/ C. Methicillin
/ D. Penicillin G
/ E. Trimethoprim-sulfamethoxazole

Explanation - Q: 2.3 Close

The correct answer is D. Penicillin G is still effective in treating most
streptococcal isolates, although resistance is an emerging problem.
Ceftriaxone (choice A) is a third-generation cephalosporin and would not be
the best agent for Streptococcus. As the cephalosporins progress from first
to second to third generation, they have increasing gram-negative activity
and decreasing gram-positive activity. Thus, penicillin is a better choice.
Gentamicin (choice B) and trimethoprim-sulfamethoxazole (choice E) are
effective against gram-negative organisms and thus would not be used in
this case.
Methicillin (choice C) is used to treat organisms that are beta-lactamase
positive such as Staphylococcus. Thus, penicillin is more appropriate to treat
this infection.


A 52-year-old man presents to the general medicine clinic complaining of
progressive hearing loss in his right ear. He says that over the past
year, he has had difficulty hearing conversations in noisy places such as
restaurants. In the past few months, he has experienced occasional
ringing in his right ear. His hearing in his left ear has not been affected. He
denies headache, vertigo, otalgia, facial pain, or weakness. He has
no medical problems and does not take any medications. Otoscopic examination
reveals pearly gray tympanic membranes bilaterally without
any bulge or retraction. A Weber's test is performed, and the patient reports
hearing the vibration loudest in his left ear.
Question 1 of 6
Which of the following is the most likely diagnosis?
/ A. Acoustic neuroma
/ B. Cholesteatoma
/ C. Meniere disease
/ D. Meningioma
/ E. Otosclerosis
Explanation - Q: 3.1 Close

The correct answer is A. Acoustic neuroma is the most common cause of
unilateral sensorineural hearing loss in adults. The result of the Weber's test
indicates a sensorineural deafness in his right ear (see explanation for
question 2 for more details). Actually, the term acoustic neuroma is
something of a misnomer, since it is not "acoustic" (originating from the
auditory nerve), nor is it a neuroma (it is a schwannoma). Some have
recommended that it be called a "vestibular schwannoma." They account for
about 80% of tumors in the cerebellopontine angle; the remaining 20% are
primarily meningiomas. Acoustic neuromas most often occur as isolated
lesions in patients 40 to 60 years of age, but may also be a manifestation of
neurofibromatosis type 2 (in which case they are typically bilateral). It is a
slowly growing, benign tumor that arises from the vestibular portion of cranial
nerve VIII. Because destruction of vestibular fibers is slow and the patients
have time to equilibrate, most patients present with hearing loss, rather than
vertigo. As small tumors, they can present with hearing loss and tinnitus, and
as they grow and continue to impinge on structures in the cerebellopontine
angle, they may cause additional cranial nerve dysfunction and cerebellar
dysfunction.
Cholesteatomas (choice B) are epidermoid inclusion cysts that occur in the
middle ear and petrous apex. They most typically start as an infection in the
middle ear that accumulates dead cells and keratin. They usually present
with intermittent drainage from the ear and conductive hearing loss.
Meniere disease (choice C) is characterized by repeated episodes of vertigo
lasting from minutes to days. This disease is also accompanied by tinnitus
and progressive sensorineural hearing loss. The etiology is thought to be an
increase in the volume of labyrinthine endolymph, but the exact
pathophysiology is not known.
Meningioma (choice D) is the second most common tumor of the
cerebellopontine angle.
Otosclerosis (choice E) is caused by bony changes in the tympanic cavity
that result in immobility of the stapes. Conductive hearing loss is its most
distinctive clinical feature. Tinnitus and vertigo can also occur. Auditory
symptoms usually begin between ages 15-35 and familial occurrence is
common. This disease is bilateral in the majority of cases. Additionally, a
patient with conductive hearing loss will hear the vibration produced in the
Weber's test louder in the diseased ear.




Question 2 of 6
Which of the following findings would most likely be observed in this patient?
/ A. Air conduction equals bone conduction in his left ear
/ B. Air conduction equals bone conduction in his right ear
/ C. Air conduction is greater than bone conduction in his right ear
/ D. Bone conduction is greater than air conduction in his left ear
/ E. Both air and bone conduction are enhanced in his right ear

Explanation - Q: 3.2 Close

The correct answer is C. This patient has sensorineural deafness in his
right ear due to his acoustic neuroma. In the normal ear, air conduction is
greater than bone conduction. An ear with sensorineural deafness will still
have air conduction greater than bone conduction, however both will be
qualitatively decreased.
This can be determined with the Rinne test, in which a vibrating tuning fork is
placed on the mastoid bone (bone conduction) behind an ear until it is no
longer heard by the patient. Then, the tuning fork is held at the external
auditory canal (air conduction). In sensorineural deafness, the Rinne test is
still normal, though both components may be qualitatively decreased. In
conduction deafness, bone conduction will be better than air conduction
(abnormal Rinne test).
Another type of auditory testing is the Weber test. In the Weber test, a
vibrating tuning fork is placed on the middle of the forehead and the patient
compares the loudness of the sound in both ears. This delivers vibrations via
bone conduction, thereby bypassing the ossicles. The Weber test compares
bone conduction in the two ears. If the sound is not heard in the middle, the
sound is said to be lateralized, and a hearing loss is present. In patients with
unilateral sensorineural hearing loss, such as in acoustic neuromas, the
sound is softer on the affected side, and is louder or "localized" to the
unaffected ear. This patient has sensorineural hearing loss on the right side
and during a Weber test, the sound will lateralize to the unaffected or left ear.
Conversely, a conduction-deaf ear will hear the sound as louder than in the
normal ear (you can do this experiment yourself by putting an ear plug in one
ear). Therefore, a patient with unilateral conduction deafness hears the
vibration louder in the diseased ear, whereas a patient with unilateral
sensorineural deafness hears the vibration louder in the normal ear.
His left ear is normal, therefore, air conduction should be greater than bone
conduction, ruling out choices A and D.
Air conduction equals bone conduction in his right ear (choice B) is incorrect
because air conduction is still greater than bone conduction with
sensorineural deafness.
Both air and bone conduction are enhanced in his right ear (choice E) is
incorrect because both should be diminished in his right ear.



Question 3 of 6
The patient's disease may be associated with which of the following disease
processes?
/ A. Hypothyroidism
/ B. Neurofibromatosis type 2
/ C. Paget disease
/ D. Von HippeI Lindau syndrome
/ E. Wallenberg syndrome

Explanation - Q: 3.3 Close

The correct answer is B. Neurofibromatosis type 2 is an autosomal
dominant disorder characterized by a propensity to develop bilateral eighth
nerve schwannomas or multiple meningiomas. The other diseases listed are
not particularly associated with acoustic neuromas.


Question 4 of 6
The patient is lost to follow-up, but returns a couple of years later complaining of
worsening hearing loss and tinnitus in his right ear and
increasing dysequilibrium. On examination, stimulation of his right cornea with a
wisp of cotton does not elicit blinking in either eye, whereas
stimulation of his left cornea produces blinking in both eyes. What is the most
likely location of this new lesion?
/ A. Left abducens nerve
/ B. Left facial nerve
/ C. Left trigeminal nerve
/ D. Right abducens nerve
/ E. Right facial nerve
/ F. Right trigeminal nerve

Explanation - Q: 3.4 Close

The correct answer is E. In the corneal reflex, a wisp of cotton is touched to
the cornea. Afferent information travels up the ipsilateral trigeminal nerve (V),
and efferent information travels down the facial nerve (VII) bilaterally to elicit
the blink. The fact that this patient had no blink reflex in either eye when his
right eye was stimulated, indicates a problem in the afferent arc of the reflex
(right V).
A left abducens nerve (VI) (choice A) lesion would result in an inability to
abduct the left eye. (The abducens nerve innervates the lateral rectus
muscle.)
If there was a left facial nerve (VII) (choice B) lesion, stimulation of his right
cornea would have produced a blink reflex in the right eye; stimulation of his
left cornea would have also produced a blink reflex only in his right eye.
If there was a lesion in the left trigeminal nerve (choice C), stimulation of the
left cornea would not elicit a reflex in either eye; stimulation of the right
cornea would elicit a blink reflex in both eyes.
A right abducens nerve (VI) (choice D) lesion would result in an inability to
abduct the left eye. (The abducens nerve innervates the lateral rectus
muscle.)
If there was a right facial nerve (VII) (choice E) lesion, stimulation of his right
cornea would have produced a blink reflex in the left eye; stimulation of his
left cornea would have also produced a blink reflex only in his left eye.


Question 5 of 6
Additional testing reveals that he does not taste sugar or salt well on the anterior
two-thirds of the right side of his tongue. Which of the
following nerves is most likely lesioned?
/ A. Right facial nerve
/ B. Right glossopharyngeal nerve
/ C. Right hypoglossal nerve
/ D. Right trigeminal nerve
/ E. Right vagus nerve

Explanation - Q: 3.5 Close

The correct answer is A. The right facial nerve (VII) innervates taste buds in
the right anterior two-thirds of the tongue.
The right glossopharyngeal nerve (IX) (choice B) innervates taste buds in
the right posterior one-third of the tongue.
The right hypoglossal nerve (XII) (choice C) innervates intrinsic and extrinsic
muscles of the right tongue, thereby mediating tongue movement.
The right trigeminal nerve (V) (choice D) conveys somatosensory
information from the anterior two-thirds of the right tongue.
The right vagus nerve (X) (choice E) innervates taste buds in the right
epiglottic region.

Question 6 of 6
Which of the following is the most appropriate therapy for this patient?
/ A. Endolymphatic shunting
/ B. Hydrochlorothiazide
/ C. Positional exercises
/ D. Scopolamine
/ E. Surgical resection

Explanation - Q: 3.6 Close

The correct answer is E. Surgical resection by a variety of approaches is
the only treatment for acoustic neuromas. 75% of acoustic neuromas will
grow at a gradual pace if left unresected. As these tumors enlarge, they may
cause symptoms by compressing other cranial nerves in the region and lead
to further disability. Because of this, surgical resection is usually
recommended. If hearing is absent or poor at the time of surgical resection,
even successful removal of the acoustic neuroma will not restore any hearing
already lost. Hearing preservation is best when the tumor is less than 2 cm in
size and does not involve the internal auditory canal fundus or cochlear
aperture.
Endolymphatic shunting (choice A) is a surgical technique used to treat
Meniere disease. It reduces the volume of labyrinthine endolymph, which is
thought to be the cause of vertigo in Meniere disease. It is not a treatment for
acoustic neuromas.
Hydrochlorothiazide (choice B) is a diuretic, which may help prevent attacks
in patients with Meniere disease, however, it does not have a role in acoustic
neuroma therapy.
Positional exercises (choice C) are used to treat symptoms associated with
benign positional vertigo and have no role in the treatment of acoustic
neuromas.
Scopolamine (choice D) is an anticholinergic medication used to treat
symptoms in vertigo in patients with Meniere disease. It has no role in the
treatment of acoustic neuromas.

A 59-year-old woman with a 10-year history of type 2 diabetes mellitus is noted
by her physician to have bilateral pitting edema of the ankles
and feet. No erythema is noted. On questioning, the patient also reports
shortness of breath on exertion and states that she has been using 3
pillows at night in order to sleep comfortably.
Question 1 of 6
This patient's symptoms are most suggestive of which of the following?
/ A. Cellulitis
/ B. Congestive heart failure
/ C. Gynecologic cancer
/ D. Lipedema
/ E. Thrombophlebitis

Explanation - Q: 1.1 Close

The correct answer is B. The combination of leg edema and pulmonary edema (as
indicated by shortness of breath and sleeping on multiple pillows) strongly suggests
that the patient has congestive heart failure involving both ventricles. Heart failure is
a pathologic state in which an abnormality of cardiac function leads to failure of the
heart to pump blood throughout the body at a rate sufficient to meet the body's
requirements. Some of the adaptive mechanisms that compensate for the failing
heart include increasing preload (through the Frank-Starling mechanism),
myocardial hypertrophy (to restore elevated ventricular wall stress to within normal
limits), redistribution of cardiac output from non-vital organs to vital organs, and
neurohumoral adjustments. Congestion develops behind the failing ventricle, with
left ventricular failure causing signs and symptoms of pulmonary congestion, and
right ventricular failure causing signs and symptoms of systemic congestion.
Cellulitis (choice A) is usually markedly erythematous.
Gynecologic cancer (choice C) and thrombophlebitis (choice E) are causes of
unilateral leg edema.
Lipedema (choice D) is not a true edema (and would not show pitting), but is
instead the deposition of fatty tissues around the ankles of obese individuals.


Question 2 of 6
A chest x-ray film demonstrates pulmonary venous congestion and interstitial
edema indicative of pulmonary edema. Which of the following
physiologic mechanisms is most likely the immediate cause of the pulmonary
edema?
/ A. Damage to endothelial cells
/ B. Damage to the epithelial lining of the alveoli
/ C. EIevated pulmonary capillary pressure
/ D. Low serum albumin
/ E. Poor lymphatic drainage of fluid

Explanation - Q: 1.2 Close

The correct answer is C. The cause of pulmonary edema in congestive
heart failure is an increase in the hydrostatic pressure at the level of the
capillaries of the lung. This increased pressure serves to drive fluid out of the
capillaries and into the alveoli. The other reasons cited in the choices can
also cause pulmonary edema, but occur in other clinical settings.
Damage to endothelial cells (choice A) can occur in vasculitis.
Damage to the epithelial lining of the alveoli (choice B) can occur in
pneumonia and respiratory distress syndrome.
Low serum albumin (choice D) can be seen in liver and kidney disease.
Poor lymphatic drainage (choice E) can be seen in lungs with chronic
damage due to severe emphysema or fibrosis.




Question 3 of 6
A chest x-ray film additionally demonstrates a markedly enlarged cardiac shadow
with dilation of both the right and left ventricles.
Echocardiography shows dilated, hypokinetic ventricles. The physician
prescribes several medications, including captopriI. Which of the
following best characterizes captopriI?
/ A. ACE inhibitor
/ B. AIpha blocker
/ C. Beta blocker
/ D. Diuretic
/ E. Positive inotrope

Explanation - Q: 1.3 Close

The correct answer is A. ACE inhibitors, including captopril, enalapril, and
lisinopril, are becoming mainstays in the treatment of congestive heart
failure, more formally known as dilated congestive cardiomyopathy. These
agents inhibit the conversion of angiotensin I to angiotensin II, thereby
interrupting the renin-angiotensin-aldosterone loop. The functional result is
vasodilation, with a reduction in both cardiac afterload and cardiac preload.
Furthermore, the ACE inhibitors, such as captopril and enalapril, are proven
to delay the progression of this condition by protecting the ventricles from
deleterious remodeling.
Alpha-blockers (choice B) are not commonly used in CHF, although
carvedilol has some alpha-blocking activity.
Carvedilol is a non-cardioselective alpha- and beta-blocker (choice C)
approved by the FDA for the treatment of both ischemic and non-ischemic
heart failure; its alpha blocking effects confer vasodilatory activity.
Catecholamine production is enhanced in the early development of CHF to
compensate for decreasing cardiac output. Beta-blockers are beneficial in
heart failure, since they diminish the chronic sympathetic nervous system
stimulation; they may also be beneficial through the following mechanisms:
improving cardiac output, increasing diastolic filling time, and decreasing
arrhythmias.
Loop diuretics (choice D), such as bumetanide and furosemide, are also
considered to be first-line agents in the treatment of heart failure. These
agents are indicated for the treatment of edema associated with CHF,
hepatic cirrhosis, and renal disease, as well as treatment of hypertension
(furosemide and torsemide).
Digoxin (choice E) causes a positive inotropic effect by inhibiting the Na
+
/K
+

ATPase in cardiac cell membranes. This inhibition leads to increased
intracellular sodium, which alters the driving force for the Na
+
/Ca
2+
exchange
mechanism, leading to a decrease of calcium removal from the cell. This
results in increased calcium storage in the sarcoplasmic reticulum, which
when released, increases contractile force. Digitalis also modifies autonomic
outflow.

Question 4 of 6
An increase in which of the following is the most likely explanation for the edema
in her legs?
/ A. Interstitial colloid osmotic pressure
/ B. Lymph flow
/ C. PIasma colloid osmotic pressure
/ D. Right atrial pressure
/ E. Stroke volume
Explanation - Q: 1.4 Close

The correct answer is D. Right atrial pressure rises in congestive heart
failure, which elevates venous pressure throughout the body. This increase
in venous pressure can cause excessive fluid loss from the microcirculation
and the development of peripheral edema.
Fluid loss from the capillaries washes protein molecules from the interstitial
compartment and thereby decreases interstitial colloid osmotic pressure
(choice A).
Increased lymph flow (choice B) is a consequence rather than a cause of
the edema.
Increased plasma colloid osmotic pressure (choice C) would tend to
decrease the development of edema.
Stroke volume (choice E) has no effect on the formation of peripheral
edema.


Question 5 of 6

As this patient's condition progresses, she is started on digoxin. Her most recent
digoxin blood level was 2.0 ng/mL. She will be at the highest
risk for developing digoxin toxicity if she has which of the following conditions?
/ A. Hypokalemia
/ B. Hyponatremia
/ C. Hypophosphatemia
/ D. Vitamin B12 deficiency
/ E. Vitamin K deficiency

Explanation - Q: 1.5 Close

The correct answer is A. Digoxin is a cardiac glycoside indicated for the
treatment of congestive heart failure, atrial fibrillation, and atrial flutter. The
therapeutic drug serum levels for digoxin are 0.5 to 2.2 ng/mL. Toxicity
typically occurs when digoxin levels are > 2.5 ng/mL, unless the patient is
hypokalemic, then toxicity can occur at any therapeutic concentration.
Hypokalemia sensitizes the myocardium to digoxin and may reduce the
positive inotropic effects of the medication. Other signs and symptoms of
digoxin toxicity include nausea, vomiting, anorexia, and appearance of
yellow-green halos in the visual field, as well as the development of cardiac
arrhythmias.
Although hyponatremia (choice B) and hypophosphatemia (choice C) can
result in the development of other pathological disturbances, they do not
potentiate digoxin toxicity.
Vitamin B
12
deficiency (choice D) is associated with the development of
pernicious anemia.
Vitamin K deficiency (choice E) is associated with the development of
clotting disorders. Furthermore, vitamin K deficiency is known to potentiate
warfarin toxicity.

Question 6 of 6
This patient's condition is associated with which of the following 5-year mortality
rates?
/ A. 5%
/ B. 30%
/ C. 50%
/ D. 70%
/ E. 95%

Explanation - Q: 1.6 Close

The correct answer is D. In temperate zones, dilated congestive
cardiomyopathy is most commonly the result of diffuse coronary artery
disease with diffuse ischemic myopathy, which produces chronic myocardial
fibrosis with diffuse loss of myocytes. Other causes are very diverse and can
include a wide variety of infections, granulomatous disease, metabolic
disorders, drugs and toxins, cancers, connective tissue disorders, familial
neuromuscular disorders, and pregnancy. The dilated chambers usually
function poorly, leading to a low ejection fraction. Tachycardia and increased
end-diastolic volume initially maintain cardiac output, but often eventually fail
with disease progression to adequately supply the tissues with oxygenated
blood. Overall, dilated congestive cardiomyopathy is more often fatal than
many cancers, with a 5-year mortality rate of 70%.

A 55-year-old woman consults a physician because of edema involving her lower
legs. While both of her ankles are swollen, the left one is
much more swollen than the right. Physical examination additionally
demonstrates a swollen, tender, bluish, cord-Iike structure immediately
below the skin on her left lower leg extending from her ankle to near her knee.
Edema at sites other than the ankles is not apparent on physical
examination. The patient has not been having any other symptoms. The woman
works as a nurse and had a long airplane trip during the
previous week.
Question 1 of 5
The tender cord-Iike structure is most likely which of the following?
/ A. Artery
/ B. Ligament
/ C. Lymphatic channel
/ D. Tendon
/ E. Vein

Explanation - Q: 2.1 Close

The correct answer is E. The structure is most likely an inflamed vein.
Large superficial veins are common on the legs.
The superficial arteries (choice A) of the legs are usually much less
apparent and located deeper than are the veins.
Ligaments (choice B) attach bone to bone, and can be palpated, but not
usually seen, in the feet and at the knee.
Lymphatic channels (choice C) are usually invisible on surface examination,
unless they are markedly inflamed and accompanied by skin erythema. Then
they appear as red lines.
Tendons (choice D) also produce cord-like structures, but they are pale in
color and do not appear blue through the skin.
A 55-year-old woman consults a physician because of edema involving her lower
legs. While both of her ankles are swollen, the left one is
much more swollen than the right. Physical examination additionally
demonstrates a swollen, tender, bluish, cord-Iike structure immediately
below the skin on her left lower leg extending from her ankle to near her knee.
Edema at sites other than the ankles is not apparent on physical
examination. The patient has not been having any other symptoms. The woman
works as a nurse and had a long airplane trip during the
previous week.

Question 2 of 5

Which of the following is the most likely cause of the increased edema in the
patient's left lower leg?
/ A. Congestive heart failure
/ B. Lymphatic obstruction
/ C. Pulmonary failure
/ D. Renal failure
/ E. Venous obstruction


Explanation - Q: 2.2 Close

The correct answer is E. While on paper, leg edema can have a very large
number of potential causes, an acute exacerbation of leg edema limited to
one leg and accompanied by an obviously damaged vein should make you
think of superficial thrombophlebitis. This condition occurs when thrombosis
of a vein induces an accompanying inflammation of the vein (which
essentially always happens). Superficial thrombophlebitis that occurs in
previously healthy legs does not always induce ankle edema because the
venous return via other routes is usually adequate. However, exacerbation of
ankle edema can, as in this case, be the presenting complaint if a patient's
legs are already damaged. In this case, the working history of being a nurse
implies that the patient may have a history of long periods of walking and
standing, which may lead to dependent edema in the ankles. Superimposed
on this is a recent history of a long airline flight, which can act as a trigger for
thrombophlebitis secondary to the prolonged immobilization experienced on
the flight.
Congestive heart failure (choice A) and renal failure (choice D) would
produce bilateral ankle edema, and possibly edema visible in other sites.
Lymphatic obstruction (choice B) can also produce localized edema, but this
woman has an obviously inflamed vein, and that is the more likely cause.
Pulmonary failure (choice C), unless due to pulmonary edema secondary to
congestive heart failure, would be unlikely to be associated with ankle
edema.

Question 3 of 5
Which of the following is the most serious complication of this condition?
/ A. Focal arterial obstruction
/ B. Involvement of deep leg veins
/ C. Obstruction of a joint space
/ D. Rupture of an involved tendon
/ E. Secondary lymphangiosarcoma

Explanation - Q: 2.3 Close

The correct answer is B. Superficial thrombophlebitis often resolves
spontaneously without sequelae, but should always be taken very seriously
because the same patients who develop superficial thrombophlebitis may
either have an extension of the clot into the deep veins (with increased risk of
potentially fatal pulmonary embolism) or may develop a separate deep
venous clot concurrently or at a later date. In hospitalized patients with
superficial thrombophlebitis, it is thought that as many as 10% eventually
develop pulmonary embolus (typically from an undiagnosed or later deep
vein thrombosis), and of this subset of patients with pulmonary embolus,
20% die.
The artery (choice A), joint space (choice C), and tendons (choice D) are
not directly involved by this disease process.
Lymphangiosarcoma (choice E) is a rare complication of chronic lymphatic
obstruction.



Question 4 of 5
Which of the following techniques is most useful for initial evaluation of the
presence of this complication?
/ A. Computed tomography
/ B. D-dimer levels in serum
/ C. Duplex ultrasonography
/ D. Magnetic resonance imaging
/ E. Prothrombin time

Explanation - Q: 2.4 Close

The correct answer is C. The accurate diagnosis of deep vein thromboses
is a problematic area of medicine, because no completely reliable diagnostic
technique is easily available. Various techniques of physical diagnosis have
been proposed, but can be misleading in most people's hands, and leave an
erroneous impression that the patient does not have deep vein involvement
when it is actually present. Both Duplex ultrasound (a combination of real-
time and Doppler ultrasound techniques) and plethysmography (using
volume and pressure changes to look at venous filling) can be used to
diagnose deep vein thromboses, and are often used in conjunction as they
tend to miss and pick up slightly different cases.
Computed tomography (choice A) and magnetic resonance venography
(choice D) can also be used, but are very expensive and not warranted in
most cases.
Serum indicators of clotting problems, such as D-dimers (choice B) and
prothrombin time (choice E) are too nonspecific to be helpful in diagnosing
deep vein thrombosis.






Question 5 of 5
Following the appropriate studies, which demonstrated the most serious
complication of this condition, the decision is made to anticoagulate
the patient. Which of the following is the most appropriate initial choice of
medications?
/ A. Intravenous Coumadin
/ B. Intravenous heparin
/ C. Oral aspirin
/ D. OraI Coumadin
/ E. Oral heparin

Explanation - Q: 2.5 Close

The correct answer is B. A patient with a known deep venous thrombosis is
usually initially anticoagulated with intravenous heparin, then switched over
the next days to weeks to oral Coumadin (choice D). These patients may
also be treated with fibrinolytic agents, particularly if the deep vein clot is
extending or has recently developed.
Heparin is not available orally (choice E), and Coumadin is not usually given
intravenously (choice A).
Aspirin (choice C) is not effective in preventing venous clots (which are
triggered predominantly by stasis).

A fifty-year-old alcoholic man presents to the emergency department complaining
of shortness of breath and ankle swelling. He has a history of
intravenous drug use and had a previous episode of upper GI bleeding. On
examination, he is a malnourished man in mild distress. His sclera
are icteric, and his abdomen is distended. He has moderate pretibial edema, and
his mentation is slightly impaired. Laboratory studies show
a conjugated hyperbilirubinemia.
Question 1 of 5
Which of the following is the most likely diagnosis?
/ A. Choledocholithiasis
/ B. Congestive heart failure
/ C. Hemolytic anemia
/ D. Hepatic cirrhosis
/ E. Renal failure

Explanation - Q: 3.1 Close

The correct answer is D. This patient's constellation of symptoms and signs
can be explained by cirrhosis of the liver with portal hypertension. His
shortness of breath and distended abdomen are a product of ascites from
portal hypertension and liver cirrhosis. As ascites fluid accumulates, intra-
abdominal pressure increases. This can lead to decreased diaphragmatic
excursion, atelectasis, and shortness of breath. His history of GI bleeding
suggests varices, which are a result of portal hypertension. The jaundice
results from decreased flow of bile out of the cirrhotic liver. His altered
mentation is from the liver's decreased ability to detoxify nitrogenous
metabolites.
Choledocholithiasis (choice A), or stones in the common bile duct, can
cause a conjugated hyperbilirubinemia, but it is typically accompanied by
severe right upper quadrant pain and it does not cause ascites.
Congestive heart failure (choice B) would cause edema and shortness of
breath, but it would not cause jaundice. Severe heart failure (especially right
heart failure) can cause hepatic congestion, and mild ascites, but given the
patient's history and symptoms, the CHF diagnosis can be excluded.
Hemolytic anemia (choice C) causes an unconjugated hyperbilirubinemia.
Hemolysis liberates free heme, which is a precursor to bilirubin. The rate of
production of bilirubin from this heme exceeds the liver's conjugating
capacity and unconjugated hyperbilirubinemia with jaundice results. It does
not produce ascites.
The patient's edema could be the result of renal failure (choice E), but renal
failure would fail to explain the rest of his symptoms. Renal failure does not
cause jaundice or GI bleeding.
Question 2 of 5
A serum analysis is performed and it is found that the patient is infected with
Hepatitis C. Which of the following best describes the Hepatitis C
virus?
/ A. FIavivirus
/ B. Picornavirus
/ C. Poxvirus
/ D. Reovirus
/ E. Rhinovirus

Explanation - Q: 3.2 Close

The correct answer is A. The hepatitis C virus is a Flavivirus-like organism.
It is less likely than hepatitis B to cause a fulminant infection, but it is more
likely to cause a chronic infection. 60-90% of cases become chronic, and
ultimately 40% progress to cirrhosis. The synergy of hepatitis C and alcohol
is particularly damaging, and this certainly contributed to this patient's
pathology.
The picornavirus is a small RNA virus ( "pico" + RNA) (choice B). The most
well known virus in this group causes polio.
Poxviruses (choice C) do not cause hepatitis. They are known to cause
molluscum contagiosum and smallpox.
The reovirus (choice D) family consists of two distinct classes of virus; the
rotavirus and the enteric calicivirus. Rotavirus can cause diarrheal illness,
particularly in children. The enteric caliciviruses cause gastroenteritis in
adults and children. Neither causes hepatitis C.
Rhinoviruses (choice E) are a cause of " the common cold."


Question 3 of 5

Which of the following best describes the pathophysiology underlying this
patient's distended abdomen?
/ A. Decreased intra-abdominal pressure, decreased plasma oncotic pressure
/ B. Decreased portal hydrostatic pressure, decreased plasma oncotic pressure
/ C. Decreased portal hydrostatic pressure, increased plasma oncotic pressure
/ D. Increased portal hydrostatic pressure, decreased plasma oncotic pressure
/ E. Increased portal hydrostatic pressure, increased plasma oncotic pressure

Explanation - Q: 3.3 Close

The correct answer is D. Many things contribute to the accumulation of
ascites fluid, but the exact mechanism is not completely understood. Known
contributors include: increased portal hydrostatic pressure (from portal
hypertension secondary to obstructed portal flow), decreased plasma oncotic
pressure (from hypoalbuminemia secondary to failing hepatic synthesis), and
a renal retention of sodium (secondary to a perceived hypovolemia from
splanchnic sequestration of fluid).
None of the other answer combinations occur in ascites.


Question 4 of 5
During the patient's hospitalization, his mental status deteriorates. He becomes
unconscious and a flapping tremor is elicited at the wrist.
Scant blood is passed per rectum. His vital signs remain stable, and his
hemoglobin falls 1g/dL. EIectrolytes are normaI. Which of the following
explains his altered mental state?
/ A. Encephalopathy secondary to gastrointestinal bleeding
/ B. Hypovolemia secondary to rapid accumulation of ascites fluid
/ C. Hypoxia secondary to gastrointestinal bleeding
/ D. Poor cerebral perfusion secondary to gastrointestinal bleeding.
/ E. Severe azotemia secondary to renal failure

Explanation - Q: 3.4 Close

The correct answer is A. This patient suffers from hepatic encephalopathy
precipitated by gastrointestinal bleeding. In this patient, the gastrointestinal
bleeding increased the amount of nitrogenous substrate available to colonic
flora. These flora metabolize nitrogenous compounds into amines, which
may act as neuromodulators. In patients with normal portal circulation, the
liver would metabolize these compounds into nontoxic substances. In this
patient, the portal hypertension causes shunting of blood into the systemic
circulation without passing through the liver, producing hepatic
encephalopathy. The asterixis, or flapping tremor, confirms this diagnosis.
The fluid shifts in ascites are slow (compare with choice B), and would not
cause altered mental status. The kidneys "sense" this fluid shift as
hypovolemia and retain fluid accordingly.
Hypoxia (choice C) is a common cause of altered mental status. It is unlikely
that a drop in hemoglobin of only 1g/dL would cause hypoxia in the absence
of any other catastrophic event.
This patient has suffered a minor GI bleed. Given that the vitals are stable
and the hemoglobin has fallen only slightly, it is unlikely that there is
hemodynamic instability that would cause an altered level of consciousness
(choice D).
There is no evidence of renal failure in this scenario. The electrolytes are
normal, and thus azotemia from renal failure (choice E) is an unlikely cause
for this patient's altered mental status


Question 5 of 5
Lactulose syrup is administered to this patient after he developed altered mental
status. Which of the following best describes how lactulose
will help restore mental function?
/ A. Lactulose acts as an osmotic diuretic to decrease intracranial pressure
/ B. Lactulose acts as an osmotic laxative to aid in evacuation of gastrointestinal
blood
/ C. Lactulose is bactericidal to intestinal flora
/ D. Lactulose restores the integrity of the blood-brain barrier
/ E. Lactulose stabilizes axonal transmission of neural impulses

Explanation - Q: 3.5 Close

The correct answer is B. Lactulose is administered to patients with hepatic
encephalopathy for several reasons. Following a GI bleed, removal of protein
(and thus nitrogenous toxins) from the colonic lumen is paramount. Lactulose
is a non-absorbable disaccharide that acts as an osmotic laxative to remove
nitrogenous toxins (and their proteinaceous precursors) from the colon. In
addition, bacterial metabolism of lactulose decreases the gut pH, which
protonates the amines to the less absorbable ammonium form.
Lactulose is not absorbed from the gut so it could not act as an osmotic
diuretic (choice A). Mannitol is an osmotic diuretic used to decrease
intracranial pressure.
Lactulose is not bactericidal to intestinal flora (choice C).
Lactulose is not absorbed from the gut and thus cannot act on the blood-
brain barrier (choice D).
Lactulose is not absorbed from the gut and thus cannot affect axonal
transmission (choice E).

A 46-year-old Caucasian man presents with a history of fatigue and poor
concentration for the last few months. Past medical history is
significant for kidney stones and a treated peptic ulcer years ago. He has
smoked 1 pack of cigarettes each day for the last 15 years. Review
of systems was positive for bilateral hand pain for the past several months, which
was not alleviated by ibuprofen. The patient denies taking
any other medications. Physical examination is unremarkable. Thyroid function
tests and blood glucose are normaI. The serum calcium is 11
mg/dL and serum phosphorus is 2.6 mg/dL. Parathyroid hormone (PTH) is 800
pg/mL. Urine calcium is 425 mg/24 hr.

Question 1 of 5

Which of the following is the most likely diagnosis?
/ A. Primary hyperparathyroidism
/ B. Sarcoidosis
/ C. Secondary hyperparathyroidism due to ectopic secretion of a PTH-Iike
substance
/ D. Secondary hyperparathyroidism due to renal failure
/ E. Vitamin D excess

Explanation - Q: 1.1 Close

The correct answer is A. Primary hyperparathyroidism is most commonly
caused by parathyroid adenomas (85% of the cases), followed by
hyperplasia (12-15%), and rarely, carcinoma (<3%). It is characterized by
excess PTH production, which results in increased serum calcium and
decreased phosphate levels. It is a relatively common disease of the middle-
aged and elderly. About 85% of the affected patients are asymptomatic and
evidence for the disorder is often found during routine laboratory screening.
One of the initial presentations of sarcoidosis (choice B) is hypercalcemia.
Mononuclear cells in granulomas produce increased 1,25-cholecalciferol
(1,25(OH)
2
D3), resulting in increased calcium absorption from the gut.
Phosphate and PTH levels are not affected.
Secondary hyperparathyroidism may manifest in different ways depending
the underlying etiologies. Ectopic secretion of PTH-like substances (choice
C) is associated with a low PTH, because the endogenous PTH is
suppressed by the hypercalcemia from the former. Renal failure (choice D)
exhibits hypocalcemia initially, which leads to the stimulation of PTH
secretion. Phosphate tends to be elevated because of decreased excretion
from the renal insufficiency.
Vitamin D excess from ingestion (choice E) results in hypercalcemia
because more 1,25(OH)
2
D3 is produced; therefore, PTH becomes
suppressed.

Question 2 of 5

Bilateral hand x-ray films are obtained. They show subperiosteal bone resorption
and some cyst formation. What do these x-ray findings
suggest?
/ A. Hungry bone syndrome
/ B. Osteogenesis imperfecta
/ C. Osteitis fibrosa cystica
/ D. Osteomalacia
/ E. Paget disease of bone

Explanation - Q: 1.2 Close

The correct answer C. Osteitis fibrosa cystica is a characteristic, but rarely
seen, manifestation of primary hyperparathyroidism. The cystic changes in
the bone are due to osteoclastic resorption, and fibrous replacement of
resorbed bone may lead to the formation of nonneoplastic tumor-like masses
(brown tumor) on x-ray films.
Hungry bone syndrome (choice A) is a another rare phenomenon that can
occur after parathyroid surgery. In this disorder, there is calcium uptake into
the bones, so that the serum calcium falls.
Osteogenesis imperfecta (choice B) is a heritable disease associated with
brittle bones, blue sclerae, and dental abnormalities. Patients often have a
history of multiple fractures.
Osteomalacia (choice D) can be secondary to vitamin D deficiency; it is
caused by defective mineralization of the bone matrix. On x-ray films, there is
a "ground glass" appearance of the bony trabeculae and the cortices are
thinned.
Paget disease of the bone (choice E) is a fairly common disease of the
elderly; it is often detected by elevated alkaline phosphatase levels. Lytic
lesions are seen on x-ray films.



Question 3 of 5

Which of the following would be the most appropriate management for this
patient?
/ A. Bisphosphonates
/ B. Calcitonin
/ C. FIuid hydration with diuretics
/ D. Observation with yearly measurement of serum calcium and creatinine
/ E. Parathyroidectomy

Explanation - Q: 1.3 Close

The correct answer is E. This patient would be a candidate for
parathyroidectomy. Criteria for surgery include age less than 50 to minimize
the complications of untreated hypercalcemia, and having a serum calcium >
11 mg/dL. In addition, urolithiasis, impaired renal function, marked
hypercalciuria, and osteoporosis are further indications for surgery. Most
patients with primary hyperparathyroidism tend not to be symptomatic from
their hypercalcemia.
Bisphosphonates (choice A) are not used to manage the chronic
hypercalcemia stemming from primary hyperparathyroidism but are more
useful treating hypercalcemia associated with malignancy.
Calcitonin (choice B), as with bisphosphonates, is useful in the treatment of
neoplastic hypercalcemia.
For more urgent and symptomatic hypercalcemia (Ca
2+
> 12-13 mg/dL, ECG
changes with short QT intervals, etc.), fluids and diuretics (choice C) are
first-line therapy, especially if the patient seems dehydrated as well as
hypercalcemic.
Observation (choice D) alone is not appropriate. Chronic hypercalcemia can
lead to further calcium deposition in tissues; eventually, untreated
hypercalcemia can lead to coma and cardiac arrest.


Question 4 of 5
If this patient had decreased serum PTH Ievels, which of the following would be
the most likely diagnosis?
/ A. Graves Disease
/ B. Lithium use
/ C. Malignancy
/ D. Sarcoidosis
/ E. Thiazide diuretics

Explanation - Q: 1.4 Close

The correct answer is C. Malignancy (especially bronchogenic squamous
cell carcinoma, multiple myeloma, and renal cell carcinoma) may produce a
PTH-like hormone that functions like PTH and results in hypercalcemia.
However, the hypercalcemia (via negative feedback) suppresses PTH levels
to less than 20 pg/mL. Given the patient's smoking history, one may consider
obtaining a chest x-ray film, CBC, and urinalysis to screen for the above
malignancies.
Moderate hypercalcemia of unknown causes can be seen in patients with
Graves disease (choice A). It is characterized by the presence of a diffusely
enlarged goiter, exophthalmos (eyes protruding out), and pretibial myxedema
as well as decreased TSH.
Lithium (choice B) can increase the PTH threshold such that a higher level
of serum calcium is required to shut off PTH production. The patient does not
have a history of manic depressive disorder and denies taking any
medications other than ibuprofen.
Sarcoidosis (choice D), along with other granulomatous diseases, can
present with hypercalcemia, but the phosphate levels are not affected.
Furthermore, the patient can have alveolar infiltrates and hilar adenopathy on
chest x-ray films as well as respiratory complaints. An elevated ACE level
can confirm the diagnosis.
Thiazides (choice E) can decrease calcium excretion, but this effect tends to
be temporary; furthermore, patients on thiazides can also have low sodium
and potassium levels.



Question 5 of 5

If this patient also stated that his mother had thyroid cancer surgery and his
brother had uncontrolled hypertension, which of the following
diagnoses would be most likely?
/ A. Familial benign hypercalcemic hypocalciuria
/ B. MEN Type l
/ C. MEN Type lI/IIa
/ D. MEN Type llI/IIb
/ E. Pseudohypoparathyroidism

Explanation - Q: 1.5 Close

The correct answer is C. A patient with parathyroid hyperplasia along with a
family history of medullary thyroid cancer and pheochromocytoma
(uncontrolled hypertension) may have MEN II/IIa syndrome/Sipple syndrome.
Like all the MEN syndromes, it is autosomal dominant with variable
expression.
Familial benign hypercalcemic hypocalciuria (choice A) is also autosomal
dominant; patients may have normal PTH levels with elevated serum
calcium. It is benign, and differentiated from hyperparathyroidism by having a
low urinary calcium.
MEN type 1/Wermer syndrome (choice B) has a variety of symptoms
caused by hyperplasia/adenomas/cancers of parathyroid, pituitary, and islet
cells of the pancreas.
MEN type III/IIb (choice D) is similar to MEN type II/IIa because it is also
associated with medullary thyroid carcinoma and pheochromocytoma;
however, parathyroid hyperplasia is rare. It is distinguished by the presence
of mucosal neuromas.
Pseudohypoparathyroidism (choice E) is a hereditary disorder associated
with hypoparathyroidism because of tissue resistance to PTH. Therefore, the
PTH levels are high with low serum calcium and phosphate levels. Patients
can have short stature and moon facies as well as having characteristic short
4th fingers.



A 52-year-old woman presents to her physician's office complaining of an
enlarging nose, thickening of her tongue, and coarsening of her
facial features. She had started noticing the gradual change 2-3 years earlier,
accompanied by soreness of the hands. No change in shoe size
or enlargement of the limbs was reported. The patient denies having headaches.
When she was 42 years old, she was diagnosed with chronic
bronchitis. At the age of 51, she underwent thyroid surgery for multinodular
goiter. On examination, her blood pressure is 140/90 mm Hg and
her pulse is 68/min. A chest x-ray film shows a welI-demarcated opacity, 5 cm in
diameter, Iocated in the posterobasal part of the right
pulmonary lobe.

Question 1 of 5

Which of the following is the most likely diagnosis?
/ A. Gigantism
/ B. McCune-AIbright syndrome
/ C. NAME syndrome
/ D. Paraneoplastic syndrome
/ E. Pituitary adenoma

Explanation - Q: 2.1 Close

The correct answer is D. This patient has acromegaly secondary to a
paraneoplastic syndrome. Paraneoplastic syndromes refer to a large group
of medical problems in patients suffering from cancer. They are defined as
clinical syndromes that result from systemic effects of substances produced
by the tumor. The symptoms are mostly endocrine, but may be
neuromuscular, cutaneous, hematologic, renal, gastrointestinal or
miscellaneous, depending on the chemical nature of the substance
produced. Tumors can produce antibodies, hormones, hormone-like
substances or hormone precursors, fetal proteins, or cytokines. Endocrine
symptoms usually resemble the more common endocrine disorders (e.g.,
Cushing syndrome, acromegaly). Lung tumors can cause several types of
endocrine paraneoplastic syndrome. Cushing syndrome and SIADH are
related to the ectopic production of hormone-like substances by small cell
cancer of the lung. Hypercalcemia, caused by the secretion of parathyroid
hormone related peptide (PTHrP), and acromegaly, caused by an ectopic
secretion of growth hormone, are endocrine paraneoplastic syndromes
associated with squamous cell carcinoma of the lung. Acromegaly caused by
an ectopic secretion of growth hormone (GH) is difficult to differentiate from
that of pituitary origin. Provocation tests (oral glucose, TRH test, GHRH test)
may be normal or may yield paradoxical results. The usual episodic pattern
of secretion is missing in cases of ectopic GH secretion. Long-acting
somatostatin analogues and dopamine agonists are used in the treatment of
this condition. Surgical or other treatment of the lung tumor represents
definitive therapy for the patient.
Hypersecretion of GH in childhood will result in gigantism (excessive linear
growth; choice A); onset in late adolescence will produce tall stature and
acromegaly.
McCune-Albright syndrome (choice B) is manifested clinically with fibrous
dysplasia of bones, hyperpigmented skin changes, goiter, acromegaly,
hyperparathyroidism, and hypophosphatemic hyperphosphaturic rickets.
NAME syndrome (nevi, atrial myxoma, myxoid neurofibromas, and
ephelides) (choice C) is associated with acromegaly due to pituitary GH-
secreting tumors.
GH hypersecretion that occurs after epiphyseal fusion is termed acromegaly.
More than 90% of acromegaly cases are caused by pituitary adenomas
(choice E) secreting excess GH.

Question 2 of 5
The pituitary cells that normally produce the hormone involved in this patient's
disease process belong to which of the following types?
/ A. Corticotrophs
/ B. Gonadotrophs
/ C. Mammotrophs
/ D. Somatotrophs
/ E. Thyrotrophs

Explanation - Q: 2.2 Close

The correct answer is D. Cells of the anterior lobe of the pituitary have
been broadly classified, based on their staining features as chromophils,
which stain with acidic and basic dyes, and chromophobes, that have little
affinity for these stains. Chromophils are further subdivided as basophils,
which stain blue (10% of population) and acidophils, which stain red (40%).
Chromophobes (50%) are the predominant type and are thought to be either
inactive chromophil progenitors or resting, exhausted cells. Modern
immunocytochemical techniques using specific antisera against a particular
hormone are necessary to allow identification of these cell types.
Somatotrophs (acidophils) are small round cells with dense 350 nm granules,
and are believed to synthesize growth hormone.
Round or oval cells with granules and lipid droplets are called corticotrophs
(choice A) and they synthesize ACTH.
FSH and LH are produced within large and small round cells that are called
gonadotrophs (choice B).
Mammotrophs (choice C) are cells with a variable size and the presence of
dense pleomorphic 600 nm granules. These cells synthesize prolactin.
Thyrotrophs (choice E) are TSH-producing cells, which are large and
polygonal with 150 nm granules.


Question 3 of 5

Which of the following is the major inhibitor of the release of the hormone in
question?
/ A. Gastrin
/ B. GHRH (growth hormone releasing hormone)
/ C. GIP (gastric inhibitory peptide)
/ D. Secretin
/ E. Somatostatin

Explanation - Q: 2.3 Close

The correct answer is E. The cyclic tetradecapeptide hormone,
somatostatin, was first described as the major physiological inhibitor of GH
secretion, but it has been since shown that it can also inhibit the secretion of
insulin, glucagon, gastrin, and secretin. Somatostatin is found in various
parts of the brain, where it functions as a neurotransmitter/neuromodulator
affecting sensory input, locomotor activity, and cognitive function. It is also
found in the retina, where it probably acts as an inhibitory neurotransmitter.
In the hypothalamus, it is secreted into the portal hypophyseal vessels,
travels to the pituitary somatotrophs, and inhibits GH secretion.
Gastrin (choice A) is secreted by G-cells in the antrum of the stomach, and
then travels to the parietal cells to stimulate acid secretion.
GHRH (growth hormone releasing hormone) (choice B) is a 44 amino acid
peptide and is the major stimulator of GH release. Its secretion from the
hypothalamus is episodic, and these fluctuations coincide with most of the
surges in growth hormone secretion.
Fatty acids in the duodenum cause the release of GIP (choice C), which acts
directly on parietal cells to reduce acid secretion.
Duodenal cells in the presence of acid, fat, and protein, release secretin
(choice D). The major target tissue is the pancreas, but it also inhibits gastric
acid secretion.


Question 4 of 5
Some of the involved hormone's actions are mediated by somatomedins (IGFs).
Which of the following effects would most likely be a result of
the action of IGFs (insulin-Iike growth factors)?
/ A. Decreased insulin sensitivity
/ B. Epiphyseal growth
/ C. Increased GI absorption of Ca2+
/ D. Lipolysis
/ E. Na+retention

Explanation - Q: 2.4 Close

The correct answer is B. Insulin-like growth factors (IGFs), or
somatomedins, are a family of peptide hormones with mitogenic properties
and insulin-like features that mediate the effects of GH on skeletal tissue,
e.g. by stimulating epiphyseal growth. They are synthesized primarily in the
liver, but are also made in cartilage, pituitary, and brain. The principal
circulating somatomedins in humans are insulin-like growth factor I (IGF-I,
somatomedin C) and IGF-II (somatomedin A). Secretion of IGF-I is
independent of GH in utero, but it is stimulated by GH after birth. Its
concentration in plasma peaks at the time of puberty and then declines to
low levels at old age. IGF-II does not play a physiological role after birth, and
its concentration is constant during postnatal development. Some of GH's
actions are direct, and some are indirect, i.e., mediated by IGF-I. IGF-I
stimulates proliferation of chondrocytes, sulfate incorporation, and collagen
synthesis. It stimulates both the differentiation and proliferation of myoblasts,
and stimulates amino acid uptake and protein synthesis, thereby playing an
important role in muscle growth. GH plays a permissive role by converting
the cartilage stem cells into cells that respond to IGF-I. IGF-I possesses
insulin-like activity, increases protein synthesis, stimulates DNA thymidine
incorporation, and expresses antilipolytic actions. Synthetic IGF-I
(somatomedin-1) can be used in children with growth disorders caused by
GH insensitivity. The other actions listed are not thought to be mediated by
somatomedins.
Human GH increases hepatic glucose output and exerts anti-insulin effects
(choice A) in muscle. Long-exposure to this hormone is usually associated
with hyperinsulinemia, and produces the state of insulin resistance. The
mechanism is not clear, but involves receptor and postreceptor interactions.
GH increases intestinal absorption of calcium (choice C).
GH increases circulating FFA (free fatty acids) levels (choice D), which is an
important energy source for cells during hypoglycemia, fasting or stressful
situations.
GH causes Na
+
retention (choice E) by inhibiting excretion of sodium by
kidneys.

Question 5 of 5

The lung carcinoma seen on the chest x-ray film is most likely which of the
following pathologic types?
/ A. Adenocarcinoma
/ B. Large cell carcinoma
/ C. Pancoast tumor
/ D. Small cell carcinoma
/ E. Squamous cell carcinoma

Explanation - Q: 2.5 Close

The correct answer is E. Paraneoplastic syndromes are clinical syndromes
resulting from tumor-produced hormones and occur in 10-15% of cancer
patients. Lung cancers can cause several paraneoplastic syndromes based
on the humoral factor being produced. Squamous cell carcinomas are one of
the most common primary malignancies of the lung and are often seen in
smokers. They usually arise from central bronchi, producing a hilar mass.
Hypercalcemia, caused by the secretion of parathyroid hormone-related
peptide (PTHrP), and acromegaly, caused by an ectopic secretion of growth
hormone, are endocrine paraneoplastic syndromes associated with
squamous cell carcinoma of the lung.
Adenocarcinoma (choice A) often forms on scars, or in lungs with interstitial
disease. It is the most prevalent form in the U.S. (35% of cases). It often
induces fibrotic changes, usually accompanied by hilar and mediastinal node
involvement. Adenocarcinoma is less strongly associated with smoking than
squamous cell carcinoma.
Large cell lung carcinoma (choice B) may be of a giant cell or a clear cell
variant. Giant cell carcinoma is a large cell type with a component of highly
pleomorphic, multinucleated cells. It is particularly aggressive and carries a
very poor prognosis.
Apical localization, with tumor invading the brachial plexus and sympathetic
chain, pain in the shoulder, and Horner syndrome (ipsilateral miosis, ptosis
and anhidrosis) are pathognomonic for Pancoast tumor (choice C). These
tumors most commonly represent a local extension of a squamous cell
carcinoma to the upper part of the lung.
Small-cell (oat-cell) carcinoma (choice D) occurs almost exclusively in
smokers. It is a very aggressive type and often metastasizes before the
tumor reaches a large size. Microscopically, small cells with minimal
cytoplasm are seen. SIADH (syndrome of inappropriate secretion of
antidiuretic hormone) is most common in cases with small cell lung cancer.


A frantic mother brings her 2-week-old daughter to the emergency department
because of protracted vomiting. She states her baby has been
vomiting for the last few days and was not tolerating any Pedialyte, milk, or
water. The baby had been "very fussy" but had not been feverish.
The course of her pregnancy was uneventful and she was vaginally delivered
without any complications.On examination, the baby appears ilI,
but well developed. Her blood pressure is 50/30 mm Hg, pulse is 176/min, and
respiratory rate is 35/min. Her oral mucosa look dry and she is
not tearing much. Her anterior fontanelle appears sunken, and mild tenting can
be elicited in her skin. An enlarged clitoris and partial fusion of
the labial folds is noted. Serum electrolytes are significant for sodium of 123
mEq/L, chloride of 92 mEq/L, and bicarbonate of 27 mEq/L.

Question 1 of 5

Which of the following is most likely diagnosis?
/ A. Congenital adrenal hyperplasia
/ B. Hermaphroditism
/ C. Mixed gonadal dysgenesis
/ D. Pyloric stenosis
/ E. Viral gastroenteritis
Explanation - Q: 3.1 Close

The correct answer is A. Congenital adrenal hyperplasia (CAH) is an
autosomal recessive disease typified by adrenal insufficiency (hyponatremic
hypovolemia and/or shock) from decreased aldosterone production and
abnormal sexual development. In the classic form, female newborns present
with ambiguous genitalia ranging from an enlarged clitoris, partial to
complete fusion of the labioscrotal folds, and presence of a urogenital sinus.
Adrenal crisis with severe salt wasting may occur. For patients with salt
wasting congenital adrenal hyperplasia, both glucocorticoids (to replace
cortisol) and mineralocorticoids (to replace aldosterone) should be used in
addition to IV fluids for resuscitation.
True hermaphroditism (choice B) is a condition in which both an ovary and a
testis are present. The external genitalia can display all gradations of the
male-to-female spectrum so an enlarged clitoris and labioscrotal fusion can
be seen. However, there is no associated adrenal crisis.
Mixed gonadal dysgenesis (choice C) is the 2nd most common cause of
ambiguous genitalia; however, there is no associated adrenal insufficiency. It
may become more apparent during sexual maturation during puberty.
Babies with pyloric stenosis (choice D) can have projectile bilious vomiting in
the first few days of their birth and can become severely dehydrated.
However, they do not have ambiguous genitalia.
Similarly, patients with viral gastroenteritis (choice E) can be dehydrated
from vomiting, but do not have ambiguous genitalia.

Question 2 of 5
Which of the following is the most common cause of ambiguous genitalia?
/ A. 5-alpha-reductase deficiency
/ B. 11-hydroxylase deficiency
/ C. Maternal ingestion of virilization drugs during pregnancy
/ D. Mosaic 45,X/46,XY
/ E. 21-hydroxylase deficiency

Explanation - Q: 3.2 Close

The correct answer is E. 21-hydroxylase deficiency is the most common
cause of ambiguous genitalia and accounts for 80-95% of CAH. There are 2
types of deficiencies; the classic and salt wasting form tends to present in the
newborn but the non-classic form can occur in late childhood and
adolescence and is less associated with salt wasting. The ambiguous
genitalia results from the virilization effects of excess DHEA produced.
In the testes, 5-alpha-reductase (choice A) converts testosterone to
dihydrotestosterone and a single mutation in the enzyme could result in
ambiguous genitalia.
11-hydroxylase deficiency (choice B) can present similarly to late-onset 21-
hydroxylase deficiency, with hirsutism and abnormal menses, but it is less
common.
Maternal ingestion of progesterone (choice C) has been associated with
virilization in utero.
Mosaic 45X/46,XY (choice D) is the karyotype for mixed gonadal
dysgenesis.


Question 3 of 5

Which of the following results would be diagnostic of this patient's condition?
/ A. Decreased serum ACTH
/ B. Decreased serum aldosterone
/ C. Increased serum DHEA
/ D. Increased serum 17-OH-progesterone
/ E. Increased urinary 17-ketosteroids

Explanation - Q: 3.3 Close

The correct answer is D. Patients with 21-hydroxylase deficiencies have
elevated 17-OH-progesterone because the enzyme deficiency prevents the
formation of aldosterone, so the pathway is shunted toward the formation of
DHEA and cortisol. Since 21-hydroxylase is also needed for cortisol
synthesis, 17-OH-progesterone is accumulated. Therefore, patients have
both aldosterone and cortisol deficiency and go into adrenal crisis.
Since the cortisol is low, ACTH (choice A) is elevated in patients with 21-
hydroxylase deficiency, resulting in the hyperplasia of adrenals.
Decreased aldosterone (choice B), elevated DHEA (choice C), and
elevated urinary 17-ketosteroids (choice E) can be seen in both 11- and 21-
hydroxylase deficiencies. 17-OH-progesterone is more specific for a 21-
hydroxylase deficiency.

Question 4 of 5
If a young woman had 11-hydroxylase deficiency, which of the following
presentations would be most likely?
/ A. Adrenal insufficiency
/ B. EIevated 11-deoxycortisol
/ C. Hyperkalemia and hypotension
/ D. Increased aldosterone
/ E. Normal menses and hair growth

Explanation - Q: 3.4 Close

The correct answer is B. Patients with 11-hydroxylase deficiency present
with features of androgen excess, rather than adrenal insufficiency (choice
A). 11-hydroxylase deficiency results in the accumulation of 11-
deoxycorticosterone and 11-deoxycortisol. Since 11-deoxycorticosterone is
an active mineralocorticoid, it has the properties of aldosterone and patients
can have hypertension and hypokalemia (compare with choice C).
Therefore, the measurable aldosterone level is low (compare with choice D)
in these patients.
Hirsutism and abnormal menses (compare with choice E), pre- or
postpubertally, tend to be the cause of investigation and diagnosis in these
patients.


Question 5 of 5

Which of the following would help differentiate between polycystic ovary disease
and late-onset 21-hydroxylase deficiency?
/ A. Abnormal menses or primary amenorrhea
/ B. FSH/LH ratio
/ C. Hirsutism
/ D. PIasma androgens (testosterone and DHEA)
/ E. Urinary 17-ketosteroids

Explanation - Q: 3.5 Close

The correct answer is B. Polycystic ovarian disease (PCOD) is classically
associated with a ratio of LH/FSH > 3. These are normal in CAH.
Both PCOD and CAH are part of the workup for any abnormal menses or
primary amenorrhea (choice A) and hirsutism (choice C), which occurs from
the elevated plasma androgens (choice D). Increased plasma androgens
tend to lead to elevated urinary excretion of 17-ketosteroids (choice E).







A 19-year-old Hispanic man is brought in to the emergency department by his
family because of 3-4 days of nausea, vomiting, and fatigue.
The patient also complained of diffuse abdominal cramping as well as a few
watery stools over the last few days. He admitted to "partying a
Iittle" with his friends prior to the onset of symptoms and drank a "few beers" as
well as eating at local taco stands in Tijuana. Since then, he
has been urinating frequently and drinking juice, but he has not been eating
much. When he is able to eat, it does not exacerbate his
abdominal pain. He denies any fever, chills, dysuria, or constipation. The
patient's mother, who is diabetic, is very concerned about her son's
habits and lifestyle. On examination, the patient appears slightly lethargic, but
answers questions appropriately. His lips and oral mucosa are
parched and dry and there is a slightly stale "sweet" odor on his breath. His pulse
is 101/min and respirations are 18/min. The patient has mild
diffuse tenderness in his abdomen, but no peritoneal signs, including rebound
tenderness, can be elicited. No hepatosplenomegaly is
detected and the rectal examination is normaI.

Question 1 of 5
Which of the following is the most likely diagnosis?
/ A. Acute appendicitis
/ B. AIcoholic hepatitis
/ C. AIcoholic pancreatitis
/ D. Diabetic ketoacidosis
/ E. Traveler's diarrhea


Explanation - Q: 4.1 Close

The correct answer is D. The patient is presenting with new onset diabetes
(Type 1) in the form of diabetic ketoacidosis (DKA).There is often a history of
diabetes in the family. The patient is exhibiting classic symptoms such as
polydipsia and polyuria, as well as "fruity" breath from the acidosis. The
stupor the patient is manifesting can progress into coma if left untreated.
Patients with DKA also have nonspecific abdominal pain and cramps. Often,
DKA can be precipitated when diabetic patients become acidotic and
dehydrated from an alcoholic binge.
If the patient had acute appendicitis (choice A), he would likely initially
present with periumbilical pain, which can then migrate down to the right
lower quadrant. Low grade fever, nausea, vomiting, a sense of constipation,
and anorexia are common. Patients can have positive psoas signs (pain with
hyperextending right thigh while lying on the left hip) and obturator signs
(flexing and rotating the hip) along with uncomfortable rectal exams.
However, acute appendicitis can have many atypical presentations and can
mimic gynecological disorders and gastroenteritis.
Patients with alcoholic hepatitis (choice B) can vary from being
asymptomatic with an enlarged liver to being critically ill. Often symptoms
include anorexia, nausea, jaundice, as well as hepatomegaly. They can also
have abdominal tenderness and ascites, and encephalopathy can also be
present.
Patients with pancreatitis (choice C) often have nausea, vomiting, abdominal
pain, and can become severely dehydrated. Abdominal pain is often
exacerbated by eating and alleviated by remaining NPO (nothing by mouth).
Traveler's diarrhea (choice E) is associated with nausea/vomiting/watery
stools, but is not particularly associated with polydipsia/polyuria/fruity breath.
Furthermore, most patients do not become severely dehydrated and are able
to tolerate some food

Question 2 of 5
Laboratory results show:
Sodium 136 mEq/L
GIucose 437 mg/dL
Potassium 4.8 mEq/L
Bicarbonate 15 mEq/L
Chloride 98 mEq/L
Urea nitrogen (BUN) 9 mg/dL
Creatinine 0.5 mg/dL
Leukocyte count 18,000/mm3
Which is the most appropriate next step in management?
/ A. Cefazolin and gentamicin
/ B. Cimetidine
/ C. IV fluids and insulin
/ D. Observation only
/ E. Prochlorperazine

Explanation - Q: 4.2 Close

The correct answer is C. Patients with diabetic ketoacidosis should be
treated with IV fluids and insulin in order to reverse the ketoacidosis and
correct the volume depletion.
Even though the patient has an elevated white blood cell count, this
leukocytosis is most likely due to ketoacidosis. Therefore, antibiotics (choice
A) are not needed unless the patient demonstrates other signs of infection,
i.e., fever, or positive blood or urine cultures.
Cimetidine (choice B) is used to decrease acid production; this is only useful
if the patient had gastroesophageal reflux disease (GERD) reflux and
alcoholic gastritis.
Observation (choice D) alone would result in a worsening of the patient's
ketoacidosis and would worsen the stupor, leading to coma and death.
Prochlorperazine (choice E) would decrease the patient's nausea and
vomiting, but would not treat the underlying problem.

Question 3 of 5
Which of the following test results would best confirm the likely diagnosis?
/ A. Acidic pH on arterial blood gas analysis
/ B. High anion gap
/ C. Low serum bicarbonate
/ D. Positive serum ketones
/ E. Positive urine ketones

Explanation - Q: 4.3 Close

The correct answer is D. Serum ketone positivity is an important criterion
used to separate diabetic ketoacidosis from hyperglycemic hyperosmolar
nonketotic coma (HHNC) seen in type 2 diabetics. In addition to the
bicarbonate level, it is also used as a measure to gauge the resolution of the
ketoacidosis; it disappears after treatment. Ketogenesis results from insulin
deficiency and glucagon excess; insulin deficiency favors lipolysis, leading to
elevated plasma fatty acids and ketone bodies (beta-hydroxybutyrate and
acetoacetate).
An acidic pH on ABG (choice A) is present in any acidotic state, and is not
specific for ketoacidosis.
A high anion gap (choice B) can be seen in other states, e.g., uremia,
methanol poisoning, salicylate ingestion. It is not specific to diabetic
ketoacidosis.
Low bicarbonate (choice C) is associated with any form of primary or
compensatory metabolic acidosis, and is not specific for ketoacidosis.
Positive urine ketones (choice E) can be seen in both diabetic ketoacidosis
and HHNC; it can also accompany starvation ketoacidosis as well as alcohol-
induced acidosis. Therefore, the presence of ketonuria is not diagnostic for
diabetic ketoacidosis.




Question 4 of 5

Prior to instituting therapy, blood samples are sent for repeat electrolyte
determination and arterial blood gas analysis, yielding the following
results:

Sodium 136 mEq/L
Potassium 4.8 mEq/L
Bicarbonate 15 mEq/L
Chloride 98 mEq/L

Arterial blood gases (ABG):
pH 7.3
pCO2 31
Bicarbonate 15 mEq/L
O2 saturation 98% on room air

Which of the following acid-base disorders is present in this patient?

/ A. High anion gap metabolic acidosis
/ B. High anion gap metabolic acidosis with compensatory respiratory alkalosis
/ C. Metabolic alkalosis
/ D. Metabolic alkalosis with compensatory respiratory acidosis
/ E. Normal anion gap metabolic acidosis with respiratory alkalosis

Explanation - Q: 4.4 Close

The correct answer is B. Diabetic ketoacidosis is a form of high anion gap
metabolic acidosis. In general, metabolic alkalosis is characterized by
elevated bicarbonate (>24) so choices C and D can be eliminated. There
are instances in which patients with chronic metabolic alkalosis would have
compensatory respiratory acidosis so that the HCO
3
-
is within normal limits
(around 23-24).
Conversely, low HCO
3
-
(<24) is suggestive of metabolic acidosis. To
differentiate amongst the remaining 3 choices, one must calculate the anion
gap. The anion gap formula is [Na
+
- (Cl
-
+ HCO
3
-
)]. Normal anion gap is 12.
For this patient, his anion gap is [136-(98+15)] = 23. Therefore, he has high
anion gap metabolic acidosis. This rules out choice E.
Next, one needs to determine whether the patient has compensatory
respiratory alkalosis. An easy way of determining the equilibrium between
PaCO
2
(from ABG) and HCO
3
-
is as follows:

For example, if a drop of HCO
3
-
of 10 (in metabolic acidosis) occurs, a
compensatory respiratory alkalosis (breathing rapidly to remove the excess
acid in the body) should occur and a drop of 10 would be present on the
PaCO
2
on the ABG.
For this patient, his primary acid-base disorder is a metabolic acidosis from
overproduction of the ketoacids. His HCO
3
-
level has dropped by 9 and so his
PaCO
2
should compensate by dropping 9 (40-9 = 31). Therefore, the patient
has high anion gap metabolic acidosis with compensatory respiratory
alkalosis (compare with choice A).

Question 5 of 5

Repeat labs after therapy begins shows a glucose 285 mg/dL, and potassium of
3.1 mEq/L. Which of the following is the cause for the drop in
potassium after treatment?
/ A. Diarrhea
/ B. Dilutional effect
/ C. Protracted vomiting
/ D. Renal tubular acidosis
/ E. Reversal of acidosis

Explanation - Q: 4.5 Close

The correct answer is E. Patients with DKA may commonly have normal, or
even elevated potassium levels on initial labs. This hyperkalemia is due to
decreased insulin, which shifts K
+
extracellularly, and as well as
hyperosmolality (intracellular K
+
concentration of the dehydrated cell
increases and K
+
diffuses extracellularly). As acidosis is reversed via insulin
and fluids, K
+
is shifted intracellularly. Therefore, it is common to give
patients K
+
supplements even if their K
+
levels are normal when they first
present.
Profound diarrhea (choice A) can result in mild hypokalemia but it should
have presented in the initial labs.
Hypokalemia from protracted vomiting (choice C) should have been present
on initial labs and does not manifest after hydration. Hydration alone should
not drop the potassium by 1.7 from a dilutional effect (choice B) since a drop
of 1 mEq/L suggests at least a total body deficit of about 350 mEq.
Renal tubular acidosis (choice D) can result in metabolic acidosis and have
resulting hypokalemia; diabetics can have type 4 RTA, but this is associated
with hyperkalemia. There is no indication that the patient has long-standing
diabetes resulting in RTA, or has any form of bicarbonate wasting through
the kidneys.

A 36-year-old woman complains to her physician of chronic fatigue. On physical
examination, the patient is noted to have generalized pallor and koilonychia
(concavity of the outer nail surface).

Question 1 of 8

The combination of generalized pallor and chronic fatigue specifically
suggests which of the following general problems?

/ A. Anemia
/ B. Cancer
/ C. Granulomatous disease
/ D. Heart failure
/ E. Respiratory failure

Explanation - Q: 1.1 Close

The correct answer is A. This combination specifically suggests that
anemia is present, and should trigger at least a screening evaluation for this
condition. The other conditions listed in the choices may or may not also be
present, either incidentally or as a complication of or cause of the anemia.



Question 2 of 8


On further questioning, the patient notes that she has had a craving for ice lately,
often keeping a cup of ice handy to "chew on." Her hematocrit
is 25%. Which of the following is the most likely diagnosis?
/ A. Folate deficiency
/ B. Iron deficiency
/ C. Malaria
/ D. Sickle cell disease
/ E. Vitamin B12 deficiency

Explanation - Q: 1.2 Close

The correct answer is B. One of the more helpful clues to look for when
questioning people about a possible anemia is pagophagia (craving ice to
suck or chew), which is seen in up to one half of the patients with moderate
iron deficiency anemia. This may occur because the ice is soothing to these
patient's mouths, which are often irritated. Cheilosis (dry scaling and fissuring
of the lips), glossitis (inflammation of the tongue), and koilonychia (nails
whose outer surface is concave) may also be present. Pica refers to eating
of non-food substances such as dirt or paint. Pagophagia is surprisingly
specific for iron deficiency anemia, and is not seen with unusual frequency in
the other forms of anemia listed. While mild iron deficiency anemia is
sometimes normocellular, moderate to severe iron deficiency usually is
associated with many hypochromic microcytes. None of the other disorders
cause a craving for ice.



Question 3 of 8

Examination of a peripheral blood smear would most likely reveal which of the
following?
/ A. Microcytes
/ B. Schistocytes
/ C. Sickled cells
/ D. Spherocytes
/ E. Spur cells

Explanation - Q: 1.3 Close

The correct answer is A. While mild iron deficiency anemia is sometimes
normocellular, moderate to severe iron deficiency anemia usually shows
many hypochromic microcytes.
Schistocytes (choice B) are seen in traumatic hemolytic anemia and
immunohemolytic anemia.
Sickled cells (choice C) are seen in the various sickle cell syndromes.
Spherocytes (choice D) are seen in hereditary spherocytosis.
Spur cells (choice E) are seen in spur cell anemia and paroxysmal nocturnal
hemoglobinuria.

Question 4 of 8
Which of the following conditions would be most likely to predispose for this
patient's disorde
/ A. Dermoid cyst of the ovary
/ B. E.Coli bladder infection
/ C. Intraductal carcinoma of the breast
/ D. Menorrhagia
/ E. Squamous cell carcinoma of the skin

Explanation - Q: 1.4 Close

The correct answer is D. Iron deficiency anemia in this country is most
often seen in the setting of chronic blood loss. Menorrhagia, or prolonged,
heavy menstrual flow, and occult gastrointestinal bleeding are the most
commonly encountered predisposing conditions. The other conditions listed
in the choices are distracters that do not have a particular tendency to cause
chronic bleeding


Question 5 of 8
The deficient substance in this patient is typically absorbed at which of the
following sites?
/ A. Colon
/ B. Distal small bowel
/ C. Esophagus
/ D. Proximal small bowel
/ E. Stomach

Explanation - Q: 1.5 Close

The correct answer is D. The proximal small bowel is the site of absorption
of many vitamins and minerals, including iron. The absorptive epithelial cells
of the proximal small bowel alter iron absorption to match the body losses,
and contain considerable intracellular iron in patients with adequate iron. In
iron deficient subjects (and also in hemachromatosis, possibly contributing to
the pathology), the stainable iron stores in these cells are nearly absent. The
iron that is absorbed can be in the forms of heme, ferric iron, and ferrous iron
(using different pathways). Unlike with many substances, the entire control of
body levels is at the stage of absorption, since there is no physiologic
mechanism other than bleeding for removal of large amounts of iron from the
body.
The colon (choice A) primarily absorbs water and electrolytes.
The distal small bowel (choice B) notably absorbs vitamin B12.
The esophagus (choice C) and stomach (choice E) are not absorptive sites.

Question 6 of 8
The substance deficient in this patient is typically delivered to non-intestinal cells
by which of the following?
/ A. Ceruloplasmin
/ B. Erythropoietin
/ C. Hematoxylin
/ D. Hemosiderin
/ E. Transferrin

Explanation - Q: 1.6 Close

The correct answer is E. The iron which is delivered to other tissues is
primarily delivered bound to transferrin. The transferrin-iron complex enters
the cell within an endosome which forms after the complex binds to a
receptor on the cell surface. Subsequent acidification of the endosome
contents releases the iron so that it can be transported across the endosome
membrane and into the cytoplasm of the cell.
Ceruloplasmin (choice A) is a copper-transporting molecule.
Erythropoietin (choice B) is a hormone that stimulates erythrocyte
production.
Hematoxylin (choice C) is a commonly used stain in histology.
Hemosiderin (choice D) is a common storage form of iron in tissues.


Question 7 of 8
The patient is prescribed supplementation for her deficiency. Her 3-year-old child
finds the bottle of pills, and ingests some of them. If the child
ingested enough to become clinically poisoned, which of the following symptoms
would most likely be seen earliest in the course?
/ A. Hepatic cirrhosis
/ B. Intestinal obstruction
/ C. Seizures
/ D. Shock
/ E. Vomiting and explosive diarrhea

Explanation - Q: 1.7 Close

The correct answer is E. Poisoning with iron supplements is always a
potential problem in households with young children and a mother with iron-
deficiency anemia, and the problem is exacerbated by the fact that some of
the commonly available iron supplements are small round pills with bright red
coating that very much resemble small candies such as M&M's and Skittles.
Fortunately, most commonly the poisoning is mild, although fatal cases of
poisoning have been encountered with doses as small as 130 mg of
elemental iron. The advice to parents about keeping the medication well out
of reach of little hands is obvious, but should be emphasized by the clinician
at the time of prescribing the supplementation.
The clinical course following iron poisoning has been divided into four
stages. Stage I occurs within 6 hours, and can be characterized by vomiting,
hematemesis, explosive diarrhea, irritability, and abdominal pain (choice E).
The presence of shock (choice D) or coma within the first 6 hours is
considered a grave prognostic sign. Other clinical features that can be
present in stage I if iron levels are particularly high include tachypnea,
tachycardia, hypotension, and metabolic acidosis. Stage II occurs 6-24 hours
after ingestion and is characterized by a latent period of apparent (but
deceptive) clinical improvement. Stage III, which typically begins 12 to 48
hours after ingestion is a life-threatening stage characterized by liver
damage, shock (choice D), hypoperfusion, seizures (choice C),
hypoglycemia, fever, ECG changes, bleeding disorders, lethargy, coma,
acidosis, and sometimes death. Stage IV which occurs 2 to 5 weeks later in
those patients that develop late complications may manifest with
gastrointestinal obstruction (choice B), hepatic cirrhosis (choice A), or
permanent CNS damage.

Question 8 of 8
Which of the following is the most appropriate pharmacotherapy for her child?
/ A. Deferoxamine
/ B. Dimercaprol
/ C. Edetate (EDTA)
/ D. Penicillamine
/ E. Protamine
/ F. Succimer

Explanation - Q: 1.8 Close

The correct answer is A. Deferoxamine is an iron chelator that binds the
absorbed iron, but not the iron in iron-carrying proteins such as hemoglobin,
myoglobin, hemosiderin, or ferritin. It is given parenterally; IV is generally the
preferred route.
Dimercaprol (choice B) is a chelator used for arsenic and mercury
poisoning.
Edetate (EDTA) (choice C) is a chelator used for lead poisoning.
Penicillamine (choice D) is a chelator used for copper poisoning (and Wilson
disease). It is sometimes used for adjunctive therapy for gold, arsenic, and
lead poisoning.
Protamine (choice E) is used for heparin overdose.
Succimer (choice F) is used in the treatment of lead poisoning; it can be
given orally. It can also be used for arsenic and mercury poisoning if used
soon after exposure.




A 52-year-old woman presents with complaints of fatigue, mood changes, diffuse
joint aches, and dry skin. She also states that she has
gained about 10 pounds in the last few months, without any change in appetite.
Her last menstrual period was 2 months ago and she has been
having very irregular menses in the last 8 months. Review of systems is
significant for constipation, but she denies hot flashes or insomnia.
Physical examination is significant for an obese woman who appears tired. An
asymmetric goiter is noted, but no bruit is appreciated. Her
joint examination is unremarkable; there is no swelling/redness and no trigger
points are painful to palpation. Labs were sent.
Question 1 of 6
Which of the following is the most likely diagnosis?
/ A. Depression
/ B. Diabetes mellitus
/ C. Fibromyalgia
/ D. Hypothyroidism
/ E. Perimenopause

Explanation - Q: 2.1 Close

The correct answer is D. Any patient with symptoms of fatigue/dry
skin/weight loss/ constipation coupled with the presence of a goiter should be
investigated for hypothyroidism. About 8% of the population over the age of
65 has hypothyroidism. It can be diagnosed by a high serum TSH.
Depression (choice A) can account for most the above symptoms but
medical conditions such as hypothyroidism, anemia, and diabetes should be
excluded first.
Diabetes (choice B) can produce multiple symptoms and should be included
in any workup of fatigue/weight gain, etc.
Fibromyalgia (choice C) is a chronic pain syndrome characterized by
difficulty sleeping and having at least 11 out of 16 specific tender points
designated by the American College of Rheumatology. Hypothyroidism and
fibromyalgia can coexist and the former is always included in the initial
workup of the latter.
Perimenopausal women (choice E) can have similar complaints as those
with hypothyroidism but they will all have hot flashes. Hypothyroid patients
are actually cold intolerant.


Question 2 of 6
The most common cause for this patient's condition is associated with which of
the following?
/ A. High antimitochondrial antibodies
/ B. High antithyroid peroxidase (anti-TPO) antibodies
/ C. High RAIU
/ D. Low anti-smooth muscle antibodies
/ E. Low antithyroglobulin antibodies

Explanation - Q: 2.2 Close

The correct answer is B. Hashimoto thyroiditis is the most common cause
of hypothyroidism. It is believed to be a result of an autoimmune response (T
cell-mediated hypersensitivity) against the thyroid. Clinically, it is associated
with an asymmetric nodular goiter with a high TSH and low T4. Like Graves'
disease, it has a strong autoimmune association and has high titers of
antithyroid peroxidase antibodies and high antithyroglobulin antibodies
(compare to choice E).
Antimitochondrial antibodies (choice A) are associated with primary biliary
cirrhosis.
A high RAIU (radioactive iodine uptake scan, choice C) is only seen in
patients who are hyperthyroid and is not used to diagnosis hypothyroidism.
Anti-smooth muscle antibodies (choice D) are associated with autoimmune
hepatitis.


Question 3 of 6

Which of the following would a thyroid biopsy most likely show?
/ A. Thyroid follicle destruction and fibrosis by macrophages and giant cells
/ B. Thyroid follicle destruction by lymphocytes; presence of Hurthle cells
/ C. Thyroid follicles with finger-Iike growths and "Orphan Annie eyes"
/ D. Thyroid follicles with large colloid and flattened epithelial cells
/ E. Thyroid follicles with scant colloid and hyperplastic epithelial cells

Explanation - Q: 2.3 Close

The correct answer is B. Thyroid follicular infiltration by lymphocytes along
with the presence of Hurthle cells is characteristic of Hashimoto disease.
Hurthle cells are the surviving follicular epithelial cells that are transformed
into large cells with abundant pink cytoplasm.
Thyroid follicle destruction by macrophages and giant cells (choice A) is
found in subacute thyroiditis.
Finger-like growths or papillary formation of the thyroid follicles (choice C) is
characteristic of papillary carcinoma of the thyroid; Orphan Annie eyes (clear
nuclei in the follicular epithelial cells) are pathognomonic.
Thyroid follicles with large colloid centers (choice D) are typical for toxic
multinodular goiter.
Initially, the euthyroid multinodular goiter can also have findings similar to
Graves' disease, with hyperplastic epithelial cells with scant central colloids
(choice E).



Question 4 of 6
Which of the following is the most appropriate pharmacotherapy?
/ A. Amitriptyline
/ B. Aspirin or NSAIDS
/ C. Conjugated estrogen and progesterone
/ D. Levothyroxine
/ E. Selective serotonin reuptake inhibitors (SSRI)

Explanation - Q: 2.4 Close

The correct answer is D. Levothyroxine (T4) is the most commonly used
form of the thyroid hormone for supplement. It is converted to T3 (active
form) in the peripheral tissues and has a half-life of 7 days.
Amitriptyline (choice A), a tricyclic antidepressant, is used to correct the
disturbed stage 4 sleep in patients with fibromyalgia.
Aspirin/NSAIDS (choice B) is used to treat the discomfort of subacute
thyroiditis.
Hormone replacement (choice C) is very helpful to alleviate the hot flash
symptoms associated with perimenopause.
SSRI's (choice E), such as fluoxetine (Prozac), are used to treat depression.


Question 5 of 6
The patient had unknown cardiac risk factors and developed a myocardial
infarction after therapy was initiated. Which of the following is the
most likely mechanism by which the therapy lead to this complication?
/ A. Enhances adrenergic effects
/ B. Enhances lipid turnover
/ C. Increases calcium mobilization in bone
/ D. Increases metabolic rate
/ E. Regulates body temperature

Explanation - Q: 2.5 Close

The correct answer is A. Thyroxine has multiple effects on the body such
as improving the lipid profile (choice B); increasing calcium mobilization from
the bone (choice C), which can cause chronic hyperthyroid patients to be
osteoporotic; increasing the metabolic rate (choice D); and regulating body
temperature (choice E). It also enhances adrenergic actions thereby
increasing heart rate and cardiac contractility. For older patients or those with
coronary artery disease, thyroid supplements must be given in smaller and
slower titrations to prevent this complication.

Question 6 of 6

If the patient had a very tender and nodular goiter with pain radiating to the ear,
which of the following would be the most likely diagnosis?
/ A. Graves' disease
/ B. Hashimoto's thyroiditis
/ C. Lymphocytic thyroiditis
/ D. Subacute thyroiditis
/ E. Toxic multinodular goiter

Explanation - Q: 2.6 Close

The correct answer is D. Patients with subacute thyroiditis have very tender
asymmetrical goiters with pain radiating to the ear. It usually starts after a
viral infection. The patient may range from being hyperthyroid to hypothyroid,
but eventually will become euthyroid.
Graves' disease (choice A) occurs in patients who are hyperthyroid with a
goiter.
Hashimoto thyroiditis (choice B) is not associated with a painful goiter and
does not occur after a viral infection.
Lymphocytic thyroiditis (choice C) is a painless thyroiditis, which is self-
limited; it can progress from hyperthyroid to hypothyroid and then to the
euthyroid state. Some consider lymphocytic thyroiditis and subacute
thyroiditis variants of the same thyroid inflammation.
Patients with toxic multinodular goiter (choice E) will be hyperthyroid instead
of hypothyroid.



A 23-year old female student notices double vision while studying for her
examinations. She goes to see an ophthalmologist, who said she
had a "Iazy eye." These symptoms come and go for the next 2 years without
becoming significant. Then, she starts to have difficulties with
chewing food, and has troubles with getting in and out of the car. SIowly,
problems with using her arms and legs develop. She thinks she is out
of shape and needs more exercise. She begins spilling drinks and missing her
mouth when using utensils. Her vision is double by afternoon
each day, and by the evening, one eye is closed. The next morning, her situation
is improved, but again worsens by the evening. Finally, she
goes to see a neurologist. On examination, ptosis is noted. The ptosis worsens
when she is asked to sustain an upward gaze, and closing her
eyes for a short period improves it. Her voice has a nasal quality. Sensory
examination and deep tendon reflexes are normaI.

Question 1 of 5

Which of the following is the most likely diagnosis?
/ A. Botulism
/ B. Guillain-Barr syndrome
/ C. Lambert-Eaton syndrome
/ D. Myasthenia gravis
/ E. Wernicke syndrome

Explanation - Q: 3.1 Close

The correct answer is D. Myasthenia gravis (MG) is a chronic autoimmune
disease of neuromuscular transmission. It is characterized by fluctuating
weakness of commonly used voluntary muscles, causing symptoms such as
ptosis, diplopia, dysphagia, drooping head, poor posture, difficulty climbing
stairs, and difficulty chewing and talking. Weakness is improved by rest and
worsens with activity. Short-acting anti-cholinesterases transiently improve
the weakness. The disorder can occur at all ages, sometimes in association
with a thymic tumor, or thyrotoxicosis, rheumatoid arthritis, and systemic
lupus erythematosus. Extraocular muscles and other cranial muscles
(masticatory, facial, and pharyngeal) are especially likely to be affected, and
the respiratory and limb muscles may also be involved. Pupillary responses,
sensation, and deep tendon reflexes are normal. Symptoms often fluctuate in
intensity during the day, and these diurnal variations are superimposed on a
tendency to have spontaneous relapses and remissions. The disorder
follows a slowly progressive course and may have a terminal outcome due to
respiratory complications.
Botulism (choice A) is a potentially fatal paralytic illness caused by the
bacterium Clostridium botulinum. Botulism would not slowly worsen over two
years.
Guillain-Barr syndrome (choice B) is an acute autoimmune demyelinating
neuropathy characterized by progressive symmetric ascending weakness,
bilateral hyporeflexia, paresthesias, pain, cranial nerve involvement,
dysautonomia, and eventually, respiratory insufficiency.
Lambert-Eaton syndrome (choice C) is an autoimmune disease of
neuromuscular transmission, often associated with malignancy, which is
characterized by weakness of proximal muscles and facilitation (strength
improvement) after exercise.
Chronic progressive external ophthalmoplegia (choice E) is a disorder
characterized by slow progressive paralysis of extraocular muscles.


Question 2 of 5
The underlying pathophysiology in this disease involves which of the following
mechanisms?
/ A. Antibodies to nicotinic acetylcholine receptors
/ B. Antibodies to voltage-gated calcium channels
/ C. Bacterial toxin
/ D. Infection-induced autoimmune response
/ E. Thiamine deficiency

Explanation - Q: 3.2 Close

The correct answer is A. Myasthenia gravis (MG) is an acquired
autoimmune disorder characterized by autoantibodies directed to
acetylcholine receptors (AChR) at the neuromuscular junction. Specific
acetylcholine receptor autoantibodies lead to reduced impulse transmission
in the neuromuscular junction. Cholinergic receptors of smooth and cardiac
muscle (muscarinic) have a different antigenicity and therefore are not
affected by the disease. T-helper cells, which have been activated in the
thymus, probably stimulate the production of these antibodies. They react
mainly with the alpha subunit of the receptor, reducing the functional activity
at the postsynaptic membrane. There are several postulated mechanisms of
autoimmune damage: modulation (increased endocytosis and degradation of
AChR), blocking the binding site for ACh on AChR, and initiating
complement-binding to ACh-AChR complex and damage to postjunctional
membranes.
In Lambert-Eaton myasthenic syndrome antibodies induce downregulation of
voltage-gated calcium channels (VGCC) at the presynaptic terminal of
neuromuscular junction (choice B). This reduces calcium-dependent
acetylcholine release, which leads to fatigable muscle weakness and loss of
tendon reflexes.
Clostridium botulinum neurotoxin causes botulism by binding to peripheral
cholinergic synapses and preventing neurotransmitter release, which leads
to muscle paralysis (choice C).
Guillain-Barr syndrome is considered to be a postinfectious, autoimmune
disease (choice D). The mechanism most likely involves cross-reactivity
between antibodies against infectious agents and axonal and/or myelin
components e.g., GM1, GM2, GD1b.
Thiamine deficiency is associated with Wernicke encephalopathy, (choice E)
characterized by ataxia, ophthalmoplegia, and confusion. This disorder
usually occurs in alcoholics, or those with severe nutritional deficiencies.


Question 3 of 5
The structures primarily affected in this disease belong to which of the following
superfamilies?
/ A. Cytokine receptors
/ B. Intracellular receptors
/ C. Ligand-gated ion channels
/ D. Receptors linked to effectors via G proteins
/ E. Receptors located on a membrane-spanning enzyme

Explanation - Q: 3.3 Close

The correct answer is C. The structures involved in this disease are the
nicotinic cholinergic receptors, which are ligand-gated ion channels. The
receptor located at the neuromuscular junction is a pentamer composed of 5
polypeptide units (2, 1, 1, and 1). Each polypeptide has four
transmembrane-spanning regions, which form a cylindrical structure when all
five are joined together. ACh binds to the subunits, producing a
conformational change that results in the opening of the channel, allowing
sodium to enter the cell.
Cytokine receptors (choice A) respond to a variety of peptide ligands,
including erythropoietin, several types of interferon, growth hormone, and
other regulators of growth and differentiation. Their mechanism is similar to
the receptor tyrosine kinases (e.g., insulin receptor), however, in this case,
the tyrosine kinase is not actually part of the receptor. Instead, a separate
tyrosine kinase (Janus kinases; JAKs) binds noncovalently to the receptor.
Once the receptor is activated, the cytokine receptors dimerize (like the EGF-
receptor), activating the JAKs to phosphorylate tyrosine residues on the
receptor. This leads to activation of another set of proteins (STATs, signal
transducers and activators of transcription), which dimerize, detach from the
receptor, and travel to the nucleus where it regulates specific gene
transcription.
Intracellular receptors (choice B) can bind lipid-soluble agents that diffuse
across the lipid bilayer (e.g., steroid hormones), which then stimulate gene
transcription by binding to specific DNA sequences. A specialized
transmembrane signalling device is not needed with this type of receptor.
Receptors linked to effectors via G proteins (choice D) are comprised of one
polypeptide chain that crosses the membrane seven times. The receptor
activates a G protein, which mediates a variety of effects. There are several
different types of G proteins, e.g., Gs (stimulates adenylyl cyclase), Gi
(inhibits adenylyl cyclase, opens cardiac K
+
channels), and Gq (activates
phospholipase C).
Receptors located on a membrane-spanning enzyme (choice E) are
polypeptides with an extracellular hormone-binding domain and an
intracellular enzyme domain, which may be a tyrosine kinase, a serine
kinase, or a guanylyl cyclase. The two domains are connected via a
hydrophobic spanning region. Insulin and many growth factors stimulate
receptors that dimerize and initiate a tyrosine kinase signalling pathway.


Question 4 of 5
Which of the following would be likely to exacerbate this patient's symptoms?
/ A. Aminoglycosides
/ B. Edrophonium
/ C. Neostigmine
/ D. Pyridostigmine
/ E. Thymectomy

Explanation - Q: 3.4 Close

The correct answer is A. Patients with MG are extremely sensitive to the
action of curariform drugs or other drugs that interfere with neuromuscular
transmission. Aminoglycosides (e.g., streptomycin, neomycin, gentamycin,
tobramycin, amikacin) are antibacterial agents that inhibit protein synthesis
by acting directly on the ribosome. Ototoxicity and neurotoxicity are the most
serious side effects. Neuromuscular blockade may result when an
aminoglycoside is used in high doses, or with curariform drugs. The
mechanism is probably decreased presynaptic release of ACh and
decreased sensitivity of the postsynaptic membrane. Many other agents can
exacerbate MG symptoms, e.g., chloroquine, ciprofloxacin, muscle relaxants,
botulinum toxin, quinidine, procainamide, phenytoin, and d-penicillamine.
Edrophonium (choice B) is a rapidly-acting, short-duration, parenteral
cholinesterase inhibitor. It is used as a diagnostic test for MG.
Anticholinesterase agents are used for symptomatic therapy of myasthenia
gravis. Neostigmine (choice C) is a synthetic reversible acetylcholinesterase
inhibitor that is used for long-term therapy of MG. Neostigmine does not
penetrate blood-brain barrier, and provides an additional direct nicotinic
agonist effect. Pyridostigmine (choice D) is another anticholinesterase drug
used in the treatment of MG.
The role of the thymus in the pathogenesis of MG is not clear, but it was
found that 75% of MG patients has some type of thymic abnormality. That is
the rationale for thymectomy (choice E) as a therapeutic tool. Thymectomy
usually improves symptoms, and should be considered in patients younger
than 60, unless the disease has affected only extraocular muscles. Plasma
exchange may also be used in patients unresponsive to other treatments, but
produces only short-term clinical improvement.

Question 5 of 5

Administration of which of the following agents would confirm the diagnosis of
this disease?
/ A. Edrophonium
/ B. Nicotine
/ C. Pilocarpine
/ D. Pralidoxime
/ E. Succinylcholine

Explanation - Q: 3.5 Close

The correct answer is A. An edrophonium (Tensilon) IV challenge (double
blind) test is used to confirm the diagnosis of MG. Edrophonium is a drug of
choice for diagnosing MG because of its rapid onset of action and
reversibility. If the patient has MG, an improvement in muscle strength that
lasts 5 minutes or so will usually be seen. Other uses include assessing the
treatment of MG with longer-acting cholinesterase inhibitors (too much or too
little), differentiating cholinergic and myasthenic crises (in myasthenic crisis,
edrophonium improves muscle strength, but in cholinergic crisis, it further
reduces muscle strength) and reversing the effects of nondepolarizing
blocking agents after surgery.
Nicotine (choice B) is a natural alkaloid found in tobacco that mimics the
effects of ACh at nicotinic receptors. It has no therapeutic actions but is
important for its toxicity.
Pilocarpine (choice C) is a direct-acting muscarinic receptor agonist. It is
used in glaucoma to produce pupillary miosis and ciliary muscle contraction,
thereby increasing aqueous humor outflow.
Pralidoxime (2-PAM, pyridine-2-aldoxime methyl chloride, choice D)
reverses the effects of organophosphates, which are irreversible
anticholinesterase agents. Treatment must be within hours because of the
"aging" phenomenon, i.e., establishing a form that cannot be reversed.
Succinylcholine (choice E) is a depolarizing neuromuscular blocking agent.
It has the briefest duration of action of all neuromuscular blocking drugs.
Therefore, it is a drug of choice for endotracheal intubation, electroconvulsive
shock therapy, and termination of laryngospasm.

A 62-year-old woman presents to the emergency department with a high fever,
weakness, and malaise. She denies vomiting, dysuria, cough,
or diarrhea. She has been receiving a six week course of antibiotics at home for
osteomyelitis of her left foot. Her semi-permanent intravenous
Iine in her arm has not been replaced for the past three weeks. Her temperature
is 38 C (102 F), blood pressure is 150/80 mm Hg, pulse is
96/minute, and respirations are 16/minute. She has a grade 3/6 holosystolic
murmur heard best over the left fifth intercostal space, in the
midclavicular line. Her lungs are clear and her abdominal examination is normaI.
Her fingernails show small longitudinal splinter-type lesions.
The skin at the intravenous site looks normaI. When her previous charts are
reviewed, her cardiovascular examination did not note any
murmurs. A chest x-ray film showed no effusion. Laboratory studies show:
Hemoglobin= 12.2g/dL
Hematocrit= 35.3 %
White blood cell count (WBC)= 16,100/mm3
PIatelets= 283,000/mm3
Urine is negative for blood, protein, nitrites, and leukocyte esterase.
Three blood cultures are sent off before the patient is started on nafcillin and
gentamycin.


Question 1 of 6
Which of the following is the most likely diagnosis?
/ A. Bacterial endocarditis
/ B. Cardiomyopathy
/ C. Cellulitis
/ D. Osteomyelitis
/ E. Pyelonephritis

Explanation - Q: 1.1 Close

The correct answer is A. The patient has had high fevers, general malaise,
and weakness. She has had two foci of infection (the osteomyelitis and the
venous line) and she presents with a new cardiac murmur, splinter
hemorrhages, a raised white cell count, an increased erythrocyte
sedimentation rate, and three positive blood cultures. The remainder of her
sepsis work up is negative (i.e., urinalysis, chest x-ray). All of this is most
suggestive of bacterial endocarditis.
Cardiomyopathy (choice B) typically manifests itself as heart failure with
pulmonary edema, shortness of breath, and lower limb edema, which does
not describe this patient.
Cellulitis (choice C) is infection of the skin, and can be secondary to skin-
borne staphylococci. One would typically see warmth and erythema around
the intravenous line site as well as proximal migration of erythema from the
infection site. This patient's skin examination was normal. In addition, one
would not normally expect a new heart murmur to accompany cellulitis.
Osteomyelitis (choice D) is infection of the bone, and is typically caused by
staphylococci. This patient has already had extensive antibiotic therapy for
the past six weeks. It is unlikely that the osteomyelitis would flare up this
quickly. In addition, the presence of a new cardiac murmur would make the
diagnosis of endocarditis more likely.
Pyelonephritis (choice E) is infection of the upper urinary tract. Symptoms
would include high fever, vomiting, and dysuria along with abdominal pain.
The signs would include costovertebral angle tenderness (this patient had a
normal abdominal examination). In addition, there would be proteinuria,
positive leukocyte esterase, and positive nitrites in the urinalysis (this
patient's urinalysis was essentially negative). Overall, the patient's story
points away from this diagnosis.


Explanation - Q: 1.2 Close

The correct answer is B. While virtually any bacterial organism can cause
bacterial endocarditis, the vast majority of infections are caused by gram-
positive cocci. Coagulase-positive staphylococci (Staphylococcus aureus)
commonly cause bacterial endocarditis in patients with prosthetic valves and
in injection drug users. In this patient's case, however, she had been
exposed to a staphylococcal infection due to the extended period of time that
the venous cannula was in place for her intravenous antibiotics. Another
source of a staphylococcal infection could have been the osteomyelitis of her
left foot.
Approximately 5 to 10% of patients who are not intravenous drug users and
who have a native-valve endocarditis are infected by slow-growing, gram-
negative bacilli such as Hemophilus parainfluenza(choice A). Others include
Hemophilus aphrophilus, and Cardiobacterium hominis. It is more likely that
this patient has a staphylococcal infection.
Infection with a coagulase-negative staphylococcus like Staphylococcus
epidermidis(choice C) is a relatively common cause of bacterial endocarditis
in patients with valvular prostheses.
Streptococcus bovis(choice D) occasionally causes bacterial endocarditis in
patients with an underlying colon malignancy or intestinal disorder. No
information suggesting gastrointestinal problems was declared by the
patient.
Streptococcal viridans(choice E) is the most common cause of endocarditis
involving native valves in patients with congenital heart disease and in
patients who are not intravenous drug users. This patient is more likely to
have a staphylococcal infection based on the reasons given above.


Question 3 of 6

The patient's murmur most likely corresponds to which of the following?
/ A. Aortic valve
/ B. Interventricular septum
/ C. Mitral valve
/ D. Pulmonic valve
/ E. Tricuspid valve

Explanation - Q: 1.3 Close

The correct answer is C. The cardiac apex corresponds to the point of
maximal pulsation and generally coincides with the audible location of the
mitral valve. In this case, the audible marking is the right fifth intercostal
space, mid-clavicular line.
The aortic valve (choice A) can be auscultated over the right second
intercostal border.
The interventricular septum (choice B) does not have a reliable 'auscultatory'
marking like the heart valves. However, due to its complex embryological
origin, the membranous part of the septum may be the site for a ventricular
septal defect, which, if small, can easily be auscultated as a holosystolic
murmur along the left sternal border. This usually a pediatric phenomenon.
The pulmonic valve (choice D) can be auscultated over the left second
intercostal border.
The tricuspid valve (choice E) can be auscultated over the left fifth
intercostal space along the left sternal border.


Question 4 of 6
Which of the following is the most likely mechanism of action of gentamicin?
/ A. Binding irreversibly to ribosomes, inhibiting initiation of protein synthesis
/ B. Binding irreversibly to ribosomes, inhibiting translocation
/ C. BIocking access of transfer RNA to messenger RNA
/ D. BIocking peptidoglycan synthesis
/ E. Inactivation of the elongation factor, preventing translocation

Explanation - Q: 1.4 Close

The correct answer is A. Gentamicin is an aminoglycoside, and exerts its
effect by binding to the 30S subunit of the ribosome, distorting ribosomal
structure, and thereby interfering with the initiation of protein synthesis via
initiation factors.
Binding irreversibly to ribosomes to inhibit translocation (choice B) is the
mechanism of action of macrolides such as clindamycin and erythromycin. In
this case, they would bind to the 50S subunit of the bacterial ribosome.
Translocation (i.e., the advancement of the ribosomal unit towards the 3'-end
of the messenger RNA by three nucleotides) is then inhibited.
Blocking access of the aminoacyl-transfer RNA to the messenger RNA-
ribosome complex (choice C) is how tetracyclines work. This is done through
interaction with smaller ribosomal subunits.
Blocking peptidoglycan synthesis (choice D) is the mechanism of action of
penicillin. The initial step of penicillin action is binding of the drug to cell wall
receptors for penicillin. Transpeptidation is thus blocked, and the synthesis of
peptidoglycan is inhibited.
Inactivation of a eukaryotic elongation factor, thus preventing translocation
(choice E), is the mechanism of action of diphtheria toxin.


Question 5 of 6
Which of the following is an important side effect of gentamicin?
/ A. BIeeding
/ B. Dry mouth
/ C. Hearing loss
/ D. Vertigo
/ E. Visual disturbance

Explanation - Q: 1.5 Close

The correct answer is C. Ototoxicity and nephrotoxicity are the most
important side effects of aminoglycosides like gentamicin. Typically, one
would measure the patient's peak gentamicin level to make sure that it is not
at toxic levels. Bleeding, visual disturbances, dry mouth, and vertigo are not
typical side effects of gentamicin.
Bleeding (choice A) would be an important side effect of warfarin, aspirin,
clopidogrel, and heparin. A patient would have such medications held if they
are about to have an operation. Heparin has the shortest half-life, and is
ideal for hourly control if the patient requires anticoagulation (e.g., if they
have mitral valve stenosis or atrial fibrillation).
Dry mouth (choice B) would be seen with anticholinergic medications such
as oxybutynin. Also lithium can cause dry mouth and a metallic taste that
may also cause polydipsia.
Not many drugs have vertigo (choice D) as a side-effect, however it is
commonly seen, along with nausea and vomiting, with alcohol.
Visual disturbances (choice E) may occur sometimes in patients taking
digoxin. Usually the patient would describe 'yellowing' or 'haloing' of vision.



Question 6 of 6
Over the next few days, the blood cultures are positive for gram-positive cocci in
clusters and the patient continues to have fevers and a high
white cell count despite antibiotic therapy. To avoid antibiotic resistance of the
organism, nafcillin is discontinued and vancomycin started. The
spread of bacterial resistance in this case is via which of the following?
/ A. Fimbriae
/ B. FIagellae
/ C. PIasmids
/ D. Spores
/ E. Teichoic acid

Explanation - Q: 1.6 Close

The correct answer is C. Staphylococcal organisms that are resistant to
penicillin produce a beta-lactamase that destroys the drug. The genetic
material that allows the organism to produce the beta-lactamase enzyme is
carried on plasmids. These are small and specialized genetic elements that
are capable of replication within at least one cell line. The plasmid may be
transferred from one cell to another and the genetic coding for the enzyme
can be conveyed throughout the entire staphylococcal population, thereby
giving rise to nafcillin resistance.
Fimbriae (choice A) are actually components of gram-negative bacteria, so
they would not be found in staphylococcal species. Fimbriae are rigid surface
appendages that are composed of subunits called pilins. Their role is in the
adherence of symbiotic bacteria to host cells, or (as sex fimbriae), the
attachment of donor and recipient cells during bacterial conjugation.
Flagellae (choice B) are thread-like appendages composed entirely of
protein, 12 to 30 nanometers in diameter. They are a means of locomotion
for the organisms that possess them. They can be monotrichous (single
polar filament), lophotrichous (multiple polar flagella), or peritrichous (flagella
distributed over the entire cell).
Spores (choice D) are specialized cell structures formed by certain types of
bacteria that allow for survival in extreme environments (e.g., freezing or
boiling conditions). They are not involved in the transfer of genetic material
and are not involved in the development of resistance against antibiotics.
Teichoic acids (choice E) are not involved in the development of drug
resistance. They are water-soluble polymers, containing ribitol or glycerol
residues joined through phosphodiester linkages and carrying one or more
amino acid or sugar substituents. They are seen primarily in gram-positive
cell walls.























The 6-year-old daughter of a businessman who travels internationally with his
family is taken to an emergency department because of a
paroxysm of high fever, shaking, chills, and sweats. The child is also
experiencing cough, fatigue, malaise, arthralgias, and myalgias. The child
has had several recent similar episodes, but the family waited to return to the
United States before seeking medical attention. An alert
hematology technician notes the intracellular forms seen in the image above
when reviewing the child's peripheral blood smear.

Question 1 of 8

Which of the following is the most likely diagnosis?
/ A. Babesiosis
/ B. Cysticercosis
/ C. Lyme disease
/ D. Malaria
/ E. Trichinellosis

Explanation - Q: 2.1 Close

The correct answer is D. The most likely diagnosis is malaria. Malaria is an
ancient disease that has been nearly eradicated in industrialized nations, but
remains a major cause of illness in the third world and may account for up to
10% of deaths in children in endemic areas. Many children in endemic areas
may develop a partial immunity, leading to repeated, frequent, low-level
infections. In the United States, imported cases of malaria are most
frequently seen. The clinical presentation illustrated is typical of a malarial
paroxysm, which usually begins with 1-2 hours of shivering and chills,
followed by development of a high fever. The patient's body temperature
then will usually drop to normal or below normal levels after he experiences
excessive diaphoresis. The fever and chills cycle will often then repeat at an
interval of 2 or 3 days. The parasites seen in the image are the ring forms
that are present in erythrocytes.
Babesiosis (choice A) is also due to an intraerythrocytic parasite, which can
have a similar appearance to the malarial parasite, but characteristically (and
helpfully, to the microscopist) occurs in tetrads.
Cysticercosis (choice B) is due to pork tapeworm larvae and causes cystic
spaces (particularly in the brain and subcutaneous tissues) which contain
recognizable scolices and hooklets.
Lyme disease (choice C) is caused by a spirochete, and causes multiorgan
system disease with prominent skin and joint manifestations.
Trichinellosis (choice E) is caused by a larval nematode with a predilection
for encysting in muscle.


Explanation - Q: 2.2 Close

The correct answer is C. Malaria can be caused by several malarial
parasites, including Plasmodium ovale, Plasmodium vivax, Plasmodium
malariae, and Plasmodium falciparum.
Babesia(choice A) cause babesiosis, which is a intraerythrocytic infection
that is usually milder than malaria.
Chlamydia(choice B) usually cause genitourinary and eye infections.
Rickettsia(choice D) cause the various forms of typhus and Rocky Mountain
spotted fever.
Toxoplasma(choice E) cause severe prenatal infection and chorioretinitis.





Explanation - Q: 2.3 Close

The correct answer is A. The Anopheles mosquito is the vector for malaria.
The parasites are present in the mosquito salivary glands, and are
introduced when saliva is injected into the mosquito bite.
Demodex(choice B) is a mite that lives in hair follicles, usually without
causing disease.
Ixodes(choice C) deer ticks carry Lyme disease and babesiosis.
Loxosceles(choice D) is the poisonous brown recluse spider.
Sarcoptes(choice E) is the mite that causes scabies.


Question 4 of 8
Within minutes of being introduced into the body, the infective form of the
organism enters cells in which of the following organs?
/ A. Brain
/ B. Kidney
/ C. Liver
/ D. Pancreas
/ E. Salivary gland




Explanation - Q: 2.4 Close

The correct answer is C. It is a surprise to most medical students that the
immediate target of the malarial parasites introduced by the mosquito is not
the erythrocyte. Instead, the infective form enters the hepatocytes to form
another asexual form called schizonts. Over one to three weeks, these
schizonts then mature and multiply in the liver (pre-erythrocytic or hepatic
schizont). Finally, the host cells burst, and release the parasites (then called
merozoites) into the blood. The other tissues listed in the choices are not
involved in this process.

Question 5 of 8
What is the form of the parasite that enters the human body?
/ A. Female gametocyte
/ B. Male gametocyte
/ C. Merozoite
/ D. Schizont
/ E. Sporozoite

Explanation - Q: 2.5 Close

The correct answer is E. The asexual forms known as sporozoites are
introduced into the bloodstream.
Schizonts (choice D) are the asexual form of the malarial parasite in liver
cells. Merozoites (choice C) are the asexual form of the parasite released
from the liver cells and picked up by erythrocytes. The merozoites then may
reproduce asexually within the erythrocytes for awhile, before a few go on to
develop sexually into male or female gametocytes (choices A and B). When
a second mosquito bites the patient, it will then pick up these sexual forms
and play host to the sexual phase of the parasite.




Explanation - Q: 2.6 Close

The correct answer is B. Different Plasmodium species cause somewhat
different clinical patterns. The different species can be distinguished
microscopically on examination of blood films containing the sexual forms of
the parasites. The fever paroxysms that characterize malaria are produced
when large numbers of erythrocytes, at roughly the same time, rupture and
release pyrogens. Plasmodium falciparum causes the most life-threatening
form of malaria. This form of malaria is the most common form in Africa, and
is also common world-wide. The incubation period is 5-12 days, and drug
resistant strains are becoming more common. A commercially prepared
dipstick that identifies Plasmodium falciparum (but not the other species) is
available.
Pseudomonas aeruginosa(choice A) is a gram-negative bacterium.
Plasmodium malariae(choice C) needs approximately 72 hours for each
erythrocyte cycle leading to hemolysis (hence the alternative name quartan
malaria), while the other malarial forms each have a 48-hour periodicity. P.
malariae is found worldwide, but is relatively uncommon.
P. ovale(choice D) is usually found in Africa. Its incubation period is 2 to 4
weeks, and it can cause infections that last more than a decade.
The most common form of malaria in Asia and the Americas is due to P.
vivax(choice E). P. vivax infection has an incubation period of 8-13 days,
and can cause occasional life-threatening splenic rupture.



Question 7 of 8

Which of the following is a feared complication of the most severe form of this
patient's disease?
/ A. BIackwater fever
/ B. IgA nephropathy
/ C. Kidney stones
/ D. Postinfectious glomerulonephritis
/ E. Pyelonephritis

Explanation - Q: 2.7 Close

The correct answer is A. Blackwater fever is due to intravascular hemolysis
with resulting hemoglobinemia, which leads to both hemoglobinuria and
hemozoinuria (hemoglobin altered by malarial parasites and excreted in
urine) with resulting darkly colored urine ("black water"). These substances
are toxic to the kidney, and up to 1/3 of adults who become infected with P.
falciparum develop secondary acute renal failure. The other renal conditions
listed are unrelated to malaria.



Question 8 of 8

The other organ which is at particular risk in the most severe form of this patient's
disease is which of the following?

/ A. Brain
/ B. Colon
/ C. Stomach
/ D. Thyroid
/ E. Testes

Explanation - Q: 2.8 Close

The correct answer is A. Cerebral malaria, due to P. falciparum, may cause
altered mental status, seizures (which may also be due to high fever alone),
and coma. It is the most common cause of death in malaria patients, with a
death rate, in individuals who develop the complication, of 15% in children
and 20% in adults. Other complications of P. falciparum infection can include
hemolysis, anemia, noncardiogenic pulmonary edema (particularly in
pregnant women), profound hypoglycemia (particularly in young children and
pregnant women), lactic acidosis, and coagulopathy. The organs listed in the
other choices are not particularly vulnerable during falciparum malaria.



A 42-year-old man is admitted to the hospital with a severe asthma attack
complicated by pneumonia. The patient's condition requires
emergent intubation after a respiratory and cardiac arrest. The patient has severe
hypotension and develops renal failure, requiring
hemodialysis. Multiple metabolic abnormalities are corrected, the patient receives
high dose steroids for several days, and requires
Iorazepam for sedation. One evening, the patient is noted to be "restless and
agitated, moving in bed, and attempting to pull out his tubes."
Hydroxyzine, and an hour later, trazodone, is given to "help with sleep." The
patient sleeps for an hour, then awakens, thrashing and pulling out
his endotracheal tube. At this point, he is believed to be delirious from the
steroids and given haloperidoI, benztropine, and lorazepam, and is
reintubated. The next day, the nurse notes the patient is "uncomfortable in bed"
and "stiff as a board" and "really out of it." The doctor is called
to emergently evaluate the patient, who is febrile at 40 C (104 F), and has
systolic blood pressures ranging from 80 to 175 mm Hg.







Question 1 of 5

Which of the following drugs is most likely responsible for this syndrome?
/ A. Benztropine
/ B. Haloperidol
/ C. Hydroxyzine
/ D. Lorazepam
/ E. Trazodone

Explanation - Q: 3.1 Close

The correct answer is B. Haloperidol is associated with neuroleptic
malignant syndrome (NMS), a rare, but potentially fatal side effect. If NMS is
on the differential, the antipsychotic medicine is stopped immediately and
other (more common) causes of fever, elevated muscle enzymes, and
altered mental status are investigated (especially infections and myocardial
infarction). Although, high potency neuroleptics such as haloperidol are more
frequently associated with NMS, all antipsychotic agents (typical and
atypical) can cause this syndrome.
Benztropine (choice A) is often given with haloperidol to premedicate for any
extrapyramidal side effects (especially dystonias). Haloperidol decreases the
effect of dopamine (the result is a relative imbalance between dopamine and
acetylcholine). Drugs that block acetylcholine (benztropine and hydroxyzine)
treat side effects caused by dopamine blockade.
Hydroxyzine (choice C) is an older medication with anticholinergic and
antihistaminergic side effects. Sometimes given for anxiety or insomnia
(because of its low risk profile compared to addictive drugs), it can cause
anticholinergic delirium in elderly patients and bothersome side effects in
younger patients (dry mouth, constipation). Like benztropine, hydroxyzine
could mitigate extrapyramidal side effects, but does not cause NMS.
Lorazepam (choice D) is a treatment for seizures and alcohol withdrawal. It
can cause confusion and delirium in elderly patients by its direct CNS effects.
Trazodone (choice E) is an atypical antidepressant medicine. It is
associated with orthostatic hypotension and priapism.


Question 2 of 5
Which of the following is the most likely diagnosis?
/ A. Akathisia
/ B. Dystonia
/ C. Neuroleptic malignant syndrome
/ D. Parkinsonism
/ E. Tardive dyskinesia

The correct answer is C. Neuroleptic-induced malignant syndrome includes
severe muscle rigidity, elevated temperature, and other related findings (changes
in level of consciousness ranging from confusion to coma, labile blood pressure,
and elevated creatine kinase) developing in association with the use of
neuroleptic medication. The neuroleptic medicine should be discontinued. Fever
and muscle rigidity are treated with antipyretics, amantadine, bromocriptine, and
dantrolene. Fluids are given to support the blood pressure and prevent renal
failure (from the myoglobin load placed on the kidneys). Fever and elevated
CPKs warrant a work-up for infection and myocardial infarction; both very
common occurrences in hospitalized medically ill patients. The one-year
estimated prevalence of NMS ranges from 0.02 to 2.4 percent.
Akathisia (choice A), caused by neuroleptics, is a subjective complaint of
restlessness, usually accompanied by fidgeting and pacing. The neuroleptic
medicine is sometimes discontinued. Beta blockers are used to treat akathisia.
Dystonia (choice B), an extrapyramidal side effect that can be caused by
neuroleptics, is a spasm of the muscles of the neck, head, limbs, or trunk. IV
benztropine or diphenhydramine is given if pharyngeal muscles are involved
(respiratory distress can be fatal).
Parkinsonism (choice D), which can be caused by neuroleptics, includes resting
tremor, cogwheeling rigidity, and akinesia or bradykinesia. It results from D2
blockade in the basal ganglia (causing in an imbalance of dopamine and
acetylcholine). Amantadine, benztropine, or diphenhydramine are used.
Tardive dyskinesia (choice E) is also caused by neuroleptics, and includes involuntary
rhythmic, choreiform movements of the jaw or extremities.

Question 3 of 5

This patient is started on dantrolene for muscle rigidity. This medication works via
which of the following mechanisms?
/ A. AIpha-1 receptor blockade
/ B. Dopaminergic agonist
/ C. Dopaminergic antagonist
/ D. Dissociates excitation-contraction coupling
/ E. Muscarinic receptor blockade

Explanation - Q: 3.3 Close

The correct answer is D. Dantrolene dissociates excitation-contraction
coupling in skeletal muscle. The prolonged muscle rigidity associated with
NMS can cause CPKs in the 10,000 -100,000 U/L range.
Alpha-1 receptor blockers (choice A) (prazosin, terazosin) are utilized in
management of hypertension and BPH. Several antipsychotic medicines and
some antidepressant medications have alpha -1 receptor blockade as an
unwanted side effect.
Dopaminergic agonists (choice B) (e.g., bromocriptine) are used in the
treatment of Parkinson disease, but amantadine (increases dopamine
release) and bromocriptine have been used in NMS as well.
Dopaminergic receptor blockers (choice C) (e.g., haloperidol, risperidone,
etc.) exert an antipsychotic effect by blockade of D2 receptors. These agents
would exacerbate this patient's symptoms.
Muscarinic receptor blockers (choice E) (benztropine, scopolamine,
atropine, oxybutynin) are used in the treatment of Parkinson disease, motion
sickness, and post-operative bladder spasms. Several antipsychotic and
antidepressant medications have anticholinergic side effects (dry mouth,
constipation).

Question 4 of 5

Which of the following drugs can also be used to treat this patient?
/ A. Benztropine
/ B. Bromocriptine
/ C. FIuphenazine
/ D. L-dopa
/ E. Risperidone


Explanation - Q: 3.4 Close

The correct answer is B. Bromocriptine is a dopamine agonist that can help
reverse the effects of haloperidol. Additional treatment should include
discontinuing all antipsychotic medications and supportive measures.
Patients should receive ventilatory and circulatory support if necessary,
cooling blankets and NSAIDs to reduce fever, IV fluids to correct volume
depletion and hypotension, and hydration and alkalinization of the urine to
prevent renal failure if rhabdomyolysis occurs.
Benztropine (choice A) is an antimuscarinic agent that can reverse some of
the extrapyramidal side effects of neuroleptics. However, it does not have a
role in the treatment of NMS.
Fluphenazine (choice C) is a typical antipsychotic, which could exacerbate
this patient's symptoms, and has no role in the treatment of NMS.
L-dopa (choice D) is a precursor to dopamine and could theoretically help a
patient with NMS. However, the treatment of NMS is not an indication for L-
dopa.
Risperidone (choice E) is an atypical antipsychotic. Atypical antipsychotics
can also cause NMS, and are not used in the treatment of this syndrome.

Question 5 of 5

The patient later develops severe nausea and vomiting related to cholelithiasis,
and develops an acute muscle spasm in his face and neck
when treated with an antiemetic medication. Which of the following is associated
with this symptom?
/ A. Dolesetron
/ B. Dronabinol
/ C. Hydroxyzine
/ D. Ondansetron
/ E. Prochlorperazine

Explanation - Q: 3.5 Close

The correct answer is E. This patient is having a dystonic reaction, resulting
from neuroleptic drug therapy. Prochlorperazine and inapsine belong to the
phenothiazine class of neuroleptic medications, and are used to treat nausea
or psychosis. Dystonia, an extrapyramidal side effect, is characterized by
intermittent spasmodic, or sustained involuntary contractions of muscles in
the face, neck, extremities, trunk, and pelvis. These reactions are rarely life-
threatening, but patients find them uncomfortable and distressing.
Anticholinergics are the treatment of choice; benzodiazepines may also be
helpful.
Dolesetron and ondansetron (choices A and D) are antiemetics that
selectively antagonize 5HT3 receptors.
Dronabinol (choice B) is a cannabinoid with antiemetic and appetite-
stimulating properties.
Hydroxyzine (choice C) is an anticholinergic/antihistaminergic with
antiemetic properties.

A 25-year old man complains to his physician of vague gastric discomfort that
has been troubling him daily for several years. His symptoms
are never very severe, but seem to be somewhat relieved by eating food.

Question 1 of 5

The patient's physician orders a screening test, which is later reported as
positive, for antibodies directed against an organism known to
cause chronic gastritis. Which of the following organisms was most likely
identified?


Explanation - Q: 1.1 Close

The correct answer is C. Helicobacter pylori has been increasingly
implicated as the primary cause of nonerosive gastritis, and has also been
implicated in peptic ulcer disease, gastric carcinoma, and gastric lymphoma.
Helicobacter infection appears to be often acquired in childhood, and may
persist for decades. Formerly, a gastric biopsy was always required to make
the diagnosis. Now, however, an early, noninvasive step in diagnosis is to
send serum to detect IgA and IgG antibodies directed against Helicobacter
pylori. As in other antibody-based tests, the absence of the antibodies can be
helpful in excluding infection, but the presence of antibodies may be due to
either past or present Helicobacter infection.
Entamoeba histolytica(choice A) is a cause of amebic dysentery.
Escherichia coli(choice B) is a normal gut constituent that may also cause
gastroenteritis.
Pseudomonas aeruginosa(choice D) most commonly causes a severe form
of pneumonia.
Salmonella typhi(choice E) is the causative organism of typhoid fever.


Question 2 of 5
The patient undergoes esophagogastroduodenoscopy for further evaluation of
his symptoms. Upon entering the stomach, the endoscopist
sees that the stomach has multiple coarse folds. These are called which of the
following?
/ A. Ampulla
/ B. Haustra
/ C. PIicae
/ D. Rugae
/ E. Villi

Explanation - Q: 1.2 Close

The correct answer is D. The folds of the stomach are known as rugae.
Ampulla (choice A) means "entrance." For example, the ampulla of Vater, in
the duodenum, is a small papillary structure that contains the opening of the
common bile duct and pancreatic duct.
Haustra (choice B) refers to the large saccules or pouches seen in the large
bowel.
Plicae (choice C) are the transverse folds seen in the small intestine.
Villi (choice E) are mucosal projections seen in the small intestine.

Question 3 of 5

The stomach is found to be diffusely erythematous, with the worst areas being in
the antrum. No focal lesions are noted. Endoscopic biopsy
taken from the antral stomach demonstrates the organisms to which the patient
has made antibodies. Where would these organisms be most
Iikely found in the histological specimen?
/ A. Between fibrocytes in the submucosa
/ B. Between the epithelial cells of the mucosal lining
/ C. In endothelial cells
/ D. In the mucus layer above the mucosa
/ E. Within macrophage cytoplasm in the submucosa

Explanation - Q: 1.3 Close

The correct answer is D. Often, the Helicobacter organisms occur in the
greatest numbers in the antral stomach, therefore this is a good site for
biopsy. Helicobacter pylori organisms are confined to the mucus layer found
on the surface of the mucosa and within the lumina of gastric glands. They
cause disease by using the mucus layer as a food source and causing focal
defects; you can remember this loosely as "they eat holes in the mucus."
This then exposes the cells of the gastric epithelium to the acid and enzymes
of the stomach, predisposing for gastritis and peptic ulcer disease. While
neutrophils, lymphocytes, and plasma cells are often seen in the gastric
mucosa and submucosa in infected patients, the Helicobacter organisms do
not actually invade the mucosa, and thus cannot be found in any of the other
sites noted in the answer options.

Question 4 of 5

Following the report of the biopsy studies, antibiotic therapy is begun. Which of
the following regimens would be most likely used to eradicate
the organism?
/ A. Bismuth subsalicylate and tetracycline
/ B. CIarithromycin alone
/ C. Metronidazole alone
/ D. Ranitidine bismuth citrate and clarithromycin
/ E. Tetracycline and metronidazole

Explanation - Q: 1.4 Close

The correct answer is D. Helicobacter pylori can be difficult to eradicate,
and therapy usually requires multiple agents given for 2 or more weeks. One
of the initial effective regimens, which is still commonly used, included
bismuth subsalicylate (Pepto-Bismol

), and the antibiotics tetracycline and


metronidazole (choices B, C, and E). More recently, the combination or
ranitidine bismuth citrate (which contains a complex of the H2 antagonist
ranitidine, bismuth, and citrate) and the antibiotic clarithromycin has also
been found to be effective, and is being increasingly used (often because
patients would rather take pills than frequently drink Pepto-Bismol).

Question 5 of 5

The patient's physician also considered using a therapeutic regime that includes
a proton pump inhibitor. Which of the following would he most
Iikely prescribe?
/ A. Filgrastim
/ B. FIumazenil
/ C. Methimazole
/ D. Omeprazole
/ E. Tamoxifen

Explanation - Q: 1.5 Close

The correct answer is D. Regimens for treating chronic gastritis and peptic
ulcer disease that incorporate a proton pump inhibitor, typically either
omeprazole or lansoprazole as part of a triple drug therapy, are also in
development, and potentially offer a shorter duration of therapy of only 1
week.
Filgastrim (choice A) is a cytokine also known as G-CSF that induces
granulocytic proliferation and is used for marrow recovery.
Flumazenil (choice B) is used as an antidote for benzodiazepine poisoning.
Methimazole (choice C) is an antithyroid drug.
Tamoxifen (choice E) is an estrogen receptor antagonist used in breast
cancer therapy.

A 35-year-old man consults a physician because of several years of chronic
gastric discomfort that is partially relieved by the ingestion of food
and antacids. The patient is treated with antacids and dietary modification, but
his symptoms fail to significantly improve over the next several
months. Esophagogastroduodenoscopy is then performed, which demonstrates
the presence of multiple ulcers in the stomach and duodenum.
These ulcers average 1.5 cm diameter and have a "punched out" appearance
with sharply defined borders without raised edges.

Question 1 of 6

This patient's ulcers are most likely which of the following types?
/ A. Aphthous ulcers
/ B. Curling ulcers
/ C. Gastric cancers
/ D. Hunner ulcers
/ E. Peptic ulcers

Explanation - Q: 2.1 Close

The correct answer is E. Peptic ulcers characteristically have the
appearance described in the question, and this patient's history is compatible
with peptic ulcer disease.
Aphthous ulcers (choice A) are usually tiny, punctate ulcers that can be
seen in chronic erosive gastritis and may be idiopathic, or caused by drugs
(notably aspirin and NSAIDS), Crohn disease, or viral infection.
Curling ulcers (choice B) are stress ulcers, often of the duodenum, seen in
patients with extensive superficial burns, intracranial lesions, or severe bodily
injury.
Gastric cancers (choice C) are usually described as having "heaped up"
borders, but an overlap in appearance with peptic ulcers can occur.
However, it would be unlikely for this relatively young man to have multiple
gastric carcinomas.
Hunner ulcers (choice D) are found in the urinary bladder.


Question 2 of 6

The patient's basal gastric acid secretory rate is measured, and is found to be
abnormally high (80% of the amount of acid secreted after
maximum stimulating dose of histamine). Which of the following cell types is
responsible for this hypersecretion of acid in the stomach?
/ A. Chief cells
/ B. Lymphocytes
/ C. Mucous cells
/ D. Neuroendocrine cells
/ E. Parietal cells
Explanation - Q: 2.2 Close

The correct answer is E. Parietal cells secrete the hydrochloric acid that
acidifies the stomach. To do so, they use a hydrogen-potassium-ATPase that
exchanges H
+
for K
+
across the luminal side of the cell membrane. Some
cases of peptic ulcer disease are related to excessive acid production (as in
this case), while others are related to damage to the mucosal barrier lining
the stomach, often by the Helicobacter pylori organism.
Chief cells (choice A) secrete proteolytic enzymes, including pepsinogen I
and II.
Lymphocytes (choice B) are present in the stomach, but do not participate
directly in digestion.
Mucous cells (choice C) secrete the mucus that protects the gastric
epithelium.
Neuroendocrine cells (choice D) are present in the gastric epithelium and
elsewhere in the gastrointestinal tract, and may secrete a variety of
hormones.

Question 3 of 6
Abnormally high serum levels of which of the following would most likely account
for the patient's multiple ulcers?
/ A. Cholecystokinin
/ B. Gastrin
/ C. GIucagon
/ D. Motilin
/ E. Secretin


Explanation - Q: 2.3 Close

The correct answer is B. Gastrin is normally secreted by the
neuroendocrine cells of the stomach in response to the presence of partially
digested protein in the stomach. Gastrin stimulates the secretion of
hydrochloric acid and pepsinogen.
Cholecystokinin (choice A) is secreted by the neuroendocrine cells of the
small intestine and stimulates the gallbladder to eject bile and the pancreas
to secrete enzymes and alkaline fluid.
Glucagon (choice C) is secreted by the pancreatic islet cells and has actions
opposing those of insulin, tending to raise blood glucose levels.
Motilin (choice D) is secreted by the neuroendocrine cells of the small
intestine and increases gastrointestinal motility.
Secretin (choice E) is secreted by the neuroendocrine cells of the small
intestine and stimulates the pancreas to secrete pancreatic juices and the
liver to secrete bile.


Question 4 of 6
Which of the following is the most likely diagnosis?
/ A. Adams-Stokes syndrome
/ B. Beckwith-Wiedemann syndrome
/ C. Crigler-Najjar syndrome
/ D. Mallory-Weiss syndrome
/ E. Zollinger-EIIison syndrome

Explanation - Q: 2.4 Close

The correct answer is E. This patient with multiple peptic ulcers and
elevated serum gastrin levels most likely has Zollinger-Ellison syndrome.
This condition is due to gastrin-secreting tumors, usually found either in the
pancreas or the duodenum. The tumors are usually small, and may be hard
to find. Endoscopic ultrasound may identify tumors that were not detected
with other methods such as CT scan or conventional ultrasound. Multiple
tumors are common and the tumors are malignant 50% of the time. The
peptic ulcers in Zollinger-Ellison syndrome tend to behave more aggressively
than those in other disorders, and are often multiple or in atypical locations.
Ideally, the tumor is resected; if this is not possible, the H
+
/K
+
ATPase
inhibitor omeprazole can markedly reduce parietal cell acid secretion. Total
gastrectomy is reserved for intractable cases.
Adams-Stokes syndrome (choice A) is characterized by a slow or absent
pulse with syncope and convulsions seen as a complication of advanced AV
block or sick sinus syndrome.
Beckwith-Wiedemann syndrome (choice B) is an autosomal recessive
condition characterized by macroglossia and gigantism.
Crigler-Najjar syndrome (choice C) is a hereditary liver disease, that, in its
severe form, is fatal in infancy.
Mallory-Weiss syndrome (choice D) is an esophageal laceration with
bleeding, caused by severe retching and vomiting.



Question 5 of 6

Individuals with this patient's condition also have an increased incidence of which
of the following?
/ A. Marfanoid habitus
/ B. Medullary carcinoma of the thyroid
/ C. Mucosal neuromas
/ D. Parathyroid adenoma
/ E. Pheochromocytoma

Explanation - Q: 2.5 Close

The correct answer is D. Zollinger-Ellison syndrome, particularly if caused
by multiple gastrin-producing neuroendocrine tumors, is frequently a
component of multiple endocrine neoplasia, Type I (MEN I). MEN I is a
familial disease linked to a tumor-suppressor gene on chromosome 11,
which is characterized by parathyroid adenomas (more than 90% of
individuals), pancreatic islet cell tumors (30-75%), and pituitary adenomas
(50-65%). The pancreatic islet cell tumors often secrete gastrin, thereby
causing the Zollinger-Ellison syndrome. The gastrin-secreting tumors and the
pituitary adenomas are not part of MEN types IIA and IIB.
Marfanoid habitus (choice A) and mucosal neuromas (choice C) are
components of MEN IIB.
Medullary carcinoma of the thyroid (choice B) and pheochromocytoma
(choice E) are components of both MEN IIA and MEN IIB.


Question 6 of 6
Which of the following would be the most appropriate pharmacotherapy?
/ A. Erythropoietin
/ B. Famotidine
/ C. FIuorouracil
/ D. Omeprazole
/ E. Sucralfate

Explanation - Q: 2.6 Close

The correct answer is D. The patient in this question was diagnosed with
Zollinger-Ellison syndrome. One of the most important initial measures in the
treatment of this condition is to control the acid hypersecretion. Proton pump
inhibitors, such as omeprazole, rabeprazole, pantoprazole, and lansoprazole,
are given at a dose of 40-120 mg/day, and titrated to achieve a basal acid
output of <10 mEq/hr. At this level, there is complete symptomatic relief and
ulcer healing.
Erythropoietin (choice A) is a glycoprotein, normally produced in the
kidneys, that stimulates stem cells to differentiate into rubriblasts, increases
the rate of mitosis, increases the release of reticulocytes from the bone
marrow and increases hemoglobin (Hgb) formation. EPO is indicated for the
treatment of anemia associated with chronic renal failure, zidovudine
therapy, chemotherapy, and for reduction of allogenic blood transfusions in
surgery patients.
Famotidine (choice B) is an H2-antagonist that decreases gastric acid
secretion by competitively and reversibly inhibiting histamine receptors on
parietal cells. Although famotidine could potentially be used in the treatment
of Zollinger-Ellison syndrome, a proton pump inhibitor is a more appropriate
treatment measure.
Fluorouracil (choice C) or 5-FU, is an example of an antimetabolite (choice
B) that blocks the methylation reaction of deoxyuridylic acid to thymidylic
acid. This results in interference with DNA synthesis, and to a lesser extent,
RNA synthesis. 5-FU is indicated for the palliative management of colon
rectum, breast, stomach and pancreatic carcinoma.
Sucralfate (choice E) is a complex of aluminum hydroxide and sulfated
sucrose that reacts with gastric acid to form a viscous, adhesive paste-like
substance that is resistant to further reaction with acid. It is indicated for the
short-term treatment of duodenal ulceration, and has been used for the
short-term treatment of gastric ulcers. Sucralfate alone would not be
appropriate therapy in a patient with Zollinger-Ellison syndrome.


A patient complains of early satiety and chronic epigastric pain to his physician.
He has also noticed that he has lost 20 pounds over the past
few months. On physical examination, he is noted to have an enlarged Virchow's
node. CT scan of the abdomen is performed, showing a
Iarge, fungating soft tissue mass protruding off of the lesser curvature of the
stomach. The gastric cardia and proximal fundus are narrowed by
the mass.

Question 1 of 6

Most of the blood reaching the affected organ in a normal individual is derived
from which of the following vessels?
/ A. Celiac artery
/ B. Inferior mesenteric artery
/ C. Left hepatic artery
/ D. Right hepatic artery
/ E. Superior mesenteric artery

Explanation - Q: 3.1 Close

The correct answer is A. The vascular supply of the stomach is important,
in part because the lymphatic drainage tends to follow the blood vessels. The
stomach has a complex vascular supply with many anastomotic channels.
However, most of the blood that reaches the stomach travels through the
celiac artery (also called the celiac trunk) and its various branches. The
celiac artery arises from the aorta, and then promptly divides into its three
terminal branches: the common hepatic artery, the splenic artery, and the left
gastric artery. The left gastric artery supplies the lesser curvature. The
hepatic artery gives rises to the gastroduodenal artery, from which the right
gastroepiploic artery (which supplies the pyloric region) arises. It then divides
into the left hepatic artery (choice C) and right hepatic artery (choice D),
neither of which supply the stomach. The third branch of the celiac, the
splenic artery, gives rise to the short gastric arteries (which supply the cardia)
and the left gastroepiploic arteries (which supply the greater curvature).
The superior mesenteric artery system (choice E) does communicate with
the celiac system through the pancreaticoduodenal vessels, but is not the
major source of the blood supply.
The inferior mesenteric artery (choice B) system does not have direct
anastomoses with the celiac system.





Question 2 of 6
Biopsy would most likely show which of the following?
/ A. Adenocarcinoma
/ B. Fibrosarcoma
/ C. Leiomyosarcoma
/ D. Lymphoma
/ E. Squamous cell carcinoma

Explanation - Q: 3.2 Close

The correct answer is A. Worldwide, gastric carcinoma is the second most
common cause of cancer-related deaths (primarily in Asia), and the
fourteenth most common cancer in the United States. Between 90 and 95%
of all gastric malignancies are adenocarcinomas. Gastric cancer is often
initially asymptomatic, but with advanced disease, a variety of signs and
symptoms may occur, including early satiety, hematemesis, epigastric pain,
weight loss, and anemia.
Lymphomas (choice D) are the second most common malignancy;
leiomyosarcomas (choice C) and squamous cell carcinomas (choice E) also
occur.
Fibrosarcoma (choice B) is rare in the stomach.




Question 3 of 6

Which of the following nodes is a Virchow node.
/ A. Left supraclavicular
/ B. Perirectal pouch
/ C. Right axilla
/ D. Right supraclavicular
/ E. Umbilical

Explanation - Q: 3.3 Close

The correct answer is A. Virchow's node refers to the single lymph node
enlargement of the left supraclavicular node. This node is difficult to localize
due to its deep location. The best way to find it is by asking the patient to
perform a Valsalva maneuver and palpate behind the anterior head of the
sternocleidomastoid. Its clinical significance is that it may indicate the
presence of metastatic disease from the abdomen, most commonly from the
stomach.
Blumer's shelf is the term used for metastatic cancer from the abdomen
(often stomach) in the perirectal pouch (choice B).
There are no Virchow nodes in the axilla (choice C).
Involvement of the right supraclavicular node (choice D) is not considered to
be a Virchow node.
Metastases to the area near the umbilicus (choice E) from intra-abdominal
cancer (often stomach) can produce Sister Mary Joseph's node.



Explanation - Q: 3.4 Close

The correct answer is C. Helicobacter pylori has been implicated as a
contributing agent in gastric carcinoma. The link is that Helicobacter pylori
predisposes for chronic gastritis, which in turn has a 6-fold increased risk for
gastric carcinoma. Helicobacter pylori is also associated with an increased
risk of peptic ulcer disease and gastric lymphoma (because of the state of
chronic inflammation seen). Other risk factors for gastric cancer include
Asian-type diets (with high intakes of pickled vegetables, salted fish,
excessive dietary salt, and smoked meats), previous gastric surgery, and
genetic factors. The other agents listed in the choices cause gastroenteritis,
but not an increased risk of gastric carcinoma.



Explanation - Q: 3.5 Close

The correct answer is A. The lesser curvature of the antropyloric region is
the most common site for gastric carcinoma. About 50% of gastric
carcinomas are found in the antropyloric region, about 25% are found in the
cardia, and the remaining 25% in the body and fundus of the organ. The
lesser curvature is affected more frequently than the greater curvature, but
an ulcerating lesion on the greater curvature is more likely malignant.

Question 6 of 6

If, on further evaluation, this patient is found to have direct tumor invasion into
the pancreas, two hepatic metastases, and involvement of 10
regional lymph nodes. It should be staged as which of the following?
/ A. T1, N1, M2
/ B. T1, N2, M2
/ C. T2, N0, M0
/ D. T3, N3, M0
/ E. T4, N2, M1

Explanation - Q: 3.6 Close

The correct answer is E. Staging of gastric carcinoma follows the TNM
system, as follows. TX - primary tumor cannot be assessed; T0 - no evidence
of primary tumor; Tis - carcinoma in situ; T1 - tumor invades lamina propria
or submucosa; T2 - tumor invades muscularis propria or subserosa; T3-
tumor penetrates serosa without invasion of adjacent structures; T4 - tumor
invades adjacent structures. NX- regional lymph nodes cannot be assessed;
N0 - no regional lymph node metastases; N1 - metastases in 1-6 regional
lymph nodes; N2 - metastases in 7-15 regional lymph nodes; N3 -
metastases in more than 15 regional lymph nodes. MX - distant metastases
cannot be assessed; M0 - no distant metastases; M1 - distant metastases.
Unfortunately, most gastric carcinomas in the United States are at a high
stage at the time of diagnosis, and most of the patients who undergo gastric
resection later have a recurrence of their disease and die. (The problem is
that the stomach is thin and is close, by both lymphatic spread and direct
extension, to many critical structures that cannot be safely and cleanly
resected.) However, with aggressive management with en bloc resection
combined with chemotherapy and/or radiation therapy, the 4-year survival
rate of these patients has been increasing. Aggressive gastric cancer
screening in Japan (which has a high incidence of gastric cancer) with
gastroduodenoscopy with biopsies has identified a larger percentage of early
carcinomas, which are more responsive to treatment.

A 60-year-old man comes to the emergency department complaining of bright
red blood per rectum. The bleeding began abruptly several
hours prior to his visit. He has light-headedness when he stands up rapidly, but
has no abdominal pain, cramping, fever, nausea, or vomiting.
He has no history of previous episodes of bleeding or abdominal pain, but has a
history of coronary artery disease and takes aspirin as a
"blood thinner." He is afebrile, slightly hypotensive and tachycardic, but stable.
On examination, he has decreased skin turgor, and dry mucous
membranes. He has no abdominal tenderness. Rectal examination is positive for
gross blood.
Question 1 of 5
Which of the following is the most likely diagnosis?
/ A. Arteriovenous malformation
/ B. Diverticulitis
/ C. Infectious colitis
/ D. Ischemic colitis
/ E. UIcerative colitis

Explanation - Q: 1.1 Close

The correct answer is A. Painless hematochezia or bright red lower GI
bleeding can come from many sources. While bright red lower GI bleeding
tends to indicate lower GI bleeding (bleeding distal to the ligament of Treitz),
brisk upper GI bleeding can also be the source. The clinical manifestations of
such bleeding range from negligible to hemodynamic instability, depending
upon the rate of bleeding. The differential diagnosis for painless
hematochezia includes AV malformations, gastric erosions, esophageal
varices, esophagitis, duodenal or gastric ulcer, hemorrhoids, diverticulosis,
and colonic neoplasm.
Diverticulitis (choice B) occurs when a colonic outpouching or diverticulum
becomes inflamed. Patients tend to be elderly and present with fever,
abdominal pain, and abdominal tenderness on examination. While painful,
these lesions do not bleed significantly (unlike their uninflamed counterparts
in diverticulosis).
Infectious colitis (choice C) may present as rectal bleeding, but this bleeding
is typically accompanied by pain, cramping, and fever. Causative organisms
may include Salmonella, Shigella, Campylobacter jejuni, E. coli, and
Entamoeba histolytica.
Ischemic colitis (choice D) may have rectal bleeding, but the hallmark of
ischemic colitis is severe abdominal pain out of proportion to examination
findings.
Ulcerative colitis (choice E) presents as abdominal pain and diarrhea, which
may be bloody or nonbloody. In addition, the onset of the disease tends to be
earlier, so this patient would likely have had previous episodes of pain.


Question 2 of 5
After the patient has stabilized, a colonoscopy is performed to elucidate the origin
of the bleeding. Several star-shaped branching vessels
measuring 0.2 to 1.0 cm are seen in the colonic submucosa. BIeeding is stopped
by electrocoagulation. A diagnosis of lower gastrointestinal
bleeding is given. Which anatomic landmark demarcates upper gastrointestinal
bleeding from lower gastrointestinal bleeding?
/ A. IIeocecal valve
/ B. Ligament of Treitz
/ C. Papilla of Vater
/ D. Pylorus
/ E. Splenic flexure of the colon

Explanation - Q: 1.2 Close

The correct answer is B. The ligament of Treitz, or the peritoneal ligament,
which separates the third (retroperitoneal) portion of the duodenum from the
fourth (peritoneal) portion of the duodenum, traditionally demarcates upper
GI bleeding from lower GI bleeding. Bleeding proximal to this landmark tends
to produce melena or black tarry stools. Bleeding distal to this landmark
tends to produce hematochezia or red blood per rectum.
The ileocecal valve (choice A) separates the terminal ileum from the cecum.
The papilla of Vater (choice C) is where the pancreatic duct and common
bile duct empty into the duodenum.
The pylorus (choice D) is the sphincter separating the stomach from the
duodenum.
The splenic flexure of the colon (choice E) marks the transition from
transverse colon to the descending colon.

Question 3 of 5
The aspirin taken by this patient represents a contributor to his condition. Which
of the following best describes the mechanism of action of
aspirin?
/ A. Aspirin decreases the serum level of factor VIII
/ B. Aspirin decreases the serum level of factor IX
/ C. Aspirin irreversibly inhibits platelets
/ D. Aspirin irreversibly inhibits thrombin
/ E. Aspirin reversibly inhibits platelets
/ F. Aspirin reversibly inhibits thrombin

Explanation - Q: 1.3 Close

The correct answer is C. Patients with gastrointestinal bleeding must be
assessed for anatomic as well as physiologic and pharmacologic sources of
bleeding. Aspirin acts as an anticoagulant by irreversibly inhibiting platelets,
preventing the formation of a clot by blocking platelet adhesion and
aggregation. Since this platelet mass acts as a matrix for fibrin clot formation,
blocking platelets prevents clot formation. This mechanism has been utilized
in patients with atherosclerotic disease to prevent intravascular clot
formation, but may aggravate bleeding conditions such as this.
Aspirin does not decrease the serum level of factor VIII (choice A). Factor
VIII deficiency is the pathophysiology behind hemophilia A.
Factor IX deficiency (choice B) is associated with hemophilia.
Aspirin does not inhibit thrombin (choices D and F). Thrombin is the enzyme
responsible for cleaving fibrinogen to fibrin.
Aspirin's effects on platelets are not reversible (choice E), and a new
population of functional platelets must replace the inhibited platelets before
coagulation is fully restored.

Question 4 of 5
Which of the following is an important mechanism in short-term blood pressure
maintenance?
/ A. BIood pressure regulation occurs slowly by endocrine mechanisms only
/ B. Decreased stretch in the carotid bodies decreases sympathetic and
increases parasympathetic discharge to the heart
/ C. Decreased stretch in the carotid bodies increases sympathetic and
decreases parasympathetic discharge to the heart
/ D. Decreased stretch in the carotid sinus decreases sympathetic and
increases parasympathetic discharge to the heart
/ E. Decreased stretch in the carotid sinus increases sympathetic and decreases
parasympathetic discharge to the heart

Explanation - Q: 1.4 Close

The correct answer is E. As blood pressure falls in this patient with
hypovolemia, many short term and long term mechanisms work to raise the
falling pressure. In the short term, the baroreceptors found in the carotid
sinus and aortic arch regulate blood pressure by modulating the autonomic
nervous system. As pressure falls in this patient, the baroreceptors sense
this change as a decrease in stretch in the vessel walls. Afferent fibers from
the baroreceptors then "report" this change to the medullary cardiovascular
center. This center responds by increasing sympathetic discharge and
decreasing parasympathetic discharge to the heart and resistance vessels.
This acts to restore the blood pressure by increasing heart rate, stroke
volume, and vascular resistance.
While endocrine mechanisms (choice A) restore mean arterial pressure for
the long term, the sympathetic mechanisms outlined above restore pressure
toward baseline much more rapidly.
Choices B and C are incorrect. The carotid bodies contain chemoreceptors
(not stretch receptors) that detect changes in PO
2
, PCO
2
, and pH. They
restore these parameters to normal by acting through the medullary centers
to change heart rate, stroke volume, vascular resistance, and ventilatory
parameters.
The decrease in pressure triggers an increase in sympathetic discharge and
decrease in parasympathetic discharge (compare with choice D).



Question 5 of 5
Normal saline is administered to this patient and his blood pressure and heart
rate normalize. One of the goals in fluid resuscitation is to
optimize cardiac parameters according to Starling's Law. Starling's Law
describes which of the following?
/ A. The relationship between end diastolic volume and contractility
/ B. The relationship between heart rate and stroke volume
/ C. The relationship between preload and afterload
/ D. The relationship between stroke volume and end systolic volume
/ E. The relationship between systemic vascular resistance and cardiac output

Explanation - Q: 1.5 Close

The correct answer is A. Starling's law of the heart describes the
relationship between end diastolic volume or preload and cardiac
contractility. It states that cardiac contractility is maximized at a particular
preload. It also states that cardiac contractility declines as the preload is
increased or decreased from this optimum. The basis for this principle is that
at a particular preload, the myocardium is "stretched" to a point that
maximizes the number of actin and myosin units that may interact in a given
contraction.
Choice B is incorrect. Heart rate x stroke volume = cardiac output
Choice C is incorrect. Preload is related to end diastolic volume and
passive wall tension exerted on the diastolic ventricle.
Choice D is incorrect. End diastolic volume - end systolic volume = stroke
volume
Choice E is incorrect. Mean arterial pressure = cardiac output x total
peripheral resistance.




A 9-day-old baby is noted to be lethargic and has been feeding poorly. Over the
next day, the baby develops bilious vomiting, a distended
tender abdomen, and bloody stools.

Question 1 of 5
Which of the following diseases would most likely cause gastrointestinal bleeding
in a neonate?
/ A. Crohn disease
/ B. Cystic fibrosis
/ C. Diverticulitis
/ D. Necrotizing enterocolitis
/ E. UIcerative colitis

Explanation - Q: 2.1 Close

The correct answer is D. Necrotizing enterocolitis is a feared complication
of infancy. It has an incidence of 1 to 5% in neonatal intensive care unit
admissions. The condition is a necrotizing disease of the small intestine, and
sometimes, the colon. The pathogenesis is still not clear, but may involve an
ischemic insult leaving the bowel susceptible to bacterial overgrowth.
Necrotizing enterocolitis may develop suddenly, with features suggesting
neonatal sepsis, or more slowly, over a period of one or two days. The case
description illustrates typical features.
Crohn disease (choice A) and ulcerative colitis (choice E) may present as
early as in the teenage years, but not usually in infancy.
Cystic fibrosis (choice B) is a cause of meconium ileus and later
malabsorption, but does not typically present with gastrointestinal bleeding.
Diverticulitis (choice C) is usually a disease of middle-aged to older adults.




Question 2 of 5
Which of the following is considered the most important risk factor for this
patient's disease?
/ A. Perinatal asphyxia
/ B. Polycythemia
/ C. Prematurity
/ D. Respiratory distress syndrome
/ E. Shock

Explanation - Q: 2.2 Close

The correct answer is C. Prematurity is the most important risk factor for
necrotizing enterocolitis, although term infants also sometimes develop the
condition. Clinical series have reported that between 60 and 95% of affected
babies are premature, and the incidence is markedly increased in babies
born at lower gestational ages.
Many other purported risk factors have also been cited but seem to have a
lesser effect, including perinatal asphyxia (choice A), respiratory distress
syndrome (choice D), umbilical catheterization, hypothermia, shock (choice
E), patent ductus arteriosus, cyanotic congenital heart disease, polycythemia
(choice B), thrombocytosis, anemia, exchange transfusion, congenital GI
anomalies, chronic diarrhea, non-breast milk formula, nasojejunal feedings,
hypertonic formula, and colonization with necrogenic bacteria. It may simply
be that any already fragile baby, particularly if premature, who has other
significant underlying disease, is at increased risk for developing necrotizing
enterocolitis.


Question 3 of 5
A plain radiograph of the abdomen demonstrates gas within the bowel walI
(pneumatosis). Which of the following would most likely be
associated with this finding?
/ A. Air in the biliary tract
/ B. BIood in the biliary tract
/ C. Gas in the hepatic veins
/ D. Gas in the mediastinum
/ E. Gas in the portal vein

Explanation - Q: 2.3 Close

The correct answer is E. Portal venous gas is seen in association with
pneumatosis intestinalis, most commonly with necrotizing enterocolitis. The
physiology of this is that the portal vein, via the mesenteric veins, drains
nutrient-rich blood from the gut to the liver. In the case of necrosis with air in
the bowel wall, air migrates into the portal venous system and to the liver. On
CT, this has the characteristic appearance of peripheral lucencies following
the portal venous system intrahepatically. In cases of more severe
pneumatosis, the bowel may rupture and lead to pneumoperitoneum.
Note: Although this item may have seemed difficult, it was, in essence, a
straightforward pathophysiology question, i.e., "Where would gas in the wall
of the intestine go?" The distracter explanations give additional information
concerning the radiographic appearance of the other conditions (the
following will most likely NOT be tested on Step I of the USMLE).
Pneumobilia, or air in the biliary tract (choice A), would be seen after
instrumentation of the biliary system, such as after an endoscopic retrograde
cholangiopancreatogram (ERCP). Other causes include a gas-forming
infection within the biliary tree or previous sphincterotomy (endoscopic
opening of the sphincter of Oddi). Pneumobilia has a distinct appearance on
CT: there is gas located centrally in the liver within the ducts.
Hemobilia, or blood in the biliary tract (choice B), would be seen after
instrumentation of the biliary system, such as after an endoscopic retrograde
cholangiopancreatogram (ERCP), from a biliary or hepatic tumor, or
secondary to a hypocoagulable state. Hemobilia is found at endoscopy, and
is generally not visible on plain radiographs. High attenuation material may
be seen within the bile ducts on a CT scan, suggesting hemobilia.
Hepatic venous gas (choice C) would not be seen with pneumatosis
because the hepatic veins drain the liver into the inferior vena cava (IVC).
Gas from the bowel wall gets trapped in the portal veins and does not
traverse the liver to get into the hepatic veins.
Pneumomediastinum (choice D) is usually from thoracic trauma causing
rupture of the esophagus or pneumothorax. Gas within the soft tissues of the
head and neck may dissect to the mediastinum. Rarely, pneumoperitoneum
may lead to secondary pneumomediastinum. Pneumatosis without
pneumoperitoneum would not lead to pneumomediastinum.


Question 4 of 5
The baby's condition continues to deteriorate, and the decision is made to
surgically resect the affected GI segment. Resection of which of the
following areas of the gastrointestinal tract would most likely produce severe
long-term malabsorption?
/ A. Ascending colon
/ B. Duodenum
/ C. Jejunum
/ D. Stomach
/ E. Terminal ileum
Explanation - Q: 2.4 Close

The correct answer is E. Babies with early necrotizing enterocolitis are
sometimes successfully managed medically with fluids, bowel rest, and
correction of anemia and thrombocytopenia that may have developed
secondary to the gastrointestinal bleeding. Surgical resection may be
necessary in more severe cases of necrotizing enterocolitis, but may have a
mortality of 30-40% in these deathly ill infants. Unfortunately, necrotizing
enterocolitis most often affects the terminal ileum, which is also the site most
necessary to prevent long-term malnutrition. In practice, more than 50% of
the bowel must usually be removed before substantial malabsorption occurs.
The ileum is the site that is most active in nutrient (particularly fats)
absorption, vitamin B12 absorption, and conjugated bile salt absorption.
The ascending colon (choice A) is good at absorbing water and has a small
capacity for absorbing carbohydrates.
The duodenum (choice B) and jejunum (choice C) are also important
absorptive sites, but are less of a long-term problem because the ileum
appears to usually have the capacity to replace their absorptive function after
proximal small intestine resection.
Nutrient absorption does not usually occur in the stomach (choice D).


Question 5 of 5
The baby's resected gastrointestinal segment would be most likely to show which
of the following on pathologic examination?
/ A. Distended macrophages with PAS-positive granules in the submucosa
/ B. Gangrenous intestinal wall
/ C. Granuloma formation
/ D. Neoplastic epithelial proliferation
/ E. Outpouching of intestinal mucosa through the muscular layer

Explanation - Q: 2.5 Close

The correct answer is B. The intestinal wall in early necrotizing enterocolitis
shows edema, hemorrhage, and necrosis. In more advanced disease,
gangrenous necrosis of the full bowel thickness is seen, and necrotic
inflammatory debris may adhere to the mucosal surface. Some cases show
evidence of reparative change, such as epithelial regeneration and
granulation tissue formation, suggesting that the lesion may have evolved
over several days before becoming clinically obvious.
Choice A is a feature of Whipple disease.
Choice C is a feature of Crohn disease.
Choice D is a feature of colonic polyps and cancers.
Choice E is a feature of diverticulitis.


A 55-year-old man with a history of coronary artery disease and alcoholism
presents to the emergency department complaining that he
vomited bright red blood twice this morning. He denies previous episodes of
bleeding or abdominal pain. On examination, he is a
malnourished man in acute distress. His blood pressure is 90/50 mm Hg and his
pulse is 110/min. His mucous membranes are dry and his
sclera are icteric. Abdominal examination reveals a distended abdomen with an
enlarged, palpable spleen. Purplish striae are seen around
the umbilicus. On rectal examination, Iarge hemorrhoids are seen, but the stool is
negative for blood.
Question 1 of 4
Which of the following is the most likely diagnosis?
/ A. Erosive gastritis
/ B. Esophageal varices
/ C. Infectious enteritis
/ D. Mallory Weiss tear
/ E. Peptic ulcer disease

Explanation - Q: 3.1 Close

The correct answer is B. While all of the answer choices listed must be
considered in the differential, upper gastrointestinal bleeding from
esophageal varices is most likely. This patient displays many of the stigmata
of hepatic disease and portal hypertension: icteric sclera, hemorrhoids,
distended umbilical veins (caput medusae), and a history of alcoholism. In
this setting, esophageal varices would be the most likely. To make this
diagnosis definitively, however, one needs to examine the gastrointestinal
tract endoscopically.
Erosive gastritis (choice A) is a source of upper gastrointestinal hemorrhage,
but it seldom bleeds so profusely that the patient becomes hemodynamically
unstable.
Infectious disease in the gastrointestinal tract (choice C) may produce
hemorrhage, but it tends to produce lower GI bleeding.
Mallory Weiss tears (choice D) produce upper GI bleeding. This tearing of
the gastroesophageal junction occurs in alcoholics, but usually a history of
retching precedes bleeding. No such history is elicited here.
Peptic ulcer disease (choice E) can produce brisk upper GI bleeding. It is
less likely in this case because this patient has no history of GI pain.



Question 2 of 4
Which of the following coagulation factors would most likely be unaffected in this
patient?
/ A. Factor ll
/ B. Factor VII
/ C. Factor IX
/ D. Factor XIII
/ E. Von Willebrand's factor


Explanation - Q: 3.2 Close

The correct answer is E. Von Willebrand's factor is a coagulation factor
produced by the vascular endothelium and megakaryocytes. It is the only
protein in the cascade that is not synthesized in the liver. vWF mediates the
adhesion of platelets to the vessel wall basement membrane after vascular
injury. Patients with a deficiency of von Willebrand's factor have a tendency
to bleed. It is an autosomal dominant disease, and the ristocetin cofactor
activity test is the best way to clinically assess vWF function.
Factor II (choice A) is produced in the liver. Deficiency is very rare, but can
produce spontaneous or posttraumatic bleeding.
Factor VII (choice B) is produced in the liver.Severe factor VII deficiency is a
very rare cause of bleeding.
Factor IX (choice C) is produced in the liver. A factor IX deficiency is known
as hemophilia B, which is an X-linked disease.
Factor XIII (choice D) is produced in the liver. A deficiency of factor XIII
produces delayed bleeding and poor wound healing.


Question 3 of 4
Which of the following anatomic relationships provides the basis for the patient's
hemorrhoids?
/ A. Coronary vein anastomosis with the esophageal plexus
/ B. Inferior rectal vein anastomosis with the iliac vein
/ C. Paraumbilical vein anastomosis with the inferior epigastric vein
/ D. Superior mesenteric vein anastomosis with the splenic vein
/ E. Superior rectal vein anastomosis with the inferior and middle rectal vein

Explanation - Q: 3.3 Close

The correct answer is E. The patient's hemorrhoids are a consequence of
his portal hypertension. The patient has a cirrhotic liver, which impedes
circulation in the portal system. As the pressure rises in the portal system,
blood in the portal circulation begins to backflow into the caval circulation. At
the sites at which the portal system anastomoses with the caval circulation,
venous engorgement occurs. At one such site, the confluence of the superior
rectal vein (portal) with the middle and inferior rectal vein (caval), this venous
engorgement leads to hemorrhoids.
The coronary vein anastomosis with the esophageal venous plexus (choice
A) provides the anatomic basis for the esophageal varices seen in portal
hypertension. As pressure builds in the portal system, venous engorgement
occurs, and varices are produced in the distal esophagus. These varices can
be the site of life-threatening upper GI bleeding.
The anastomosis of the inferior rectal vein with the iliac vein (choice B) is a
caval-caval anastomosis and would not be affected by portal hypertension.
The anastomosis of the paraumbilical vein and the inferior epigastric vein
(choice C) is the portal-caval anastomosis responsible for the purplish striae
or caput medusae seen on this patient's abdomen. This circulatory route is
an embryologic remnant, and is only patent when portal pressure rises high
enough to re-open this pathway.
The anastomosis of the superior mesenteric vein and the splenic vein
(choice D) marks the origin of the portal vein. It may have an elevated
pressure, but it is not the basis for hemorrhoids.



Question 3 of 4
Which of the following anatomic relationships provides the basis for the patient's
hemorrhoids?
/ A. Coronary vein anastomosis with the esophageal plexus
/ B. Inferior rectal vein anastomosis with the iliac vein
/ C. Paraumbilical vein anastomosis with the inferior epigastric vein
/ D. Superior mesenteric vein anastomosis with the splenic vein
/ E. Superior rectal vein anastomosis with the inferior and middle rectal vein

Explanation - Q: 3.3 Close

The correct answer is E. The patient's hemorrhoids are a consequence of
his portal hypertension. The patient has a cirrhotic liver, which impedes
circulation in the portal system. As the pressure rises in the portal system,
blood in the portal circulation begins to backflow into the caval circulation. At
the sites at which the portal system anastomoses with the caval circulation,
venous engorgement occurs. At one such site, the confluence of the superior
rectal vein (portal) with the middle and inferior rectal vein (caval), this venous
engorgement leads to hemorrhoids.
The coronary vein anastomosis with the esophageal venous plexus (choice
A) provides the anatomic basis for the esophageal varices seen in portal
hypertension. As pressure builds in the portal system, venous engorgement
occurs, and varices are produced in the distal esophagus. These varices can
be the site of life-threatening upper GI bleeding.
The anastomosis of the inferior rectal vein with the iliac vein (choice B) is a
caval-caval anastomosis and would not be affected by portal hypertension.
The anastomosis of the paraumbilical vein and the inferior epigastric vein
(choice C) is the portal-caval anastomosis responsible for the purplish striae
or caput medusae seen on this patient's abdomen. This circulatory route is
an embryologic remnant, and is only patent when portal pressure rises high
enough to re-open this pathway.
The anastomosis of the superior mesenteric vein and the splenic vein
(choice D) marks the origin of the portal vein. It may have an elevated
pressure, but it is not the basis for hemorrhoids.



Question 4 of 4
Which of the following structures are found in the portal triad?
/ A. Hepatic vein, common hepatic artery, common bile duct
/ B. Portal vein, celiac artery, common bile duct
/ C. Portal vein, common hepatic artery, common bile duct
/ D. Portal vein, falciform ligament, common bile duct
/ E. Portal vein, sinusoids, bile canaliculi

Explanation - Q: 3.4 Close

The correct answer is C. The portal triad contains the portal vein, common
hepatic artery, and common bile duct. It is found in the fold of peritoneum,
called the hepatoduodenal ligament, that separates the greater and lesser
abdominal sacs.
None of the other choices offer a complete answer:
The hepatic vein (choice A) drains the liver into the inferior vena cava (IVC).
The celiac artery (choice B) supplies blood to the anatomic foregut. One of
its branches, the common hepatic artery, travels in the porta hepatis.
The falciform ligament (choice D) is the remnant of the umbilical vein that
passes from the anterior abdominal wall to the superior surface of the liver.
Bile canaliculi (choice E) are microscopic channels that drain bile from the
hepatocytes.


A 25-year-old man presents to the emergency department complaining of
passing bright red blood per rectum. He reports no prior episodes
of gastrointestinal bleeding, but he has had occasional lower abdominal pain and
diarrhea for the past ten months. He reports a 7 kg weight
Ioss since the onset of these symptoms. He denies sick contacts. On
examination, he is febrile with moderate, diffuse abdominal pain to
palpation and percussion. Rectal examination is positive for blood.
Question 1 of 4
Which of the following is the most likely diagnosis?
/ A. Chronic pancreatitis
/ B. Duodenal ulcer
/ C. Infectious colitis
/ D. Inflammatory bowel disease
/ E. Ischemic colitis

Explanation - Q: 4.1 Close

The correct answer is D. Patients with inflammatory bowel disease can
present with a variety of symptoms. While ulcerative colitis and Crohn
disease patients may have distinct presentations, mixed presentations are
common. Distinguishing Crohn disease and ulcerative colitis is difficult,
based on clinical findings. Symptoms typical for Crohn disease include:
abdominal pain, fever, diarrhea, weight loss, and anal disease. Symptoms
typical for ulcerative colitis include: bloody diarrhea, fever, and weight loss.
Chronic pancreatitis (choice A) presents as epigastric pain that radiates to
the back, weight loss, and steatorrhea. In many cases, a history of
alcoholism is present. Thus the location of this patient's pain and his lower GI
bleeding are not consistent with chronic pancreatitis.
Duodenal ulcer (choice B) may present as epigastric pain, and with severe
disease, severe bleeding may be present. Patients with duodenal ulcer rarely
have diarrhea and weight loss. Thus this diagnosis is unlikely.
Infectious colitis (choice C) presents as abdominal pain and bleeding. Many
infectious agents may cause GI bleeding, including Salmonella, Shigella,
Campylobacter jejuni, and E. coli. The chronic nature of this patient's
complaints and the lack of sick contacts suggests a different diagnosis.
Ischemic colitis (choice E) presents as acute onset of severe abdominal pain
often with copious bright red blood per rectum. On examination, they display
the classic finding of "pain out of proportion to examination." They are
typically elderly patients with a history of atherosclerotic or embolic disease.


Question 2 of 4
A colonoscopy is performed and mucosal ulceration with bleeding extending
continuously from the rectum to the cecum is seen. The terminal
ileum is spared. Had the terminal ileum been affected, the patient would have
been at risk for which of the following conditions?
/ A. Diabetes mellitus
/ B. Folate deficiency
/ C. Iron deficiency anemia
/ D. Kwashiorkor
/ E. Pernicious anemia

Explanation - Q: 4.2 Close

The correct answer is E. Pernicious anemia is a hypochromic,
megaloblastic anemia that may be associated with neurologic complications.
It occurs as a result of a lack of vitamin B 12. The B12/intrinsic factor
complex is absorbed in the terminal ileum by active transport. If this patient's
ulcerative colitis extended into the terminal ileum, this condition could
complicate his disease. Crohn disease almost invariably affects the terminal
ileum, and this malabsorptive condition is more common in that setting.
Ulcerative colitis usually affects only the colon, but ileal extension has been
observed.
Diabetes mellitus (choice A) is an endocrine condition, and is unrelated to
the absorptive capacity of the terminal ileum.
Folate (choice B) is absorbed in the proximal small intestine. Ileal
involvement would not affect its absorption.
Iron (choice C) is also absorbed in the proximal small intestine. Ileal
involvement would not affect its absorption.
Kwashiorkor (choice D) is protein malnutrition. Protein is absorbed
throughout the small intestine. Ileal involvement would not affect its
absorption.


Question 3 of 4
At colonoscopy the colonic mucosa appears granular, and is ulcerated.
Numerous crypt abscesses and pseudopolyps are observed. Which of
the following is the most likely diagnosis?


Explanation - Q: 4.3 Close

The correct answer is E. Granular, flat mucosa with ulcers, crypt
abscesses, and pseudopolyps are characteristic findings in ulcerative colitis.
Celiac disease (choice A) is a disease of the intestine resulting from a
hypersensitivity to the protein gluten. The intestinal mucosa is smooth and
atrophic.
Clostridium difficile colitis (choice B) or "pseudomembranous colitis" is a
colonic infection seen after extensive antibiotic use, which disturbs the
colonic flora, promoting overgrowth of C. difficile. Fibrinous
pseudomembranes are seen in the colon at colonoscopy.
Endoscopic evaluation of Crohn disease (choice C) reveals swollen mucosa
with transverse fissures and linear ulcers. Biopsy findings demonstrate
transmural involvement with granuloma formation.
Diverticula are outpouchings of the intestinal mucosa. They may bleed, or
they may become infected, leading to a painful condition, diverticulitis. The
findings here do not suggest diverticulosis (choice D).

Question 4 of 4
Several months pass and this patient's symptoms progress. He continues to
have frequent bloody diarrhea and abdominal pain. Abruptly, this
patient experiences the acute onset of severe abdominal pain and is taken to the
emergency department by friends. In the emergency
department, he is febrile, and his abdomen is rigid, with severe pain to palpation
and percussion. Laboratory findings are consistent with
dehydration. Amylase and lipase are normaI. Which of the following most likely
explains this patient's new findings?
/ A. Abdominal aortic aneurysm rupture
/ B. Acute pancreatitis
/ C. Bowel perforation and peritonitis
/ D. Sepsis from fulminant infectious colitis
/ E. Severe ischemic colitis

Explanation - Q: 4.4 Close

The correct answer is C. This patient's chronic course with acute
exacerbation suggests that this patient has viscus perforation with peritonitis
secondary to exacerbation of his ulcerative colitis. The inflammatory
processes in ulcerative colitis can be so severe that erosion from
inflammation can cause colonic perforation. Bowel contents then leak into
the peritoneal cavity, causing peritonitis. Peritonitis is characterized by fever,
severe abdominal pain, abdominal tenderness to palpation and percussion,
and rigidity of the abdominal wall.
Abdominal aortic aneurysm rupture (choice A) presents as abdominal pain
that radiates to the back. It is accompanied by hemodynamic instability that
may deteriorate to shock. This presentation is not consistent with findings in
this patient.
Acute pancreatitis (choice B) can cause severe abdominal pain and fever.
The pain, however is usually epigastric and radiating to the back. Typically
nausea and vomiting accompany pancreatitis. Amylase and lipase are
elevated.
This patient does not display the symptoms of sepsis (choice D). In sepsis,
patients are febrile with hemodynamic instability.
Ischemic colitis (choice E) can lead to perforation and peritonitis, and if the
patient had symptomatology consistent with ischemic colitis, it could be the
source of this patient's peritonitis. This patient did not report bright red blood
per rectum, or "pain out of proportion to examination," making ischemic colitis
less likely.









A 47-year-old, darkly pigmented man with a known history of alcohol abuse
begins vomiting large quantities of blood and is brought by
ambulance to the emergency department.
Question 1 of 6

In the emergency department, the man is found to have a temperature of 36.7 C
(98.1 F), blood pressure of 65/40 mm Hg and dropping
rapidly, a weak pulse of 130/min, and respirations of 29/min. These vital signs
suggest that which of the following is developing?
/ A. Congestive heart failure
/ B. Meningitis
/ C. Pneumonia
/ D. Septicemia
/ E. Shock

Explanation - Q: 5.1 Close

The correct answer is E. The patient's low and dropping blood pressure,
tachycardia, high respiratory rate, and slightly below normal body
temperature are all consistent with impending shock. At this point, the other
conditions listed in the choices have not yet been ruled out, but clinically, the
patient should begin to be immediately treated for the shock, even if the
therapeutic workup for underlying conditions must be temporarily deferred.

Question 2 of 6
A blood sample is drawn and an IV Iine is started. While the patient is being
cross-matched, the physical examination is continued. The
patient's sclerae are noted to be icteric and his nail beds and palms have a
yellowish hue. A caput medusa is noted. Which of the following is
the most accurate description of a caput medusa?
/ A. Ecchymoses over the mastoid process
/ B. Paradoxical increase in venous distension and pressure during inspiration
/ C. Reflex movement of the eyes in the opposite direction to that in which the
head is moved
/ D. Small bony masses found on the terminal phalanges
/ E. Varicose veins radiating from the area of the umbilicus

Explanation - Q: 5.2 Close

The correct answer is E. Medusa was a goddess with snakes instead of
hair on her head. The caput medusa (Medusa's head) is an old term still in
fairly common use for numerous varicose veins radiating over the abdomen
from the area of the umbilicus.
Choice A describes Battle's sign, which is suggestive of basal skull fracture.
Choice B describes Kussmaul's sign, which is seen in constrictive
pericarditis.
Choice C describes the doll's eye sign, which is looked for in the evaluation
of comatose patients and suggests functional integrity of the brainstem
tegmental pathways and cranial nerves involved in eye movement.
Choice D describes Heberden's nodules, which are seen in osteoarthritis.


Question 3 of 6
Caput medusa specifically suggests which of the following diagnoses?
/ A. BIadder infection
/ B. Duodenal ulcer
/ C. Gastric ulcer
/ D. Pancreatitis
/ E. Portal hypertension

Explanation - Q: 5.3 Close

The correct answer is E. The caput medusa develops when severe portal
hypertension induces dilation of the anastomotic channels between the portal
venous system and the systemic venous system, some of which involve the
superficial veins near the umbilicus. The other answers are distracters.


Question 4 of 6

Which of the following is the most common cause of this patient's disorder in the
United States?
/ A. Hepatic cirrhosis
/ B. Hepatic vein thrombosis
/ C. Hepatocellular carcinoma
/ D. Metastatic disease to the liver
/ E. Portal vein thrombosis

Explanation - Q: 5.4 Close

The correct answer is A. The overwhelmingly most common cause of portal
hypertension in the United States is hepatic cirrhosis, which is usually due to
either alcoholism or hepatitis viral infection. In this patient's case, the
diagnosis of cirrhosis is further clinically substantiated by his jaundice, as
evidenced by his sclera, nail beds, and palms. (Look in these areas on
individuals in whom dark skin pigmentation may mask the jaundice
generally.) The other entities listed are occasional causes of portal
hypertension.
Question 5 of 6
Endoscopic studies demonstrate that this patient has bleeding esophageal
varices, and the bleeding is successfully stopped with
sclerotherapy. What percentage of patients with bleeding esophageal varices
have another episode of variceal bleeding at a subsequent
time?
/ A. 5%
/ B. 25%
/ C. 40%
/ D. 70%
/ E. 95%


Explanation - Q: 5.5 Close

The correct answer is D. Patients who have had one episode of bleeding
from esophageal varices have an approximately 70% chance of developing a
second incident of bleeding, and one third of these episodes of rebleeding is
fatal.


Question 6 of 6

Following blood transfusions and sclerotherapy, the patient initially feels
reasonably well and is able to converse with medical personneI. Over
the next 12 hours, while he does not begin to rebleed, his mental status
deteriorates. Arterial blood levels of which of the following would be
most helpful in confirming the likely diagnosis?
/ A. Ammonia
/ B. Angiotensin l
/ C. Calcitonin
/ D. Carbon monoxide
/ E. Ceruloplasmin


Explanation - Q: 5.6 Close

The correct answer is A. Hepatic encephalopathy is seen in end-stage
cirrhosis patients, and can either present or worsen in the presence of
gastrointestinal bleeding. The blood in the upper gastrointestinal tract
behaves essentially as a high protein load, and increases the absorption of
ammonia and nitrogen, which cannot be appropriately metabolized by the
liver. GI bleeding may also predispose for inadequate renal function
secondary to hypotension.
Angiotensin I (choice B) is part of the renin-angiotensin-aldosterone system
for blood pressure and sodium ion control.
Calcitonin (choice C) is a hormone secreted by the thyroid, which may be
increased in medullary carcinoma of the thyroid.
Carbon monoxide (choice D) increases in the blood in smokers.
Ceruloplasmin (choice E) is a copper-carrying protein monitored in patients
with Wilson disease.


A 2-year-old child is seen in the emergency department because of bright red
blood per rectum. This is the third time this has happened, and
on the previous episode, no lesion was identified on colonoscopy. During this
visit, the child is scheduled for small bowel barium studies,
which show an outpouching of the distal ileum about 2 feet proximal to the
cecum.
Question 1 of 7
Which of the following is the most likely diagnosis?
/ A. Abnormally located appendix
/ B. Crohn disease
/ C. Diverticulosis
/ D. Meckel diverticulum
/ E. Potter syndrome

Explanation - Q: 6.1 Close

The correct answer is D. This patient has a Meckel diverticulum. Meckel
diverticula can be asymptomatic through life, or may come to medical
attention because of a bleeding peptic ulcer, acute inflammation, rupture,
strangulation, or intussusception of the Meckel diverticulum. Diagnosis, as in
this case, may be difficult because the ileum is difficult to visualize. In some
cases, small bowel barium studies may successfully identify the lesion.
While the appendix (choice A) can have variations in location, these tend to
involve the side of the cecum into which it opens. Also, the appendiceal
lumen remains narrow in aberrant locations.
Crohn disease (choice B) can involve the distal ileum and cause
gastrointestinal bleeding, but would not cause an isolated outpouching of the
ileum.
Diverticulosis (choice C) refers to acquired diverticula, and is usually a
disease of older individuals.
Potter syndrome (choice E) refers to the cluster of bilateral renal agenesis,
oligohydramnios, limb deformities, facial deformities, and pulmonary
hypoplasia.

Question 2 of 7
The prevalence of this patient's anatomic anomaly in the US population is which
of the following?
/ A. 2%
/ B. 6%
/ C. 15%
/ D. 40%
/ E. 80%

Explanation - Q: 6.2 Close

The correct answer is A. The usually cited prevalence for Meckel
diverticulum is 2%, although it actually varies from 0.2% to 4%. Many
medical students remember five "2s" associated with Meckel diverticulum: 2
inches long, 2 feet from the ileocecal valve, 2% of the population, commonly
presents in the first 2 years of life, and may have 2 types of epithelium.


Question 3 of 7
This patient's anatomic anomaly is thought to be embryologically derived from
which of the following?
/ A. Mesonephric duct
/ B. Mullerian duct
/ C. Paramesonephric duct
/ D. Vitelline duct
/ E. Wolffian duct

Explanation - Q: 6.3 Close

The correct answer is D. The vitelline duct or yolk stalk embryologically
connects the midgut to the yolk sac. The duct usually disappears by the
seventh gestational week, but if it fails to obliterate, several lesions can be
produced, including Meckel diverticulum, a persistent vitelline duct that
drains as a fistula through the anterior abdominal wall at the umbilicus, a
fibrous band, or a vitelline duct cyst. The other ducts are genital ducts.
The mesonephric (wolffian) duct (choices A and E) develops into seminal
vesicles, epididymis, ejaculatory duct, and ductus deferens; the
paramesonephric (mullerian) duct (choices B and C) develops into the
fallopian tube, uterus, and part of the vagina.



Question 4 of 7
Which of the following is the most common type of ectopic tissue seen in this
patient's anatomic anomaly?
/ A. Endometrial tissues
/ B. Gastric mucosa
/ C. Jejunal mucosa
/ D. Pancreatic tissue
/ E. Rectal mucosa

Explanation - Q: 6.4 Close

The correct answer is B. Meckel diverticula often have ectopic tissues in
them, the most common of which is heterotopic gastric mucosa. The tissues
listed in the other choices can also be seen, as well as colonic mucosa.


Question 5 of 7
The ulceration that was the source of bleeding in this patient is most likely related
to acid secretion by which of the following cell types?
/ A. Chief cells
/ B. Mucous neck cells
/ C. Parietal cells
/ D. Surface epithelial cells
/ E. Zymogenic cells

Explanation - Q: 6.5 Close

The correct answer is C. In gastric mucosa, whether in the stomach, or in
an ectopic location, it is the parietal cells that secrete acid. This acid
secretion is particularly likely to cause peptic ulceration in a Meckel
diverticulum or the adjacent ileum, because the secretion of protective
mucus is likely to be markedly inadequate in this setting, and the distal small
intestinal mucosa is not equipped to handle an acid environment.
The chief cells, also called zymogenic cells (choices A and E) secrete
pepsinogen.
The mucous neck cells and surface epithelial cells (choices B and D)
secrete mucus.


Question 6 of 7
The acid-secreting cells are stimulated by which of the following hormones?
/ A. Cholecystokinin
/ B. Gastric inhibitory peptide
/ C. Gastrin
/ D. Secretin
/ E. Vasoactive intestinal polypeptide

Explanation - Q: 6.6 Close

The correct answer is C. The polypeptide hormone gastrin is secreted by
the duodenum and pyloric antrum. Its release is stimulated by the presence
of digested protein in the stomach and duodenum. Gastrin stimulates acid
secretion from the parietal cells of the gastric glands and pepsinogen
secretion from the chief cells.
Cholecystokinin (choice A) is secreted by the endocrine cells of the
duodenum and proximal jejunum, and stimulates pancreatic enzyme
synthesis and secretion, increases gall bladder emptying, and decreases
gastric emptying.
Gastric inhibitory peptide (choice B) inhibits gastrin release and gastric acid
secretion, and causes insulin release from the endocrine pancreas.
Secretin (choice D) is produced in the crypts of Lieberkhn of the
duodenum, and stimulates pepsinogen secretion from the stomach, and fluid
and bicarbonate release from the pancreas.
Vasoactive intestinal polypeptide (choice E) induces smooth muscle
relaxation, modifies the composition of pancreatic juice and bile, and inhibits
gastric acid secretion and absorption from the intestinal lumen.

Question 7 of 7
Stimulation of which receptor on the acid-secreting cell leads to increased acid
secretion?
/ A. Epinephrine receptor
/ B. Histamine-1 receptor
/ C. Histamine-2 receptor
/ D. Prostaglandin E2 receptor
/ E. Somatostatin receptor

Explanation - Q: 6.7 Close

The correct answer is C. Acid secretion by parietal cells can be stimulated
by the gastrin receptor, the histamine-2 (H2) receptor, and the acetylcholine
receptor. Drugs with anti-H2 receptor activity are used to treat peptic ulcer
disease. The histamine that stimulates the H2 receptors is probably derived
from enterochromaffin cells.
Epinephrine and histamine-1 receptors (choices A and B) do not appear to
have a physiologic role in gastric acid secretion.
Substances capable of reducing gastric acid secretion include prostaglandin
E
2
(choice D), secretin, and somatostatin (choice E).

A 44-year-old construction worker is brought to the hospital following a fall and a
back injury. On his third day in the hospitaI, he appears
irritable and edgy and demands to leave. His behavior gradually becomes worse,
and he is put into restraints in order to prevent further back
injury. A psychiatrist called to consult on this patient reviews the chart and notes
no prior history of psychiatric illness. He smokes and drinks
moderately, and uses no street drugs, per notes in the chart. His previous
physical health was good. The man appears confused and talks to
the physician about having seen snakes in his room. He is disoriented to time
and place, is perspiring profoundly and is tremulous. The man
seems agitated, confused, and is hallucinating. His blood pressure, pulse, and
respiratory rate are gradually increasing.

Question 1 of 4

Which of the following is the most likely diagnosis?
/ A. Conversion disorder
/ B. Delirium tremens
/ C. Delusional disorder
/ D. Pathological intoxication
/ E. Schizophrenia

Explanation - Q: 1.1 Close

The correct answer is B. It would be important to know when the patient's
last drink was in order to assess the potential risk for the development of
withdrawal symptoms. The signs of autonomic instability, along with visual
hallucinations, confusion, and disorientation indicate delirium tremens, since
the patient has probably being drinking heavily in the past. This justifies the
need to keep him in the hospital against his will.
Conversion disorder (choice A) is defined by one or more symptoms
affecting voluntary motor or sensory function, suggesting a neurological
condition. It is precipitated by psychological stressors. The symptoms are not
intentionally produced nor explained by other medical conditions. The
symptoms include motor or sensory deficits, seizures, or mixed symptoms
that cause significant impairment in everyday functioning.
Delusional disorder (choice C) is characterized by one sole fixed and
unshakable delusion. The delusion is nonbizarre, and has to be present at
least a month. Apart from the impact of delusion, the functioning is not
markedly impaired in other areas of life.
Pathological intoxication (choice D) occurs when the subject has just taken
alcohol and has an excessive intoxication reaction to a small amount. It
presents in the same way as other alcohol intoxications.
Schizophrenia (choice E) is defined by the presence of active symptoms of
delusions, hallucinations, disorganized speech or behavior in the past month.
The continuous signs must be present longer than six months and cause
significant impairment in social or occupational functioning.



Question 1 of 4
Which of the following is the most likely diagnosis?
/ A. Conversion disorder
/ B. Delirium tremens
/ C. Delusional disorder
/ D. Pathological intoxication
/ E. Schizophrenia

Explanation - Q: 1.1 Close

The correct answer is B. It would be important to know when the patient's
last drink was in order to assess the potential risk for the development of
withdrawal symptoms. The signs of autonomic instability, along with visual
hallucinations, confusion, and disorientation indicate delirium tremens, since
the patient has probably being drinking heavily in the past. This justifies the
need to keep him in the hospital against his will.
Conversion disorder (choice A) is defined by one or more symptoms
affecting voluntary motor or sensory function, suggesting a neurological
condition. It is precipitated by psychological stressors. The symptoms are not
intentionally produced nor explained by other medical conditions. The
symptoms include motor or sensory deficits, seizures, or mixed symptoms
that cause significant impairment in everyday functioning.
Delusional disorder (choice C) is characterized by one sole fixed and
unshakable delusion. The delusion is nonbizarre, and has to be present at
least a month. Apart from the impact of delusion, the functioning is not
markedly impaired in other areas of life.
Pathological intoxication (choice D) occurs when the subject has just taken
alcohol and has an excessive intoxication reaction to a small amount. It
presents in the same way as other alcohol intoxications.
Schizophrenia (choice E) is defined by the presence of active symptoms of
delusions, hallucinations, disorganized speech or behavior in the past month.
The continuous signs must be present longer than six months and cause
significant impairment in social or occupational functioning.
Question 2 of 4
Which of the following is the most appropriate pharmacotherapy?
/ A. Carbamazepine
/ B. Chlordiazepoxide
/ C. Disulfiram
/ D. Iron supplement
/ E. Phenobarbital

Explanation - Q: 1.2 Close

The correct answer is B. Chlordiazepoxide is a long-acting benzodiazepine
that is used for detoxification from alcohol in uncomplicated cases. It has
several metabolites that are long-acting, thus making it somewhat difficult,
because of impaired liver metabolism, to efficiently manage the detoxification
without risking the accumulation of the drug and its metabolites.
Carbamazepine (choice A) is an anticonvulsant that, according to several
studies, is as effective as benzodiazepines in controlling symptoms
associated with alcohol withdrawal. The potential risk of adverse side effects,
including the induction of liver enzymes, limits its clinical usefulness for this
application.
Disulfiram (choice C) inhibits the enzyme aldehyde dehydrogenase, leading
to elevated levels of acetaldehyde upon alcohol ingestion. It has been used
for long-term treatment of alcoholism in order to maintain abstinence, but it
has not been used for detoxification.
Iron supplementation (choice D) is not a part of the usual treatment of an
alcoholic patient. Thiamine, folic acid, and magnesium usually are part of the
standard treatment.
Phenobarbital (choice E) is, like all barbiturates, metabolized by the liver,
and causes induction of hepatic enzymes. It can be used for uncomplicated
detoxification from other barbiturates or benzodiazepines, but is not used for
detoxification from alcohol.

Question 3 of 4
The nurse obtains collateral information from his family, which support the
physician's diagnosis. They also report that he had blackouts in the
past. Which of the following best describes blackouts related to this patient's
condition?
/ A. Anterograde amnesia
/ B. Confabulation
/ C. Hypermnesia
/ D. Remote memory loss
/ E. Retrograde amnesia
Explanation - Q: 1.3 Close

The correct answer is A. Alcohol-related blackouts refer to discrete
episodes of anterograde amnesia following alcohol intoxication. During the
blackout, remote memory is intact, as well as the ability to perform tasks.
However, individuals experience a specific short-term memory deficit, such
that they are unable to recall new information or events that happened in the
minutes before the blackout.
Confabulation (choice B) can be seen in chronic alcoholics with dementia, in
whom gaps in memory are filled with the events that never happened.
Hypermnesia (choice C) describes an exaggerated degree of retention and
recall, and is not seen in blackouts.
Remote memory loss (choice D) may happen in later stages of dementia
related to alcohol abuse, but is not seen initially, and certainly is not typical
for blackouts.
Retrograde amnesia (choice E) refers to amnesia prior to a point in time,
and is characterized by the inability to recall previously remembered
knowledge.

Question 4 of 4
After he had been stabilized, the psychiatrist discusses the use of disulfiram after
discharge with the patient. Disulfiram exerts its
pharmacologic effect on which of the following?
/ A. AIcohol dehydrogenase
/ B. AIdehyde dehydrogenase
/ C. Cytochrome P-450
/ D. GABA-benzodiazepine complex
/ E. Mu opioid receptors


Explanation - Q: 1.4 Close

The correct answer is B. Disulfiram inhibits aldehyde dehydrogenase,
resulting in accumulation of acetaldehyde following ethanol ingestion. This
reaction causes flushing, headache, hypotension, tachycardia, sweating,
anxiety, and confusion. It is used temporarily to establish a pattern of
sobriety.
Alcohol dehydrogenase (choice A) is not affected by disulfiram. It
metabolizes alcohol to acetaldehyde, which is then converted to acetate via
aldehyde dehydrogenase.
Cytochrome P-450 enzymes (choice C) are not directly affected by
disulfiram. However, other concomitant medications can affect the
metabolism of disulfiram.
The GABA-benzodiazepine complex (choice D) is activated by alcohol, not
by disulfiram.
Mu opioid receptors (choice E) are not involved in the mechanism of action
of disulfiram.



A 24-year-old man is brought to the emergency department by the police after
taking a sledgehammer to the electrical outlets in his bedroom
and placing tinfoil on the walls to "scramble alien radio transmissions." His
parents called 911 after he had stated "I know who you really are,
you are a shapeshifter" and lunged at his father. The patient returned to his
parent's home 7 months ago after failing two consecutive
semesters at college and losing his job at the library for destroying film projectors
"to stop the Martian voices." Since then he has been cared
for at home. He spends most of the time in his room, replaying CDs "Ioaded with
top secret information." In the emergency department, the
patient sits calmly and quietly and reports he is glad his cause wilI "finally get the
government's attention." The patient and his family deny any
history of depressive symptoms or substance use. He has no significant past
medical history, no allergies, and family history is
noncontributory. He reports that aliens have entered his room and removed his
heart and parts of his brain while he was sleeping at night.
Physical examination is remarkable for a malodorous, disheveled appearance,
but is otherwise normal and routine laboratory studies are
normaI.


Question 1 of 6
Which of the following is the most likely diagnosis?
/ A. Bipolar disorder
/ B. Brief psychotic disorder
/ C. Chronic paranoid schizophrenia
/ D. Major depressive disorder with psychotic features
/ E. Schizoaffective disorder

Explanation - Q: 2.1 Close

The correct answer is C. Chronic paranoid schizophrenia is characterized
by delusions, hallucinations (usually auditory), and possibly, disorganized
behavior or speech, and negative symptoms (e.g., apathy, lack of attention to
hygiene). Delusions are grandiose (saving the world) and persecutory. Other
types of schizophrenia include disorganized, catatonic, and residual.
Bipolar disorder (choice A) is associated with depressive episodes, which
the history does not support. Manic patients may be grandiose, but do not
usually have a set of persecutory delusions. Manic patients have a
characteristic presentation of loud speech and racing thoughts, and present
with an expansive or irritable mood. This patient is calm and quiet when his
environment does not seem threatening to his delusional beliefs.
Brief psychotic disorder (choice B) is characterized by symptoms of one
month duration or less. His symptoms began at least seven months ago.
Major depressive disorder (choice D) would include depressed mood, or
anhedonia, and a total of five depressive symptoms. Depressed patients
exhibit mood congruent delusions- "I have a deadly disease, I'm bankrupt,
etc." Having the power to save the world is a grandiose delusion.
Schizoaffective disorder (choice E) would include some episodes of mood
symptoms (mania or depression) not in the presence of psychotic symptoms.
There is no history of mood symptoms in this patient.


Question 2 of 6
The patient is stabilized on medication and released from the hospital fourteen
days later on a new medication. He returns the next day,
terrified, drooling, with the left side of his face in spasm, and his neck "frozen."
Which of the following is the most likely diagnosis?
/ A. Acute dystonic reaction
/ B. Akathisia
/ C. Neuroleptic-induced parkinsonism
/ D. Neuroleptic malignant syndrome
/ E. Tardive dyskinesia

Explanation - Q: 2.2 Close

The correct answer is A. This patient is having an acute dystonic reaction
resulting from his neuroleptic medication. This generally occurs in the first
week, most often in the first 48 hours. It is characterized by an involuntary
spasm of muscles, usually in the head and neck. It can involve the larynx
and pharynx and needs immediate treatment to protect the airway. This
patient is drooling, which suggests that he cannot swallow.
Akathisia (choice B) is a neuroleptic side-effect best described as a
subjective feeling of restlessness, and patients often respond with pacing or
constant fidgeting. Akathisia may be underdiagnosed, as it is difficult to
distinguish from states of agitation.
Neuroleptic-induced parkinsonism (choice C) is characterized by
bradykinesia, cogwheeling rigidity, postural instability, and tremor. Elderly
patients are at a higher risk for this.
Neuroleptic malignant syndrome (choice D) presents as confusion in
addition to autonomic instability, hyperthermia, and rigidity. It is a medical
emergency.
Tardive dyskinesia (choice E) is a late onset side-effect of neuroleptics in
patients who have usually taken medicines for a long time.



Question 3 of 6
Which of the following agents is most likely to relieve the patient's symptoms?
/ A. Benztropine
/ B. CIomipramine
/ C. Phenelzine
/ D. Propanolol
/ E. Vitamin E

Explanation - Q: 2.3 Close

The correct answer A. Benztropine is an anticholinergic agent given to treat
acute dystonic reaction. Dopamine blockade by neuroleptics causes a
relative imbalance between dopamine and acetylcholine, with balance
theoretically restored by decreasing acetylcholine.
Clomipramine (choice B) is a tricyclic antidepressant with some serotonergic
properties. It is used to treat severe OCD.
Phenelzine (choice C) is a MAO inhibitor. It is associated with hypertensive
crisis.
Propanolol (choice D) is a beta blocker used to treat akathisia.
Vitamin E (choice E) is used to prevent tardive dyskinesia.

Question 4 of 6
The patient's side effect is related to dopamine blockade in which of the following
pathways?
/ A. Corticospinal
/ B. Mesocortical
/ C. Mesolimbic
/ D. Nigrostriatal
/ E. Tuberoinfundibular

Explanation - Q: 2.4 Close

The correct answer is D. Blockade of D2 receptors in the nigrostriatal
pathway is thought to produce extrapyramidal reactions, the symptoms of
which range from akathisia to tremor to dystonia.
The corticospinal pathway (choice A) is involved in movement disorders, but
it is not related to extrapyramidal movement disorders induced by dopamine
blockade in the nigrostriatal pathway (i.e., movement disorders "outside" the
"pyramids").
Blockade of D2 receptors in the mesocortical pathway (choice B) can
produce apathy and withdrawal, lack of motivation and cognitive blunting,
and may compound the negative symptoms of schizophrenia.
Blockade of D2 receptors in the mesolimbic pathway (choice C) is thought to
mediate the reduction in positive symptoms. It is the therapeutic mechanism
of action.
Blockade of D2 receptors in the tuberoinfundibular pathway (choice E) can
cause prolactinemia with resultant galactorrhea, breast enlargement,
amenorrhea, and sexual dysfunction.

Question 5 of 6
After experiencing the side-effect above, the patient is switched to a new atypical
antipsychotic, and has a recurrence of his psychotic
symptoms. The patient notes that his medicine "wears off" in about 10 hours, and
when switched to twice-daily dosing, his psychotic
symptoms remit. Which of the following drugs was he most likely given?
/ A. AIprazolam
/ B. CIozapine
/ C. OIanzepine
/ D. Quetiapine
/ E. Thioridazine

Explanation - Q: 2.5 Close

The correct answer is D. Quetiapine's half-life is only about 6 hours. Some
patients have been successfully switched to once daily dosing after
stabilization, but reemergence of psychotic symptoms on quetiapine may
suggest underdosing, or the need for multiple daily doses.
Alprazolam (choice A) is a benzodiazepine with a rapid onset of action and
a short half life. It is high risk for addiction.
Clozapine (choice B) is an atypical antipsychotic associated with
agranulocytosis and seizures and has a half-life of about 12 hours.
Olanzepine (choice C) is an atypical antipsychotic is associated with weight
gain, dyslipidemia, and diabetes and has a half-life of about 33 hours.
Thioridazine (choice E) is a typical antipsychotic that has a maximum daily
dose of 800 mg to reduce the possibility of retinal deposits.


Question 6 of 6
Unfortunately, the patient's psychosis responds poorly to several different
antipsychotic medications. He is started on a different antipsychotic
medicine for refractory disease. He returns to the emergency room six months
later with a sore throat and chills. His leukocyte count is
2,300/mm3and his absolute neutrophil count is <100/mm3. Which of the
following medications is most likely responsible for this side effect?
/ A. CIozapine
/ B. OIanzepine
/ C. Quetiapine
/ D. Risperidone
/ E. Venlafaxine

Explanation - Q: 2.6 Close

The correct answer is A. Clozapine is associated with agranulocytosis at
(0.5-2%) and seizures. Clozapine therapy requires frequent blood draws to
evaluate for a decrease in the WBC. This side effect is potentially fatal.
Clozapine is usually reserved for refractory schizophrenia.
Olanzepine (choice B) is associated with weight gain, dyslipidemia, and
diabetes.
Risperidone (choice D) is associated with extrapyramidal symptoms, felt to
occur less often than with the older antipsychotic medicines (e.g.,
haloperidol).
Quetiapine (choice C) is associated with orthostatic hypotension.
Venlafaxine (choice E) is an antidepressant medicine and is not used to
treat psychosis.



A 23-year-old right-handed man presents to his family physician complaining of
forgetfulness. He states that he has been a bicycle messenger
for 5 years in the same city and has, over the past 6 months, periodically become
disoriented and lost. His girlfriend confirms that he has
become more forgetful and also states that he also been acting strangely, such
as trying to unlock his bicycle lock with his finger. He says he
does not remember doing these things. His employer referred him for a drug
screen, which was negative. He was then sent to a
psychotherapist who started him on thioridazine (MellariI), after which his
forgetfulness and bizarre behavior increased. In addition, since
starting this medication, his girlfriend has witnessed three episodes in which he
was unresponsive for 5 minutes, during which, he was
smacking his lips, and after which, he was confused for about an hour. During
these episodes, he did not fall down or exhibit any shaking
movements. Before these episodes occur, he notices an unpleasant taste and a
smell akin to burning rubber. His family physician refers him
for an MRI of the brain, which reveals no abnormalities. An
electroencephalogram (EEG) shows 1.5-2.5 Hz spike and wave discharges
Iocalized to the right temporal lobe. A positron emission tomography (PET) scan
reveals hypometabolism in the area of the right temporal
Iobe.
Question 1 of 5
Which of the following is the most likely diagnosis?
/ A. Absence seizures
/ B. Complex partial seizures
/ C. Creutzfeldt-Jakob disease
/ D. GIioblastoma multiforme
/ E. Schizophrenia


Explanation - Q: 3.1 Close

The correct answer is B. A patient with progressive memory problems,
behavioral changes, episodes of abnormal responsiveness to the
environment, automatisms, abnormal autonomic features followed by
amnesia for the event, and postictal confusion lasting about an hour,
probably has temporal lobe epilepsy manifested as complex partial seizures.
These seizures may be preceded by a well-defined aura, often involving
gustatory and/or olfactory hallucinations. Complex partial seizures usually
present in adolescents and young adults, and more than 70% of these
patients have a temporal lobe origin for their seizures. Approximately 2/3 of
patients with temporal lobe epilepsy have a normal MRI of the brain. These
patients are sometimes erroneously diagnosed with a psychiatric disorder,
however antipsychotics, such as thioridazine (Mellaril), can lower the seizure
threshold and may exacerbate their problem.
Absence seizures (choice A) usually present in the first decade of life and
are characterized by momentary lapses in awareness accompanied by
motionless staring, automatisms, and arrest of any ongoing activity. These
seizures begin and end abruptly (usually less than 30 seconds) without any
aura or postictal confusion. EEG often reveals characteristic 3 Hz spike and
wave discharges.
Creutzfeldt-Jakob disease (choice C) is a rare prion-transmitted disease,
which results in dementia that rapidly progresses over months, with death
occurring in less than a year. The EEG characteristically exhibits periodic
biphasic or triphasic, high amplitude sharp waves.
A glioblastoma multiforme (choice D) could present with memory or
behavioral problems, depending upon its location, as well as seizures.
Glioblastoma multiforme is the most common primary brain tumor and
usually presents in the 4th to 6th decades of life. However, the seizures are
often generalized tonic-clonic. In addition, an MRI of the brain would most
likely reveal an enhancing mass lesion with surrounding edema.
Schizophrenia (choice E) is a psychiatric disorder characterized by psychotic
symptoms that significantly impair functioning and that involve disturbances
in feeling, thinking, and behavior. Delusions and hallucinations, usually
auditory, are often present. The disorder usually presents in the late teens
and early twenties. Most schizophrenic patients have normal EEGs, but
some have decreased alpha, and increased theta and delta activity. PET
scanning may also reveal frontal and parietal lobe hypometabolism.
Symptoms are usually improved and not worsened by antipsychotics, such
as thioridazine (Mellaril).

Question 2 of 5
The most appropriate treatment for this patient would involve a drug that would
accomplish which of the following?
/ A. AIkylate DNA
/ B. Increase the frequency of chloride channel opening
/ C. Post-synaptic blockade of CNS dopamine type 2 receptors
/ D. Preferentially inhibit reverse transcriptase
/ E. Use-dependent blockade of sodium channels

Explanation - Q: 3.2 Close

The correct answer is E. Antiepileptic drugs, such as phenytoin and
carbamazepine, block voltage-dependent sodium channels and impair high-
frequency action-potential generation. These drugs are used to treat
patients, such as the one described above, with complex partial epilepsy.
Alkylation of DNA (choice A) is accomplished by nitrosoureas, such as
carmustine, which are used to treat brain tumors like glioblastoma
multiforme.
Increasing the frequency of Cl
-
channel opening (choice B), which then
facilitates GABA
A
activity, is accomplished by benzodiazepines, such as
diazepam, which may be used in the treatment of status epilepticus.
Post-synaptic blockade of CNS dopamine type 2 receptors (choice C) is
accomplished by antipsychotic drugs, such as thioridazine. These drugs are
used in the treatment of schizophrenia.
Preferential inhibition of reverse transcriptase (choice D) is accomplished by
reverse transcriptase inhibitors, such as zidovudine (AZT), which are used in
the treatment of HIV.

Question 3 of 5
If surgery is required to treat the above patient, which of the following would be
the most likely deficit to result from a right temporal lobectomy?
/ A. Conduction aphasia
/ B. Left hemiplegia
/ C. Left inferior quadrantic anopsia
/ D. Left superior quadrantic anopsia
/ E. Receptive aphasia

Explanation - Q: 3.3 Close

The correct answer is D. A right temporal lobectomy will disrupt the optic
radiations of Meyer's loop as they run in the temporal lobe. This will result in
a left superior quadrantic anopsia.
A conduction aphasia (choice A) would result from an injury to the arcuate
fasciculus in the dominant (left) hemisphere connecting Broca's area to
Wernicke's area. This manifests as an inability to repeat, while maintaining
good comprehension and fluent speech.
A left hemiplegia (choice B) would result from an injury to the right primary
motor cortex or right corticospinal tract, such as from a right middle cerebral
artery stroke.
A left inferior quadrantic anopsia (choice C) would result from a right parietal
lesion.
A receptive aphasia (choice E) would result from an injury to Wernicke's
area, located in the left superior temporal gyrus. Right-handed people are
nearly always left brain language-dominant, while 85% of left-handed people
are left brain language-dominant. A right temporal lobectomy performed on
the patient described above would probably not result in a language deficit.



Question 4 of 5
Histological examination of diagnostic tissue from this patient would most likely
reveal which of the following?
/ A. Hippocampal gliosis and atrophy
/ B. Intranuclear and intracytoplasmic inclusions
/ C. Neurofibrillary tangles and neuritic plaques
/ D. Pseudopalisading necrosis
/ E. Spongiform changes with astrocytosis, but without inflammation

Explanation - Q: 3.4 Close

The correct answer is A. Hippocampal gliosis and atrophy can result from
mesial temporal sclerosis, a cause of temporal lobe epilepsy.
Intranuclear and intracytoplasmic inclusions (choice B) are a feature of a
viral encephalitis, such as in subacute sclerosing panencephalitis (SSPE).
Neurofibrillary tangles and neuritic plaques (choice C) are a feature of
Alzheimer disease.
Pseudopalisading necrosis (choice D) is a feature of glioblastoma
multiforme.
Spongiform changes with astrocytosis, but without inflammation (choice E) is
a feature of Creutzfeldt-Jakob disease.


Question 5 of 5
The patient's memory and behavior problems may be related to interruptions in
the interconnections between the major limbic structures.
Which of the following represents the circuit most likely responsible for the
formation of memory?
/ A. Amygdala septal area, Iateral preoptic and hypothalamic areas, and the
nucleus of the diagonal band
/ B. Dentate nucleus, contralateral ventral lateral and ventral posterolateral
thalamic nuclei, motor cortex
/ C. Hippocampal formation, mamillary body, anterior nucleus of the thalamus,
cingulate gyrus, entorhinal cortex
/ D. OIfactory tract pyriform cortex, entorhinal cortex, hippocampal formation,
septal area
/ E. Subthalamus, globus pallidus, centromedian nucleus of the thalamus,
putamen
Explanation - Q: 3.5 Close

The correct answer is C. The interconnections between the hippocampal
formation to the mamillary body by the way of the fornix, to the anterior
nucleus of the thalamus by way of the mamillothalamic tract, to the cingulate
gyrus, to the entorhinal cortex describes the Circuit of Papez. This circuit was
hypothesized by Papez in the 1930s, and was thought to control emotions.
This interpretation is now believed to be oversimplified, and the circuit is
thought to play a larger role in the formation of new memories.
Amygdala to the septal area, the lateral preoptic and hypothalamic areas,
and the nucleus of the diagonal band (choice A) represents some of the
amygdalofugal projections, which are involved in emotions.
Dentate nucleus to the contralateral ventral lateral and ventral posterolateral
thalamic nuclei to the motor cortex (choice B) represents cerebellar efferent
fibers concerned with coordination of movement.
Olfactory tract to the pyriform cortex to the entorhinal cortex to the
hippocampal formation to the septal area (choice D) describes one of the
olfactory pathways.
Subthalamus to the globus pallidus to the centromedian nucleus of the
thalamus to the putamen (choice E) represents some of the pallidofugal fiber
system important in the modulation of somatic motor activity.


Question 1 of 6
Which of the following mechanisms of action is responsible for this syndrome?
/ A. BIockade of dopamine receptors
/ B. BIockade of NMDA receptors
/ C. Decreased release of serotonin in the synaptic cleft
/ D. Increased release of monoamines in the synaptic cleft
/ E. Potentiates GABA at the GABAA receptor.

Explanation - Q: 4.1 Close

The correct answer is D. Amphetamines increase the release of
norepinephrine, dopamine, and serotonin in the synaptic cleft. Release
occurs through the uptake carrier.
Blockade of dopamine receptors (choice A) is the mechanism of action
responsible for the efficacy and some side effects of the antipsychotic
medications.
Blockade of NMDA receptors (choice B) is the mechanism of action of
phencyclidine (PCP).
Amphetamine increases, rather than decreases (choice C) serotonin
release.
Benzodiazepines and barbiturates potentiate the action of GABA at the
GABA
A
receptor (choice E).

Question 2 of 6
Use of the substance causing her symptoms is also associated with which of the
following?
/ A. Ataxia
/ B. Analgesia
/ C. Anorexia
/ D. Hypoactivity
/ E. Pruritus

Explanation - Q: 4.2 Close

The correct answer is C. Anorexia, euphoria, talkativeness, alertness, and
aggression are associated with amphetamine use. Visual and tactile
hallucinations may be produced by prolonged usage or overdose. Lateral
gaze nystagmus is associated with alcohol use or sleep deprivation.
Ataxia (choice A) is associated with alcohol and sedative/hypnotic
intoxication.
Analgesia (choice B) is associated with opioids and PCP use.
Hypoactivity (choice D) is associated with opioid and sedative/hypnotic use.
Pruritus (choice E) is associated with opioid use.

Question 3 of 6
Chronic use of this substance often mimics which of the following psychiatric
disorders?
/ A. AIzheimer disease
/ B. Attention deficit disorder
/ C. Borderline intellectual functioning
/ D. Conversion disorder
/ E. Schizophrenia

Explanation - Q: 4.3 Close

The correct answer is E. Amphetamines can produce paranoia and
delusions very similar to those seen in schizophrenia. However, visual
hallucinations, which are uncommon in schizophrenia, may be produced by
amphetamines.
Alzheimer disease (choice A) produces a progressive memory loss and a
decrease in executive functioning skills (balancing a checkbook, doing
laundry). It can be mimicked by depression or other (vascular, alcohol-
induced, or B12 deficiency) dementias.
Attention deficit disorder (choice B) is treated with amphetamines.
Borderline intellectual functioning (choice C) usually presents with school or
work difficulties.
Conversion disorder (choice D) presents with a neurologic symptom in the
context of a release from an intrapsychic conflict.


Question 4 of 6
Which of the following would increase the excretion of this drug?
/ A. Hyperbaric oxygen
/ B. Hyperventilation
/ C. Hypoventilation
/ D. Urine acidification
/ E. Urine alkalinization

Explanation - Q: 4.4 Close

The correct answer is D. Amphetamines are excreted by the kidneys. Urine
acidification greatly increases the excretion of amphetamines and
methamphetamines (weak bases).
Hyperbaric oxygen (choice A) is the treatment for carbon monoxide
poisoning.
Hyperventilation (choice B) may result in perioral tingling and can be
associated with anxiety.
Hypoventilation (choice C) can result with intoxication from alcohol,
barbiturates, benzodiazepines, or opioids.
Urine alkalinization (choice E) improves the excretion of acetaminophen and
barbiturates (weak acids).




Question 5 of 6
The patient develops hypertension. The etiology of this is related to which of the
following neurotransmitters?
/ A. Acetylcholine
/ B. Dopamine
/ C. Histamine
/ D. Norepinephrine
/ E. Serotonin

Explanation - Q: 4.5 Close

The correct answer is D. Norepinephrine in the periphery is a pressor and
increases blood pressure. Norepinephrine in the brain is associated with
sleep, mood, and energy. The euphoric and reinforcing effects of
amphetamine are thought to be related to increased release of dopamine in
the brain.
Acetylcholine (choice A) blockade in the brain is associated with
anticholinergic delirium (when blocked by anticholinergic drugs).
Dopamine (choice B) in the brain is associated with reward, motivation, and
psychosis.
Histamine (choice C) blockade in the brain is associated with weight gain
and sedation. Peripheral histamine is associated with the development of
gastric ulcers.
Serotonin (choice E) in the brain is associated with anxiety and depressive
disorders.

Question 6 of 6
Which of the following symptoms distinguishes intoxication with this drug from
intoxication with jimson weed ( Datura sp.) ?
/ A. Bronchoconstriction
/ B. Decreased gastrointestinal activity
/ C. Dry skin
/ D. Mydriasis
/ E. Tachycardia

Explanation - Q: 4.6 Close

The correct answer is C. Jimson weed (a Datura sp.), which has
anticholinergic activity, is used as a recreational hallucinogen by some
misguided individuals, and has lead to anticholinergic poisoning, and even
death. Many symptoms produced by amphetamine and anticholinergics are
similar, however, one distinguishing difference is diaphoresis. Amphetamine
causes diaphoresis, whereas anticholinergics produce a dry, warm skin.
Both amphetamine and jimson weed would be expected to cause
bronchodilation, not bronchoconstriction (choice A), due to sympathetic
activation and parasympathetic blockade, respectively.
Both amphetamine and jimson weed would be expected to decrease
gastrointestinal activity (choice B), due to sympathetic activation and
parasympathetic blockade, respectively.
Both amphetamine and jimson weed would be expected to produce
mydriasis (choice D), due to sympathetic activation (of the radial dilator ms.)
and parasympathetic blockade (of the ciliary ms.), respectively.
Both amphetamine and jimson weed would be expected to cause
tachycardia (choice E), due to sympathetic activation and parasympathetic
blockade, respectively.


A 28-year-old graduate student presents to the university health center
complaining of headache. She has had multiple episodes of severe
headache over the past three years. She describes the headache as a pounding
pain behind her eyes and along the lateral aspects of her
head. Prior to the headaches, she almost always sees small flashes of bright
light that form enlarging patterns, then clear over time. She often
feels nauseated during the headache and occasionally vomits. She has tried
multiple over-the-counter pain medications with minimal relief.
She has no other medical problems and takes no other medications. She denies
fever, weakness, or loss of sensation. Her vital signs are
normaI. Physical examination, including a full neurologic examination, is normaI.

Question 1 of 6

Which of the following is the most likely diagnosis?
/ A. CIuster headache
/ B. Meningitis
/ C. Migraine headache
/ D. Sinusitis
/ E. Tension headache

Explanation - Q: 1.1 Close

The correct answer is C. This patient is describing signs and symptoms of
classic migraine headache or migraine with aura. The aura is an episode of
transient neurologic symptoms that precede the headache. Auras are most
commonly visual, and include scotomas, scintillations, and visual field
defects. During the headache, common symptoms include nausea, vomiting,
and photophobia.
Cluster headache (choice A) describes a syndrome of a brief, very severe,
unilateral headache that lasts from ten minutes to less than two hours. The
headaches affect men more commonly than women and occur at night, often
awakening the patient from sleep. It typically starts as a burning sensation
over the lateral aspect of the nose and is associated with ipsilateral
conjunctival injection, lacrimation, nasal stuffiness, and Horner syndrome.
Meningitis (choice B) often presents with headache. The lack of fever,
however, suggests that the patient's headaches are not due to an infectious
etiology. In addition, the chronic nature of the headaches is not typical of
meningitis. Meningitis classically presents with an acute onset of headache
associated with fever, nuchal rigidity, and neurologic signs.
Sinusitis (choice D) is an inflammatory process that presents with headache
and pressure or pain typically over the frontal or maxillary sinuses.
Percussion of these sinuses can exacerbate the pain. Inflammation of the
ethmoid or sphenoid sinuses presents as a deep midline pain behind the
nose. Sinusitis is not associated with visual symptoms or nausea.
Tension headache (choice E) is a general term used to describe chronic
headaches of unclear pathophysiology that lack characteristic features of
migraine or cluster headache. Tension is thought to be the cause of these
headaches and may be related to contraction of neck and scalp muscles. It is
described as a nonthrobbing, bilateral, occipital head pain, which is not
associated with nausea or visual disturbances.

Question 2 of 6
A CT of the head in this patient would most likely show which of the following?
/ A. Air fluid levels in the sinuses
/ B. Contrast enhancement of the meninges
/ C. Normal findings
/ D. Posterior fossa tumor
/ E. Subarachnoid hemorrhage

Explanation - Q: 1.2 Close

The correct answer is C. There are no anatomic abnormalities associated
with migraine headaches, with or without aura. If a CT of the head were
performed in this patient, it would most likely demonstrate normal findings.
Air fluid levels in the sinuses (choice A) are seen in the setting of acute
sinusitis. It is not an expected finding in a patient with migraine headaches.
Contrast enhancement of the meninges (choice B) can be seen in the
setting of acute meningitis, other inflammatory processes involving the
meninges, and metastatic disease to the meninges. In many instances,
however, the head CT will be unremarkable. MRI of the brain is a more
sensitive diagnostic test to evaluate for meningeal enhancement in
suspected meningitis. There is no reason to expect this finding in a patient
with migraine headaches.
Posterior fossa tumors (choice D) are the most common brain tumors of
childhood and are much less common in adults. These tumors can present
with headache, nausea, and vomiting. This is not an expected finding in a
patient with classic signs and symptoms of migraine headache.
Subarachnoid hemorrhage (choice E) can be secondary to ruptured
aneurysm or trauma to the head. Patients with subarachnoid hemorrhage
present with acute onset of headache that they usually describe as the worst
headache of their life. There is no reason to expect this finding in a patient
with migraine headaches.

Question 3 of 6
The visual symptoms this woman experienced are thought to be the result of
localized decreased blood flow to the visual cortex. The visual
cortex is located in which of the following parts of the brain?
/ A. Brainstem
/ B. Frontal lobe
/ C. Occipital lobe
/ D. Parietal lobe
/ E. Temporal lobe

Explanation - Q: 1.3 Close

The correct answer is C. While the mechanism for the development of
migraines is still not well defined, it has been shown that the various types of
aura appear to be related to decreased blood flow to different areas of the
brain. Auras are transient, reversible neurologic defects that may produce
visual, somatosensory, motor, or language alterations. Visual auras are the
most common form, and may include flashing lights, scintillating scotoma,
and fortification spectrums. In the case of visual aura symptoms, decreased
blood flow to the visual cortex, located in the occipital lobe, at the occipital
pole of the cerebral hemispheres, has been demonstrated.
Associate the frontal lobe (choice B) with control of movements; the parietal
lobe (choice D) with receptive speech and the interpretation of sensation;
the temporal lobe (choice E) with hearing; and the brainstem (choice A) with
a large variety of basic body functions and reflexes.

Question 4 of 6
The patient's headache is interrupted using sumatriptan. This drug acts by
activation of which of the following?
/ A. AIpha adrenergic receptors
/ B. Beta adrenergic receptors
/ C. Cholinergic receptors
/ D. Dopamine receptors
/ E. Serotonin receptors

Explanation - Q: 1.4 Close

The correct answer is E. Sumatriptan is a prototype abortive drug used to
interrupt migraine headaches acutely. It activates serotonin receptors (5-
HT1d subtype) and has a 70% success rate in interrupting migraine
headaches. Sumatriptan ameliorates the entire symptom complex of
migraine, including headache, aura, nausea, vomiting, and photosensitivity.
Drugs with direct effects on alpha adrenergic receptors (choice A) and
cholinergic receptors (choice C) are not usually used in migraine therapy.
Beta blockers, but not agonists (choice B), such as propanolol are
sometimes used in migraine prophylaxis.
Dopamine antagonists, but not agonists (choice D), including
metoclopramide and prochlorperazine are sometimes used for abortive
therapy of migraines.



Question 5 of 6
A potential side effect of sumatriptan is which of the following?
/ A. Angina
/ B. Arrhythmia
/ C. Bradycardia
/ D. Gastrointestinal bleeding
/ E. Hypotension

Explanation - Q: 1.5 Close

The correct answer is A. Angina is a known side effect of sumatriptan and
the frequency of occurrence is reported to be approximately 5%. Sumatriptan
is a selective serotonin receptor agonist but can cause vasoconstriction in a
number of different parts of the body, including the extracranial vessels as
well as coronary arteries. As such, sumatriptan is contraindicated in patients
with ischemic heart disease and Prinzmetal's angina.
Arrhythmia (choice B) is not a known side effect of sumatriptan. It is a side
effect of amitriptyline, which is a drug used in the prophylaxis of migraine.
Bradycardia (choice C) is not a known side effect of sumatriptan. It is a side
effect of beta blockers, which are used in the prophylaxis of migraine.
Gastrointestinal bleeding (choice D) is not a known side effect of
sumatriptan. It is a side effect of nonsteroidal anti-inflammatory medications,
which are used for analgesia in migraine.
Hypotension (choice E) is not a known side effect of sumatriptan. It is a side
effect of some calcium channel blockers, such as verapamil, which can be
used in the prophylaxis treatment of migraines.






Question 6 of 6

The patient returns to the clinic three months later and reports that the
sumatriptan works welI. She says, however, that her headaches are
occurring more frequently and asks if there is a medication that can prevent the
headaches. An effective drug for prophylaxis is which of the
following?
/ A. Caffeine
/ B. Ergotamine
/ C. Meperidine
/ D. Prednisone
/ E. Propranolol

Explanation - Q: 1.6 Close

The correct answer is E. There are a number of effective migraine
prophylactic agents. These include beta blockers such as propranolol,
antidepressants such as amitriptyline, and anticonvulsants such as valproic
acid. Prophylactic medications should be considered for patients who
experience headaches two or more times a month, patients who experience
prolonged headaches, and for patients who are intolerant to their
medications for acute attacks.
Caffeine (choice A) is an ingredient that is found in several drugs that treat
the acute onset of migraine headache. It has no known role in the prevention
of migraine headaches.
Ergotamine (choice B) is a serotonin agonist and partial alpha agonist used
to treat migraine headaches, in the acute setting, by a similar mechanism to
sumatriptan. It has no known role in the prevention of migraine headaches.
Meperidine (choice C) is a narcotic analgesic that is used to treat the acute
onset of migraine headaches. It has no known role in the prevention of
migraine headaches.
Prednisone (choice D) is a corticosteroid that can be used to treat cluster
headaches. It has no known role in the prevention of migraine headaches.









A 70-year-old woman of Scandinavian descent consults a physician because she
has been having numerous headaches for the past several
months. These headaches began abruptly and increased in severity and duration
over a several week period. They are sometimes
accompanied by facial pain on the lateral aspect of her forehead. During the
period when the headaches first began, she experienced malaise
and fever. Physical examination is notable for a tender, thickened blood vessel
running cranially along her lateral temple anterior to and above
her ear.
Question 1 of 5
The involved blood vessel is most likely which of the following?
/ A. Facial artery
/ B. Lingual artery
/ C. Occipital artery
/ D. Posterior auricular artery
/ E. Superficial temporal artery

Explanation - Q: 2.1 Close

The correct answer is E. The artery is the superficial temporal artery, which
is often just called the temporal artery. It runs from the parotid gland upward
in front of the tragus of the ear together with the auriculotemporal nerve, and
divides into anterior and posterior branches that supply the temporal area of
the scalp.
The facial artery (choice A) arises below the corner of the jaw and then
crosses the mandible to run diagonally toward the nose.
The lingual artery (choice B) arises below the corner of the jaw and supplies
the tongue.
The occipital artery (choice C) and the posterior auricular artery (choice D)
both course backward behind the ear.

Question 2 of 5
The involved blood vessel is a branch arising directly from which of the following
blood vessels?
/ A. Basilar artery
/ B. Common carotid artery
/ C. External carotid artery
/ D. Internal carotid artery
/ E. Vertebral artery

Explanation - Q: 2.2 Close

The correct answer is C. The temporal artery is a terminal branch (together
with the posterior auricular artery) of the external carotid artery. The external
carotid artery supplies the external aspect of the face and head, and its
branches include the lingual artery, the facial artery, the superficial temporal
artery, the posterior auricular artery, and the occipital artery.
The basilar artery (choice A) arises from the union of the two paired
vertebral arteries (choice E); both supply the brainstem and the rest of the
brain through the Circle of Willis.
The common carotid artery (choice B) gives rise to the internal and external
carotid arteries.
The internal carotid artery (choice D) supplies the brain via the Circle of
Willis.


Question 3 of 5
Biopsy of the involved blood vessel would be most likely to show which of the
following?
/ A. Arteriolosclerosis
/ B. Giant cell arteritis
/ C. Polyarteritis nodosa
/ D. Takayasu arteritis
/ E. Wegener granulomatosis

Explanation - Q: 2.3 Close

The correct answer is B. The most likely diagnosis is giant cell arteritis,
which is characterized microscopically by granulomatous destruction, with
giant cell formation, of the wall of the vessel. The condition is also commonly
known as temporal arteritis, although this term is presently being
discouraged because the inflammatory process may involve many other
similar sized arteries both within and outside of the head. The clinical
presentation illustrated in the case summary is typical. The condition is fairly
uncommon (18 cases per 100,000 in the population aged 50 years or more),
and so will be suspected more often than proved. The diagnosis is
established by biopsy of a fairly long segment (2 cm or more) of the temporal
artery, since the lesion is spotty and may be missed with smaller biopsies.
(There is enough collateral blood supply to the scalp that distal infarction of
scalp tissues does not occur.)
Arteriolosclerosis (choice A) involves arterioles rather than larger vessels,
and is most commonly diagnosed in the kidney.
Polyarteritis nodosa (choice C) produces localized inflammation of blood
vessels in many sites in the body, and while it might possibly involve the
temporal artery, it does not have a particular predilection for doing so.
Takayasu arteritis (choice D) is a granulomatous involvement of the aorta
and its branches, and is most common in Asia or in people of Asian descent.
Wegener granulomatosus (choice E) would characteristically also produce
prominent lung involvement.

Question 4 of 5
Which of the following is the most serious complication of this disease process?
/ A. BIindness
/ B. Face and neck pain
/ C. Jaw claudication
/ D. Skin necrosis
/ E. Widespread vessel tenderness

Explanation - Q: 2.4 Close

The correct answer is A. Visual symptoms that can be seen in giant cell
arteritis include blurred vision, diplopia, visual hallucinations, and transient or
permanent blindness. These symptoms are thought to be related to the
involvement of the ciliary arteries and/or the central retinal artery. In large
part, because of the fear of recurrence with the possibility of permanent
blindness, temporal arteritis is treated with a prolonged steroid course that
may run for a year or longer. The conditions listed in the other choices can
also occur, but are not usually as serious as the risk of blindness.



Question 5 of 5
The patient has also been experiencing severe morning stiffness, which causes
her to have to "rolI" out of bed in the morning. Her shoulder
girdle and pelvic girdle are most strikingly involved. She experiences the pain as
"muscle pain," but later serum studies show no elevation of
the muscle marker creatine kinase. Physical examination for arthritis-related
findings is unremarkable, but her erythrocyte sedimentation rate
is found to be markedly high. Which of the following is the most likely diagnosis?
/ A. Gout
/ B. Osteoarthritis
/ C. Polymyalgia rheumatica
/ D. Rheumatoid arthritis
/ E. Still disease

Explanation - Q: 2.5 Close

The correct answer is C. There is a known association between giant cell
arteritis and polymyalgia rheumatica, and in fact, some authors claim that the
two conditions are actually different ends of the same disease spectrum. The
clinical description given in the question is typical. Polymyalgia rheumatica
appears to be much more common than giant cell arteritis, so patients with
giant cell arteritis are much more likely to have coexisting polymyalgia
rheumatica than vice versa.
Gout (choice A) usually appears clinically quite different, with obvious
involvement of one or a small number of joints.
While polymyalgia rheumatica is often misdiagnosed as osteoarthritis
(choice B), rheumatoid arthritis (choice D), or adult-onset Still disease (the
adult form of juvenile rheumatoid arthritis, choice E), the prominence of the
muscle complaints, the absence of obvious joint deformity, and the
predilection for involvement of shoulder and pelvic girdles should suggest the
correct diagnosis.

A 69-year-old man presents to the emergency department with a headache. He
states that the headache is the worst headache he has ever
had and has been constant for the past three hours. The patient has a past
medical history of hypertension and benign prostate hypertrophy.
Review of systems reveals a possible seizure two months ago. Medications
include atenolol and occasional ibuprofen. Vital signs are normaI.
Physical examination is notable for papilledema bilaterally and a clumsy gait.
Question 1 of 4
Which of the following is the most likely diagnosis?
/ A. Arteriovenous malformation
/ B. Ganglioglioma
/ C. GIioblastoma multiforme
/ D. Meningococcal meningitis
/ E. Metastatic renal cell carcinoma

Explanation - Q: 3.1 Close

The correct answer is C. Glioblastoma multiforme is the most common
brain tumor in adults. In adults, glioblastomas are noted most frequently in
the frontal lobe with the temporal lobe second in frequency. Childhood
glioblastomas of the cerebral hemispheres are also located most often in the
frontal lobe; with the second most frequent site being the parietal lobe.
Glioblastomas account for 50% of all gliomas and arise after age 50 in most
patients. Younger patients tend to have a better prognosis than the elderly.
Radiation and chemotherapy appear to extend the life of the patient.
Glioblastoma multiforme is the highest grade of astrocytoma, and may
present with papilledema, headaches, seizure, or personality changes. The
next step is to obtain diagnostic imaging studies such as a computed
tomography (CT) scan or magnetic resonance imaging (MRI) scan. The
tumor would typically be a large, irregular, necrotic, enhancing mass within
the brain parenchyma.
Arteriovenous malformations (choice A) are relatively uncommon brain
lesions that often present with seizures in patients less than forty years of
age.
Gangliogliomas (choice B) are rare, benign brain tumors that present with
seizures in patients less than forty years of age.
Meningitis (choice D) usually presents with photophobia, fever, and
headache. Meningitis would be on the differential diagnosis in this case.
Metastatic disease (choice E) is common in this older age group. Usually
there are signs or symptoms from the primary neoplasm first, but an isolated
brain metastasis may be the initial presentation. Renal cell carcinoma is a
much less common cause of a brain mass than glioblastoma multiforme.


Question 2 of 4
A CT scan with intravenous contrast shows a large, enhancing mass of the left
temporaI, frontal and parietal lobes. Biopsy of this lesion would
most likely show which of the following?
/ A. Atypical astrocytes with mild pleomorphism
/ B. BIepharoplasts in a sheet configuration
/ C. Normal astrocytes
/ D. Pseudopalisading astrocytes with necrosis
/ E. Tubules and rosettes of blepharoplasts

Explanation - Q: 3.2 Close

The correct answer is D. Markedly pleomorphic astrocytes in a
pseudopalisading configuration with necrosis is a classic appearance for
glioblastoma multiforme. There are often multiple gemistocytes (large
astrocytes) present as well.
Atypical astrocytes with mild pleomorphism (choice A) are characteristic of a
low grade astrocytoma, not a high grade astrocytoma, like glioblastoma
multiforme.
Blepharoplasts (choice B and E) are the key cells seen in ependymoma, a
less aggressive tumor of the ependyma, which lines the ventricles. Typically,
blepharoplasts are arranged in tubules and rosettes around blood vessels.
Astrocytes (choice C) are the most common cell in the normal brain. Normal
astrocytes make up only a small portion of the cells in a glioma

Question 3 of 4
Besides surgery, which of the following would be the most appropriate
pharmacotherapy in this patient?
/ A. Aspirin
/ B. Coumadin
/ C. Dexamethasone
/ D. Doxycycline
/ E. Heparin

Explanation - Q: 3.3 Close

The correct answer is C. A high potency steroid like dexamethasone or
prednisolone is indicated to lower intracranial pressure on the brain. A low
potency steroid like cortisone would not be effective. Steroids interrupt the
normal inflammatory cascade of the body, and thus reduce brain swelling
from causes such as a tumor or trauma. A low potency nonsteroidal anti-
inflammatory agent like aspirin or ibuprofen would not reduce intracranial
pressure to any measurable extent.
Aspirin (choice A) is a low potency anti-inflammatory agent that also has an
effect on platelets. It might help with headache, but would not relieve
symptoms of increased intracranial pressure.
Coumadin (choice B) is an oral anticoagulant that is contraindicated in a
patient with a brain tumor because of the risk of potentially fatal intracranial
hemorrhage within the tumor.
Doxycycline (choice D) is an antibiotic with no known role in the treatment of
brain tumors.
Heparin (choice E) is an intravenous anticoagulant that is contraindicated in
a patient with a brain tumor because of the risk of potentially fatal intracranial
hemorrhage within the tumor.

Question 4 of 4
If this patient was left untreated for six months, what would be the most likely
new presenting symptom?
/ A. Anosmia
/ B. Cardiac arrhythmias
/ C. Left hemiparesis
/ D. Right hemiparesis
/ E. Sudden death

Explanation - Q: 3.4 Close

The correct answer is D. Right hemiparesis is the most likely outcome,
because a frontoparietal lesion on the left would likely affect the motor strip
controlling the entire right side of the body. Specific body parts affected
would depend on the exact neural circuits damaged. The patient would likely
also experience sensory deficits on the right side of the body.
Anosmia (choice A) is characteristic of lesions in the inferior frontal lobes or
the bones of the anterior cranial fossa interfering with the first cranial nerves,
which convey the sense of smell.
Lesions affecting the autonomic centers of the medulla, or a lesion of the
pituitary causing an electrolyte imbalance, could conceivably cause an
arrhythmia, but cardiac arrhythmias (choice B) are most commonly caused
by primary dysfunction of the cardiac conduction system or electrolyte
imbalances, rather than brain lesions.
Left hemiparesis (choice C) would arise from a lesion of the right frontal
region.
Sudden death (choice E) is a rare effect of brain tumors. A tumor would
have to compress the medulla (directly, or via a mass effect) to stop
respiration or cardiac activity. A posterior cranial fossa mass lesion could
cause sudden death from cerebral herniation.

A 39-year-old woman presents to the emergency department after collapsing at a
party. An interview with her boyfriend indicates that she
complained of a severe headache prior to her collapse. He states that she has no
significant past medical history and takes occasional
vitamin supplements. Her blood pressure is 200/120 mm Hg, pulse is 37/min,
and respirations are 5/min. The patient is unresponsive to
commands or painful stimuli. There is moderate papilledema. The remainder of
the examination is unremarkable. An electrocardiogram
demonstrates normal sinus rhythm without T wave inversions or ST segment
changes.
Question 1 of 5
Which of the following is the most likely diagnosis?
/ A. Anterior communicating artery aneurysm rupture
/ B. Atonic seizure
/ C. Cocaine induced myocardial infarction
/ D. Posterior inferior cerebellar artery aneurysm rupture
/ E. Vein of Galen malformation

Explanation - Q: 4.1 Close

The correct answer is A. This patient is presenting with a loss of
consciousness, bradycardia, hypertension, and decreased respirations.
While loss of consciousness has a wide differential diagnosis, the triad of
bradycardia, hypertension, and decreased respirations is known as
Cushing's triad, and is indicative of increased intracranial pressure. The
finding of papilledema confirms that there is increased intracranial pressure.
The differential diagnosis at this point is a spontaneous hemorrhage due to
aneurysm rupture, trauma, vascular malformation rupture, or possibly a
massive ischemic stroke. Of the choices given, anterior communicating
artery aneurysm rupture is the most likely diagnosis. Aneurysms are
outpouchings of the arteries of the Circle of Willis that occur most commonly
at the anterior communicating artery, middle cerebral artery, or posterior
communicating artery. They most commonly present with hemorrhage or
headache. In this case, there is likely hemorrhage and increased intracranial
pressure leading to secondary brain herniation. Treatment of this patient
consists of lowering intracranial pressure and treating the aneurysm
surgically.
An atonic seizure (choice B) is a fainting spell in which the patient becomes
hypotonic, but recovers over a short interval. There would be no signs of
increased intracranial pressure.
Myocardial infarction (choice C) from cocaine or other etiology would usually
have electrocardiogram abnormalities and there would be no signs of
increased intracranial pressure.
Posterior inferior cerebellar artery aneurysm rupture (choice D) is a
possibility but these aneurysms are rare, compared to anterior
communicating artery aneurysms.
Vein of Galen malformations (choice E) are a remnant of the fetal circulation
that presents in children as a posterior fossa mass. Actual hemorrhage of
these lesions is relatively rare.

Question 2 of 5
Which of the following conditions would predispose this patient to having this
condition?
/ A. Atherosclerosis
/ B. Diabetes
/ C. Hemophilia
/ D. Marfan syndrome
/ E. Protein C deficiency
/ F. Protein S deficiency



Explanation - Q: 4.2 Close

The correct answer is D. Connective tissue diseases, such as Marfan
syndrome, weaken blood vessel walls and predispose to aneurysms of any
blood vessels in the body. Marfan syndrome is an autosomal dominant
disorder that has been linked to the FBN1 gene on chromosome 15. FBN1
encodes the protein fibrillin, which is involved in the formation of elastic fibers
found in connective tissue. Without the structural support provided by fibrillin,
many tissues are weakened, with severe consequences, e.g., aneurysm
formation.
Atherosclerosis (choice A) is not thought to be associated with intracranial
aneurysms, which are believed to form from congenitally weak areas at the
junctions of blood vessels. Aortic aneurysms are closely associated with
atherosclerosis.
Diabetes (choice B) is not thought to be associated with intracranial
aneurysms. However, diabetes leads to an increased incidence of
atherosclerosis, which may lead to aortic aneurysms.
The hemophilias (choice C) are blood clotting disorders that do not
predispose patients to aneurysms. These patients bleed profusely from even
minor vessel trauma, however.
Protein C deficiency (choice E) and protein S deficiency (choice F) are
blood clotting disorders leading to thrombosis of arteries and veins. This
does not predispose a patient to aneurysms, however.

Question 3 of 5
A CT scan would most likely demonstrate blood in which of the following areas?
/ A. Fourth ventricle
/ B. Lateral ventricles
/ C. Subarachnoid space
/ D. Subdural space
/ E. Superior sagittal sinus
/ F. Third ventricle

Explanation - Q: 4.3 Close

The correct answer is C. The subarachnoid space consists of the space
between the pia, which adhere to the brain, and the arachnoid membrane.
The circle of Willis, including the anterior communicating artery, lies in the
subarachnoid space. Subarachnoid hemorrhage is a common presenting
symptom of ruptured intracranial aneurysms. Aneurysmal subarachnoid
hemorrhage is usually within the basilar cisterns, where the circle of Willis
lies, while posttraumatic subarachnoid hemorrhage is usually over the
cerebral convexities. Hemorrhage into the epidural or subdural space is
usually secondary to trauma. Epidural hematomas occur from injury to the
middle meningeal artery and subsequent hematoma formation, and are
usually associated with a fracture of the temporal bone.
Intraventricular hemorrhage (choices A, B, and F) is a much less common
presentation of a ruptured aneurysm. Usually there will be subarachnoid
hemorrhage and intraventricular hemorrhage, rather than isolated
intraventricular hemorrhage. Intraventricular hemorrhage often leads to
ependymitis and hydrocephalus from dysregulation of the normal
cerebrospinal fluid production and resorption physiology.
Subdural hematomas (choice D) are usually secondary to trauma, not
bleeding aneurysms. Subdural hematomas form from injury to the bridging
veins between the venous sinuses and the cortical draining veins. Subdurals
are common in elderly patients because they usually have some degree of
brain atrophy and these bridging veins are stretched thin.
There is normally blood present in the superior sagittal sinus (choice E),
which drains the cortical veins from the top of the cerebrum.

Question 4 of 5
Which of the following drugs could have precipitated this patient's condition?
/ A. Cocaine
/ B. Hashish
/ C. Lysergic acid diethylamide (LSD)
/ D. Morphine
/ E. Pindolol


Explanation - Q: 4.4 Close

The correct answer is A. The key here is to find the drug that leads to
hypertension, and thus is likely to cause an aneurysm to rupture. Cocaine
leads to episodic hypertension due to its sympathomimetic effects. It may be
snorted, smoked, or injected. Cocaine use is associated with cardiac
arrhythmia, myocardial infarction, stroke, and cerebral or aortic aneurysm
rupture. Although not a cause of intracranial aneurysm formation, it may lead
to aneurysm rupture. Cocaine is used for its central effects on dopaminergic
neurons, and the sympathomimetic effects described above are unwanted
side effects.
Hashish (choice B) and marijuana contain delta-9- tetrahydrocannabinol
(THC), which is used for its effects on the central nervous system. Other
physical effects include reddening of the eyes, dryness of the mouth and
throat, moderate increase in the heart rate, tightness of the chest (if the drug
is smoked), drowsiness, unsteadiness, and muscular incoordination.
Hypertension is not a common effect of THC.
Lysergic acid diethylamide (LSD) (choice C) is a psychotropic amide with
many poorly-understood central nervous system effects. Significant
hypertension does not generally occur with LSD.
Morphine (choice D) is an opiate analgesic, and would tend to lower blood
pressure, rather than increase it.
Pindolol (choice E) is a nonselective beta-adrenergic receptor blocker. In
addition, pindolol has partial agonist activity, with significantly greater agonist
than antagonist effects at beta-2 receptors. It has negative inotropic and
chronotropic effects and thus is used as an antihypertensive agent. It would
help prevent hypertension.



Question 5 of 5
Which of the following is more likely to be present in patients with this condition
than in normal persons?
/ A. Early AIzheimer disease
/ B. Fronto-temporal brain atrophy
/ C. Medullary thyroid carcinoma
/ D. Osteosarcoma
/ E. Renal cysts

Explanation - Q: 4.5 Close

The correct answer is E. Patients with adult polycystic kidney disease have
a much higher incidence of berry aneurysms than the general population.
Hypertension that may accompany the eventual renal failure can contribute
to aneurysm rupture and subarachnoid hemorrhage.
Early Alzheimer-like changes (choice A) are observed in patients with Down
syndrome.
Fronto-temporal brain atrophy (choice B) is seen in Pick disease.
Medullary thyroid carcinoma (choice C) is seen with increased frequency in
multiple endocrine neoplasia (MEN) IIa and IIb.
Osteosarcoma (choice D) is more frequent in patients with familial
retinoblastoma.


Question 5 of 5
Which of the following is more likely to be present in patients with this condition
than in normal persons?
/ A. Early AIzheimer disease
/ B. Fronto-temporal brain atrophy
/ C. Medullary thyroid carcinoma
/ D. Osteosarcoma
/ E. Renal cysts

Explanation - Q: 4.5 Close

The correct answer is E. Patients with adult polycystic kidney disease have
a much higher incidence of berry aneurysms than the general population.
Hypertension that may accompany the eventual renal failure can contribute
to aneurysm rupture and subarachnoid hemorrhage.
Early Alzheimer-like changes (choice A) are observed in patients with Down
syndrome.
Fronto-temporal brain atrophy (choice B) is seen in Pick disease.
Medullary thyroid carcinoma (choice C) is seen with increased frequency in
multiple endocrine neoplasia (MEN) IIa and IIb.
Osteosarcoma (choice D) is more frequent in patients with familial
retinoblastoma.



A forty-year-old woman presents to the emergency department complaining of
two days of severe headache, fever, and stiff neck. On
examination, the patient displays nuchal rigidity and Brudzinski's sign.

Question 1 of 5

Which of the following is the most likely diagnosis?
/ A. Embolic stroke
/ B. Guillain-Barr syndrome
/ C. Hemorrhagic stroke
/ D. Meningitis
/ E. Vascular headache



Explanation - Q: 5.1 Close

The correct answer is D. Meningitis is characterized by fever, headache,
nuchal rigidity, and CNS dysfunction including confusion, delirium, lethargy,
coma, and cranial nerve dysfunction. Brudzinski's sign indicates meningeal
irritation: as the patient's neck is flexed, the patient flexes the hip and knee.
Choices A and C are incorrect. A stroke is a vascular accident and would
thus have an acute onset. Focal neurologic findings would be elicited. Fever
would not be found in stroke.
Guillain-Barr syndrome (choice B) is a peripheral neuropathy, and thus
peripheral, rather than central nervous system findings, would be present.
Vascular headache (choice E) presents as severe headache, often
throbbing, which is accompanied by nausea and photophobia. Visual aura
may precede the headache, and focal neurologic findings may complicate
the course. Fever is not present.


Question 2 of 5

Lumbar puncture is performed and the cerebrospinal fluid is examined. The fluid
is turbid, and laboratory findings include elevated opening
pressure, neutrophilic pleocytosis, markedly elevated protein, and decreased
glucose. Which of the following is the most likely etiologic
agent?



Explanation - Q: 5.2 Close

The correct answer is E. The CSF analysis greatly aids in finding the
etiology of meningitis. In bacterial meningitis, the CSF has decreased
glucose, elevated protein, and a proliferation of neutrophils. In addition, a
Gram's stain should be performed and will often reveal and allow
characterization of the bacteria. S. pneumoniae is a common bacterial
pathogen seen in this patient's age group. Antibiotic therapy to cover this
organism can now be started pending cultures and sensitivity.
Cryptococcus neoformans(choice A) is responsible for chronic fungal
meningitis. The CSF would have lymphocytes with only a slight elevation in
protein, with normal or slightly decreased glucose.
Viral meningitis, from e.g., Herpes simplex (choice B), would produce a
slight elevation in CSF lymphocytes, normal glucose, and slightly elevated
protein.
Listeria monocytogenes(choice C) is a bacterial pathogen, but it is found in
the pediatric population, and thus would be extremely unlikely in this patient.
Tuberculous meningitis (choice D) has an indolent course and a delayed
onset. The CSF in TB meningitis would have slight lymphocytic elevation,
markedly elevated protein, and slightly decreased glucose.

Question 3 of 5
Cerebrospinal fluid glucose concentration is normally approximately what fraction
of serum glucose concentration?
/ A. 1/3
/ B. 1/2
/ C. 2/3
/ D. 1
/ E. 4/3

Explanation - Q: 5.3 Close

The correct answer is C. CSF glucose is derived from serum, and is a
reflection of the serum concentration during the previous 2-4 hours. Glucose
is often abnormally low in cases of bacterial meningitis. In normal-pressure
hydrocephalus, CSF glucose is usually normal. Normally, CSF glucose is
about 2/3 of the serum glucose concentration.



Question 4 of 5
Which of the following would be the most appropriate pharmacotherapy?
/ A. Acyclovir
/ B. Ceftriaxone
/ C. Cephazolin
/ D. Penicillin
/ E. Valicyclovir

Explanation - Q: 5.4 Close

The correct answer is B. When selecting an antimicrobial, it is important to
select an agent that is effective against the likely organisms and will also
penetrate the blood-brain barrier. Ceftriaxone will be effective against the
likely organisms causing bacterial meningitis in a 40-year-old (Neisseria
meningitidis and Streptococcus pneumoniae). In addition, it can cross the
blood-brain barrier and access the CNS.
Acyclovir (choice A) and valicyclovir (choice E) are antivirals and would be
inappropriate in bacterial meningitis.
Cephazolin (choice C) and penicillin (choice D) are effective against gram-
positive organisms, but they do not cross the blood-brain barrier.


Question 5 of 5
Which of the following best describes the order of meningeal layers from the skull
to the cerebral cortex?
/ A. Arachnoid, dura mater, pia mater
/ B. Arachnoid, pia mater, dura mater
/ C. Dura mater, arachnoid, pia mater
/ D. Dura mater, pia mater, arachnoid
/ E. Pia mater, dura mater, arachnoid

Explanation - Q: 5.5 Close

The correct answer is C. The dura mater is the most substantial layer of the
meninges and is the most distal from the brain. The next layer is the
arachnoid layer. The pia is a thin tissue layer applied directly to the surface
of the brain. Cerebrospinal fluid is found between the arachnoid and pia
layers.

A pediatrician sees a 4-month-old boy for the first time. He had been delivered at
home by his maternal grandmother, who had been a midwife
in Southeast Asia before the family immigrated to the United States. According to
the mother, the baby had been born on time and had
weighed about 7 pounds at birth. Physical examination is remarkable for a
continuous murmur heard best at the upper left sternal border. A
thrilI, analogous to a kitten's purring, can be felt over the left side of the baby's
chest.

Question 1 of 6

The infant's murmur is suggestive of which of the following diagnoses?
/ A. Coarctation of the aorta
/ B. Hypoplastic left ventricle
/ C. Mitral valve stenosis
/ D. Patent ductus arteriosus
/ E. Tricuspid valve stenosis

Explanation - Q: 1.1 Close

The correct answer is D. This is the murmur that is characteristic of patent
ductus arteriosus.
Coarctation of the aorta (choice A) may produce a soft bruit heard over the
coarctation site, often heard best in the back.
Hypoplastic left ventricle (choice B) per se does not usually produce a
murmur, although a compensating patent ductus arteriosus or septal defect
may produce a murmur.
Mitral valve stenosis (choice C) can produce an rumbling apical diastolic
murmur.
Tricuspid valve stenosis (choice E) can produce a presystolic murmur heard
at the left sternal edge in the 4th intercostal space.


Question 2 of 6
The infant's pulse at the wrist is noted to be full and to have a widened pulse
pressure. On chest x-ray, the left atrium, Ieft ventricle, and
ascending aorta are enlarged, and there is increased pulmonary blood flow. This
infant's murmur is most likely related to which of the
following?
/ A. BIood flowing across the aortic valve
/ B. BIood flowing from the aorta to the pulmonary artery
/ C. BIood flowing from the left ventricle to the right ventricle
/ D. BIood flowing from the pulmonary artery to the aorta
/ E. BIood flowing from the right ventricle to the left ventricle

Explanation - Q: 1.2 Close

The correct answer is B. The ductus arteriosus is an important prenatal
vessel that connects the aorta to the pulmonary artery, and allows blood in
the (non-breathing) fetus to bypass the lungs. This vessel normally closes
within a few hours to days of birth. In a baby without other congenital cardiac
malformations, the blood flowing through a patent ductus arteriosus flows
from the aorta to the pulmonary artery. In babies with other congenital
cardiac malformations, the blood may flow in either direction (including
choice D), depending on the specific malformation present. In this case, the
widened pulse pressure, the enlarged chambers on the left side of the heart,
and the increased pulmonary blood flow all suggest that the blood is flowing
from the systemic circulation to the pulmonary system.
Blood flow across the aortic valve (choice A) per se would not cause
increased pulmonary blood flow.
Blood flowing through a ventricular septal defect (choices C and E) would
tend to produce enlargement of both ventricles without ascending aorta
enlargement.


Question 3 of 6
Which of the following sets of changes depict the blood oxygen tensions at
various locations in this patient compared to those of a healthy
infant?



Explanation - Q: 1.3 Close

The correct answer is A. Oxygen-rich blood from the aorta flows through
the patent ductus arteriosus, increasing the oxygen tension in the pulmonary
artery. Blood becomes fully oxygenated in the lungs in a normal way so that
the oxygen tension of blood in the left ventricle, aorta, and vena cava is
normal.


Question 4 of 6
This baby's lesion accounts for approximately what percentage of congenital
heart defects?
/ A. 1%
/ B. 10%
/ C. 25%
/ D. 50%
/ E. 95%

Explanation - Q: 1.4 Close

The correct answer is B. Patent ductus arteriosus accounts for
approximately 10% of all congenital heart defects. Many of the affected
babies are premature. Other associations include birth asphyxia, rubella,
coarctation of the aorta, ventricular septal defect, and trisomies 18 and 21.



Question 5 of 6
If this baby's problem had been identified in his first week of life, which of the
following medications might have been used to try to medically
correct his problem?
/ A. Acetaminophen
/ B. Codeine
/ C. Hydrocortisone
/ D. Indomethacin
/ E. Morphine

Explanation - Q: 1.5 Close

The correct answer is D. Before closing a patent ductus arteriosus, it is
important to establish that the ductus is not compensating for other cardiac
malformations, since a patent ductus arteriosus is beneficial in some other
congenital lesions, including tetralogy of Fallot, Eisenmenger syndrome due
to right ventricular hypertrophy, a large interventricular septal defect, or an
interrupted aortic arch (in which the patent ductus supplies blood to the distal
aorta). Indomethacin is the drug most commonly used to treat a patent
ductus arteriosus, and it is usually used in the first few weeks of life. An
alternative drug sometimes used is ibuprofen. These drugs act by inhibiting
by decreasing the activity of cyclooxygenase and thereby inhibiting the
production of prostaglandins. The drugs cause the ductus to narrow, and
may have decreasing urine output as a side effect. If the baby is at risk of
pulmonary edema, diuretics and/or modest fluid restriction may also be
added. The indomethacin course may need to be repeated if it is not initially
effective. In babies in whom medical treatment was ineffective or in whom
the diagnosis was made late, surgical correction by either ligation of the
vessel or implanting of a coil within the vessel is often used.
Acetaminophen (choice A), codeine (choice B), and morphine (choice E)
are analgesics used for pain control that do not have any effect on the ductus
arteriosus.
Hydrocortisone (choice C) is a corticosteroid and would not be used to close
a patent ductus.


Question 6 of 6
Untreated patients with this baby's condition have been associated with which of
the following mortality rates by 20 years of age?
/ A. 5%
/ B. 20%
/ C. 50%
/ D. 80%
/ E. 95%

Explanation - Q: 1.6 Close

The correct answer is B. While patients with very small amounts of blood
flow through a patent ductus arteriosus may do fine throughout life, you
should be aware that this is not a completely benign condition. Complications
of patent ductus arteriosus include left heart failure, pulmonary hypertension,
right heart hypertrophy and failure, bacterial endocarditis, myocardial
ischemia, and necrotizing enterocolitis. It is estimated that the untreated
mortality rate by age 20 years is 20%. Additionally, there is an estimated
untreated mortality rate of 42% by age 45 and 60% by age 60.


A 65-year-old man presents to his primary care physician complaining of
dyspnea, chest pain, and several syncopal episodes. His symptoms
have worsened over the past few months and his third syncopal episode
prompted this visit. On examination, a systolic ejection murmur is
auscultated with an ejection click in the right second intercostal space. Rales are
present at the lung bases. He has a history of rheumatic fever
in his twenties.
Question 1 of 4
Which of the following is the most likely diagnosis?
/ A. Aortic regurgitation
/ B. Aortic stenosis
/ C. Mitral stenosis
/ D. Tension pneumothorax
/ E. Thoracic aortic dissection

Explanation - Q: 2.1 Close

The correct answer is B. Exertional dyspnea, angina pectoris, and syncope
are the cardinal symptoms of aortic stenosis. Exam findings of systolic
ejection murmur with ejection click confirm this diagnosis. Patients
experience dyspnea because of the pulmonary edema generated by
increased pulmonary capillary pressure, transmitted from increased left heart
pressures. Angina pectoris results from left ventricular hypertrophy and
increased interventricular pressures. Thus, an increased myocardial mass
increases myocardial oxygen demand, while the increased wall pressures
decrease perfusion. Ischemia results. Syncope results when the impeded left
ventricle cannot meet peripheral perfusion demands.
Patients would have a diastolic murmur with aortic regurgitation (choice A).
Patients will have fatigue and dyspnea with mitral stenosis (choice C), but
they have a diastolic murmur.
Patients with tension pneumothorax (choice D) present emergently with
acute onset of shortness of breath and hemodynamic instability.
Thoracic aortic dissection (choice E) presents as acute onset of "tearing"
chest pain that radiates to the back. Murmur may also be present.


Question 2 of 4
Patients with syncope cannot maintain sufficient cardiac output to meet
peripheral perfusion demands. Which of the following best describes
cardiac output?
/ A. Cardiac output = end diastolic volume - end systolic volume
/ B. Cardiac output = heart rate X mean arterial pressure
/ C. Cardiac output = heart rate X stroke volume
/ D. Cardiac output = stroke volume X mean arterial pressure
/ E. Cardiac output = systemic vascular resistance X mean arterial pressure

Explanation - Q: 2.2 Close

The correct answer is C. Cardiac output = heart rate X stroke volume.
Choice A is incorrect: Stroke volume = end diastolic volume - end systolic
volume
Choice B is incorrect: Double product (estimation of cardiac work) = mean
arterial pressure X heart rate
Choices D and E are nonsense distracters.


Question 3 of 4
Which of the following might explain the angina pectoris in this patient
/ A. Increased ventricular wall tension limits perfusion
/ B. Left ventricular hypertrophy accelerates atherosclerotic diseas
/ C. Pulmonary hypertension decreases the PO2 of arterial blood
/ D. Stenotic valves occlude the coronary arteries
/ E. Twisting of the heart on its axis limits coronary flow

Explanation - Q: 2.3 Close

The correct answer is A. The angina pectoris seen in aortic stenosis is
caused by left ventricular hypertrophy. The ventricle must generate greater
pressures to overcome the occluded outflow tract, and hypertrophy occurs.
This contributes to cardiac ischemia in several ways. As noted above, an
increased myocardial mass increases myocardial oxygen demand, while the
increased wall tension decreases perfusion. The myocardium is perfused
during diastole, and coronary perfusion relies on this relaxation. When mural
diastolic pressures remain elevated in hypertrophy, perfusion is limited and
ischemia results.
Hypertrophy does not accelerate atherosclerosis (choice B).
Pulmonary hypertension (choice C), seen in aortic stenosis, results in
pulmonary edema and a thus a mild hindrance to alveolar gas exchange.
One would not expect the PO
2
to decrease enough to cause angina.
Stenotic valves do not occlude the coronary arteries (choice D).
The heart does not twist to occlude the coronary arteries (choice E).

- - -

Question 4 of 4
Which of the following sets of changes depict the mean arterial pressure (MAP),
Ieft ventricular peak systolic pressure (LVPSP), pulmonary
wedge pressure (PWP), and left atrial pressure (LAP) in this patient, compared to
a healthy individuaI?



Explanation - Q: 2.4 Close

The correct answer is A. In aortic stenosis, the blood is ejected from the left
ventricle through a smaller than normal opening. Because the resistance to
ejection of blood is high, the left ventricular peak systolic pressure can
sometimes increase to over 250 mm Hg with normal pressures in the aorta.
The increase in left ventricular pressure raises left atrial pressure as well as
pulmonary wedge pressure (which is a clinical index of left atrial pressure).


Question 1 of 4
Which of the following is the most likely diagnosis?
/ A. Aortic regurgitation
/ B. Infective endocarditis
/ C. Mitral regurgitation
/ D. Mitral stenosis
/ E. Myocardial infarction

Explanation - Q: 3.1 Close

The correct answer is C. Mitral regurgitation traditionally presents as fatigue
and exertional dyspnea. The history of myocardial infarction suggests that
the origin of the regurgitation is papillary muscle dysfunction. The exam
findings of midsystolic murmur at the apex also support this diagnosis. The
JVD, rales, and lower extremity edema indicate that the condition has
advanced to produce congestive heart failure.
Aortic regurgitation (choice A) will present with similar symptoms, but will
have a diastolic murmur.
There is nothing in this patient's history to suggest infective endocarditis
(choice B). Patients may have a murmur, but it would be accompanied by
fever, chills, and leukocytosis.
Mitral stenosis (choice D) would present as fatigue and dyspnea, but it
would also generate diastolic murmur.
While this patient has a history of MI (choice E) and another infarction is
always possible, the patient has had no acute symptoms of MI (e.g., chest
pain or pressure, jaw pain radiating down the left arm, diaphoresis).

Question 2 of 4
The filamentous structures that connect the valve leaflets to the papillary muscle
are which of the following?
/ A. Bundle of His
/ B. Chordae tendineae
/ C. Crista galli
/ D. Purkinje fibers
/ E. Trabeculae carneae

Explanation - Q: 3.2 Close

The correct answer is B. The filamentous structures that connect the valve
leaflets to the papillary muscle are the chordae tendineae. They are integral
in valve function as the papillary muscle contracts during systole, pulling on
the chordae. This pulling prevents the valve leaflets from everting into the
atrium, which would lead to regurgitation.
The Bundle of His (choice A) conducts electrical impulses through the atrial
septum, from the SA node to the to the AV node.
The crista galli (choice C) is a bony projection in the cribriform plate of the
skull.
Purkinje fibers (choice D) conduct electrical impulses from the AV node
down the interventricular septum.
Trabeculae carneae (choice E) are the irregular folds and ridges of the
myocardium.


Question 3 of 4
Valvular damage and altered flow dynamics put this patient at risk for infective
endocarditis. If blood cultures are positive for Staphylococcus Aureus, which of
the following would best treat this infection?
/ A. Amphotericin B
/ B. Cephazolin
/ C. Gentamicin
/ D. Nafcillin
/ E. Penicillin G
Explanation - Q: 3.3 Close

The correct answer is D. Staphylococcus aureus is a gram-positive coccus,
and therapy should be directed accordingly. Beta lactam antibiotics interfere
with cell wall synthesis in gram-positive bacteria, but Staphylococcus aureus
has the enzyme beta-lactamase, which makes it resistant to many beta
lactam antibiotics. Thus, one needs a beta lactamase-resistant penicillin,
such as nafcillin, to treat this infection.
Amphotericin B (choice A) is an antifungal agent and thus would not be
effective in this case.
Cephazolin (choice B) is a first generation cephalosporin, and thus is
effective against gram-positive organisms. It, however, is susceptible to beta
lactamases possessed by Staph. aureus, making it a poor choice for this
infection.
Gentamicin (choice C) is an aminoglycoside, and is used to treat gram-
negative infections.
Penicillin (choice E) is effective against gram-positive organisms, but is also
susceptible to beta lactamase.



Question 4 of 4

Which of the following sets of changes depict the left atrial pressures (LAP) at the
end of ventricular systole and at the end of ventricula
diastole of this patient, compared to a healthy individuaI?



Explanation - Q: 3.4 Close

The correct answer is B. Mitral regurgitation is characterized by a greatly
elevated left atrial pressure toward the end of systole caused by backward
flow of blood from the left ventricle into the left atrium through the leaky mitral
valve. The left atrial pressure is normal at the end of diastole with mitral
regurgitation because blood flows unimpeded from the atrium into the
ventricle when the mitral valve is open.




A 46-year-old woman presents to the emergency department with fever, chills,
and chest discomfort for 2 weeks. She admits to a 20 year
history of intravenous drug use, and her last use was 3 weeks ago. Her
temperature is 38.3 C (101 F), blood pressure is 120/52 mm Hg, pulse
is 120/min, and respirations are 26/min. Her jugular venous pulse is normaI, but
there are bibasilar crackles on lung examination. Cardiac
examination reveals a rapid but regular rhythm and a new decrescendo, blowing
diastolic murmur heard best over the left sternal border. The
electrocardiogram shows sinus tachycardia.

Question 1 of 7

Which of the following is the most likely cause of her cardiac murmur?
/ A. Aortic regurgitation
/ B. Aortic stenosis
/ C. Mitral regurgitation
/ D. Mitral stenosis
/ E. Mitral valve prolapse

Explanation - Q: 4.1 Close

The correct answer is A. The patient's murmur is typical of an aortic
regurgitation murmur. Aortic regurgitation, also known as aortic insufficiency,
can be caused by rheumatic heart disease, syphilitic aortitis, endocarditis, or
an aortic aneurysm. It is due to retrograde blood flow from the aorta into the
left ventricle through an incompetent aortic valve.
Aortic stenosis (choice B) would result in a systolic ejection murmur heard
best at the right second intercostals space. It can be caused by a congenital
bicuspid valve, age-related degenerative changes, or rheumatic heart
disease.
Mitral regurgitation (choice C) presents as a holosystolic murmur best heard
over the apex. It is caused by rheumatic heart disease, mitral valve prolapse,
endocarditis, or papillary muscle damage from a myocardial infarction.
Mitral stenosis (choice D) produces a low-pitched diastolic murmur heard
best over the apex. It is often preceded by an opening snap. It is usually due
to rheumatic heart disease.
Mitral valve prolapse (choice E) is due to the stretching of the posterior
mitral valve leaflets resulting in prolapse of the valve. This change produces
the classic midsystolic click that is sometimes is followed by a murmur.


Question 2 of 7
This valvular abnormality is also associated with which of the following signs?
/ A. Apical diastolic murmur
/ B. Ghon complex
/ C. Narrow pulse pressures
/ D. Opening snap
/ E. Systolic crescendo-decrescendo murmur

Explanation - Q: 4.2 Close

The correct answer is A. The Austin Flint murmur is caused by contact of
the aortic regurgitant jet with the left ventricular wall. It is a low-pitched, apical
diastolic murmur.
A Ghon complex (choice B) occurs in the setting of a primary tuberculosis
infection or exposure to tuberculosis. These are tuberculous granulomas that
occur in the lung with lobar or perihilar lymph node involvement.
In aortic regurgitation, the pulse pressure is widened, not narrowed (choice
C). The narrow pulse pressure is due to the regurgitation of blood back into
the left ventricle during diastole.
An opening snap (choice D) occurs in mitral stenosis. It precedes the low-
pitched diastolic murmur associated with mitral stenosis.
A systolic crescendo-decrescendo murmur (choice E) occurs in aortic
stenosis. The time at which the murmur peaks indicates the severity of the
lesion. Murmurs that peak late in systole are indicative of more severe
stenosis.


Question 3 of 7
What is the most likely underlying cause of the patient's fever, chest discomfort,
and new murmur?
/ A. Aortic dissection
/ B. Endocarditis
/ C. Myocardial infarction
/ D. Pulmonary embolus
/ E. Tension pneumothorax
Explanation - Q: 4.3 Close

The correct answer is B. The presence of a new murmur in the setting of a
patient with fevers and a history of intravenous drug abuse strongly suggests
endocarditis. Positive blood cultures and an echocardiogram showing
vegetations would be diagnostic.
Aortic dissection (choice A) would present as sudden onset of severe chest
pain, which often radiates to the back. Patients can have hypotension,
depending on the severity of the dissection. Patients can also have unequal
pulses in their extremities if the dissection affects one of the major arteries
branching off the aortic arch. Patients can develop aortic regurgitation, but
they are more likely to dissect into the pericardial sac and develop cardiac
tamponade. Aortic dissection is also not associated with fevers and a new
murmur.
A myocardial infarction (choice C) usually presents with severe squeezing
left-sided chest pain that can radiate down the left arm. Patients are
generally middle-aged and can have risk factors for cardiac disease such as
hypertension, diabetes, hypercholesterolemia, or a history of tobacco use.
The electrocardiogram can vary from nonspecific T wave changes to ST
segment elevation.
Pulmonary embolus (choice D) can present with chest pain that is pleuritic in
nature. It is not associated with fevers or a new murmur. Also, patients often
have risk factors for a hypercoagulable state.
Tension pneumothorax (choice E) can present with chest pain, but it is not
associated with fevers or a new murmur. Furthermore, there should be an
absence of breath sounds over the affected part of the lung.

Question 4 of 7
Which of the following is associated with the underlying diagnosis?
/ A. ArgylI-Robertson pupil
/ B. Bouchard's nodes
/ C. Heberden's nodes
/ D. Roth spots
/ E. Xanthomas

Explanation - Q: 4.4 Close

The correct answer is D. Roth spots are white spots of coagulated fibrin in
the retina, which are found on funduscopic exam. They are due to micro-
emboli from the cardiac vegetations that occur in endocarditis.
An Argyll-Robertson pupil (choice A) is pathognomonic for tertiary syphilis.
The pupil constricts with accommodation, but it is not reactive to light.
Bouchard's nodes (choice B) and Heberden's nodes (choice C) are found in
osteoarthritis. They are osteophytes that form at either the proximal
interphalangeal joint (Bouchard's nodes) or distal interphalangeal joint
(Heberden's nodes).
Xanthomas (choice E) are signs of hyperlipidemia. They are lipid deposits
that occur on the skin, especially on eyelids, and tendons.



Question 5 of 7
Which of the following sets of changes depict the systolic pressure (SP), diastolic
pressure (DP), pulse pressure (PP), stroke volume (SV),
and end-diastolic volume (EDV) in this patient as compared to a healthy
individuaI?



Explanation - Q: 4.5 Close

The correct answer is E. In aortic regurgitation, blood flows backward
through the aortic valve during diastole, when the valve is closed. The
arterial pulse pressure is widened, often to over 100 mm Hg, and, in extreme
cases the systolic pressure can be elevated to over 250 mm Hg with a
depression in the diastolic pressure. The aortic pressure falls greatly during
diastole, because blood from the aorta regurgitates backward into the
ventricle through the leaky aortic valve. The increase in systolic pressure
most likely results from the large increase in stroke volume, which is
secondary to an increase in the end-diastolic volume.





Question 6 of 7
The patient is treated with an angiotensin receptor blocker to decrease afterload.
Which of the following was she most likely given?
/ A. Benazepril
/ B. Captopril
/ C. Irinotecan
/ D. Losartan
/ E. Meclizine

Explanation - Q: 4.6 Close

The correct answer is D. Losartan is an angiotensin receptor blocker. It
antagonizes the action of angiotensin II on AT 1 receptors.
Benazepril (choice A) and captopril (choice B) are both angiotensin
converting enzyme inhibitors. They interfere with the conversion of
angiotensin I to angiotensin II.
Irinotecan (choice C) is a chemotherapeutic agent used in the treatment of
metastatic colon cancer. It binds to topoisomerase I-DNA complexes,
preventing religation of single-stranded breaks.
Meclizine (choice E) is an anticholinergic agent that is used in the treatment
of motion sickness and dizziness.


Question 7 of 7
The patient decompensates further and is treated with nitroprusside to decrease
afterload. After several days, the patient develops a
worsening generalized cyanosis despite a normal oxygen tension and does not
respond to oxygen. Methemoglobin levels are measured and
found to be 40%. Which of the following is the most appropriate
pharmacotherapy?
/ A. FIumazenil
/ B. GIucagon
/ C. Methylene blue
/ D. N-acetylcysteine
/ E. Naloxone

Explanation - Q: 4.7 Close

The correct answer is C. Methemoglobinemia can be caused by many
different medications, including nitrites, nitroprusside, sulfonamides, and
antimalarials. Symptoms include headache, fatigue, lethargy, dyspnea, and
tachycardia. The diagnosis is suggested by a normal oxygen tension and
generalized cyanosis that does not respond to oxygen. If there are signs of
hypoxia, or the methemoglobin levels are greater than 30%, the patient
should be treated with methylene blue. Methylene blue converts ferric iron to
ferrous iron, producing hemoglobin.
Flumazenil (choice A) is used in the treatment of benzodiazepine overdose.
It antagonizes benzodiazepine receptors.
Glucagon (choice B) is used in the treatment of beta receptor blocker
overdose.
N-acetylcysteine (choice D) is used in the treatment of acetaminophen
overdose. It replenishes glutathione stores, which are depleted in
acetaminophen overdose.
Naloxone (choice E) is used in the treatment of opiate overdose. It
antagonizes opiate receptors.



A 30-year-old Hispanic woman reports to her primary care physician complaining
of progressive dyspnea. She reports that she has a two-year
history of exertional shortness of breath that has now worsened and affects her
with even modest amounts of activity. Occasionally, a dry
cough accompanies her dyspnea. Today, she had a mild episode of hemoptysis,
which prompted her visit. She denies fever, chills, or sputum
production. Her medical history is significant for a febrile illness with sore throat
and joint pain at the age of fifteen in Mexico. On examination,
she is a welI-developed female in no acute distress. A crescendo diastolic rumble
is present at the apex of the heart and fine crackles are
auscultated at the lung bases.

Question 1 of 5

Which of the following is the most likely diagnosis?
/ A. Mitral stenosis
/ B. Myocardial infarction
/ C. Pneumonia
/ D. Pulmonary effusion
/ E. Pulmonary embolus

Explanation - Q: 5.1 Close

The correct answer is A. This patient displays the classic signs, symptoms,
and history for mitral stenosis. She has progressive dyspnea with hemoptysis
and an apical diastolic murmur. Her dry cough and basilar crackles also
suggest pulmonary edema, a result of pulmonary hypertension, which is a
consequence of her mitral stenosis.
Exertional dyspnea and pulmonary edema support the diagnosis of
myocardial infarction (choice B) but hemoptysis does not. The chronic,
progressive course of her symptoms also does not support MI as a
diagnosis.
The patient denies fever, chills, and sputum production, which are the
hallmarks of pneumonia (choice C).
While a pulmonary effusion (choice D) may cause progressive dyspnea, the
progression is usually more rapid than in this case. In addition, pulmonary
effusion is not associated with hemoptysis.
Pulmonary embolus (choice E) may present in a variety of ways, but
classically presents as acute onset of shortness of breath and pleuritic chest
pain. In severe cases, patients may have hemoptysis. They may also have a
history of calf pain, immobility, and a hypercoaguable state.



Question 2 of 5

Which of the following organisms is most likely responsible for her history of
fever, sore throat, and joint pain?


Explanation - Q: 5.2 Close

The correct answer is A. This patient has several findings that suggest that
she suffered from rheumatic fever at age fifteen. Patients with rheumatic
fever may have a variety of symptoms that include: fever, pharyngitis, chest
pain (carditis), arthralgia, subcutaneous nodules, macular rash (erythema
marginatum), and Sydenham's chorea. This patient had many of the acute
symptoms of rheumatic fever in her history as well as some of the chronic
sequelae. A history of rheumatic fever, coupled with a diastolic murmur,
suggests that her mitral disease is a result of rheumatic processes. Group A
Streptococcus infection has been implicated in rheumatic fever and was the
cause of her febrile illness.
Group B Streptococcus (choice B) is associated with neonatal infections and
postpartum endometriosis. It is not implicated in rheumatic fever.
While H. influenzae (choice C) can cause upper respiratory infections,
meningitis, epiglotitis, and pneumonia in the pediatric setting, it does not
cause arthralgia or rheumatic disease.
Staphlococcus. aureus (choice D) causes a variety of conditions such as
staphylococcal scalded skin syndrome, toxic shock syndrome, wound
infection, osteomyelitis, and endocarditis. It does not cause rheumatic
conditions.
Streptococcus viridans (choice E) is associated with dental caries and does
not cause rheumatic fever.


Question 3 of 5
An echocardiogram is performed and it shows left atrial enlargement, an
increased left atrioventricular pressure gradient, and a diminished
mitral valve orifice. Which of the following electrocardiographic findings is
consistent with left atrial enlargement?
/ A. Peaked T waves
/ B. Prolonged PR interval
/ C. Prolonged QR interval
/ D. Q waves
/ E. Tall and peaked P waves

Explanation - Q: 5.3 Close

The correct answer is E. Tall and peaked P waves are a characteristic
finding for any condition that results in left atrial enlargement. In mitral
stenosis, this dilation results from chronic increased left atrial pressure as the
stenotic valve hinders flow into the left ventricle.
Peaked T waves (choice A) occur in hyperkalemia.
Mitral stenosis does not affect PR interval. Prolonged PR interval (choice B)
may indicate heart block.
The QRS complex (choice C) is typically normal in mitral stenosis unless
severe pulmonary hypertension is present and a right axis deviation occurs.
Q waves (choice D) are a classic finding in myocardial infarction and do not
occur in mitral stenosis.

Question 4 of 5
In addition to the above findings, atrial fibrillation is seen. As a result, the patient
will be at increased risk for which of the following conditions?
/ A. Deep vein thrombosis
/ B. Dilated cardiomyopathy
/ C. Portal hypertension
/ D. Renal failure
/ E. Stroke

Explanation - Q: 5.4 Close

The correct answer is E. Patients with mitral stenosis often develop atrial
fibrillation secondary to the atrial distention caused by the valve dysfunction.
Under these conditions, the altered flow in the atrium leads to the formation
of thrombi, which may embolize. Typical sites of embolization may include
brain, kidney, spleen, and extremities. Thus, this patient will be at increased
risk for embolic stroke.
Deep vein thrombosis (choice A) is not caused by atrial fibrillation. Factors
that precipitate deep vein thrombosis are described by Virchow's triad:
endothelial damage, stasis, and hypercoaguable states.
Atrial fibrillation does not put this patient at increased risk for dilated
cardiomyopathy (choice B). Common causes for dilated cardiomyopathy
include: viral myocarditis (especially Coxsackie virus), alcohol, cocaine, and
thyroid disease.
There is no association between atrial fibrillation and portal hypertension
(choice C).
While atrial fibrillation may cause renal infarcts, many significant infarctions
would be necessary to precipitate renal failure (choice D).



Question 5 of 5

Which of the following sets of changes depict the mean pulmonary artery
pressure (MPAP), pulmonary wedge pressure (PWP), peak left
ventricular pressure (PLVP), and mean arterial pressure (MAP) in this patient,
compared to a healthy individuaI?




Explanation - Q: 5.5 Close

The correct answer is E. The pulmonary wedge pressure (which is used as
an estimate of left atrial pressure) and the pulmonary artery pressure are
both elevated in mitral stenosis. Left ventricular pressure and mean arterial
pressure are both normal.


A 28-year-old man is evacuated by ambulance from his work to the emergency
department of a local hospital after he abruptly develops
massive hemoptysis. When seen in the emergency department, he is already
hypotensive and requires transfusion to maintain blood
pressure. Emergency department personnel perform intubation with assisted
ventilation to maintain his respiratory function. A chest x-ray film
shows diffuse bilateral alveolar infiltrates. According to the patient's wife, the man
had a number of minor episodes of hemoptysis with blood-
tinged sputum during the past year, but discounted them and refused to seek
medical attention.
Question 1 of 6
Which of the following is the most likely diagnosis?
/ A. Bronchogenic carcinoma
/ B. Goodpasture syndrome
/ C. Idiopathic pulmonary fibrosis
/ D. Pneumonia
/ E. Tuberculosis

Explanation - Q: 1.1 Close

The correct answer is B. Goodpasture syndrome is a hypersensitivity
disorder of unknown cause that can cause hemoptysis and dyspnea
secondary to bleeding from the alveolar capillary bed. The pulmonary
hemorrhage may be mild to massive. Most patients, as in this case, are
young men.
Bronchogenic carcinoma (choice A) and tuberculosis (choice E) can cause
pulmonary hemorrhage, which is usually not massive, but can be excluded
because a mass lesion was not seen on chest x-ray.
Idiopathic pulmonary fibrosis (choice C) and pneumonia (choice D) could
both cause diffuse pulmonary changes visible on x-ray, but would be unlikely
to cause massive hemoptysis.
















Question 2 of 6




The man is admitted to the hospitaI, and during his hospitalization, is also found
to have hematuria, proteinuria, red cell casts in the urine, and
rising serum creatinine. A hematoxylin and eosin stained section of a renal
biopsy is shown above. Which of the following is the most likely
diagnosis?

/ A. End-stage renal disease
/ B. GIomerulonephritis
/ C. KimmelstieI-Wilson nodules
/ D. Pyelonephritis
/ E. Tubulointerstitial nephritis

Explanation - Q: 1.2 Close

The correct answer is B. In addition to lung involvement, Goodpasture
syndrome is also characterized by severe renal involvement which may
precede, coexist with, or follow lung involvement. Renal biopsy
characteristically shows rapidly progressive glomerulonephritis with crescent
formation in Bowman's spaces.
End-stage renal disease (choice A) with marked scarring of the renal cortex
and loss of glomeruli can be seen after many years of kidney disease of
many types.
Kimmelstiel-Wilson nodules (choice C) are characteristic of diabetic
nephropathy.
Pyelonephritis (choice D) is an infection of the kidney, and can complicate a
urinary tract infection or be due to hematogenous dissemination of bacteria.
Tubulointerstitial nephritis (choice E) can be seen with drug reactions and
toxins.



Question 3 of 6



A second section from the renal biopsy was stained with immunofluorescent
antibodies to human lgG (and human complement) with the result
shown. Which of the following is most likely the antigen against which the lgG
antibody is directed?
/ A. Collagen
/ B. EIastin
/ C. Fibronectin
/ D. Laminin
/ E. Vitronectin

Explanation - Q: 1.3 Close

The correct answer is A. The photomicrograph shows the linear
immunofluorescence characteristic of Goodpasture syndrome. The antigenic
determinant in Goodpasture syndrome is on the alpha-3 chain of type IV
(basement membrane type) collagen.
Elastin (choice B), fibronectin (choice C), laminin (choice D) and vitronectin
(choice E) are all found in the extracellular matrix. These molecules could
conceivably be antigenic, but have not been implicated in Goodpasture
syndrome.
Question 4 of 6
The antigenic determinant against which the lgG is specifically directed is located
in the C-terminal domain of one peptide chain of the
molecule. The nucleotide sequence encoding this region is known, and a small
portion of the intron (Iower case) and exon (UPPER CASE)
sequence of this region is shown below:
c a t t a g G A A C T C T T G
How many amino acids of the this antigen are translated from the in-frame
sequence shown?
/ A. 1
/ B. 2
/ C. 3
/ D. 4
/ E. 5

Explanation - Q: 1.4 Close

The correct answer is C. The intron sequence (lower case) is a non-coding
region between exons. Therefore, the intron portion of the sequence above
(cattag) does not encode any amino acids in the Goodpasture antigen. The 9
nucleotides of the in-frame exon sequence shown (UPPER CASE) would be
translated into 3 amino acids (3 nucleotides/amino acid) of the Goodpasture
antigen. None of the 3 codons shown (GAA, CTC, TTG) is a stop codon.
If all the nucleotides shown were part of an exon, the sequence would code
for only 1 amino acid (choice A) because the second triplet (tag) is a stop
codon.


Question 5 of 6
Which of the following would be the most appropriate pharmacotherapy for this
patient?
/ A. Erythropoietin
/ B. Famciclovir
/ C. Foscarnet
/ D. Isoniazid
/ E. Methylprednisolone

Explanation - Q: 1.5 Close

The correct answer is E. Corticosteroids and cyclophosphamide are used
to prevent the formation of new antibodies and control the inflammatory
response in Goodpasture syndrome. Intravenous administration of
corticosteroids, such as methylprednisolone or prednisone, or
immunosuppressive therapy with cyclophosphamide is usually carried out for
several days.
Erythropoietin (choice A) is a glycoprotein, normally produced in the
kidneys, that stimulates red blood cell production. It is indicated for treatment
of anemia associated with chronic renal failure, zidovudine therapy in HIV-
infected patients, and for cancer patients on chemotherapy. It is
contraindicated in patients with uncontrolled hypertension and
hypersensitivity to mammalian cell-derived products or to human albumin.
Famciclovir (choice B) is a synthetic acyclic purine nucleoside analog that
interferes with DNA synthesis in herpes simplex virus (types 1 and 2) and
varicella zoster virus. It is used in the treatment of genital herpes in patients
who are not immunocompromised.
Foscarnet (choice C) is an agent that inhibits the replication of all known
herpes viruses, including cytomegalovirus, herpes simplex virus (types 1 and
2), human herpesvirus 6, Epstein-Barr virus and varicella zoster virus. This
agent is indicated for the treatment of cytomegalovirus (CMV) retinitis,
mucocutaneous herpes simplex virus (HSV), and acyclovir-resistant HSV in
immunocompromised patients.
Isoniazid (choice D) is an agent indicated for the treatment of all forms of
tuberculosis (TB) as well as the prophylaxis of tuberculosis. This agent is
associated with the development of severe and sometimes fatal hepatitis.
Peripheral neuropathy is the most common side effect seen with isoniazid
therapy and is associated with a symmetrical numbness and tingling of the
extremities. Pyridoxine administration can treat/prevent this condition.


Question 6 of 6
Over the next six months, the patient is treated with repeated plasmapheresis.
Which of the following is the rationale of this therapy for this
patient?
/ A. Decrease blood clotting factors
/ B. Increase hematocrit
/ C. Increase platelets
/ D. Protect against infection
/ E. Reduce circulating autoantibodies

Explanation - Q: 1.6 Close

The correct answer is E. In plasmapheresis, the patient's blood is removed
(in small amounts over time). The blood cells are collected and the plasma is
discarded, and then the blood cells are resuspended in someone else's
plasma before being returned to the patient's body. Plasmapheresis is a very
effective, albeit expensive, way of washing autoantibody out of the patient's
body, and together with pharmacologic immunosuppression has markedly
changed the formerly dismal prognosis of Goodpasture syndrome. Surviving
patients often stop producing clinically significant amounts of autoantibody
within 12 to 18 months, although some patients may require longer-term
immunosuppressive therapy.



A fifty-year-old man presents to his primary care doctor complaining of chronic
cough with hemoptysis, weakness, frequent urination, thirst,
and a decreased ability to concentrate. Review of systems reveals fever, chills,
night sweats, and a twenty-pound weight loss. He has a
seventy pack-year smoking history. On examination, his lungs are clear to
auscultation, but neurologic examination reveals global hyporeflexia.
A chest x-ray film reveals a 4-cm hilar non-cavitary opacity in the left lung. Serum
electrolytes show K+ = 2.3 mEq/L. Calcium is within normal
Iimits.
Question 1 of 6
Which of the following is the most likely diagnosis?
/ A. Adenocarcinoma
/ B. Bronchioloalveolar carcinoma
/ C. Mesothelioma
/ D. Small cell carcinoma
/ E. Squamous cell carcinoma

Explanation - Q: 2.1 Close

The correct answer is D. This patient has classic findings for small cell
carcinoma of the lung (also known as oat cell carcinoma). Patients with small
cell carcinoma typically present with constitutional symptoms, cough, and
hemoptysis. Paraneoplastic syndromes are common with this cancer, and
may often be the first symptoms to present. This patient has hypokalemia
and many of the symptoms that accompany it: muscular weakness, frequent
urination, thirst, and decreased ability to concentrate. This patient's
radiographic findings (non-cavitary hilar mass) also support the diagnosis of
small cell carcinoma.
Adenocarcinomas (choice A) tend to present as peripheral masses, and
paraneoplastic syndromes are rare.
Bronchioloalveolar carcinoma (choice B) is a subtype of adenocarcinoma. It
arises from the peripheral airways, and while it can present as a discrete
mass, it typically is indistinct radiographically as it grows down the airway
surfaces. Again, paraneoplastic syndromes are rare.
Mesothelioma (choice C) is a pleural malignancy associated with asbestos
exposure. They do not present as hilar masses. 50% metastasize, but death
usually results from local extension. They tend to produce large effusions
that may obscure the mass on chest x-ray films.
Squamous cell carcinoma (choice E), like small cell carcinoma, has a strong
association with smoking. These tumors may present in a similar fashion,
with a more central lung lesion, but paraneoplastic syndromes with
squamous cell carcinoma tend to be related to the secretion of a PTH-like
substance, resulting in hypercalcemia. Thus the diagnosis of small cell
carcinoma is more likely.



Question 2 of 6
Which of the following substances secreted from the lesion is the most likely
cause of this patient's electrolyte disturbance?
/ A. ACTH
/ B. AIdosterone
/ C. Angiotensin ll
/ D. Epinephrine
/ E. PTH

Explanation - Q: 2.2 Close

The correct answer is A. ACTH is often secreted by small cell lung
carcinoma. The clinical results are those seen from uninhibited ACTH
secretion: the adrenal gland increases the production of glucocorticoids and
mineralocorticoids. Thus, patients experience hypokalemia, hyperglycemia,
and hypertension. The chronic sequelae of this process (Cushingoid
symptoms) are not usually seen, due to the metabolic demands of the
neoplasm and the short survival of affected individuals.
None of the other hormones listed above are commonly secreted by small
cell cancers.


***
Question 3 of 6
As the patient's condition progresses, the patient grows increasingly hoarse.
Which of the following is the most likely reason for the patient's
hoarseness?
/ A. Enlargement of the mass has severely decreased inspiratory volume.
/ B. Extension of the mass into the larynx
/ C. Impingement on cranial nerve XII
/ D. Impingement on the recurrent laryngeal nerve
/ E. Metastasis into the speech centers of the brain

Explanation - Q: 2.3 Close

The correct answer is D. As seen with many hilar masses, impingement on
the recurrent laryngeal nerve produces hoarseness. The recurrent laryngeal
nerve courses inferiorly to the aortic arch and then turns superiorly to
innervate the intrinsic muscles of the larynx (except the cricothyroid muscle).
When this nerve sustains unilateral damage, hoarseness results, as patients
have difficulty abducting the vocal cords. Bilateral damage results in acute
breathlessness, because both of the vocal cords move to the midline and
block the airway.
Enlargement of the mass (choice A) to a size that would affect speech
would likely produce pulmonary collapse, not hoarseness.
Extension of the mass from the hilum of the lung to the larynx (choice B)
would be unlikely. Many critical structures are in this course, and the patient
would likely not survive long enough to become hoarse from direct extension.
Impingement on cranial nerve XII (choice C) would affect movement of the
tongue. Not only would metastasis or direct extension to the anatomic site of
this nerve be unlikely, this event would not make a patient hoarse.
Metastasis to the speech centers (choice E) would produce more profound
speech difficulties. The characteristics of the aphasia would depend upon
which speech center is affected ( i.e., fluent vs. expressive aphasia).


Question 4 of 6
This patient is given cisplatin as part of his chemotherapeutic regimen. Which of
the following is a known adverse effect of cisplatin?
/ A. Cardiotoxicity
/ B. Hemorrhagic cystitis
/ C. Nephrotoxicity
/ D. Profound myelosuppression
/ E. Pulmonary fibrosis

Explanation - Q: 2.4 Close

The correct answer is C. Cisplatin is a heavy metal compound used in the
treatment of small cell carcinoma of the lung. Cisplatin cross-links DNA,
though it is not a true alkylating agent. The dominant adverse effect seen
with its administration is nephrotoxicity, as it is toxic to both proximal and
distal renal tubule epithelium. It produces only modest myelosuppression.
Doxorubicin causes cardiotoxicity (choice A) and congestive heart failure
may result.
Cyclophosphamide is a chemotherapeutic agent known to cause
hemorrhagic cystitis (choice B).
The myelosuppression with cisplatin is modest. Many other
chemotherapeutic agents are known to severely suppress the marrow
(choice D).
Bleomycin is known to cause pulmonary fibrosis (choice E).


Question 5 of 6
In addition, this patient is given etoposide. Which of the following best describes
the mechanism of action of etoposide?
/ A. Etoposide cross-Iinks DNA
/ B. Etoposide induces single- and double-stranded breaks in DNA
/ C. Etoposide inhibits dihydrofolate reductase
/ D. Etoposide inhibits microtubule formation
/ E. Etoposide inhibits topoisomerase ll

Explanation - Q: 2.5 Close

The correct answer is E. Etoposide acts by inhibiting the enzyme
topoisomerase II. Topoisomerases are enzymes that create and repair
breaks in DNA during replication. This enzyme is needed to relieve topologic
and conformational changes as the DNA is "unzipped" during replication and
transcription.
Alkylating agents, such as cyclophosphamide, cross-link DNA (choice A).
The antitumor antibiotics, such as bleomycin, act by inducing breaks in DNA
(choice B).
Methotrexate is an antimetabolite chemotherapeutic agent that inhibits
dihydrofolate reductase (choice C), an enzyme needed to produce purine
nucleotides. Thus the "purine shortage" produced hinders cell replication.
The Vinca alkaloids exert their effects by inhibiting microtubule formation
(choice D). Without microtubules, cells cannot mobilize their chromosomes,
and thus mitosis is inhibited.




Question 6 of 6
The patient has been aware of his diagnosis and prognosis for several weeks
now. He makes the statement "I had successes and failures but
I'm pretty sure l got as much living out of the last 50 years as anybody could."
This statement suggests that the patient is experiencing which of
the following of Erikson's stages?
/ A. Ego integrity vs. despair
/ B. Generativity vs. self absorption
/ C. Identity vs. role confusion
/ D. Industry vs. inferiority
/ E. Intimacy vs. isolation

Explanation - Q: 2.6 Close

The correct answer is A. In the ego integrity vs. despair stage, individuals
reconcile their achievements and failures and face the fact that any human
life is limited. Persons completing this task find self-worth in reviewing their
life events. Persons stuck at this task cannot admit that time has run out.
Generativity vs. self-absorption (choice B) usually occurs from age 30-65. It
involves noting ones contributions as a parent (in the broadest sense) and
eventually "passing the torch" to the next generation. Persons stuck in this
phase refuse to give up any power to their successors, and maintain ultimate
authority without making a place for the next generation.
Identity vs. role confusion (choice C) occurs during adolescence, and is
usually defined by emotional and or geographic separation from the actual
parents, with an internal identity based on both similarities and differences
from parental traits and values.
Industry vs. inferiority (choice D) is a stage in which school age (6-12)
children explore peer groups and learn that industry (hard work) generally
pays off.
Intimacy vs. isolation (choice E) is the stage, in the early twenties, when
young adults connect to their peers and significant others in a meaningful
and mature way, putting aside fear and inhibitions.



A 53-year-old man consults a physician because he has begun coughing up
sputum tinged with fresh blood. He does not initially report any
other symptoms to his physician. When his physician comments on his
hoarseness and cough, the patient discounts these symptoms and
attributes them to his long smoking history. He cannot say when they began or
became worse.
Question 1 of 6
The chronic hoarseness suggests dysfunction of which of the following?
/ A. Palate
/ B. Pharynx
/ C. Tongue
/ D. Trachea
/ E. Vocal cords


Explanation - Q: 3.1 Close

The correct answer is E. Hoarseness specifically suggests dysfunction of
the vocal cords, which produce the sounds that are then articulated to
speech with the pharynx (choice B), palate (choice A), tongue (choice C),
teeth, and lips. While vocal cord dysfunction is specifically suggested by
hoarseness, the anatomic lesion may or may not be present at the level of
the vocal cords. Vocal paralysis may be the result of local tumor or trauma,
intracranial lesions affecting the nucleus ambiguus or its supranuclear tracts,
and lesions at the base of the skull, neck, or upper portion of the thorax that
involve either the vagus nerve or the recurrent laryngeal nerves.
Air from the trachea (choice D) passes into the larynx to reach the vocal
cords, but tracheal dysfunction does not produce hoarseness.

Question 2 of 6
Laryngoscopy reveal a fungating tumor of the larynx that is located between the
false and true vocal cords. This tumor is in which of the
following sites?
/ A. Aryepiglottic fold
/ B. Infraglottic compartment
/ C. Piriform recess
/ D. Supraglottic compartment
/ E. Ventricle

Explanation - Q: 3.2 Close

The correct answer is E. The interior of the larynx is divided into 3
compartments: the supraglottic compartment (choice D) above the false
vocal cords, the ventricle between the false and true vocal cords, and the
infraglottic compartment (choice B) below the true vocal cords.
The aryepiglottic fold (choice A) is the upper free border of the quadrangular
membrane found in the supraglottic compartment.
The piriform recess (choice C) lies behind the thyroid laminae and the lateral
wall of the supraglottic compartment.
Question 3 of 6
Biopsy of the mass demonstrates a malignancy. Which of the following is the
most likely diagnosis?
/ A. Adenocarcinoma
/ B. Lymphoma
/ C. Oat cell carcinoma
/ D. Sarcoma
/ E. Squamous cell carcinoma

Explanation - Q: 3.3 Close

The correct answer is E. The epithelial lining of the larynx is squamous
epithelium, and the vast majority of cancers of the larynx are squamous cell
carcinomas. Squamous cell carcinoma can present with hoarseness, cough,
hemoptysis, or difficulty swallowing. If the symptoms develop insidiously (as
in the case), the patient may not seek medical attention until late in the
course. While very small laryngeal cancers can be treated successfully with
surgery and/or radiation, larger ones are much more problematic, in large
part because complete resection of the cancer and any lymph node
metastases can be difficult to impossible to perform without compromising
the many vital neck structures. Patients treated with partial laryngectomy
may retain some speech ability. Patients treated with total laryngectomy can
often learn to speak again using esophageal speech (gradual belching of air
through the pharyngoesophageal junction), a tracheoesophageal fistula (one
way valve between the trachea and the esophagus which makes a sound
when air is forced across it), or an electrolarynx (sound source held against
the neck). With all three techniques, the sound produced is then turned into
articulation by the patient's pharynx, palate, tongue, teeth, and lips. The other
tumors listed in the choices are very uncommon in the larynx.


Question 4 of 6
In addition to smoking, which of the following is an accepted risk factor for this
patient's tumor?
/ A. AIcohol use
/ B. Cocaine use
/ C. Coffee use
/ D. Marijuana use
/ E. Tea use

Explanation - Q: 3.4 Close

The correct answer is A. The only two risk factors that you will need to
associate with laryngeal carcinoma are smoking and alcohol use. Cocaine
(choice B), coffee (choice C), and tea (choice E) have not been linked to
laryngeal cancer.
Marijuana (choice D) has been suggested as a risk factor for oral cancer in a
few cases, but a causal association has not been established.



Question 5 of 6
The patient returns to clinic three weeks after receiving the news that his cancer
is inoperable. His wife reports that he has been more
withdrawn, eating and sleeping poorly and "just seems to have lost all hope." On
examination, the patient moves very little, never makes eye
contact, and admits to "some" depressed mood, Ioss of appetite, and sleep
disruption. Which of the following symptoms is of most concern
regarding his risk of suicide?
/ A. Decreased appetite
/ B. Decreased energy
/ C. Diminished concentration
/ D. Guilty and worthless feelings
/ E. Hopelessness

Explanation - Q: 3.5 Close

The correct answer is E. Hopelessness is an ominous sign and is
associated with a higher risk for suicide.
Decreased appetite (choice A), decreased energy (choice B), and
diminished concentration (choice C) are symptoms indicating the presence
of a major depressive episode, or are the result of the medical illness.
Guilty and worthless feelings (choice D) are symptoms of depression. They
do not predict risk for suicide.


*** Commercial version is infinite. Order at http://www.structurise.com/kleptomania ***
Question 6 of 6
Some patients present with hoarseness due to a tumor at the lung apex that
involves a nerve that is a branch of which of the following?
/ A. Accessory nerve
/ B. GIossopharyngeal nerve
/ C. Hypoglossal nerve
/ D. Phrenic nerve
/ E. Vagus nerve



Explanation - Q: 3.6 Close

The correct answer is E. The larynx has complex innervation. The vocal
cords and most of the area of the larynx below them are supplied by the
recurrent laryngeal nerve, which is a branch of the vagus nerve that passes
beneath the subclavian artery before returning to the neck to innervate the
larynx. From above the larynx, the superior laryngeal nerve arises from the
vagus and divides into the internal laryngeal nerve and the external laryngeal
nerve. The innervation above the vocal cords is by the internal laryngeal
branch of the vagus nerve. The cricothyroid muscle (the only muscle of the
larynx not supplied by the recurrent laryngeal nerve) is supplied by the
external laryngeal branch of the superior laryngeal, which also branches off
the vagus, but which contains motor fibers originally derived from the
accessory nerve (choice A).
The glossopharyngeal nerve (choice B) supplies the pharynx.
The hypoglossal nerve (choice C) supplies the tongue.
The phrenic nerve (choice D) supplies the diaphragm.

A 60-year-old woman presents to her doctor's office with complaints of
headaches and progressive vision loss in her left eye over the past 3
months. On further questioning, she also complains of an inability to smell the
flowers in her garden. Her family also notes that she has become
more short-tempered over the past 6 months and that, while she was previously
famously polite and good-natured, she now curses and insults
people regularly. Her ophthalmological examination is significant left monocular
vision loss and right papilledema. She is also found to have
anosmia. The rest of her examination is unremarkable. An MRI of the brain
reveals a mass lesion.
Question 1 of 5
Which of the following is the most likely cause of the patient's vision loss?
/ A. Pressure on the left occipital lobe
/ B. Pressure on the left optic nerve
/ C. Pressure on the right occipital lobe
/ D. Pressure on the right optic nerve
/ E. Pressure on the right temporal lobe

Explanation - Q: 1.1 Close

The correct answer is B. A lesion compressing the left optic nerve would
cause left optic nerve atrophy and monocular vision loss. This patient's
presentation describes a Foster-Kennedy syndrome, in which there is
anosmia and optic atrophy in one eye, and papilledema in the other eye. This
presentation classically is seen with olfactory groove meningiomas, which
arise from the dura covering the floor of the anterior fossa, and compress the
frontal lobes superiorly, and disrupt the olfactory tracts and optic nerve as
they extend posteriorly.
Pressure on the left occipital lobe (choice A) would cause a right
homonymous hemianopsia, not a monocular vision loss.
Pressure on the right occipital lobe (choice C) would cause a left
homonymous hemianopsia, not a monocular vision loss.
Pressure on the right optic nerve (choice D) would cause a right monocular
vision loss.
Pressure on the right temporal lobe (choice E) would cause a left superior
homonymous quadrantanopsia, not a monocular vision loss.






A 60-year-old woman presents to her doctor's office with complaints of
headaches and progressive vision loss in her left eye over the past 3
months. On further questioning, she also complains of an inability to smell the
flowers in her garden. Her family also notes that she has become
more short-tempered over the past 6 months and that, while she was previously
famously polite and good-natured, she now curses and insults
people regularly. Her ophthalmological examination is significant left monocular
vision loss and right papilledema. She is also found to have
anosmia. The rest of her examination is unremarkable. An MRI of the brain
reveals a mass lesion.
Question 2 of 5
Which of the following would most likely be seen on further ophthalmologic
examination
/ A. Light shone in her left eye produces pupillary constriction in both eyes
/ B. Light shone in her left eye produces pupillary constriction in her left eye only
/ C. Light shone in her left eye produces pupillary constriction in her right eye
only
/ D. Light shone in her right eye produces pupillary constriction in both eyes
/ E. Light shone in her right eye produces pupillary constriction in her left eye
only
/ F. Light shone in her right eye produces pupillary constriction in her right eye
only

Explanation - Q: 1.2 Close

The correct answer is D. A lesion of the left optic nerve would prevent her
left eye from reacting to direct (light shone in her left eye) because of a
disruption in the afferent limb of the pupillary light reflex pathway. This
patient is blind in the left eye, so light shone in the left eye will not transmit a
stimulus through the optic nerve to the pretectal nucleus to the Edinger-
Westphal nucleus and through the oculomotor nerve to the pupillary
constrictor muscle. The damage to the afferent limb will similarly prevent the
consensual response from occurring in her right eye. Taken together, this
excludes choices A, B, and C.
However, the afferent limb of the pupillary light reflex pathway in the right eye
and the efferent limbs in both the right and left eyes are intact. Therefore,
shining light in her right eye would produce both a direct response in her right
eye and a consensual response in her left eye (choice D).

Question 3 of 5
Which of the following is the most likely cause of the patient's loss of smelI?
/ A. Pressure on the left occipital lobe
/ B. Pressure on the left temporal lobe
/ C. Pressure on the olfactory tracts
/ D. Pressure on the right occipital lobe
/ E. Pressure on the right temporal lobe

Explanation - Q: 1.3 Close

The correct answer is C. This patient's presentation describes a Foster-
Kennedy Syndrome, in which there is anosmia and optic atrophy in one eye
and papilledema in the other eye. This presentation classically is seen with
olfactory groove meningiomas, which arise from the dura covering the floor of
the anterior fossa and compress the frontal lobes superiorly, and disrupt the
olfactory tracts and optic nerve as they extend posteriorly.
Pressure on the left occipital lobe (choice A) could cause a right
homonymous hemianopsia, not a loss of smell.
Pressure on the left temporal lobe (choice B) could cause a right superior
homonymous quadrantanopsia, not a loss of smell. Seizures originating from
the temporal lobe can be preceded by an aura consisting of an olfactory
hallucination.
Pressure on the right occipital lobe (choice D) could cause a left
homonymous hemianopsia, not a loss of smell.
Pressure on the right temporal lobe (choice E) could cause a left superior
homonymous quadrantanopsia, not a loss of smell. Seizures originating from
the temporal lobe can be preceded by an aura consisting of an olfactory
hallucination.

Question 4 of 5
What is the most likely cause of the patient's change in personality?
/ A. AIzheimer disease
/ B. Pressure on the frontal lobes
/ C. Pressure on the left occipital lobe
/ D. Pressure on the left temporal lobe
/ E. Pressure on the right occipital lobe

Explanation - Q: 1.4 Close

The correct answer is B. Lesions affecting the frontal lobes can cause
personality changes such as decreased social consciousness, inappropriate
behavior, decreased tact, and elevated mood.
Alzheimer disease (choice A) is the most common cause of dementia in the
elderly, and onset usually occurs around 80 years of age. There is no sex
predilection. Inheritance is usually sporadic, but occasionally may be
dominant, associated with multiple chromosomal abnormalities. Symptoms
usually start with forgetfulness and then lead to confusion, followed by
ideomotor apraxia, dysnomia, and akinetic mutism.
Pressure on the left occipital lobe (choice C) could cause a right
homonymous hemianopsia, not a loss of smell.
Pressure on the left temporal lobe (choice D) could cause a right superior
homonymous quadrantanopsia, a potential partial contralateral hearing loss,
decreased auditory learning, difficulty with learning, and a Wernicke's
(receptive) aphasia. Wernicke's aphasia is marked by an inability to
comprehend written and spoken language, while maintaining fluent
paraphasic speech. Often the speech produced contains malformed and
inappropriate words, with impaired repetition and an inability to write or read.
Pressure on the right occipital lobe (choice E) could cause a left
homonymous hemianopsia, not a change in personality.


Question 5 of 5
The mass is surgically removed, and the pathology is consistent with a
meningioma. Which of the following features would be present upon
microscopic examination of the tissue?
/ A. Amyloid plaques and neurofibrillary tangles
/ B. Basophilic psammoma bodies and whorls of cells
/ C. Chicken-wire vascular pattern and cells with perinuclear clearing
/ D. Groups of spindle cells in a school of fish-Iike pattern and Verocay bodies
/ E. Pseudopalisading necrosis and endovascular proliferation

Explanation - Q: 1.5 Close

The correct answer is B. These are pathologic characteristics of
meningiomas. Meningiomas are slow-growing tumors that arise from
arachnoid cap cells. They account for 15% of primary intracranial tumors,
and have a peak age of presentation between 40 and 60 years of age.
Woman are more commonly affected, and the incidence is increased by
radiation and neurofibromatosis type 2. Meningiomas almost always have
dural attachments, and are supplied by branches from the external carotid
artery. They can invade dura and bone, and can occasionally cause a
hyperostotic reaction in adjacent bone. Psammoma bodies are laminated,
concentric calcified spherules, which are also features of papillary
adenocarcinoma of the thyroid, serous papillary cystadenocarcinoma of the
ovary, and malignant mesothelioma.
Amyloid plaques and neurofibrillary tangles (choice A) are pathologic
features of Alzheimer disease. The neurofibrillary tangles and amyloid
plaques are intracytoplasmic, and most frequently found in the hippocampus
and adjacent temporal lobe. Alzheimer disease is the most common cause of
dementia in the elderly and onset usually occurs around 80 years of age.
A chicken-wire vascular pattern and cells with perinuclear clearing (choice
C) are the pathologic features of oligodendrogliomas. Oligodendrogliomas
account for 10% of gliomas, with a peak age of presentation between 35 to
40 years. They have no sex predominance. On microscopy, their cells exhibit
round nuclei with a scant cytoplasm, giving them a halo, or fried egg
appearance. This is an artifact seen in permanent sections. There is also a
background of thin vessels.
Groups of spindle cells in a "school of fish"-like pattern and Verocay bodies
(choice D) are the pathologic features of schwannomas. Schwannomas
arise from Schwann cells and account for 7% of intracranial tumors. Since
Schwann cells provide the myelin in the peripheral nervous system, these
tumors are attached to the cranial nerves when they occur in the head or to
peripheral nerves when they occur elsewhere. Schwannomas contain no
axons, and on microscopic examination, they exhibit a biphasic pattern of
compact Antoni A and loose Antoni B areas. Verocay bodies are anuclear
material with palisading cells in Antoni A areas.
Pseudopalisading necrosis and endovascular proliferation (choice E) are
pathologic features of glioblastoma multiforme (GBM). GBMs comprise 50%
of astrocytomas, and are the most frequently occurring primary brain tumor.
The peak age for their occurrence is 45 to 60 years of age, with a male
predominance. They are highly aggressive tumors with a poor prognosis.

A 33-year-old right-handed man with a past history of cocaine abuse is brought
into the emergency department by his friends after they found
him unable to speak. On examination, he is alert and is able to follow commands,
but is frustrated by his inability to speak and write. He is also
unable to repeat phrases. His cranial nerves are intact and his strength and
sensation is normal and symmetric in all extremities.
Question 1 of 3
Which of the following is the most likely diagnosis?
/ A. Broca expressive aphasia
/ B. Conductive aphasia
/ C. GIobal aphasia
/ D. Transcortical sensory aphasia
/ E. Wernicke receptive aphasia

Explanation - Q: 2.1 Close

The correct answer is A. With an expressive (Broca) aphasia, there is an
inability to speak or write with impaired repetition (unable to write from
dictation, but able to copy letters). A Broca aphasia is caused by a lesion in
the dominant inferior frontal gyrus (Broca's area).
Conductive aphasia (choice B) is caused by a disruption of the arcuate
fasciculus in the subcortical white matter of the upper Sylvian fissure under
the supramarginal gyrus. There is fluent, paraphasic speech, and impaired
repetition. Patients with a conductive aphasia understand language (both
written and spoken) and are aware of their problem.
Global aphasia (choice C) is caused by a lesion affecting both Broca's area
and Wernicke's area (usually caused by a large middle cerebral artery
stroke).
Transcortical sensory aphasia (choice D) is caused a lesion in the posterior
parietal-occipital region. A transcortical sensory aphasia is characterized by
impaired language reception, paraphasic fluent speech, and a hemianopia.
Wernicke receptive aphasia (choice E) is the inability to comprehend
language (written or spoken). There is impaired repetition. Speech is fluent,
but malformed, with inappropriate words. A Wernicke aphasia is caused by a
lesion in the superior temporal gyrus of the dominant hemisphere.


A 33-year-old right-handed man with a past history of cocaine abuse is brought
into the emergency department by his friends after they found
him unable to speak. On examination, he is alert and is able to follow commands,
but is frustrated by his inability to speak and write. He is also
unable to repeat phrases. His cranial nerves are intact and his strength and
sensation is normal and symmetric in all extremities.
Question 2 of 3
What is the most likely location of the lesion causing this patient's speech deficit?
/ A. Frontal and temporal lobes
/ B. Inferior frontal gyrus
/ C. Posterior parieto-occipital lobe
/ D. Superior temporal gyrus
/ E. Supramarginal gyrus and underlying white matter

Explanation - Q: 2.2 Close

The correct answer is B. Broca's area is located in the dominant inferior
frontal gyrus.
The frontal and temporal lobes (choice A) are the locations for both Broca's
and Wernicke's areas. A lesion involving both of these area would result in a
global aphasia.
A posterior parieto-occipital lobe (choice C) lesion would result in a
transcortical sensory aphasia.
The superior temporal gyrus (choice D) is the location of Wernicke's area, in
the dominant temporal lobe.
The supramarginal gyrus and underlying white matter (choice E) is the
location of the arcuate fasciculus. A lesion in this area would result in a
conductive aphasia.



Question 3 of 3
The lesioned area is supplied by which of the following arteries?
/ A. Anterior cerebral artery
/ B. Anterior choroidal artery
/ C. Middle cerebral artery
/ D. Posterior cerebral artery
/ E. Posterior communicating artery

Explanation - Q: 2.3 Close

The correct answer is C. The middle cerebral artery arises from the internal
carotid artery, and makes up part of the anterior circle of Willis. The middle
cerebral artery supplies the caudate nucleus, the putamen, the globus
pallidus, and the internal capsule, through the lateral lenticulostriate arteries.
As the middle cerebral artery travels distally it supplies the lateral convexity
of the hemisphere and underlying insula, including Broca's and Wernicke's
areas. A proximal middle cerebral artery occlusion in the dominant
hemisphere would result in a stroke presenting with a global aphasia and a
contralateral hemiparesis.
The anterior cerebral artery (choice A) arises from the internal carotid artery
and makes up part of the anterior circle of Willis. The proximal anterior
cerebral artery supplies the superior surface of the optic nerve, the optic
chiasm, the anterior hypothalamus, the anterior commissure, and the fornix
via the medial lenticulostriate arteries. Just distal to the anterior
communicating artery, the medial distal striate artery (the recurrent artery of
Heubner) arises to supply the head of the caudate, the anterior limb of the
internal capsule, the anterior putamen and globus pallidus, and the inferior
frontal lobe. The distal anterior cerebral artery is the usual location of
occlusions by thrombotic emboli. This segment supplies the anterior 2/3 of
the medial cortex. Occlusion of the distal right anterior cerebral artery would
result in weakness in the left lower extremity, because the leg is represented
medially on the motor homunculus.
The anterior choroidal artery (choice B) arises from the internal carotid
artery, and travels posteriorly to supply the choroid plexus of the temporal
horns of the lateral ventricles, the hippocampus, the amygdala, optic tract,
the lateral geniculate body, the globus pallidus, and the anterior portion of
the posterior limb of the internal capsule.
The posterior cerebral arteries (choice D) are formed from the bifurcation of
the basilar artery. They supply the midbrain, the posterior thalamus, and the
occipital lobe, including the visual cortex. An occlusion of the posterior
cerebral artery results in a contralateral homonymous hemianopia with
macular sparing, since the occipital pole subserving the macula can receive
collateral blood supply from the middle cerebral artery.
The posterior communicating artery (choice E) arises from the carotid siphon
and travels posteriorly to join the posterior cerebral artery. It supplies the
optic chiasm and tract, the hypothalamus, subthalamus, and the anterior half
of the anterior thalamus.


A 45-year-old right-handed man is brought into his physician's office by his wife
who states that he has been acting strangely. She states that
recently, he has been consistently trying to walk to work without a shoe or sock
on his left foot, and he keeps leaving his wedding ring and
watch at home. She also notes that he has developed a severe limp favoring his
left leg. He denies that anything is wrong. On examination, his
vital signs are within normal limits. His cranial nerves are intact. He is pleasant
and alert, and his speech is fluent. He is able to follow
commands, however he requires significant encouragement in order to follow
commands when testing his left arm and left leg. He is noted to
have some mild weakness in his left arm and more pronounced weakness in his
left leg.

Question 1 of 3
Which of the following is the most likely location of this lesion?
/ A. Bilateral amygdala nuclei
/ B. Bilateral frontal lobes
/ C. Left inferior parietal lobule
/ D. Reticular activating system
/ E. Right superior parietal lobule

Explanation - Q: 3.1 Close

The correct answer is E. A lesion involving the right superior parietal lobule
results in a contralateral hemineglect, in which there is a lack of awareness
about the left half of the body. Deficits affecting the neglected half of the
body are often treated with denial and indifference by the patient. Other
manifestations include: dressing apraxia, constructional apraxia, and
topographic memory loss.
Bilateral amygdala nuclei (choice A) ablation results in Klver-Bucy
syndrome, characterized by a tendency to examine objects orally, tameness,
diet change, psychic blindness, fearlessness, curiosity, and hypersexuality.
Bilateral frontal lobe (choice B) lesions can result in a frontal lobe syndrome
characterized by inappropriate behavior, loss of initiative, the release of
primitive reflexes (e.g., sucking, groping, and grasping), gait apraxia,
incontinence, and abulia.
The left inferior parietal lobule (choice C) includes the supramarginal gyrus
and angular gyrus. A lesion of the left inferior parietal lobule may result in a
Gerstmann's syndrome with right/left dissociation, finger agnosia, acalculia,
and agraphia.
The reticular activating system (choice D) is located in the middle and lateral
pons and midbrain. It sends signals via the spinal cord to maintain tone in the
antigravity muscles and also to various subcortical structures including the
thalamus for diffuse spread. It is responsible for keeping the brain "on." If it is
disrupted by a lesion, then the patient can fall into a coma.


Question 2 of 3
Damage to what area of the brain is the most likely cause of the patient's left-
sided weakness?
/ A. Left postcentral gyrus
/ B. Left precentral gyrus
/ C. Left supramarginal gyrus
/ D. Right postcentral gyrus
/ E. Right precentral gyrus
/ F. Right supramarginal gyrus

Explanation - Q: 3.2 Close

The correct answer is E. The right precentral gyrus contains the primary
motor cortex (area 4), which contributes to the corticospinal tract, and
controls the voluntary movements of contralateral muscles. A lesion involving
the right precentral gyrus results in a left upper motor neuron lesion.
The left postcentral gyrus (choice A) contains the primary somatosensory
cortex (areas 1,2, and 3), which receives input from the ventral posterior
nucleus of the thalamus. A lesion affecting the left postcentral gyrus results
in a right-sided sensory loss.
The left precentral gyrus (choice B) contains the primary motor cortex (area
4), which contributes to the corticospinal tract and controls the voluntary
movements of contralateral muscles. A lesion involving the left precentral
gyrus results in a right upper motor neuron lesion.
The right postcentral gyrus (choice D) contains the primary somatosensory
cortex (areas 1,2, and 3), which receives input from the ventral posterior
nucleus of the thalamus. A lesion affecting the right postcentral gyrus results
in a left-sided sensory loss.
The supramarginal gyri (choices C and F) is responsible for the interrelation
of somatosensory, auditory, and visual input. A lesion of the supramarginal
gyrus results in an ideomotor apraxia, and ideational apraxia, a facial
apraxia, and a conduction aphasia (if located on the dominant side).




Question 3 of 3

Which of the following is the most likely finding on the physical examination of
this patient?
/ A. Atrophy of the left leg
/ B. CIonus in the left leg
/ C. Decreased reflexes in the left leg
/ D. Decreased tone in the left leg
/ E. Fasciculations of the left leg

Explanation - Q: 3.3 Close

The correct answer is B. Clonus is an upper motor neuron sign
characterized by rhythmic contractions of muscles in response to a sudden,
passive movement. This patient has a cortical lesion causing a upper motor
neuron weakness in his left side. Physical findings that indicate an upper
motor neuron lesion are: hyperreflexia, increased tone, clasp-knife spasticity,
the presence of a Babinski sign, and clonus.
The remaining options (choices A, C, D, and E) are all lower motor neuron
signs.







A 54-year-old right-handed woman with chronic atrial fibrillation runs out of her
warfarin and does not renew her prescription. Two weeks later,
she experiences a sudden inability to comprehend language. She can produce
fluent speech, and some intelligible phrases, but can not
repeat a phrase, follow commands, or read or write.

Question 1 of 3

What is the most likely location of the lesion causing this patient's speech deficit?
/ A. Frontal and temporal lobes
/ B. Inferior frontal gyrus
/ C. Posterior parieto-occipital lobe
/ D. Superior temporal gyrus
/ E. Supramarginal gyrus and underlying white matter

Explanation - Q: 4.1 Close

The correct answer is D. Wernicke's area is located in the dominant
temporal lobe, in the superior temporal gyrus.
The frontal and temporal lobes (choice A) are the locations for both Broca's
and Wernicke's areas. A lesion involving both of these area would result in a
global aphasia.
The inferior frontal gyrus (choice B) contains Broca's area and is located in
the dominant inferior frontal gyrus.
A posterior parieto-occipital lobe (choice C) lesion would result in a
transcortical sensory aphasia.
The supramarginal gyrus and underlying white matter (choice E) is the
location of the arcuate fasciculus. A lesion in this area would result in a
conductive aphasia.

Question 2 of 3
Which of the following aphasias most closely fits this patient's clinical
presentation?
/ A. Broca's expressive aphasia
/ B. Conductive aphasia
/ C. GIobal aphasia
/ D. Transcortical sensory aphasia
/ E. Wernicke's receptive aphasia




Explanation - Q: 4.2 Close

The correct answer is E. This patient is unable to comprehend language
(written or spoken) and exhibits impaired repetition. In this condition, speech
is fluent, but malformed, with inappropriate words. These are the hallmarks of
Wernicke's (receptive) aphasia, caused by a lesion in the superior temporal
gyrus of the dominant hemisphere.
Broca's (expressive) aphasia (choice A) is characterized by an inability to
speak or write, with impaired repetition (unable to write from dictation, but
able to copy letters). Broca's aphasia is caused by a lesion in the dominant
inferior frontal gyrus (Broca's area).
Conductive aphasia (choice B) is caused by disruption of the arcuate
fasciculus, in the subcortical white matter of the upper Sylvian fissure under
the supramarginal gyrus. There is fluent paraphasic speech and impaired
repetition. Patients with a conductive aphasia understand language (both
written and spoken), and are aware of their problem.
Global aphasia (choice C) is caused by a lesion affecting both Broca's area
and Wernicke's area (usually caused by a large middle cerebral artery
stroke).
Transcortical sensory aphasia (choice D) is caused a lesion in the posterior
parietal-occipital region. A transcortical sensory aphasia is characterized by
impaired language reception, paraphasic fluent speech, and a hemianopia.


Question 3 of 3

Which of the following signs or symptoms, if present, would help to confirm the
likely location of this persons lesion?
/ A. Ataxia
/ B. Bitemporal hemianopia
/ C. Left hemiparesis
/ D. Left tongue fasciculation
/ E. Superior quadrantanopia


Explanation - Q: 4.3 Close

The correct answer is E. The simultaneous presence of several different
signs ("neighborhood signs"), although very different in nature, can help
determine the location of the lesion. In this case, the patient has Wernicke's
aphasia, a condition caused by a lesion in the superior temporal gyrus. It is
not surprising that such a patient would also have superior quadrantanopia,
which results from damage to the visual radiations that course through the
temporal lobe toward the occipital lobe. The remaining symptoms result from
lesions far removed from this location.
Ataxia (choice A) can result from a cerebellar lesion.
Bitemporal hemianopia (choice B) can result from an optic chiasm lesion.
Left hemiparesis (choice C) can result from lesions in a variety of areas,
such as the right precentral gyrus, posterior limb of the internal capsule, crus
cerebri (midbrain), basilar pons, and medullary pyramid.
Left tongue fasciculations (choice D) can result from a lesion of the left
hypoglossal nucleus (located in the medulla) or hypoglossal nerve (exiting
from the medulla).

A 35-year-old man has hypertension, which has been difficult to control with
medication. Periodically, he experiences periods when he
develops intense symptoms including racing heart, Iightheadedness, flushing,
diaphoresis, clammy skin, headache, and a sense of impending
doom. He has gone to the emergency department of a local hospital several
times during these episodes, but by the time he is seen several
hours later, the symptoms have long passed, and nothing can be found on
physical examination or serum chemistry studies.
Question 1 of 6
The patient's physician orders a 24-hour urine to be collected, which is found to
contain significantly elevated levels of vanillylmandelic acid.
This compound is a degradation product of which of the following?
/ A. Acetylcholine
/ B. Cholesterol
/ C. Epinephrine
/ D. Serotonin
/ E. Testosterone

Explanation - Q: 1.1 Close

The correct answer is C. Vanillylmandelic acid (VMA) is a degradation
product of both epinephrine and norepinephrine. It is secreted into the urine,
and 24 hour screening for this metabolite (often along with the dopamine
degradation product homovanillic acid, HVA) is used to look for excessive
catecholamine secretion. The half-lives of epinephrine, norepinephrine, and
dopamine are all very short and consequently direct measurement in serum
of these species only provides a clinical answer if a patient is having a
hypertensive paroxysm at the time. Two enzymes in these degradative
pathways of which you should be aware are catechol-O-methyl transferase
(COMT: adds methyl groups to hydroxyl moieties) and monamine oxidase
(MAO: removes amino groups, oxidizing the nearby carbon to an aldehyde).
Acetylcholine (choice A) is usually not measured in the clinical laboratory.
Cholesterol (choice B) is measured in serum, and is unrelated to this
patient's disease.
Serotonin (choice D) is produced by carcinoid tumors, and can be measured
directly in serum, or its metabolite, 5-hydroxyindoleacetic acid (5-HIAA), can
be measured in urine.
Testosterone (choice E) can be measured directly in serum, and its
precursors and metabolites (including DHT, androstenedione, 3-alpha-
androstenediol glucuronide, DHEA) can also be measured in serum.


Question 2 of 6
Which of the following is the most likely diagnosis?
/ A. Leiomyosarcoma
/ B. Lymphoma
/ C. Neuroblastoma
/ D. Pheochromocytoma
/ E. Small cell carcinoma

Explanation - Q: 1.2 Close

The correct answer is D. This patient most likely has a pheochromocytoma.
Pheochromocytoma is a rare tumor that is often suspected and seldom
found. (Its incidence on the USMLE is very much higher than its incidence in
general practice.) The tumors can secrete catecholamines such as
epinephrine, norepinephrine, and/or dopamine, and the urinary determination
of metabolites (see previous question) is the most reliable method of
specifically suggesting the diagnosis. The vast majority of patients with
pheochromocytoma have either paroxysmal or persistent hypertension.
Patients who have a strong paroxysmal character to the secretion may have
episodes similar to those described in the question stem. Neuroblastoma
(choice C) can also secrete catecholamines, but is a tumor of childhood.
Lymphomas (choice B) and leiomyosarcomas (choice A, malignant tumor of
smooth muscle) do not secrete catecholamines.
Small cell carcinoma (choice E) is known for the large number of
paraneoplastic syndromes it can produce, but does not usually secrete
catecholamines and would be very unusual in a man this young.


Question 3 of 6
The tumors producing this patient's symptoms are usually found in which of the
following sites?
/ A. Adrenal gland
/ B. Pancreas
/ C. Salivary gland
/ D. Thymus
/ E. Thyroid gland

Explanation - Q: 1.3 Close

The correct answer is A. Approximately 80% of pheochromocytomas are
found in the adrenal medulla, with the remainder being found in a wide
variety of other tissues derived from neural crest cells, including, among
others, the paraganglia of the sympathetic chain, along the aorta, in the
carotid body, in the genital urinary tract, and in the brain. The other sites
listed in the choices are not particularly prone to develop
pheochromocytoma.

Question 4 of 6
Roughly what percentage of this patient's tumor type behave in a malignant
fashion?
/ A. 5-10%
/ B. 20-30%
/ C. 50-60%
/ D. 70-80%
/ E. 90-95%


Explanation - Q: 1.4 Close

The correct answer is A. 5-10% of pheochromocytomas overall are
malignant; the proportion of malignant cases is higher (30%) in the extra-
adrenal pheochromocytomas. 10% (20% in children) of the adrenal tumors
are bilateral. Pheochromocytomas typically weigh 50-200 grams
(uncommonly up to several kilograms) and are composed of nests of
chromaffin cells that often appear cytologically bizarre, even when the
tumors do not metastasize. Surgical resection is the preferred treatment
modality; alpha and beta blockers are used to block the effects of the excess
catecholamines prior to and during surgery.

Question 5 of 6
This patient's lesion has been associated with which of the following thyroid
disorders?
/ A. Follicular carcinoma
/ B. Graves disease
/ C. Hashimoto disease
/ D. Medullary carcinoma
/ E. Papillary carcinoma

Explanation - Q: 1.5 Close

The correct answer is D. Both pheochromocytoma and medullary
carcinoma of the thyroid can occur as part of familial multiple endocrine
neoplasia, types IIA (Sipple syndrome) and IIb (mucosal neuronal
syndrome). The medullary carcinoma is derived from the parafollicular C
cells that secrete calcitonin, rather than thyroxine. Pheochromocytoma also
has associations with neurofibromatosis and von Hippel-Lindau syndrome.
Follicular carcinoma (choice A) and papillary carcinoma (choice E) of the
thyroid are cancers of the thyroid follicular epithelium, and are not associated
with pheochromocytoma.
Graves disease (choice B) and Hashimoto disease (choice C) are both
autoimmune thyroid disorders with no association with pheochromocytoma.


Question 6 of 6
This patient is scheduled for surgical removal of the tumor. Which of the following
agents should be administered before the surgery?
/ A. Iodide
/ B. Lorazepam
/ C. Phenoxybenzamine
/ D. Propylthiouracil
/ E. Spironolactone

Explanation - Q: 1.6 Close

The correct answer is C. Phenoxybenzamine, a nonselective alpha-
blocking agent related to the nitrogen mustards, is indicated for the treatment
of pheochromocytoma. It binds covalently to both alpha-1 and alpha-2-
adrenergic receptors. After this medication is injected, a few hours must
elapse before actual blockade occurs. By blocking alpha-receptors,
phenoxybenzamine prevents the constriction of peripheral blood vessels,
producing a reflex tachycardia. The desired action in patients with
pheochromocytoma is related to the "epinephrine reversal," seen in classic
drug traces. In epinephrine reversal, unopposed epinephrine increases blood
pressure. After administration of an alpha antagonist (e.g.,
phenoxybenzamine, phentolamine), subsequent epinephrine administration
decreases blood pressure because of unopposed beta stimulation.
Iodide (choice A) inhibits the release and the biosynthesis of T
4
and T
3
.
Iodide decreases the size and vascularity of the thyroid gland, hence makes
it the preoperative treatment of choice for Graves disease.
Benzodiazepines, such as lorazepam (choice B), potentiate the effects of
gamma-aminobutyrate (GABA), and are used for sedation, hypnosis, muscle
relaxation, and anxiolysis. Although these agents may help to calm the
patient, they would not ameliorate the patient's condition.
Propylthiouracil (choice D) inhibits the synthesis of the thyroid hormones.
This agent does not inactivate existing T
4
and T
3
; however, it is able to inhibit
the peripheral conversion of T
4
to T
3
. It is indicated for long-term hyperthyroid
therapy, which may lead to disease remission, as well as short-term
treatment before thyroidectomy or radioactive iodine therapy.
Spironolactone (choice E) is indicated for the treatment of edematous states
as well as the prophylaxis and treatment of hypokalemia. It is commonly
combined with other non-potassium sparing diuretics to prevent the
appearance of hypokalemia.



A 41-year-old woman comes to the physician's office complaining of fatigue,
muscle weakness, cramping, headaches, polydipsia, and
polyuria. She has been treated for hypertension for 6 years, and her doctors
have told her that she has renal problems. Beta-blockers, calcium
channel blockers, and diuretics have been used to control her hypertension.
There is a family history of renal disease and hypertension. Her
blood pressure is 240/140 mm Hg and her pulse is 85/min. The remainder of her
examination is normaI. A routine chemical panel shows
hypokalemia, hypernatremia, and metabolic alkalosis.

Question 1 of 5

Pathologic examination of this patient would most likely reveal which of the
following findings?
/ A. Adrenal adenoma
/ B. Adrenal carcinoma
/ C. Bilateral nodular hyperplasia
/ D. Multiple adrenal adenomas
/ E. Unilateral nodular adrenal hyperplasia

Explanation - Q: 2.1 Close

The correct answer is A. 50% of patients with Conn syndrome present with
a solitary adenoma of the adrenal cortex zona glomerulosa, which secretes
aldosterone (aldosteronoma). Aldosterone-secreting adenomas are usually
less than 2 cm in diameter with a bright yellow appearance. 60% of these
lesions are found in the left adrenal gland. Histologically, these tumors are
composed of lipid-laden zona glomerulosa cells in cords, although compact
cells can also be seen. The unaffected cortex is not atrophic, since
aldosterone does not feedback on the production of ACTH.
Rarely, aldosterone can be secreted by adrenocortical carcinoma (choice
B).
Approximately 40% of patients with Conn syndrome have bilateral
hyperplasia (choice C) of the zona glomerulosa. The hyperplasia may be
micronodular, macronodular, or a mixture of both.
10% of the patients have multiple benign tumors (choice D), with the same
appearance as a solitary adenoma.
A few patients have unilateral nodular adrenal hyperplasia (choice E) that is
similar in function to adenoma.


Question 2 of 5
This patient is found to have an excessive amount of an adrenal hormone in her
plasma. Which of the following factors is a major regulator of
the synthesis of this hormone?
/ A. ACTH
/ B. Atrial natriuretic peptide
/ C. Dopamine
/ D. Renin-angiotensin system
/ E. Sodium

Explanation - Q: 2.2 Close

The correct answer is D. The major factors stimulating aldosterone
production and release by the zona glomerulosa are angiotensin II and the
serum potassium concentration. Angiotensinogen, the precursor of
angiotensin peptides, is synthesized by the liver. In the circulation, renin,
secreted by juxtaglomerular cells, cleaves four amino acids from
angiotensinogen, forming the decapeptide angiotensin I (AI). AI is cleaved by
angiotensin-converting enzyme (ACE) to form an octapeptide, angiotensin II
(AII). In the zona glomerulosa of the adrenal cortex, AII stimulates the
production of aldosterone. The mechanism of AII action involves an increase
in activity of aldosterone synthase, the key enzyme in the biosynthesis of
aldosterone. AII is the principal stimulator of aldosterone production when
intravascular volume is reduced. Also found in the circulation, the des-ASP
heptapeptide (angiotensin III) is as active as angiotensin II in stimulating
aldosterone release but has much less pressor activity. Potassium is also a
major physiologic regulator of aldosterone secretion; hyperkalemia also
increases the activity of aldosterone synthase. The mechanism of potassium
effect may involve depolarization and activation of voltage-gated calcium
channels. AII produces quantitatively the most significant increase in
aldosterone production, but the hormone secretion is most sensitive to small
changes in serum potassium ion concentrations.
ACTH (choice A) stimulates aldosterone secretion, but does not appear to
play a significant role in the physiological regulation of mineralocorticoid
homeostasis.
Atrial natriuretic peptide (choice B) antagonizes the AII-stimulated release of
aldosterone.
Dopamine (choice C), acting locally as a paracrine agent, inhibits secretion
of aldosterone.
The extent of sodium (choice E) reduction that is necessary to alter
aldosterone secretion is rarely seen in pathologic, let alone physiologic
situations.


Question 3 of 5
Spironolactone is used in the treatment of patients with this disease. Which of the
following is the most important adverse reaction of
spironolactone therapy?
/ A. Anti-androgen
/ B. Cardiac arrhythmia
/ C. Dehydration
/ D. Hyperkalemia
/ E. Skin reaction

Explanation - Q: 2.3 Close

The corrects answer is A. Spironolactone is a competitive antagonist of the
aldosterone receptor. Hypokalemia and hypertension in patients with primary
aldosteronism can be controlled by spironolactone, 50-100 mg/d. Although
spironolactone is an effective aldosterone receptor antagonist, it is not
without side effects that can limit its use in the chronic treatment of this
disease. The most important are anti-androgenic reactions. Spironolactone
acts as an anti-androgen by decreasing the production of testosterone by the
adrenal gland and by preventing DHT (dihydrotestosterone) from binding to
its androgen receptor. As a result of this, in the long run, gynecomastia
occurs in more than 10% of the treated men. Impotence, loss of libido, and
menstrual irregularities are also common side effects of spironolactone
therapy. On the other side, these spironolactone features are the basis for its
usage in the treatment of hirsutism, acne, and alopecia. Spironolactone is
also used by transsexuals in the feminizing regimen because of its anti-
androgenic actions. Eplerenone is a new aldosterone antagonist that may
overcome the limitations of spironolactone.
Cardiac arrhythmia (choice B) is not a frequent adverse reaction to
spironolactone treatment, and it is seen only in the presence of significant
hyperkalemia.
Dehydration (choice C) is usually very mild, and can be prevented with
adequate water intake.
Hyperkalemia (choice D) develops in 5-10% of treated patients, especially if
renal function is compromised, or the patient is diabetic, or elderly.
Skin reactions (choice E), mostly urticaria, are rare side effects of
spironolactone therapy.


A 41-year-old woman comes to the physician's office complaining of fatigue,
muscle weakness, cramping, headaches, polydipsia, and
polyuria. She has been treated for hypertension for 6 years, and her doctors
have told her that she has renal problems. Beta-blockers, calcium
channel blockers, and diuretics have been used to control her hypertension.
There is a family history of renal disease and hypertension. Her
blood pressure is 240/140 mm Hg and her pulse is 85/min. The remainder of her
examination is normaI. A routine chemical panel shows
hypokalemia, hypernatremia, and metabolic alkalosis.
Question 4 of 5
The serum potassium level in this patient is found to be 2.5 mEq/L. Which of the
following EKG changes would most likely be expected?
/ A. Prolonged QT interval
/ B. Prominent U waves, flattened T waves
/ C. Shortened QT interval
/ D. ST segment elevation, convex upwards
/ E. TalI, peaked T waves

Explanation - Q: 2.4 Close

The correct answer is B. In hypokalemia, the triad of prominent U waves,
low amplitude T waves, and ST segment depression is a typical finding.
High-amplitude positive U waves are the usual EKG features that can be
expected in this patient. The origin of the U wave is still unclear, although
most authors correlate the U wave with the phenomenon of after
depolarizations in the ventricles. U waves become as tall as T waves at a
serum level of about 3.0 mEq/L, and at about 2.0 mEq/L, become taller than
T waves. T waves begin to flatten at a serum potassium level of about 3.0
mEq/L, and eventually may either fuse with the U waves or become inverted.
Ventricular arrhythmias may occur with hypokalemia in the presence of
digitalis.
The EKG in hypocalcemia typically shows prolongation of the QT interval
(choice A). T wave peaking or inversion can also be seen.
Shortening of the QT interval (choice C) is seen in patients with
hypercalcemia.
ST segment elevation, convex upwards (choice D) is pathognomonic for
acute myocardial infarction.
A hyperkalemic state is characterized primarily by tall, peaked T waves
(choice E). There may be also wide, flat P waves, lowering of the R wave,
and increased depth of the S wave.


Question 5 of 5
The mineralocorticoid receptor (MR) displays the same affinity for glucocorticoid
hormones as it does for the hormone that is in excess in this
patient. The sensitivity of the MR to this hormone depends on which of the
following enzymes?
/ A. AIdosterone synthase
/ B. C17,20-Iyase
/ C. 11-beta hydroxysteroid dehydrogenase type 2 (HSD2)
/ D. Na+/K+-ATPase
/ E. 17-alpha hydroxylase

Explanation - Q: 2.5 Close

The correct answer is C. Mineralocorticoid receptors (MR) are members of
a superfamily of steroid/thyroid/retinoid/orphan (STRO) receptors. MR are
intracellular and act as ligand-activated transcription factors to regulate gene
expression. The human MR is a 984 amino acid protein, and the cDNA has
been cloned and sequenced. The MR shows the same affinity for
aldosterone and glucocorticoids. Since the plasma concentration of cortisol is
much higher than that of aldosterone, a mechanism is necessary to protect
MRs from constant occupancy by glucocorticoid hormones. This mechanism
depends on the activity of 11-beta-hydroxysteroid dehydrogenase type 2
(HSD2), which converts cortisol (and corticosterone) into 11-dehydro
metabolites in the endoplasmic reticulum. These are not ligands for MR. This
permits the MR to be occupied by aldosterone as a function of its serum
levels. In the absence of, or inhibition of HSD2, the MR will be occupied by
glucocorticoids, and a permanent Na+ reabsorption will occur. This concept
can be seen at work in patients with excessive ingestion of licorice.
Glycyrrhizic acid, an active hypertensive component in licorice, and its
derivative 18-beta-glycyrrhetinic acid, inhibit HSD2 activity. Cortisol activation
of MR induces sodium retention, potassium excretion, and hydrogen ion
excretion in the kidney, mimicking the symptoms of aldosteronism:
hypertension, hypokalemia, and metabolic alkalosis.
Aldosterone synthase (choice A) converts a CH3 group at C18 of the steroid
molecule to an aldehyde group, from whence comes the name aldosterone.
This enzyme is expressed exclusively in zona glomerulosa of adrenal cortex.
Aldosterone synthase is encoded by the gene CYP11B2 and has 11-beta-
hydroxylase, 18-hydroxylase, and 18-hydroxy-dehydrogenase activity.
Activity of C17,20-lyase (choice B) is responsible for producing the
androgens, dehydroepiandrosterone (DHEA) and androstenedione.
Na+/K+-ATPase (choice D) is located in basolateral membranes of the distal
tubular cells and generates the electrochemical gradient that drives diffusion
through the sodium and potassium channels. Aldosterone stimulates gene
expression of mRNA for this enzyme.
The synthesis of cortisol requires 17-alpha hydroxylation of pregnenolone by
17-alpha hydroxylase (choice E), which is expressed only in the zona
fasciculata.


A 34-year-old man undergoing a routine physical examination is found to have a
blood pressure of 165/105 mm Hg. The measurement is
repeated 40 minutes later, and is 162/103 mm Hg. The physician asks the patient
to return the next week and the week following, and each
time repeats the evaluation yielding the following results: 170/102, 168/107,
175/108, 167/102 mm Hg.
Question 1 of 7
This patient's blood pressure should be classified as which of the following?
/ A. Optimal
/ B. Normal
/ C. High-normal
/ D. Stage 1 (mild) hypertension
/ E. Stage 2 (moderate) hypertension
/ F. Stage 3 (severe) hypertension

Explanation - Q: 3.1 Close

The correct answer is E. This patient has moderate hypertension. It is
important not to try to diagnose hypertension based on the finding of a single
abnormal blood pressure result, particularly since many patients feel
uncomfortable during medical examination and may have transient blood
pressure elevations. One protocol commonly used is to take two or more
blood pressure readings on the first examination, and then have the patient
come back twice, taking two or more blood pressure readings on the
subsequent examinations. The average systolic and diastolic pressures from
the visits after the first one are then used to stage the degree of
hypertension:
Optimal blood pressure (choice A): systolic less than 120 and diastolic less
than 80.
Normal blood pressure (choice B): systolic less than 130 and diastolic less
than 85.
High normal (choice C): systolic 130-139 or diastolic 85-89.
Stage 1 (mild) hypertension (choice D): systolic 140-159 or diastolic 90-99.
Stage 2 (moderate) hypertension (choice E): systolic 160-179 or diastolic
100-109.
Stage 3 (severe) hypertension (choice F): systolic more than 180 or diastolic
more than 110.
If the systolic and diastolic values fall into different stages, the patient is
considered to have the higher stage.


Question 2 of 7
How many Americans over the age of 5 have high blood pressure?
/ A. 50 thousand
/ B. 500 thousand
/ C. 5 million
/ D. 50 million
/ E. 150 million

Explanation - Q: 3.2 Close

The correct answer is D. 50 million Americans aged 6 and older have high
blood pressure. This corresponds to one in five Americans, or one in four
adults. The incidence of hypertension is highest among African Americans,
but other races known to have an increased incidence of hypertension
include Mexican Americans, American Indians, native Hawaiians, and some
Asian Americans. It is suspected that dietary salt intake, obesity, and genetic
differences all play a role in these racial differences. About 30% of people
with high blood pressure know that they have it, and only about 24% of
patients with high blood pressure have it controlled to pressures of less than
140/90 mm Hg (considered optimal). The remainder are either untreated or
inadequately treated. The medical implications of this are obvious.


Question 3 of 7
What percentage of these patients have essential hypertension?
/ A. Less than 5%
/ B. 10-15%
/ C. 40-50%
/ D. 70-80%
/ E. 90-95%

Explanation - Q: 3.3 Close

The correct answer is E. Major risk factors for essential hypertension
(cause is unknown) include diabetes mellitus, family history of hypertension
or cardiovascular disease, high cholesterol, obesity, smoking, high salt diet,
alcohol use, and stress. Additionally, older individuals, African Americans,
and males are at increased risk. Nonetheless, in 90-95% of individuals with
hypertension, the cause is never identified, and the individual is considered
to have essential or primary hypertension. This means that, in practice,
although physicians should always keep in mind the possibility of other,
specific causes of hypertension (e.g., renal artery stenosis,
pheochromocytoma, other endocrine disease, coarctation of the aorta), these
other conditions will not usually be found.


Question 4 of 7
Years of untreated hypertension would be most likely to cause which of the
following cardiac changes?
/ A. Large vegetations on cardiac valves
/ B. Left ventricular hypertrophy
/ C. Patent foramen ovale
/ D. Pericarditis
/ E. Pulmonary stenosis

Explanation - Q: 3.4 Close

The correct answer is B. High blood pressure directly killed 43,000
Americans, and contributed to the deaths of 227,000 additional Americans.
Direct deaths are due to processes like stroke, ruptured berry aneurysm, and
ruptured aortic aneurysm. Many of the indirect deaths are related to the
vascular damage that high blood pressure causes. High blood pressure is a
major contributor to the formation of atherosclerotic plaques, and has a much
more than additive effect when added to other atherosclerotic risk factors,
such as obesity and diabetes mellitus. In addition to the large vessel
damage, small arterioles can also be damaged, which are most apparent in
the kidney and eye. The cardiac complication of hypertension that you are
most likely to be asked about on a USMLE examination is left ventricular
hypertrophy. This occurs in 15 to 20% of patients with hypertension,
apparently as a result of the response to the various stimuli that accompany
blood pressure elevation. The left ventricular hypertrophy may be either
concentric (involving the entire chamber) or eccentric (involving localized
sites, often including the septum). Concentric hypertrophy is thought to be an
indicator of poor prognosis, and may lead to the development of first
diastolic, and then later systolic dysfunction of the cardiac muscle. In addition
to the left ventricular hypertrophy, other changes have been associated with
hypertension, including left atrial abnormalities, aortic insufficiency, heart
failure, myocardial ischemia, and cardiac arrhythmias.
Large vegetations on cardiac valves (choice A) suggests endocarditis.
Patent foramen ovale (choice C) is a congenital lesion.
Pericarditis (choice D) can be due to infection, uremia, and inflammatory
disorders.
Pulmonary stenosis (choice E) is usually congenital in origin.


*** Commercial version is infinite. Order at http://www.structurise.com/kleptomania ***
Question 5 of 7
Patients with hypertension would be most likely to have which of the following
findings on renal biopsy?
/ A. Crescent formation
/ B. Hyaline arteriosclerosis
/ C. KimmelstieI-Wilson nodules
/ D. Papillary necrosis
/ E. Subepithelial electron-dense humps

Explanation - Q: 3.5 Close

The correct answer is B. Hyaline arteriosclerosis is the characteristic lesion
seen on biopsy in patients with essential hypertension. This lesion is
associated with thickening of the wall (due to deposition of serum
components) and narrowing of the lumen of the afferent arterioles that enter
the glomerulus. The lesion may be associated with a secondary
obsolescence of the glomeruli.
Crescent formation (choice A) is seen in rapidly progressive
glomerulonephritis.
Kimmelstiel-Wilson nodules (choice C) are a feature of diabetic glomeruli.
Papillary necrosis (choice D) can be seen in analgesic abuse, diabetes
mellitus, pyelonephritis, sickle cell disease, and urinary tract obstruction.
Subepithelial electron-dense humps (choice E) are a feature of post-
infection glomerulonephritis.



Question 6 of 7
Examination of the eye of a patient with long-standing hypertension shows
"cotton wool spots." These are due to which of the following?
/ A. Crystal deposition in the lens
/ B. Hemorrhage in the lens
/ C. Hemorrhage in the retina
/ D. Ischemia of the lens
/ E. Ischemia of the retina

Explanation - Q: 3.6 Close

The correct answer is E. Early changes seen on ocular examination of
hypertensive individuals can include narrowing of arteries and arteriovenous
junction changes. Late changes seen can include deposits of lipids in the
eye, cotton wool spots, bleeding in the eye (which may cause retinal
detachment), venous occlusion (which may cause transient or permanent
loss of vision), and new vessel growth. The cotton wool spots are actually
small areas of transient retinal ischemia, and usually resolve within days.
Cotton wool spots are also commonly seen in diabetics and HIV patients.
Crystal deposition in the lens (choice A) produces a cataract.
Choices B and D are distracters.
Hemorrhage in the retina (choice C) can produce "flame lesions" and retinal
detachment.


Question 7 of 7
The medical therapy of hypertension offers innumerable choices but, in this case,
the decision was made to treat the patient with an ACE
inhibitor. Which of the following drugs belongs to this class?
/ A. Atenolol
/ B. Diltiazem
/ C. Enalapril
/ D. Hydrochlorothiazide
/ E. Losartan

Explanation - Q: 3.7 Close

The correct answer is C. Life style modifications (decreased salt intake,
weight reduction, more exercise) offer some help in reducing blood pressure,
but most patients with high blood pressure eventually require medications.
We now have an enormous array of medications that can be used in the
treatment of hypertension. This is good for the patients, because it means
that it is almost always possible to find (with enough time) some effective
medication that a particular patient can tolerate. In actual practice, most
physicians develop a few favorite drugs with which they often start patients,
and then modify the medications based on blood pressure response, patient
complaints, and coexisting diseases. Classes of hypertensive medications
include diuretics, beta-blockers, long-acting Ca
++
blockers, ACE-inhibitors,
angiotensin II receptor blockers, and alpha-adrenergic blockers. Examples of
ACE-inhibitors include captopril, benazepril, enalapril, fosinopril, lisinopril,
moexipril, quinapril, ramipril, and trandolapril. These agents act by disrupting
the renin-angiotensin system by blocking angiotensin converting enzyme.
Atenolol (choice A) is a beta-1 blocker and has direct effects on blood
vessels and cardiac function.
Diltiazem (choice B) is a calcium channel blocker, which partially inhibits
smooth and cardiac muscle function.
Hydrochlorothiazide (choice D) is a diuretic and acts by decreasing the
volume of blood in the vascular space.
Losartan (choice E) is an angiotensin II receptor blocker, and disrupts the
renin-angiotensin system by blocking the actions of angiotensin II. Some
patients who have trouble tolerating ACE inhibitors do well with angiotensin II
receptor blockers.


A 19-year-old woman presents to her doctor's office for an annual physical
examination. She has been previously healthy and is currently doing
well without complaints. She is a non-smoker and has no significant past medical
history or family history. Her temperature is 36.9 C (98.5 F),
blood pressure is 160/90 mm Hg (confirmed in all extremities), pulse is 84/min,
and respirations are 16/min. Her pulses are symmetric and
equaI, her cardiac and pulmonary examinations are unremarkable, and there is
an abdominal bruit with a systolic and diastolic component.

Serum chemistry reveals:

Sodium 145 mEq/L
Potassium 3.1 mEq/L
Chloride 102 mEq/L
Bicarbonate 28 mEq/L
BIood urea nitrogen 14 mg/dL
Creatinine 1.0 mg/dL
GIucose 80 mg/dL





Question 1 of 5

Which of the following is the most likely cause of her elevated blood pressure?
/ A. Coarctation of the aorta
/ B. Cushing syndrome
/ C. Pheochromocytoma
/ D. Renovascular hypertension
/ E. Thyrotoxicosis

Explanation - Q: 4.1 Close

The correct answer is D. Secondary hypertension should be considered in
a young previously healthy patient with new onset hypertension. In this case,
renovascular hypertension should be highly suspected in the presence of an
abdominal bruit on physical exam. Abdominal bruits, particularly those with a
systolic and diastolic component, are specific for renovascular hypertension
due to renal artery stenosis. Renovascular hypertension is the most common
cause of correctable secondary hypertension.
Coarctation of the aorta (choice A) is a major cause of hypertension in
young children. It is characterized by diminished peripheral pulses,
particularly in the lower extremities. The hypertension is often limited to the
upper extremities, as the narrowing usually occurs distal to the origin of the
subclavian arteries.
Cushing syndrome (choice B) can cause hypertension, but one should find
other suggestive findings on physical examination, including central obesity,
moon facies, dorsal hump, purple striae, and ecchymoses.
Pheochromocytoma (choice C) can also cause secondary hypertension, but
patients have paroxysms of blood pressure elevation, which correlate with
symptomatic episodes of sweating and palpitations.
Thyrotoxicosis (choice E) can also cause secondary hypertension, but one
should expect to find other symptoms of hyperthyroidism such as
restlessness, tremor, heat intolerance, hyperdefecation, weight loss in spite
of increased appetite, and menstrual abnormalities in females.


Question 2 of 5
Which of the following is the most likely cause of the low potassium in this
patient?
/ A. Acidosis
/ B. Gastrointestinal wasting
/ C. Hyperaldosteronism
/ D. Inadequate nutritional intake
/ E. Thyrotoxicosis

Explanation - Q: 4.2 Close

The correct answer is C. In renovascular hypertension, there is decreased
perfusion of the renal tissue, which activates the renin-angiotensin system.
This, in turn, stimulates the oversecretion of aldosterone, which acts on the
distal convoluted tubule to enhance sodium reabsorption in exchange for
potassium excretion.
Alkalosis, rather than acidosis (choice A), is associated with hypokalemia.
Gastrointestinal wasting (choice B) can cause hypokalemia but there is no
indication of diarrhea or vomiting in this patient.
Inadequate nutritional intake (choice D) can also cause hypokalemia, but
this patient gives no history of poor nutrition.
Thyrotoxicosis (choice E) can cause hyperdefecation resulting in diarrhea,
which could cause hypokalemia, but this patient does not have signs or
symptoms suggestive of thyrotoxicosis.


Question 3 of 5
The patient is initially treated with atenolol to lower her blood pressure. Which of
the following is the mechanism of action of this medication?
/ A. AIpha-1 receptor blockade
/ B. AIpha-2 receptor stimulation
/ C. Calcium channel blockade
/ D. Nonselective beta blockade
/ E. Selective beta-1 receptor blockade

Explanation - Q: 4.3 Close

The correct answer is E. Atenolol is a selective beta-1 receptor blocker. It
acts to decrease cardiac output, decrease heart rate, and decrease
contractility, thus decreasing blood pressure. Metoprolol and esmolol are
also selective beta-1 receptor blockers.
Alpha-1 receptor blockers (choice A), such as terazosin and doxazosin, are
used in the treatment of hypertension and benign prostatic hypertrophy.
Alpha-2 receptor stimulation (choice B) is a mechanism of action used by
clonidine. It is a centrally acting antihypertensive agent that lowers blood
pressure and heart rate.
Calcium channel blockade (choice C) in vascular smooth muscle causes
vasodilation, and in the heart, causes a decrease in contractility. Examples of
calcium channel blockers include nifedipine, verapamil, and diltiazem.
Nonselective beta blockers (choice D) block beta-2 receptors in addition to
beta-1 receptors. Examples of nonselective beta-blockers include
propranolol, timolol, pindolol, and labetalol.


Question 4 of 5
Which of the following tests would help confirm the diagnosis in this patient?
/ A. Bilateral arteriography with renal vein and systemic renin measurements
/ B. Renal biopsy
/ C. Thyroid function studies
/ D. Twenty-four hour urine evaluation for creatinine clearance
/ E. Urine catecholamine levels

Explanation - Q: 4.4 Close

The correct answer is A. Bilateral arteriography with renal vein and
systemic renin measurements is the most definitive diagnostic procedure for
the diagnosis of renal artery stenosis. Arteriography can demonstrate renal
artery stenosis and comparison of renin levels help confirm the diagnosis.
Renal biopsy (choice B) would be done if renal parenchymal disease such
as glomerulonephritis were suspected. In renovascular hypertension, the
pathology occurs in the renal artery and/or one of its major branches.
Thyroid function studies (choice C) would be helpful if this patient had signs
and symptoms suggestive of thyrotoxicosis.
Twenty-four hour urine evaluation for creatinine clearance (choice D) would
estimate the glomerular filtration rate in this patient, but it would not be
helpful in suggesting a possible cause of the hypertension.
Urine catecholamine levels (choice E) would be helpful if this patient were
suspected of having a pheochromocytoma.








Question 5 of 5

Biopsy of the affected tissue would likely reveal which of the following?
/ A. Adrenocortical adenoma
/ B. Atrophic thyroid follicles with dense, focal lymphocytic infiltration
/ C. Fibromuscular dysplasia
/ D. Hyperplasia of the zona glomerulosa of the adrenal gland
/ E. Psammoma bodies

Explanation - Q: 4.5 Close

The correct answer is C. Fibromuscular dysplasia is an intrinsic structural
abnormality of the arterial wall. It generally occurs in young women. When it
occurs in the renal vein and/or one of its major branches, it causes renal
artery stenosis. The renal artery stenosis activates the renin-angiotensin
system, causing an increase in aldosterone secretion and hypertension.
Adrenocortical adenoma (choice A) is an aldosterone-secreting tumor of the
adrenal gland. Hyperplasia of the zona glomerulosa of the adrenal gland
(choice D) also results in increased secretion of aldosterone. They both
cause a primary hyperaldosteronism and hypertension, but are both
associated with low renin levels. Patients with renovascular hypertension
have increased release of renin, which causes secondary
hyperaldosteronism.
Atrophic thyroid follicles with dense, focal lymphocytic infiltration (choice B)
are seen in Hashimoto thyroiditis.
Psammoma bodies (choice E) are seen in papillary carcinoma, a malignant
tumor of the thyroid gland.

A 32-year-old woman presents to an emergency department and loses
consciousness while going through the financial screening process.
According to her husband, she has been having severe vomiting and diarrhea.
He took her to the emergency room when she also began to
have severe pain in her back, abdomen, and legs. Physical examination
demonstrates marked hypotension, dry mucus membranes, Ioss of
skin turgor, and a generalized hyperpigmentation of the skin that is most
noticeable on scars, skin folds, knees, knuckles, and lips.
Question 1 of 5
Which of the following features of this woman's presentation is most suggestive
of the likely diagnosis?
/ A. Abdominal pain
/ B. Hyperpigmentation
/ C. Hypotension
/ D. Loss of consciousness
/ E. Vomiting

Explanation - Q: 1.1 Close

The correct answer is B. One of the tricks of physical diagnosis is to have a
sense of which features of a patient's presentation are most suggestive of
specific illnesses. This woman is obviously severely ill, but the majority of her
signs and symptoms (including those listed in choices A, C, D, and E) are
very nonspecific and may be seen in many different diseases affecting a
variety of organ systems. The most specific finding is the unusual pattern of
hyperpigmentation.


Question 2 of 5
Which of the following is the most likely diagnosis?
/ A. Addison disease
/ B. Diabetes mellitus type 2
/ C. Hyperthyroidism
/ D. Hypoparathyroidism
/ E. Pheochromocytoma

Explanation - Q: 1.2 Close

The correct answer is A. While roughly 75% of patients with Addison
disease have been previously diagnosed when they develop a crisis, the
remaining 25% have not. Most patients' symptoms are nonspecific (although
usually severe) and many physicians have never seen a case (Addison
disease is rare, with an incidence of 1 in 100,000). These facts together
mean that this potentially fatal crisis is very often misdiagnosed, and vital
time is wasted while working the patient up as a probable sepsis, surgical
abdomen, gastrointestinal illness, or back pain case. An alert physician who
notices the hyperpigmentation of both sun-exposed and unexposed skin and
also knows to associate this with Addison disease could save this patient's
life. (A helpful clue in patients with naturally pigmented skin is to look for the
specific areas of hyperpigmentation mentioned in the case history.) Those
patients who do not present with a crisis may come to medical attention
because of chronic fatigue and muscle weakness, loss of appetite with
weight loss, chronic nausea and vomiting, hyperkalemia picked up on routine
serum chemistry screens, hypotension with fainting spells, or irritability and
depression. Other features can include craving of salty foods, hypoglycemia,
and irregular menstrual periods. The symptoms usually develop slowly, and
may be ignored until exacerbated by illness or accident, thereby producing
an Addisonian crisis. Once Addison disease is suspected, the diagnosis can
be confirmed with an ACTH stimulation test, in which cortisol secretion is
measured after exogenous ACTH is administered.
A patient with undiagnosed type 2 diabetes mellitus 2 (choice B) might also
lose consciousness (most likely due to hyperosmolar coma) in the
emergency room, but would not have the skin pigmentation, pain, or nausea
and vomiting seen in this case. Hyperthyroidism (choice C) causes heat
intolerance, nervousness, and can occasionally produce an arrhythmia
severe enough to cause loss of consciousness, but would not cause the skin
pigmentation, pain, or nausea and vomiting seen in this case.
Hypoparathyroidism (choice D) causes hypocalcemia with tetany, but would
not cause this patient's symptom pattern.
Pheochromocytoma (choice E) could cause nausea and vomiting,
abdominal pain, and loss of consciousness secondary to arrhythmia, but
would not cause hyperpigmentation.


Question 3 of 5
This patient's severe hypotension is most likely related to abnormal serum levels
of which of the following hormones?
/ A. AIdosterone
/ B. Epinephrine
/ C. Insulin
/ D. Parathyroid hormone
/ E. Thyroxine

Explanation - Q: 1.3 Close

The correct answer is A. The hormones that are deficient in Addison
disease are cortisol (always) and aldosterone (sometimes). Aldosterone is a
mineralocorticoid produced by the adrenal cortex that normally helps the
body maintain blood pressure and water and salt balance. It acts by
stimulating the kidney to retain sodium and excrete potassium. Hypotension
and hyperkalemia can develop when aldosterone levels are too low. Cortisol
also helps to maintain blood pressure and cardiovascular function, and has
additional actions, including modulating the inflammatory response and
opposing insulin's actions in the regulation of protein, carbohydrate, and fat
metabolism. Treatment of patients with Addison disease usually includes
glucocorticoid replacement (often with oral hydrocortisone) and may include
mineralocorticoid replacement (typically with oral fludrocortisone).
Epinephrine (choice B) stimulates the adrenergic system and tends to
increase blood pressure secondary to cardiotropic and vasoactive effects.
Insulin (choice C) affects the metabolism of carbohydrate, fat, and protein,
but does not directly alter blood pressure.
Parathyroid hormone (choice D) affects serum calcium levels.
Thyroxine (choice E) tends to increase metabolic rate, and, when deficient,
may be associated with low blood pressure, but would not produce the
clinical presentation seen with this patient.


Question 4 of 5
Which of the following is the most common cause in the United States of this
patient's probable condition?
/ A. Amyloidosis
/ B. Autoimmune disease
/ C. Metastatic cancer
/ D. Surgical removal
/ E. Tuberculosis

Explanation - Q: 1.4 Close

The correct answer is B. Primary adrenal insufficiency, or Addison disease,
only develops when at least 90% of the adrenal cortex has been destroyed.
While, at the time that Addison initially described the disease, tuberculosis
(choice E) was the most common cause (and still causes approximately
20% of cases), medical control of tuberculosis has made this cause much
less likely, and a gradual autoimmune destruction of the adrenal cortex now
causes approximately 70% of all cases of Addison disease in developed
countries. Less common causes of Addison disease include chronic
infections (usually fungal), metastatic cancer (choice C), amyloidosis
(choice A), and surgical removal of the adrenal glands (choice D).
Secondary adrenal insufficiency differs from primary adrenal insufficiency by
the lack of skin pigmentation, and is usually due to a lack of pituitary-
secreted ACTH, most commonly either following cessation of glucocorticoid
therapy for other diseases, or removal of an ACTH-producing tumor of the
pituitary gland. Secondary adrenal insufficiency often eventually resolves
spontaneously, although interim hormonal support is usually required.


Question 5 of 5
If this patient had been a child who had a familial form of this disease, which of
the following would most likely also be affected?
/ A. Exocrine pancreas
/ B. Pancreatic islets
/ C. Parathyroid gland
/ D. Salivary glands
/ E. Thyroid gland

Explanation - Q: 1.5 Close

The correct answer is C. The autoimmune form of Addison disease can
occur either as an isolated condition or as part of a polyendocrine deficiency
syndrome, which is possibly inherited, since multiple family members may
develop endocrine deficiencies. These associations are important because
patients with autoimmune Addison disease should be initially evaluated, and
then periodically screened for development of other endocrine deficiencies.
Type I polyendocrine deficiency develops in childhood, and may include
adrenal insufficiency, hypoparathyroidism (choice C), pernicious anemia,
chronic active hepatitis, chronic Candida infections, and slow sexual
development. Type II polyendocrine deficiency (Schmidt syndrome) develops
in young adults and may include adrenal insufficiency, hypothyroidism
(choice E), diabetes mellitus type I (affecting the pancreatic islets, choice
B), vitiligo, and slow sexual development. The exocrine pancreas (choice A)
and the salivary glands (choice D) are not affected in either type.



A 32-year-old woman goes to the local emergency department because she has
been feeling increasingly ilI. When she tries to stand to go
into the examining room, she loses consciousness and falls to the floor. Her
blood pressure is 70/40 mm Hg. Her temperature is 40.1 C (104
F), and her skin shows a diffuse, sunburn-Iike, erythema. IV fluids are started,
and the woman's husband is questioned. He reports that her
symptoms began approximately 24 hours previously and that she has been
experiencing headache, sore throat, profound lethargy, vomiting,
profuse diarrhea, muscle pain, and the rash. Examination of the patient's vagina
demonstrates a blood-filled tampon. Gram's stain of a vaginal
smear shows nearly complete replacement of the normal vaginal flora by gram-
positive cocci.
Question 1 of 6
Which of the following is the most likely diagnosis?
/ A. Kawasaki syndrome
/ B. Meningococcemia
/ C. Reye syndrome
/ D. Rocky Mountain spotted fever
/ E. Toxic shock syndrome

Explanation - Q: 2.1 Close

The correct answer is E. This woman's presentation is typical for toxic
shock syndrome. In patients who survive, the rash eventually becomes
desquamating. Toxic shock syndrome is a rare, but very dangerous disease
with a 5% mortality rate, primarily related to the complications of shock. Many
cases have been related to the use of super absorbent tampons, which favor
bacterial overgrowth. Since removal of some of the more strongly implicated
tampons from the market, the incidence of toxic shock syndrome has
dropped and now appears to be about 3 cases/100,000 menstruating
women. Cases can also be seen complicating postpartum or postoperative
infections. Gram-positive cocci would not cause any of the other conditions
listed.
Kawasaki syndrome (choice A) usually occurs in children, and while it
causes severe rash, it does not usually cause shock.
Meningococcemia (choice B) and Rocky Mountain spotted fever (choice D)
both usually cause macular rashes.
Reye syndrome (choice C) can cause rash, but it is almost always seen in
individuals less than 18 years of age.


Question 2 of 6
This condition is most closely associated with which of the following organisms?





Explanation - Q: 2.2 Close

The correct answer is C. Almost all cases of classic toxic shock syndrome
have been associated with exotoxin-producing strains of phage group 1
Staphylococcus aureus. Streptococcus pyogenes can also, less commonly,
produce a toxic shock-like syndrome. The toxin implicated in staphylococcal
toxic shock syndrome has been called toxic shock syndrome toxin-1 (TSST-
1). It is thought that women who develop toxic shock syndrome following
tampon use have vaginas that were previously colonized by small numbers
of the toxin-producing bacteria, and that then as the vaginal conditions
change, bacterial overgrowth with the staphylococci occurs. Toxic shock
syndrome is unusual, in that sepsis-like systemic effects are seen even
though true bacteremia does not occur, just absorption of a very potent toxin.
Antibiotic therapy is usually included in treatment of toxic shock, but the
intent is to eradicate the colonization and prevent recurrence, rather than
treat an active, true infection.
Moraxella catarrhalis(choice A) is a gram-negative coccus that forms part of
the normal nasal flora, and has been occasionally implicated as a cause of
otitis media, bronchitis, and bronchopneumonia.
Neisseria gonorrhoeae(choice B) is a gram-negative coccus that causes
gonorrhea.
Streptococcus agalactiae(choice D) is a group B, beta-hemolytic
streptococcus that colonizes the vagina (with no harm to the mother) and
causes neonatal septicemia and meningitis.
Streptococcus pyogenes(choice E) is a group A, beta-hemolytic
streptococcus that also produces potent toxins, and has been implicated in
pharyngitis, scarlet fever, impetigo, rheumatic fever, and acute
glomerulonephritis.



Question 3 of 6
The exotoxin implicated in this patient's disease is unusual in that it is a
superantigen. This means which of the following?
/ A. It can bind to the inside surfaces of the normal peptide grooves of both lgA
and the TCR molecule
/ B. It can bind to the inside surfaces of the normal peptide grooves of both lgE
and the TCR molecule
/ C. It can bind to the inside surfaces of the normal peptide grooves of both the
MHC class l molecule and the TCR molecule
/ D. It can bind to the outside surfaces of the normal peptide grooves of both lgG
and the TCR molecule
/ E. It can bind to the outside surfaces of the normal peptide grooves of both the
MHC class ll molecule and the TCR molecule

Explanation - Q: 2.3 Close

The correct answer is E. Both the staphylococcal toxic shock syndrome
toxin-1 (TSST-1) and the streptococcal super antigen (SSA) are unusual in
that they tend to trigger a massive release of cytokines in response to
relatively small amounts of toxin. The reason is that, unlike normal antigens
that bind inside the normal peptide grooves of the MHC class II molecule on
the antigen presenting cell and the T cell receptor (TCR) molecule of the T
lymphocyte, these antigens recognize sites outside the grooves on these two
molecules. The result is that they can, in effect, "turn on" almost every T cell
they find, thereby producing the massive release of cytokines.
IgA (choice A), IgE (choice B), MHC class I molecules (choice C), and IgG
(choice D) are not involved in these processes.


Question 4 of 6
One of the compounds that has been implicated in the severe hypotension seen
in this patient's disease is tumor necrosis factor (TNF)-alpha.
This molecule is produced by which of the following cell types?
/ A. B Iymphocytes
/ B. Eosinophils
/ C. Macrophages
/ D. Neutrophils
/ E. T Iymphocytes

Explanation - Q: 2.4 Close

The correct answer is E. Tumor necrosis factor alpha is produced by T
lymphocytes, and it acts by increasing the fluid leakage out of the
vasculature as part of its more generalized stimulation of inflammatory
processes. It is also thought the TSST-1 may directly alter capillary
permeability. Macrophages (choice C) produce IL-1, which is also important
in triggering the severe hypotension by a similar mechanism to that of TNF-
alpha.
B lymphocytes (choice A), eosinophils (choice B), and neutrophils (choice
D) do not produce TNF-alpha and are not as important in triggering the
hypotension of toxic shock.





Question 5 of 6
Which of the following is the most appropriate pharmacotherapy?
/ A. Chloramphenicol
/ B. Gentamicin
/ C. Nafcillin
/ D. Penicillin
/ E. Tetracycline

Explanation - Q: 2.5 Close

The correct answer is C. Treatment measures for TSS include rehydration,
management of cardiac and renal failure (if present), administration of
antistaphylococcal agents (e.g., nafcillin), as well as removal of the source of
the toxin, such as removal of the tampon or drainage of an abscess.
Chloramphenicol (choice A) is an older antibacterial agent used in the
treatment of severe infections only when less toxic agents cannot be used.
Examples of such infections include Salmonella infections, H. influenzae
infections, and various types of meningitis.
Gentamicin (choice B) is a bactericidal aminoglycoside antibiotic used
primarily in the treatment of serious gram-negative infections. This agent
requires active transport across the cell membrane to exhibit activity. In an
acidic environment, the drug becomes ionized and less is transported to the
interior of the cell. Gentamicin is a concentration-dependent killer: the drug
only needs to be in contact with the bacteria for a short period of time to
exert a killing effect (a property that may help minimize toxicity). It also exerts
a postantibiotic effect in which the antimicrobial action is "seen" after drug
concentration falls below minimum inhibitory concentration.
Penicillin (choice D) is a bactericidal antibiotic with broad gram-negative and
gram-positive coverage as well as moderate anaerobic coverage. It inhibits
the biosynthesis of cell wall mucopeptides. Most hospital isolates of Staph
aureus are resistant to penicillin.
Tetracycline (choice E) has mild-to-moderate gram-negative and gram-
positive coverage. It provides coverage for many rickettsial, spirochetal, and
chlamydial infections. Tetracycline acts by inhibiting protein synthesis by
binding to the 30S ribosome. It is primarily used for uncomplicated
gonococcal infections and acne.



Question 6 of 6
The mortality in this patient's condition is high, in large part because multi-organ
failure may develop. Which of the following serum laboratory
results would suggest developing renal failure?
/ A. AIanine aminotransferase (ALT) 105 U/L
/ B. Aspartate aminotransferase (AST) 100 U/L
/ C. Creatinine 2.5 mg/dL
/ D. Serum sodium 141 mEq/L
/ E. Total creatine kinase 550 U/L

Explanation - Q: 2.6 Close

The correct answer is C. The multi-organ failure seen in toxic shock
syndrome is, in large part, due to the very difficult-to-manage hypotension as
fluid shifts out of the vascular space and into the tissues. This produces the
paradox that you can get a markedly edematous patient who is actually
hypotensive secondary to fluid losses within the body. Patients may require
10 liters or more of IV fluids per day in order to prevent multiorgan failure.
Impending renal failure is indicated by dropping urine output and rising BUN
and serum creatinine levels.
The liver is also very vulnerable, and damage is indicated by rising ALT
(choice A) and AST (choice B) levels.
Serum sodium (choice D) is usually not affected, since the fluid replacement
is usually with normal saline or similar isotonic fluids.
Muscle damage may be a prominent feature and cause leakage of creatine
kinase (choice E).



A 20-year-old college student presents to the campus health service complaining
of a purulent penile discharge. When his culture reveals
gonococcal infection, he returns to the clinic for treatment, and is given an
intramuscular injection of ceftriaxone. While checking out, he tells the
receptionist that he feels ilI. He complains of feeling weak, dizzy, and short of
breath, as well as a "funny" sensation around his mouth. On
physical examination, he is sweaty and pale. His lips appear swollen. His blood
pressure cannot be auscultated; via palpation, his systolic
pressure is estimated at 74 mm Hg. His pulse is 116/min, and respirations are
30/min.
Question 1 of 5
This is an example of which of the following types of hypersensitivity reactions?
/ A. Type l
/ B. Type ll
/ C. Type lll
/ D. Type IV

Explanation - Q: 3.1 Close

The correct answer is A. IgE antibody is produced in response to an initial
challenge by an antigen, and binds to Fc receptors on mast cells and
basophils. Reexposure to the antigen causes degranulation of mast cells and
release of vasoactive amines (e.g., histamine and adenosine), chemotactic
cytokines, and enzymes (e.g., proteases and kinins). Mast cells go on to
synthesize and release arachidonic acid metabolites (leukotrienes,
prostaglandin D2) and platelet-activating factor. These multiple mediators, in
turn, initiate a rapid inflammatory response, resulting in recruitment of
neutrophils and eosinophils, and production of increased vascular
permeability, tissue edema, and epithelial cell injury. The clinical
manifestations of anaphylaxis are urticaria, tissue swelling and angioedema,
laryngeal edema, bronchoconstriction, and hypotension, as well as
abdominal cramping and diarrhea. Other Type I type hypersensitivity
reactions are more localized, as in some forms of bronchial asthma, allergic
rhinitis, and allergic gastroenteritis.
Type II hypersensitivity reactions (choice B) include antibody-dependent
cytotoxic responses, in which antibody (IgG or IgM) binds to antigen on a cell
surface, resulting in either the activation of complement, or cell-mediated
cytotoxicity via nonspecific neutrophils, monocytes, eosinophils, and NK cells
that bind to the Fc fragment of IgG. Examples of Type II reactions are
transfusion reactions, autoimmune hemolytic anemias, erythroblastosis
fetalis, and Goodpasture syndrome. Type II hypersensitivity also includes
noncytotoxic antibody-mediated dysregulation such as the anticholinergic
receptor antibodies of myasthenia gravis and the anti-TSH receptor
antibodies of Graves disease.
Type III hypersensitivity reactions (choice C) are immune-complex mediated
reactions in which antibody-antigen complexes are formed, thereby
activating complement. The antigen may be endogenous or exogenous.
Examples are serum sickness, systemic lupus erythematosus, and some
acute glomerulonephritides.
Type IV hypersensitivity (choice D) also called delayed-type hypersensitivity,
is entirely cell-mediated and requires the presentation of antigen on a cell
surface, resulting in CD4 cell activation followed by CD8 cell-mediated direct
cytotoxicity. The most common example of this is the cutaneous tuberculin
reaction: this is also involved in contact dermatitis and in rejection of solid
organ transplants.





Question 2 of 5
Which of the following immunoglobulins initiates this patient's disease process?
/ A. IgA
/ B. IgD
/ C. IgE
/ D. IgG
/ E. IgM

Explanation - Q: 3.2 Close

The correct answer is C. Antigen binds to IgE on the surface of mast cells
and basophils, initiating anaphylaxis.
IgA (choice A) is secreted primarily by mucosal plasma cells and is the
principal immunoglobulin present in secretions. It is a major component of
immunologic defense against viruses. Approximately 1 in 800 people is IgA-
deficient.
IgD (choice B) acts as an antigen-binding receptor on B cells, although very
small quantities also circulate.
IgG (choice D) represents approximately 75% of circulating antibody and is
the predominant antibody made after rechallenge of a host with antigen, as
well as a major component of antibacterial immunity, initiating neutrophil and
monocyte responses as well as activating complement.
IgM (choice E) is the antigen-binding receptor on the B cell. IgM also
circulates as a pentamer. Since no class-switching is required to form it, IgM
constitutes the primary antibody response on initial host antigen challenge.
IgM activates complement efficiently, and is an important mediator of
rheumatoid arthritis and of immune complex diseases.


Question 3 of 5
As resuscitation efforts begin, 0.5 mg of epinephrine is administered
subcutaneously. Which adrenergic receptors, in which tissues, are
responsible for the beneficial effect of epinephrine in this patient?
/ A. AIpha-1 receptors in vascular smooth muscle, alpha-2 receptors in the
heart, and beta-1 receptors in bronchial smooth muscle
/ B. AIpha-1 receptors in vascular smooth muscle, alpha-2 receptors at
presynaptic nerve terminals, beta-1 receptors in bronchial smooth
muscle
/ C. AIpha-1 receptors in vascular smooth muscle, beta-1 receptors in the heart,
and beta-2 receptors in bronchial smooth muscle
/ D. AIpha-2 receptors in vascular smooth muscle and beta-2 receptors in
vascular smooth muscle
/ E. Beta-1 receptors in the heart alone

Explanation - Q: 3.3 Close

The correct answer is C. Epinephrine is a nonselective adrenergic agonist
and a valuable resuscitative drug because of its effects at multiple adrenergic
receptor subtypes. In the treatment of anaphylaxis, epinephrine increases
myocardial contractility, accelerates heart rate, causes constriction of
vascular smooth muscle, and causes relaxation of bronchial smooth muscle.
The principal pharmacologic effects of epinephrine that are beneficial in
anaphylaxis are mediated via: alpha-1 receptors in vascular smooth muscle,
resulting in vasoconstriction, beta-1 receptors in the heart, resulting in
increased contractility, and beta-2 receptors in bronchial smooth muscle,
resulting in relaxation and relief of bronchoconstriction. (One simple
mnemonic for the respective locations of beta-1 and beta-2 receptors is "one
heart, two lungs.") Beta-2 receptors are also found, however, in vascular
smooth muscle (especially in skeletal muscle beds), where, just as in
bronchial smooth muscle, they promote relaxation. (Epinephrine dilates
skeletal muscle vascular beds to maximize oxygen delivery for the "fight-or-
flight" response.) The resulting vasodilation in skeletal muscle vascular beds
would, by itself, tend to decrease blood pressure, which might tend to worsen
the effects of anaphylactic shock, but this effect is mitigated by the intense
alpha-1 receptor stimulation, causing vasoconstriction in multiple beds. The
principal action of alpha-2 receptors is at the presynaptic nerve terminal,
where receptor stimulation reduces the release of norepinephrine from the
nerve terminal. Epinephrine does stimulate these receptors, but it does not
really contribute to the beneficial actions of epinephrine in resuscitation.
Alpha-2 receptors are not located in the heart (except at presynaptic nerve
terminals), alpha-2 receptors do not have a significant beneficial effect in
resuscitation, and beta-1 receptors are not located in bronchial smooth
muscle, but are located in the heart (compare with choice A).
Beta-1 receptors are located in the heart, not in bronchial smooth muscle.
The adrenergic receptor that produces bronchial smooth muscle relaxation is
beta-2 (compare with choice B).
Alpha-2 receptors are located primarily at presynaptic nerve terminals, and
because the beta-2 receptors in vascular smooth muscle cause vasodilation,
this not a helpful effect of epinephrine in this case (compare with choice D).
In the patient with anaphylaxis, epinephrine is resuscitative also by raising
blood pressure via alpha-1 receptors in vascular smooth muscle and
relieving bronchoconstriction via beta-2 receptors in bronchial smooth
muscle (compare with choice E).

Question 4 of 5
The patient is also given a dose of hydrocortisone. Which of the following is the
most likely molecular mechanism by which steroids have their
effect?
/ A. Adenosine receptor blockade
/ B. Direct mast cell stabilization
/ C. Histamine receptor blockade
/ D. Inhibition of cyclooxygenase
/ E. Inhibition of phospholipase A2
/ F. Leukotriene receptor blockade

Explanation - Q: 3.4 Close

The correct answer is E. Steroids inhibit phospholipase A2, which cleaves
cell membrane phospholipids into arachidonic acid. Inhibition of this enzyme,
in turn, prevents the formation of thromboxane, as well as all the
prostaglandins and leukotrienes.
Adenosine receptor blockade(choice A) is one of the mechanisms of action
of the methylxanthines, including caffeine, theophylline, aminophylline, and
theobromine. Adenosine is a potent bronchoconstrictor. Theophylline and
aminophylline are used in the treatment of asthma.
Prevention of mast cell degranulation (choice B) is thought to be the
mechanism of action of cromolyn sodium, an inhaled drug used in asthma.
Histamine receptor blockade (choice C) is a central part of the therapy for
anaphylaxis, and is accomplished by using both H1 receptor blockers such
as diphenhydramine as well as H2 receptor blockers like cimetidine.
Inhibition of cyclooxygenase (choice D) is the principal mechanism of action
of the nonsteroidal anti-inflammatory agents, which generally inhibit both the
COX1 (constitutive) and COX2 (inducible) isoforms of this enzyme,
preventing the conversion of arachidonic acid into prostaglandin and
thromboxane precursors. Newer NSAIDs are relatively selective for COX2,
and therefore do not inhibit prostaglandin formation in normal tissue to the
same degree that they do in inflamed tissues.
Leukotriene receptor blockade (choice F) is the mechanism of action of a
new class of asthma drugs such as montelukast and zafirlukast.



Question 5 of 5
At discharge from the hospital several days later, the patient is warned that he is
allergic to cephalosporins. Which other drug should he be
warned not to take in the future?
/ A. Aztreonam
/ B. Gentamicin
/ C. Indomethacin
/ D. Penicillin
/ E. Trimethoprim-sulfamethoxazole

Explanation - Q: 3.5 Close

The correct answer is D. Penicillins and cephalosporins share a common
beta-lactam ring, which is the most likely antigen responsible for triggering
the anaphylaxis cascade in this patient.
Aztreonam (choice A) is a monobactam antibiotic which, although closely
related to the penicillins and cephalosporins, does not appear to have
antigenic cross-reactivity.
Gentamicin (choice B) is an aminoglycoside antibiotic and is not chemically
related to cephalosporins. Aminoglycosides are frequently prescribed in
conjunction with beta-lactam antibiotics because of their synergy against
gram-positive bacteria.
Indomethacin (choice C) is a nonsteroidal anti-inflammatory agent, not an
antibiotic or related to the cephalosporins. NSAIDs and aspirin can, however,
trigger severe bronchospasm and upper airway obstruction, as well as
angioedema in susceptible individuals, most frequently asthmatic patients,
probably as the result of an imbalance in prostaglandin and leukotriene
production. This is not an IgE-mediated reaction and this patient should not
be at significantly increased risk.
The combination of trimethoprim, (a diaminopyrimidine) and
sulfamethoxazole (a sulfonamide) (choice E) provides two antimicrobials
with different targets in the bacterial folic acid synthesis pathway, to create a
synergistic and widely prescribed combination. There is no chemical relation
to or cross-reactivity with cephalosporins or penicillins.


A 63-year-old man is brought into the emergency department with shortness of
breath over the past 4 hours, although the patient's family
states that he had complained of some shortness of breath and dry cough over
the past 2 months. The patient denies any significant past
medical history, fever, chills, or chest pain, and has not been taking any
medications. Review of systems is positive for a 10 pound weight loss
in the last 5 months and social history pertinent for a 20-pack-year smoking
history, but the patient had quit smoking 10 years prior. Physical
examination reveals a thin man who is alert and awake but in obvious distress
and breathing shallowly. His temperature is 37.2 C (99 F),
blood pressure is 90/75 mm Hg, pulse is 122/min, and respirations are 20/min.
The neck veins are prominent with a decline during inspiration.
The cardiac examination is significant for tachycardia and distant heart sounds,
but no murmurs/gallops/rubs are heard. Lung examination is
significant for decreased breath sounds at the bases, but no crackles or wheezes
are heard. No peripheral edema is seen. Labs are sent.
ECG reveals a sinus tachycardia at 118/min, Iow voltage QRS complexes, and
electrical alternans of the P, QRS, and T waves. A chest x-ray
film shows an enlarged cardiac silhouette, but minimal pulmonary vascular
congestion.

Question 1 of 3

Which of the following is characteristic of the patient's condition?
/ A. Pulsus alternans
/ B. Pulsus bisferiens
/ C. Pulsus et tardus
/ D. Pulsus paradoxus
/ E. Quincke's pulses

Explanation - Q: 4.1 Close

The correct answer is D. This patient has cardiac tamponade. This disorder
is characterized by elevation of intracardiac pressures, limitation of
ventricular filling, and reduction of cardiac output. The patient's presentation
is highly suggestive of this disorder, with dyspnea, tachycardia, hypotension,
neck vein distention with pulsations, and muffled heart sounds. Severe,
acute tamponade is characterized by falling arterial pressure, and rising
venous pressure, but slowly developing tamponade may resemble
congestive heart failure. The jugular venous pressure shows a prominent x
descent; in constrictive pericarditis, the y descent is prominent. The ECG in
tamponade tends to show tachycardia, but with small QRS complexes
because the pericardial fluid hinders the transmission of the impulses.
Electrical alternans (beat-to-beat alternation in ECG components) of the P,
QRS, and T waves is suggestive of effusion, often with tamponade. The
chest x-ray film tends to show a globular heart, but minimal pulmonary
edema. Pulsus paradoxus can be present in patients with cardiac
tamponade. It is present if a > 10 mm Hg decline in systolic pressure is
measured during inspiration, and is caused by impaired left ventricular filling.
Pulsus alternans (choice A) is a regular alteration in pulse pressure
amplitude in severe left ventricular dysfunction.
Pulsus bisferiens (choice B) refers to two palpable peaks in the pulse of
those with mixed aortic regurgitation and stenosis and hypertrophic
obstructive cardiomyopathy.
Pulsus et tardus (choice C) is a delayed carotid upstroke seen in aortic
stenosis.
Quincke's pulses (choice E) are subungual capillary pulsations seen in
patients with aortic regurgitation.


Question 2 of 3
Which of the following features on the echocardiogram would be diagnostic?
/ A. Anterior wall hypokinesis
/ B. Collapse of the right ventricle during diastole
/ C. Diastolic dysfunction
/ D. Left ventricular hypertrophy
/ E. Pericardial fluid collection

Explanation - Q: 4.2 Close

The correct answer is B. A patient with cardiac tamponade has pericardial
effusion (choice E) on echocardiography, but right ventricle/atrium collapse
during diastole is specific for tamponade. Patients with pericardial effusion
only (no tamponade) can be asymptomatic.
Anterior wall hypokinesis (choice A) can occur in patients with anterior
myocardial infarction.
Diastolic dysfunction (choice C) is impaired right or left ventricular filling from
long-standing hypertension.
Left ventricular hypertrophy (choice D) is enlargement of the left ventricular
wall, often from hypertension.


Question 3 of 3
Which of the following is the most appropriate immediate treatment for this
patient?
/ A. Antibiotics
/ B. Diuresis
/ C. Heparin
/ D. Pericardiocentesis
/ E. Pericardial window



Explanation - Q: 4.3 Close

The correct answer is D. The most immediate treatment for this disorder is
pericardiocentesis (placing a needle into pericardial space to aspirate fluid) if
a patient shows any signs of hemodynamic compromise.
Antibiotics (choice A) are not indicated unless there are any other signs of
infections.
Diuresis (choice B) is important for heart failure, but is not appropriate for
this patient with pending cardiovascular collapse if not treated.
Heparin (choice C) is used for anticoagulation for pulmonary embolus and
deep vein thrombus.
Pericardial window (choice E) is used in chronic pericardial effusion and
constrictive pericarditis to drain the recurrent fluid accumulation.


A 45-year-man was brought in by paramedics for vomiting blood. He was not
responding to the questions by the medical staff. The patient had
a heavy odor of alcohol on his breath. On examination, his temperature is 37.7 C
(99.8 F), blood pressure is 75/35 mm Hg, and pulse is
120/min. There is blood crusting around the patient's mouth and he is only
oriented to person, but not place or time. His neck veins are
flattened, and his heart examination shows tachycardia without any murmurs. His
lung and abdomen examinations are not significant. Rectal
examination is guaiac negative. No peripheral edema is seen. A nasogastric tube
is placed and about 1.5 Iiter of normal saline is used until
the nasogastric lavage is clear of blood. Laboratory results show:

Sodium 143 mEq/L
Potassium 4.3 mEq/L
Chloride 96 mEq/L
Bicarbonate 19 mEq/L
BUN 36 mg/dL
Creatinine 1.3 mg/dL
GIucose 89 mg/dL

A complete blood count (CBC) and a chest x-ray film are pending. His ECG
shows normal sinus rhythm at 117/min, and no other changes.






Question 1 of 6

Which of the following is the most likely diagnosis?
/ A. Adrenal insufficiency
/ B. AIcohol withdrawal
/ C. Cardiogenic shock
/ D. Hypovolemic shock
/ E. Septic shock

Explanation - Q: 5.1 Close

The correct answer is D. The patient is suffering from hypovolemic shock
secondarily from an upper GI bleed.
Adrenal insufficiency (choice A) can present with severe hypotension in the
setting of a patient who stopped taking his chronic steroid dosage. It can
occur after severe stress, trauma, or infections. It is often associated with
hyponatremia and hyperkalemia.
Alcohol withdrawal (choice B) should be considered in the management of
this patient; he may go into delirium tremens (DTs) in the next 24-48 hours.
Cardiogenic shock (choice C) is suggested by symptoms of heart failure,
e.g., bulging neck veins and crackles on lung examination. There may be
some old or acute changes on the ECG.
Septic shock (choice E) can be a possibility for investigation if the patient
does not respond to fluids/blood products.


Question 2 of 6
Which of the following would rule out shock?
/ A. AItered mental status
/ B. Cold extremities
/ C. Low systolic pressure
/ D. Metabolic acidosis
/ E. Urine output > 40 cc/hr

Explanation - Q: 5.2 Close

The correct answer is E. Patients in shock tend to be oliguric (urine output
< 20 cc/hr).
Patients in shock can have altered mental status (choice A), cold and
clammy extremities from peripheral vasoconstriction (choice B), low systolic
pressure (choice C), and metabolic acidosis (choice D) from the buildup of
the serum lactate from anaerobic metabolism.



Question 3 of 6
Which of the following is the most appropriate initial therapy?
/ A. Dopamine
/ B. Hydrocortisone
/ C. IV FIuids
/ D. Nitroglycerin
/ E. Norepinephrine

Explanation - Q: 5.3 Close

The correct answer is C. IV fluids are the first and foremost important
intervention for hypovolemia; whole blood products would be the next step
for hypovolemic shock from hemorrhage.
Dopamine (choice A) is used in patients with cardiogenic shock or pump
failure.
Hydrocortisone (choice B) is necessary, in addition to fluids, for patients with
suspected adrenal insufficiency.
Nitroglycerin (choice D) is used to relieve chest pain, and may be used in
hypertensive crises and urgencies.
Norepinephrine (choice E) has alpha adrenergic/vasoconstricting properties
used in septic shock in which the SVR (systemic vascular resistance) is low.



Question 4 of 6
Which of the following is the patient's mean arterial pressure?
/ A. 35
/ B. 40
/ C. 48
/ D. 60
/ E. 75

Explanation - Q: 5.4 Close

The correct answer is C. The mean arterial pressure (MAP) estimates the
perfusion of the body and brain; a pressure > 60 (choice D) ensures
sufficient perfusion. It is measured by 2/3 diastolic + 1/3 systolic pressure =
2/3 (35) + 1/3 (75) = 70/3 + 25 = 48.3 The normal ranges are 75 to 110.
Choice A is the diastolic blood pressure.
Choice B is the pulse pressure = systolic bp-diastolic bp.
Choice E is the systolic blood pressure.



Question 5 of 6



Which of the following would most likely represent this patient's pulmonary artery
catheterization (Swan-Ganz) values?
/ A. A
/ B. B
/ C. C
/ D. D
/ E. E


Explanation - Q: 5.5 Close

The correct answer is B. Hypovolemic shock is characterized by a low
wedge pressure (PCWP) because of low LV preload/filling leading to low
stroke volume, leading to low cardiac output (CO), leading to high systemic
vascular resistance (SVR).
Patient A has normal pulmonary artery catheterization values: normal PCWP
(12-15 mm Hg), CO (3.5-5.5 L/min), and SVR (800-1200 dyne/sec/cm
2
).
Patient C is in cardiogenic shock; the hallmark is low CO, leading to high
PCWP (pump backs up) and high SVR.
Patient D has septic shock with the distinguishing low SVR, leading to low
PCWP, leading to high CO for compensation.
Patient E has obstructive shock (from massive pulmonary embolus/tension
pneumothorax) with low filling pressure, leading to low wedge pressure,
leading to low CO, leading to high SVR as compensation.


Question 6 of 6



Which of the following is the best treatment for Patient C in the chart above?
/ A. Dopamine
/ B. Hydrocortisone
/ C. IV FIuids
/ D. Nitroglycerin
/ E. Norepinephrine

Explanation - Q: 5.6 Close

The correct answer is A. Patient C has cardiogenic shock, and therefore
dopamine would be appropriate to increase pump activity.
Hydrocortisone (choice B) is used in patients with adrenal insufficiency.
IV fluids (choice C) would cause even further cardiac pump failure.
Nitroglycerin (choice D) is used to relieve chest pain, and may be used in
hypertensive crises and urgencies.
Norepinephrine (choice E) has alpha adrenergic/vasoconstricting properties,
and is used in septic shock in which the SVR is low.

An 8-month-old girl is seen in the emergency department because her parents
are concerned about her very listless behavior and the fact that
her skin has a yellow discoloration. The child is the daughter of Vietnamese
immigrants to the United States, and has received no medical
care since her birth. On physical examination, the child is noted to be jaundiced.
The head shows prominence of the mandible, maxillary
overbite eminences, and frontal bossing. Hepatosplenomegaly is present. BIood
studies demonstrate a hemoglobin of 6.8 g/dL. A peripheral
blood smear shows predominantly a microcytic hypochromic anemia, although a
wide variety of red cell changes are noted by the examining
technician, including anisocytosis, poikilocytosis, target cells, ovalocytes,
basophilic stippling, polychromasia, macrocytes, and nucleated red
cells.
Question 1 of 5
An x-ray film of her head shows thinned cortices with widened marrow spaces of
the bones of the skulI. This would most likely be due to which
of the following processes?
/ A. Bony tumor
/ B. Cartilaginous tumor
/ C. Genetic abnormality of bone development
/ D. Hyperparathyroidism
/ E. Marrow expansion

Explanation - Q: 1.1 Close

The correct answer is E. When you see thinned cortices with widened
marrow spaces, you should think of processes that can cause marrow
expansion, such as leukemias and hemolytic anemias.
Bony and cartilaginous tumors (choices A and B) would be more likely to
produce masses visible on x-ray.
Genetic abnormalities of bone development (choice C), such as
osteogenesis imperfecta, may produce bones with abnormal patterns of
calcification and evidence of multiple fractures, but do not usually produce
thinned cortices with widened medullary spaces.
Hyperparathyroidism (choice D) can cause lytic bone lesions, but does not
usually cause diffuse thinning of bone.


Question 2 of 5

This patient's jaundice is most likely due to which of the following?
/ A. Gallstones
/ B. Hepatitis A
/ C. Hepatitis B
/ D. Hepatitis C
/ E. Hemolytic anemia

Explanation - Q: 1.2 Close

The correct answer is E. While many associate jaundice with liver disease,
you need to remember that the hemolytic anemias also cause jaundice (and
can cause hepatosplenomegaly secondary to extramedullary
hematopoiesis). Pigmented gallstones (choice A) related to excessive
excretion of the heme degradative product bilirubin can be seen in patients
with chronic hemolytic anemia, but would be very unusual in a young child.
None of the information in this patient's history and clinical examination
except the jaundice and hepatosplenomegaly suggests that she has hepatitis
(choices B, C, and D); more specifically, the marked anemia and marrow
expansion of the cranium would not be seen in hepatitis.


Question 3 of 5
Which of the following conditions is most likely the cause of the patient's
peripheral blood smear findings?
/ A. B12 deficiency
/ B. Folate deficiency
/ C. Iron deficiency
/ D. Sickle cell anemia
/ E. Thalassemia

Explanation - Q: 1.3 Close

The correct answer is E. The severe anemia and very complex peripheral
smear pattern is most consistent with severe thalassemia. Milder cases of
thalassemia may resemble either iron deficiency (choice C) with microcytic
cells, or folate or vitamin B12 deficiencies (choices A and B) with macrocytic
cells. In these milder cases, the wide variety of red cell shapes and profound
anemia seen in severe cases are not present, and the diagnosis of
thalassemia is usually made after a failure of iron, folate, or B12 therapy to
correct the anemia.
Sickle cell anemia (choice D) would show sickled cells on peripheral smear.


Question 4 of 5
Hemoglobin electrophoresis studies are ordered, and HbF is found to be the
predominant form, with only very small amounts of other
hemoglobins present. This is most consistent with which of the following
underlying genetic defects?
/ A. 1 defective alpha globin chain gene
/ B. 1 defective beta globin chain gene
/ C. 2 defective alpha globin chain genes
/ D. 2 defective beta globin chain genes
/ E. 3 defective alpha globin chain genes

Explanation - Q: 1.4 Close

The correct answer is D. The thalassemias are a group of chronic, inherited
blood diseases that are characterized by defective hemoglobin synthesis and
resultant ineffective erythropoiesis. Severe cases (thalassemia major) tend to
present as illustrated in the case history by 12 months of age; intermediate
cases (thalassemia intermedia) present at 2-4 ages; and milder cases
(thalassemia minor and thalassemia minima) may be completely or nearly
asymptomatic throughout life. The hemoglobin molecule contains two alpha
chains (coded by 4 copies of the gene on 2 chromosomes) and two beta
chains (coded by 2 copies of the gene on 2 chromosomes). Thalassemia can
be produced by either defective alpha chain or defective beta chain
production. The fact that the child can make fetal hemoglobin, HbF, means
that he is able to make alpha chains (thereby excluding choices A, C, and
E), which occur in both adult and fetal hemoglobin. The child has severe,
rather than mild, disease (thalassemia major) and is not making normal
hemoglobin. This means that she most likely has 2 defective beta globin
chain genes rather than 1 (choice B). If you encounter a question similar to
this on the USMLE, you should analyze it as illustrated above; however, for
your own information, a patient with 2 defective beta globin genes and 1
defective alpha globin gene might have a similar electrophoresis pattern. The
thalassemias are found predominantly in the equatorial belt corresponding to
the distribution of malaria (thalassemia trait has been postulated to be
protective against falciparum malaria). Immigration from the Indian
subcontinent and Southeast Asia (including Vietnam) is increasing the
number of cases of thalassemia found in the United States; the earlier US
pool of thalassemia had been dominated by Italian immigrants and those
from other countries near the Mediterranean Sea.


Question 5 of 5
This child will require life-Iong transfusions to correct the otherwise fatal anemia,
which may cause death either due to anemia itself or due to
septicemia. The child is consequently at severe risk of developing iron overload,
which also has potentially severe to fatal complications. To
reduce the rate at which iron overload occurs, the child should be treated, after
the age of 3, with nightly subcutaneous infusions of which of the
following?
/ A. Desferrioxamine
/ B. Dimercaprol
/ C. Edetate calcium disodium
/ D. Penicillamine
/ E. Succimer

Explanation - Q: 1.5 Close

The correct answer is A. Desferrioxamine is given intravenously as a
chelating agent to treat severe iron load, and subcutaneously, with slow
infusion (over 8 hours nightly via a small pump) in milder overload cases.
The use of this drug markedly extends the life span of children with
thalassemia major (who otherwise typically die before age 10), but is very
problematic for these young children and their families because the nightly
prolonged injections are painful. There is a great deal of interest in
developing a safe oral iron chelating agent, but none is yet available in this
country and agents in use in other countries have had severe toxicity
problems. We have already extended these children's life-span up to 30 or
40 years, and no one yet knows how far we will be able to go as better
agents and/or delivery systems become available. The other choices listed
are also chelating agents, but do not work with iron.
Dimercaprol (choice B) is used to treat a variety of poisonings with
antimony, arsenic, chromium derivatives, bismuth, copper, gold, mercury,
nickel, tungsten, and zinc.
Edetate calcium disodium (choice C) is used to treat a variety of metal
poisonings with cadmium, chromium, copper, lead, manganese, nickel,
radium, selenium, tungsten, uranium, vanadium, and zinc.
Penicillamine (choice D) is used to treat a variety of poisonings with
chromium derivatives, cadmium, cobalt, copper, lead, mercury, nickel, and
zinc.
Succimer (choice E) is used to treat poisonings with lead, arsenic, and
mercury.



A 35-year-old woman is evaluated for jaundice in an emergency department. For
several days, the patient has had mild flu-Iike symptoms of
anorexia, nausea and vomiting, fatigue, Iow-grade fever, and malaise. This
morning, she noted that her urine was brown in color, and she has
also today developed moderate, steady, pain of the right upper quadrant of her
abdomen. She has not had any similar episodes in the past.
On physical examination, the patient is noted to be jaundiced and to have an
enlarged, tender liver. BIood chemistry studies are notable for
alanine aminotransferase (ALT) of 15,000 mIU/L, aspartate aminotransferase
(AST) of 11,000 mIU/L, and alkaline phosphatase of 100 U/L.
Question 1 of 6
This patient's dark urine is due to the presence of which of the following?
/ A. Bacteria
/ B. Bilirubin
/ C. Hemosiderin
/ D. Ketone bodies
/ E. Melanin

Explanation - Q: 2.1 Close

The correct answer is B. Darkly discolored urine due to the presence of
bilirubin (a degradative product of heme that is normally excreted via bile into
feces) may precede obvious jaundice in patients with acute hepatitis.
Bacteria (choice A) can cause urine to become cloudy or whitish.
Hemosiderin (choice C) can also cause brown discoloration in urine, but
would be seen in a setting in which urinary tract hemorrhage was present.
Ketone bodies (choice D), a feature of diabetic ketoacidosis, do not cause
urine discoloration.
Melanin (choice E) is a black pigment that is rarely spilled into the urine in
patients with metastatic melanoma.


Question 2 of 6
Which of the following is the most likely diagnosis?
/ A. Acute hepatitis
/ B. Chronic hepatitis
/ C. Gallstone disease
/ D. Hepatic cirrhosis
/ E. Wilson disease

Explanation - Q: 2.2 Close

The correct answer is A. This person's markedly elevated AST and ALT
with modest elevation of alkaline phosphatase strongly suggests that she
has acute hepatitis. The clinical presentation with flu-like symptoms that
progress to jaundice is also typical.
Chronic hepatitis (choice B) and cirrhosis (choice D) would present more
insidiously and would not have the extremely high elevations of AST and
ALT.
Gallstone disease (choice C) can cause acute abdominal pain, and
occasionally jaundice (if a small stone occludes the common bile duct), but
would not usually cause the very high elevations of AST and ALT seen in this
patient.
Wilson disease (choice E) in adults usually causes a chronic hepatitis that
may progress to cirrhosis.



Question 3 of 6
The patient has recently returned from a 6-month tour working with infants and
young children in a daycare facility in southern Mexico. During
the past two years, she has not had any sexual encounters, has not used drugs,
and has not received blood products. Which of the following
best describes the most likely pathogen?
/ A. Enveloped, defective circular RNA virus
/ B. Enveloped, DNA virus in the Hepadnavirus family
/ C. Enveloped, RNA virus in the FIavivirus family
/ D. Naked capsid, RNA virus in the Calicivirus family
/ E. Naked capsid, RNA virus in the Picornavirus family


Explanation - Q: 2.3 Close

The correct answer is E. Viral infection is an important cause of acute
hepatitis, and in this case, the individual's history of exposure to infants in an
area endemic for hepatitis A is very suggestive. Unlike many of the hepatitis
viruses, hepatitis A is spread through a fecal-oral route. The patient's history
also excludes other probable routes of spread, such as through sexual
transmission, use of intravenous drugs, or use of blood products. The
diagnosis can be confirmed by serum measurement of anti-hepatitis A
immunoglobulin M. The hepatitis A virus is an unencapsulated, single-
stranded, positive sense, linear RNA enterovirus in the Picornavirus family.
The virus replicates (apparently exclusively) in hepatocytes and is excreted
via bile into the stool.
Enveloped, defective circular RNA virus (choice A) describes hepatitis D.
Hepatitis D, also called delta hepatitis, is a defective hepatitis virus that must
coinfect or superinfect with hepatitis B to cause disease. It can cause severe
acute hepatitis, but is unlikely in this patient because the usual routes of
spread are parenteral or sexual.
Enveloped, DNA virus in the Hepadnavirus family (choice B) describes
hepatitis B, while enveloped RNA virus in the Flavivirus family (choice C)
describes hepatitis C. Hepatitis B and hepatitis C are usually spread by
parenteral or sexual routes, and usually do not cause severe acute hepatitis.
Naked capsid, RNA virus in the Picornavirus family (choice D) describes
hepatitis E. Hepatitis E is spread by the fecal-oral route, but usually causes
mild disease unless the patient is pregnant.


Question 4 of 6
What percentage of adults who develop this infection have symptomatic
disease?
/ A. 10%
/ B. 25%
/ C. 50%
/ D. 75%
/ E. 95%

Explanation - Q: 2.4 Close

The correct answer is D. In adults who acquire hepatitis A infection,
symptomatic cases are common (75%) and most of these symptomatic
adults develop jaundice.

Question 5 of 6
What percentage of children less than 2 years of age who develop this infection
have symptomatic disease?
/ A. 10%
/ B. 25%
/ C. 50%
/ D. 75%
/ E. 95%

Explanation - Q: 2.5 Close

The correct answer is A. In marked contrast to the situation with adults,
90% of children under the age of 2 who acquire hepatitis A infection
(common in endemic areas) are asymptomatic.








Question 6 of 6
Overdose with which of the following could also be responsible for this patient's
disease?
/ A. Acetaminophen
/ B. Aspirin
/ C. Caffeine
/ D. Cocaine
/ E. Codeine

Explanation - Q: 2.6 Close

The correct answer is A. Severe acute hepatitis can be caused by
processes other than viral infection, including acute drug-induced liver injury
(notably acetaminophen), drug-induced hypersensitivity reactions (notably
sulfasalazine hypersensitivity), and other viruses (CMV, EBV, and HIV).
Acetaminophen overdose can cause dose-dependent, potentially fatal
hepatic necrosis that can mimic the fulminant hepatitis of severe hepatitis A
infection. Other serious side effects of acetaminophen overdose include
renal tubular necrosis, hypoglycemic coma, and thrombocytopenia.
Aspirin (choice B) overdose can cause early CNS overstimulation followed
by coma, respiratory failure, and severe electrolyte disturbances.
Caffeine (choice C) overdose can cause CNS stimulation, difficulty
breathing, arrhythmias, and GI symptoms.
Cocaine (choice D) overdose can cause tremors, convulsions, delirium, and
arrhythmias.
Codeine (choice E) overdose can cause muscle spasticity, respiratory
failure, coma, and shock.


A 37-year-old man is evaluated by a family practitioner because of jaundice. On
physical examination, the man is noted to have an enlarged
Iiver with an irregular edge on palpation. UItrasound examination shows diffuse
echogenicity and nodularity of the liver without specific discrete
masses.
Question 1 of 7
Which of the following is the most likely diagnosis?
/ A. Acute hepatitis
/ B. Chronic persistent hepatitis
/ C. Cirrhosis
/ D. Hepatocellular carcinoma
/ E. Steatosis

Explanation - Q: 3.1 Close

The correct answer is C. The diffuse echogenicity and nodularity without
specific discrete masses are typical of a cirrhotic liver, as is the irregular liver
edge. Cirrhotic livers may be enlarged, or smaller than normal, depending
upon the degree to which regeneration is occurring.
Acute hepatitis (choice A) can cause an enlarged, tender liver, but does not
cause nodularity.
Chronic persistent hepatitis (choice B) is an inflammation of the portal areas
seen in some chronic cases of viral hepatitis that, by definition, does not
progress to cirrhosis, and consequently does not show nodularity.
Hepatocellular carcinoma (choice D) can complicate cirrhosis, but is unlikely
in this particular case, because the liver does not contain specific discrete
masses.
Steatosis (choice E), or fatty liver, does not cause nodularity of the liver.


Question 2 of 7

Serum serology studies demonstrate the following:
IgG anti-HAV: +
IgM anti-HAV:
IgM HBcAb:
IgG HBcAb: +
HBsAg: +
HBeAG: +
HBV DNA: +
anti-HCV:

This suggests that this patient is presently infected with which of the following?

/ A. Both hepatitis A and hepatitis B viruses
/ B. Both hepatitis A and hepatitis C viruses
/ C. Hepatitis A virus only
/ D. Hepatitis B virus only
/ E. Hepatitis C virus only

Explanation - Q: 3.2 Close

The correct answer is D. This patient is presently infected with hepatitis B
virus, and was formerly exposed to hepatitis A. He has not been exposed to
hepatitis C. The hepatitis antibodies and antigens tend to be somewhat
confusing, but can be sorted through with a little care. "Ag" on the end of the
abbreviation indicates an antigen, while "Ab" indicates an antibody formed
against the antigen. Anti-HAV, or antibodies to Hepatitis A virus, can be
either in IgM form (indicating recent infection) or IgG form (indicating past
infection). Hepatitis A virus does not cause chronic infection or cirrhosis, so
the patient is not currently infected with this virus. Hepatitis B serology is
complex. There are 3 significant antigens: "c"- the core antigen, "e" - the e
antigen also found in the core of the virus, and "s"- the surface antigen.
Chronic hepatitis, including cirrhosis, due to hepatitis B, is usually
characterized by persistent circulating HBsAg, HBeAg, and HBV DNA. There
are also usually HBcAb (antibodies to core antigen), often in the IgG form.
Hepatitis C exposure is indicated by the presence of HCV antibodies, which
this patient does not have.


Question 3 of 7
What percentage of adults who acquire this patient's current viral infection(s)
develop a chronic infection?
/ A. Less than 5%
/ B. 10-20%
/ C. 30-50%
/ D. 60-70%
/ E. 90% or more

Explanation - Q: 3.3 Close

The correct answer is A. Hepatitis B infection can take a variety of forms.
The infection usually begins as a clinical or subclinical acute infection that
may resolve completely, persist with little accumulating damage, or progress
to hepatic cirrhosis. In adults, less than 5% of individuals who become
infected develop chronic disease, with risk of eventual development of
cirrhosis. The virus is spread through contaminated blood and body fluids.



Question 4 of 7
Approximately what percentage of children who acquire hepatitis B in the
perinatal period develop a chronic infection?
/ A. 5%
/ B. 20%
/ C. 50%
/ D. 70%
/ E. 90%

Explanation - Q: 3.4 Close

The correct answer is E. While the rate at which adults with new hepatitis B
infection develop a chronic infection is less than 5%, that of neonates is 90%,
and that of children 1-5 years of age is 20-50%. The neonates usually
acquire the infection through vertical transmission from the mother.


Question 5 of 7
Which of the following is a defective virus that requires this patient's virus(es) for
propagation?
/ A. Cytomegalovirus
/ B. Hepatitis D virus
/ C. Hepatitis E virus
/ D. Herpes simplex l
/ E. Herpes simplex ll

Explanation - Q: 3.5 Close

The correct answer is B. The hepatitis D virus is a defective hepatitis virus
that can coinfect or superinfect hepatitis B liver disease, causing more
severe disease with increased risk of progression to chronic disease and
cirrhosis. The other viruses listed can infect the liver, but do not require
coinfection with hepatitis B to cause disease.


Question 6 of 7
Which part of this patient's virus(es) is specifically required by the defective virus
that uses it for propagation?
/ A. HBcAb
/ B. HBcAg
/ C. HBeAg
/ D. HBsAb
/ E. HBsAg

Explanation - Q: 3.6 Close

The correct answer is E. Hepatitis D can replicate independently within the
liver cell, but requires the presence of hepatitis B surface antigen (HBsAg)
for release of the virus particles in an infective form.
HBcAb (choices A) and HBsAb (choice D) are antibodies to antigens, rather
than antigens themselves, and do not participate in the hepatitis D life cycle.
The hepatitis B core and e antigens (choices B and C) are not apparently
required.


Question 7 of 7
Which of the following agents blocks viral reverse transcriptase and can be used
to treat this patient's infection?
/ A. Amantadine
/ B. Lamivudine
/ C. Oseltamivir
/ D. Prednisone
/ E. Zanamivir

Explanation - Q: 3.7 Close

The correct answer is B. Formerly, hepatitis B infection was a very
frustrating disease because almost no effective therapy existed. Alpha
interferon was one of the first successful therapies for chronic hepatitis B
infection, and has been extensively used since the mid-1980's, although
unfortunately, the therapy response rate is only 30-40%. The thymidine
analog lamivudine is used in treatment of chronic hepatitis B infection, and
acts by blocking viral replication by competitive inhibition of viral reverse
transcriptase. This and other new similar antiviral agents (e.g., famciclovir,
lobucavir, adefovir dipivoxil) offer the promise of much better control of
hepatitis B infection, since these agents directly block the propagation of
hepatitis B.
The anti-viral agent amantadine (choice A) is used in treatment of hepatitis
C and acts by inhibiting uncoating of the virus.
The corticosteroid prednisone (choice D) is sometimes used in treatment of
cholestatic hepatitis A infection.
Zanamivir (choice E) and oseltamivir (choice C) are active against influenza
A and B viruses, and act by preventing clumping of virions, thereby reducing
the likelihood that the virus will penetrate infected cells.


Routine physical examination of a 19-year-old man demonstrates mild jaundice
and scleral icterus. On questioning, the patient reports he feels
welI. He says he occasionally gets a slightly yellowish tinge to his skin and eyes,
but the yellow seems to go away spontaneously after several
days. Screening chemistry studies are remarkable only for serum bilirubin of 2
mg/dL. On further evaluation, this bilirubin is found to be
predominately unconjugated. Liver enzymes are not elevated, and a complete
blood count (CBC) is within normal limits. On questioning, the
man says he does not use alcohoI, has not had unprotected sex, and does not
feel ilI. Liver biopsy is unremarkable.
Question 1 of 7
Bilirubin is a degradative product of which of the following?
/ A. Cholesterol
/ B. GIycosaminoglycans
/ C. Hemoglobin
/ D. Melanin
/ E. Steroid hormones

Explanation - Q: 4.1 Close

The correct answer is C. Bilirubin is derived from the degradation of the
heme moiety of hemoglobin. The heme ring is opened to produce biliverdin,
which is then converted to bilirubin. The other answers are distracters.


Question 2 of 7
Which of the following is the most likely diagnosis?
/ A. Acute viral hepatitis
/ B. Chronic viral hepatitis
/ C. Gilbert syndrome
/ D. Hemochromatosis
/ E. Wilson disease

Explanation - Q: 4.2 Close

The correct answer is C. Gilbert syndrome is the most common inherited
(although the pattern of inheritance may be hard to define) cause of
unconjugated hyperbilirubinemia. Episodes of clinical jaundice in Gilbert
syndrome tend to develop when there is a physiologic stressor such as
dehydration, fasting, menstrual periods, stress, illness, or vigorous exercise.
Some patients have vague, mild symptoms of abdominal cramps, fatigue,
and malaise. Between episodes, the patients may have variable (but low)
degrees of hyperbilirubinemia, sometimes with occasional normal values for
serum bilirubin. The diagnosis of Gilbert disease is established by excluding
other, more serious, causes of hyperbilirubinemia. All of the other liver
diseases listed in the choices would have abnormal liver biopsies and
elevated serum liver enzymes.


*** Commercial version is infinite. Order at http://www.structurise.com/kleptomania ***
Question 3 of 7
What is the incidence of this patient's disease in the United States?
/ A. Less than 0.001% of the population
/ B. 0.02-0.05% of the population
/ C. 0.4-0.5% of the population
/ D. 3-7% of the population
/ E. More than 15% of the population
Explanation - Q: 4.3 Close

The correct answer is D. Gilbert syndrome is very common, with an
incidence of 3-7% of people in the United States.


Question 4 of 7
The function of the defective enzyme in this patient's disease is which of the
following?
/ A. Adding glucuronyl residues to bilirubin
/ B. Converting bilirubin to urobilinogen
/ C. Converting biliverdin to bilirubin
/ D. Oxidation of a methane bridge in the porphyrin ring
/ E. Removing the globin chains from verdoglobin

Explanation - Q: 4.4 Close

The correct answer is A. The biochemical defect in Gilbert syndrome is a
decreased activity of the bilirubin conjugating system that converts bilirubin
to a form more soluble in bile. The affected enzyme is called bilirubin-uridine
diphosphate glucuronyl transferase (bilirubin-UGT). This enzyme conjugates
bilirubin, producing the more soluble forms: bilirubin monoglucuronides and
bilirubin diglucuronides. This enzyme is one of a family that also conjugates
(and thus render more easily excreted) a variety of carcinogens, drugs,
hormones, and neurotransmitters. The other choices are other steps in the
degradation of hemoglobin.
Hemoglobin degradation begins with oxidation of a methane bridge in the
porphyrin ring (choice D) to form verdoglobin.
Next, the globin chains are removed from verdoglobin (choice E), producing
biliverdin, which is subsequently converted to bilirubin (choice C).
The bilirubin is then conjugated and excreted into the bile, and bacteria act
on it to produce urobilinogen (choice B).


Question 5 of 7
The long-term prognosis of people with this patient's disease is which of the
following?
/ A. Life span shortened on average by 5 years
/ B. Life span shortened on average by 10 years
/ C. Life span shortened on average by 20 years
/ D. Life span shortened on average by 30 years
/ E. Normal life span

Explanation - Q: 4.5 Close

The correct answer is E. Patients with Gilbert syndrome have a normal life-
span because there is no associated morbidity or mortality. Because of the
lack of associated morbidity or mortality, it is now recommended that no
medications be used in treatment of this disease.


Question 6 of 7
If the patient had instead presented with a hereditary conjugated
hyperbilirubinemia and was found to have a black liver on laparotomy, which
of the following would be the most likely diagnosis?
/ A. Crigler-Najjar syndrome type l
/ B. Crigler-Najjar syndrome type ll
/ C. Dubin-Johnson syndrome
/ D. Gilbert syndrome
/ E. Rotor syndrome

Explanation - Q: 4.6 Close

The correct answer is C. Dubin-Johnson syndrome is a rare benign form of
hereditary hyperbilirubinemia with asymptomatic jaundice that has the
unusual feature of causing the liver to be darkly pigmented as the result of
deposition of an intracellular melanin-like substance. This has occasionally
been disconcerting in the surgical suite, when a patient undergoing operation
for some other purpose is incidentally found to have a "black liver". The
hyperbilirubinemia of Dubin-Johnson syndrome is conjugated, rather than
unconjugated, like Gilbert and Crigler-Najjar syndromes (choices A, B, and
D).
Rotor syndrome (choice E) is another, rare, form of benign hereditary
conjugated hyperbilirubinemia that clinically resembles Dubin-Johnson
syndrome, but without the black liver.


Question 7 of 7
If the patient had instead died of a hereditary cause of hyperbilirubinemia in
infancy, which of the following would be the most likely diagnosis?
/ A. Crigler-Najjar syndrome type l
/ B. Crigler-Najjar syndrome type ll
/ C. Dubin-Johnson syndrome
/ D. Gilbert syndrome
/ E. Rotor syndrome



Explanation - Q: 4.7 Close

The correct answer is A. All of the diseases listed are hereditary
hyperbilirubinemias, but only Crigler-Najjar syndrome type I causes death in
infancy. The affected babies have severe hyperbilirubinemia, and typically
die before 1 year of age of kernicterus (bilirubin damage to developing brain).
Fortunately, this is a rare disease.
Crigler-Najjar syndrome type II (choice B) is also rare, and features a
hyperbilirubinemia that is less severe than in Type I, but is still much more
marked than in Gilbert syndrome. These patients usually live into adulthood
without neurologic damage, and may at least partially respond to therapy
with barbiturates.
Dubin-Johnson syndrome, Gilbert syndrome, and Rotor syndrome (choices
C, D, and E) are all benign forms of hyperbilirubinemia.


A 27-year-old man presents to a dermatologist because his skin has become
chronically itchy. He has also been experiencing chronic,
progressive fatigue. On physical examination, no specific skin lesions are seen,
but the patient is noted to be mildly jaundiced. He is referred,
for further evaluation, to an internist specializing in liver disease. Serum
chemistry studies demonstrate elevated serum alkaline phosphatase
and bilirubin with minimally increased transaminases. The mitochondrial antibody
test is negative. Endoscopic retrograde cholangiography
demonstrates multiple short strictures and saccular dilatations involving the
intrahepatic and extrahepatic bile ducts.

Question 1 of 6

Which of the following is the most likely diagnosis?
/ A. Ascending cholangitis
/ B. Bile duct tumor
/ C. Primary biliary sclerosis
/ D. Primary sclerosing cholangitis
/ E. Viral hepatitis
/ F. Wilson disease

Explanation - Q: 5.1 Close

The correct answer is D. This presentation is typical for primary sclerosing
cholangitis, which is an inflammatory disease that affects the bile duct
system, most often in young men. Most of the findings are fairly nonspecific,
but the characteristic cholangiographic picture illustrated is the clue that
gives the diagnosis, and will probably be mentioned in any case history
about the disease that you encounter.
Ascending cholangitis (choice A) is a bacterial infection of the bile duct
system, and does not produce the characteristic sacculations of primary
sclerosing cholangitis.
A bile duct tumor (choice B) can produce a diffuse dilation of the biliary tree
above it, secondary to back-pressure effects, but would not produce the
alternating strictures and sacculations seen in this case.
Primary biliary sclerosis (choice C) affects the intrahepatic, but not the
extrahepatic bile ducts.
Viral hepatitis (choice E) does not usually affect the larger intrahepatic bile
ducts and the extrahepatic duct system.
Wilson disease (choice F) typically produces fatty change, hepatitis, or
cirrhosis, rather than cholangitis. Also, it is often accompanied by neurologic
dysfunction




Question 2 of 6
A positive mitochondrial antibody test would have suggested which of the
following diagnoses?
/ A. Hepatitis A infection
/ B. Hepatitis B infection
/ C. Hepatitis C infection
/ D. Primary biliary sclerosis
/ E. Wilson disease

Explanation - Q: 5.2 Close

The correct answer is D. Antibodies directed against mitochondria are fairly
specific for primary biliary sclerosis (an important differential diagnosis for
this patient's disease), and are not seen in the other liver diseases listed in
the choices.


Question 3 of 6
The presence of which of the following antibodies would have more specifically
suggested the disease this patient has?
/ A. Anti-centromere antibodies
/ B. Anti-double-stranded DNA antibodies
/ C. Anti-Golgi antibodies
/ D. Anti-ribonucleoprotein antibodies
/ E. Perinuclear antineutrophil cytoplasmic antibodies

Explanation - Q: 5.3 Close

The correct answer is E. The perinuclear antineutrophil cytoplasmic
antibodies, also called p-ANCA, are most characteristic of primary sclerosing
cholangitis and ulcerative colitis.
Anti-centromere antibodies (choice A) suggest the CREST variant of
scleroderma.
Anti-double-stranded DNA antibodies (choice B) suggest systemic lupus
erythematosus.
Anti-Golgi antibodies (choice C) are seen most often in systemic lupus
erythematosus and Sjgren syndrome.
Anti-ribonucleoprotein antibodies (choice D) are seen most often in systemic
lupus erythematosus and mixed connective tissue disease.



Question 3 of 6
The presence of which of the following antibodies would have more specifically
suggested the disease this patient has?
/ A. Anti-centromere antibodies
/ B. Anti-double-stranded DNA antibodies
/ C. Anti-Golgi antibodies
/ D. Anti-ribonucleoprotein antibodies
/ E. Perinuclear antineutrophil cytoplasmic antibodies

Explanation - Q: 5.3 Close

The correct answer is E. The perinuclear antineutrophil cytoplasmic
antibodies, also called p-ANCA, are most characteristic of primary sclerosing
cholangitis and ulcerative colitis.
Anti-centromere antibodies (choice A) suggest the CREST variant of
scleroderma.
Anti-double-stranded DNA antibodies (choice B) suggest systemic lupus
erythematosus.
Anti-Golgi antibodies (choice C) are seen most often in systemic lupus
erythematosus and Sjgren syndrome.
Anti-ribonucleoprotein antibodies (choice D) are seen most often in systemic
lupus erythematosus and mixed connective tissue disease.


Question 4 of 6
This patient's liver disease is most strongly associated with which of the
following?
/ A. Amoebic colitis
/ B. Celiac disease
/ C. Tropical sprue
/ D. UIcerative colitis
/ E. Whipple disease

Explanation - Q: 5.4 Close

The correct answer is D. There is a specific association between ulcerative
colitis and primary sclerosing cholangitis. Half to three-quarters of patients
with primary sclerosing cholangitis also have inflammatory bowel disease
(ulcerative colitis more commonly than Crohn disease), and approximately
5% of patients with inflammatory bowel disease also have primary sclerosing
cholangitis. The pathophysiologic basis for this association remains unclear.
The other intestinal diseases listed are not specifically associated with
primary sclerosing cholangitis.


Question 5 of 6
Which of the following HLA types is seen with increased frequency in patients
with this patient's disease?
/ A. HLA-A3
/ B. HLA-B8
/ C. HLA-B27
/ D. HLA-B35
/ E. HLA-Cw6

Explanation - Q: 5.5 Close

The correct answer is B. Primary sclerosing cholangitis is associated with
increased frequencies of class I antigen HLA-B8 and of class II antigen HLA-
DR3. HLA-B8 is also associated with other autoimmune disorders, and may
partially account for the fact that approximately one-quarter of patients with
primary sclerosing cholangitis also have other autoimmune diseases outside
the liver and colon. Therapy for primary sclerosing cholangitis remains
problematic, with many patients progressing to cirrhosis within 10-15 years.
Existing therapies often attempt to modulate the immune system with
steroids or antimetabolites.
HLA-A3 (choice A) is seen with increased frequency in patients with
idiopathic hemochromatosis.
HLA-B27 (choice C) is seen with increased frequency in patients with acute
anterior uveitis, ankylosing spondylitis, and Reiter syndrome.
HLA-B35 (choice D) is seen with increased frequency in patients with
duodenal ulcer and subacute thyroiditis
HLA-Cw6 (choice E) is seen with increased frequency in patients with
psoriasis vulgaris.

Question 6 of 6
Which of the following histologic findings on liver biopsy is considered to be most
specific for this patient's disease?
/ A. Concentric obliterative fibrosis of interlobular bile ducts
/ B. Ductular proliferation
/ C. Individual hepatocyte necrosis
/ D. Periductal concentration of mononuclear cells
/ E. Regenerating nodules of hepatic parenchyma

Explanation - Q: 5.6 Close

The correct answer is A. All of the findings illustrated can be seen in
primary sclerosing cholangitis at different stages, but most are non-specific
markers of liver injury. The only finding considered to be specific for primary
sclerosing cholangitis is concentric obliterative fibrosis of interlobular ducts.


A 5-year-old boy is seen by his primary care physician because of progressive
weakness of his lower extremities, clumsy walking, and
decreased motor skills. He also has a 10-year-old brother with similar, but more
severe symptoms. On physical examination, he shows an
awkward gait and has a positive Gowers maneuver (rises from a sitting position
by walking his hands up his legs.) There is
pseudohypertrophy of his calves.
Question 1 of 6
Which of the following is the most likely diagnosis?
/ A. Central core disease
/ B. Duchenne muscular dystrophy
/ C. Myasthenia gravis
/ D. Myotonic dystrophy
/ E. Spinal muscular atrophy

Explanation - Q: 1.1 Close

The correct answer is B. Duchenne muscular dystrophy is a progressive X-
linked condition that occurs in boys. There is a progressive degeneration of
muscles: most commonly the pelvic and shoulder girdles. These children are
usually wheelchair-bound by 10 years of age and die by 20 years of age. The
weak calf muscles become atrophic and are replaced by fibrofatty tissue,
which results in pseudohypertrophy of the calves.
Central core disease (choice A) is a congenital disease characterized by
hypotonia, decreased deep tendon reflexes, and delayed motor skills.
Myasthenia gravis (choice C) is an acquired autoimmune disease that is
more common in woman than men (2:1). Onset is usually in young adults.
Patients complain of easy muscular fatigability.
Myotonic dystrophy (choice D) is the most common form of adult muscular
dystrophy. Patients have progressive muscle weakness and sustained
muscle contractions.
Spinal muscular atrophy (choice E) is a autosomal recessive denervating
condition. Patients exhibit extreme weakness at birth and rarely live beyond
1 year.

Question 2 of 6
What is the pattern of inheritance for this child's disorder?
/ A. Autosomal dominant
/ B. Autosomal recessive
/ C. Sporadic
/ D. X-Iinked recessive
/ E. X-Iinked dominant
Explanation - Q: 1.2 Close

The correct answer is D. Duchenne muscular dystrophy is inherited in a X-
linked recessive pattern. The history indicates that he has a brother with the
condition. Neither parent has the condition since patients only live until about
20 years old



Question 3 of 6
The prevalence of this condition is approximately 1:3000. Which of the following
most accurately describes prevalence?
/ A. The disease risk in the exposed group divided by the disease risk in the
unexposed group
/ B. The number of new cases divided by the total population at risk
/ C. The number of new cases in a population per unit time
/ D. The number of new cases subtracted from the total number of existing
cases per unit time
/ E. The total number of cases in a population at a given time


Explanation - Q: 1.3 Close

The correct answer is E. Prevalence is the total number of cases in a
population at a given time.
The disease risk in the exposed group divided by the disease risk in the
unexposed group (choice A) is the relative risk.
The number of new incidents divided by the total population at risk (choice
B) is the incidence rate.
The number of new cases in a population per unit time (choice C) is the
incidence.
The number of new cases subtracted from the total number of existing cases
per unit time (choice D) does not have a definition.




Question 4 of 6
Which of the following is the genetic abnormality in this disorder?
/ A. 11:22 chromosomal translocation
/ B. Defective CFTR gene
/ C. Defective dystrophin gene
/ D. Defective fibrillin gene
/ E. Deletion of the short arm of chromosome 5




Explanation - Q: 1.4 Close

The correct answer is C. Duchenne muscular dystrophy is associated with
a defect in the dystrophin gene. Normal dystrophin is undetectable in these
patients. The genetic underpinning of this deficiency includes deletions or
duplications in the dystrophin gene in about 65% of patients and point
mutations in about 25%.
An 11:22 chromosomal translocation (choice A) is seen in Ewing sarcoma.
The CFTR gene (choice B) is associated with cystic fibrosis.
Defects in the fibrillin gene (choice D) result in Marfan syndrome.
A deletion of the short arm of chromosome 5 (choice E) is seen in Cri-du-
chat syndrome



Question 5 of 6
Which of the following tests would be necessary to make the definitive
diagnosis?
/ A. CT scan of the head
/ B. MRI
/ C. Muscle biopsy
/ D. Muscle strength testing
/ E. X-ray films

Explanation - Q: 1.5 Close

The correct answer is C. Muscle biopsy is needed to make the definitive
diagnosis of Duchenne muscular dystrophy. Immunostaining for dystrophin
reveals the characteristic absence of the protein associated with this
disease.
CT scan (choice A), MRI (choice B), muscle strength testing (choice D),
and x-rays (choice E) will not give a specific diagnosis of Duchenne
muscular dystrophy.



Question 6 of 6
Many different types of muscle contractions exist. Contractions in which muscle
tension is generated, but the length of the muscle does not
change is which of the following?
/ A. Concentric
/ B. Eccentric
/ C. Isokinetic
/ D. Isometric
/ E. Isotonic

Explanation - Q: 1.6 Close

The correct answer is D. In isometric contractions, muscle tension is
generated but the length of the muscle does not change.
Concentric contractions (choice A) are a type of isotonic contraction in which
the muscle shortens during the contraction.
Eccentric contractions (choice B) are also a type of isotonic contraction in
which the muscle lengthens during the contraction.
In isokinetic contractions (choice C), muscle tension is generated as the
muscle contracts at a constant velocity over a full range of motion.
In isotonic contractions (choice E), muscle tension is constant over a full
range of motion.

A 6-year-old boy is taken to a physician because he has developed pain in his
right hip. On physical examination, the physician feels a large
mass near the iliac crest. PIain x-ray films demonstrate a large lytic lesion of the
ilium. MRI studies show that the tumor appears to arise in the
bone, but extends into the adjacent soft tissues. A Iarge incisional biopsy
demonstrates a tumor composed of sheets of smalI, round, blue
cells.


Question 1 of 7

Which of the following is the most likely diagnosis?
/ A. Chondrosarcoma
/ B. Ewing sarcoma
/ C. Giant cell tumor of bone
/ D. Malignant fibrous histiocytoma
/ E. Osteosarcoma

Explanation - Q: 2.1 Close

The correct answer is B. Ewing sarcoma is the second most common type
of bone tumor (after osteosarcoma) in children and adolescents. The tumor
is usually a lytic bone lesion, and often both invades the medullary cavity of
the bone and extends into extraosseous tissues. Grossly, the tumor is often
tan-white in color and shows focal areas of hemorrhage and necrosis.
Microscopically, it is composed of sheets of small, round cells with scanty
cytoplasm that may appear clear because of their glycogen content.
Chondrosarcoma (choice A) is composed of malignant hyaline and myxoid
cartilage.
Giant cell tumor of bone (choice C) contains multinucleated giant cells in a
background of mononuclear stromal cells.
Malignant fibrous histiocytoma (choice D) has a background of spindled
fibroblasts in a storiform pattern admixed with bizarre, multinucleated tumor
giant cells.
Osteosarcoma (choice E) is characterized by anaplastic to well-differentiated
tumor cells that are focally making bone.




Question 2 of 7
Which of the following is the most common bone to be affected by this tumor?
/ A. Femur
/ B. Humerus
/ C. Mandible
/ D. Maxilla
/ E. Radius

Explanation - Q: 2.2 Close

The correct answer is A. While any bone can be affected by Ewing
sarcoma, the most common site is the diaphysis of the femur. Other common
sites include the flat bones of the pelvis and the tibia. The other bones listed
in the choices are not as common sites of involvement.








Question 3 of 7
The peak age of incidence for this tumor is which of the following?
/ A. Less than 1 year of age
/ B. 2 to 3 years of age
/ C. 5 to 8 years of age
/ D. 10 to 15 years of age
/ E. More than 20 years of age

Explanation - Q: 2.3 Close

The correct answer is D. Ewing sarcoma has a peak incidence of 10-15
years and is only rarely observed in children less than 4 years or adults older
than 30 years.

Question 4 of 7
A few Homer-Wright pseudorosettes are observed on biopsy. These suggest
differentiation along which of the following lines?
/ A. Bone
/ B. Cartilage
/ C. Nervous tissue
/ D. Skeletal muscle
/ E. Smooth muscle


Explanation - Q: 2.4 Close

The correct answer is C. Homer-Write pseudorosettes are circular
arrangements of tumor cells around a central fibrillary space (composed of
neurites), and these pseudorosettes specifically suggest differentiation along
neural lines.
Differentiation into bone (choice A) and cartilage (choice B) would be
suggested by the presence, respectively, of immature bone spicules or
cartilage.
Skeletal muscle differentiation (choice D) would be suggested by the
presence of rhabdomyoblasts (immature skeletal muscle cells) with cross-
striations.
Differentiation along smooth muscle lines (choice E) is hard to pick up on
routine light microscopic examination (except for the presence of nonspecific
elongated tumor cells with elliptical nuclei), and usually requires
immunohistochemistry for confirmation.


Question 5 of 7
Which of the following immunohistochemical markers would be expected to be
positive in biopsies of this tumor?
/ A. CD3
/ B. CD21
/ C. Myogenin
/ D. S-100
/ E. WT-1


Explanation - Q: 2.5 Close

The correct answer is D. S-100 protein and neuron-specific enolase, both
of which are markers for tumors with neuroendocrine differentiation, are
usually positive in Ewing sarcoma. The tumor also usually contains glycogen,
which can be helpful in the distinction with other "small blue cell tumors"
(including lymphomas, neuroblastoma, and Wilms tumor) of childhood.
CD3 (choice A) is a T cell marker.
CD21 (choice B) is a B-cell marker.
Myogenin (choice C) is a marker for rhabdomyosarcoma.
WT-1 (choice E) is the gene product marker of Wilms tumor.



Question 6 of 7
Which of the following genetic abnormalities is associated with this tumor?
/ A. Defective gene at 11p13
/ B. Defective gene at 13q14
/ C. t(X;18)
/ D. t(2;13)
/ E. t(11;22)

Explanation - Q: 2.6 Close

The correct answer is E. Approximately 90% of Ewing sarcoma cases have
a t(11;22) translocation. This translocation fuses the EWS gene in band
22q12 with the transcription factor gene FLI1 in band 11q24, producing a
hybrid transcript and a chimeric protein.
A defective gene at 11p13 (choice A) (the WT-1 gene), is associated with
Wilms tumor.
A defective gene at 13q14 (choice B) is the Rb gene, which is associated
with retinoblastoma and osteosarcoma.
t(X;18) (choice C) is a translocation associated with synovial sarcoma.
t(2;13) (choice D) is a translocation associated with alveolar
rhabdomyosarcoma.


Question 7 of 7
Which of the following tumors is most closely related to this patient's tumor?
/ A. Neuroblastoma
/ B. Primary lymphoma of bone
/ C. Primitive neuroectodermal tumor
/ D. Retinoblastoma
/ E. Wilms tumor

Explanation - Q: 2.7 Close

The correct answer is C. All of the tumors listed can have areas on biopsy
with sheets of "small blue cells" which appear similar to those seen in Ewing
sarcoma. This is particularly true if only a small biopsy sample is available for
evaluation, which increases the chance that clearly diagnostic areas are not
found. Primitive peripheral neuroectodermal tumor and Ewing sarcoma are
thought to be very similar tumors, and share the 11;22 translocation.
Primitive peripheral neuroectodermal tumor differs from Ewing sarcoma in
that it arises in brain, but it appears to be derived from the same type of stem
cells as Ewing sarcoma.
Neuroblastoma and retinoblastoma (choices A and D) show neural
differentiation like Ewing sarcoma, but usually do not have large amounts of
glycogen.
Lymphomas (choice B) may appear histologically similar, but have lymphoid
markers on immunohistochemistry.
Wilms tumor (choice E) characteristically has at least some areas that form
gland-like structures.








A 55-year-old man presents to his primary care physician with pain and swelling
of his right great toe. He reports he has had this pain for
approximately 2 days and it is getting worse. He also states that he had a similar
episode of this 4 years ago, but in the interim, he has been
symptom free. He was given something for the prior episode, but does not recall
the name. He denies trauma, fever, chills or sweats, and has
been afebrile. On physical examination, he is afebrile and his right great toe is
swollen at the metatarsaI-phalangeal joint. There is decreased
range of motion. X-ray films are unrevealing. He has no other joint involvement.
A joint aspiration is performed.
Question 1 of 7
Which of the following types of crystals in the joint aspirate would confirm the
likely diagnosis?
/ A. Negatively-birefringent needle-shaped crystals
/ B. Negatively-birefringent oval crystals
/ C. Negatively-birefringent rhomboidal shaped crystals
/ D. Positively-birefringent needle-shaped crystals
/ E. Positively-birefringent rhomboidal crystals

Explanation - Q: 3.1 Close

The correct answer is A. Negatively-birefringent needle-shaped crystals are
classically found in the diagnosis of gout.
Positively-birefringent rhomboidal crystals (choice E) are diagnostic for
pseudogout, or calcium pyrophosphate crystal deposition.
The other choices (choice B, C, D) are not diagnostic or commonly occurring
combinations.

Question 2 of 7
What is the pathologic process leading to crystal deposition in this patient's
disease?
/ A. Hypercalcemia
/ B. Hyperkalemia
/ C. Hyperlipidemia
/ D. Hypertension
/ E. Hyperuricemia

Explanation - Q: 3.2 Close

The correct answer is E. Gout is a disorder of nucleic acid metabolism
resulting in hyperuricemia, which leads to monosodium urate crystal
deposition. The endproduct of the catabolism of purines is uric acid. Gout
can be the result of the overproduction of purines, increased catabolism of
nucleic acids, or decreased urinary excretion of uric acid.
Hypercalcemia (choice A) and hyperkalemia (choice B) are not directly
associated with the development of gout.
Although there is no genetic basis for hyperlipidemia (choice C) causing
gout, 80% of patients with gout have hyperlipidemia.
Up to fifty percent of patients with gout may have hypertension (choice D),
however hypertension does not cause gout.



Question 3 of 7
The physician prescribes colchicine for this patient. Which of the following is the
mechanism of action of this drug?
/ A. BIocks phospholipase A2
/ B. Impairs leukocyte chemotaxis and degranulation
/ C. Inhibits cyclooxygenase
/ D. Inhibits reabsorption or uric acid
/ E. Inhibits xanthine oxidase

Explanation - Q: 3.3 Close

The correct answer is B. Colchicine depolymerizes microtubules, thereby
impairing leukocyte chemotaxis and degranulation. All of the other options
describe mechanisms of other drugs used in gout.
Corticosteroids block phospholipase A2 (choice A).
Aspirin and NSAIDs inhibit cyclooxygenase (choice C).
Probenecid inhibits reabsorption of uric acid (choice D).
Allopurinol inhibits xanthine oxidase (choice E) and decreases the
conversion of xanthine to uric acid.


Question 4 of 7
Which of the following is the most common side effect of colchicine?
/ A. Cardiac side effects
/ B. Gastrointestinal side effects
/ C. Hepatic side effects
/ D. Pulmonary side effects
/ E. Renal side effects

Explanation - Q: 3.4 Close

The correct answer is B. Gastrointestinal side effects namely nausea,
vomiting, and diarrhea are the most common side effect of colchicine
treatment.
The remainder of the choices cardiac (choice A), hepatic (choice C),
pulmonary (choice D), and renal (choice E) are less likely.


Question 5 of 7
Which of the following organs is most commonly additionally affected by this
patient's disease?
/ A. Heart
/ B. Kidney
/ C. Liver
/ D. Lung
/ E. Spleen

Explanation - Q: 3.5 Close

The correct answer is B. The kidneys are commonly affected in patients
with gout. In fact, 10% of deaths in patients with gout are related to the
kidney. Approximately 25% of patients with gout have kidney stones.
The heart (choice A), liver (choice C), lung (choice D) and spleen (choice
E) are less commonly affected than the kidney.


Question 6 of 7
If the patient had been 85-years-old, and had involvement of the knee that had
slowly developed over several days, which of the following types
of crystals would support the likely diagnosis in this case?
/ A. Negatively-birefringent needle-shaped crystals
/ B. Negatively-birefringent oval crystals
/ C. Negatively-birefringent rhomboidal crystals
/ D. Positively-birefringent needle-shaped crystals
/ E. Positively-birefringent rhomboidal crystals



Explanation - Q: 3.6 Close

The correct answer is E. Positively-birefringent rhomboidal crystals are
classically found in cases of pseudogout. Pseudogout, or calcium
pyrophosphate dihydrate (CPPD) disease, tends to affect older individuals,
tends to affect the knee, wrist, shoulder, ankle, elbow, and hands, and may
develop more insidiously. Nevertheless, the similarities between gout and
pseudogout require aspiration of the joint for diagnosis.
Negatively-birefringent needle-shaped crystals (choice A) are diagnostic for
gout.
The other choices (choice B, C, D) are not diagnostic or commonly occurring
combinations.



Question 7 of 7
Which of the following is the recommended treatment for the patient with the
preceding presentation?
/ A. AIIopurinol
/ B. Colchicine
/ C. NSAIDs
/ D. Prednisone
/ E. Probenecid

Explanation - Q: 3.7 Close

The correct answer is C. Pseudogout does not require specific treatment,
and is treated symptomatically most commonly with NSAIDs.
Allopurinol (choice A), colchicine (choice B), prednisone (choice D) and
probenecid (choice E) may be used for the treatment of gout, but are not
indicated in the treatment of pseudogout.


An 88-year-old woman is brought to the emergency department by ambulance
after a falI 12 hours prior to presentation. She complains of pain
in her left wrist, Ieg, and lower back. She has recently been told by her primary
care physician that she has osteoporosis. On physical
examination, her left wrist is tender and has a deformity. She is tender in her low
lumbar region. Her left lower extremity is shortened and
externally rotated. She is has normal motor strength, sensation, and pulses in her
upper and lower extremities. X-ray films reveal a dorsally
angulated and displaced fracture of her distal radius. X-ray films of her lumbar
spine reveal a compression fracture of the T12 vertebra. X-ray
films of her left leg reveal a left femur fracture.
Question 1 of 5
The basic abnormality affecting this patient's bones is which of the following?
/ A. Abnormal bone remodeling
/ B. Abnormal collagen synthesis and failure of crosslinking
/ C. Abnormal collagen synthesis with normal crosslinking
/ D. Reduction in bone formation with defective bone mineralization
/ E. Reduction in bone mass with normal bone mineralization
/ F. Reduction in osteoclast function

Explanation - Q: 4.1 Close

The correct answer is E. Osteoporosis is a reduction in bone mass with
normal bone mineralization. There is a quantitative defect of bone. However,
the bone that is present has normal mineralization.
Abnormal bone remodeling (choice A) is characteristic of Paget disease
This woman's condition is not due to abnormal collagen synthesis, with
(choice C) or without (choice B) normal cross-linking. Abnormal collagen
synthesis associated with a failure of cross-linking is the description of
osteogenesis imperfecta.
Reduction in bone formation with defective bone mineralization (choice D) is
defined as osteomalacia.
Reduction in osteoclast function (choice F) is the definition for osteopetrosis,
or Marble bone disease


Question 2 of 5
Which of the following is the predominant collagen type in bone?
/ A. Type l
/ B. Type ll
/ C. Type lll
/ D. Type IV
/ E. Type X

Explanation - Q: 4.2 Close

The correct answer is A. Type I collagen is the predominant collagen in
bone (this can be remembered easily by "bone").
Type II collagen (choice B) is found in cartilage.
Type III collagen (choice C) is found in skin, blood vessels, uterus, and fetal
tissue.
Type IV collagen (choice D) is found in the basement membrane.
Type X collagen (choice E) is found in the epiphyseal plate.

Question 3 of 5
Which of the following best describes the patient's wrist fracture?
/ A. Bennett's fracture
/ B. Colles fracture
/ C. Jones fracture
/ D. Salter-Harris fracture
/ E. Smith fracture

Explanation - Q: 4.3 Close

The correct answer is B. Dorsally displaced distal radius fractures are
commonly called Colles fractures.
Bennett's fractures (choice A) are fractures of the base of the first
metacarpal.
Jones fracture (choice C) are fractures of the fifth metatarsal in the foot.
Salter-Harris fractures (choice D) are a classification system of growth plate
(physeal) fractures in children.
Smith fractures (choice E) are volar displaced fractures of the distal radius.

Question 4 of 5
X-ray films also reveal a femur fracture. Which of the following types of femur
fracture results in the highest incidence of avascular necrosis?
/ A. Distal femur
/ B. Femoral neck
/ C. Femoral shaft
/ D. Greater trochanter
/ E. Lesser trochanter

Explanation - Q: 4.3 Close

The correct answer is B. Dorsally displaced distal radius fractures are
commonly called Colles fractures.
Bennett's fractures (choice A) are fractures of the base of the first
metacarpal.
Jones fracture (choice C) are fractures of the fifth metatarsal in the foot.
Salter-Harris fractures (choice D) are a classification system of growth plate
(physeal) fractures in children.
Smith fractures (choice E) are volar displaced fractures of the distal radius.

Question 5 of 5
AII of the patient's fractures are treated appropriately. In addition to narcotics,
she is started on celecoxib for pain. What is the advantage of
celecoxib over other nonsteroidal anti-inflammatory medications?
/ A. Greater analgesic effect
/ B. Greater anti-inflammatory effect
/ C. Greater antipyretic effect
/ D. Less cardiovascular toxicity
/ E. Less gastrointestinal irritation


Explanation - Q: 4.5 Close

The correct answer is E. Celecoxib selectively inhibits the COX-2 isoform of
cyclooxygenase. It spares the COX-1 isoform, which helps maintain the
gastric mucosa, thus resulting in less gastrointestinal irritation.
There is no conclusive evidence that one NSAID medication has greater
analgesic (choice A), anti-inflammatory (choice B) or antipyretic (choice C)
effects.
There is also no evidence that celecoxib has less cardiovascular toxicity
(choice D) compared to other NSAIDS

A 32-year-old man is involved in a high speed motorcycle accident. He sustains
multiple injuries, including a pelvic fracture and an open left
femur fracture. He is taken urgently to the operating room for irrigation and
debridement of his wounds. They are unable to stabilize his
fractures at the time of admission because he is medically unstable. On hospital
day number two, he is doing welI, however later that evening,
he becomes confused, tachypneic, dyspneic, and develops petechiae. An
electrocardiogram is normaI. Chest radiographs are normaI.
Question 1 of 7
Which of the following is the most likely diagnosis?
/ A. Fat embolism
/ B. Pneumonia
/ C. Pneumothorax
/ D. Pulmonary contusion
/ E. Pulmonary embolism

Explanation - Q: 5.1 Close

The correct answer is A. Fat embolism is usually seen 24-72 hours after
trauma. Classic signs include tachypnea, confusion, and petechiae.
Additional signs include tachycardia, hypoxemia, and pulmonary edema. Its
incidence may be decreased by early skeletal stabilization of long bone
fractures. Treatment includes pulmonary support.
Pneumonia (choice B) would most likely show up on the chest x-ray film,
which was normal in this situation.
Pneumothorax (choice C) would cause pulmonary symptoms, but a
significant pneumothorax will show up on chest x-ray films. Also if the patient
had a pneumothorax from the initial accident, he most likely would have been
symptomatic from it at initial presentation.
A pulmonary contusion (choice D) is possible, and should always be
considered in patients with high-energy trauma, however the patient's
symptoms of tachypnea, confusion, and petechiae are classic for fat
embolus.
Patients with a pulmonary embolus (choice E) usually present with pleuritic
chest pain; tachypnea and tachycardia are hallmark signs of a pulmonary
embolus. ECG changes such as a right bundle branch block with right axis
deviation are seen in approximately 25% of patients.



Question 2 of 7
The patient finally improves medically, and has his fractures stabilized. After
surgery, he is placed on gentamicin (an aminoglycoside) and
cefazolin (a cephalosporin) for prophylaxis from infection. Which of the following
is the mechanism of action of aminoglycosides?
/ A. BIocks cell wall synthesis
/ B. BIocks nucleotide synthesis
/ C. BIocks peptidoglycan synthesis
/ D. BIocks protein synthesis at the 30S ribosomal subunit
/ E. BIocks protein synthesis at the 50S ribosomal subunit

Explanation - Q: 5.2 Close

The correct answer is D. Aminoglycosides block protein synthesis by acting
at the 30S ribosomal subunit.
Penicillins and cephalosporins block cell wall synthesis (choice A) by
inhibition of peptidoglycan cross-linking.
Sulfonamides blocks nucleotide synthesis (choice B).
Antibiotics such as bacitracin and vancomycin block peptidoglycan synthesis
(choice C).
Clindamycin and macrolide antibiotics block protein synthesis at the 50S
ribosomal subunit (choice E).

Question 3 of 7
Three weeks postoperatively, the patient is still hospitalized and immobilized. A
nurse reports that he has swelling and pain in his right calf. On
physical examination, his right calf is 3 cm in diameter larger than the left calf,
and there is a palpable cord in his posterior calf. There is a
positive Homans' sign (dorsiflexion of the foot produces calf pain). Which of the
following is the most likely diagnosis?
/ A. Antibiotic allergy
/ B. Compartment syndrome
/ C. Congestive heart failure
/ D. Deep vein thrombosis
/ E. Late fracture

Explanation - Q: 5.3 Close

The correct answer is D. This patient has unilateral leg swelling and pain,
with a palpable cord, and a positive Homans' sign. This suggests a deep vein
thrombosis (although a cord may not be palpable in many cases of deep vein
thromboses of the calf). The risk factors for DVT are described by Virchow's
triad of stasis, endothelial injury, and hypercoagulable state. This patient's
surgery has rendered him immobile, and thus he has stasis. In addition, his
recent operation predisposes him to this condition. Diagnosis can be
confirmed by venography or duplex ultrasonography.
Antibiotic allergies (choice A) are common, however, they are usually not
isolated to one anatomic region and usually do not cause pain and swelling.
It is important to always be aware of compartment syndrome (choice B), but
compartment syndrome three weeks after injury is less likely.
Congestive heart failure (choice C) commonly occurs in the postoperative
period. However, congestive heart failure presents with bilateral leg swelling,
not unilateral swelling. Pain is not a frequent problem.
Late fractures (choice E) do not commonly occur without another trauma.
However, it is important to look for missed fractures, especially in patients
with multiple distracting injuries.


Question 4 of 7
That evening, the patient develops pleuritic chest pain, tachypnea, and
tachycardia. Pulse oximetry reveals a oxygen saturation of 74%. An
electrocardiogram is performed, which reveals a right bundle branch block with
right axis deviation. A chest x-ray film is normaI. Which of the
following is the most likely diagnosis?
/ A. Cardiac arrhythmia
/ B. Fat embolism
/ C. Myocardial infarction
/ D. Pneumonia
/ E. Pulmonary embolism


Explanation - Q: 5.4 Close

The correct answer is E. Pleuritic chest pain, tachypnea, and tachycardia
are hallmark signs of a pulmonary embolus. ECG changes such as a right
bundle branch block with right axis deviation are seen in approximately 25%
of patients.
Cardiac arrhythmia (choice A) would be less likely because the ECG does
not show any type of arrhythmia.
Fat embolism (choice B) is usually seen 24-72 hours after trauma. Classic
signs include tachypnea, confusion, and petechiae. Additional signs include
tachycardia, hypoxemia, and pulmonary edema.
Myocardial infarction (choice C) is seen in the postoperative period, however
the ECG does not have any signs of a recent myocardial infarction.
Pneumonia (choice D) would most likely show up on chest x-ray films, which
were normal in this situation.


Question 5 of 7
Intravenous heparin is started as the treatment for the above condition. Which of
the following is the mechanism of action of heparin?
/ A. Binds to antithrombin lll
/ B. Converts plasminogen to plasmin
/ C. Inhibits platelet aggregation
/ D. Interferes with vitamin K-dependent clotting factors
/ E. Mechanically compresses blood vessels

Explanation - Q: 5.5 Close

The correct answer is A. Heparin acts indirectly by binding to antithrombin
III. As heparin binds to antithrombin III, a complex is formed that enhances
the proteolytic activity of antithrombin III.
Thrombolytics such as streptokinase, urokinase, and tissue plasminogen
activator convert plasminogen to plasmin (choice B), which, in turn, breaks
down fibrin.
Ticlopidine inhibits platelet aggregation (choice C).
Warfarin interferes with vitamin K-dependent clotting factors (choice D).
Compression boots mechanically compress blood vessels (choice E).



Question 6 of 7
Which of the following laboratory tests would be most useful in monitoring the
anticoagulation effect of heparin?
/ A. Activated partial thromboplastin time (aPTT)
/ B. BIeeding time
/ C. International normalized ratio (INR)
/ D. PIatelet count
/ E. Prothrombin time (PT)

Explanation - Q: 5.6 Close

The correct answer is A. The activated partial thromboplastin time (aPTT)
is used to monitor the intrinsic pathway of the coagulation cascade. It is the
test of choice to monitor the effects of heparin on the coagulation cascade.
A bleeding time (choice B) is most helpful as an indicator of platelet
abnormality, either in number or function.
The international normalized ratio (INR) (choice C) is the patient PT divided
by the midpoint of the laboratory PT reference range. This allows for
standardization of the PT from laboratory to laboratory. The INR is a good
measure of the extrinsic pathway, and can be used to monitor anticoagulant
therapy with warfarin.
A platelet count (choice D) is a direct count of the number of platelets, and is
commonly used to evaluate and treat platelet disorders
The prothrombin time (choice E) is used to monitor anticoagulant therapy
with warfarin, as part of a general screen for coagulation system disorders,
and as a liver function test. An abnormal value in the PT indicates a disorder
in the extrinsic pathway of the coagulation cascade.



Question 7 of 7

If the test reveals that the heparin dose is supratherapeutic, which of the
following would be the most appropriate pharmacotherapy?
/ A. Acetylcysteine
/ B. FIumazenil
/ C. Naloxone
/ D. Pralidoxime
/ E. Protamine sulfate

Explanation - Q: 5.7 Close

The correct answer is E. Protamine sulfate is administered in the setting of
heparin overdose when the patient is at risk for hemmorhage. It acts by
ionically binding heparin to form a stable complex, which neutralizes the
anticoagulant effects of heparin.
Acetylcysteine (choice A) is administered in acetominophen overdose. It
acts by breaking disulfide bonds and replenishing glutathione.
Flumazenil (choice B) is given in the setting of benzodiazopine overdose. It
antagonizes the benzodiazepine binding site on GABA-A receptors.
Naloxone (choice C), an opioid antagonist, is given in opiate overdose.
Pralidoxime (choice D) is a acetylcholinesterase regenerator and is used as
an antidote for organophosphate poisoning.



A 17-year-old boy is playing football and is tackled by another player. The
opponent hits the lateral aspect of his knee. He presents to a
primary care clinic 1 week after the injury, complaining of swelling and pain in his
right knee. On physical examination of his right knee, there is
a large effusion. There is increased laxity (as compared to his uninjured knee) of
his knee when his knee is passively placed in a valgus
(abducted) position. In addition, there is significant anterior translation of his tibia
with respect to his femur when his knee is tested at ninety
degrees of flexion.

Question 1 of 3

Injury to which of the following structures most likely accounts for the increased
laxity of his knee when his knee is passively placed in a valgus
(abducted) position?

/ A. Anterior cruciate ligament
/ B. Lateral collateral ligament
/ C. Medial collateral ligament
/ D. Patellar ligament
/ E. Posterior cruciate ligament


Explanation - Q: 6.1 Close

The correct answer is C. The medial collateral ligament originates from the
medial femoral epicondyle and inserts on the medial proximal tibia. Its
function is to prevent valgus instability of the knee. When a valgus stress is
placed on the tibia, the medial collateral ligament is stressed and prevents
displacement.
The anterior cruciate ligament (choice A) passes from the medial surface of
the lateral femoral condyle to the anterior intercondylar area of the tibia. Its
primary function is to prevent anterior translation of the tibia in reference to
the femur (or posterior displacement of the femur in reference to the tibia)
The lateral collateral ligament (choice B) originates on the lateral femoral
epicondyle and inserts on the head of the fibula. It functions to prevent varus
instability of the knee. When a varus stress (adduction) is placed on the tibia,
the lateral collateral ligament becomes taut and prevents displacement.
The patellar ligament (choice D) runs from the inferior pole of the patella to
the tibial tubercle, and is commonly referred to as the patellar tendon.
However, this structure connects bone to bone, which is the definition of a
ligament, not a tendon.
The posterior cruciate ligament (choice E) passes from the lateral surface of
the medial femoral condyle to the posterior intercondylar area of the tibia. Its
primary function is to prevent posterior translation of the tibia in reference to
the femur.







Question 2 of 3
Injury to which of the following structures would most likely account for the
increased anterior translation of the tibia in reference to the femur?
/ A. Anterior cruciate ligament
/ B. Lateral collateral ligament
/ C. Medial collateral ligament
/ D. Patellar ligament
/ E. Posterior cruciate ligament

Explanation - Q: 6.2 Close

The correct answer is A. The anterior cruciate ligament passes from the
medial surface of the lateral femoral condyle to the anterior intercondylar
area of the tibia. Its primary function is to prevent anterior translation of the
tibia in reference to the femur (or posterior displacement of the femur in
reference to the tibia). The Lachman test is the most sensitive examination
maneuver for evaluating a anterior cruciate ligament tear. The test is
performed with the knee flexed to 30 degrees and the tibia is translated
anterior with respect to the femur. It is positive if there is increased
translation as compared to the opposite side, or more translation than
normal.
The lateral collateral ligament (choice B) originates on the lateral femoral
epicondyle and inserts on the head of the fibula. It functions to prevent varus
instability of the knee. When a varus stress (adducted) is placed on the tibia,
the lateral collateral ligament becomes taut and prevents displacement.
The medial collateral ligament (choice C) originates from the medial femoral
epicondyle and inserts on the medial proximal tibia. Its function is to prevent
valgus instability of the knee. When a valgus stress is placed on the tibia, the
medial collateral ligament is stressed and prevents displacement.
The patellar ligament (choice D) runs from the inferior pole of the patella to
the tibial tubercle, and is commonly referred to as the patellar tendon.
However, this structure connects bone to bone, which is the definition of a
ligament, not a tendon.
The posterior cruciate ligament (choice E) passes from the lateral surface of
the medial femoral condyle to the posterior intercondylar area of the tibia. Its
primary function is to prevent posterior translation of the tibia in reference to
the femur.





Question 3 of 3

In addition to the two ligaments injured, which of the following additional injuries
is also likely present in this individuaI?
/ A. Lateral femoral condyle fracture
/ B. Lateral meniscal tear
/ C. Medial femoral condyle fracture
/ D. Medial meniscal tear
/ E. Medial tibial plateau fracture

Explanation - Q: 6.3 Close

The correct answer is D. This is commonly referred to as the "unhappy triad
of knee injury." Medial meniscal injury, in combination with anterior cruciate
ligament and medical collateral ligament injuries, is commonly referred to as
the unhappy triad of the knee. The medial meniscus is attached to the medial
collateral ligament and injury to the medial meniscus is associated with
medial collateral ligament injuries. However, both injuries do occur in
isolation. The medial meniscus is a c-shaped cartilaginous structure that acts
as a cushion or shock absorber. It is more commonly torn than the lateral
meniscus.
The lateral meniscus (choice B) is also a cartilaginous incomplete circular
structure that also acts as a shock absorber. It is not associated with the
unhappy triad of the knee and is less frequently injured than the medial
meniscus.
Lateral femoral condyle fracture (choice A), medial femoral condyle fracture
(choice C), and medial tibial plateau fractures (choice E) are not associated
with the unhappy triad. However, they do occur with trauma to the knee.



A 48-year-old man with a history of diabetes presents to the emergency
department with a poorly healing right foot ulcer. The ulcer has been
present for 1 month, and has never been treated. The patient denies fever or
chills, and complains of mild pain at the ulcer site. His
temperature is 37.6 C (99.6 F), blood pressure is 158/86 mm Hg, pulse is 94/min,
and respirations are 18/min. The patient has diminished
popliteaI, posterior tibiaI, and dorsalis pedis pulses bilaterally, but the right side is
worse than the left side. He has soft tissue swelling around
the first toe with a 1 x 1 cm ulcer at the tip of the toe and surrounding erythema
and purulent discharge. An x-ray film of the foot shows soft
tissue swelling and signs of early bone destruction in the first metatarsaI.


Question 1 of 6

What is the most likely cause of the bone destruction evidenced on the x-ray
film?
/ A. Osteitis deformans
/ B. Osteoarthritis
/ C. Osteomalacia
/ D. Osteomyelitis
/ E. Osteoporosis




Explanation - Q: 7.1 Close

The correct answer is D. Osteomyelitis is an infection of the bone that can
be caused either by hematogenous spread or contiguous spread from a local
focus of infection. In this case, the patient has a poorly healing ulcer in his
right toe, which likely has spread to the metatarsal bone. Osteomyelitis can
be caused by any microorganism, but is most often due to pyogenic bacteria
or mycobacteria.
Osteitis deformans (choice A) is also known as Paget disease of the bone. It
is an inflammatory disease of unknown etiology, characterized by abnormal
and chaotic bone resorption and formation. It most commonly occurs in the
spine, pelvis, skull, and long bones of the lower extremities.
Osteoarthritis (choice B) is the most common form of arthritis. It is a chronic
noninflammatory disease characterized by degeneration of articular cartilage.
It is accompanied by new bone formation rather than bone destruction.
Osteomalacia (choice C) is due to vitamin D deficiency in adults. It is
associated with defective calcification of the bone matrix. Patients can
experience bone pain, which often occurs in the ribs and thighs. X-ray films
show diffuse radiolucency.
Osteoporosis (choice E) is a disease of the skeleton characterized by a
decrease in bone mass. There is both impaired bone synthesis and
increased bone resorption. It predisposes patients to fractures in weight-
bearing bones, particularly the upper femur and vertebral column.






Question 2 of 6
Which of the following physical findings would help support the diagnosis?
/ A. Ability to probe directly to bone from the ulcer site
/ B. Bouchard's nodes
/ C. Craniotabes
/ D. Heberden's nodes
/ E. Rheumatoid nodules

Explanation - Q: 7.2 Close

The correct answer is A. The ability to probe directly to bone or visualize
bone at an infected wound increases the likelihood of underlying
osteomyelitis. A common cause of osteomyelitis is contiguous spread from a
local focus of infection.
Bouchard's nodes (choice B) are osteophytes at the proximal
interphalangeal joints of the fingers. Heberden's nodes (choice D) are
osteophytes at the distal interphalangeal joints of the fingers. They are both
seen in osteoarthritis.
Craniotabes (choice C) is a thinning and softening of the occipital and
parietal bones of the skull. It is associated with rickets, which is caused by
vitamin D deficiency in children.
Rheumatoid nodules (choice E) are a manifestation of rheumatoid arthritis.
The nodules form subcutaneously in bursae, and along tendon sheaths.

Question 3 of 6
Which of the following laboratory test results would help confirm the diagnosis?
/ A. EIevated erythrocyte sedimentation rate (ESR)
/ B. Joint fluid evaluation
/ C. Low albumin
/ D. Positive antinuclear antibody titer
/ E. Positive rheumatoid factor

Explanation - Q: 7.3 Close

The correct answer is A. The erythrocyte sedimentation rate (ESR) is an
acute phase reactant, and is often elevated when there is inflammation or
infection. The ESR is elevated in most cases of osteomyelitis, even in cases
when constitutional symptoms or leukocytosis are absent.
Joint fluid evaluation (choice B) can be done if there is synovitis present and
the patient is suspected of having an infectious or inflammatory arthritis. This
patient has no such signs or symptoms.
Low albumin (choice C) is a sign of either poor nutritional status, increased
protein loss as in nephritic syndrome, or decreased synthesis due to liver
disease.
Positive antinuclear antibody titer (choice D) is a nonspecific finding, but can
be associated with an underlying rheumatologic condition such as systemic
lupus erythematosus or rheumatoid arthritis.
Positive rheumatoid factor (choice E) is found in patients with rheumatoid
arthritis.


Question 4 of 6
Which of the following tests would help assess the extent of the disease?
/ A. Bone mineral density study
/ B. Full body bone survey
/ C. MRI of the foot
/ D. Serum protein electrophoresis
/ E. Urine protein electrophoresis

Explanation - Q: 7.4 Close

The correct answer is C. MRI is a sensitive test for acute osteomyelitis. It
can yield information about the activity and anatomic extent of the infection. It
can also reveal other anatomical abnormalities, such as fistulas and
abscesses.
A bone mineral density study (choice A) is used to diagnose osteoporosis
and follow its response to treatment.
A full body bone survey (choice B) is used to evaluate diseases that can
affect multiple skeletal bones, such as metastatic tumor or multiple myeloma.
Osteomyelitis, on the other hand, generally affects only one specific area,
unless there is diffuse hematogenous spread from a focus such as
endocarditis.
Serum protein electrophoresis (choice D) and urine protein electrophoresis
(choice E) are both used to analyze the presence of immunoglobulins in the
serum and urine. They can be used to help diagnose multiple myeloma.


Question 5 of 6
Gram's stain and culture of the affected tissue reveal gram-positive cocci that are
coagulase-positive. Which of the following is the most likely
organism?



Explanation - Q: 7.5 Close

The correct answer is D.Staphylococcus aureus is a gram-positive coccus
that is often involved in osteomyelitis, especially those that are secondary to
a contiguous spread from a local focus. The ability of Staphylococcus aureus
to produce coagulase differentiates it from other gram-positive cocci such as
group A streptococci (choice A) and Staphylococcus epidermis(choice E),
which are not able to produce coagulase. In addition, Staphylococcus
epidermis is part of the normal skin flora.
Neisseria gonorrhoeae(choice B) is a gram-negative coccus that is also
capable of causing osteomyelitis, as well as septic arthritis in sexually active
people.
Pseudomonas aeruginosa(choice C) is a gram-negative rod. It can also
cause osteomyelitis that is often associated with puncture wounds of the
foot, thermal burns, and intravenous drug abusers.


Question 6 of 6
The patient is treated with oxacillin. Which of the following is the mechanism of
action of this medication?
/ A. Bactericidal agent that inhibits cell wall synthesis
/ B. Bactericidal agent that inhibits DNA gyrase and topoisomerase IV
/ C. Bacteriostatic agent that inhibits cell wall synthesis
/ D. Inhibition of protein synthesis by blocking translocation
/ E. Monobactam antibiotic that inhibits cell wall synthesis

Explanation - Q: 7.6 Close

The correct answer is A. Oxacillin is a bactericidal antibiotic that inhibits cell
wall synthesis by binding to penicillin-binding proteins. It also blocks
transpeptidase cross-linking of the cell wall, and activates autolytic enzymes.
As it inhibits cell wall synthesis, it kills bacteria and is therefore bactericidal. It
is not bacteriostatic (choice C). Bacteriostatic antibiotics retard the growth
and multiplication of microorganisms, but do not kill them in large numbers.
Fluoroquinolones are bactericidal agents that inhibit DNA gyrase and
topoisomerase IV (choice B). Examples of fluoroquinolones include
ciprofloxacin, levofloxacin, and norfloxacin. They are active against gram-
negative rods, Neisseria, and some gram-positive organisms.
Macrolides inhibit protein synthesis by blocking translocation (choice D). The
macrolides include erythromycin, clarithromycin, and azithromycin. They are
active against gram-positive cocci, Mycoplasma, Legionella, and Chlamydia.
Aztreonam is a monobactam antibiotic that inhibits cell wall synthesis
(choice E). Unlike other beta lactam antibiotics, it is resistant to beta-
lactamases. It is active against most gram-negative rods, but it has no
activity against gram-positive organisms or anaerobes.


A 2 1/2-year-old boy is brought to the emergency department by his parents
because he is has stopped bearing weight on his right lower
extremity. His parents report that he has been irritable for the last 12 hours and
started limping 6 hours ago. He now refuses to put any weight
on the affected leg. They also report a low grade fever for the past 24 hours. On
physical examination, his temperature is 39.1 C (102.4 F). The
child is lying with his right hip slightly flexed, externally rotated, and in abduction.
He does not allow any passive motion of his hip. Laboratory
studies reveal a white blood cell count of 14,000/mm3. The erythrocyte
sedimentation rate is 58 mm/hr. X-ray films of the right hip and pelvis
are normaI.


Question 1 of 7

Which of the following is the most likely diagnosis?
/ A. Hip fracture
/ B. Hip sprain
/ C. Legg-Calve-Perthes disease
/ D. Septic arthritis
/ E. SIipped capital femoral epiphysis

Explanation - Q: 8.1 Close

The correct answer is D. There is no history of trauma, and this child has
new onset of pain with associated fevers. In addition, the white blood cell
count and erythrocyte sedimentation rate are elevated. The child is lying with
his right hip slightly flexed, externally rotated, and in abduction. This position
maximizes joint volume and is the most comfortable in septic arthritis of the
hip. Negative hip x-ray films do not confirm or exclude the diagnosis of septic
arthritis. Joint aspiration is often diagnostic. The joint fluid is often cloudy or
bloody, and the white count is typically elevated, usually above 20,000/mm
3
,
often above 100,000/mm
3
.
A hip fracture (choice A) would be unlikely in the setting of normal x-ray films
and no trauma.
A hip sprain (choice B) is nonspecific, and most likely would not cause this
much pain, or fevers and an elevated white blood cell count.
Legg-Calve-Perthes disease (choice C) is a noninflammatory deformity of
the femoral head. It is usually seen in boys 4 to 8 years old. This child is too
young for this condition. There is a vascular insult that leads to osteonecrosis
of the proximal femoral epiphysis. There are usually radiographic signs of
proximal femoral epiphysis collapse.
Slipped capital femoral epiphysis (choice E) is seen most commonly in
obese males during adolescence. The femoral epiphysis remains in the
acetabulum and is displaced from the remainder of the femoral neck. Most
cases will be diagnosed by plain radiographs. The x-ray films in this child are
negative, and he is far too young for this condition.




Explanation - Q: 8.2 Close

The correct answer is E. In children greater than 2 or 3 years of age,
Staphylococcus aureus is the most common cause of septic arthritis of the
hip. Approximately 50% of cases in this age group is caused by
Staphylococcus aureus. Approximately 25% are caused by streptococci.
Borrelia burgdorferi(choice A) is a spirochete that is transmitted by the deer
tick, Ixodes, and causes Lyme disease.
Group B streptococci (choice B) are gram-positive cocci in chains, which are
the most common cause of septic arthritis of the hip in healthy neonates.
Streptococci are found in approximately 50% of affected children in this age
group.
Hemophilus influenzae(choice C) has in the past been the most common
cause of septic hip arthritis in children 3 months to 3 years of age. However,
with the advent of H. influenzae type B vaccines in the late 1980s, the
incidence of infection secondary to Hemophilus influenzae has dramatically
decreased. Children who are not immunized are at risk of Hemophilus
influenzae infection.
Neisseria gonorrhoeae(choice D) is the most common cause for septic
arthritis in sexually active adolescents, but not in toddlers. The knee is most
commonly affected.




Explanation - Q: 8.3 Close

The correct answer is D. Neisseria gonorrhoeae is a gram-negative diplococcus
that is the most common cause of septic arthritis in sexually active adolescents.
Newborns are also at risk during birth. Adolescents will present with tenosynovitis
of the hands and feet as well as migratory polyarthralgias. The knee is most
commonly affected.
Borrelia burgdorferi(choice A) produces arthritis in association with Lyme disease,
but is not the most common cause in a 17-year-old.
Group B streptococci (choice B) are more likely to cause septic arthritis of the hip in
otherwise healthy neonates, not in a 17-year-old.
Hemophilus influenzae(choice C) was a common cause of septic hip arthritis in
children 3 months to 3 years of age prior to the advent of the type B H. influenzae
vaccine.
In children greater than 2 or 3 years of age, Staphylococcus aureus(choice E) is the
most common cause of septic arthritis of the hip, but N. gonorrhoeae would be more
likely in a sexually active adolescent.
Question 4 of 7
If the patient was had not received any scheduled immunizations, which of the
following would be the most likely pathogen?


Explanation - Q: 8.4 Close

The correct answer is C. Hemophilus influenzae in the past has been the
most common cause of septic hip arthritis in children 3 months to 3 years of
age. However, with the advent of H. influenzae type B vaccines in the late
1980's, the incidence of infection secondary to Hemophilus influenzae has
dramatically decreased. Children who are not immunized are at increased
risk for Hemophilus influenzae. The other organisms are causes of septic
arthritis, but are not the most common cause in an unvaccinated 2-year-old.




Explanation - Q: 8.5 Close

The correct answer is B. Group B streptococci are gram-positive cocci in
chains that are the most common cause of septic arthritis of the hip in
healthy neonates. Streptococci are found in approximately 50% of affected
children in this age group. The other organisms are causes of septic arthritis,
but are not the most common pathogens in otherwise healthy neonates.



Question 6 of 7

Which of the following most accurately describes the appearance of Neisseria
gonorrhoeae in Gram's-stained preparations?

/ A. Gram-negative diplococci
/ B. Gram-negative rods
/ C. Gram-positive cocci in chains
/ D. Gram-positive cocci in clusters
/ E. Gram-positive rods


Explanation - Q: 8.6 Close

The correct answer is A. Neisseria gonorrhoeae is identified as a gram-
negative diplococcus on Gram's stain.
An example of gram-negative rods (choice B) would be Escherichia coli.
Streptococci are gram-positive cocci in chains (choice C).
Staphylococci are gram-positive cocci in clusters (choice D).
Listeria and Corynebacterium are gram-positive rods (choice E).



Question 7 of 7

Which of the following culture media is commonly used to identify
Haemophilus Influenzae ?

/ A. Bordeaux-Gengou (Potato) agar
/ B. Charcoal yeast extract agar
/ C. Chocolate agar
/ D. Lowenstein-Jensen agar
/ E. MacConkey's agar


Explanation - Q: 8.7 Close

The correct answer is C. Chocolate agar (agar to which sheep's blood is
added, then heated) may be used to isolate Hemophilus influenzae.
Bordeaux-Gengou (Potato) agar (choice A) can be used to isolate
Bordetella pertussis.
Charcoal yeast extract agar (choice B) can be used to isolate Legionella
pneumophila.
Lowenstein-Jensen agar (choice D) is used for the isolation of
Mycobacterium tuberculosis.
MacConkey's agar (choice E) is used for the isolation of lactose fermenting
enterics

A patient undergoing chest x-ray following an automobile accident is found to
have an enlarged mediastinum with bilateral hilar and right
paratracheal adenopathy. The patient has been asymptomatic, but careful
examination demonstrates an enlarged cervical lymph node. This
node is biopsied and demonstrates involvement by smalI, non-caseating
granulomas.
Question 1 of 5
Occasional giant cells with stellate inclusions are seen within the granulomas.
These are most likely which of the following?
/ A. Anitschkow cells
/ B. Aschoff bodies
/ C. Asteroid bodies
/ D. Paget's cells
/ E. Schaumann's bodies

Explanation - Q: 1.1 Close

The correct answer is C. The cells described are asteroid bodies, and are
classically associated with sarcoidosis, although they are not completely
specific for this disease.
Anitschkow cells (choice A) are a form of activated histiocyte with an
unusual wavy chromatin. They are found within inflammatory lesions called
Aschoff bodies (choice B) in the hearts of patients with acute rheumatic
heart disease.
Paget's cells (choice D) are individual adenocarcinoma cells found within the
squamous epithelium of the skin.
Schaumann's bodies (choice E) are laminated concretions sometimes found
in sarcoidosis.


Question 2 of 5
Acid fast and Grocott silver stain performed on the biopsy are negative. Which of
the following is the most likely diagnosis?
/ A. Aspergillosis
/ B. Candidiasis
/ C. Histoplasmosis
/ D. Sarcoidosis
/ E. Tuberculosis

Explanation - Q: 1.2 Close

The correct answer is D. Acid fast stains are used to identify mycobacteria,
and the Grocott silver stain is used to identify fungal organisms. The
combination of non-caseating granulomas and negative stains for
mycobacteria and fungi is most consistent with sarcoidosis. This condition is
a multisystem disease characterized by formation of non-caseating
granulomas in a variety of body sites. The etiology is unknown, and the
symptoms and signs depend on the site and degree of involvement. Patients
may present with lymphadenopathy, fever, weight loss, arthralgias, skin
lesions, erythema nodosum, or organ (lungs, spleen, liver, bone marrow)
involvement. Mediastinal lymphadenopathy, with bilateral hilar and right
paratracheal adenopathy, is a particularly common finding on chest x-ray
films in these patients.
The Grocott stain would be positive for fungal organisms that cause
aspergillosis (choice A), candidiasis (choice B), and histoplasmosis (choice
C).
Acid fast stains for mycobacteria would be positive in tuberculosis (choice
E).



Question 3 of 5
This patient is most likely in which of the following age groups?
/ A. 0-5 years
/ B. 10-15 years
/ C. 20-40 years
/ D. 55-65 years
/ E. 80-90 years

Explanation - Q: 1.3 Close

The correct answer is C. The peak incidence of sarcoidosis is in persons
aged 20 to 40 years. While the disease occurs worldwide and in all races, it
is most common in Northern Europeans (notably Swedish) and American
blacks, and can be found in both men and women. Patients of other age
groups develop sarcoidosis much less commonly.


Question 4 of 5
If an enlarged peripheral lymph node had not been available for biopsy, biopsy of
which of the following sites would have been most likely to
safely yield the diagnosis?
/ A. Colon
/ B. Liver
/ C. Pancreas
/ D. Spleen
/ E. Stomach
Explanation - Q: 1.4 Close

The correct answer is B. Percutaneous liver biopsy can demonstrate
hepatic granulomas in about 70% of patients, and is the preferred biopsy site
if an enlarged peripheral lymph node is not available for biopsy.
The other sites listed (choices A, C, and E) are less commonly involved by
sarcoidosis.
The spleen (choice D) is also often involved, but splenic biopsy tends to be
dangerous because of the spleen's predilection for hemorrhage.


Question 5 of 5
The most appropriate therapy for this patient would include which of the
following?
/ A. Cyclophosphamide
/ B. Methotrexate
/ C. Naproxen
/ D. Oxycodone
/ E. Prednisone

Explanation - Q: 1.5 Close

The correct answer is E. The drugs of choice for the treatment of
sarcoidosis are the corticosteroids. Indications for the treatment of
sarcoidosis with corticosteroids, such as prednisone, include the classical
constitutional signs and symptoms, hypercalcemia, iritis, arthritis, central
nervous system involvement, granulomatosis hepatitis, cutaneous lesions, as
well as symptomatic pulmonary lesions. Long-term therapy is often required
over months to years. Immunosuppressive therapy with medications such as
cyclosporin has been tried after corticosteroid therapy fails.
Cyclophosphamide (choice A) is an alkylating agent related to the nitrogen
mustards. It is indicated primarily for malignant diseases; however, it has
been used in treatment of severely resistant cases of rheumatoid arthritis,
but not sarcoidosis.
Methotrexate (choice B) is an agent used to treat severe, active, classical or
definite rheumatoid arthritis in adults who have an insufficient response with
conventional therapies. It is also used as an antineoplastic agent for a
number of cancers, but has not been shown to provide any benefit to a
patient with sarcoidosis.
Naproxen (choice C) is a nonsteroidal anti-inflammatory drug (NSAID) that
acts by inhibiting cyclooxygenase, leading to decreased production of
prostaglandins, as well as blocking platelet aggregation. This agent is
indicated for treatment of mild-moderate pain (acute and chronic) caused by
inflammation.
Oxycodone (choice D) is an opioid analgesic that stimulates opioid
receptors. Oxycodone is indicated for relief of moderate to severe pain





An 11-year-old child presents with an abdominal mass. CT scan demonstrates
that the mass is due to massively enlarged retroperitoneal
Iymph nodes. Biopsy of one of the nodes show that it has been effaced by smalI,
round, dark blue cells with scanty cytoplasm, which are
interspersed with isolated larger individual cells with larger regular nuclei and
abundant cytoplasm, forming the pattern shown above. On
immunohistochemical typing, the small cells are positive for CD20.

Explanation - Q: 2.1 Close

The correct answer is A. Burkitt lymphoma is found endemically in Africa,
where the mean age at diagnosis is 7 years. It is also found sporadically
elsewhere, including in the U.S., with a mean age of 11 years. Roughly 2/3 of
cases are male. African cases often involve the jaw, while American cases
more commonly involve the abdomen. Overall, with modern aggressive
therapeutic regimens, Burkitt lymphoma now has a survival rate of about
60%, with cases with limited disease having a survival rate of 90%. The
uncommon adult cases tend to present at a later stage than childhood cases,
and have a correspondingly poorer prognosis.
Hodgkin lymphoma (choice B) has clearly larger cells with very abnormal-
appearing nuclei in a setting of lymphocytes, neutrophils, and eosinophils.
Neuroblastoma (a childhood epithelial tumor, choice C), Szary syndrome (T
cell lymphoma with cutaneous manifestations, choice D), and Wilms tumor
(a childhood tumor with epithelial and stromal components, choice E) would
all have tumor cells that express CD20, but they would not have the starry
sky appearance.


Question 2 of 5
The darkly staining cells between the clear areas in the photomicrograph shown
above are mostly which of the following?
/ A. Benign B Iymphocytes
/ B. Benign epithelial cells
/ C. Malignant B Iymphocytes
/ D. Malignant epithelial cells
/ E. Malignant T Iymphocytes

Explanation - Q: 2.2 Close

The correct answer is C. The small cells in the background of the starry sky
pattern are malignant B lymphocytes, and mark with the B-cell marker CD20.
These are the true tumor cells. Epithelial cells and T lymphocytes do not
mark with CD20.



Question 3 of 5
The larger cells that make the clear areas in the pattern are which of the
following?
/ A. Benign B cells
/ B. Benign macrophages
/ C. Benign T cells
/ D. Malignant macrophages
/ E. Malignant T cells

Explanation - Q: 2.3 Close

The correct answer is B. The larger cells are benign macrophages, which
"eat" the debris produced by the rapidly dividing malignant B cells. The other
answers are distracters.





Question 4 of 5
Which of the following translocations is most often associated with this
neoplasm?
/ A. t(2;8)
/ B. t(2;22)
/ C. t(8;14)
/ D. t(8,22)
/ E. t(14,22)




Question 5 of 5
Infection with which of the following viruses is associated with this disease?
/ A. Cytomegalovirus
/ B. Epstein-Barr virus
/ C. Herpes simplex l
/ D. Herpes simplex ll
/ E. Human papilloma virus

Explanation - Q: 2.5 Close

The correct answer is B. Epstein-Barr virus is closely linked to the African
form of Burkitt lymphoma and portions of Epstein-Barr viral DNA have also
been found in some sporadic cases. The precise role played by the infection
is unclear.
Human papilloma virus (choice E) is linked to warts, condyloma, cervical
cancer, and some forms of penile cancer. The other viruses listed do not
have tumor associations you need to remember.



A 25-year-old woman consults a physician because she has noticed swelling in
one armpit. The woman has also been experiencing fever and
malaise. Physical examination demonstrates lymphadenopathy in the left axilla.
No other lymphadenopathy is noted. The only other notable
physical finding is a healing scratch with a 4-mm diameter crusted papule on the
same arm, which the patient attributes to her cat and says is
one week old. Lymph node biopsy from the arm shows that the node contains
granulomas, "stellate abscesses," and nonspecific inflammatory
infiltrates. Follow-up silver stain of the histologic sections shows the presence of
bacteria.




Question 1 of 5

Which of the following is the most likely diagnosis?
/ A. Burkitt lymphoma
/ B. Cat-scratch disease
/ C. Hodgkin lymphoma
/ D. Sarcoidosis
/ E. Tuberculosis

Explanation - Q: 3.1 Close

The correct answer is B. This patient has cat-scratch disease. Once the
diagnosis is suspected, it can be relatively easily confirmed by the presence
of the bacteria seen on silver stain of the lymph node. However, you should
be aware that in real life, the diagnosis of cat-scratch is often delayed or
even completely missed, because the trivial scratch produced by the cat may
be unnoticed by the patient or doctor and may have healed by the time of
examination. Additionally, the lymphadenopathy with complex histologic
pattern may suggest either lymphoma (choices A and C, note that
misdiagnosis as Hodgkin disease would be more likely than misdiagnosis as
Burkitt lymphoma, which has a very different histologic picture) or more
conventional granulomatous disease (choices D and E). Features
suggestive (but not diagnostic) of cat-scratch disease on initial histologic
examination of the lymph node include granulomas, "stellate abscesses"
(star-shaped collections of neutrophils with tissue destruction), and
nonspecific inflammatory infiltrates.



Question 2 of 5

The causative organism of this patient's condition is which of the following?
. `



Explanation - Q: 3.2 Close

The correct answer is A. The causative organism of catscratch disease is
Bartonella (formerly Rochalimaea) henselae, which was identified in 1992,
based on serologic responses of the patients. The organism is sensitive to
many antibiotics, including erythromycin, doxycycline, rifampin, gentamicin,
cefoxitin, mezlocillin, and trimethoprim-sulfamethoxazole.
Borrelia burgdorferi(choice B) causes Lyme disease.
Pneumocystis carinii(choice C) causes pneumonia in AIDS and other
immunosuppressed patients.
Treponema pallidum(choice D) causes syphilis.
Yersinia pestis(choice E) causes bubonic plague.




Explanation - Q: 3.3 Close

The correct answer is C. The organism is apparently transferred between
cats via fleas. This route of exposure does not appear to be important in cat-
to-human transmission. The other routes of exposure listed in the choices do
not appear to be common in cat-to-cat transmission.



Question 4 of 5
The organism that causes this disease is most closely related to one that causes
which of the following in AIDS patients?
/ A. Bacillary angiomatosis
/ B. Hairy leukoplakia
/ C. Infectious mononucleosis
/ D. Kaposi sarcoma
/ E. Thrush

Explanation - Q: 3.4 Close

The correct answer is A. Bacillary angiomatosis causes reddish, berry-like
lesions of the skin of AIDS patients that can mimic the lesions of Kaposi
sarcoma. The association between Bartonella and bacillary angiomatosis in
AIDS patients was actually known before its association with cat-scratch
disease. In fact, it was this association that initially led researchers to include
Bartonella among those whose serology was tested in the cat-scratch
patients. We presently call the organism that causes bacillary angiomatosis
(and also bacillary peliosis hepatitis and trench fever) Bartonella quintana.
Hairy leukoplakia (choice B) is an oral cavity lesion caused by Epstein-Barr
virus in AIDS patients.
Epstein-Barr virus also causes infectious mononucleosis (choice C) and
some cases of Burkitt's lymphoma in non-immunosuppressed individuals.
Kaposi sarcoma (choice D) is caused by human herpes virus 8.
Thrush (choice E) is caused by Candida.


Question 5 of 5
In addition to the biopsy studies described, which of the following tests can assist
in the diagnosis of this patient's disease?
/ A. Growth on chocolate agar
/ B. Indirect fluorescent antibody
/ C. Lactose fermentation
/ D. Kveim test
/ E. Stool for ova and parasites

Explanation - Q: 3.5 Close

The correct answer is B. An indirect fluorescent antibody test performed on
serum is now available for identifying Bartonella. This test is 84-88%
sensitive and 94-96% specific, but shows cross-reactivity between B.
henselae and B. quintana. This is not a problem clinically, since the diseases
produced by these strains are quite different. Low positive results usually
indicate past exposure, while high positive results suggest recent exposure.
Some research work has also been done with PCR-based diagnosis, but this
is not yet readily available. To date, clinical laboratories cannot routinely
culture Bartonella species (it has been done in research settings), so growth
on chocolate agar (used for Haemophilus, choice A) and lactose
fermentation (choice C) (commonly used in evaluation of enteric rods, are
not appropriate tests.
The Kveim test (choice D) is used in sarcoidosis.
The organism is a bacterium rather than a larger parasite, so examination of
stool for ova and parasites (choice E) is not relevant.


A 17-year-old boy is taken by his mother to his pediatrician because he has had
chronic fatigue of two months duration, which has been
accompanied by intermittent low-grade fevers. On questioning, the boy reports
that he has had one episode of intense itchiness and two
episodes of waking up with night sweats during this period. Physical examination
is remarkable only for a fever of 37.6 C (99.7 F). A chest x-
ray film shows marked mediastinal widening without masses or other lesions in
the lung fields. BIood studies show moderate
polymorphonuclear leukocytosis, Iymphocytopenia, and eosinophilia.

Question 1 of 6



The patient is sent for mediastinoscopy with biopsy. During this procedure, a
small incision is made immediately above the sternum and a
rigid tube is inserted into the chest cavity for visualization of the mediastinum by
video camera. Which of the following structures would be most
at risk for trauma during this procedure?
/ A. Accessory nerve
/ B. Digastric muscle
/ C. Thymus
/ D. Thyroid gland
/ E. Trapezius muscle

Explanation - Q: 4.1 Close

The correct answer is C. The thymus is the only organ on the list that is
likely to be traumatized. The location of the incision immediately above the
sternum is specifically chosen to reduce the chance of injury to the thyroid
(choice D), which lies higher in the neck.
The accessory nerve (choice A) runs in the lateral neck toward the shoulder.
The digastric muscle (choice B) is located more superiorly in the neck and
extends from the temporal bone to the hyoid and then to the mandible.
The trapezius muscle (choice E) is located more laterally in the neck and
extends from the base of the skull to the shoulder and back.


Question 2 of 6
Several biopsies of lymph nodes are taken and sent for pathologic examination.
In these biopsies, the normal lymphoid tissue has been
replaced by a mixed population of morphologically normal cells including
histiocytes, Iymphocytes, monocytes, plasma cells, and eosinophils.
Admixed with these cells are infrequent numbers of large binucleate cells with
large prominent eosinophilic nucleoli (see above).
Immunophenotyping demonstrates that these cells are positive for CD 15 and CD
30. The CD15 and CD 30 positive cells are most likely
which of the following?
/ A. Aschoff cells
/ B. Langhans giant cells
/ C. Megakaryocytes
/ D. Reed-Sternberg cells
/ E. Strap cells

Explanation - Q: 4.2 Close

The correct answer is D. These cells are Reed-Sternberg cells, and you will
very likely see them either in a photograph or in a verbal description on the
USMLE. Reed-Sternberg (RS) cells are typically binucleate (or contain
bilobed nuclei) with prominent "owl's eyes" nucleoli. These cells must be
identified before the diagnosis of Hodgkin disease can be made. The RS cell
and RS variants express CD30, a lymphoid activation antigen. CD15, a
granulocyte antigen, is also uniformly positive in RS cells.
Aschoff cells (choice A) are modified histiocytes with a ribbon-like pattern of
chromatin dispersal, which are seen in rheumatic fever.
Langhans giant cells (choice B) are cells with large numbers of nuclei seen
in granulomatous diseases such as tuberculosis.
Megakaryocytes (choice C) contain many nuclei and are usually found in
bone marrow.
Strap cells (choice E) are an elongated form of immature myocyte seen in
myosarcoma.


Question 3 of 6
Which of the following is the most likely diagnosis?
/ A. Chronic myelogenous leukemia
/ B. Hodgkin lymphoma
/ C. Non-Hodgkin lymphoma
/ D. Sarcoidosis
/ E. Tuberculosis



Explanation - Q: 4.3 Close

The correct answer is B. Reed-Sternberg cells seen in a background of
mixed normal cells, including eosinophils, indicate Hodgkin disease. There
are around 7000 new cases of Hodgkin disease annually. The disease has a
bimodal age distribution with peaks in adolescence to young adulthood and
old age. Histological subtypes of Hodgkin disease include lymphocyte
predominant, nodular sclerosis, mixed cellularity, and lymphocyte-depleted.
Although cells resembling Reed-Sternberg cells on hematoxylin and eosin
stained tissues can be uncommonly seen in a wide variety of settings
(including some non-Hodgkin lymphomas and some tumors), for the USMLE,
you can probably safely assume that any time large, binucleate cells with
prominent nucleoli are described, they are Reed-Sternberg cells and the
disease is Hodgkin disease.
Chronic myelogenous leukemia (choice A) does not usually involve lymph
nodes and does not have Reed-Sternberg-like cells.
In real life (as indicated above) uncommon types of non Hodgkin's lymphoma
(choice C), notably T-cell rich B-cell lymphoma, may closely mimic Hodgkin
disease histologically, but if you see a description on a test similar to the one
in the question stem, pick Hodgkin disease.
Sarcoidosis (choice D) and tuberculosis (choice E) usually have some lung
involvement and their histological description would refer to granulomas.


Question 4 of 6
As part of the staging of the patient's disease, a bone marrow biopsy is
performed. Which of the following bones would be an appropriate site
for obtaining the biopsy?
/ A. Cervical vertebrae
/ B. Femur
/ C. Humerus
/ D. IIiac bone
/ E. Lumbar vertebrae


Explanation - Q: 4.4 Close

The correct answer is D. The iliac bone is used most commonly for bone
marrow biopsy. The sternum is a less common substitute site, which is used
if iliac biopsy fails. The other sites are not usually used for bone marrow
biopsies unless sampling of a radiologically demonstrated lesion is
attempted.




Question 5 of 6
Biopsy studies demonstrate that the patient's disease additionally involves bone
marrow, Iiver, and periaortic nodes in the lower abdomen.
Using the Ann Arbor classification, he has which of the following disease stages?
/ A. IA
/ B. IIB
/ C. IIIA2
/ D. IIIB1
/ E. IVB

Explanation - Q: 4.5 Close

The correct answer is E. In the Ann Arbor staging of Hodgkin and non
Hodgkin lymphoma, stage I involves one lymph node only; stage II involves
two or more lymph nodes on the same side of the diaphragm, stage III
involves the lymph nodes, spleen, or both, on both sides of the diaphragm
(with subclass 1 above the renal vessels and subclass 2 in the lower
abdomen), and stage IV shows extranodal involvement in sites such as bone
marrow, lung, and liver. "A" is added if the patient is free of systemic
symptoms and "B" is added if he has systemic symptoms such as weight
loss, intense pruritus, fever, and night sweats. "B" symptoms are more
common in higher stage disease.
Radiotherapy alone to an extended field can be used to treat some stage I
and IIA disease; combined chemotherapy and radiotherapy or chemotherapy
alone are usually used for patients with higher stages. Even in advanced
disease, complete remission can be obtained in up to 70-80% of patients,
and more than half of those who survive are disease-free at 10 to 15 years.




Question 6 of 6

Based on the staging of the patient's condition, the most appropriate and best
tolerated (Ieast toxic) treatment would be which of the following?
/ A. Cisplatin only
/ B. Methotrexate only
/ C. Radiation only
/ D. Doxorubicin, bleomycin, vincristine, dacarbazine (ABVD)
/ E. Mechlorethamine, vincristine, procarbazine, prednisone (MOPP)

Explanation - Q: 4.6 Close

The correct answer is D. The patient was diagnosed with Stage IVB
Hodgkin disease. Radiation therapy (choice C) is used as a single mode of
therapy in the treatment of "low-risk" patients with stages IA and IIA disease.
The addition of limited chemotherapy for some patients receiving radiation is
also showing some promise. Most patients with Hodgkin disease, including
those with advanced stages of the disease, are best treated with combination
chemotherapy using doxorubicin, bleomycin, vincristine and dacarbazine
(ABVD) (choice D). This particular combination therapy has been proven to
be the more effective and less toxic than traditional therapies, such as
mechlorethamine, vincristine, procarbazine and prednisone (MOPP) (choice
E). Specifically, ABVD causes less secondary leukemia and less
reproductive sterility than MOPP. Several other shorter and more intensive
regimens have shown great promise in the treatment of Hodgkin disease.
Cisplatin (choice A) is an antineoplastic agent that acts by binding to DNA,
producing intrastrand cross-links, and forming DNA adducts. The result of the
cross-linkage of DNA strands blocks the synthesis of DNA, RNA, and protein.
This agent is commonly used in the treatment of solid tumors, such as
metastatic testicular carcinoma, ovarian carcinoma, as well as bladder
carcinoma. Aside from immunosuppression, the most common side effect of
this agent is severe persistent emesis.
Methotrexate (choice B) is an agent that competitively inhibits dihydrofolic
acid reductase and is indicated for the treatment of rheumatoid arthritis in
adults who have an insufficient response with conventional therapies, and for
antineoplastic therapeutic treatment regimens e.g., for ALL, cancers of the
head, neck, and breast. It is contraindicated in pregnancy, nursing mothers,
hepatic insufficiency, as well as in patients with AIDS, blood dyscrasias,
bone marrow hypoplasia, leukopenia, and severe anemia.


An 18-year-old college student consults her university health clinic because she
has felt ill for the preceding month. She had developed what
initially appeared to be a bad cold with fever, severe pharyngitis, and a transient
rash, but she had not recovered promptly from the illness. On
physical examination she is noted to have generalized lymphadenopathy with
hepatosplenomegaly. A peripheral blood smear shows
numerous atypical lymphocytes. A Monospot test is positive.



Question 1 of 5
Which of the following is the most likely diagnosis?
/ A. Cat-scratch disease
/ B. Chickenpox
/ C. German measles
/ D. Infectious mononucleosis
/ E. Measles


Explanation - Q: 5.1 Close

The correct answer is D. This patient has infectious mononucleosis. The
presentation illustrated is typical. Patients may also have nausea and
anorexia without vomiting. In early cases, a helpful diagnostic clue may be
the presence of uvular edema, which is uncommon in most types of
pharyngitis. You should be aware that in real life, most cases of infectious
mononucleosis are probably missed because they are milder and clear more
readily. Very severe cases can also exist, which may even rarely cause
death, usually secondary to hepatic necrosis or splenic rupture. You should
also be aware that both clinically, and on the peripheral blood smear,
infectious mononucleosis may closely resemble lymphoma (the atypical
lymphocytes look very odd), and misdiagnosis is always a potential problem
and has gone in both directions (patients with infectious mononucleosis
subjected to chemotherapy, and patients with lymphoma neglected for
months because someone thought they just had infectious mononucleosis).
Treatment is usually supportive as no antiviral agent is presently effective
against the causative virus.
Cat-scratch disease (choice A) can cause lymphadenopathy and a transient
early rash, but would not have a positive Monospot test.
Measles (choice E), German measles (choice C), and chickenpox (choice
B) can also cause febrile illness with skin manifestations in a teenager, but
none of these diseases would last as long as in this patient. Also, these
diseases are more likely to be associated with a prominent rash.










Question 3 of 5
The organism described is in the same family as which of the following?
/ A. California encephalitis virus
/ B. Cytomegalovirus
/ C. Hanta virus
/ D. Parainfluenza virus
/ E. Rabies virus

Explanation - Q: 5.3 Close

The correct answer is B. The Herpes virus family is composed of double-
stranded DNA viruses, and contains a number of medically significant
viruses, including herpes simplex I and II, varicella-zoster, cytomegalovirus,
Epstein-Barr virus, and herpes virus 8, which causes Kaposi sarcoma. The
other viruses listed are all RNA viruses.


Question 4 of 5
Which of the following is also thought to be related to infection with the most
likely pathogen?
/ A. Burkitt lymphoma
/ B. Cervical carcinoma
/ C. Condyloma
/ D. Hodgkin lymphoma
/ E. Oat cell carcinoma of the lung
Explanation - Q: 5.4 Close

The correct answer is A. There is a tie between Epstein-Barr virus infection
and Burkitt lymphoma. The tie is most apparent in the epidemic African form
of the disease, which tends to involve the jaw, but sporadic American cases
(tending to involve the abdomen) have also been found by PCR techniques
to often contain DNA from Epstein-Barr virus. You should also be aware that
patients with old exposures to Epstein-Barr virus may harbor small amounts
of the virus (possibly incorporated into their genome). If these individuals
then go on to develop AIDS or have an organ transplant, they become
vulnerable to lymphoproliferative disorders that may then progress to frank
lymphoma (not necessarily the Burkitt type).
Condyloma (choice C) and cervical cancer (choice B) are linked to human
papilloma virus infection.
Hodgkin lymphoma (choice D) and oat cell carcinoma (choice E) have
never been convincingly linked to a specific viral infection.





Question 5 of 5

A patient with a similar presentation to this patient's, but with a negative
Monospot test, would be most likely to be infected with which of the
following?





Explanation - Q: 5.5 Close

The correct answer is C. "Heterophile-negative" infectious mononucleosis,
in which the Monospot test is negative, is usually caused by
cytomegalovirus.
Candida albicans(choice A) can cause skin infection, thrush, vaginitis, and
disseminated infections.
The severe pharyngitis of some cases of infectious mononucleosis can
produce a "pseudomembrane" in the throat that is similar to that of
diphtheria, caused by Corynebacterium diphtheriae(choice B). However,
diphtheria is now rare in the United States, and does not usually cause the
degree of lymphadenopathy seen in this patient.
Herpes simplex (choice D) can cause vesicular disease of the skin and
mucous membranes.
Yersinia enterocolitis (choice E) can infect the lymph nodes of the intestinal
mesentery.




An 18-month-old child is referred to a children's hospital because of 8 days of
high sustained fever. The fever began abruptly, and has not
responded to antibiotic therapy. On physical examination, the child is noted to be
very irritable. The cervical lymph nodes are markedly
enlarged. A polymorphous rash with erythema of the extremities is seen, as is
desquamation of the fingers and toes. The lips are swollen and
fissured, and within the mouth, the tongue has a strawberry appearance and the
pharynx appears edematous. Bilateral conjunctivitis is also
noted.
Question 1 of 4
Which of the following is the most likely diagnosis?
/ A. Aran-Duchenne disease
/ B. Bowen disease
/ C. Charcot-Marie-Tooth disease
/ D. Christmas disease
/ E. Kawasaki disease

Explanation - Q: 6.1 Close

The correct answer is E. This child is showing the classic features of
Kawasaki disease, a febrile illness of childhood. It is a systemic vasculitis of
unknown etiology that affects small- and medium-sized blood vessels of the
body. To make the diagnosis of Kawasaki disease, the fever must have been
present at least 5 days, and at least 4 of the following additional findings
must be seen: changes in the extremities, bilateral conjunctivitis,
polymorphous rash, cervical lymphadenopathy, and changes in the lips and
oral cavity.
Aran-Duchenne disease (choice A) is amyotrophic lateral sclerosis.
Bowen's disease (choice B) is an intraepidermal carcinoma.
Charcot-Marie-Tooth disease (choice C) is peroneal muscular atrophy.
Christmas disease (choice D) is hemophilia B.


Question 2 of 4
In the United States, this child's condition has the highest incidence rate in which
of the following populations?
/ A. African-Americans
/ B. Japanese-Americans
/ C. Jewish-Americans
/ D. Native Americans
/ E. Swedish-Americans

Explanation - Q: 6.2 Close

The correct answer is B. Kawasaki disease has an increased incidence in
Japanese Americans, and was, in fact, initially described in Japan by
Tomisaku Kawasaki in 1967. The other answers are distracters.



Question 3 of 4
Approximately how many children in the United States are hospitalized for this
patient's condition yearly?
/ A. 30
/ B. 300
/ C. 3000
/ D. 30,000
/ E. 300,000

Explanation - Q: 6.3 Close

The correct answer is C. Approximately 3000 children are hospitalized
annually in the United States for Kawasaki disease. Epidemics appear to
occur at roughly 3-year intervals primarily in the late winter and spring. The
peak incidence of age in the United States is 18-24 months, with a male-to-
female ratio of 1.5:1. Modern thinking suggests (without proof) that the
condition may be due to an initial infectious process that then develops an
autoimmune component, which does much of the actual damage.



Question 4 of 4

This child's condition tends to predispose for which of the following
complications?
/ A. Abdominal aortic aneurysm
/ B. Berry aneurysms of the circle of Willis
/ C. Coronary artery aneurysm
/ D. Cystic medial necrosis
/ E. Dissecting aortic aneurysm


Explanation - Q: 6.4 Close

The correct answer is C. Kawasaki disease can involve the small arteries
supplying the heart, and the damage produced can induce coronary artery
aneurysm formation. The degree to which the aneurysms develop (and
become a potential site of thrombosis with risk of myocardial infarction and
death) determines the degree of long-term disability experienced by the
children. Additionally, early in the disease, myocarditis, congestive heart
failure, pericarditis with pericardial effusion, valvular insufficiency, and
dysrhythmias may occur. Modern therapy to try to limit the development of
complications includes gamma globulin and aspirin as anti-inflammatory
agents and long-term anticoagulation to prevent the development of
thromboses in any damaged coronary arteries. Transfer to a tertiary care
center is recommended for any children that develop cardiovascular
complications. Kawasaki disease has now surpassed rheumatic fever as the
leading cause of acquired heart disease in the United States among children
younger than 5 years.
The aorta (choices A, D, and E) and the vessels of the brain (choice B) are
not particularly vulnerable in this disease.

A 38-year-old woman complains to her physician of chronic pelvic pain that is
much worse during her menstrual periods.
Pelvic examination demonstrates an enlarged uterus with multiple palpable
masses. UItrasound shows that a large number of balI-Iike masses up to 6 cm
diameter are present in the uterus.

Which of the following is the most likely cause of the balI-Iike masses in her
uterus?

/ A. Cervical adenocarcinoma
/ B. Cervical squamous cell carcinoma
C. Leiomyoma
/ D. Leiomyosarcoma
/ E. Uterine adenocarcinoma

The correct answer is C. Leiomyomas are benign ball-like masses that
commonly form in the uterus.
Cervical and uterine carcinomas (choices A, B, and E) tend to initially form a
fairly superficial layer of cancer on the surface of the endocervix or uterine cavity,
which may later become deeply infiltrative. Usually, no ball-like mass is
produced.
Leiomyosarcomas (choice D) are very rare tumors, which form masses that are
usually less regular in geometry than leiomyomas.

Most cases of this type of tumor are found at which of the following sites?
/ A. Intramurally in the cervix of the uterus
/ B. Intramurally in the fundus or body of the uterus
/ C. Submucosally in the cervix of the uterus
/ D. Submucosally in the fundus or body of the uterus
/ E. Subserosally in the fundus or body of the uterus

The correct answer is B. The vast majority of leiomyomas are found in the
uterine fundus or body; with only 3% being found in the cervix (choices A and
C). Of the leiomyomas in the fundus or body, 95% are intramural, with the
remainder being located subserosally (choice E) or submucosally (choice D).

What percentage of women over 30 years old who still have a uterus are thought
to have these tumors?
/ A. 0-5%
/ B. 10-15%
/ C. 20-50%
/ D. 60-80%
/ E. 95-100%
The correct answer is C. Leiomyomas are the most frequently diagnosed
gynecologic tumor. It is thought that 20-50% of the population described in the
question has leiomyomas, although the tumors do not cause clinical problems in
many of these individuals. Problems that can occur that are related to
leiomyomas include menorrhagia, abdominal cramping, symptoms related to
pressure on the bladder or colon, rare cases of secondary polycythemia, and
infertility and pregnancy complications. The pregnancy complications are of
particular concern, and may include spontaneous abortion, intrauterine growth
retardation, preterm labor, uterine dyskinesia during labor, obstruction of the birth
canal, and postpartum hemorrhage. Submucosal leiomyomas and very large
intramural leiomyomas are the ones that are most likely to interfere with a
pregnancy. Historically, leiomyosarcomas were thought to arise from malignant
degeneration of a leiomyoma, but modern thinking among pathologists suggests
that most leiomyosarcomas are actually new tumors, that may, by coincidence,
occur in a uterus that has leiomyomas in it.


The woman undergoes a hysterectomy, and pathologic examination
demonstrates a uterus weighing nearly 1 kg, nearly 10 times the expected
weight. The uterus is massively distorted by over 20 balI-Iike tumors. Histological
sections of these tumors show whorls of spindle shaped cells. These are most
likely which of the following?
/ A. Benign fibroblasts
/ B. Benign smooth muscle cells
/ C. Malignant fibroblasts
/ D. Malignant nerve cells
/ E. Malignant smooth muscle cells


The correct answer is B. Surgery is only indicated in patients with leiomyomas
if they are causing clinical problems. While many smaller uteruses also have
leiomyomas, uteruses as large as this woman's are not uncommon. The tumors
are composed of spindle shaped cells, and may also show areas of
degeneration, hemorrhage, or calcification, if the tumor size has compromised
blood flow to the tumor tissues. While the cell of origin of spindle-shaped cells
can often be a problem in pathology, in the uterus, the situation is usually much
simpler, because the vast majority of tumors with spindle-shaped cells are
leiomyomas, composed of benign smooth muscle cells.
Fibromas (choice A), fibrosarcomas (choice C), and neurofibrosarcomas
(choice D) are very rare in the uterus.
Malignant smooth muscle cells (choice E) occur in the much rarer
leiomyosarcomas.


These tumors appear to be under some degree of hormonal controI, and typically
have receptors for which of the following?
/ A. Androgen only
/ B. Estrogen only
/ C. Progestin only
/ D. Both estrogen and progestin
/ E. Both progestin and androgen
/ F. Estrogen, progesterone, and androgen


The correct answer is D. Leiomyomas are hormonally sensitive tumors that
have both estrogen and progestin (but not androgen) receptors. They may
change size during pregnancy (usually enlarge, but sometimes shrink), often
shrink with menopause, and may regrow with hormonal therapy after
menopause.


A 27-year-old woman consults a gynecologist because of chronic pelvic pain,
which has been severe enough on several occasions to cause her to go a nearby
emergency department. On both previous occasions, the emergency department
physician suggested that she had had a "ruptured ovarian cyst," but no
diagnostic studies had been performed. The woman seems nearly desperate
during the examination, and complains that "no one seems to be able to figure
out what is wrong with me." She has had painful menstrual periods since her
teenage years, and does think that her now chronic pelvic pain is worst during
her periods. The pain is often accompanied by diarrhea, and her internist thought
that she might have irritable bowel syndrome. Gynecologic examination is
notable for some fullness in both adnexa.


Laparoscopy demonstrates multiple chocolate brown-colored 2 mm to 2 cm
lesions on her ovaries, the serosal surface of her uterine fundus, and on nearby
bowel loops. These are most likely due to which of the following?
/ A. Endometriosis
/ B. Endosalpingosis
/ C. Leiomyomas
/ D. Metastatic ovarian cancer
/ E. Metastatic uterine cancer


The correct answer is A. Chocolate brown discoloration suggests focal
hemorrhage, and the most likely diagnosis is endometriosis. Endometriosis
affects more than 5 million women in the United States, and can be a cause of
pelvic pain, painful intercourse, infertility, fatigue, painful urination or bowel
movements during periods, and diarrhea or constipation. Many patients have
complex, frustrating medical histories before finally being diagnosed.
Laparoscopy is the most reliable method of determining the presence of
endometriosis.
Endosalpingosis (choice B), or implants of tissue resembling the lining of
fallopian tubes, is much less common and less likely to bleed than is
endometriosis.
Leiomyomas (choice C) form ball-like masses in the uterus.
Metastatic ovarian (choice D) or uterine (choice E) cancer would be unlikely in a
woman this young.

Pathologic examination of a biopsy of these lesions would most likely show which
of the following?
/ A. Benign stroma, benign glands, and hemosiderin deposition
/ B. Malignant glands, reactive stroma, and hemosiderin deposition
/ C. Malignant stroma, no glands, and hemosiderin deposition
/ D. Whorled masses of benign smooth muscle cells
/ E. Whorled masses of malignant smooth muscle cells


The correct answer is A. Foci of endometriosis contain tissue resembling the
endometrial lining, usually with benign glands, benign stroma, and hemosiderin
deposition secondary to "menstruation" with hemorrhage in the lesion. In some
cases, only typical stroma resembling that of endometrium and hemosiderin is
seen. Common sites of implantation include the surfaces of the ovaries, fallopian
tubes, uterine corpus, ligaments of the uterus, and the linings of spaces between
the uterus and bladder and the uterus and rectum. Less common sites of
implantation include the bladder, bowel, vagina, cervix, vulva, abdominal scars,
and even lung, extremities, and other locations. A variety of mechanisms for
development of endometriosis have been hypothesized, including retrograde flow
of menstrual material into the pelvis, spreading through lymphatics or blood,
genetic predisposition, metaplasia of other tissues to tissue resembling
endometrium, and maturation of rests of primitive endometrium at abnormal
sites. Possibly, multiple mechanisms may have the same end result in different
patients. Additionally, there appears to be a developing link between
endometriosis and allergies, chemical sensitivities, frequent yeast infections, and
autoimmune disorders (lupus, Hashimoto thyroiditis, eczema, asthma,
fibromyalgia).
Malignant glands (choice B) suggest adenocarcinoma.
Malignant stroma (choice C) suggests sarcoma, of which leiomyosarcoma
(described in choice E) is one type.
Leiomyomas are whorled masses of benign smooth muscle cells (choice D).
The patient also complains of having been trying to unsuccessfully conceive a
child for several years. This problem would be most likely related to which of the
following in her case?
/ A. Failure of an egg to mature during her menstrual cycle
/ B. Failure of her ovary to secrete estrogen
/ C. Failure of her ovary to secrete progesterone
/ D. Failure of her pituitary to secrete gonadotropic hormones
/ E. Scarring of both adnexa


The correct answer is E. The lesions of endometriosis are very irritating to
adjacent structures, which trigger both the pain experienced and a fibrotic
response to the hemorrhage. This scarring can completely entrap the fallopian
tubes and ovaries, and either prevent pregnancy, or, if sperm are able to pass
through the damaged fallopian tubes, predispose for ectopic pregnancy.
The problems are not related to either pituitary (choice D) or ovarian (choices A,
B, and C) functioning per se.
The woman decides that she prefers an attempt at medical control before
deciding whether or not to have surgery. Which of the following would be the
most appropriate pharmacotherapy?
/ A. AIIopurinol
/ B. Danazol
/ C. Diazepam
/ D. Digoxin
/ E. Tetracycline

The correct answer is B. Depending upon the severity of the disease, therapy
for endometriosis can involve pain medication, pharmacologic manipulation of
the implants, surgical or laser ablation of individual implants, or surgical removal
of affected pelvic organs. Drug therapy for endometriosis can include
combination estrogen/progestin oral contraceptives, progestins, steroids such as
danazol, or GnRH agonists such as nafarelin and leuprolide. Danazol, a partial
agonist at progestin, androgen, and glucocorticoid receptors, suppresses ovarian
function. Danazol may be effective in control of endometriosis, but many patients
have problems with side effects that may include weight gain, acne, voice
changes, hirsutism, emotional instability, and liver dysfunction.
Allopurinol (choice A) is used in the treatment of gout.
Diazepam (choice C) is an anti-anxiety agent (benzodiazepine).
Digoxin (choice D) is a cardiotropic agent.
Tetracycline (choice E) is an antibiotic.
A 31-year-old woman comes to the emergency department because of
abdominal pain and vaginal spotting. She states that the pain began 2 days ago
and has been worsening since. The spotting occurred this morning. Her last
menstrual period was 6 weeks ago. She has no medical problems. She takes no
medications and has no known drug allergies. Her temperature is 37 C (98.6 F),
blood pressure is 90/50 mm Hg, pulse is 110/minute, and respirations are
14/minute.
Abdominal examination reveals significant lower abdominal tenderness.
Speculum examination shows scant blood in the vagina with a closed cervical os.
Bimanual examination demonstrates significant left adnexal tenderness. Urine
hCG is positive. Serum hCG is 5,000 mIU/mL. Pelvic ultrasound shows a normal
uterus with a left adnexal mass surrounded by free fluid.


Which of the following is the most likely diagnosis?
/ A. Appendicitis
/ B. Ectopic pregnancy
/ C. Ovarian cancer
/ D. Pelvic inflammatory disease
/ E. Tubo-ovarian abscess


The correct answer is B. This patient has a presentation that is most consistent
with ectopic pregnancy. The most common presenting symptoms of patients with
ectopic pregnancy are abdominal/pelvic pain and vaginal bleeding. However,
many patients have ectopic pregnancies and are asymptomatic until they are
discovered incidentally on ultrasound or until they rupture. Examination of a
patient with ectopic pregnancy often demonstrates tenderness on the side of the
ectopic pregnancy. Occasionally, a mass can be appreciated on that side. The
keys to the diagnosis of ectopic pregnancy are the serum hCG value and the
ultrasound. This patient has a serum hCG level of 5,000 mIU/mL. At this hCG
level, a pregnancy would be visualized if it were, in fact, in the uterus. This
patient has an ultrasound showing nothing in the uterus with a mass in the left
adnexa. This constellation of symptoms and findings are most consistent with the
diagnosis of ectopic pregnancy.
Appendicitis (choice A) can also present with abdominal pain. However, the pain
and findings are most often on the right side of the patient's abdomen and this
patient has a mass and tenderness on the left side. Furthermore, vaginal
bleeding and elevated hCG levels are not associated with appendicitis.
Occasionally, a patient with ovarian cancer (choice C) will have an hCG-
secreting tumor. However, when a woman of childbearing age presents with
abdominal pain and vaginal spotting and has a positive hCG value, the diagnosis
is almost always pregnancy-related (either ectopic pregnancy, intrauterine
pregnancy, or spontaneous abortion) and not ovarian cancer.
Pelvic inflammatory disease (choice D) is rare, but does sometime occur during
pregnancy. This patient, however, has findings more consistent with ectopic
pregnancy than pelvic inflammatory disease.
Tubo-ovarian abscess (choice E) can often be seen on ultrasound as an adnexal
mass. However, in this patient's case, with her positive hCG value, the adnexal
mass is much more likely to be an ectopic pregnancy.


In most cases, the disease process of this patient affects which of the following
anatomic structures?
/ A. Cervix
/ B. Fallopian tube
/ C. Ovary
/ D. Peritoneal cavity
/ E. Vagina


The correct answer is B. Approximately 98% of the time, ectopic pregnancies
are located in the fallopian tube. It is believed that improper transport of the
fertilized ovum through the fallopian tube is the most common pathophysiology
leading to ectopic pregnancy. Several risk factors are associated with such
improper transport, including pelvic inflammatory disease and prior tubal surgery.
Pelvic inflammatory disease and prior tubal surgery result in intraperitoneal
scarring and adhesions that can alter the finely tuned transport mechanisms in
the fallopian tubes that are responsible for ensuring that the fertilized ovum
makes it to the endometrial cavity.
Occasionally an ectopic pregnancy can be located in the cervix (choice A) but
this is very uncommon. The cervix is one of the three segments of the uterus (the
other two being the body and the fundus). An ectopic pregnancy in this location
can be difficult to address surgically.
Some ectopic pregnancies are located on or near the ovary (choice C). The
ovary lies near the fallopian tube in the broad ligament. Ectopic pregnancies on
the ovary are also very uncommon.
Very rarely an ectopic pregnancy will be located in the peritoneal cavity (choice
D) where it may attach to the surrounding intestines. There are case reports of
some of these pregnancies making it to term, although this is extremely rare.
Vaginal bleeding is common with ectopic pregnancy, as the abnormal pregnancy
is unable to support the lining of the uterus, and bleeding results. An ectopic
pregnancy in the vagina (choice E), however, is not known to occur.

The patient is started on methotrexate. This medication works via which of the
following mechanisms?
/ A. BIocks the conversion of folic acid to tetrahydrofolate
/ B. Disrupts the synthesis of purines
/ C. Functions as an androgen receptor antagonist
/ D. Functions as an estrogen antagonist
/ E. Inhibits xanthine oxidase



The correct answer is A. Methotrexate covalently binds to the enzyme
dihydrofolate reductase. By binding to this enzyme, methotrexate effectively
blocks the conversion of folic acid to tetrahydrofolate. Tetrahydrofolate is an
important intermediary in the synthesis of deoxyribonucleic acid (DNA).
Methotrexate is used to treat certain tumors, including choriocarcinoma, and
some leukemias. It has also been used to treat severe rheumatoid arthritis,
psoriasis, and other skin disorders. Historically, ectopic pregnancy was treated
only with surgery. Increasingly, however, it is now treated with methotrexate
when a patient is deemed to be an appropriate candidate.
Mercaptopurine is used to treat leukemia. It disrupts the synthesis of purines
(choice B), namely adenine and guanine.
Flutamide is one of the medications that functions as an androgen receptor
antagonist (choice C). It has been used in the treatment of prostate cancer.
Bicalutamide is another antiandrogen that has been used to treat prostate
cancer.
Tamoxifen functions as an estrogen antagonist (choice D). It is used to treat and
to prevent breast cancer. It is not used in the treatment of ectopic pregnancy.
Allopurinol inhibits the xanthine oxidase enzyme (choice E). It is used in the
treatment of gout.


Which of the following findings would be a contraindication for methotrexate use
in this patient?
/ A. AIanine aminotransferase = 336 U/L
/ B. BIood type A negative
/ C. Creatinine = 0.7 mg/dL
/ D. HCG = 1,000 mIU/mL
/ E. Progesterone = 5 ng/mL


The correct answer is A. Methotrexate has much potential toxicity and can
cause numerous adverse effects. Nausea and vomiting are common complaints
of patients who are given this medication. Methotrexate can also cause
myelosuppression with a resulting low white blood cell count and/or anemia.
Therefore, a complete blood count should be checked prior to administering this
drug to a patient. The drug is eliminated through the kidneys, and renal damage
can occur. Therefore, renal function should be monitored. Methotrexate is also
known to cause hepatic toxicity. Therefore, liver function tests (LFTs) should be
obtained prior to initiating the medication. If the LFTs are elevated (e.g., an ALT
of 336 U/L, as in the above example) then the medication should not be given.
Also, patients receiving methotrexate treatment should avoid alcohol.
Having an A-negative blood type (choice B) is not a contraindication to receiving
methotrexate. A patient who is Rh negative should receive RhoGAM if she is
bleeding during the pregnancy.
Having a creatinine = 0.7 mg/dL (choice C) is not a contraindication to receiving
methotrexate. This represents normal renal function.
Having an hCG = 1,000 mIU/mL (choice D) would not be a contraindication to
methotrexate use for this patient. There is much controversy regarding the exact
level of hCG that would be a contraindication to methotrexate use for a patient.
Some experts use a level of 6,000 mIU/mL. An hCG = 1,000 mIU/mL is generally
considered to be an acceptable level at which to use methotrexate.
Having a progesterone = 5 ng/mL (choice E) would not be a contraindication to
methotrexate use for this patient. A progesterone value < 5 ng/mL is almost
always associated with a nonviable pregnancy (i.e., an ectopic pregnancy or a
spontaneous abortion).






A 21-year-old woman presents to the emergency department complaining of
pelvic pain, a yellow-green vaginal discharge, and fever, all of which have been
worsening over the last 24 hours. She has no frequency or dysuria. She has no
medical problems. Her past surgical history is significant for a cesarean delivery
2 years ago performed for a nonreassuring fetal heart rate tracing. She takes no
medications and is allergic to sulfa drugs. She is sexually active with 2 male
partners who sometimes use condoms. She works as a medical assistant. Her
temperature is 38.2 C (100.7 F), blood pressure is
100/60 mm Hg, pulse is 110/minute, and respirations are 12/minute. Her
abdominal examination is significant for diffuse tenderness, rebound, and
guarding. Speculum examination demonstrates a copious greenish vaginal
discharge that appears to be coming from the cervix. Pelvic examination is
significant for cervical motion tenderness and adnexal tenderness. Laboratory
evaluation shows:
Urine hCG: negative
Urinalysis: negative
Leukocytes: 15,000/mm3
Hematocrit: 39%
PIatelets: 200,000/mm3


Which of the following is the most likely diagnosis?
/ A. Ectopic pregnancy
/ B. Gonococcal cervicitis
/ C. Pelvic inflammatory disease (PID)
/ D. Spontaneous abortion
/ E. Urinary tract infection (UTI)



The correct answer is C. Pelvic inflammatory disease (PID) is a significant
cause of morbidity among sexually active menstruating women. PID refers to an
upper genital tract infection, especially of the endosalpingeal cells that line the
fallopian tubes. When the infection involves the fallopian tubes only, it is referred
to as a salpingitis. When it involves the ovaries as well, it is then referred to as a
salpingo-oophoritis. The endometrium is also often involved (endometritis). The
peak incidence of the disease is in the 15- to 24-year-old-group. This patient
presents with the typical symptoms of PID, particularly abdominal/pelvic pain.
Fever and vaginal discharge are often present in gonococcal PID, but may be
absent in other forms of PID. The diagnosis of PID is made when the patient has
abdominal tenderness, cervical motion tenderness, and adnexal tenderness plus
a temperature > 38 C (100.4 F), or leukocytosis (>10,000/mm
3
), or laboratory
documentation of chlamydial or gonorrheal infection. Treatment is with
antibiotics.
Ectopic pregnancy (choice A) is ruled out with the negative urine pregnancy test.
An ectopic pregnancy is a pregnancy that is implanted abnormally-- most often in
the fallopian tubes. As a pregnancy, it secretes human chorionic gonadotropin
(hCG), which can be found in the blood or urine. When this is not present,
ectopic pregnancy is ruled out.
Gonococcal cervicitis (choice B) presents with findings localized to the cervix.
This patient has findings that go beyond a cervicitis. Given her abdominal
tenderness with rebound, cervical motion tenderness, and adnexal tenderness,
she is manifesting involvement of the fallopian tubes and peritoneum. While the
gonococcus may be the offending organism in this case, this patient has more
than a gonococcal cervicitis.
A patient with a spontaneous abortion (choice D) can present in a variety of
ways, but most commonly she will present with complaints of vaginal bleeding or
the passage of tissue from the vagina. A spontaneous abortion represents a
failed pregnancy and this patient has no evidence of a failed pregnancy, and an
abundance of evidence for PID.
A patient with a urinary tract infection (UTI) (choice E) will usually present
complaining of frequency, urgency, or dysuria. Also, the urinalysis will typically
show white blood cells. This patient has no urinary complaints and a negative
urinalysis.


One of the antibiotics that this patient is started on is doxycycline. This antibiotic
works via which of the following mechanisms?
/ A. Inhibition of bacterial cell wall synthesis
/ B. Inhibition of bacteriaI DNA gyrase
/ C. Inhibition of bacterial protein synthesis
/ D. Inhibition of ergosterol synthesis
/ E. Inhibition of viraI DNA synthesis after conversion by thymidine kinase



The correct answer is C. Doxycycline is a member of the tetracycline family of
antibiotics. These bacteriostatic antibiotics are broad-spectrum in nature and are
used in a variety of conditions. The mechanism of action of the tetracycline family
of antibiotics is that they prevent the attachment of transfer RNA to the 50S
ribosomal subunit, thus inhibiting bacterial protein synthesis. The tetracycline
family of medications is used in the treatment of syphilis, Chlamydia, gonorrhea,
amebiasis, urinary tract infections, as well as pelvic inflammatory disease. Along
with its use in pelvic inflammatory disease, doxycycline is also well-known for its
use in the treatment of Lyme disease.
Penicillins and cephalosporins are two families of antibiotics that work via the
mechanism of inhibition of bacterial cell wall synthesis (choice A). Cefotetan (a
third-generation cephalosporin) or cefoxitin (a second generation cephalosporin)
are often used with doxycycline in the treatment of PID.
Fluoroquinolones are antibiotics that function through the inhibition of bacterial
DNA gyrase (choice B). They are used to treat a variety of organisms including
Escherichia coli, Proteus mirabilis, and Neisseria gonorrhoeae. They are often
used in the treatment of urinary tract infections.
Miconazole is the drug that works through the inhibition of ergosterol synthesis
(choice D). By doing so, it disrupts fungal cell wall synthesis. It is often used in
the treatment of topical fungal infections, but can also be used intravenously in
the treatment of systemic fungal infections.
Acyclovir is the drug that functions through the inhibition of viral DNA synthesis
after conversion by thymidine kinase (choice E). It is used predominantly in the
treatment of herpes and varicella-zoster infections.


Doxycycline is not used during pregnancy because of possible adverse effects
on which of the following fetal structures?
/ A. Heart
/ B. Kidneys
/ C. Limbs
D. Teeth
/ E. Tendons


The correct answer is D. Tetracyclines are known to chelate with calcium ion.
This characteristic results in these drugs becoming incorporated into dental
enamel and bone. When used by pregnant women, the tetracyclines have been
shown to cause staining of teeth and suppression of skeletal development.
These drugs, therefore, should not be used by pregnant women or children
younger than 8 years of age. Also, because of the interaction of the tetracyclines
with calcium, calcium-containing supplements and antacids should not be used
at the same time as tetracycline antibiotics. At least 2 hours separation between
the ingestion of the 2 substances should be given.
Cardiovascular birth defects are among the most common birth defects. Lithium,
which has been used to treat manic depressive disorder and other psychiatric
problems, has been associated with Ebstein's anomaly, a defect of the fetal heart
(choice A).
Angiotensin converting enzyme (ACE) inhibitors are medications commonly used
in patients with hypertension. This family of medications includes drugs such as
captopril, enalapril, lisinopril, quinapril, and several others. These medications
should not be used during pregnancy because of the adverse effects they have
on fetal kidneys (choice B). If used in pregnancy, they can cause severe fetal
renal damage and death.
Thalidomide is a medication that was first used in Europe in the 1950s (and later
the United States) for the treatment of insomnia and nausea in pregnant women.
Its use resulted in the birth of thousands of babies with stunted growth of the
limbs (choice C). Even one dose of the medication can be associated with birth
defects.
Fluoroquinolones should not be used during pregnancy because of possible
effects on fetal tendons (choice E). Studies in dogs have shown that use of
these drugs in pregnancy can cause arthropathies.

This patient's condition can lead to adhesions in the pelvis and around what other
structure?
/ A. Brain
/ B. Heart
/ C. Kidneys
/ D. Liver
/ E. Lungs
The correct answer is D. The Fitz-Hugh-Curtis syndrome is a syndrome of
perihepatitis that approximately 5% of women with pelvic inflammatory disease
will develop. The features of the Fitz-Hugh-Curtis syndrome are right upper
quadrant pain and tenderness, sometimes with mildly abnormal liver function test
results. When laparoscopy is performed on a patient with Fitz-Hugh-Curtis
syndrome, fibrous, "violin string" adhesions can be seen extending from the
dome of the liver to the diaphragm. The syndrome is most commonly associated
with gonococcal or chlamydial PID. It is believed that the intraperitoneal spread
of the infection (and inflammation) is the cause of the condition. Because there is
a direct anatomic connection between the liver and pelvic structures, direct
intraperitoneal spread of infection and inflammation can occur.
The brain (choice A) is not an intraperitoneal structure. Infection of the brain
from pelvic inflammatory disease would be exceedingly rare and would require a
mechanism other than direct intraperitoneal spread.
The heart (choice B) is also not an intraperitoneal structure. In order to develop
a myocarditis (infection of the myocardium) or an endocarditis (infection of the
endocardium), would require a mechanism other than direct intra-abdominal
spread.
The kidneys (choice C) are a retroperitoneal structure. Direct intraperitoneal
extension in a patient with pelvic inflammatory disease could not cause nephritis.
The lungs (choice E) are also not an intraperitoneal structure. Pelvic
inflammatory disease leading to pneumonitis would also be rare.


A 58-year-old woman comes to the physician because of vaginal bleeding. She
states that she had her last menstrual period 11 years ago and since that time,
she has not had any vaginal bleeding, until now. She has no other complaints.
Her past medical history is significant for diabetes and hypertension. She has
never had surgery. She currently takes glyburide and captopriI. She was taking
estrogen replacement therapy for hot flashes untiI 5 years ago. She has no
known drug allergies. On physical examination, she is a 5 foot, 2 inch female
who weighs 220 pounds. Speculum examination reveals some old blood in the
vaginal vault. Pelvic examination is limited by the patient's body habitus. The
remainder of the physical examination is within normal limits. An endometrial
biopsy is performed, that shows crowded endometrial glands with severe
cytologic and architectural abnormalities, and greater than 50% solid areas.



Which of the following is the most likely diagnosis?
/ A. Ectopic pregnancy
/ B. Endometrial atrophy
/ C. Endometrial carcinoma
/ D. Pelvic inflammatory disease
/ E. Polycystic ovary syndrome


The correct answer is C. This patient's presentation and findings are most
consistent with endometrial carcinoma. Endometrial carcinoma is the most
common gynecologic malignancy. It is approximately two times more than
ovarian cancer and roughly three times more common than invasive cervical
cancer. The major risk factors for the development of endometrial cancer are
obesity and the use of unopposed estrogen replacement therapy. Hypertension
and diabetes also appears to be significant risk factors. Other risk factors include
infertility, nulliparity, early menarche, and late menopause. This patient is at
significant risk given her hypertension, diabetes, obesity, and history of exposure
to unopposed estrogen. Diagnosis is made by endometrial biopsy. Treatment is
surgical: total abdominal hysterectomy with bilateral salpingo-oophorectomy and
staging. In patients who are poor candidates for surgery, radiation alone can be
used, but the cure rate is low.
Ectopic pregnancy (choice A) can cause abnormal vaginal bleeding. However,
for an ectopic pregnancy to occur, ovulation must take place in the woman. A
woman, such as this patient, who is postmenopausal, is extremely unlikely to
ovulate and, therefore, ectopic pregnancy is not the most likely diagnosis.
Furthermore, her symptoms, findings and pathology results establish the
diagnosis of endometrial cancer.
Endometrial atrophy (choice B) can cause vaginal bleeding in postmenopausal
women. However, in these cases, the biopsy demonstrates atrophy, and not
endometrial cancer as in this patient.
Pelvic inflammatory disease (choice D) is a disease predominantly of young,
sexually active women. Its peak incidence is in the 15 to 24-year-old group. It is
characterized by abdominal tenderness, cervical motion tenderness, and adnexal
tenderness with confirmatory laboratory findings. This patient's presentation is
not consistent with pelvic inflammatory disease.
Polycystic ovary syndrome (choice E) appears to place women at increased risk
for the eventual development of endometrial hyperplasia and cancer. However,
this patient is no longer "at risk." She, in fact, has endometrial cancer.


Administration of which of the following hormones might have prevented this
patient's condition?
/ A. Follicle stimulating hormone
/ B. Human chorionic gonadotropin
/ C. Insulin
/ D. Progesterone
/ E. Testosterone



The correct answer is D. One hypothesis for the development of endometrial
cancer is that unopposed estrogen leads to the proliferation and eventual
cancerous change of the endometrial lining. It has been well-demonstrated that
postmenopausal women who have a uterus (i.e., postmenopausal women who
have not had a hysterectomy) have an increased probability of developing
endometrial cancer that correlates directly with the dose and duration of
unopposed estrogen use. The increased risk has also been shown to persist
even after the estrogen has been stopped. "Unopposed" estrogen refers to
estrogen that is not opposed by progesterone. This patient has no past surgical
history, that is, she has never had a hysterectomy. She was taking unopposed
estrogen during her 50s. She should have also been on a progestin at that time
to prevent the development of endometrial cancer.
Follicle stimulating hormone (choice A) is produced by the anterior pituitary and
acts upon the ovary. It is not given with estrogen to postmenopausal women to
prevent the development of endometrial hyperplasia and cancer.
Human chronic gonadotropin (choice B) is produced by the trophoblast cells of
the developing embryo to support the corpus luteum so that the corpus luteum
will continue to secrete hormones to support the pregnancy during its first several
weeks. It is not given to postmenopausal women to oppose the effects of
estrogen.
Insulin (choice C) is a peptide hormone secreted by the pancreas and is
essential for glucose metabolism and regulation.
Testosterone (choice E) is used by some physicians in postmenopausal women
to enhance sexual libido and well-being. It is not used like progesterone to
oppose the effects of estrogen on the endometrium.

One of the risks of the endometrial biopsy that was performed on this patient is
perforation of the uterus. The endometrial biopsy device is placed through the
cervix and into the endometrial cavity. If complete perforation occurs, what is the
sequence of layers that the biopsy device would penetrate prior to entering the
peritoneal cavity?
/ A. Endometrium, myometrium, serosa
/ B. Ovary, fallopian tube, broad ligament
/ C. Round ligament, cardinal ligament, uterosacral ligament
/ D. Serosa, myometrium, endometrium
/ E. Uterine body, fundus, cervix

The correct answer is A. The uterus is a hollow, muscular organ that lies
between the bladder and the rectum in the true pelvis. The uterus can be divided
into three major segments: the uterine fundus, the uterine corpus (or body), and
the cervix. The fallopian tubes enter the uterine fundus laterally in the region
called the cornua. Moving from the inside (or hollow) portion of the uterus to the
peritoneal cavity, the layers that would be penetrated are the endometrium, the
myometrium, and the serosa. The endometrium is the portion of the uterus that
proliferates during the menstrual cycle and sheds (the menses) if pregnancy
does not occur, or supports the pregnancy if conception and implantation take
place. The myometrium is comprised of three layers: an inner layer of
longitudinal smooth muscle, a middle layer of circular smooth muscle (the
stratum vasculare), and an outer layer of longitudinal and circular smooth muscle
(the stratum supervascularae). The final layer is the uterine serosa.
The ovary, fallopian tube, and broad ligament (choice B) are found
posterolateral to the uterus. A biopsy device that perforated the uterus could also
injure these structures, but the direct path of perforation is through the
endometrium, myometrium, and serosa.
The round ligament, cardinal ligament, and uterosacral ligament (choice C) are
supporting structures attached to the uterus. Again, while a perorating biopsy
instrument could possibly involve these structures, this would not be the direct
path of perforation.
The serosa, myometrium, and endometrium (choice D) are the correct structures
that would be perforated, but they are in the wrong order. Starting within the
uterus, the first layer is the endometrium, next is the myometrium and finally the
serosa.
The uterine body, fundus, and cervix (choice E) describe the three general parts
of the uterus.


Estrogen is believed to play a crucial role in the development of this disease.
Which of the following substances is the precursor of this hormone?
/ A. Arachidonic acid
/ B. Argininosuccinate
/ C. Cholesterol
/ D. Oxaloacetate
/ E. Succinate


The correct answer is C. There are 5 principal types of steroid hormones:
glucocorticoids, mineralocorticoids, estrogens, progestins, and androgens. These
steroids hormones are synthesized from cholesterol in the cytosol and the
mitochondria. The main organs of steroid biosynthesis are the adrenal cortex,
ovaries, testes, and the placenta. The first step in the synthesis of the steroid
hormones, which is also the rate limiting step, is the conversion of cholesterol to
pregnenolone. This reaction is catalyzed by the desmolase complex. Several
other intermediate steps must occur for the eventual formation of the steroid
hormones. Defects in these pathways can result in congenital adrenal
hyperplasia.
Arachidonic acid (choice A) is a precursor in the formation of prostaglandins,
thromboxanes, and leukotrienes. These substances cause a multitude of
physiologic responses.
Argininosuccinate (choice B) is an intermediate compound in the urea cycle.
Urea is the major excretory product of nitrogen metabolism in human beings.
Argininosuccinate is formed when aspartate, ATP, and citrulline react in a
reaction catalyzed by argininosuccinate synthetase.
Oxaloacetate (choice D) and succinate (choice E) are compounds found in the
Krebs cycle (also called the citric acid cycle or the tricarboxylic acid cycle). This
cycle represents the final common pathway for the degradation of fatty acids,
amino acids, and carbohydrates.



A 57-year-old woman presents to a physician with chronic pelvic pain of many
years duration. On further questioning, the woman also reports urinary
frequency, constipation, pain with intercourse, and bloating. On physical
examination, a large mass is felt in the pelvic area. Follow-up ultrasound
examination demonstrates that the mass involves the right adnexa and
is composed of multiloculated cystic spaces.


Prior to sending the patient to surgery, the primary care physician wants to order
a serum tumor marker to screen for ovarian cancer. Which of the following would
be the best choice?
/ A. Adrenocorticotropic hormone
/ B. CA-125
/ C. Galactosyltransferase
/ D. Ribonuclease
/ E. S-100


The correct answer is B. The diagnosis of ovarian cancer is often delayed,
because symptoms often do not occur until late in the disease when the mass is
large and applies pressure to other abdominal organs. The symptoms illustrated
in the case are typical. Patients may also develop ascites with shortness of
breath, a variety of gastrointestinal symptoms related to impaired motility
secondary to pressure, and, if the ovarian cancer is a type that secretes
hormones, menstrual irregularities or abnormal hair growth. The most widely
studied tumor marker in ovarian cancer is CA-125. This marker is not of
particular use in screening of the general population for two reasons: 1) small
cancers often do not cause serum elevations of the marker; and 2) serum levels
of the marker can be elevated by a variety of cancers (including those of ovary,
breast, pancreas, colon, and lung) and benign conditions (including
endometriosis, pregnancy, liver disease, and congestive heart failure). However,
if there is already a strong suspicion or a known history of ovarian cancer, CA-
125 can be helpful in establishing if the cancer burden (if present) is large, and
whether following the CA-125 levels following surgery can be used to monitor for
recurrent disease. While small tumors are not always picked up (50% of women
with small ovarian cancers have normal CA-125), more than 80% of women with
advanced ovarian cancer have CA-125 elevations.
Adrenocorticotropic hormone (choice A) can be elevated in ectopic hormone-
producing lung cancers.
Galactosyltransferase (choice C) can be elevated in a wide variety of cancers,
including those from lung, breast, esophagus, stomach, pancreas, and colon. It is
not used as a marker for ovarian cancer.
Ribonuclease (choice D) is a marker for pancreatic cancer.
S-100 (choice E) is a marker for melanoma and neuroendocrine tumors.


In this particular case, serum levels of the chosen tumor marker are within the
normal range, which is interpreted to mean that the woman either does not have
ovarian cancer or has a smaller amount of cancer, which is probably confined to
the ovary. The woman is taken to surgery and a 20-cm diameter adnexal mass is
removed and sent for intraoperative examination. The specimen submitted to
pathology is roughly the size and shape of a large melon, has a smooth external
surface, shows a pale color on cross-section, and is nearly completely replaced
by a large number of cystic spaces of varying sizes from nearly microscopic to
over 6 cm diameter. These cysts contain cloudy white gelatinous material that
slowly oozes fluid and the cyst walls are generally thin. Which of the following is
the most likely diagnosis at this point in the evaluation?
/ A. Benign or malignant Brenner tumor
/ B. Benign or malignant granulosa celI-theca cell tumor
/ C. Dermoid cyst or immature teratoma
/ D. Mucinous cystadenoma or mucinous cystadenocarcinoma
/ E. Serous cystadenoma or serous cystadenocarcinoma



The correct answer is D. In real life, the gross examination of a specimen tends
to be most helpful if it provides enough clues to tentatively place the lesion into a
broad category. Usually, unless clear-cut invasion is seen grossly, the gross
examination cannot "prove" whether the lesion is benign or malignant. In this
case, a large ovarian mass virtually completely replaced by cystic spaces filled
with gelatinous material, and whose stroma appears pale is most likely a cystic
mucinous lesion, such as mucinous cystadenoma or mucinous
cystadenocarcinoma.
Brenner tumors (choice A), which are predominately benign, are often solid (but
may be somewhat cystic with less complete replacement of the tumor than is
illustrated in this case) and the stroma tends to be firm and white, resembling the
normal stroma of the ovary.
The various granulosa cell and theca cell tumors (choice B) are usually benign
(rarely malignant, most commonly as a granulosa cell tumor) and tend to form
solid or partially cystic tumors that have a yellow hue if the tumor is
endocrinologically active.
Dermoid cysts (choice C, also called mature cystic teratomas) are typically filled
with cheesy white material (derived from shedding of the surface of skin) and
hair; while the related (and malignant) immature teratomas (choice C) are
uncommon in the ovary and tend to be mostly or completely solid.
Serous cystadenomas and cystadenocarcinomas (choice E) closely resemble
the mucinous variants, but the cystic spaces are filled with clear fluid rather than
gelatinous material.


The cells lining the cystic spaces in this tumor are most closely related to which
of the following normal cell types?
/ A. Decidual cell in the ovarian stroma
/ B. Fibroblastic ovarian stroma cell
/ C. Oocyte in preantral follicle
/ D. Ovarian surface epithelial cell
/ E. Smooth muscle cell in ovarian stroma



The correct answer is D. The ovary is vulnerable to a surprisingly large variety
of primary tumors. Ovarian tumors are broadly classified into epithelial tumors
(resembling the epithelial cells of the ovary and including the serous tumors,
mucinous tumors, endometrioid tumors, clear cell tumors, and transitional cell
tumors); the sex cord-stromal tumors (related to choices B, D, and E, and
including the granulosa cell tumors, the thecoma-fibroma tumors, the Sertoli cell
tumors, the androblastomas, and the steroid cell tumors); and the germ cell
tumors (related to choice C, and including the teratomas, dysgerminomas, yolk
sac tumors, and mixed germ cell tumors). In this patient's large cystic mucinous
tumor, the cells lining the cystic spaces are epithelial in nature, and thought to be
most closely related to the epithelial cells on the surface of the ovary.
Incidentally, peritoneum in other sites can undergo metaplasia to an epithelium
resembling that on the surface of the ovary, and rarely, tumors that histologically
resemble mucinous and serous cystic tumors of the ovary can arise in other sites
involving peritoneum.


Extensive sampling reveals an area of frank cancer within the tumor, and a
metastasis is also found on the surface of the uterus. Following surgery, the
decision is made to add carboplatin. This drug acts by which of the following
mechanisms?
/ A. BIocking microtubule assembly
/ B. Cross-Iinking DNA
/ C. Inhibiting topoisomerase ll
/ D. Interrupting folate metabolism
/ E. Substituting for a nucleotide in DNA or RNA


The correct answer is B. Most cases of ovarian mucinous cystadenocarcinoma
of the ovary that are not of a very low stage receive chemotherapy in addition to
surgery. The most common agent chosen is carboplatin, which is a drug similar
to cisplatin and acts by a similar mechanism. Both of these drugs cross-link DNA.
Vinblastine is an example of a chemotherapeutic agent that acts by blocking
microtubule assembly (choice A).
Etoposide is an example of a chemotherapeutic agent that acts by inhibiting
topoisomerase II (choice C).
Methotrexate is an example of a cancer chemotherapeutic agent that acts by
interrupting folate metabolism (choice D).
Examples of cancer chemotherapeutic agents that act by substituting for a
nucleotide (choice E) include 5-fluorouracil, cytarabine, and 6-mercaptopurine.


Which of the following genes has been most closely associated with familial
cases of ovarian cancer?
/ A. BRCA1
/ B. NF1
/ C. NF2
/ D. VHL
/ E. WT1


The correct answer is A. Roughly 5-10% of cases of ovarian cancer occur in
individuals who are a member of a cancer family. In familial cases of ovarian
cancer, the cancers tend to appear at a younger age, but are thought to possibly
have, on average, a slightly better prognosis than do isolated cases. Families
with multiple cases of breast cancer due to genetic mutations in BRCA1 or
BRCA2 also have an increased incidence of ovarian cancers. The BRCA1
mutation is particularly virulent, and up to 30-40% of women with this mutation
develop ovarian cancer. (There are also families in which the women have a
BRCA1 mutation and develop ovarian, but not breast cancer at an increased
rate.) The BRCA1 and BRCA2 mutations have a particularly high incidence in
Ashkenazi Jews. Another cancer syndrome, the Lynch II syndrome (also known
as the hereditary nonpolyposis colorectal cancer syndrome), is also associated
with ovarian cancer (and cancers of the uterus, breast, stomach, and pancreas).
In both the breast cancer families and the Lynch II families, the genetic pattern
seen is usually autosomal dominant with variable penetrance. Prophylactic
oophorectomy is often offered women in cancer families who have reached the
age of 35 and have had their children.
NF1 (choice B) is associated with neurofibromatosis type I, neuroblastoma,
melanoma, and colon cancer.
NF2 (choice C) is associated with neurofibromatosis type II, acoustic neuromas,
and meningiomas.
VHL (choice D) is associated with von Hippel-Lindau disease, renal cell
carcinoma, and pheochromocytoma.
WT1 (choice E) is associated with Wilms tumor.


Which of the following tends to decrease the risk of ovarian cancer?
/ A. Age greater than 50 years
/ B. Caucasian race
/ C. Combined oral contraceptive pills
/ D. History of breast cancer
/ E. No history of pregnancy


The correct answer is C. Decreased risk of developing ovarian cancer is
associated with processes that seem to interrupt ovulation, including the use of
the combined oral contraceptive pill, increasing parity, and breast feeding.
An increased risk of ovarian cancer is seen with age greater than 50 years
(choice A), no history of pregnancy (choice E), breast cancer (choice D), and
Caucasian race (choice B). Less well established and less common risk factors
include exposure of the genitalia to asbestos and talc, mumps virus, and
estrogen therapy in postmenopausal women.
Several months later, the patient returns to the hospital and is found to have
widespread metastases. The patient is currently arousable and oriented. The
patient has designated a family member as her health care power of attorney
(HCPOA). The most appropriate first step is to determine which of the following?
/ A. If the patient is interested in a hospice referral
/ B. If the patient understands what is likely to happen
/ C. The patient's main coping style
/ D. Who is the HCPOA designee
/ E. Why the patient chose her HCPOA




The correct answer is B. The Health Care Power of Attorney designates
someone to make the patient's medical decisions in the event that the person
becomes incompetent or unable to make decisions. If this patient is awake and
understands her situation, the HCPOA is not yet activated.
Hospice (choice A) can be chosen by the patient when she desires palliative,
rather than attempted curative therapy. The immediate concern in this case is to
address the patient wishes in the very near future.
Coping styles (choice C) can impede diagnosis and treatment of disease and
even add to the disease state. The immediate concern in this case is to find out
what the patient wishes should she code.
It is important to know the designee (choice D) because he or she becomes the
decision maker when the patient is no longer able. However, determining if this
patient is still able to make her own decisions is the immediate concern.
Why the patient chose her HCPOA (choice E) is irrelevant.


The patient is successfully stabilized and returns home. She is admitted to a
nursing home three months later. One day, her physician finds her somnolent
and difficult to arouse. The Health Care Power of Attorney (HCPOA) designee
has told the physician the he wants nothing done except supportive care. Which
of the following is the most appropriate next step?
/ A. Order an EEG
/ B. Discuss intravenous fluids
/ C. Order a head CT scan
/ D. PIace a central line
/ E. Transfer to the emergency room



The correct answer is B. Fluids and pain control are consistent with supportive
care.
Additional tests (e.g., EEG, choice A) are not consistent with supportive care.
A head CT scan (choice C) implies diagnosis and treatment. It is not consistent
with supportive care.
Placing a central line (choice D) implies aggressive treatment and is not
consistent with supportive care.
Transferring to the emergency room (choice E) implies diagnosis and treatment.
It is not consistent with supportive care.

As the physician is speaking with the designated HCPOA, a frantic staff member
enters the room and reports an estranged sibling has called on the telephone,
requesting that "everything be done" until he can arrive. He has stated he wilI
"sue the pants off that doctor if they give up on his sister." The designated
HCPOA reiterates the plan for supportive care. The most appropriate next step is
which of the following?

/ A. Begin medical treatment to buy time for the family to all agree to the same
plan
/ B. Call the EMS to take the patient to the hospital
/ C. Discuss the plan for supportive care and honor the designated HCPOA's
request
/ D. Tell the family you will turn the brother into adult protective services
/ E. Refuse to speak to the designated HCPOA because he does not know what
he is doing



The correct answer is C. The designated HCPOA's decision is the final one and
the legal agreement has been set up to prevent the patient from any measures
she did not want to be subjected to, despite the wishes of any other family
members, doctors, etc.
Beginning medical treatment (choice A) is inappropriate and illegal given the
clear wishes of the HCPOA.
Calling the EMS (choice B) is inappropriate and illegal given the clear wishes of
the HCPOA.
Telling the family you will turn the brother into adult protective services (choice
D) is inappropriate and not indicated. It is within the rights of the patient's HCPOA
to proceed with supportive care as planned.
Refusing to speak to the designated HCPOA because he does not know what he
is doing (choice E) is not acceptable for a competent physician. It is acceptable
to explain your recommendations or concerns, but the final decision rests with
the HCPOA.






A 17-year-old girl is evaluated by a gynecologist because she has never had a
menstrual period. On physical examination, the girl is noted to
be 5'2" talI, have slight webbing of her neck, and a broad chest with widely
spaced nipples. The girI's breasts show papilla elevation only. In her
pubic area, only villus hair is seen. No mature axillary hair is seen.
Question 1 of 5
This girI's body is at which of the following Tanner stages of development?
/ A. Stage 1
/ B. Stage 2
/ C. Stage 3
/ D. Stage 4
/ E. Stage 5

Explanation - Q: 1.1 Close

The correct answer is A. Tanner stages are used to define the degree of
sexual maturation of a girl's body. Stage 1 is the prepubertal stage, and is
characterized by elevation of only the papilla of the breast, and pubic hair
consisting only of fine villus hair. The child's height usually increases at a
basal rate of 5-6 cm per year.
Stage 2 (choice B) corresponds to the beginning of puberty and is
characterized by palpable breast buds, enlargement of the areola, minimal
coarse, pigmented hair mainly on the labia, and accelerated rate of height
increase of typically 7-8 cm per year.
Stage 3 (choice C) is characterized by elevation of breast contour with
areolar enlargement, dark curly hair over mons pubis, axillary hair
development, and peak height increase of about 8 cm/year. Acne vulgaris
may develop in this stage.
Stage 4 (choice D) is characterized by formation, by the areola, of a
secondary mound on the breast, adult quality pubic hair with no spread to the
junction of the medial thigh with the perineum, and height increase of about 7
cm/year.
Stage 5 (choice E) is characterized by adult breast contour with recession of
the areola to the general contour of the breast, adult distribution of pubic hair
with spread to medial thigh, and no further increases in height.


Question 2 of 5
Most girls who are this patient's age are at which of the following Tanner stages?
/ A. Stage 1
/ B. Stage 2
/ C. Stage 3
/ D. Stage 4
/ E. Stage 5

Explanation - Q: 1.2 Close

The correct answer is E. This patient is markedly behind her peers in
sexual development, since Tanner stage 5 is usually reached by age 16.
Stage 1 (choice A) is the prepubertal stage that begins at birth.
Stage 2 (choice B) typically becomes noticeable at around 11 years (9-13
years for normal range), with breast bud enlargement often slightly preceding
early pubic hair development.
Stage 3 (choice C) typically occurs at about 12 years (9 1/2 to 14 years) and
often includes the onset of menstruation.
Stage 4 (choice D) typically occurs at about 13 years (10 1/2 to 15 1/2
years).



Question 3 of 5

Which of the following is the most likely diagnosis?
/ A. Down syndrome
/ B. Edwards syndrome
/ C. KIinefelter syndrome
/ D. Triple X syndrome
/ E. Turner syndrome

Explanation - Q: 1.3 Close

The correct answer is E. This patient has a number of the stigmata of
Turner syndrome, including primary amenorrhea (no menstrual periods ever),
failure of onset of puberty, short stature, webbed neck, and widely spaced
nipples. Other features that may be a part of Turner syndrome include
multiple pigmented nevi, short 4th metacarpals and metatarsals, prominent
finger pads, nail hypoplasia, and some diminution of perceptual ability,
usually without frank mental retardation. Patients with Turner syndrome are
now treated with oral sex hormone replacement, and will go through puberty
and begin menstruation. They are usually infertile (exceptions being rare
mosaic patients with some residual ovarian function), but recent work
suggests that they may be able to carry a baby produced by in vitro
fertilization, if appropriate hormonal support is given throughout pregnancy.
Growth hormone supplementation begun by about age 9 if possible will
increase height.
Down syndrome (choice A) is characterized mental retardation,
characteristic facies (slanted eyes, small head, flattened occiput, flattened
nose), single palmar (simian) crease, and short fingers.
Edwards syndrome (choice B) is characterized by severe mental retardation
and usually death in infancy.
Klinefelter syndrome (choice C) is characterized by male phenotype, tall
stature, small testes, and a predisposition for learning difficulties.
Triple X syndrome (choice D) is characterized by a phenotypically
apparently normal female who may or may not have sterility and menstrual
irregularities


Question 4 of 5
Which of the following genotypes would be most likely present in this patient?
/ A. 45,XO
/ B. 47,XXX
/ C. 47,XXY
/ D. Trisomy 18
/ E. Trisomy 21

Explanation - Q: 1.4 Close

The correct answer is A. Turner's syndrome is a sex chromosome
abnormality, which in about half of live birth cases, is due to 45,XO
karyotype. The remainder are usually mosaics, with 45,XO/46,XX or
45,XO/47,XXX karyotypes. It is thought that approximately 98% of Turner's
conceptions die in utero early in pregnancy.
47,XXX (choice B) is called triple X syndrome.
47,XXY (choice C) is the karyotype for Klinefelter syndrome.
Trisomy 18 (choice D) is also known as Edwards syndrome.
Trisomy 21 (choice E) is also known as Down syndrome.







Question 5 of 5
This patient's condition is most strongly associated with which of the following
congenital anomalies?
/ A. Atrial septal defect
/ B. Coarctation of the aorta
/ C. Patent ductus arteriosus
/ D. Tetralogy of Fallot
/ E. Ventricular septal defect

Explanation - Q: 1.5 Close

The correct answer is B. 35% of patients with Turner's syndrome have
coarctation of the aorta, which may be asymptomatic. An easy screening test
is to check the pulses at both ankles and both wrists to note any discrepancy
in pulse strength that might suggest the presence of a segment of aortic
narrowing. Coarctation of the aorta is also associated with bicuspid aortic
valve, intracranial berry aneurysms, ventricular septal defect, and acquired
intercostal aneurysms.
Atrial septal defect (choice A) can occur as part of the autosomal dominant
condition Holt Oram syndrome, which also is associated with upper extremity
bony abnormalities.
Patent ductus arteriosus (choice C) is associated with a very large number
of infectious and genetic congenital conditions, with or without other
congenital heart disease.
Tetralogy of Fallot (choice D) is associated with Down syndrome.
Ventricular septal defect (choice E) is a component of tetralogy of Fallot, and
can also be associated with Down syndrome, Patau syndrome, Edwards
syndrome, and Holt-Oram syndrome.


A 34-year-old woman consults a physician because her menstrual periods have
been irregular for the last 5 to 10 years. Her cycles are often
greater than six weeks in length, with eight or fewer periods in a year. The
periods often vary in character, with lengthy bleeding episodes,
scant or heavy periods, or frequent spotting.
Question 1 of 4
One of the findings noted on this woman's physical examination is the presence
of poorly defined large patches of darkened skin on the back
of her neck and in skin creases under her arms and breasts. These darkened
skin patches have a very slightly rough, velvety texture. The
Iesions are not itchy or irritated in feeling, and the patient had been only vaguely
aware of them. Which of the following skin diseases would be
most likely to produce this type of skin manifestation?
/ A. Acanthosis nigricans
/ B. Lichen planus
/ C. Malignant melanoma
/ D. Psoriasis vulgaris
/ E. Szary syndrome

Explanation - Q: 2.1 Close

The correct answer is A. The lesions are those of acanthosis nigricans.
While they appear hyperpigmented, there is actually little or no increase in
melanin in these areas, and the dark coloring is instead the result of
papillomatous epidermal and superficial dermal hyperplasia.
Lichen planus (choice B) is an inflammatory skin condition characterized by
itchy, purple papules or plaques.
Malignant melanoma (choice C) is a malignant skin tumor (loosely a
malignant mole) that can produce darkly pigmented skin, but it would be very
unusual to have multiple lesions or lesions under the breasts or arms. Also,
while the edges of a melanoma may be feathery, the lesion's circumference
is usually relatively well-defined.
Psoriasis vulgaris (choice D) is a proliferative disease of the skin and
produces clearly defined plaques with a deep red color ('salmon-colored")
and silvery scale and often involves the limbs.
Szary syndrome (choice E) is an intensely erythematosus, itchy, skin
reaction that involves the whole body and is a reaction to a T-cell lymphoma


Question 2 of 4
In addition to being associated with the condition causing this patient's menstrual
irregularities, the patient's skin condition can be associated
with which of the following?
/ A. Gastric carcinoma
/ B. Gout
/ C. Myocardial infarction
/ D. Rheumatoid arthritis
/ E. UIcerative colitis

Explanation - Q: 2.2 Close

The correct answer is A. Acanthosis nigricans can be subdivided into
"benign" (as this patient has) and "malignant" variants. The "benign" form
typically has milder lesions with localized involvement of the back of the
neck, arm pits, areas below the breast, groin, vulva, between the thighs, and
sometimes on the hands, elbows, and knees. One or several patches of
involved skin may be seen. This form is associated with endocrine
abnormalities, including insulin-resistance, obesity, hyperandrogen states,
and other endocrine abnormalities. The "malignant" form of acanthosis
nigricans is not actually a malignant skin disease, but it is so-called because
it is associated with underlying carcinomas, often of the gastrointestinal tract
(frequently stomach). This form tends to produce much more severe skin
lesions, with a much broader distribution (sometimes involving almost all of
the skin), and patients with "malignant" acanthosis nigricans often die within
2 years of their underlying cancer.
Gout (choice B) patients can have tophi, composed of deposits of urate with
the accompanying skin reaction; the tophi are most often recognized on the
external ear, although they can occur in other locations.
Myocardial infarction (choice C) does not usually produce a skin
manifestation unless congestive heart failure develops, in which case,
edema of the ankles may develop.
Rheumatoid arthritis (choice D) is associated with subcutaneous rheumatoid
nodules, vasculitis, and leg ulcers.
Inflammatory bowel disease, e.g., ulcerative colitis (choice E), is associated
with oral mucosal ulcerations (aphthous ulcers) and pyoderma granulosum
(causes a severe ulcer of the lower leg).



Question 3 of 4

Physical examination reveals hirsutism. Pelvic examination demonstrates
bilateral enlargement of the adnexa to approximately 3 times the
size of normaI. Follow-up ultrasound examination demonstrates a "string of
pearls" appearance in both ovaries. Given these findings, and the
patient's menstrual irregularities, hirsutism, and skin changes, which of the
following is the most likely diagnosis?
/ A. Bilateral dermoid cysts
/ B. Bilateral hydrosalpinx
/ C. Bilateral mucinous cystadenocarcinoma
/ D. Multiple leiomyomas
/ E. Polycystic ovary disease

Explanation - Q: 2.3 Close

The correct answer is E. The diagnosis that best accounts for the patient's
complex findings is polycystic ovary disease (also known as Stein-Leventhal
syndrome). The clinical presentation of polycystic ovary disease can include
menstrual problems (amenorrhea, infrequent menses, oligomenorrhea,
irregularity), infertility (secondary to infrequent or absent ovulation),
symptoms of androgen excess (hirsutism, alopecia, acne), chronic pelvic
pain or palpable mass, obesity, and acanthosis nigricans. The polycystic
ovaries are typically enlarged 1.5 to 3 times, and the finding of the "string-of-
pearls" (e.g., small cystic spaces lined up in a row under the ovarian surface)
on ultrasound examination is also typical.
Dermoid cysts (benign ovarian teratomas that may produce hair and skin,
choice A), hydrosalpinx (pathologically dilated fallopian tube, choice B), and
cyst adenocarcinomas (a form of malignant ovarian tumor that does not
usually secrete hormones, choice C) can cause adnexal masses, but would
not produce the hirsutism or acanthosis nigricans seen in this case.
Leiomyomas (choice D) are solid uterine tumors.


Question 4 of 4

Which of the following most accurately describes the pathophysiology of the
adnexal masses present in this patient's condition?

/ A. Areas of focal abscess formation
/ B. Areas of focal necrosis
/ C. Follicles in various stages of maturation
/ D. Small cysts within a benign tumor
/ E. Small cysts within a malignant tumor

Explanation - Q: 2.4 Close

The correct answer is C. The cysts that give polycystic ovary disease its
name are actually ovarian follicles in varying states of maturation. One of the
underlying mechanisms leading to the formation of the polycystic ovaries
appears to be a benign, nontumorous, proliferation of ovarian stromal tissue
under the influence of a high androgen state, which then makes it physically
much harder for a follicle to rupture the ovarian surface and release its egg.
Any menstrual cycle in which the egg fails to leave the ovary is anovulatory,
and has no chance of conception on that cycle. Further, the corpus luteum
fails to develop, and the progesterone levels remain low rather than rising.
The cysts are not related to true tumor (choices D and E), necrosis (choice
B), or abscess formation (choice A).


A 38-year-old woman is seen by a gynecologist on a routine visit. A complete
history is taken and is non-contributory except for the fact that
the patient's menstrual irregularities apparently developed insidiously over the
preceding decade. Screening physical examination is
performed. Routine serum chemistries and complete blood count are sent, as are
Pap smear studies. Pelvic ultrasound studies are also
performed. Pregnancy test is negative. Endometrial biopsy shows proliferative
endometrium. AII other results are within normal limits, except
for evidence of a mild iron deficiency anemia.




Question 1 of 3
Which of the following is the most likely diagnosis?
/ A. Cervical carcinoma
/ B. Cervical dysplasia
/ C. Dysfunctional uterine bleeding
/ D. Dysfunctional uterine bleeding
/ E. Turner syndrome

Explanation - Q: 3.1 Close

The correct answer is C. This patient most likely has dysfunctional uterine
bleeding. Other, potentially more serious, diseases have been mostly
excluded by the patient's extensive evaluation (more than many patients
receive in this setting).
Cervical dysplasia or carcinoma (choices A and B) usually produce
abnormal Pap smears, and sometimes produce an abnormal vaginal
examination.
Endometrial carcinoma (choice D) would usually be evident in either the
endometrial biopsy or on the pelvic ultrasound examination.
Patients with the genetic Turner syndrome (choice E) do not menstruate.


Question 2 of 3
In most patients with this patient's disease, the condition is etiologically related to
which of the following?
/ A. Anovulatory cycles
/ B. High androgen levels
/ C. High progesterone levels
/ D. Persistent corpus luteum
/ E. Persistent secretory endometrium

Explanation - Q: 3.2 Close

The correct answer is A. Approximately 90% of patients with dysfunctional
bleeding have anovulatory menstrual cycles. In these patients, the failure of
ovulation leads to a failure of the corpus luteum to form (compare with
choice D), and consequently a failure of normal progesterone secretion
(compare with choice C) and a failure to develop secretory endometrium
(compare with choice E). Estrogen levels remain high but androgen levels
(choice B) do not usually play a significant role. The unopposed estrogen
secretion stimulates endometrial proliferation, and the thickened
endometrium eventually outgrows its blood supplies and begins to die,
producing the prolonged menstrual blood flow. In the remaining 10% of
patients who have dysfunctional uterine bleeding and ovulatory cycles, the
prolonged progesterone secretion is associated with irregular endometrial
shedding. Some of these latter patients also have polycystic ovary disease
and may have abnormal androgen metabolism.


Question 3 of 3
The physician decides to treat this patient with an oral drug that will stop
endometrial growth and support and organize the endometrium to
allow organized sloughing on the next menstrual cycle after withdrawal of the
drug. Which of the following is the most appropriate
pharmacotherapy?
/ A. Estrogens, conjugated
/ B. GIyburide
/ C. Hydralazine hydrochloride
/ D. Levothyroxine
/ E. Medroxyprogesterone acetate

Explanation - Q: 3.3 Close

The correct answer is E. Medroxyprogesterone acetate (Provera) is the
drug of choice for most patients with anovulatory dysfunctional uterine
bleeding. It does not control the acute episode of bleeding, but it does allow
the establishment of a more stable endometrium, and the bleeding following
the withdrawal of the Provera usually has a more normal character.
Conjugated estrogens (choice A) are also used to treat dysfunctional uterine
bleeding, and in pharmacologic doses, cause the rapid growth of endometrial
tissue over a denuded epithelial surface.
Glyburide (choice B) is a sulfonylurea drug that is used as an anti-diabetic
agent.
Hydralazine (choice C) is an antihypertensive.
Levothyroxine (choice D) is used to replace thyroid hormone.


A 7-year-old Caucasian girl is evaluated by a pediatrician because she has just
begun to menstruate. The mother states that her daughter
started developing breasts at age five.


Question 1 of 6

In the normal physiology of puberty, which of the following is the earliest change
seen, indicating that puberty is beginning physiologically?
/ A. Appearance of pulsatile LH release during sleep
/ B. Circulating FSH Ievels increase progressively
/ C. Difference between sleeping and waking LH secretory patterns disappears
/ D. Increase in circulating estradiol
/ E. Serum prolactin concentrations increase modestly

Explanation - Q: 4.1 Close

The correct answer is A. While the physical changes of puberty are most
obvious to the observer, puberty progresses under the control of a variety of
hormones. The earliest measurable change to herald the onset of puberty is
a pulsatile release of LH (luteinizing hormone, from the pituitary) during
sleep. It has been speculated that this is linked to a pulsatile release of
gonadotropin releasing hormone (GnRH,) from the hypothalamus. As
puberty continues, these LH peaks increase in frequency and amplitude, and
also begin to be found in the waking period, until by the end of puberty, the
waking and sleeping patterns of LH release are very similar (choice C). The
response of LH to GnRH (gonadotropin-releasing hormone, from the
hypothalamus) also increases through puberty. FSH (follicle stimulating
hormone) levels increase less markedly than do LH levels, but do
progressively increase (choice B). The rising levels of LH and FSH also
trigger an increase in circulating estradiol (choice D), which is responsible
for the development of secondary sexual characteristics in a female. Serum
prolactin concentrations also increase modestly during puberty (choice E).

Question 2 of 6
On physical examination, the girl is noted to have large breast mounds for her
age with an approximately 4 inch diameter and 1 1/2 inch high
palpable breast mound. No secondary mound is seen. This child's breasts should
be classified as which of the following Tanner breast
stages?
/ A. Stage 1
/ B. Stage 2
/ C. Stage 3
/ D. Stage 4
/ E. Stage 5

Explanation - Q: 4.2 Close

The correct answer is C. Sexual development is usually classified in terms
of the Tanner stages. This child has tanner stage 3 breasts.
Tanner stage 1 breasts (choice A) are preadolescent breasts in which only
the papillae are elevated.
Tanner stage 2 breasts (choice B) would be smaller than this child's breast
and have an elevated bud and papilla with a small mound and increased
areolar diameter.
Tanner stage 3 breasts (choice C), such as this child has, are larger than
stage 2 breasts and have increased palpable glandular tissue.
Tanner stage 4 breasts (choice D) differ from stage 3 breasts in that the
areola and papilla are elevated to form a second mound above the level of
the rest of the breast.
Tanner stage 5 breasts (choice E) are mature adult breasts (and much
larger than stage 3 breasts), in which the areola recesses to the mound of
breast tissue, with projection of only the papilla being evident.


Question 3 of 6
The child's pubic area shows a small area of moderately dark, slightly coarse,
curly hair along the labia to the pubic junction. The hair is more
mature than vellous hair, but does not have a completely adult texture. This
child's pubic hair should be classified as which of the following
Tanner pubic hair stages?
/ A. Stage 1
/ B. Stage 2
/ C. Stage 3
/ D. Stage 4
/ E. Stage 5



Explanation - Q: 4.3 Close

The correct answer is C. This child's pubic hair is also in Tanner stage 3.
Stage 1 (choice A, preadolescent) is vellus hair only (similar to that on the
anterior abdominal wall).
Stage 2 (choice B) shows sparse growth of long, slightly pigmented, downy
hair. These hairs develop along the labia.
Stage 3 (choice C), this child's stage, is characterized by darker, coarser, a
little curled, and is found up to the pubic junction.
Stage 4 (choice D) is adult-type hair, but with a smaller distribution than in
most adults, and with no spread to the medial surface of the thighs.
Stage 5 (choice E) is characterized by adult-type hair distributed as an
inverse triangle with spread to the medial surface of the thighs.



Question 4 of 6

Which of the following is the most likely diagnosis?
/ A. Polycystic ovary syndrome
/ B. Precocious puberty
/ C. Primary amenorrhea
/ D. Secondary amenorrhea
/ E. Turner syndrome

Explanation - Q: 4.4 Close

The correct answer is B. Caucasian girls usually begin to develop breasts,
and then pubic hair between the ages of 8 and 13 years. Most girls beginning
menstruating at 12 to 13 years of age. This child has an early pubarche
(beginning of genital hair changes), an early thelarche (beginning of breast
development), and an early menarche (beginning of menstruation),
consistent with precocious puberty. Precocious puberty is the term used for
the appearance of secondary sexual maturation in Caucasian girls younger
than 7 years, African-American girls younger than 6 years, or boys younger
than 9 years. Precocious puberty is much more common in girls than in boys.
Approximately 80% of cases of precocious puberty in the United States are
due to a premature activation of the hypothalamic-pituitary-gonadal axis. The
term "central precocious puberty" is sometimes used for these cases, which
are contrasted with cases of "precocious pseudopuberty" [also known as
"gonadotropin-independent precocious puberty" and still classified as
"precocious puberty" by many authors] in which the sexual development
occurs as a consequence of other disease.
Polycystic ovary syndrome (choice A) can be associated with virilism in adult
women, but is not associated with precocious puberty.
Amenorrhea refers to a failure to menstruate, either never (primary
amenorrhea, choice C) or after having previously started (secondary
amenorrhea, choice D).
Turner syndrome (choice E) has XO genetics and is associated with primary
amenorrhea.


Question 5 of 6
Which of the following would be the most appropriate pharmacotherapy?
/ A. Epoetin
/ B. Dapsone
/ C. GIycopyrrolate
/ D. Leuprolide
/ E. Pindolol

Explanation - Q: 4.5 Close

The correct answer is D. Precocious puberty is usually medically treated,
primarily to prevent social ostracization of the child and also to allow time for
the patient's height to reach near normal to normal adult height (growth in
stature often ends when puberty does). The mainstay of therapy is now the
gonadotropin-releasing hormone agonists, including leuprolide and nafarelin.
These medications successfully suppress LH and FSH levels (and thus
ovarian and testicular steroidogenesis) because the pituitary responds to
pulsatile GnRH cues, rather than constant ones.
Epoetin (choice A) is the pharmacologic name for erythropoietin, used to
stimulate red cell growth.
Dapsone (choice B) is an antimicrobial agent used in leprosy.
Glycopyrrolate (choice C) is an anticholinergic medication used to suppress
GI secretions.
Pindolol (choice D) is an antihypertensive agent.







Question 6 of 6

In some children, a similar condition can be part of a syndrome that is also
characterized by polyostotic fibrous dysplasia and caf-au-Iait skin
pigmentation. Which of the following is the most likely diagnosis in these
individuals?
/ A. CREST syndrome
/ B. Crigler-Najjar syndrome
/ C. Cushing syndrome
/ D. Dandy-Walker syndrome
/ E. McCune-AIbright syndrome

Explanation - Q: 4.6 Close

The correct answer is E. McCune-Albright syndrome is a rare syndrome
characterized by fibrous dysplasia (a type of bony defect), caf-au-lait skin
pigmentation (also seen in neurofibromatosis), and autonomous endocrine
hyperfunction. The endocrine abnormalities most commonly take the form of
gonadotropin-independent precocious puberty, but may also include
hyperthyroidism, hypercortisolism, and pituitary gigantism. These patients
are also vulnerable to chronic liver disease and sudden death (possibly
related to cardiac arrhythmias).
CREST syndrome (choice A) is a variant of scleroderma with calcinosis,
esophageal motility problems, telangiectasias, Raynaud's phenomenon, and
sclerodactyly.
Crigler-Najjar syndrome (choice B) is a hereditary defect in bilirubin
metabolism.
Cushing syndrome (choice C) refers to the physical problems seen as a
result of hypercortisolism.
Dandy-Walker syndrome (choice D) is a congenital malformation of the
brain.

A 47-year-old man presents to a psychiatrist with signs and symptoms of
depression. His son, who brought him in, said that he has become absent-
minded, and has difficulty concentrating or remembering things. During the
session, the patient makes sudden jerky movements with his hands and
grimaces frequently. The psychiatrist also notes that the patient has problems
with coordination and balance, and observes stuttering and slightly slurred
speech during the interview. The man's mother is a healthy, 75-year-old retired
saleswoman; his father committed suicide at the age of 60.

What are the major characteristics of this patient's disease?
/ A. Anterograde amnesia, ataxia, ophthalmoplegia
/ B. Autosomal dominant, choreiform movements, and behavioral changes
/ C. Autosomal recessive, progressive dementia, hyperoral behavior
/ D. Progressive memory loss, progressive aphasia, impaired executive function
/ E. Tremor, rigidity, bradykinesia


Explanation - Q: 1.1 Close

The correct answer is B. This patient suffers from Huntington disease
(chorea). Huntington disease (HD) is a progressive neurodegenerative
disease primarily affecting neurons in the basal ganglia. HD affects about 1
in 10,000 individuals, and is transmitted in an autosomal dominant fashion.
Its average onset is from 30-40 years of age, and the duration of the disease
is typically about 15 years, but shows a wide range. Early onset (juvenile HD)
is associated with more rapid progression, rigidity, and ataxia. The main
motor features of HD are irregular, sudden limb and facial jerks. The disease
may begin with "piano-playing" movements of the fingers or slight facial
twitching, but symptoms are slowly progressive and become uncontrollable.
The speech slowly becomes incomprehensible and swallowing is difficult.
The gait is poorly coordinated, with a dancing-like (choreiform) pattern.
Although patients appear to be off-balance, the ability to balance is actually
well-preserved. Depression, apathy, social withdrawal, and irritability are
common. Memory is typically affected at later stages of the disease. Since
HD is incurable, the ultimate result is death. The family history is relevant,
since with autosomal dominant inheritance, the chance of inheriting the
Huntington gene is 50%. The patient's mother does not have the disease, so
the patient's father must have had it. A history of family members who
commit suicide in middle-age is often associated with HD.
Wernicke-Korsakoff syndrome is due to profound thiamine deficiency.
Initially, confusion, ataxia, and ophthalmoplegia dominate the clinical picture,
but if the thiamine deficiency is not corrected, an amnestic syndrome with
predominantly anterograde amnesia supervenes (choice A).
Autosomal recessive inheritance, progressive dementia, hyperoral behavior,
emotional disinhibition, and language disturbances are clinical features of
Pick disease (choice C).
Alzheimer disease is the most common cause of dementia, in which
cognitive and behavioral impairment interfere with social and occupational
functioning. Although, other neurological systems can be affected (e.g.,
impaired visuospatial skills and spastic paraparesis), as the disease
progresses, the most prominent feature is progressive memory loss (choice
D).
Parkinson disease is characterized by a combination of tremor, rigidity,
bradykinesia, and a distinctive disturbance of gait and posture (choice E).


Where are the primary sites of cell degeneration in this patient's disease?
/ A. Basal ganglia and thalamus
/ B. Frontal and temporal cortical regions
/ C. Neostriatum and cerebral cortex
/ D. Substantia nigra pars compacta and locus coeruleus
/ E. Upper and lower motor neurons


Explanation - Q: 1.2 Close

The correct answer is C. Cell death in the caudate and putamen
(neostriatum) produces the chorea. Release of the globus pallidus from
striatal inhibition results in suppression of subthalamic activity, which may
also contribute to the choreiform movements. Impaired cognitive function and
ultimate dementia is caused by the loss of cerebral cortical cells, and
possibly, by disruption of the cognitive portions of the basal ganglia. Atrophy
of the caudate nuclei can be seen using neuroimaging studies in the mid-
and late stages of the disease, while more diffuse cortical atrophy is visible
late in the disease. Microscopically, there are no significant pathologic
features other than some gliosis. Histochemically, there is a marked
decrease of GABA and its synthetic enzyme glutamic acid decarboxylase
throughout the basal ganglia. The levels of other transmitters, including
substance P and enkephalins, are also decreased.
Pathologic changes of cavitary degeneration in the basal ganglia with
extensive gliosis and neuronal loss, in association with a marked decrease of
the copper content in this region of the brain are characteristic of Wilson
disease. Similar changes may be seen in the thalamus (choice A).
Pick disease is defined pathologically by severe atrophy, neuronal loss, and
gliosis. Swollen neurons and argentophilic neuronal inclusions (Pick bodies)
affect the frontal and temporal cortical areas (choice B).
In Parkinson disease, loss of substantia nigra pars compacta neurons leads
to the depletion of dopamine in the striatum, which ultimately decreases
thalamic excitation of the motor cortex. Pigmented neurons of the locus
coeruleus also degenerate (choice D).
Amyotrophic lateral sclerosis is ultimately a diffuse disease, but the onset is
often focal and asymmetric. At the onset, bulbar motor neurons can be
involved, or spinal anterior horn cells can be affected. Later, spread to other
motor areas produces the classic combination of upper and lower motor
neuron degeneration (choice E).


Which of the following is the most appropriate pharmacotherapy for this patient?
/ A. Acetylcholinesterase inhibitors
/ B. Anticholinergics
/ C. Dopamine antagonists
/ D. Dopamine precursors
/ E. Thiamine


Explanation - Q: 1.3 Close

The correct answer is C. Although, no therapy is currently available to
delay the onset of symptoms or to prevent the progression of the disease,
symptomatic treatment of HD patients may improve the quality of life and
prevent complications. Dopamine antagonists (antipsychotic agents) are
used for the psychosis, paranoia, and delusional states that occur in HD,
although they are used in lower doses than is often required for primary
psychiatric disorders. Antipsychotic agents may also improve choreic
movements in patients with HD. HD patients on antipsychotics have to be
monitored, because the drugs can worsen the rigidity seen in these patients.
Another approach is to decrease levels of dopamine via reserpine, although
patients need to be monitored for hypotension and depression.
Tetrabenazine, another dopamine-depleting agent, seems to be the most
effective suppressant of choreiform movements, but this drug is not available
in the U.S. and is categorized as investigational. Other possibly useful drug
treatments in HD are SSRIs and carbamazepine for depression.
Centrally acting acetylcholinesterase inhibitors (e.g., rivastigmine,
galantamine, donepezil, tacrine) (choice A) are used in the treatment of
Alzheimer disease.
Anticholinergics (choice B), such as benztropine or trihexyphenidyl, are
used in Parkinson disease.
L-dopa, a dopamine precursor (choice D), is used in Parkinson disease.
Thiamine (choice E) is used in Wernicke-Korsakoff syndrome.



Which of the following genes is involved in the pathogenesis of this disease?
/ A. ATP7B, Iocated on chromosome 13
/ B. D23116, Iocated on chromosome 2q33-q34
/ C. IT15, Iocated on chromosome 4p
/ D. N-acetyI-transferase 2, Iocated on chromosome 8
/ E. 6p21, Iocated on chromosome 6



Explanation - Q: 1.4 Close

The correct answer is C. Huntington disease is a genetic disorder of the
central nervous system, and is inherited as an autosomal dominant
condition. In affected individuals, one gene of the gene pair is not functioning
properly and "dominates" the other working gene. The HD gene, called IT15,
is located on chromosome 4p.
The gene responsible for Wilson disease is located on chromosome 13, and
it is called ATP7B (choice A).
Many familial cases of amyotrophic lateral sclerosis are linked to the
mutations in the gene region on chromosome 2 (choice B) that codes for
superoxide dismutase.
The gene that has the strongest connection with Parkinson disease is N-
acetyl-transferase 2 located on chromosome 8 (choice D), which codes for
the enzyme responsible for degradation of toxins in the body.
Region 6p21 on chromosome 6 (choice E) has been strongly linked to
multiple sclerosis.



DNA samples from the patient and his unaffected mother are amplified by the
polymerase chain reaction (PCR) using primers specific for the 5' coding region
of the gene involved in his disease. Gel electrophoresis of their PCR products is
shown on the left in the diagram.


If DNA from his deceased father had been available for testing, which option
represents the most likely PCR pattern of the father?
/ A. A
/ B. B
/ C. C
/ D. D
/ E. E


Explanation - Q: 1.5 Close

The correct answer is A. Huntington disease is caused by the rapid
expansion of a triplet repeat (CAG) in the 5' coding region of the IT15 gene
for huntingtin. The normal huntingtin allele has 5 adjacent CAG codons
encoding 5 adjacent glutamine residues in the protein. In families with
Huntington disease, the CAG sequence has expanded somewhat with each
generation, and the polyglutamine tract in the huntingtin protein is longer.
One would most likely expect the expanded CAG sequence to be shorter in
the father than in his son, and the corresponding PCR-amplified fragment to
migrate faster (lower on the gel) during electrophoresis.
Choices B, C, D, and E might represent results of a paternity test to
determine whether the man was his father. In this case, the PCR products
would be generated to amplify a region with a known polymorphic marker,
often a short tandem repeat (microsatellite). The patterns (B and C) would
be consistent with the man as father, while patterns (D and E) would be
inconsistent with the man being his father.


The class of mutation causing this disease is often associated with a
characteristic feature in a family pedigree. Which of the following is most likely to
be a characteristic of pedigrees caused by this class of mutation?
/ A. AIIelic heterogeneity
/ B. Anticipation
/ C. Consanguinity
/ D. Imprinting
/ E. Multifactorial inheritance


Explanation - Q: 1.6 Close

The correct answer is B. In diseases caused by triplet repeat expansions,
the symptoms worsen and the age of onset decreases in successive
generations, a characteristic known as anticipation. Important triplet repeat
expansion diseases include Huntington disease, Fragile X syndrome, and
myotonic dystrophy. In each, the pedigree typically shows anticipation.
Allelic heterogeneity (choice A) indicates that different types of mutations in
a gene cause a particular disease. For example, there are over 400
mutations in the glucose 6-phosphate dehydrogenase gene known to cause
some form of hemolytic anemia. Each family would generally have only one
of these mutations. All cases of Huntington disease are caused by a triplet
repeat expansion in the same 5' locus.
Consanguinity (choice C) is a more common feature of autosomal recessive
diseases, in which two copies of an affected gene must be inherited to
produce the disease phenotype. This is more likely when there are
incestuous or consanguineous (between first or second cousins) matings.
Genetic imprinting (choice D) occurs when a gene is normally expressed
only from either the paternally inherited, or from the maternally inherited
chromosome. In such a case, if the normally active gene is deleted from the
chromosome, a disease such as Prader-Willi (deletion in paternal 15) or
Angelman syndrome (deletion in maternal 15) may occur. In Huntington
disease, for unknown reasons, the triplet usually expands more rapidly when
the affected chromosome is passed through a male, but this is not imprinting.
In a family with Huntington disease, and in normal families, both IT15 alleles
on the maternal and the paternal chromosomes are active (expressed). The
preferential expansion of the triplet in Fragile X is through the mother, and in
the more severe forms of myotonic dystrophy, through the mother.
Multifactorial inheritance (choice E) refers to a disease in which there are
many factors contributing to phenotypic disease. Multiple genes may be
involved, as well as nongenetic environmental factors. Coronary artery
disease and cancer would be examples of multifactorial diseases. Huntington
disease is solely attributed to the triplet repeat expansion in the IT15 gene.


The patient is sent for imaging studies. Which of the following will most likely be
found?
/ A. Decrease in 18F-dopa uptake
/ B. Diffuse corticaI/cerebral atrophy
/ C. Enlarged lateral ventricles
/ D. Hypodensities in the putamen
/ E. T2 hyperintensities


Explanation - Q: 1.7 Close

The correct answer is C. Enlarged, dilated lateral ventricles are visible on
head CT scan. CT scan or MRI may reveal loss of the normally convex
prominence of the caudate nucleus into the lateral ventricles. Although no
single imaging technique is necessary for diagnosis, in patients with typical
symptoms, a CT scan strongly supports the diagnosis of Huntington disease.
A reliable imaging marker of this disease is measurement of the bicaudate
diameter by MRI or CT scan. Other imaging studies e.g., positron emission
tomography (PET) scanning and proton magnetic resonance spectroscopy
may show abnormalities, but they are used very rarely as a part of the
workup for HD.
PET decrease in
18
F-dopa uptake (choice A) in the contralateral putamen is
characteristic for Parkinson disease.
Diffuse cortical/cerebral atrophy (choice B) is seen on brain MRI in patients
with Alzheimer disease.
CT of the head showing hypodensities in the putamen (choice D) is seen in
patients with Wilson disease.
Typical multiple sclerosis lesions appear as T2 hyperdensities (choice E) in
the periventricular regions.



A 63-year-old man experiences the sudden onset of a right facial droop and an
inability to speak, while talking to his niece.
His niece, who is a physician, rushes him to the local emergency department. On
arrivaI, his vital signs are within normal Iimits. On examination, he cannot speak,
and he is noted to have a marked right facial droop and right arm weakness. An
emergent head CT scan does not reveal any abnormalities.


This patient's disease most likely involves which of the following arteries?
/ A. Left anterior cerebral artery
/ B. Left middle cerebral artery
/ C. Left posterior cerebral artery
/ D. Right posterior inferior cerebellar artery
/ E. Right superior cerebellar artery


Explanation - Q: 2.1 Close

The correct answer is B. The middle cerebral artery supplies the lateral
convexity of the cerebral hemisphere and the underlying insula. It supplies
both Broca's and Wernicke's speech areas in the dominant hemisphere. It
also supplies the face, arm, and trunk areas of the motor and sensory
homunculus. Occlusion can result in right-sided face and arm weakness and
aphasia, as seen in this patient.
An occlusion of the left anterior cerebral artery (choice A) would result in
right lower extremity weakness. The anterior cerebral artery supplies the
medial surfaces of the frontal and parietal lobes, as well as the corpus
callosum.
An occlusion of the left posterior cerebral artery (choice C) would result in a
contralateral hemiparesis and a right homonymous hemianopsia. The
posterior cerebral artery supplies the midbrain, the posterior half of the
thalamus, the occipital lobe, and the inferior surface of the temporal lobe.
An occlusion of the right posterior inferior cerebellar artery (choice D) would
result in a lesion in the lateral tegmentum of the medulla, called a Wallenberg
syndrome (lateral medullary syndrome). This is characterized by ipsilateral
palsies of cranial nerves V (ipsilateral hemifacial numbness), IX (ipsilateral
decreased gag and taste), X (dysphagia and hoarseness), and XI, as well as
an ipsilateral Horner syndrome, cerebellar ataxia, and nystagmus. There
would also be a loss of pain and temperature sense over the contralateral
body.
Occlusion of the right superior cerebellar artery (choice E) results in a lateral
superior pontine syndrome. This is characterized by ipsilateral ataxia (due to
lesions of the superior and middle cerebellar peduncles), dysmetria (due to
lesion of the dentate nucleus), contralateral loss of pain and temperature
(due to lesions in the spinothalamic and trigeminothalamic tracts), ipsilateral
Horner syndrome, and contralateral loss of proprioception and vibration
sense in the trunk and leg.


Which of the following would the patient most likely be able to do on further
evaluation?
/ A. Name objects
/ B. Raise his eyebrows symmetrically
/ C. Repeat words
/ D. Stick out his tongue without deviation
/ E. Write words


Explanation - Q: 2.2 Close

The correct answer is B. Corticobulbar fibers have bilateral input to the
upper face division of the facial nucleus, while the lower face division of the
facial nucleus has contralateral input. Therefore, an upper motor neuron
lesion involving these corticobulbar fibers, such as a cortical stroke, would
result in weakness of the lower half of the contralateral face, while sparing
the upper half of the contralateral face due to this bilateral input. A lower
motor neuron lesion involving the facial nucleus or the facial nerve (i.e., a
Bell's palsy) would result in weakness of the entire ipsilateral half of the face.
The ability to name objects (choice A) requires an intact Broca's area, which
seems to be involved in the patient's stroke, since he is unable to speak.
The ability to repeat words (choice C) requires an intact Wernicke's area,
Broca's area, and arcuate fasciculus, any one or all of which would be
involved in a left-sided middle cerebral artery distribution stroke. The patient
described above clearly has difficulty speaking, and would most likely be
unable to repeat words.
The ability to stick out his tongue without deviation (choice D) would most
likely not be preserved in this patient with a stroke resulting in contralateral
face and arm weakness. The tongue area is adjacent to the face area on the
motor homunculus, and a lesion affecting one would most likely affect both.
This would result in an upper motor lesion, and would cause the tongue to
deviate away from the side of the lesion.
The ability to write words (choice E) requires an intact Broca's area, which
seems to be involved in the patient's stroke, since he is unable to speak.


In the emergency department, it is decided that the patient is a candidate for
emergent pharmacologic thrombolysis.
Which of the following is the mechanism of action of this treatment?
/ A. Activates antithrombin lll
/ B. Facilitates the conversion of plasminogen to plasmin
/ C. Impairs the synthesis of vitamin K-dependent clotting factors
/ D. Irreversibly inhibits cyclooxygenase
/ E. Irreversibly inhibits the ADP pathway involved in fibrinogen binding


Explanation - Q: 2.3 Close

The correct answer is B. Thrombolytics, such as t-PA and urokinase, aid in
the conversion of plasminogen to plasmin, which then goes on to cleave
thrombin and fibrin clots and actively lyse clots. It has been shown that giving
tPA within 3 hours of the onset of a stroke can improve outcome and is
approved by the US Food and Drug Administration for treating acute
ischemic stroke within 3 hours of onset of symptoms.
Activation of antithrombin III (choice A) is the mechanism of action of
heparin, which is an anticoagulant, not a thrombolytic. It prevents the
formation of new blood clots, but does not actively lyse formed clots. It is
given parenterally, and has a rapid onset of action.
Impairing the synthesis of vitamin K-dependent clotting factors (choice C) is
the mechanism of action of warfarin. Warfarin is an anticoagulant, not a
thrombolytic. It prevents the formation of new blood clots, but does not
actively lyse formed clots. Unlike heparin, warfarin is given orally, and has a
slow onset of action.
Irreversibly inhibiting cyclooxygenase (choice D) is the mechanism of action
of aspirin. This inhibits platelet aggregation and prevents clot formation. It
has a slow onset of action, and does not actively lyse formed clots.
Irreversibly inhibiting the ADP pathway involved in fibrinogen binding (choice
E) is the mechanism of action of ticlopidine. It inhibits platelet aggregation
and prevents clot formation. It has a slow onset of action and does not
actively lyse formed clots.


After emergent pharmacologic thrombolysis, the patient's symptoms quickly
resolve. However, he begins complaining of a headache and nausea. A head CT
reveals a 2 cm x 2 cm blood clot in the patient's right cerebellar hemisphere.
What would be the best therapy to reverse the patient's coagulopathy?
/ A. Aminocaproic acid
/ B. Dialysis
/ C. PIatelets
/ D. Protamine
/ E. Vitamin K


Explanation - Q: 2.4 Close

The correct answer is A. Aminocaproic acid is useful in enhancing
hemostasis when fibrinolysis contributes to bleeding. The antifibrinolytic
effects of aminocaproic acid appear to be exerted principally via inhibition of
plasminogen activators, and to a lesser degree through antiplasmin activity.
Dialysis (choice B) would not correct the thrombolytic effect on fibrinolysis.
Platelets (choice C) would not correct the thrombolytic effect on fibrinolysis.
Protamine (choice D) is used to reverse the anticoagulant action of heparin,
which is not a thrombolytic. Protamine combines ionically with heparin to
form a stable complex devoid of anticoagulant activity.
Vitamin K (choice E) is the antidote for warfarin toxicity.

What would be the most likely pronounced neurologic deficits that would be
observed in this patient with a right cerebellar hemorrhage?
/ A. Contralateral dyscoordination
/ B. Contralateral hemiparesis
/ C. Ipsilateral dyscoordination
/ D. Ipsilateral hemiparesis
/ E. Truncal ataxia


Explanation - Q: 2.5 Close

The correct answer is C. An ipsilateral cerebellar lesion causes ipsilateral
dyscoordination because it sends efferents from the dentate nucleus to the
contralateral cerebral cortex by way of crossed fibers.
Contralateral dyscoordination (choice A) would not result from a hemispheric
cerebellar lesion.
Contralateral hemiparesis (choice B) would not result from a hemispheric
cerebellar lesion. It could result from an upper motor neuron lesion proximal
to the medullary decussation.
Ipsilateral hemiparesis (choice D) would not result from a hemispheric
cerebellar lesion. It could result from an upper motor neuron lesion distal to
the medullary decussation.
Truncal ataxia (choice E) would be seen with a vermian lesion.



A 15-year-old boy is taken to a psychiatrist because he has had a recent sudden
deterioration in his schoolwork and a recent marked increase in aggressive
behavior. The psychiatrist performs a physical examination that demonstrates a
mild hand tremor, partial loss of the gag reflex, and impaired coordination. He
additionally demonstrates hepatomegaly and mild jaundice.

Which of the following diseases would be most likely to cause both hepatic and
neurologic dysfunction?
/ A. AIzheimer disease
/ B. Huntington disease
/ C. Parkinson disease
/ D. Sydenham chorea
/ E. Wilson disease



Explanation - Q: 3.1 Close

The correct answer is E. All of the diseases listed have prominent
neurologic manifestations, but only Wilson disease also usually affects the
liver. Wilson disease is an autosomal recessive disease with an incidence of
1 in 30,000 that commonly presents in adolescence. The patient may come
to medical attention because of psychiatric symptoms of all degrees (school
performance deterioration, suicidal or homicidal impulses, depression, and
rarely, illness mimicking schizophrenia or manic-depressive illness),
neurologic dysfunction (tremors, difficulty walking, difficulty talking, or
difficulty swallowing), or hepatic problems (jaundice, hepatomegaly, or
abdominal pain). The metabolic basis of Wilson disease is an abnormality of
copper metabolism, which leads to accumulation of toxic amounts of copper.
One of the sites where copper accumulates is the liver, and biopsy of liver
followed by chemical analysis may show copper concentrations greater than
250 micrograms/gram dry weight.
Alzheimer disease (choice A) characteristically causes mental deterioration
in the elderly.
Huntington disease (choice B) is an autosomal dominant disease with
intellectual deterioration and movement disorder that characteristically
manifests in middle age.
Parkinson disease (choice C) is an idiopathic progressive disorder with
decreased ability to move and tremor, which usually manifests in late middle-
aged or older individuals.
Sydenham chorea (choice D) is a motor disorder that can complicate
rheumatic fever.


Measurements of the serum level of which of the following would be most likely
to contribute to establishing the patient's diagnosis?
/ A. Carcinoembryonic antigen
/ B. Ceruloplasmin
/ C. Ferritin
/ D. Lead
/ E. Myelin basic protein


Explanation - Q: 3.2 Close

The correct answer is B. Ceruloplasmin is a copper-containing serum
enzyme that is present in diminished levels in both patients with, and carriers
of Wilson disease. Since the enzyme can be deficient in both heterozygotes
(with a carrier frequency of 1 in 90 and no progression to clinical disease)
and homozygotes, the establishment of deficient ceruloplasmin does not, in
itself, establish the diagnosis. However, the diagnosis can be established if
this information is taken together with other evidence of abnormal copper
metabolism, such as elevated liver copper, elevated urinary excretion of
copper, or the presence of Kayser-Fleischer rings on ocular examination.
Carcinoembryonic antigen (choice A) is a tumor marker that is most often
clinically useful in following patients with colon or lung cancer.
Ferritin (choice C) is an iron-protein complex used to transport iron from the
gastrointestinal tract to the tissues, and may be markedly elevated in
hemochromatosis.
Lead (choice D) poisoning can cause cognitive defects in children, and
encephalopathy and progressive renal disease in adults. Tremor, such as
this patient has, it not usually a feature.
Myelin basic protein (choice E) is characteristically elevated in CSF fluid in
patients with multiple sclerosis.

Which of the following findings on ocular examination would be considered most
specific for this patient's probable disease?
/ A. Corneal ulcers
/ B. Kayser-FIeischer rings
/ C. Macular degeneration
/ D. Papilledema
/ E. Retinal detachment



Explanation - Q: 3.3 Close

The correct answer is B. Kayser-Fleischer rings are deposits of copper in
Descemet's membrane of the cornea, which can be seen as gold or
greenish-gold rings and crescents on slit-lamp ophthalmologic examination.
They are considered pathognomic for Wilson disease, and the disease is
sometimes picked up initially by an ophthalmologist who notes the presence
of the characteristic rings.
Corneal ulcers (choice A) may be due to trauma, autoimmune disease, or
infection of the eye.
Macular degeneration (choice C) is a common aging problem of the eye.
Papilledema (choice D) suggests increased intracranial pressure.
Retinal detachment (choice E) can be due to trauma or hemorrhage behind
the retina.


Which of the following pharmacologic agents would be useful to this patient by
decreasing intestinal absorption of the substance causing this patient's disease?
/ A. Deferoxamine
/ B. Dimercaprol
/ C. Penicillamine
/ D. Trientine
/ E. Zinc acetate


Explanation - Q: 3.4 Close

The correct answer is E. Zinc acetate has been widely used elsewhere in
the world, and recently the FDA approved it for use in the United States. It
induces an intestinal metallothionein, which binds copper, and prevents its
absorption into the blood stream. It appears to be an important new addition,
with fewer side effects than chelating agents, for the chemotherapy of Wilson
disease.
Deferoxamine (choice A) is used to chelate iron.
Trientine (choice D), penicillamine (choice C), and dimercaprol (choice B)
are chelating agents that increase copper excretion.




A 58-year-old left-handed man is referred to a neurologist for "involuntary left
hand twitches." Between six months and a year ago, he first noticed that when
his left hand was resting, it would shake. He can stop the shaking by looking at
his hand and concentrating. The shaking does not impair his activities in any
way. He has no trouble holding a glass of water.
There is no tremor in his right hand and his lower extremities are not affected. He
has had no trouble walking and there have been no falls. There have been no
behavioral or language changes. On examination, a left hand tremor is evident
when he is distracted. Handwriting appears small and cramped. He has bilateral
cogwheel rigidity with contralateral activation, which is worse on the left. Rapid
alternating movements are bradykinetic on the left.


Which of the following is the most likely diagnosis?
/ A. Epilepsy
/ B. Guillain-Barr syndrome
/ C. Multiple sclerosis
/ D. Parkinson disease
/ E. Stroke



Explanation - Q: 4.1 Close

The correct answer is D. The tremor is of a Parkinsonian type. He also has
the classic findings of Parkinson disease (PD), which are asymmetric tremor,
rigidity, and bradykinesia. Mask-like facies, drooling, tremors, pill-rolling
motion, cogwheel rigidity, and a shuffling gait all may be present. This
disease results from the degeneration of the dopaminergic neurons in the
substantia nigra pars compacta of the midbrain.
Epilepsy (choice A) is characterized by repeated unprovoked seizures.
Hand shaking can be a focal motor seizure, but the presentation overall
makes epilepsy an unlikely diagnosis.
Guillain-Barr syndrome (choice B) is a peripheral demyelinating disease,
which usually presents as an ascending motor deficit.
Multiple sclerosis (choice C) is a central nervous system (CNS)
demyelinating disease. It presents with individual episodes of CNS deficits,
which usually recover to some extent.
Stroke (choice E) is characterized by the acute onset of a neurological
deficit and is due to infarction of neural tissue. Tremor would be an
exceedingly rare presentation for stroke, and it would not evolve over 6-12
months.


Which of the following would be the most appropriate pharmacotherapy for this
patient?
/ A. AIteplase
/ B. Carbamazepine
/ C. Carbidopa-Ievodopa
/ D. GIatiramer
/ E. Interferon beta-1A
/ F. Sertraline


Explanation - Q: 4.2 Close

The correct answer is C. Parkinson disease symptoms are due in large part
to dopamine depletion. Carbidopa-levodopa can replete dopamine and
alleviate symptoms. Levodopa is a precursor to dopamine that is
decarboxylated by dopa-decarboxylase. Inhibition of the peripheral form of
this enzyme by carbidopa allows greater amounts of the precursor to reach
the brain, where it is converted to dopamine by the uninhibited brain
decarboxylase.
Alteplase (recombinant tPA; choice A) is used to dissolve blood clots during
acute strokes or heart attacks.
Carbamazepine (choice B) is an anticonvulsant.
Glatiramer (choice D) and interferon beta-1A (choice E) are used to treat
multiple sclerosis. They have been shown to decrease attacks. Both are
thought to work through immunomodulation.
Sertraline (choice F) is a selective serotonin reuptake inhibitor (SSRI). By
increasing serotonin concentrations, it is effective for the treatment of
depression.


Another drug prescribed for this patient is tolcapone. Which of the following is the
mechanism of action of this agent?
/ A. COMT inhibitor
/ B. Dopamine agonist
/ C. Dopamine uptake blocker
/ D. MAO inhibitor
/ E. Muscarinic antagonist
/ F. Selective serotonin reuptake inhibitor



Explanation - Q: 4.3 Close

The correct answer is A. Tolcapone and entacapone are COMT inhibitors,
which prevent the peripheral conversion of L-Dopa to 3-O-methyldopa,
enhancing the availability of L-Dopa for transport across the blood-brain
barrier. These drugs are used as adjuncts to levodopa-carbidopa and can
prolong "on-time" and allow lower doses of L-Dopa to be used.
Dopamine agonists (choice A), such as bromocriptine, pergolide, and
pramipexole, directly stimulate postsynaptic dopamine D2 receptors to
relieve parkinsonian symptoms. These agents can be used as monotherapy
in early PD, or as adjuncts to levodopa-carbidopa in later stages of the
disease.
Amantadine works possibly by a variety of mechanisms, including dopamine
uptake blockade (choice C), enhancing dopamine synthesis or release,
stimulating postsynaptic dopamine receptors, or increasing dopamine
receptor sensitivity. It is used as an adjunctive therapy.
Selegiline is an irreversible inhibitor of MAO (choice D). At recommended
doses, it is relatively specific for MAO-B. It blocks the breakdown of
dopamine and therefore extends the duration of L-Dopa.
Muscarinic antagonists (choice E), such as trihexyphenidyl and benztropine,
are used adjunctively to alleviate the tremor and rigidity in parkinsonism; they
do not typically improve the bradykinesia.
Selective serotonin reuptake inhibitors (SSRIs, choice F) such as fluoxetine,
sertraline, citalopram, and paroxetine are antidepressants that act by
blocking serotonin reuptake.

Which of the following drugs would be contraindicated in this patient?
/ A. Aspirin
/ B. Chlorpromazine
/ C. Pergolide
/ D. Pramipexole
/ E. Ropinirole


Explanation - Q: 4.4 Close

The correct answer is B. Parkinson disease symptoms are based on
dopamine depletion. Chlorpromazine is a D2 dopamine receptor antagonist
and will worsen parkinsonian symptoms.
Aspirin (choice A) is commonly used as a pain reliever and for its anti-
platelet effect. It does not worsen parkinsonian symptoms.
Pergolide, pramipexole, and ropinirole (choices C, D, and E) are all
dopamine receptor agonists and are effective treatments for Parkinson
disease.
Which of the following enzymes is normally present in the neurons whose
degeneration leads to this patient's movement disorder?
/ A. Catecholamine- methyltransferase
/ B. Dopamine -hydroxylase
/ C. Phenylethanolamine-N-methyl transferase
/ D. Tryptophan hydroxylase
/ E. Tyrosine hydroxylase



Explanation - Q: 4.5 Close

The correct answer is E. Tyrosine hydroxylase is the rate-limiting enzyme in
catecholamine synthesis, which proceeds as follows:

The dopaminergic cells that degenerate in Parkinson disease contain
tyrosine hydroxylase and dopa decarboxylase.
Catecholamine-O-methyltransferase (choice A) is an extraneuronal enzyme
involved in the degradation of catecholamines.
Dopamine -hydroxylase (choice B) converts dopamine to norepinephrine,
and therefore would not be present in dopaminergic neurons. Instead, it is
present in noradrenergic neurons, such as those in the locus ceruleus. The
locus ceruleus, another pigmented nucleus, does degenerate in Parkinson
disease, however, this does not lead to any movement disorders.
Phenylethanolamine-N-methyl transferase (PNMT, choice C) converts
norepinephrine to epinephrine, and would be present in cells that synthesize
epinephrine, such as cells in the adrenal medulla and select neurons in the
brain.
Tryptophan hydroxylase (choice D) is the rate-limiting enzyme in the
synthesis of serotonin.


The brain structure whose degeneration leads to this patient's condition is
derived embryologically from which of the following structures?
/ A. Diencephalon
/ B. Mesencephalon
/ C. Metencephalon
/ D. Myelencephalon
/ E. Telencephalon



Explanation - Q: 4.6 Close

The correct answer is B. The substantia nigra (pars compacta)
degenerates in Parkinson disease, leading to the motor disorders of these
patients. The substantia nigra is located in the midbrain, i.e., the
mesencephalon. The mesencephalon exists in both the three- and five-
vesicle stages of neural tube development.
The diencephalon (choice A) first appears during the 5-vesicle stage of
neural tube development. The thalamus, subthalamus, hypothalamus, and
epithalamus are all parts of the diencephalon.
The metencephalon (choice C) first appears during the 5-vesicle stage of
neural tube development. It later becomes the pons and cerebellum.
The myelencephalon (choice D) first appears during the 5-vesicle stage of
neural tube development. It later becomes the medulla.
The telencephalon (choice E) first appears during the 5-vesicle stage of
neural tube development. It later becomes the cerebral hemispheres.




Which of the following structures is most likely lesioned in this patient?
/ A. A
/ B. B
/ C. C
/ D. D
/ E. E
/ F. F
/ G. G



Explanation - Q: 4.7 Close

The correct answer is A. The pigmented substantia nigra pars compacta
degenerates in this disease. This structure is located anterior to the
tegmentum within each cerebral peduncle of the midbrain (between the crus
cerebri and the tegmentum). The substantia nigra can be divided into two
parts: the more dorsally located cell-rich pars compacta, and the more
ventrally located pars reticulata. Neurons of the pars compacta contain
neuromelanin, which is a polymer derived from dopamine, giving the
substantia nigra its black appearance. The cytology, function, and
connections of the substantia nigra pars reticulata are similar to those of the
internal segment of the globus pallidus, and so these two nuclei are often
considered a single functional unit.
Choice B labels the medial lemniscus. These are ascending sensory tracts
that carry discriminative touch, vibration, and proprioception information.
Choice C labels the periaqueductal gray. It is thought to be involved in the
modulation of pain perception.
Choice D labels the superior colliculi, which are located in the tectum of the
midbrain. They play an important role in eye movements.
Choice E labels the medial geniculate body. The MGN are auditory relay
centers and are actually considered to be part of the thalamus, rather than
the midbrain.
Choice F labels the red nucleus. The red nuclei are obvious masses in the
midbrain tegmentum and are important in the outflow of information from the
cerebellum.
Choice G labels the corticospinal tract, located in the crus cerebri of the
midbrain. This carries descending motor tract fibers from the cortex to the
spinal cord.



Which of the following would most likely be seen if microscopic examination of
the lesioned area were performed?
/ A. Hirano bodies
/ B. Lewy bodies
/ C. Neuritic plaques
/ D. Neurofibrillary tangles
/ E. Pick bodies



Explanation - Q: 4.8 Close

The correct answer is B. In patients with this condition whose brains are
examined after death, the substantia nigra is found to be pale. Microscopic
sections taken through this region typically demonstrate intracytoplasmic
inclusion in the remaining neurons. These inclusions are called Lewy bodies
and are typically round to elongated, eosinophilic, and have a dense core
surrounded by a paler rim. Lewy bodies are characteristic of idiopathic
Parkinson disease. When viewed by electron microscopy, they are
composed of fine filaments that are densely packed at the core and loose at
the periphery. They stain by immunohistochemistry for ubiquitin, and appear
to be related to neuron degeneration.
Hirano bodies (choice A) are associated with Alzheimer disease, and consist
of actin-containing paracrystalline arrays of beaded fibers that, by light
microscope, appear as elongated, glassy, eosinophilic bodies.
Neuritic plaques (choice C) are associated with Alzheimer disease, and are
focal collections of distorted neurites around a central amyloid core.
Neurofibrillary tangles (choice D) are associated with Alzheimer disease,
and consist of bundles of filaments within the cytoplasm of neurons that often
form a "flame-shaped" inclusion.
Pick bodies (choice E) are associated with Pick disease, and are round to
oval cytoplasmic inclusions composed of filaments that stain densely with
silver stains.


Which of the following agents could produce a syndrome similar to the one seen
in this patient?
/ A. Cocaine
/ B. Carbon monoxide
/ C. Methylenedioxymethamphetamine (MDMA)
/ D. Methylphenyl tetrahydropyridine (MPTP)
/ E. Sulfonamides



Explanation - Q: 4.9 Close

The correct answer is D. An illicit lab attempting to synthesize a
meperidine-like drug in the 1980s, accidently produced MPTP (1-methyl 4-
phenyl-1,2,3,6-tetrahydropyridine), a substance that caused a rapidly
developing and irreversible form of parkinsonism. This extremely neurotoxic
substance can produce a syndrome virtually identical to Parkinson disease in
just 5-15 days, as compared to the slowly developing idiopathic Parkinson
disease, which develops over a period of years. MPTP is converted to MPP+
by MAO-B in astrocytes. MPP+ then enters dopaminergic nerve terminals
and eventually causes the destruction of dopaminergic neurons in the
substantia nigra. Although the microscopic findings are not exactly identical
(typical Lewy bodies are not produced), MPTP produces the best model of
Parkinson disease known to date.
Carbon monoxide (choice A) can cause low-density lesions of the basal
ganglia as shown on CT, however it does not affect the substantia nigra.
Cocaine (choice B) acts in part by blocking dopamine uptake. It can cause a
variety of problems (e.g., cardiovascular, psychiatric), however, it does not
directly damage the substantia nigra.
MDMA (choice C), also known as methylenedioxymethamphetamine or
Ecstasy, is a drug that is taken recreationally. It is a potent releaser of
serotonin in the brain. Animal studies reveal damage to brain serotonergic
nerve terminals, however the substantia nigra remains unaffected.
Neonatal administration of sulfonamides (choice E) and some penicillins,
can displace bilirubin from its albumin-binding site, effectively increasing the
serum concentration of free bilirubin available to cross the BBB. The basal
ganglia can be affected by this (along with other structures such as the
hippocampus, geniculate bodies, cranial nerve nuclei, and cerebellum), but
not the substantia nigra.




Vignette 5 of 8

A 58-year-old man presents to his neurologist with a one-year history of muscle
weakness and muscle twitching, particularly in his hands and legs. The patient
developed foot drop that resulted in a fall and a sprained ankle. His family states
that the man has lost weight loss, especially over the last six months. His son,
home from college, noticed slurring of his words and choking during a recent
dinner. The patient reports no impairment in bladder or bowel controI, or sexual
function. On examination, the Babinski sign is present and fasciculations are
noted. Deep tendon and jaw jerk reflexes are hyperreflexic. Sensory, cognitive,
and oculomotor functions are normaI.


Which of the following is the correct diagnosis?
/ A. Amyotrophic lateral sclerosis
/ B. Kennedy disease
/ C. Lambert-Eaton myasthenic syndrome
/ D. Multifocal motor neuropathy
/ E. Myasthenia gravis


Explanation - Q: 5.1 Close

The correct answer is A. Amyotrophic lateral sclerosis (ALS) is a specific,
rapidly progressive form of degenerative motor neuron disease in which
there are both upper and lower motor neuron signs. Amyotrophic refers to
the muscle atrophy accompanying the lower motor neuron (LMN)
degeneration. Other LMN signs include fasciculations and weakness. Lateral
sclerosis refers to the hardness to palpation of the lateral columns of the
spinal cord in autopsy specimens, where gliosis follows degeneration of the
corticospinal tracts. The clinical aspects include upper motor neuron (UMN)
signs: overactive tendon reflexes, clonus, Babinski and Hoffmann signs.
Symptoms begin with limb involvement in approximately 75-80% of patients;
the rest may present with bulbar signs. Upper and lower limb involvement
occurs at approximately equal frequency at presentation. Patients with upper
limb onset may complain of difficulty with buttons, turning keys, and picking
up small objects; patients with lower limb onset may complain of tripping or
difficulty running. Patients with bulbar onset complain of slurred speech,
hoarseness, and decreased volume of speech; later, dysphagia and drooling
occurs. Extraocular muscles and sphincters are typically spared. No changes
on sensory examination are found. Respiratory insufficiency is usually a late
event. ALS is usually fatal within 3-5 years. Riluzole, which reduces the
presynaptic release of glutamate, may slow progression of the disease. ALS
is also known as Lou Gehrig's disease, because it caused the death of this
baseball legend.
Kennedy disease (choice B) is an inherited disorder characterized by
degeneration of both motor and sensory neurons. Extraocular muscles are
spared. The genetic basis is an expanded repeat of the CAG trinucleotide in
the androgen receptor gene on the proximal portion of the X chromosome.
Lambert-Eaton myasthenic syndrome (choice C) is an autoimmune disorder
of neuromuscular transmission caused by antibodies directed against
voltage-gated calcium channels on the presynaptic motor nerve terminal.
This leads to the decreased release of acetylcholine in response to a nerve
impulse, and to weakness, especially of the proximal muscles of the limbs.
Multifocal motor neuropathy (choice D) is dominated by lower motor neuron
signs, and characterized by multiple motor conduction blocks on electrical
testing. These patients, unlike those with ALS, respond to treatment with
cyclophosphamide or IV immune globulin.
Myasthenia gravis (choice E) is an autoimmune disorder characterized by
fluctuating weakness and fatigability on exertion. Extraocular muscles are
commonly affected. The pathophysiologic substrate is a variable degree of
neuromuscular transmission block caused by autoantibodies against
acetylcholine receptors at the neuromuscular junction.



Which of the following neurologic signs would most likely represent both upper
motor neuron (UMN) and lower motor neuron (LMN) involvement?
/ A. Babinski sign
/ B. Decreased muscle tone
/ C. Fasciculations
/ D. Hoffmann sign
/ E. Weakness



Explanation - Q: 5.2 Close

The correct answer is E. Weakness is a sign of both LMN and UMN
syndromes. UMN weakness particularly involves the extensors of the upper
limb and the flexors of the lower limbs. Patients with LMN involvement
usually have marked weakness and associated muscle wasting (weight loss),
which is the consequence primarily of the loss of neuronal axonal trophic
factors, and partly of disuse.
The Babinski sign (reflex) (choice A) is present when stimulation of the sole
of the foot causes dorsiflexion of great toe with fanning of the remaining toes.
It is normal in young children, but is abnormal after about 2 years of age and
indicates damage to the UMNs.
Decreased muscle tone (choice B), muscle atrophy, and hyporeflexia are
characteristic of LMN disease, while UMN involvement shows hypertonia and
hyperreflexia.
LMN syndrome is characterized by muscle fasciculations (choice C), which
are small and local involuntary contractions of muscle that are visible under
the skin. Fasciculations in ALS are observed with the muscle at rest.
The Hoffmann sign (choice D), or heightened finger flexor reflex (thumb
flexes and adducts), suggests the presence of an UMN lesion affecting the
hands.


Degeneration of which of the following is most likely associated with this patient's
Babinski sign?
/ A. Dorsal funiculus
/ B. Dorsal horn
/ C. Lateral funiculus
/ D. Lateral horn
/ E. Ventral funiculus
/ F. Ventral horn



Explanation - Q: 5.3 Close

The correct answer is C. The lateral funiculus is spinal cord white matter
located between the dorsal and ventral horns. Important tracts in this region
include the lateral corticospinal tracts, the spinothalamic tracts, and dorsal
and ventral spinocerebellar tracts. Upper motor neurons descend in the
lateral corticospinal tracts, and degeneration of the neurons that contribute to
these tracts can lead to a variety of UMN signs, including the Babinski sign.
The dorsal funiculus (choice A) is spinal cord white matter located between
the dorsal median septum and dorsal gray horn. The dorsal columns are
located here.
The dorsal horn (choice B) is part of the gray matter of the spinal cord and
contains cell bodies with sensory functions.
The lateral horn (choice D) is part of the gray matter of the spinal cord and
contains cell bodies with autonomic functions (preganglionic neurons).
The ventral funiculus (choice E) is spinal cord white matter located between
the ventral gray horn and anterior median fissure. It contains tracts such as
the ventral corticospinal tracts and vestibulospinal tracts.
The ventral horn (choice F) is part of the gray matter of the spinal cord and
contains cell bodies with motor functions. Cell bodies of LMNs are located
here. These neurons do degenerate in ALS, but this does not produce a
Babinski sign



Degeneration of which of the following is most likely associated with this patient's
fasciculations?
/ A. Dorsal columns
/ B. Dorsal horn
/ C. Lateral corticospinal tract
/ D. Lateral horn
/ E. Ventral horn
/ F. Ventral white commissure



Explanation - Q: 5.4 Close

The correct answer is E. The ventral horn is part of the gray matter of the
spinal cord and contains cell bodies of the LMNs. Degeneration of these
neurons lead to a variety of LMN signs, including fasciculations, atrophy,
hypotonia, decreased strength, and hyporeflexia.
The dorsal columns (choice A) contain the fasciculus gracilis (legs) and
fasciculus cuneatus (arms), which carry sensory information regarding
discriminative touch, vibration sensation, and conscious proprioception.
The dorsal horn (choice B) is part of the gray matter of the spinal cord and
contains cell bodies with sensory functions.
The lateral corticospinal tract (choice C) contains descending motor fibers
(UMNs).
The lateral horn (choice D) is part of the gray matter of the spinal cord and
contains cell bodies with autonomic functions (preganglionic neurons).
The ventral white commissure (choice F) carries pain and temperature
information. The cell bodies originate in the dorsal horn, cross in the ventral
white commissure, and ascend in the lateral spinothalamic tract to the ventral
posterior lateral (VPL) nucleus of the thalamus


Neurons damaged in this patient's condition decussate in which of the following
locations?
/ A. Cerebral cortex
/ B. Internal capsule
/ C. Medulla
/ D. Midbrain
/ E. Pons
/ F. Spinal cord



Explanation - Q: 5.5 Close

The correct answer is C. Spasticity is the result of UMN dysfunction. UMNs
originate in the cerebral cortex (choice A), descend through the posterior
limb of the internal capsule (choice B), and continue down through the crus
cerebri of the midbrain (choice D), the basilar part of the pons (choice E),
the pyramids of the medulla, and then cross in the caudal medulla (pyramidal
decussation) (choice C). The 85% of the neurons that decussate descend in
the lateral corticospinal tract of the spinal cord, synapsing in the ventral horn
of the spinal cord. LMNs, which are also lesioned in this disease, do not
decussate.



The spasticity that this patient develops is both troubling and painfuI. Which of
the following medications would most directly alleviate this symptom?
/ A. Amitriptyline
/ B. Baclofen
/ C. Pyridostigmine
/ D. Riluzole
/ E. Succinylcholine

Explanation - Q: 5.6 Close

The correct answer is B. Baclofen is a GABA-B agonist and is used to
relieve spasticity. Alpha-adrenergic agonists, such as tizanidine and
clonidine, may also help, but they produce hypotension, which may limit their
use. Dantrolene and benzodiazepines may also be used.
If depression, excessive salivation, or chronic pain occurs in these patients,
amitriptyline (choice A) may be prescribed. Amitriptyline inhibits the re-
uptake of noradrenaline and serotonin at the nerve endings in the CNS. The
drug also has a strong anticholinergic effect.
Pyridostigmine (choice C) is an anticholinesterase inhibitor. It is used to
enhance muscle strength in myasthenia gravis.
Riluzole (choice D), a glutamate antagonist, is the only drug approved by the
FDA for the treatment of ALS. In therapeutic trials, riluzole prolonged survival
by 3-6 months.
Succinylcholine (choice E) is a depolarizing skeletal muscle relaxant used to
relax skeletal muscles during surgery.




Vignette 6 of 8

A 17-year-old man is brought in to the emergency department after being
stabbed in a fight. The knife wound is located in the center of the patient's back
at about the T5 IeveI. After assuring that the patient is hemodynamically stable, a
neurologic examination is preformed. The patient is alert, oriented, and fluent.
His cranial nerves are intact and the motor and sensory examinations of his
upper extremities are within normal limits. However, the patient is unable to move
his right leg. Sensory examination of his lower extremities reveals a complete
loss of tactile discrimination, proprioception, and vibratory sensation in his right
leg and a complete loss of pain and temperature sensation in his left leg.


Which of the following is the most likely diagnosis?
/ A. Anterior spinal artery disruption
/ B. Complete transection of the spinal cord
/ C. Hemisection of the left half of the spinal cord
/ D. Hemisection of the right half of the spinal cord
/ E. Syringomyelia
/ F. Tabes dorsalis



Explanation - Q: 6.1 Close

The correct answer is D. Hemisection of the cord (Brown-Sequard
syndrome) results in an ipsilateral spastic paresis, an ipsilateral loss of
vibratory and position sense, and a contralateral loss of pain and
temperature below the level of the lesion. This patient's symptoms are the
result of a right-sided thoracic spinal cord hemisection.
Anterior spinal artery disruption (choice A) would result in an infarction of the
anterior two-thirds of the spinal cord. While the posterior columns would be
spared, the infarction of the corticospinal tracts and the spinothalamic tracts
would result in bilateral lower extremity spastic paralysis and sensory loss.
Complete transection of the spinal cord (choice B) would result in bilateral
lower extremity spastic paralysis and sensory loss.
Hemisection of the left half of the spinal cord (choice C) would result in left
leg spastic paresis and loss of pain and temperature sensation in the right
leg.
Syringomyelia (choice E) is a cyst that forms around the central canal of the
spinal cord. It usually occurs in the cervical spine and 90% occur with Arnold-
Chiari I malformations. Syringomyelias also occur with trauma, tumors, and
infection. They can present with pain, lower motor neuron findings in the
upper extremities (as the disease process extends into the anterior horns of
the spinal cord), upper motor neuron findings in the lower extremities (as the
disease process extends into the lateral funiculus), and a cape-like loss of
pain and temperature sensation (from the disruption of the decussating
spinothalamic fibers through the ventral white commissure).
Tabes dorsalis (choice F) results from damage to proprioceptive and other
dorsal root fibers. It is classically caused by syphilis. Symptoms include
paresthesias, pain, and abnormalities of gait. Vibration sense is most
affected.


A Iesion in which of the following locations is most likely responsible for the
patient's loss of pain and temperature sensation?
/ A. Left corticospinal tract
/ B. Left gracile fasciculus
/ C. Left lateral spinothalamic tract
/ D. Right corticospinal tract
/ E. Right gracile fasciculus
/ F. Right lateral spinothalamic tract
/ G. Ventral white commissure



Explanation - Q: 6.2 Close

The correct answer is F. A lesion of the right lateral spinothalamic tract
would result in a loss of pain and temperature sensation in the left leg,
beginning one segment below the lesion and extending caudally.
A lesion of the left corticospinal tract (choice A) could lead to a left spastic
paresis.
A lesion of the left gracile fasciculus (choice B) could lead to loss of tactile
discrimination, and position and vibration sensation from the left leg.
A lesion involving the left lateral spinothalamic tract of the spinal cord
(choice C) would result in a loss of pain and temperature sensation in the
right leg, beginning one segment below the lesion, and extending caudally.
A lesion of the right corticospinal tract (choice D) would lead to a right
spastic paresis.
A lesion involving the ventral white commissure of the spinal cord (choice F)
would cause a bilateral loss of pain and temperature sensation within the
dermatomes of the segments affected. The ventral white commissure
contains the crossing axons of second-order sensory neurons of the lateral
spinothalamic tract. These axons originate from cell bodies within the dorsal
horn and travel through the ventral white commissure to ascend in the
ventral half of the lateral funiculus. These neurons carry input from first-order
neurons regarding pain and temperature sensation.



A Iesion in which of the following locations is most likely responsible for the
patient's loss of proprioception?
/ A. Corticospinal tract of the spinal cord
/ B. Lateral spinothalamic tract of the spinal cord
/ C. Posterior columns of the spinal cord
/ D. Spinocerebellar tract of the spinal cord
/ E. Ventral white commissure of the spinal cord



Explanation - Q: 6.3 Close

The correct answer is C. A lesion in the posterior columns would result in
an ipsilateral loss of tactile discrimination and position and vibration
sensation below the lesion. In this case, the gracile fasciculus is implicated
because the symptoms involve only the leg.
A lesion involving the corticospinal tracts of the spinal cord (choice A) would
result in an ipsilateral spastic paresis below the lesion, with upper motor
neuron signs.
A lesion involving the lateral spinothalamic tracts of the spinal cord (choice
B) would result in a contralateral loss of pain and temperature sensation,
beginning one segment below the lesion, and extending caudally.
A lesion involving the spinocerebellar tract of the spinal cord (choice D)
would result in ipsilateral leg dystaxia (dorsal spinocerebellar tract) and
contralateral leg dystaxia (ventral spinocerebellar tract).
A lesion involving the ventral white commissure of the spinal cord (choice E)
would cause a bilateral loss of pain and temperature sensation within the
dermatomes of the segments affected.



A Iesion in which of the following locations is most likely responsible for the
patient's right leg weakness?
/ A. Corticospinal tract of the spinal cord
/ B. Lateral spinothalamic tract of the spinal cord
/ C. Posterior column of the spinal cord
/ D. Spinocerebellar tract of the spinal cord
/ E. Ventral white commissure of the spinal cord



Explanation - Q: 6.4 Close

The correct answer is A. A right-sided spinal cord hemisection, which
includes damage to the corticospinal tract, would result in a right-sided
spastic paresis below the lesion with upper motor neuron signs.
A lesion involving the lateral spinothalamic tracts of the spinal cord (choice
B) would result in a contralateral loss of pain and temperature sensation,
beginning one segment below the lesion and extending caudally.
A lesion involving the posterior columns of the spinal cord (choice C) would
result in an ipsilateral loss of tactile discrimination, and position and vibration
sensation below the lesion. Bilateral posterior column degeneration is seen
with tertiary syphilis (tabes dorsalis).
A lesion involving the spinocerebellar tract of the spinal cord (choice D)
would result in ipsilateral leg dystaxia (dorsal spinocerebellar tract), and
contralateral leg dystaxia (ventral spinocerebellar tract).
A lesion involving the ventral white commissure of the spinal cord (choice E)
would cause a bilateral loss of pain and temperature sensation within the
dermatomes of the segments affected.

The neurons that transmit discriminative touch and vibration information
decussate at which of the following levels?
/ A. Medulla
/ B. Midbrain
/ C. Pons
/ D. Thalamus
/ E. Within one or two levels after entering the spinal cord



Explanation - Q: 6.5 Close

The correct answer is A. After the primary sensory fiber enters the spinal
cord, the ascending branch enters the dorsal columns and travels to the
medulla. The fibers from the legs and trunk travel medially in the fasciculus
gracilis, while those from the arm and neck travel laterally in the fasciculus
cuneatus. The fasciculus gracilis and fasciculus cuneatus make up the
posterior columns. These first order neurons synapse in the medulla (on the
nucleus gracilis and nucleus cuneatus) and then the second order neurons
decussate as the internal arcuate fibers (in the medulla) and ascend in the
medial lemniscus. The second order fibers synapse in the
ventroposterolateral (VPL) nucleus of the thalamus. Tertiary neurons then
synapse upon the somatosensory cortex.



Upper motor neurons decussate at which of the following levels?
/ A. At the junction of the medulla and the spinal cord
/ B. At the junction of the midbrain and the medulla
/ C. At the junction of the pons and the medulla
/ D. At the thalamus
/ E. Within one or two levels after entering the spinal cord



Explanation - Q: 6.6 Close

The correct answer is A. The lateral corticospinal tract originates from layer
V of the cerebral cortex (primary motor cortex, premotor cortex, and primary
sensory cortex). These axons then travel in the posterior limb of the internal
capsule and then the middle three-fifths of the crus cerebri of the midbrain.
They continue through the basal pons and the pyramids in the medulla. 85-
90% of these fibers will decussate in the pyramidal decussation, and will
continue to descend in the spinal cord as the lateral corticospinal tract; the
remaining 10-15% will descend as the anterior corticospinal tract. The
pyramidal decussation occurs in the caudal medulla near the junction of the
medulla and spinal cord.



Vignette 7 of 8

A 41-year-old man feels a slight tingling in his right leg while getting out of his
car. At the time, he ascribes it to the flu he had a week ago. He visits his friends,
and then several hours later, he starts feeling increasingly weak in his arms and
legs, and has difficulty turning the key in the ignition of his car when he leaves.
The next morning, when trying to stand up, the man collapses and is able only to
roll on the floor. His mouth feels strange, with tingling around his lips. He manges
to call paramedics, and is taken to the hospitaI. On admission, complete flaccid
paralysis of his arms and legs is found. He states that he feels "trapped in a body
that l have no control of." The next day, Ieg and back pain begin to develop. By
his fourth hospital day, the man is completely paralyzed and is put on a
ventilator.


Which of the following is the most likely diagnosis?
/ A. Botulism
/ B. Devic disease
/ C. Guillain-Barr syndrome
/ D. Multiple sclerosis
/ E. Myasthenia gravis



Explanation - Q: 7.1 Close

The correct answer is C. Guillain-Barr syndrome (GBS) is an acute
inflammatory demyelinating polyradiculopathy, predominantly affecting motor
function. It is thought to be a postinfectious immune-mediated disease
directed against the peripheral nervous system myelin, axon, or both. The
pathologic features are distinct, and involve multifocal areas of inflammation
and demyelination with cellular infiltration of macrophages and lymphocytes.
Focal demyelination causes slowing in impulse conduction. The damage can
also be axonal (denervation), which will block nerve conduction. GBS usually
follows a minor gastrointestinal or respiratory infection. It commonly presents
with symmetric ascending weakness, bilateral changes in reflexes,
involvement of cranial nerves, mild sensory changes, gait disturbance, pain,
autonomic abnormalities, and respiratory insufficiency that may require
assisted ventilation.
Botulism (choice A) is caused by Clostridium botulinum neurotoxin.
Devic disease (choice B) is a rare, chronic, inflammatory and demyelinating
disorder characterized by optic neuritis and myelitis.
Multiple sclerosis (MS) (choice D) is a demyelinating, inflammatory disease
of the central nervous system especially affecting individuals living in a
temperate climate.
Myasthenia gravis (choice E) is an autoimmune disorder of neuromuscular
transmission.
A Iumbar puncture is performed. Which of the following CSF findings would most
likely be in this patient?
/ A. High protein level and normal cell count
/ B. Large number of cells
/ C. Normal protein level
/ D. OIigoclonal bands
/ E. Tau protein



Explanation - Q: 7.2 Close

The correct answer is A. Typical CSF findings in GBS include elevated
protein, especially IgG (80 to 200 mg/dL) and the presence of a few
mononuclear leukocytes.
The CSF in Devic patients has a large number of WBC (choice B), no
oligoclonal bands, and normal intrathecal IgG synthesis.
Normal protein CSF level (choice C) is seen in patients with botulism.
Oligoclonal bands (choice D), myelin basic protein, normal glucose and
protein levels, and a slightly elevated WBC count are typical CSF finding in
multiple sclerosis.
Increased level of tau protein (choice E) is a typical CSF feature in
Alzheimer disease.



The patient develops problems with swallowing. Which of the following cranial
nerves is most likely involved?
/ A. Abducens (VI)
/ B. Accessory (XI)
/ C. TrigeminaI (V)
/ D. Trochlear (IV)
/ E. Vagus (X)




Explanation - Q: 7.3 Close

The correct answer is E. Swallowing is a complex reflex that involves
coordinated action of several cranial nerves that innervate skeletal muscles
of tongue, palate, pharynx, larynx and upper third of the esophagus, and
smooth muscle of the lower two-thirds of the esophagus. The vagus nerve
plays an important role in swallowing.
The abducens nerve (VI; choice A) innervates skeletal muscles that move
eyeball laterally (m. rectus lateralis), and afferents from the receptors in
those muscles.
The accessory nerve (XI; choice B) nerve innervates neck skeletal muscles
(m. trapezius and m. sternocleidomastoideus).
The trigeminal cranial nerve (V; choice C) innervates chewing muscles
(masticatory muscles). Afferents transmit sensory information from skin,
skeletal muscles of the face, nose and mouth, and teeth sockets.
The trochlear nerve (IV; choice D) innervates skeletal muscles that move the
eyeball downward and laterally (superior oblique).



What would be the most appropriate treatment in this case?
/ A. Botulinus antitoxin
/ B. Corticosteroids
/ C. Interferons
/ D. IV immunoglobulins
/ E. Methotrexate



Explanation - Q: 7.4 Close

The correct answer is D. Infusion of IV immunoglobulins (IVIG), 0.4
mg/kg/day for 5 days is the therapy of choice at many hospitals. Immune
globulin has numerous immunomodulatory and anti-inflammatory activities.
Therapeutic plasma exchange (TPE, plasmapheresis) is beneficial in
patients with rapidly progressive disease. It is contraindicated in patients with
cardiovascular diseases, active sepsis, and dysautonomias. The combination
of IV immunoglobulin and plasma exchange offers no significant additional
advantage.
Botulinus antitoxin (equine, choice A), trivalent, types A, B, and E is the only
treatment available for botulism and should be administered immediately.
Treatment of GBS patients with corticosteroids (choice B) is ineffective and
may actually prolong recovery time.
Beta-1 interferons (choice C) slow the progression of MS and reduce the
frequency of relapses.
Methotrexate (choice E) and corticosteroids showed promising results as
treatment for patients with Devic syndrome.



Which of the following infective agents is most likely associated with this
disease?



Explanation - Q: 7.5 Close

The correct answer is B. C. jejuni is a gram-negative rod and is
microaerophilic (requires reduced levels of oxygen). It is not commonly
carried by man, although it is often isolated from healthy cattle, chickens,
birds, etc. This organism is considered the most common cause of foodborne
illness in the US. The illness is called Campylobacteriosis or Campylobacter
enteritis. GBS is usually preceded by infections, in particular Campylobacter
jejuni. Other agents less frequently identified include Cytomegalovirus,
Mycoplasma pneumoniae, and Epstein-Barr virus. The virulence of C. jejuni
is based on the presence of specific antigens in its capsule that may cross-
react with ganglioside GM1, which is the component of normal myelin. This
process is known as molecular mimicry and it could be a possible immune
pathogenetic mechanism in developing GBS.
Lyme disease is a multisystemic inflammatory disease, with cranial
neuropathy and radiculoneuritis as neurologic manifestations. It is caused by
the spirochete Borrelia burgdorferi(choice A), which is transmitted to
humans by different type of ticks (family Ixodes).
Clostridium botulinum(choice C) is an anaerobic, gram-positive, spore-
forming rod that produces a potent neurotoxin and causes botulism in
humans and animals.
Clostridium tetani(choice D) is a bacterium that causes tetanus in humans.
Salmonella enteritidis(choice E) is a rod-shaped, motile, nonspore-forming
and gram-negative bacterium, which causes gastroenteritis and is not
associated with GBS.









Vignette 8 of 8

A 37-year-old woman presents to her doctor complaining of weakness and
clumsiness in her upper and lower extremities.
On examination, she is alert and oriented, and her speech is fluent. Her cranial
nerves are intact. She is noted to have muscle atrophy in her upper extremities,
and sensory examination reveals a cape-Iike loss of pain and temperature
sensation in her shoulders and upper arms. She also has decreased tone in her
arms and increased tone in her legs.
Reflex testing reveals her to be hyperreflexic in her lower extremities and
hyporeflexic in her upper extremities.


Which of the following is the most likely diagnosis?
/ A. Amyotrophic lateral sclerosis
/ B. Parasagittal meningioma
/ C. Poliomyelitis
/ D. Syringomyelia
/ E. Tabes dorsalis


Explanation - Q: 8.1 Close

The correct answer is D. Syringomyelia is a cyst that forms around the
central canal of the spinal cord. It usually occurs in the cervical spine and
90% occur with Arnold Chiari I malformations. Syringomyelias also occur with
trauma, tumors, and infection. They can present with pain, lower motor
neuron findings in the upper extremities (as it extends into the anterior horns
of the spinal cord), upper motor neuron findings in the lower extremities (as it
extends into the lateral funiculus), and a cape-like loss of pain and
temperature sensation (from the disruption of the decussating spinothalamic
fibers through the ventral white commissure).
Amyotrophic lateral sclerosis (choice A) is the most frequent adult-onset
progressive motor neuron disease. It results in degeneration of motor
neurons and the corticospinal tracts, which results in the expression of both
upper and lower motor neuron signs. There are no sensory changes that
occur with amyotrophic lateral sclerosis.
A parasagittal meningioma (choice B) would result in bilateral lower
extremity weakness if it compressed the motor cortex of both hemispheres
beneath it. The area of cortex that represents the legs is in the parasagittal
region of the motor homunculus.
Poliomyelitis (choice C) is an acute inflammatory viral inflammation caused
by the poliovirus, which is an enterovirus. It is transmitted by the fecal-oral
route and infection leads to destruction of the anterior horn cells (lower motor
neurons) in the spinal cord, leading to flaccid paralysis.
In tabes dorsalis (choice E) there is demyelination of the posterior columns
caused by tertiary syphilis. This results in impaired proprioception and
locomotor ataxia. Other associated symptoms are usually present such as
lightning-like pains, Argyll-Robertson pupils, and Charcot joints.


A Iesion of which of the following is the most likely cause of this patient's sensory
symptoms?
/ A. Apical portions of the precentral and postcentral gyri, bilaterally
/ B. Lateral corticospinal tracts of the spinal cord
/ C. Lateral spinothalamic tracts of the spinal cord
/ D. Posterior columns of the spinal cord
/ E. Ventral white commissure of the spinal cord


Explanation - Q: 8.2 Close

The correct answer is E. The ventral white commissure contains the
crossing axons of second-order sensory neurons of the lateral spinothalamic
tract. Primary sensory neurons carrying pain and temperature information
have cell bodies in the dorsal root ganglia and project axons to the spinal
cord via the dorsolateral tract of Lissauer. They synapse in the dorsal horn
on secondary neurons, which travel through the ventral white commissure to
ascend in the ventral half of the lateral funiculus (lateral spinothalamic tract).
A lesion involving the ventral white commissure (as would occur with a
cervical syringomyelia) would cause a bilateral loss of pain and temperature
sensation within the dermatomes of the affected segments. Bilateral deficits
occur because a lesion of this area affects tracts decussating from both sides
of the spinal cord. However, the posterior columns are spared, thus
preserving discriminative touch, proprioception, and vibration sensation. This
is called a dissociated sensory loss.
A lesion involving the apical portions of the precentral and postcentral gyri
bilaterally (choice A), such as a parasagittal meningioma, would result in
bilateral lower extremity spastic paresis with upper motor neuron signs.
A lesion involving the lateral spinothalamic tracts of the spinal cord (choice
B) would result in a contralateral loss of pain and temperature sensation,
beginning one segment below the lesion and extending caudally.
A lesion involving the lateral corticospinal tracts of the spinal cord (choice C)
would result in an ipsilateral spastic paresis below the lesion, with upper
motor neuron signs.
A lesion involving the posterior columns of the spinal cord (choice D) would
result in an ipsilateral loss of tactile discrimination and position and vibration
sensation below the lesion. Bilateral posterior column degeneration is seen
with tertiary syphilis (tabes dorsalis).

A Iesion of which of the following is the most likely cause of this patient's upper
extremity weakness?
/ A. Anterior horns of the spinal cord
/ B. Apical portions of the precentral and postcentral gyri bilaterally
/ C. Brachial plexus bilaterally
/ D. Lateral corticospinal tracts of the spinal cord
/ E. Lateral spinothalamic tracts of the spinal cord



Explanation - Q: 8.3 Close

The correct answer is A. This lower motor neuron lesion would result in
bilateral upper extremity weakness, with muscle wasting and hyporeflexia.
Such a lesion is seen in amyotrophic lateral sclerosis and, in the case of this
patient, from expansion of a syrinx into the anterior horns of the cervical
spinal cord.
A lesion involving the apical portions of the precentral and postcentral gyri
bilaterally (choice B), such as a parasagittal meningioma, would result in
bilateral lower extremity spastic paresis with upper motor neuron signs.
A lesion involving the brachial plexus bilaterally (choice C), such as bilateral
brachial plexus schwannomas or bilateral brachial plexitis, could result in
bilateral upper extremity lower motor neuron weakness, sensory
disturbances, and, in the case of brachial plexitis, pain radiating down both
arms. However, these lesions rarely occur bilaterally and would not result in
the lower extremity weakness exhibited by the patient.
A lesion involving the lateral corticospinal tracts of the spinal cord (choice D)
would result in an ipsilateral spastic paresis below the lesion with upper
motor neuron signs, such as hyperreflexia and increased tone.
A lesion involving the lateral spinothalamic tracts of the spinal cord (choice
E) would result in a contralateral loss of pain and temperature sensation,
beginning one segment below the lesion and extending caudally.



A Iesion of which of the following is the most likely cause of this patient's lower
extremity weakness?
/ A. Anterior horns of the spinal cord
/ B. Apical portions of the precentral and postcentral gyri bilaterally
/ C. Lateral corticospinal tracts of the spinal cord
/ D. Lateral spinothalamic tracts of the spinal cord
/ E. Posterior columns of the spinal cord



Explanation - Q: 8.4 Close

The correct answer is C. Extension of a syringomyelia into the lateral
funiculus of the cervical spinal cord bilaterally, affecting the lateral
corticospinal tracts, would result in a spastic paresis below the lesion, with
upper motor neuron signs as described in this patient.
A lesion involving the anterior horns of the spinal cord (choice A) would
result in a lower motor neuron lesion, causing weakness with muscle wasting
and hyporeflexia. However, this would occur at the level of the syrinx.
A lesion involving the apical portions of the precentral and postcentral gyri
bilaterally (choice B), such as a parasagittal meningioma, would result in
bilateral lower extremity spastic paresis with upper motor neuron signs.
However, given that the patient also has lower motor neuron signs in her
upper extremities, and a dissociated sensory loss, this makes the spinal cord
a much more likely candidate.
A lesion involving the lateral spinothalamic tracts of the spinal cord (choice
D) would result in a contralateral loss of pain and temperature sensation,
beginning one segment below the lesion and extending caudally.
A lesion involving the posterior columns of the spinal cord (choice E) would
result in an ipsilateral loss of tactile discrimination, and position and vibration
sensation below the lesion.


Vignette 1 of 5

A 3-month-old boy is brought to the emergency department by his parents
because he has been vomiting for 4 days. The baby appears very ilI, with dry
mucous membranes and poor skin turgor. He is poorly responsive during the
examination.
According to his mother, he has not been able to hold anything down for several
days, and his diapers have been dry for almost 24 hours. The mother describes
the vomiting as having been very forcefuI, and accompanied with gagging and
retching. She did not notice any red-brown, black, or green discoloration to the
emesis, and states that it just looked like formula. No other family members have
been ilI, and when the mother called the child's daycare, she was told that no
other children were ill with similar symptoms. Despite the vomiting, until today,
the child has behaved as if he were very hungry, and would seek the bottle again
shortly after vomiting.

Which of the following would be most likely to account for the patient's problems?
/ A. Bacterial gastroenteritis
/ B. Esophageal reflux
/ C. Gastric ulcer
/ D. Pyloric stenosis
/ E. Viral gastroenteritis



Explanation - Q: 1.1 Close

The correct answer is D. This child most likely has hypertrophic pyloric
stenosis, which causes obstruction of the pyloric channel secondary to
muscular hypertrophy in the pyloric region. This condition may run in families,
and most often does not present until the child is several weeks to months
old. Hypertrophic pyloric stenosis has an incidence of about 1 of every 500
births, and a male predominance of 4-5 to 1. The vomiting may be so severe
that the term "projectile" is appropriate. This child's presentation is typical.
The absence of discoloration to the emesis indicates that it does not contain
bile (and so the problem is above the ampulla of Vater in the duodenum) or
blood.
Both bacterial (choice A) and viral (choice E) gastroenteritis would usually
be accompanied by significant nausea, and the child would not wish to eat
after vomiting.
Esophageal reflux (choice B) can cause a gentle regurgitation of formula,
but does not cause severe vomiting with gagging and emesis.
Gastric ulcer (choice C) would be very unusual in a child of this age.



Physical examination also shows visible gastric peristaltic waves. Which
additional finding should be sought on physical examination?
/ A. Multiple petechiae
/ B. OIive shaped mass
/ C. Pain on palpation in the upper left quadrant below the liver
/ D. Spider angiomas
/ E. Tenderness at McBurney's point



Explanation - Q: 1.2 Close

The correct answer is B. The most characteristic physical finding on
abdominal examination in children with hypertrophic pyloric stenosis is the
presence of a mass about the size and shape of a small olive, which is
usually felt to the right of the umbilicus. This mass is most easily felt if the
child is calm (or better still asleep) and gentle palpation is used to prevent
guarding. The mass is produced by hypertrophy of the pyloric muscle.
Multiple petechiae (choice A) would suggest a blood clotting problem.
Pain below the liver (choice C) suggests gallstone disease (usually in
adults).
Spider angiomas (choice D) suggest alcoholic cirrhosis (usually in adults).
Tenderness at McBurney's point (choice E) in the right lower quadrant
suggests appendicitis.


The patient's history of vomiting would be most likely to produce which of the
following?
/ A. Metabolic acidosis with increased anion gap
/ B. Metabolic acidosis with normal anion gap
/ C. Metabolic alkalosis
/ D. Respiratory acidosis
/ E. Respiratory alkalosis



Explanation - Q: 1.3 Close

The correct answer is C. These children lose gastric hydrochloric acid in
the vomitus, and consequently develop a metabolic alkalosis with low
chloride levels. Other laboratory findings that may be seen are related to
dehydration and include: high specific gravity in the urine, high BUN and
creatinine, and hemoconcentration.
Metabolic acidosis with increased anion gap (choice A) can be seen in
diabetes mellitus, lactic acidosis, renal failure, and intoxication.
Metabolic acidosis with normal anion gap (choice B) can be seen with
diarrhea, renal tubular acidosis, and other renal disease.
Respiratory acidosis (choice D) can be seen with depression of the
respiratory center, severe lung disease, and diseases that impair lung
expansion.
Respiratory alkalosis (choice E) can be seen in voluntary or drug-induced
hyperventilation or resulting from hyperventilation at high altitudes.


Which of the following is the best choice for confirmation of the diagnosis?
/ A. Computed tomography
/ B. Magnetic resonance imaging
/ C. PIain x-ray film
/ D. Scintillation scan
/ E. Uitrasound


Explanation - Q: 1.4 Close

The correct answer is E. Most cases of hypertrophic pyloric stenosis are
now being confirmed with abdominal ultrasound, which, in these cases,
demonstrates a typical hypoechoic mass between the stomach and
duodenum. An upper gastrointestinal series may also be helpful, if
ultrasound studies are ambiguous.
Computed tomography (choice A) and magnetic resonance imaging (choice
B) are overly expensive and not required for diagnosis in this condition.
Plain x-ray films (choice C) may show a stomach filled with air and fluid, but
this is not considered as reliable a finding as the abdominal ultrasound
studies.
Scintillation scans (choice D) are usually used to evaluate the extent of bony
disease.



Which of the following treatments would best correct this child's disease?
/ A. IV fluids with 3 day restriction of oral intake
/ B. Gastric resection
/ C. Medical management with acetaminophen
/ D. Medical management with antacids
/ E. Pyloromyotomy



Explanation - Q: 1.5 Close

The correct answer is E. After inguinal hernia, the most common cause of
gastrointestinal surgery in very young children is hypertrophic pyloric
stenosis. The surgery that is done is called pyloromyotomy, and involves
partially cutting through the pyloric muscle, to allow it to be spread apart with
resultant dilation of the lumen of the pyloric channel. This operation only
takes about 30 minutes to perform after anesthesia is induced, and most of
the babies are allowed to go home within 24-48 hours.
Gastric resection (choice B) is not required, and various forms of medical
management (choices A, C, and D) will not correct the underlying problem
(although medical stabilization before surgery is appropriate).



Vignette 2 of 5

Over 50 children at an elementary school became violently ill several hours after
eating lunch. AII of the affected children developed severe nausea and vomiting,
and some additionally developed abdominal cramps and non-bloody diarrhea. AII
of the students felt much better by the following morning.

Which of the following items from the lunch would be most likely to be the cause
of the food poisoning?
/ A. Carrot sticks
/ B. Chicken salad
/ C. Fresh pineapple
/ D. Lemonade
/ E. Whole wheat rolls


Explanation - Q: 2.1 Close

The correct answer is B. In most cases of food poisoning causing violent
gastrointestinal symptoms, the culprit contains meat, cheese, milk or milk
products, eggs, or salad dressing. In this case, the chicken salad is the most
likely culprit.
Fresh fruits and vegetables, such as the carrots (choice A) and pineapple
(choice C), are less likely choices unless a salad dressing has been used.
Lemonade (choice D) is too acidic to grow most bacteria, and most
commercial soft drinks have enough preservatives in them to not be a
problem, even if they have been badly stored for a few hours.
Bread products (choice E) are also not usually implicated, unless they are
moist due to being incorporated into a dessert or casserole.

Gram's stain of a smear taken from the presumed source of the food poisoning
shows abundant gram-positive cocci in grape-Iike clusters. Which of the following
is the most likely causative organism?


Explanation - Q: 2.2 Close

The correct answer is C. "Gram-positive cocci in grape-like clusters" is the
classic description used in test questions about Staphylococcus.
Bacillus cereus(choice A) can cause food poisoning, but is a gram-positive
rod.
Escherichia coli(choice B) can cause diarrheal illness related to infection,
but is a gram-negative rod.
Streptococcus pneumoniae(choice D) and S. pyogenes(choice E) are gram-
positive cocci that usually occur in chains and are not a usual cause of food
poisoning or diarrheal illness


Which of the following is the most likely source of the bacteria?
/ A. Food preparer's hands
/ B. Infected animal supplying milk
/ C. Infected animal used for meat
/ D. Poorly cleaned bowl
/ E. Water used in food preparation


Explanation - Q: 2.3 Close

The correct answer is A. While in theory, staphylococci can enter a food
specimen by any of the routes listed, in practice the most common source is
bacteria from the food handler's skin. Staphylococcus aureus is present in
small numbers on the skin of many individuals, and may be present in large
numbers in persons with true staphylococcal skin infections. All commercial
food handlers should use gloves, and people with impetigo (skin infection
due to staphylococci) should not prepare food. The presentation of the case
in these questions was based in part on a real staphylococcal food poisoning
outbreak that occurred in Texas and has been reported by the US Food and
Drug Administration. 16 elementary schools were served by the same central
kitchen, and 1,364 of 5,825 children became poisoned. The culprit was the
chicken salad. The previous day, frozen chickens had been boiled for 3
hours, deboned, and then cooled to room temperature with a fan. They were
then ground into small pieces and placed in 12 inch deep aluminum pans
(which may not have cooled quickly) where they were stored overnight in a
walk-in refrigerator at a temperature thought to be 42-45 F. The next
morning, the salad was prepared. The food was then transported in thermal
containers to the various schools, where it was kept at room temperature for
several hours before being served to the children. The investigators thought
that the contamination probably occurred while the chickens were being
deboned, and then the subsequent inadequate refrigeration for much of 24
hours allowed proliferation of the staphylococci, which were easily
demonstrated in the chicken salad later.


Which of the following is the most appropriate therapy to offer most patients who
develop this type of food poisoning?
/ A. Bed rest only
/ B. IV cephalosporins
/ C. Oral ampicillin
/ D. Oral cephalosporins
/ E. Penicillin injection


Explanation - Q: 2.4 Close

The correct answer is A. Staphylococcal food poisoning is usually self-
limited, and can be treated with bed rest alone. Antibiotics (choices B to E)
are not indicated. Death occurs rarely, and when it does, the patient is
almost always in a vulnerable group, such as the very young, the elderly, or
the already seriously ill. Patients in these populations may require
hospitalization with intravenous fluid support through the illness.




Vignette 3 of 5

A 13-year-old girl is taken to the emergency department by her mother. The girl
had awoken feeling ilI, and stayed home alone. When her mother returned from
work, she found that her daughter had been vomiting more or less continuously
all day. The mother was particularly concerned because her daughter seemed to
be acting "weird" with an unusual degree of drowsiness and listlessness. When
the physician attempts to interview the girI, she is very irritable, combative, and
appears to be confused. Physical examination fails to demonstrate focal
neurologic findings, but hepatomegaly is noted.
No jaundice, fever, rash, or lymphadenopathy are noted. Drug screens are
negative; screening serum chemistries demonstrate moderate elevations (4
times upper limit of normaI) of serum liver transaminases. By the time the
laboratory results return, the girl has developed coma with progressive
unresponsiveness. Emergency CT scan of the head demonstrates nonspecific
findings including cerebral edema, gyral flattening, swollen white matter, and
ventricular compression.
Which of the following is the most likely diagnosis?
/ A. Kawasaki syndrome
/ B. Meningococcal meningitis
/ C. Reye syndrome
/ D. Rocky Mountain spotted fever
/ E. Wilson disease


Explanation - Q: 3.1 Close

The correct answer is C. Reye syndrome is a rare, but potentially
devastating disease primarily affecting individuals less than 18 years old.
Because of its rarity, it is likely to be misdiagnosed as meningitis,
encephalitis, diabetes, drug overdose, sudden infant death syndrome, head
trauma, renal or hepatic failure, poisoning, or a psychiatric disturbance. In
infants, it should be suspected when diarrhea (but not necessarily vomiting),
respiratory disturbances (hyperventilation, apneic episodes), seizures, or
hypoglycemia are accompanied by elevated liver transaminases (AST, ALT)
in the absence of jaundice. In children, adolescents, and the rare adult
patient, suspect Reye when you see a cluster of severe vomiting, elevated
serum transaminases without jaundice, and signs of neurologic dysfunction.
This patient's CT findings are typical, and nonspecifically reflect marked brain
edema with compression of fluid-filled spaces such as the ventricles and
meningeal spaces. Treatment of Reye syndrome is supportive, as no specific
therapy is available. Patients may recover completely, be left with mild to
severe residual neurologic defects, or die.
Kawasaki syndrome (choice A) is a sometimes very severe febrile illness of
children that may be associated with lymphadenopathy, rash, and late
development of coronary artery aneurysms secondary to polyarteritis.
Meningococcal meningitis (choice B) can present fulminantly, as in this
case, but nuchal rigidity would be noted in the case presentation and a CT
scan would probably show meningeal thickening.
Rocky Mountain spotted fever (choice D) can cause a fulminant illness with
a prominent petechial rash.
Wilson disease (choice E) can cause both psychiatric and hepatic
dysfunction, but tends to become symptomatic over a period of decades.


Marked elevation of which of the following would be most helpful in confirming
the suspected diagnosis?
/ A. BIood free erythrocyte protoporphyrin
/ B. BIood methemoglobin
/ C. Serum ammonia
/ D. Serum somatomedin C
/ E. Urine vanillylmandelic acid

Explanation - Q: 3.2 Close

The correct answer is C. Patients with Reye syndrome often have markedly
elevated serum NH4
+
levels as a consequence of the liver dysfunction.
These high ammonia levels probably substantially contribute to the altered
mental status seen commonly in Reye patients.
Erythrocyte protoporphyrin (choice A) is increased in some forms of
porphyria.
Methemoglobin (choice B) is an oxidized form of hemoglobin that can be
seen in hemolytic anemias.
Somatomedin C (choice D) elevation can be seen in gigantism and
acromegaly.
Vanillylmandelic acid (choice E) can be elevated in pheochromocytoma.


This patient's disease was most likely preceded by which of the following
illnesses?
/ A. Crohn disease
/ B. Diabetes mellitus
/ C. Hypothyroidism
/ D. Influenza
/ E. Juvenile rheumatoid arthritis


Explanation - Q: 3.3 Close

The correct answer is D. Many patients who develop Reye syndrome have
a recent history of a febrile viral illness, most notably influenza and varicella
(chicken pox). How this predisposes for Reye syndrome is unclear, but the
link appears compelling. Note that patients who have preceding chickenpox
may still have the rash, as Reye syndrome typically develops on the 5th or
6th day of the illness in these patients.
Reye syndrome does not appear to be an autoimmune disease, and has no
links to other diseases with a strong autoimmune component, including
Crohn disease (choice A), diabetes mellitus type I (choice B), and juvenile
rheumatoid arthritis (choice E).
Reye syndrome is also unrelated to hormonal disorders such as
hypothyroidism (choice C).





Ingestion of which of the following is most likely to have contributed to the
patient's illness?
/ A. Acetaminophen
/ B. Aspirin
/ C. Coffee
/ D. Pseudoephedrine
/ E. Tea


Explanation - Q: 3.4 Close

The correct answer is B. Aspirin and salicylate use is thought to increase
the risk of developing (still very rare) Reye syndrome by 35-fold. While few
parents now give young children aspirin, many teenagers or their parents
may consider the teens to be "adults" and use aspirin for fever and
discomfort control. Also, many over-the-counter anti-nausea medications
may contain salicylates, and patients may thus be exposed unknowingly to
them. The other agents listed in the choices have no known link to Reye
syndrome.


If this patient's liver were biopsied and a sample sent for electron microscopy,
which of the following would most likely be seen?
/ A. Abnormally formed cilia
/ B. Enlarged mitochondrial with disrupted cristae
/ C. Linear inclusions in macrophages
/ D. Membrane-bound vacuoles with complex crystalline structures
/ E. Prominent, Iong microvilli


Explanation - Q: 3.5 Close

The correct answer is B. While the pathophysiology of Reye syndrome is
still poorly understood, severe mitochondrial dysfunction (most prominent in
brain and liver) is a striking feature and is confirmed by often marked
morphological changes in mitochondria observed by electron microscopy.
Abnormal cilia (choice A) in the respiratory epithelium is a feature of
Kartagener syndrome, characterized by bronchiectasis and sometimes, situs
inversus.
Linear inclusions in macrophages (choice C) in the brain are a feature of the
hereditary condition, Krabbe disease.
Membrane-bound vacuoles with a complex crystalline structure (choice D)
can be seen in the brain of patients with the hereditary condition,
metachromatic leukodystrophy.
Prominent, long microvilli (choice E) are a feature of mesotheliomas.

Vignette 4 of 5

A 65-year-old man is struck by a car as he is staggering across the street after a
night of binge drinking. When the ambulance arrives, he is noted to be belligerent
and combative. A Iarge left temporal laceration is observed. On arrival to the
emergency department, his blood pressure is 148/78 mm Hg, his pulse is 89/min,
and his oxygen saturation is 96%.
He continues to be uncooperative, so only a cursory physical examination is
performed, which reveals a 6 cm left temporal Iaceration and no other gross
abnormalities. The laceration is sewn closed at the bedside, and the patient left
to rest. An hour later the patient's speech becomes more slurred; he begins
vomiting, and is given prochlorperazine. An hour later, he is found to be
unresponsive. His blood pressure is 194/100 mm Hg and his pulse is 55/min. His
left pupil is 6 mm and non-reactive and his right pupil is 3 mm and reactive to
light. He localizes to pain in his left upper and lower extremities, but exhibits no
movement in his right upper and lower extremities.


Which of the following is the most likely diagnosis?
/ A. AIcohol toxicity
/ B. AIcohol withdrawal
/ C. Horner syndrome
/ D. Subfalcine (cingulate) herniation
/ E. Uncal herniation


Explanation - Q: 4.1 Close

The correct answer is E. Uncal herniation occurs when a mass lesion, such
as a subdural hematoma, causes increased intracranial pressure and
displaces the brain downward. The uncus and parahippocampal gyrus are
pushed over the tentorial edge, where they compress the brain stem,
causing contralateral hemiparesis. The uncus also compresses the ipsilateral
third cranial nerve, as it enters the interpeduncular cistern at the
midbrain/pons junction, resulting in an ipsilateral third nerve palsy. Signs and
symptoms of increased intracranial pressure often are present prior to a
herniation syndrome. These include headache, nausea/vomiting, agitation,
lethargy, and eventually, coma. There may be a decrease in heart rate and
an increase in blood pressure as well.
Alcohol toxicity (choice A) is characterized by disinhibition, emotional lability,
slurred speech, and ataxia, followed by loss of consciousness and then
coma. It would be unusual for a conscious intoxicated patient to progress to
coma without consuming more alcohol.
Alcohol withdrawal (choice B) usually occurs 2-5 days after the last drink.
Autonomic hyperactivity, characterized by tachycardia, hypertension,
tremors, and anxiety, occurs first. This is followed by delirium tremens,
hallucinations, and confusion.
Horner syndrome (choice C) results from a disruption of the sympathetic
fibers that originate from the hypothalamus. These fibers travel inferiorly to
synapse on second-order neurons in the intermediolateral column of the
spinal cord, then synapse on third-order neurons in the superior cervical
sympathetic ganglion, to finally innervate the smooth muscles of the eyelids,
pupil, and sweat glands of the face and forehead. Interruption of these fibers,
which can occur with a Pancoast tumor, results in ipsilateral ptosis, miosis,
and anhidrosis.
Subfalcine (cingulate) herniation (choice D) occurs when a mass lesion
causes the cingulate gyrus to move under the free edge of the falx, and the
ipsilateral foramen of Monro becomes trapped. This causes an ipsilateral
large lateral ventricle and a contralateral small lateral ventricle.


What is the mechanism most likely responsible for the patient's unilateral
pupillary dilation?

/ A. Autonomic hyperactivity due to alcohol withdrawal
/ B. Prochlorperazine (Compazine)
/ C. Unilateral compression of preganglionic parasympathetic fibers originating
from the Edinger-Westphal nucleus
/ D. Unilateral compression of postganglionic sympathetic fibers originating from
the superior cervical ganglion
/ E. Unilateral compression of the frontal eye fields


Explanation - Q: 4.2 Close

The correct answer is C. The Edinger-Westphal nucleus gives rise to
preganglionic parasympathetic fibers. These fibers leave the midbrain and
travel on the dorsal superficial aspect of the oculomotor nerve. They then
synapse on the ciliary ganglion. The ciliary ganglion then gives rise to
postganglionic parasympathetic fibers, which terminate in the ciliary body
and the iris to cause miosis. Interruption of this pathway, such as when the
uncus compresses these fibers in uncal herniation, results in unopposed
sympathetic input to the ipsilateral eye, which results in ipsilateral pupillary
dilation.
Autonomic hyperactivity due to alcohol withdrawal (choice A) usually occurs
2-5 days after the last drink. It is manifested as tachycardia, hypertension,
anxiety, and tremors.
Prochlorperazine (Compazine) (choice B) is a phenothiazine antiemetic,
which selectively antagonizes dopamine D2 receptors. It can have atropine-
like side effects, as well as cause photosensitivity and even oculogyric crisis.
It would not cause a unilateral papillary dilation.
Unilateral compression of postganglionic sympathetic fibers originating from
the superior cervical ganglion (choice D) interrupts the sympathetic input to
the ipsilateral eye and half of the face, resulting in a Horner's syndrome
(miosis, ptosis, anhidrosis).
Unilateral compression of the frontal eye fields (choice E) results in
ipsilateral eye deviation. The frontal eye fields (Area 8) are located in the
caudal middle frontal gyrus and are responsible for the initiation of saccades
(rapid eye movements to a target of behavioral importance). Stimulation of
area 8 results in conjugate eye deviation to the contralateral side (away from
the stimulation). A lesion involving area 8 causes a transient conjugate eye
deviation to the ipsilateral side (toward the lesion).


Which of the following is a common side effect of prochlorperazine?
/ A. Diarrhea
/ B. Disulfiram-Iike reaction
/ C. Drowsiness
/ D. Miosis
/ E. Urinary urgency


Explanation - Q: 4.3 Close

The correct answer is C. Prochlorperazine is a phenothiazine antiemetic,
which commonly causes drowsiness, as well as dizziness, and can produce
extrapyramidal side effects. For these reasons, it should not be used in
patients with an altered mental status, as it further depresses the patient's
sensorium and makes it more difficult for the treating physician to accurately
follow the patient's neurologic examination. It also has atropine-like side
effects, resulting in dry mouth, constipation, and urinary retention.
None of the other choices are side effects of prochlorperazine.


Which of the following is the most likely cause for the patient's
unresponsiveness?
/ A. Injury to both of the mamillary bodies
/ B. Injury to the left parietal lobe
/ C. Injury to the left subthalamic nucleus
/ D. Injury to the reticular activating system
/ E. Injury to the right parietal lobe


Explanation - Q: 4.4 Close

The correct answer is D. Tonic input from the reticular activating system
(RAS) keeps the brain turned "on." An injury to the RAS, such as during
uncal herniation, causes the patient to fall into a coma. The RAS is located in
the middle and lateral pons and midbrain. It sends signals to the subcortical
structures (especially the thalamus), which then send diffuse inputs to the
cortex. The cortex, in turn, sends positive feedback inputs back to the RAS.
The RAS also sends signals to the spinal cord to maintain tone and activate
the spinal reflexes.
Injury to the mamillary bodies (choice A) is seen in Wernicke-Korsakoff
encephalopathy. This disorder is seen in alcoholics with B1 (thiamine)
deficiency. Wernicke encephalopathy is characterized by psychosis,
ophthalmoplegia, and confusion. Korsakoff syndrome is characterized by
memory loss, confabulation, and confusion.
Injury to the left parietal lobe (choice B) can result in a Gerstmann
syndrome. Gerstmann syndrome is characterized by right/left confusion,
finger agnosia, acalculia, and agraphia.
Injury to the left subthalamic nucleus (choice C) results in contralateral
sudden limb flailing (hemiballismus).
Injury to the right parietal lobe (choice E) can result in a left hemineglect.


On further examination, the patient exhibits decerebrate (extensor) posturing in
his upper and lower extremities. What is the most likely mechanism for this?
/ A. An injury below the level of the vestibular nuclei
/ B. An injury between the red nucleus and the vestibular nuclei
/ C. An injury to the midbrain above the red nucleus
/ D. An injury to the posterior limb of the left internal capsule
/ E. An injury to the vermis



Explanation - Q: 4.5 Close

The correct answer is B. Decerebrate (extensor) posturing is characterized
by extension, adduction, and pronation of the arms, extension of the legs,
and plantar flexion of the feet. It results from a lesion of the rubrospinal tract,
originating from the red nucleus, which maintains flexor tone in the arms and
legs. The vestibulospinal tract, which maintains extensor tone, is then
unopposed, resulting in the extensor posture.
An injury below the level of the vestibular nuclei (choice A) results in
contralateral hemiparesis (or quadriparesis if the lesion involves both
corticospinal tracts), but not in posturing.
An injury to the midbrain above the red nucleus (choice C) results in
contralateral hemiparesis (or quadriparesis if the lesion involves both
corticospinal tracts) and decorticate (flexor) posturing due to dominance of
the rubrospinal tract, which maintains flexor tone in the arms and legs.
An injury to the posterior limb of the left internal capsule (choice D) results in
a contralateral hemiparesis and decorticate (flexor) posturing due to
dominance of the rubrospinal tract, which maintains flexor tone in the arms
and legs.
An injury to the vermis (choice E) results in truncal ataxia, scanning speech,
and hypotonia.




Vignette 5 of 5

A 33-year-old woman presents to the emergency department complaining of
nausea and vomiting. She states that she has been having significant nausea
that has been worsening over the past 2 weeks. Over the past 2 days, she has
had 2 episodes of vomiting. She also notes increased fatigue. She has no
abdominal pain or vaginal bleeding. She has no other complaints. Her past
medical history is significant for occasional migraine headaches. She has never
had surgery. She takes acetaminophen as needed for headache, and has no
known drug allergies. She works as a lawyer at a local firm and lives with her
husband of three years. She has no family history of cancer or heart disease. Her
vital signs are stable.
Examination is significant for a bluish-appearing cervix on speculum examination.
The remainder of the examination, including the abdominal examination, is
benign. Laboratory evaluation shows:
Urine hCG: positive
Leukocytes: 9,000/mm3
Hematocrit: 41%
PIatelets: 250,000/mm3
Pelvic ultrasound demonstrates a gestational sac with yolk sac and fetal pole
surrounded by myometrium. There is a heart rate of 154 beats per minute.

Which of the following is the most likely diagnosis?
/ A. Appendicitis
/ B. Complete hydatidiform mole
/ C. Ectopic pregnancy
/ D. Intrauterine pregnancy
/ E. Spontaneous abortion


Explanation - Q: 5.1 Close

The correct answer is D. This patient's presentation, exam findings, and
studies are all consistent with a diagnosis of intrauterine pregnancy. First,
her presenting complaints of nausea, vomiting, and fatigue are consistent
with a first trimester pregnancy. Approximately 80% of pregnant women
experience some nausea during the pregnancy, especially in the first
trimester. This nausea is most commonly called "morning sickness" but it can
occur at any time during the day. Fatigue is also a common first trimester
complaint. On examination she has a bluish-appearing cervix. This is called
"Chadwick's sign" and it is another clue that she is pregnant. Most
definitively, though, she has a positive urine pregnancy test. With this
positive test, the diagnosis is narrowed to intrauterine pregnancy, ectopic
pregnancy, or spontaneous abortion, with mole also being a consideration.
The ultrasound that demonstrates an intrauterine pregnancy fully establishes
the diagnosis.
Appendicitis (choice A) represents an infection of the appendix. Patients
with appendicitis most commonly present with symptoms and signs of
infection, including abdominal pain, fever and chills, abdominal tenderness,
elevated temperature, and leukocytosis.
A patient with a complete hydatidiform mole (choice B) will not have an
intrauterine pregnancy with a fetal heart rate visualized on ultrasound
examination. A complete mole often appears as a "snowstorm" pattern on
pelvic ultrasound.
It is essential to "think ectopic!" whenever a woman of childbearing age
presents for medical attention. In fact, this sign ("think ectopic!") and other
such similar signs can be seen in many emergency rooms. However,
patients with ectopic pregnancy (choice C) usually complain of abdominal
pain or vaginal bleeding. On examination, they will often have abdominal and
adnexal tenderness. Pelvic ultrasound will show no intrauterine pregnancy.
This patient, with a gestational sac, yolk sac, and fetus seen surrounded by
myometrium (that is, within the uterus) can be diagnosed with an intrauterine
pregnancy.
Most patients with spontaneous abortions (choice E) present with vaginal
bleeding a passage of tissue (products of conception.) On ultrasound, no
viable pregnancy (i.e., an intrauterine pregnancy with a normal heart rate) is
seen. This patient could possibly go on to have a spontaneous abortion, but,
at this point, she can be diagnosed with an intrauterine pregnancy.


During early pregnancy, the trophoblastic cells secrete which of the following
hormones in order to maintain the corpus Iuteum?
/ A. Estrogen
/ B. Human chorionic gonadotropin (hCG)
/ C. Luteinizing hormone (LH)
/ D. Progesterone
/ E. Testosterone


Explanation - Q: 5.2 Close

The correct answer is B. At approximately the midpoint of the menstrual
cycle, the ovum is expelled. Once this occurs, the surrounding granulosa and
theca cells undergo luteinization and begin to secrete a large amount of
estrogen and progesterone. This mass of hormone secreting cells is known
as the corpus luteum (because of its yellow color). If pregnancy does not
occur, the corpus luteum will persist for 14 days, after which point it
degenerates. With the degeneration of the corpus luteum, hormonal levels
drop, the endometrium sloughs, and menstruation occurs. If a pregnancy
does occur, the trophoblastic cells of the pregnancy will secrete human
chorionic gonadotropin (hCG). This hormone goes to the corpus luteum and
prevents its degeneration, signaling it to continue its excretion of estrogen
and progesterone to maintain an environment that is favorable for
pregnancy. The presence of human chorionic gonadotropin forms the basis
for the urine and serum pregnancy test. hCG steadily increases over the first
several weeks of the pregnancy, reaching a peak at approximately 10 weeks.
After that, hCG levels decrease gradually during the remainder of the
pregnancy. During early pregnancy, most women experience some nausea.
One of the leading theories for the cause of this nausea is the hormone hCG,
because the nausea of early pregnancy appears to mirror the rise and fall of
the hCG levels--peaking at roughly 10 weeks and improving after that.
Estrogen (choice A) and progesterone (choice D) are produced by the
corpus luteum during early pregnancy. The corpus luteum does not
degenerate in a pregnant woman, as it does in the nonpregnant woman,
because the trophoblastic cells secrete human chorionic gonadotropin
(hCG). After the first months of pregnancy, the placenta takes over the role
of secreting the large amounts of estrogen and progesterone that are needed
to continue a healthy pregnancy. Once the placenta has taken over this role,
the corpus luteum degenerates.
Luteinizing hormone (LH) (choice C) is produced by the anterior pituitary
gland. Its rate of secretion is controlled mainly by the hypothalamus, through
hypothalamic luteinizing hormone-releasing hormone (LHRH, also called
GnRH) secretion. The LH surge, which occurs roughly at the midpoint of the
menstrual cycle, appears to cause ovulation, which occurs approximately 24
hours after the surge. LH also appears to induce the theca and granulosa
cells to become the corpus luteum.
Testosterone (choice E) is formed by the interstitial cells of the testicles. It is
not known to be secreted by trophoblastic cells to maintain the corpus
luteum.


Which of the following is a derivative of the ectoderm of the trilaminar embryo?
/ A. Bone
/ B. Germ cells
/ C. Muscle
/ D. Spinal cord
/ E. Spleen



Explanation - Q: 5.3 Close

The correct answer is D. The neural plate, a thickened area of ectoderm,
gives rise to the nervous system. The first step is the development of a
groove in the neural plate. On either side of this groove are the raised neural
folds. These neural folds then begin to fuse in the midline at approximately
the midpoint of the neural plate. Fusion proceeds in a rostral and caudal
direction; once completed, there still remains an open area of the "tube" at
both the rostral and caudal ends. These openings are referred to as the
rostral and the caudal neuropore. The rostral neuropore closes around day
26 and the caudal neuropore closes on approximately day 28. With the
closure of the caudal neuropore, closure of the neural tube is completed. It
has been convincingly shown that folic acid plays an important role in the
closure of the neural tube. By giving women of childbearing age folic acid
supplements, the rate of neural tube defects can be reduced dramatically.
Yet, based on the embryology of the neural tube (with closure of the tube
roughly by day 28), it is obvious that in order for folic acid supplementation to
work, it should be started pregestationally. Unfortunately, most women do
not seek prenatal care until after the neural tube has undergone its period of
closure.
Bone (choice A), muscle (choice C), and spleen (choice E) are all
derivatives of the mesoderm. This mesoderm is formed at the end of the
second week post-fertilization, when some cells of the primitive streak
migrate laterally between the endodermal and the ectodermal layer.
Germ cells (choice B) are derived from the endodermal layer of the
trilaminar embryo. They migrate as primordial sex cells from the endoderm of
the yolk sac to the urogenital ridge.


Assays for serum human chorionic gonadotropin (hCG) are commonly used as a
pregnancy test. However, there have been several reported cases of false
positive results, that is, women who test positive and are told that they are
pregnant but later turn out not to be. If the false positive rate of serum hCG for
detecting pregnancy is < 0.5%, then what is the specificity of the test?
/ A. < 0.5%
/ B. > 0.5%
/ C. < 99.5%
/ D. > 99.5%
/ E. It cannot be determined from the information given


Explanation - Q: 5.4 Close

The correct answer is D. Serum and urine hCG tests are widely used to
determine if a patient is pregnant. The development and use of these tests
has been absolutely essential for the diagnosis and management of a variety
of conditions including pregnancy, ectopic pregnancy, spontaneous abortion,
gestational trophoblastic neoplasia, and some malignancies. However, it is
important to always remember that no test is 100% perfect. Numerous case
reports attest to the fact that using hCG to test for pregnancy will sometimes
yield incorrect results.
A false positive result refers to the situation when the test states that the
patient does have the disease or condition (in this case pregnancy) when, in
fact, the patient does not have the condition. A false positive rate of 0.5%
means that out of every 100 people who are not pregnant and take the
pregnancy test, 0.5 will be told that they are pregnant. Multiplying this result
by 2 (to get rid of the 0.5 person concept), it means that of every 200 people
who are not pregnant, 1 will be falsely told that they are. A false-positive rate
of < 0.5%, therefore, means that <1 nonpregnant person per 200
nonpregnant people that take the test will be told that they are pregnant.
The specificity of a test represents the percentage of individuals who do not
have a disease (or condition like pregnancy) who test negative by the
diagnostic test. Staying with the above example, if 200 people who are not
pregnant take the test, 199 of them will be told that they are not pregnant.
This represents a specificity of 99.5%. If the false positive rate is <0.5% then
the specificity is >99.5%. In general, the false positive rate (FPR) can be
calculated by the formula FPR = (1-specificity).





Vignette 1 of 4

A 26-year-old woman presents to her physician's office with multiple complaints
for the last few months. She has been feeling very anxious and will sometimes
feel that her heart is racing. She also has been feeling really hot despite the fact
that it is the middle of winter. She has been feeling very hungry, and has not
been able to sleep for more than 3 hours a day. Her last menstrual period was
"months ago" and she has been sexually active without using any contraception.
Her medical history is significant for an episode of untreated depression 2 years
ago. She denies taking any medications. On examination, she appears agitated
and is rocking her legs throughout the entire interview. She is talking very rapidly,
and interrupts the interviewer frequently. Her blood pressure is 146/80 mm Hg,
and her pulse is 103/min. Her examination is significant for some staring and a
slightly enlarged and firm thyroid. Routine laboratory studies, a urinary pregnancy
test, and an ECG are obtained.

Which of the following is the most likely preliminary diagnosis?
/ A. Anxiety attacks
/ B. Goiter
/ C. Hyperthyroidism
/ D. Manic episode of bipolar illness
/ E. Supraventricular tachycardia


Explanation - Q: 1.1 Close

The correct answer is C. Hyperthyroidism is suggested by the clinical
presentation. Symptoms of hyperthyroidism include anxiety, tremors,
insomnia, heat intolerance, and amenorrhea. Patients may or may not have
goiters. Suppression of TSH levels and elevation of free thyroxine levels
would confirm the likely diagnosis.
Patients with anxiety attacks (choice A) can have similar symptoms as
patients with hyperthyroidism, so that ruling out the latter is important in the
diagnosis of anxiety disorder. However, the "staring" and thyroid enlargement
suggest hyperthyroidism.
Patients with goiters (choice B) can be hyperthyroid, but can be euthyroid,
without any symptoms, as well.
Given the patient's history of depression, it is possible that the patient is
undergoing the manic phase of bipolar disorder (choice D). Again,
hyperthyroidism should be ruled out first prior to further psychiatric
evaluation.
Supraventricular tachycardia (choice E) can occur in young women, and can
be associated with hyperthyroidism. However, the patient with isolated
supraventricular tachycardia without hyperthyroidism would not have the
associated positive review of systems (e.g., insomnia, polyphagia, weight
loss).
Laboratory studies are significant for a TSH Ievel of 0.02 U/mL and serum free
T4 of 2.5 ng/dL (normaI 0.8-1.45 ng/dL).
Which of the following is the most likely diagnosis?
/ A. Chronic lymphocytic thyroiditis
/ B. Graves disease
/ C. Hashimoto disease
/ D. Subacute thyroiditis
/ E. Toxic multinodular goiter


Explanation - Q: 1.2 Close

The correct answer is B. Graves disease is the most common cause of
hyperthyroidism, and occurs mostly in young women. It is an autoimmune
disease in which immune cells produce a thyroid-stimulating immunoglobulin
(IgG TSI) that binds to and stimulates the thyroid TSH receptor. Graves
disease is defined by a triad of signs: hyperthyroidism with a diffuse
symmetric goiter, ophthalmopathy (the stare), and dermopathy (pretibial
myxedema). Graves disease is associated with an elevated radioiodine
uptake scan (RAIU), and elevated thyroglobulin and anti-thyroid peroxidase
antibodies.
Chronic lymphocytic thyroiditis/chronic thyroiditis (choice A) is a painless
inflammation of the thyroid associated with a transient thyrotoxicosis
progressing to a hypothyroid state. It can be differentiated from Graves
disease by having a low RAIU.
Hashimoto disease (choice C) is the most common cause of hypothyroidism,
and often presents with a multinodular, firm, and asymmetric goiter. Like
Graves disease, it is autoimmune in nature and tends to occur in women of
the same family. There are associated high titers of antithyroid peroxidase
(anti-TPO) antibodies and antithyroglobulin antibodies.
Patients with subacute thyroiditis (choice D) have a characteristically very
painful, nodular, and asymmetrically enlarged thyroid gland. This disorder
tends to occur after a viral upper respiratory infection. Labs show a low
RAIU, but a high erythrocyte sedimentation rate. Like chronic thyroiditis,
patients can first present in the thyrotoxic stage prior to becoming
hypothyroid, and then euthyroid.
Patients with toxic multinodular goiter (choice E) are also hyperthyroid, but
with an enlarged and nodular thyroid. As in Graves disease, the RAIU is
increased.


Which of the following tests is the best to establish the diagnosis in this patient?
/ A. Radioactive iodine uptake and scan
/ B. Serum antithyroglobulin antibodies
/ C. Serum anti-thyroid peroxidase antibodies
/ D. Serum thyroid stimulating hormone (TSH)
/ E. UItrasound of the neck


Explanation - Q: 1.3 Close

The correct answer is A. Radioactive iodine uptake scan (RAIU) is the main
tool to differentiate between the different causes of hyperthyroidism. It is
elevated in Graves disease and toxic nodular goiter. It is not used if the
patient is hypothyroid.
Antithyroglobulin levels (choice B) are more commonly elevated in Graves
disease but they can present in Hashimoto disease as well. They are also
used as a marker for thyroid cancer and used as a surveillance agent in
treated patients
Elevated anti-thyroid peroxidase antibodies (choice C) are more commonly
seen in patients with Hashimoto disease but they can be present in Graves
disease too.
Serum TSH levels (choice D) would not distinguish between Graves disease
and toxic nodular goiter.
An ultrasound (choice E) is only used to determine if a nodule within a goiter
is solid or cystic.

How could a physician differentiate between a patient with this patient's disease
and a patient who surreptitiously had ingested thyroid hormones to lose weight?
/ A. Needle aspiration or biopsy of the thyroid
/ B. Radioactive iodine uptake
/ C. Serum free T4 Ievels
/ D. Serum TSH Ievels
/ E. Thyroid scan



Explanation - Q: 1.4 Close

The correct answer is B. Both these patients will have elevated free T4
levels (choice C) and suppressed TSH levels (choice D) regardless of the
cause of hyperthyroidism. However, patients taking thyroid supplements will
have a low RAIU (choice B) versus a high RAIU in patients with Graves
disease.
A thyroid scan (choice E) is a nuclear study that indicates activity in the
thyroid nodules; a hot nodule has more active thyroid function than a cold
nodule. It is only used on nodular disease.


Which of the following will provide the most rapid relief of the patient's cardiac
signs and symptoms?
/ A. L-thyroxine
/ B. Methimazole
/ C. Nadolol
/ D. Potassium iodide
/ E. Propylthiouracil


Explanation - Q: 1.5 Close

The correct answer is C. Nadolol is a non-cardioselective beta-receptor
blocker used to treat the signs and symptoms of hyperthyroidism, such as
palpitations, anxiety, tremor, and heat intolerance. Beta-blockers may also
partially inhibit the peripheral conversion of T4 to T3. However, they do not
reduce thyroid-stimulating antibodies or prevent thyroid storm. These agents
will produce a rapid reduction of the patients cardiac signs and symptoms. It
is important to note that although methimazole (choice B), potassium iodide
(choice D), and propylthiouracil (choice E) are indicated for the treatment of
Graves disease, nadolol will produce the fastest reduction of cardiac signs
and symptoms.
L-thyroxine (choice A) is indicated for treatment of hypothyroidism. The use
of this agent in this patient would worsen the patient's condition.
Methimazole (choice B) and propylthiouracil (PTU) (choice E) act by
inhibiting the synthesis of the thyroid hormones. PTU and methimazole do
NOT inactivate existing T4 and T3 and PTU is able to inhibit the peripheral
conversion of T4 to T3. The full therapeutic effect usually occurs 4 - 8 weeks
after the initiation of therapy; the symptoms should be diminished and the
thyroid hormones should be back to within normal limits. These agents are
indicated for long-term hyperthyroid therapy, which may lead to disease
remission, and for short-term treatment before thyroidectomy or radioactive
iodine therapy. PTU may also be useful for decreasing mortality due to
alcoholic liver disease by reducing the hepatic hypermetabolic state induced
by alcohol (unlabeled indication).
Potassium iodide (choice D) was the first therapeutic option for treatment of
Graves disease. It inhibits the release of T4 and T3, inhibits the biosynthesis
of T4 and T3, and decreases the size and vascularity of the thyroid gland.
The therapeutic effects begin to be seen in approximately 2 - 7 days. If used
on a chronic basis, the therapeutic effects can be observed for up to 6
weeks. T4 and T3 concentrations may return to normal for a few weeks.







Vignette 2 of 4

A 54-year-old man presents to the emergency department with palpitations and
shortness of breath. His temperature is 37 C (98.6 F), blood pressure is 102/68
mm Hg, pulse is 130/min, and respirations are 26/min. His jugular venous
pressure is elevated. Cardiac examination reveals an irregular rate and rhythm,
with a low-pitched diastolic murmur, preceded by an opening snap, heard best
over the apex. Lung examination reveals bibasilar crackles. There is also 2+
bilateral lower extremity edema. An electrocardiogram reveals an irregular
undulation of the baseline, an absence of P waves, and narrow QRS complexes
that are irregularly irregular.


Which of the following is the most likely preliminary diagnosis?
/ A. Atrial arrhythmia
/ B. First degree atrioventricular block
/ C. Normal sinus rhythm
/ D. Third degree atrioventricular block
/ E. Ventricular arrhythmia


Explanation - Q: 2.1 Close

The correct answer is A. The presence of narrow QRS complexes indicates
the rhythm is from a supraventricular source and conducted to the ventricles
through the atrioventricular conduction system. In this case, it is from the
atria, and therefore an atrial arrhythmia.
Atrioventricular block refers to an abnormality in the electrical conduction
between the atria and the ventricles. The degree of the block refers to the
severity of the conduction. In first degree atrioventricular block (choice B), all
electrical impulses are conducted with a delay. In third degree
atrioventricular block (choice D), there is no conduction of the electrical
impulse and complete heart block.
Normal sinus rhythm (choice C) is a normal rhythm. It is produced by
electrical impulses formed in the sinoatrial node. In the electrocardiogram,
these impulses are seen as P waves followed by narrow QRS complexes.
Ventricular arrhythmias (choice E) occur when the electrical impulse is
generated in the ventricles. Because the conduction of the impulse polarizes
the ventricles at different times, the QRS complex is seen as widened on the
electrocardiogram.


Which of the following best describes the patient's cardiac rhythm?
/ A. Atrial fibrillation
/ B. Atrial flutter
/ C. Multifocal atrial tachycardia
/ D. Sinus bradycardia
/ E. Sinus tachycardia

Explanation - Q: 2.2 Close

The correct answer is A. The irregular undulation of the baseline and the
absence of P waves indicate that the atria are not contracting in an
organized manner. This is known as atrial fibrillation. Atrial fibrillation is also
characterized by irregularly irregular QRS complexes.
Atrial flutter (choice B) is characterized by a rapid atrial rate between 240 to
400 beats per minute. It is often seen as a saw tooth pattern of F waves on
the electrocardiogram. Because of the refractoriness of the atrioventricular
node, the flutter waves are not generally all transmitted through to the
ventricles. The conduction of these waves can vary from 2:1 conduction or
higher.
Multifocal atrial tachycardia (choice C) is an atrial arrhythmia characterized
by different P wave shapes with varying PR intervals. It is associated with
severe underlying lung disease.
Sinus bradycardia (choice D) is a slow sinus rhythm of less than 60 beats
per minute. It is characterized by QRS complexes that follow each P wave.
Sinus tachycardia (choice E) is a rapid sinus rate of greater than 100 beats
per minute. A QRS complex also follows each P wave. It is generally a
cardiac response to pain, fever, infection, vigorous exercise, shock,
dehydration, anxiety, heart failure, or anemia.


Which of the following is the underlying cause of this patient's abnormal rhythm?
/ A. Aortic stenosis
/ B. Hyperthyroidism
/ C. Hypothyroidism
/ D. Mitral stenosis
/ E. Sepsis


Explanation - Q: 2.3 Close

The correct answer is D. The low-pitched diastolic murmur, which is
preceded by an opening snap, is diagnostic of mitral stenosis. Mitral stenosis
is most often due to rheumatic heart disease, and it often leads to atrial
arrhythmias, especially atrial fibrillation.
Aortic stenosis (choice A) is characterized by a systolic crescendo-
decrescendo murmur best heard over the second intercostal space along the
right sternal border. It is generally not associated with atrial fibrillation.
Hyperthyroidism (choice B) can also be associated with atrial fibrillation, but
in this case, the patient has no symptoms suggestive of hyperthyroidism.
These symptoms include heat intolerance, hyperdefecation, tremulousness,
hair loss, and excessive sweating.
Hypothyroidism (choice C) is not associated with atrial fibrillation. It is
associated with sinus bradycardia.
Sepsis (choice E) can be associated with atrial fibrillation, but it occurs in the
setting of a disseminated infection that causes vascular compromise.


The patient is treated with amiodarone to control his cardiac rhythm. Which of the
following is the mechanism of action of this medication?
/ A. Beta adrenergic receptor blocker
/ B. Calcium channel blocker
/ C. CIass 1B sodium channel blocker
/ D. CIass 1C sodium channel blocker
/ E. Potassium channel blocker


Explanation - Q: 2.4 Close

The correct answer is E. Amiodarone is an antiarrhythmic agent classified
as a potassium channel blocker. It acts to prolong the action potential in
phase 3.
Beta adrenergic receptor blockers (choice A) include propranolol, esmolol,
timolol, and metoprolol. As antiarrhythmic agents, they decrease cAMP
levels, decrease calcium currents, and suppress abnormal pacemakers.
Calcium channel blockers (choice B) include verapamil, diltiazem, and
bepridil. They decrease conduction velocity and increase the PR interval.
They are used in the prevention of nodal arrhythmias.
Class 1B sodium channel blockers (choice C) include lidocaine, mexiletine,
and tocainide. They act to depress the action potential and stabilize the cell
membrane. They are used in acute ventricular arrhythmias and digitalis-
induced arrhythmias.
Class 1C sodium channel blockers (choice D) include flecainide, encainide,
and propafenone. They depress the action potential at phase 0 and stabilize
cell membranes. They are used as a last resort in refractory
tachyarrhythmias due to their side effects.


Which of the following is a known adverse reaction associated with amiodarone?
/ A. Diabetes mellitus
/ B. Gastrointestinal bleeding
/ C. Hallucinations
/ D. Pulmonary disease
/ E. Renal failure


Explanation - Q: 2.5 Close

The correct answer is D. Amiodarone is associated with many adverse
reactions, including pulmonary disease. Patients can develop an alveolitis or,
in more serious cases, pulmonary fibrosis.
Diabetes mellitus (choice A) is not associated with amiodarone use.
Amiodarone can cause endocrine abnormalities which affect the thyroid
gland, resulting in hypo- or hyperthyroidism.
Gastrointestinal bleeding (choice B) is not associated with amiodarone use.
Amiodarone can cause nausea, vomiting, or abdominal discomfort, but it is
not associated with bleeding.
Hallucinations (choice C) are not associated with amiodarone use. Digoxin,
which is also used to treat atrial fibrillation, is associated with hallucinations
and delirium.
Renal failure (choice E) is not associated with amiodarone. Amiodarone is
associated with elevations in liver enzymes, or in more severe cases, it can
cause hepatotoxicity.


Because the patient is at risk for thrombosis, he is anticoagulated with warfarin.
Which of the following is the mechanism of action of warfarin?
/ A. Activation of antithrombin lll
/ B. Fibrinolysis
/ C. Inhibition of antithrombin lll
/ D. Inhibition of synthesis of clotting factors lI, VII, IX, and X
/ E. Inhibition of synthesis of clotting factors XI and XII


Explanation - Q: 2.6 Close

The correct answer is D. Warfarin inhibits vitamin K-dependent coagulation
factor synthesis. These include factors II, VII, IX, and X, as well as protein C
and S. Warfarin acts to increase the prothrombin time. Factors XI and XII are
not vitamin K-dependent factors. Therefore, warfarin does not inhibit their
synthesis (choice E).
Activation of antithrombin III (choice A) is a mechanism of action used by
heparin. It acts to prolong the partial thromboplastin time.
Fibrinolysis (choice B) is a mechanism of action used by thrombolytic agents
such as streptokinase and urokinase. These agents convert plasminogen to
plasmin, promoting fibrinolysis.
Inhibition of antithrombin III (choice C) would promote clot formation. The
goal of anticoagulation is to prevent clot formation.



Vignette 3 of 4

A 23-year-old college senior with type 1 diabetes was preparing for an 11 a.m.
examination. He had taken insulin that morning, but was very nervous about the
test and forgot if he had eaten anything for breakfast. With time, he felt his heart
pounding, his palms were sweaty, his hands were shaking, and he felt dizzy. He
wanted to take a sugar tablet, but at that moment, his friend arrived and they
rushed to schooI. In the car, he developed a headache and difficulties
concentrating on the conversation, but he thought all of these symptoms were
the result of the nervousness caused by the approaching exam. He was
confused, the speech became slurred, and finally he lost consciousness. His
friend brought him to the emergency department. On site, his blood glucose level
was 40 mg/dL. After medical intervention, he successfully recovered without
sequelae.


Administration of which of the following pancreatic hormones would be expected
to help this patient?
/ A. Amylin
/ B. GIucagon
/ C. Insulin
/ D. Pancreatic polypeptide
/ E. Somatostatin


Explanation - Q: 3.1 Close

The correct answer is B. Glucagon is a single-chain peptide of 29 amino
acids, produced by alpha cells of the endocrine pancreas, and is structurally
related to the secretin family of peptide hormones. Glucagon is synthesized
as proglucagon and processed to give glucagon within the pancreatic islets.
Within the intestinal tract, proglucagon is processed to a family of glucagon-
like peptides. The major effect of glucagon is to stimulate an increase in
glucose concentrations. The mechanism involves stimulation of hepatic
glycogenolysis and hepatic gluconeogenesis, and inhibition of glycogen
synthesis. In the liver, glucagon also activates the transport of long-chain
free fatty acids into mitochondria for oxidation and ketogenesis. It appears to
have a minor effect of stimulating lipolysis in adipose tissue, thereby
providing fatty acid fuels to most cells and conserving blood glucose. The
molecular mechanism of glucagon action involves activation of adenylate
cyclase via Gs and an increase in cAMP, which induces gene expression of
PEPCK (phosphoenolpyruvate carboxykinase), a key gluconeogenic
enzyme. On the other hand, cAMP-activated PKA (protein kinase A)
activates phosphorylase and inactivates glycogen synthase in the liver.
Glucagon is used to treat severe hypoglycemic episodes when oral glucose
is inadequate, and IV glucose is not available (emergencies away from
medical settings). The usual dose of glucagon in adults is 0.5 to 1.0 U given
SC, IM. or IV.
Amylin (choice A) is a beta cell hormone that is colocalized and cosecreted
with insulin. It appears to work in concert with insulin to regulate glycemia,
suppressing the postprandial secretion of glucagon and slowing the rate of
gastric emptying.
Insulin (choice C) is the only hormone that causes hypoglycemia.
F or D1 cells produce pancreatic polypeptide (choice D). The function of this
hormone is still uncertain, but it is suggested that it may influence
gastrointestinal function and promote intraislet homeostasis.
D cells produce somatostatin (choice E), which inhibits insulin and glucagon
secretion locally (paracrine effect) within the pancreatic islets.


Beta-receptor antagonists prevent many of the premonitory signs of this
condition, which are caused by an epinephrine response. Which of the following
signs will still be present despite beta blockade?
/ A. BIurred vision
/ B. Palpitations
/ C. Sweating
/ D. Tachycardia
/ E. Trembling


Explanation - Q: 3.2 Close

The correct answer is C. Beta receptor blockade prevents the warning
signs of hypoglycemia caused by epinephrine release. However, sweating
will still occur despite the beta blockade. Human skin contains two types of
sweat glands: eccrine (merocrine) and apocrine sweat glands. Eccrine sweat
glands are the predominant type. They are found all over the body,
particularly on the palms of the hands, soles of the feet and forehead.
Apocrine glands are located in the perigenital area, and in the axilla. They
produce a viscous, more protein-rich secretion than eccrine glands, and are
innervated by the sympathetic adrenergic nerves via alpha1 receptors.
Eccrine sweat glands are activated by sympathetic cholinergic nerves
(muscarinic M
3
receptors). Therefore, hypoglycemic sweating will still be
evident in the presence of beta-adrenergic blockade.
Beta
2
-mediated epinephrine responses an cause relaxation of the ciliary
muscle and consequent blurred vision (choice A).
Palpitations (choice B) are unpleasant sensations of irregular/and or forceful
beating of the heart. They are elicited through activation of beta
1
and beta
2

receptors that result in increased automaticity (phase 4 depolarization) and
conduction velocity, and via a separate set of beta
2
adrenergic receptors, an
increase in contractility.
Tachycardia (choice D) is a result of epinephrine-induced activation of beta
1

and beta
2
receptors located at the cells of the SA (sinoatrial) node.
Trembling (choice E) is mediated through beta-adrenergic receptors.


Which additional action occurs as a result of this epinephrine response?
/ A. Inhibition of glucose uptake
/ B. Stimulation of gluconeogenesis
/ C. Stimulation of glucose uptake
/ D. Stimulation of glycogenolysis
/ E. Stimulation of glycogen synthesis


Explanation - Q: 3.3 Close

The correct answer is D. Under normal conditions, increased glycogen
release is the major counterregulatory response to hypoglycemia. However,
in patients with diabetes mellitus, glucagon secretion in response to
hypoglycemia is defective, and under these circumstances, epinephrine
secretion becomes the critical counterregulatory factor. The absent glucagon
response results from the loss of the intraislet insulin-inhibition of glucagon
release. Stimulation of glycogenolysis in liver provides the major source for
circulating glucose, because liver cells contain glucose-6-phosphatase.
Epinephrine stimulates hepatic glycogenolysis via adrenergic beta
2

receptors, thereby increasing the delivery of glucose to the circulation. It
activates adenylate cyclase, and the increased cAMP activates protein
kinase A (PKA). PKA catalyzes the phosphorylation of phosphorylase kinase
to convert it from the inactive form b to the active form a. Subsequently,
activated phosphorylase kinase a then catalyzes the phosphorylation of
glycogen phosphorylase-b to produce an active form of the enzyme. At the
same time, PKA converts glycogen synthase to the inactive form, which
prevents resynthesis of glycogen. The hormone also stimulates
glycogenolysis in muscle through the activation of beta
2-
adrenergic
receptors. Because muscle lacks the enzyme glucose-6-phosphatase, the
glucose-6-phosphate enters the glycolytic sequence to give pyruvate.
Pyruvate is converted to lactate, and lactate produced in muscle is released
to the blood stream and transported to the liver, where it is converted to
glucose in the process of gluconeogenesis. The glucose is then returned to
the blood and can be used as an energy source by the brain and by muscle.
This cycle is termed the Cori cycle. Epinephrine also stimulates lipolysis in
adipocytes through activation of alpha-adrenergic receptors, and glycerol,
the product of fat tissue lipolysis, serves as an additional gluconeogenic
substrate.
Cortisol inhibits glucose uptake (choice A), stimulates hepatic
gluconeogenesis, stimulates protein breakdown in muscle, and stimulates
lipolysis, all of which help restore the glucose level in hypoglycemia.
However, the cortisol response is delayed, and less critical than the
epinephrine response.
Growth hormone stimulate lipolysis in adipocytes, and gluconeogenesis
(choice B) in liver, thereby acting as a second-line counterregulatory
hormone.
Insulin stimulates glucose uptake into cells (choice C), which leads to
hypoglycemia.
Insulin increases glucose storage as glycogen in liver. In skeletal muscle,
insulin promotes glycogen synthesis (choice E) by increasing glucose
transport, inducing glycogen synthase and inhibiting phosphorylase.


After he is successfully treated and released from the hospitaI, which of the
following compounds will be the major substrate for his brain metabolism?
/ A. Fatty acids
/ B. GIucose
/ C. Ketone bodies
/ D. Mannose
/ E. Triacylglycerols


Explanation - Q: 3.4 Close

The correct answer is B. The brain represents 2% of body weight, but
receives 15% of the cardiac output, consumes 20% of the total O
2
, and
utilizes 25% of the total glucose. Glucose is the major, and nearly the sole
source of energy for the brain. Therefore, plasma glucose is normally very
carefully regulated to maintain the level that ensures glucose transport into
the brain at adequate rates. Insulin is not required for brain cells to utilize
glucose. The carbohydrate reserves in brain tissues are extremely limited,
and normal function depends upon continuous glucose supply. Under normal
conditions, the brain takes three times more glucose than it needs from the
circulation. At some critical glucose concentration (approx. 70 mg/dL),
centers in the hypothalamus sense a fall in the blood glucose level, and the
release of glucagon and epinephrine is triggered. If the blood glucose level is
below 30 mg/dL, coma develops, and below 15 mg/dL, permanent brain
damage and/or death ensue.
Astrocytes, a major class of glial cells in mammalian brain, play a pivotal role
in the regulation of brain metabolism by providing neurons with anaplerotic
metabolites and substrates for energy generation. Glucose enters the brain
via an insulin-independent GLUT 1 transporter in cerebral capillaries. The
entrance of glucose into the CNS from the capillaries occurs primarily into
astrocytes, which metabolize glucose through the glycolytic pathway. This
mechanism seems to be coupled with the transport of glutamate into
astrocytes by a Na
+
-cotransporter, and this secondary active transport
produces ADP, which serves as a major allosteric effector for glycolysis.
Glycolysis produces lactate, which is taken up and metabolized by neurons,
and through oxidative phosphorylation, ATP is produced. Amino acids, lipids,
and proteins derived from glucose can be metabolized for energy only during
certain conditions.
The blood brain barrier excludes free fatty acids (FFA) (choice A),
preventing them from entering into brain metabolism.
Acetoacetate, beta-hydroxybutyrate, and acetone are known as ketone
bodies (choice C). The rate of ketone transport into the brain is too slow to
meet its energy requirements unless fasting ketone body plasma levels are
markedly increased. But, under particular conditions, such as starvation,
diabetes, or in breast-fed neonates, plasma levels of ketone bodies increase
markedly, and the brain cells can switch to ketone bodies as substrates for
their metabolism. Without ketone bodies, most of us would be unconscious
after 48 hours of fasting.
Mannose (choice D) theoretically can substitute for glucose as an alternative
substrate for brain metabolism. It crosses the blood-brain barrier and it is
converted to fructose-6-phosphate, a physiological intermediate of the
glycolytic pathway. However, mannose is not normally present in the blood,
and cannot be considered a physiologic metabolic substrate.
Brain cells can sometimes use triacylglycerols (choice E) for energy
metabolism, but the quantitative importance of this pathway is negligible.


If this patient's symptoms were due to ingestion of a drug, which of the following
agents would he have most likely ingested?
/ A. CIozapine
/ B. Diazoxide
/ C. GIucocorticoids
/ D. Pentamidine
/ E. Thiazide diuretic


Explanation - Q: 3.5 Close

The correct answer is D. Pentamidine is used to treat the Pneumocystis
carinii pneumonia that occurs commonly in immunocompromised patients,
such as cancer patients, AIDS patients, and transplant patients. Pentamidine
side effects include metallic taste, coughing, bronchospasm in heavy
smokers and asthmatics, decrease in urination, unusual bleeding or bruising,
and hypoglycemia. Some patients may develop sudden, severe low blood
pressure after receiving pentamidine. Therefore, it is recommended that the
patient lie down during the administration of the medicine. This agent is also
used in patients with cutaneous and visceral leishmaniasis (kala-azar)
caused by Leishmania donovani, and trypanosomiasis (Trypanosoma brucei,
gambiense, and rhodesiense). The drug's hypoglycemic effect is due to lytic
destruction of pancreatic beta cells, causing acute hyperinsulinemia and
hypoglycemia. Later on, insulinopenia and hyperglycemia may develop. IV
glucose should be administered during pentamidine administration, and
during the period immediately after that.
Clozapine (choice A) is a dibenzodiazepine derivative and an atypical
antipsychotic. Besides well known-side effects, such as agranulocytosis,
seizures, weight gain, constipation, and hypersalivation, recent studies show
an association of clozapine with hyperglycemia and diabetes.
Diazoxide (choice B) opens potassium channels in vascular muscle cells,
stabilizing the membrane potential and preventing smooth muscle
contraction. This leads to arteriolar dilatation and reduction of mean arterial
blood pressure. The use of this agent has been associated with
hyperglycemia and hyperosmolar nonketotic coma. Diazoxide inhibits insulin
secretion, and is used to treat hypoglycemia secondary to insulinoma.
Glucocorticoids (choice C) stimulate gluconeogenesis and decrease insulin
sensitivity, which both lead to hyperglycemia.
Hyperglycemia secondary to thiazide diuretics (choice E) is thought to be
related to the depletion of potassium. Thiazides may decrease insulin
secretion and contribute to the development of insulin resistance. Other
drugs, such as furosemide, nicotinic acid, and oral contraceptives also can
induce a hyperglycemic state.




Vignette 4 of 4

A 55-year-old man with a 30-year history of alcohol abuse is brought into the
emergency department with palpitations and chest discomfort. His temperature is
37.1 C (98.8 F), blood pressure is 98/56 mm Hg, pulse is 130/min, and
respirations are 26/min. Cardiac examination reveals tachycardia with a regular
rhythm. An electrocardiogram reveals wide, monomorphic QRS complexes and P
waves occurring independently of the QRS complexes.

What general classification of arrhythmia does this patient have?
/ A. Atrial fibrillation
/ B. Atrial flutter
/ C. Ventricular arrhythmia
/ D. 1st degree atrioventricular block
/ E. 2nd degree atrioventricular block


Explanation - Q: 4.1 Close

The correct answer is C. Ventricular arrhythmias occur when the electrical
impulse is generated in the ventricles. The presence of dissociative P waves
also indicates that the impulse is originating in the ventricles. Because the
conduction of the impulse polarizes the ventricles at different times, the QRS
complex is seen as widened on the electrocardiogram.
Atrial fibrillation (choice A) is characterized by irregularly irregular QRS
complexes. Furthermore, there is an irregular undulation of the baseline and
an absence of P waves, which would indicate that the atria are not
contracting in an organized manner.
Atrial flutter (choice B) is characterized by a rapid atrial rate between 240 to
400 beats per minute. It is often seen as a sawtooth pattern of F waves on
the electrocardiogram. Because of the refractoriness of the atrioventricular
node, the flutter waves are not generally all transmitted through to the
ventricles. The conduction of these waves can vary from 2:1 conduction or
higher.
Atrioventricular block refers to an abnormality in the electrical conduction
between the atria and the ventricles. The degree of the block refers to the
severity of the conduction. In 1st degree atrioventricular block (choice D), all
electrical impulses are conducted with a delay. In 2nd degree atrioventricular
block (choice E), only some of the impulses are conducted, and if they are
conducted, they can be conducted with a delay.


Which of the following is the actual cardiac rhythm?
/ A. Multifocal atrial tachycardia
/ B. Normal sinus rhythm
/ C. Sinus tachycardia
/ D. Ventricular fibrillation
/ E. Ventricular tachycardia


Explanation - Q: 4.2 Close

The correct answer is E. Ventricular tachycardia is usually a result of a
reentrant pathway in the cardiac conduction system. It is defined as at least 3
consecutive QRS complexes originating from the ventricles and occurring at
a rapid rate (over 100 beats per minute).
Multifocal atrial tachycardia (choice A) is an atrial arrhythmia characterized
by different P wave shapes with varying PR intervals. It is associated with
severe underlying lung disease.
Normal sinus rhythm (choice B) is a normal rhythm. It is produced by
electrical impulses formed in the sinoatrial node. In the electrocardiogram,
these impulses are seen as P waves followed by narrow QRS complexes. In
ventricular tachycardia, the P waves are dissociated from the QRS complex.
Sinus tachycardia (choice C) is a rapid sinus rate of greater than 100 beats
per minute. A QRS complex also follows each P wave. It is generally a
cardiac response to pain, fever, infection, vigorous exercise, shock,
dehydration, anxiety, heart failure, or anemia.
Ventricular fibrillation (choice D) occurs when the ventricles contract in an
unorganized manner. There are no clearly formed QRS complexes or T
waves. The patient is in cardiac arrest and does not have a pulse.


After cardioversion, the patient is treated with esmoloI. Which of the following is
the mechanism of action of esmoloI?
/ A. Calcium channel blockade
/ B. CIass 1B sodium channel blockade
/ C. Nonselective beta receptor blockade
/ D. Potassium channel blockade
/ E. Selective beta-1 receptor blockade


Explanation - Q: 4.3 Close

The correct answer is E. Esmolol selectively antagonizes beta-1 receptors.
Other selective beta-1 receptor blockers include metoprolol and atenolol.
Nonselective beta receptor blockers (choice C) include propanolol, timolol,
labetalol, and nadolol. Nonselective beta receptor blockers block beta-1 and
beta-2 receptors relatively equally.
Calcium channel blockers (choice A) include verapamil, diltiazem, and
bepridil. They decrease conduction velocity and increase the PR interval.
They are used in the prevention of nodal arrhythmias.
Class 1B sodium channel blockers (choice B) include lidocaine, mexiletine,
and tocainide. They act to depress the action potential and stabilize the cell
membrane. They are used in acute ventricular arrhythmias and digitalis-
induced arrhythmias.
Potassium channel blockers (choice D) act to prolong the action potential in
phase 3. They include amiodarone, bretylium, and sotalol.

The physician also starts the patient on bumetanide. Which of the following is
bumetanide's mechanism of action?
/ A. Angiotensin converting enzyme inhibitor
/ B. Carbonic anhydrase inhibitor
/ C. Loop diuretic
/ D. Potassium sparing diuretic
/ E. Thiazide diuretic

Explanation - Q: 4.4 Close

The correct answer is C. Bumetanide is a loop diuretic. It inhibits the
sodium, potassium, chloride cotransporter in the thick ascending limb of the
loop of Henle.
Angiotensin converting enzyme inhibitors (choice A) include benazepril,
captopril, fosinopril, and lisinopril. They interfere with the conversion of
angiotensin I to angiotensin II.
Carbonic anhydrase inhibitors (choice B), such as acetazolamide, act in the
proximal convoluted tubule. They cause a self-limited sodium bicarbonate
diuresis. They are used to alkalinize the urine, and for the treatment of
metabolic alkalosis and glaucoma.
Potassium-sparing diuretics (choice D) such as spironolactone, triamterene,
and amiloride inhibit the action of aldosterone. They are used in the
treatment of hyperaldosteronism and potassium depletion.
Thiazide diuretics (choice E), such as hydrochlorothiazide, inhibit sodium
chloride reabsorption in the early distal tubule. They are used in the
treatment of hypertension, congestive heart failure, and to inhibit renal
calcium stone formation.


Vignette 1 of 4

A 65-year-old man undergoes routine biochemical screening of his serum, which
yields the following results:

AIanine aminotransferase 30 U/L
AIbumin 4.1 g/dL
AIkaline phosphatase 950 U/L
Aspartate aminotransferase 32 U/L
Bilirubin 0.8 mg/dL
BIood urea nitrogen 15 mg/dL
Calcium 9.1 mg/dL
Chloride 102 mmoI/L
Creatinine 0.9 mg/dL
GIucose 80 mg/dL
Phosphorus 3.2 mg/dL
Potassium 4.1 mmoI/L
Prostate specific antigen 2 ng/mL
Sodium 141 mmoI/L


The abnormality shown in the laboratory values is most often seen in diseases of
which of the following?
/ A. Bone and prostate
/ B. Heart and prostate
/ C. Liver and bone
/ D. Liver and heart
/ E. Liver and prostate




Explanation - Q: 1.1 Close

The correct answer is C. This patient has an isolated elevation of the serum
enzyme alkaline phosphatase. This enzyme can nonspecifically remove
phosphate groups, including the 5'-phosphate groups of DNA and RNA, and
phosphates from individual nucleotides and proteins. It operates best at an
alkaline pH. All of the other laboratory tests illustrated in the question stem
are within normal limits, including those listed in the other choices. Alkaline
phosphatase is actually a group of related enzymes that are found in highest
concentration in liver, biliary tract, bone, intestinal mucosa, and placenta.
Most clinically significant elevations of alkaline phosphatase are due to liver
or bone disease.
Prostate (choices A, B, and E) contains acid phosphatase, which is used as
a serum marker.
Disease of the heart (choices B and D) can cause elevation of a variety of
serum enzymes including creatine kinase, but not usually including alkaline
phosphatase.





The patient returns to the physician's office where a complete, careful physical
examination is performed. This is notable for subtle changes in the skull with
frontaI "bossing," dilated scalp veins, a short, kyphotic trunk; and anterolateral
bowing of one leg. X-ray studies of the affected bones show increased density
with abnormal architecture, cortical thickening and overgrowth. Microfractures
are noted in the femur of the bowed leg. No distinct masses are noted. Which of
the following is the most likely diagnosis?
/ A. Metastatic prostate cancer
/ B. Multiple myeloma
/ C. Osteoporosis
/ D. Osteosarcoma
/ E. Paget disease



Explanation - Q: 1.2 Close

The correct answer is E. While theorists would like to imagine that every
patient's notable physical findings are picked up on the initial examination,
you should be aware that in real life, subtle clues are sometimes only noticed
after some other finding suggests the possibility of a disease. This is
particularly true when a patient's disease develops over decades, and the
patient, family, and physician may all become so accustomed to the patient's
appearance as to not notice even prominent, but gradual, changes. In this
case, a cluster of skeletal changes is noted, which had been previously
unnoticed. Paget disease of bone (not to be confused with the rare form of
breast cancer with skin involvement knows as Paget disease of breast) is a
chronic disorder of bony remodeling that can cause enlargement and
deformity of bones. The modified bones appear strong, but are actually
weak. The underlying pathophysiology appears to involve abnormal
osteoclasts that absorb too much bone, and are then countered by an also
overly active osteoblast population that compensates by laying too much
new bone down, which forms in an irregular manner. The disease affects
men and women over the age of 40, and is thought to have an incidence of
3-4% in this population (up to 10-20% of people over 60 years of age). It
develops slowly and usually only affects selected bones, rather than all of a
patient's bones. There is thought to be a genetic predisposition for the
disease, as it may occur in families. A slow-virus infection by a virus in the
measles family has also been implicated. The disease may be diagnosed
incidentally, as in this case, or when it produces symptoms related to bony
changes (often related to pressure on nerves or vessels by narrowed ostia)
such as bone pain, headache, hearing loss, nerve palsies, pathologic
fracture, or secondary arthritis. Other complications can include heart
disease related to a shunt due to increased vascularity of the bony lesions,
kidney stones, teeth changes, and rarely, sarcoma. Once the diagnosis is
established by the characteristic x-ray findings, the extent of disease can be
evaluated with bone scan.
Metastatic prostate cancer (choice A) would most likely produce multiple
mass lesion of the bones, which are often osteoblastic.
Multiple myeloma (choice B) produces lytic bone lesions.
Osteoporosis (choice C) produces diffuse thinning of bone.
Osteosarcoma (choice D) would produce a mass that often extends outside
the bone.


Bone biopsy in an involved area of bone would most likely show which of the
following?
/ A. Benign spindle cells growing in a storiform pattern
/ B. Cartilaginous nodule within a bony medullary cavity
/ C. Mosaic pattern of lamellar bone
/ D. Spirochetes visible with silver stain
/ E. Subperiosteal abscess formation



Explanation - Q: 1.3 Close

The correct answer is C. The characteristic finding on histologic
examination of a biopsy from a patient with Paget disease of bone is a
mosaic pattern of lamellar bone. This pattern, which resembles a jigsaw
puzzle in appearance, reflects the tight apposition of irregular units of bone
that adhere to one another with prominent cement lines. These findings are
characteristic of fully developed Paget disease, and are the ones most likely
to be illustrated on an examination. However, you should be aware that early
Paget disease typically shows lytic rather than proliferative bone lesions, and
it is thought that the primary defect involves the osteoclasts, which are
typically abnormally large with more than the usual number of nuclei. This is
followed by overly vigorous osteoblastic activity, which eventually produces
the characteristic mosaic bone.
Benign spindle cells growing in a storiform pattern (choice A) are
characteristic of fibrous cortical defect and nonossifying fibroma.
A cartilage nodule within a bony medullary cavity (choice B) is characteristic
of enchondroma.
Spirochetes visible with silver stain (choice D) are a feature of skeletal
syphilis.
Subperiosteal abscess formation (choice E) is often seen in pyogenic
osteomyelitis.



While in the past, only symptomatic treatment was available for this patient's
condition, it is now known that many cases can be markedly slowed by treatment
with which of the following drugs?
/ A. Benzodiazepines
/ B. Bisphosphonates
/ C. Calcium channel antagonists
/ D. HMG-CoA reductase inhibitors
/ E. MAO inhibitors


Explanation - Q: 1.4 Close

The correct answer is B. Bisphosphonate therapy has quietly revolutionized
the treatment of Paget disease, because therapy with such drugs can
suppress the abnormal osteoclast population that contributes so much to the
pathologic process. The osteoblastic process is also then secondarily
slowed, and the healing bone that forms has a more normal appearance and
strength. These drugs contain two phosphate groups linked by a carbon
atom rather than by an oxygen atom, and different members of the class
then have different attached R groups. The most established member of the
class is etidronate disodium; newer members are pamidronate sodium,
alendronate sodium, tiludronate disodium, and risedronate sodium.
Calcitonin is also used to treat Paget disease patients. Analgesics are often
added for pain control, and occasional surgeries may replace affected joints
or remove pieces of bone impinging on other structures.
Benzodiazepines (choice A) are used to treat anxiety states and sleep
disorders.
Calcium channel antagonists (choice C) are used to treat hypertension and
angina.
HMG-CoA reductase inhibitors (choice D) are used to treat hyperlipidemia.
MAO inhibitors (choice E) are used to treat depression.


Osteosarcoma is a feared complication of this patient's disease that typically
causes death within 3 years if it develops.
This complication occurs in what percentage of patients with this patient's
disease?

/ A. Less than 1%
/ B. 5-15%
/ C. 25-40%
/ D. 60-75%
/ E. More than 95%


Explanation - Q: 1.5 Close

The correct answer is A. Patients should be warned that osteogenic
sarcoma can complicate Paget disease, but they should also be assured that
this complication is rare. Osteogenic sarcoma typically presents with
increasing severe pain affecting an involved bone, and the diagnosis can be
established with radiology followed by bone biopsy.




Vignette 2 of 4

A 42-year-old man presents to his doctor with complaints of left-sided flank pain
radiating into his groin. The pain began suddenly and has increased in intensity.
He had a similar problem several years ago, but he did not seek medical
attention, and his symptoms resolved spontaneously. He denies fever, chills,
nausea, or vomiting. He has had no diarrhea or difficulty urinating. Physical
examination is remarkable for tenderness along the left flank and into the groin,
but is otherwise normaI. There are no inguinal hernias detected on examination.
A urinalysis reveals microscopic hematuria without casts. Serum chemistry
reveals:

Sodium: 143 mEq/L
Potassium: 2.4 mEq/L
Bicarbonate: 17 mEq/L
Chloride: 115 mEq/L
BIood urea nitrogen: 12 mg/dL
Creatinine: 1.0 mg/dL
GIucose: 85 mg/dL


Which of the following is most likely present in this patient?
/ A. Normal acid-base status
/ B. Metabolic acidosis
/ C. Metabolic alkalosis
/ D. Respiratory acidosis
/ E. Respiratory alkalosis



Explanation - Q: 2.1 Close

The correct answer is B. This patient's bicarbonate is lower than the normal
range of 22-28 mEq/L. This means that there is less bicarbonate in his blood
than usual. If his breathing is normal, then his CO
2
level should be about
normal. Bicarbonate is produced when CO
2
binds to an H
2
O molecule, which
then splits into a H
+
ion and a HCO
3
- (bicarbonate) ion. This reaction is
governed by an equilibrium constant proportional to the concentrations of the
products multiplied together over the concentrations of the reactants
multiplied together. By convention, the "water" concentration is ignored
because it is "constant." So, in order for the equilibrium constant to not
change value, the H
+
concentration must increase if the bicarbonate
decreases and the CO
2
remains the same. This means that an acidosis is
produced. In this case, it is a metabolic acidosis, since there is no indication
of respiratory disease (eliminating choices D and E) in this patient. Neither
a normal acid-base status (choice A) nor a metabolic alkalosis (choice C)
would have a low bicarbonate.


This patient's anion gap is which of the following?

/ A. 3.4 mEq/L
/ B. 8.4 mEq/L
/ C. 13.4 mEq/L
/ D. 21.4 mEq/L
/ E. 42.4 mEq/L



Explanation - Q: 2.2 Close

The correct answer is C. The concept of the anion gap is based on the idea
that all the charges in a solution must add to zero, since the solution is
electrically neutral. In practice, people just take the two principal cations, Na+
and K+, and the two principal anions, Cl
-
and HCO
3
-
, and subtract the sum of
the anions from the sum of the cations. When the correct formula is used, the
anion gap is ([Na
+
]+[K
+
])-([Cl
-
]+[HCO
3
-
]) = 143 + 2.4 - (17 + 115) = 13.4
mEq/L, which is within the normal range of 10-20 mEq/L when this formula is
used for the calculation. (Some authors "cheat" even further and ignore the
K
+
as well, which works on the same principal, but produces a slightly
different reference range of values.) The anion gap is due to the excess
amount of unmeasured anions in the serum, when compared to the
unmeasured cations.


Which of the following is the most likely cause of this patient's flank pain?
/ A. Acute renal failure
/ B. Acute tubular necrosis
/ C. Benign prostatic hypertrophy
/ D. Multiple myeloma
/ E. Urolithiasis



Explanation - Q: 2.3 Close

The correct answer is E. Urolithiasis will often present with flank pain
radiating to the groin. The urine can grossly appear normal, or there can be
gross blood. Invariably, urinalysis should show hematuria, unless the stone is
completely obstructing the affected ureter.
Acute renal failure (choice A) has many different etiologies, including
hypovolemia, sepsis, drugs, and acute glomerulonephritis. In acute renal
failure, the BUN and Cr are elevated, usually in a ratio of 20:1 or higher.
Acute tubular necrosis (choice B) can present with hematuria, but there
should also be muddy brown granular casts. In addition, the patient shows
no signs of dehydration or infection, which can cause acute tubular necrosis.
Benign prostatic hypertrophy (choice C), if symptomatic, should present with
obstructive symptoms such as a poor urinary stream or nocturia. It does not
present with flank pain.
Multiple myeloma (choice D) is a malignant tumor of plasma cells. Patients
often present with fatigue, weakness, and bone pain. It is associated with
type 2 RTA, but not type 1.


Given this patient's symptoms, acid base status, chloride status, and anion gap,
which of the following is the most likely cause of this patient's abnormal
laboratory values?

/ A. Diabetic ketoacidosis
/ B. Hyperosmolar non-ketotic acidosis
/ C. Type 1 renal tubular acidosis (RTA)
/ D. Type 2 renal tubular acidosis (RTA)
/ E. Type 4 renal tubular acidosis (RTA)



Explanation - Q: 2.4 Close

The correct answer is C. Type 1 RTA is a defect in hydrogen ion secretion
in the distal tubule. All patients with RTA, regardless of type, have
hyperchloremia and a normal anion gap metabolic acidosis. Patients with
type 1 RTA are also hypokalemic, and have a tendency to develop kidney
stones.
Diabetic ketoacidosis (choice A) and hyperosmolar non-ketotic acidosis
(choice B) produce an elevated anion gap metabolic acidosis. They occur in
both type 1 and type 2 diabetics, where hyperglycemia results in an osmotic
diuresis, dehydration, and electrolyte abnormalities.
Type 2 RTA (choice D) is due to a defect in bicarbonate resorption in the
proximal tubule. The bicarbonate wasting leads to a normal anion gap
acidosis. Type 2 RTA can be inherited or acquired. Acquired causes include
multiple myeloma, heavy metal poisoning, and drugs. There is not an
increased incidence of kidney stones.
Type 4 RTA (choice E) is due to a defect in ammonium excretion in the
distal tubule. It results in hyperkalemia and is often caused by
hypoaldosteronism due to low renin. It is associated with diabetic
nephropathy.




Vignette 3 of 4

A 59-year-old man presents to his physician with complaints of fatigue and mild
weakness for the past several months. He has had a few episodes of nausea
and vomiting. He also states that he has been feeling more irritable and
forgetting things easily. He has no significant past medical history and review of
systems is noncontributory. He is not taking any medications. He is married, with
two children. Physical examination is unremarkable.
Laboratory studies show:
Sodium 125 mEq/L
Potassium 4 mEq /L
Chloride 101 mEq /L
Bicarbonate 23 mEq /L
BUN 10 mg/L
Creatinine 0.3 mg/L
GIucose 98 mg/dL
Calcium 10.5 mg/dL
Magnesium 2.4 mg/dL
Phosphorus 2.6 mg/dL

Which of the following tests would be the most appropriate next step in the
workup of this patient's hyponatremia?
/ A. Fractional excretion of sodium (FENa)
/ B. Serum anion gap
/ C. Serum osmolality
/ D. Urine osmolality
/ E. Urine sodium excretion


Explanation - Q: 3.1 Close

The correct answer is C. Serum osmolality is the first step in the workup of
hyponatremia. It can be used to determine whether the hyponatremia is
secondary to pseudohyponatremia (from hyperlipidemia/multiple myeloma as
the proteins/lipids displace Na
+
) or hypertonic solutions (such as
mannitol/hyperglycemia which cause osmotic shift of water out of cells and
dilute plasma Na
+
).
FENa (choice A) is a useful tool in determining if renal insufficiency is from
acute glomerulonephritis/prerenal azotemia or other causes. It is not used to
determine the cause of hyponatremia.
Anion gap (choice B) is used to determine which type of metabolic acidosis
is present.
Similarly, urine osmolality (choice D) can be used to differentiate between
SIADH and polydipsic causes of hyponatremia.
Urine sodium excretion (choice E) is used to determine whether the kidney
is responsible for the hyponatremia; it is used in conjunction with volume
status assessment to diagnosis the cause of hyponatremia.


Further laboratory studies show:
Serum osmoIality: 250 mOsm/L
Urine osmoIality: 600 mOsm/L
Urine sodium: 80 meq/L
Thyroid-stimuIating hormone (TSH) 3.5 U/mL

Which of the following is the most likely cause of this patient's hyponatremia?
/ A. Chlorpropamide
/ B. Hypothyroidism
/ C. Meningitis
/ D. Neoplasm
/ E. OId age


Explanation - Q: 3.2 Close

The correct answer is D. Given the patient's age, a neoplasm is the most
likely cause of hyponatremia, secondary to the syndrome of inappropriate
antidiuretic hormone secretion (SIADH). Neoplasms that can cause this
disorder include multiple myeloma, and cancers of the lung, pancreas,
duodenum, and thymus. Neoplastic cells can synthesize, store, and release
ADH, which is indistinguishable from that made in the posterior pituitary.
SIADH is associated with a low osmolality (<280 mosmol/kg H2O),
euvolemia (no peripheral edema), and a high urine osmolality (> 300
mOsm/L).
Chlorpropamide (choice A) can also cause SIADH, but it is an old
antiglycemic agent for diabetes, which is no longer in use.
Hypothyroidism (choice B) can cause an SIADH presentation and can be
ruled out with a normal TSH level. There is no clinical evidence for
hypothyroidism in this man.
Any intracranial process, including meningitis (choice C), could result in
SIADH; this patient does not seem to have any signs suggestive of any CNS
infectious process.
Although old age (choice E) can result in resetting of the osmoreceptor (that
controls ADH secretion) threshold; this should be a diagnosis of exclusion,
after all other causes of hyponatremia or SIADH have been ruled out.


What additional workup should this patient receive?

/ A. Chest x-ray film and whole body CT scan
/ B. HIV test
/ C. Radioiodine uptake (RAIU)
/ D. Screening flexible sigmoidoscopy
/ E. Serum CA 19-9 Ievels



Explanation - Q: 3.3 Close

The correct answer is A.. Any older patient with SIADH needs to have a
complete workup for neoplastic disease, especially small cell cancer of the
lung. SIADH occurs in 60% of such cases. Even if the initial workup is
negative, a patient needs to undergo total body scans every 1-2 years,
because a malignancy often eventually manifests itself.
An HIV test (choice B) would be useful if the patient had any symptoms
suggestive of HIV/AIDS, since SIADH can occur with Pneumocystis
pneumonia, but he did not show any signs of this disease. Also, the patient is
59-years-old and married, so HIV would be less likely.
RAIU (choice C) is useful for differentiating between the different causes of
hyperthyroidism.
Screening flexible sigmoidoscopy (choice D) is an important screening tool
for colon cancer for patients over the age of 50, but colon cancer is not often
associated with paraneoplastic syndromes (i.e., SIADH).
Serum CA 19-9 (choice E) can be elevated in pancreatic cancer; however,
increased values are also found in acute and chronic pancreatitis and
cholangitis. A CT/MRI would be the best diagnostic tool if the patient had
symptoms suggestive of pancreatic cancer.


Which of the following would help differentiate between this patient's disease and
psychogenic polydipsia?

/ A. BIood urea nitrogen (BUN) 16 mg/dL
/ B. Serum osmolality 265 mOsm/L
/ C. Serum potassium 4 meq/L
/ D. Serum uric acid 7 mg/dL
/ E. Urine osmolality 500 mOsm/L

Explanation - Q: 3.4 Close

The correct answer is E. SIADH is characterized by a high urine osmolality,
whereas the hallmark of psychogenic polydipsia is a low urine osmolality
because the patient is drinking too much water and diluting extracellular
sodium levels. The level of ADH is often suppressed. In addition, SIADH is
associated with low BUN (<10 mg/dL) (choice A) and low uric acid (<4
mg/dL) (choice D).
Psychogenic polydipsia is associated with low serum osmolality (choice B)
also, but potassium levels (choice C) would be elevated.



Which of the following would be the most appropriate therapy for this patient?

/ A. Demeclocycline
/ B. Diuretics
/ C. FIuid restriction
/ D. IV infusion of hypertonic saline
/ E. IV infusion of normal saline


Explanation - Q: 3.5 Close

The correct answer is C. Fluid restriction to 800-1000 cc daily is the
appropriate treatment until the sodium rises to > 135 meq/L. Then, the
underlying cause for the SIADH must be investigated and treated.
Demeclocycline (choice A) is used to treat chronic SIADH by decreasing
kidney sensitivity to ADH.
Diuretics (choice B) are only used in hypotonic hypervolemic states such as
cirrhosis, to alleviate the edema symptoms.
Hypertonic saline solutions (choice D) are used very sparingly in patients
with profound hyponatremia with mental status changes. If the hyponatremia
is corrected too rapidly, the patient could suffer central pontine myelinolysis.
In reality, hyponatremic patients are typically gently corrected with normal
saline infusions.
Patients with hyponatremia and hypovolemia should be given normal saline
infusions (choice E), which would correct both of the disorders.


If the patient had presented with peripheral swelling, which of the following would
be the most likely cause of his hyponatremia?

/ A. Adrenal insufficiency
/ B. Gastrointestinal loss
/ C. Hepatic cirrhosis
/ D. Hyperlipidemia
/ E. Syndrome of inappropriate ADH secretion (SIADH)


Explanation - Q: 3.6 Close

The correct answer is C. Patients with hepatic cirrhosis have hypotonic
hypervolemia; other causes include renal failure and cardiac failure.
The presence of peripheral edema differentiates this patient from others with
hypotonic hypovolemia [such as adrenal insufficiency (choice A) and
gastrointestinal loss (choice B)] as well as hypotonic euvolemia (such as
SIADH ; choice E).
Hyperlipidemia (choice D) can cause hyponatremia, but the serum would be
but isotonic and the patient euvolemic.








Vignette 4 of 4

A 38-year-old man complains of chest pains upon exertion. He has a family
history of heart problems. He is 6'1" and 200 Ib and his blood pressure is 145/95
mm Hg, pulse is 80/min, and respirations are 16/min. Physical examination
reveals diffuse thickening of his Achilles tendons, bilaterally. The man admits to
recent pain in both Achilles tendons.
His laboratory values are:
LDL: 300 mg/dL (optimaI, < 100 mg/dL)
HDL: 65 mg/dL (optimaI, > 60 mg/dL)
Triglycerides: 140 mg/dL
Familial hypercholesterolemia is suspected.




Which of the following pedigrees would be most consistent with this diagnosis?

/ A. A
/ B. B
/ C. C
/ D. D
/ E. E



Explanation - Q: 4.1 Close

The correct answer is C. Familial hypercholesterolemia is transmitted as an
autosomal dominant disorder. It is one of the most common genetic
disorders. About 1 in 500 persons is a heterozygote. Pedigree C shows an
autosomal dominant inheritance pattern with complete penetrance. In this
pattern, affected parents can have affected children of either sex. Unaffected
individuals have only unaffected progeny. A child born to an affected parent
has a 50% probability of inheriting the trait. Other common examples of
autosomal dominants include Marfan syndrome, Huntington disease and
osteogenesis imperfecta.
Pedigree A shows an X-linked recessive inheritance pattern. The hallmark of
this inheritance is an abundance of affected males and an absence of
affected females. Males are hemizygous for the X chromosome, so the
phenotype is expressed with only one dose of the gene. Females have two
copies of the X chromosome, so they will be phenotypically normal although
they may carry the deleterious allele. Since a male inherits his X
chromosome from his mother, if he is affected, she carries the trait. X-linked
recessive traits worth remembering include color blindness, hemophilia,
glucose-6-phosphate dehydrogenase deficiency, Lesch-Nyhan syndrome
and Duchenne muscular dystrophy.
Pedigree B could show an autosomal dominant or an X-linked dominant
inheritance pattern. It is impossible to determine without a larger sample. In
the X-linked dominant inheritance pattern, if the female parent is affected,
both sexes of children can be affected. If the male parent is affected, all of
his female children will be affected with the disease because he always gives
them his X chromosome. Since he only gives a Y chromosome to his sons,
none of them will be affected. Only a few disorders are classified as X-linked
dominants. These include X-linked hypophosphatemic rickets and ornithine
transcarbamylase deficiency.
Pedigree D shows an autosomal recessive inheritance pattern. In the
autosomal recessive pattern, unaffected parents who carry the trait can have
affected children of either sex. Common autosomal recessive traits include
cystic fibrosis, phenylketonuria, the hemoglobinopathies and the
thalassemias.
Pedigree E illustrates the mitochondrial inheritance pattern. Genetic
information is transmitted through genes contained both in nuclear
chromosomes and in mitochondrial chromosomes. However, since
mitochondria are always maternally inherited, a pedigree analysis that shows
any male transmission of a disorder is not compatible with a mitochondrial
mutation.


Which of the following is the most likely mechanism for the increased cholesterol
in this patient?

/ A. Abnormal apo-E isoform
/ B. Complete absence of LDL receptors
/ C. Half the LDL receptors are dysfunctional
/ D. Lipoprotein lipase deficiency
/ E. Pancreatic lipase deficiency



Explanation - Q: 4.2 Close

The correct answer is C. Familial hypercholesterolemia is caused by
mutations that either prevent the normal synthesis of the LDL receptor or
render the receptor dysfunctional. In the homozygous form of the disease (1
in 1 million), both alleles are mutated, and there are virtually no functional
LDL receptors (choice B). These patients have severely elevated LDL levels
(> 600 mg/dL) and can have cardiac problems in the first year or two of life.
Most affected individuals have the heterozygous form of the disease. In this
case, only half the receptors are absent or dysfunctional. A mutation that
prevents the normal synthesis of the LDL receptor is classified as Class 1.
Mutations that render the LDL receptor dysfunctional are classed as Class 2
to Class 5, depending on the defect. The heterozygous patient will have
blood LDL levels in range of 200-400 mg/dL. The patient described above
probably has the heterozygous disease given his blood LDL concentration.
The LDL receptor binds both apolipoprotein B-100 (on LDL) and apo E (on
VLDL and chylomicrons). Apo E has two normal isoforms, the E-3 and E-4. A
defect that results in abnormal isoforms of Apo E (choice A) results in poor
uptake of VLDL and chylomicrons, which accumulate in the blood. LDL is
usually low because the normal transformation of VLDL to LDL is diminished.
The disorder produced by this defect is called familial
dysbetalipoproteinemia.
Lipoprotein lipase deficiency (choice D) is responsible for primary
hypertriglyceridemia. It is transmitted as an autosomal recessive trait. Blood
levels of triglycerides can be as high as 25,000 mg/dL. The liver and spleen
are usually enlarged, and foam cells can be found in the liver, spleen, and
bone marrow. LDL and HDL levels in the blood are usually low. Treatment
involves reduction of fat to 10% or less of ingested calories.
Pancreatic lipase deficiency (choice E) results in poor digestion and
absorption of dietary triglycerides. It would not produce the condition of high
LDL present in the patient.


The patient is instructed to eat a diet that limits saturated fats, trans fats, and
cholesteroI. He is also instructed to lose weight and maintain a regular exercise
program. After 2 months, his LDL Ievel has only decreased to 260 mg/dL.
Atorvastatin (Lipitor) is prescribed. Three months later his LDL Ievel is 180
mg/dL. Which of the following steps in the synthesis of cholesterol does
atorvastatin inhibit?



Explanation - Q: 4.3 Close

The correct answer is C. The first two steps in the biosynthesis of
cholesterol are shared by the pathway that produces ketone bodies. Two
molecules of acetyl CoA from -oxidation of fatty acids (or from ketogenic
amino acids) combine to form acetoacetyl CoA (choice B). This reaction is
catalyzed by acetyl CoA acetyltransferase. In the next step, a third acetyl
CoA molecule is added to produce HMG CoA (choice A). This is catalyzed
by HMG CoA synthase. In the next step, HMG CoA is reduced in the
presence of NADPH to form mevalonic acid. This step, which is the rate-
limiting step, is catalyzed by HMG CoA reductase. Statins act by inhibiting
the action of this enzyme, and therefore decrease cholesterol synthesis, and
can reduce blood cholesterol levels by as much as 50%. Mevalonic acid is
then converted to farnesyl pyrophosphate by a series of reactions that utilize
ATP and several different enzymes. Two molecules of farnesyl
pyrophosphate combine and are reduced to form the 30-carbon compound
squalene. Squalene is hydroxylated and cyclized to form lanosterol (choice
E). This step is catalyzed by squalene monoxygenase. Finally, in a series of
reactions (that may include as many as 19 steps associated with the
endoplasmic reticulum), lanosterol is converted to cholesterol (choice D).


To further decrease cholesterol levels, a bile acid sequestrant is added to the
statin regimen. One month later, blood LDL
Ievels are 140 mg/dL. Which of the following was most likely prescribed?

/ A. Cholestyramine
/ B. CIofibrate
/ C. Ezetimbe
/ D. Niacin
/ E. Probucol


Explanation - Q: 4.4 Close

The correct answer is A. Cholestyramine is an anion-exchange resin. When
taken by mouth, this compound binds bile acids and prevents their
absorption in the ileum as part of the enterohepatic circulation. Instead, the
bile acids are eliminated in the stool. When the concentration of bile acids
returning to the liver in the hepatic portal vein is low, synthesis of new
primary bile acids from cholesterol is stimulated. Hepatocytes increase
expression of LDL receptors and increase uptake of cholesterol from the
blood. When used with a statin, cholestyramine's LDL lowering effect is
additive.
Clofibrate (choice B) causes a reduction in circulating VLDL and
triglycerides, with a modest reduction in LDL and increase in HDL. Its
mechanism of action is not completely understood, but probably acts by
stimulating lipoprotein lipase. The increased hydrolysis of triglyceride in
chylomicrons and VLDL explains the reduction in plasma triglyceride and
VLDL. It also may increase hepatic LDL uptake.
Ezetimbe (choice C) is a new compound that lowers LDL by inhibiting the
absorption of cholesterol. It also reduces circulating triglycerides and raises
HDL.
Niacin (choice D) lowers lipids when used in gram quantities. It works by
inhibiting hepatic triglyceride production and VLDL secretion. It produces a
marked decrease in circulating triglycerides with a more modest decrease in
LDL and increase in HDL. It has unwanted side effects such as flushing,
palpitations, and gastrointestinal disturbances.
Probucol (choice E) lowers both circulating LDL and HDL. Its mechanism of
action is not known. It remains in body fat for several months after treatment
is discontinued. Its peak effect on plasma cholesterol takes one to three
months of treatment.


Vignette 1 of 5


A fire in a local factory brings patients to several hospitals with injuries. A 25-
year-old woman is found by firefighters in a smoke-filled enclosed office. She has
no apparent burns, but is complaining of shortness of breath and looks quite
distressed. She is intubated in the emergency department for hypoxemia and
admitted to the intensive care unit, where bronchoscopy reveals carbonaceous
material in her large airways.


Where the bronchoscope camera exits the endotracheal tube, the resident, who
has never performed a bronchoscopy before, notes that the rings of the airway
are incomplete: the airway has "c-shaped" rings, and the remainder of the
airway's circumference appears flat, with no clearly defined cartilaginous rings.
Distal to the camera is a bifurcation. What is the significance of these findings?
/ A. The camera is in the trachea
/ B. The camera is in the trachea, but the rings are abnormal
/ C. The camera is in the left mainstem bronchus
/ D. The camera is in the right mainstem bronchus
/ E. There is not sufficient information to determine the location of the camera



Explanation - Q: 1.1 Close

The correct answer is A. The only complete cartilaginous ring in the
trachea is the cricoid cartilage of the larynx. The remainder of the tracheal
cartilages are incomplete rings; they are nearly circumferential and maintain
the patency of the trachea. The posterior wall of the trachea, directly anterior
to the esophagus, is noncartilaginous and appears relatively flat. At the
carina, the mainstem bronchi divide: the left mainstem bronchus exits at an
angle to divide into left upper and lower lobes, and the right mainstem
bronchus continues at a less acute angle off the trachea, where it quickly
gives off a branch at an acute angle to the right upper lobe, then continues
on to divide into branches to the right middle and lower lobes.
Choice B is incorrect because the tracheal rings are normally incomplete at
their posterior aspect.
Choices C and D are incorrect because the cartilaginous rings of the
mainstem bronchi are normally complete, thereby excluding the possibility
that the camera is in a mainstem bronchus. The posterior membrane also
localizes the camera in the trachea.
Choice E is incorrect because the posterior membrane of the trachea is
unique to the trachea.


18 hours after ICU admission, a chest x-ray shows bilateral diffuse airspace
disease. Despite ventilation with 80% oxygen, her oxygen saturation is 90% and
her arterial blood gas reveals a PO2 of 60, a PCO2 of 45, and a pH of 7.36. High
airway pressures are required to generate small tidal volumes. What is the
pathophysiology underlying this patient's hypoxic respiratory failure?
/ A. Abnormal alveolar capillary permeability
/ B. Acute bronchospasm
/ C. EIevated left atrial pressure
/ D. Rapidly progressive pulmonary fibrosis
/ E. Surfactant deficiency



Explanation - Q: 1.2 Close

The correct answer is A. This patient has the adult respiratory distress
syndrome (ARDS), caused principally by alterations in capillary permeability.
The normal blood-gas interface permits the transudation of water into the
alveolus when the difference in hydrostatic pressure between the airspace
and the capillary exceeds the plasma oncotic pressure. This interface,
however, is not normally permeable to protein. With injury, whether direct
toxic injury (as in this case) or due to inflammatory mediators and oxidative
injury (in the case of sepsis), plasma proteins leak into the interstitial space
and the airspace. This leaves the hydrostatic gradient unopposed and
pulmonary edema ensues, despite normal hydrostatic pressures (i.e., in the
absence of elevation of left atrial pressure).
Acute bronchospasm (choice B) can result from smoke inhalation. However,
on chest x-ray films, this would appear as hyperinflated, abnormally
radiolucent lungs. In addition, bronchospasm alone seldom results in this
degree of hypoxemia.
Elevated left atrial pressure (choice C) causes cardiogenic pulmonary
edema as the result of left heart failure, which is unlikely in this otherwise
healthy 25-year-old who has sustained airspace injury as the result of
inhalation of a toxic gas.
Pulmonary fibrosis (choice D) is a chronic disorder that can produce bilateral
abnormalities on chest x-ray films, hypoxemia, and decreased pulmonary
compliance. However, fibrosis as a pathologic process does not develop
over hours and is not compatible with such an acute onset.
Surfactant deficiency (choice E) causes the neonatal respiratory distress
syndrome, which is characterized by a similar clinical picture of decreased
compliance, hypoxemia, and bilateral diffuse airspace disease. Surfactant
has never been shown to be deficient in or beneficial in the treatment of
ARDS.


Because of her poor pulmonary compliance, the decision is made to ventilate her
using small tidal volumes and a high respiratory rate in order to maintain
ventilation while avoiding the trauma potentially caused by very high airway
pressures.
If her total minute ventilation remains constant, but her tidal volume decreases
and respiratory rate increases, which of the following will occur?
/ A. Dead space increases
/ B. Dead space remains constant, but alveolar ventilation decreases
/ C. Dead space ventilation decreases and alveolar ventilation increases
/ D. Shunt decreases
/ E. Shunt remains the same, but alveolar ventilation improves


Explanation - Q: 1.3 Close

The correct answer is B. Dead space (volume that is ventilated, but does
not participate in gas exchange) remains constant, but with an increase in
respiratory rate and a decrease in tidal volume, alveolar ventilation declines.
That is, the amount of the minute ventilation that goes to the dead space
increases:
Alveolar ventilation/min = (Tidal volume - Dead space volume) x Respiratory
rate
As an example: a patient is breathing a tidal volume of 600 cc, 150 cc of
which is dead space, at 12 x/minute. This translates into a total minute
ventilation of 7.2 liters/min: alveolar ventilation/min is (600 cc-150 cc) x 12, or
5.4 liters/min. If her tidal volume decreases to 400 cc per breath, and she
breathes at 18 breaths/min, minute ventilation is still 7.2 liters/min, but
alveolar ventilation/min = (400 cc-150 cc) x 18 = 4.5 liters/min.
Choice A is incorrect because the dead space is not significantly altered by
a drop in tidal volume.
Choice C is incorrect because dead space ventilation would increase.
Before her ventilator was adjusted, dead space ventilation would be 1.8
L/min (150 cc x 12/min) and after the adjustment, it would be 2.7 L/min (150
cc x 18/min). And as shown above, alveolar ventilation would decrease.
Choice D is incorrect because with a drop in tidal volume, the shunt can
remain the same or can actually increase, if the lower tidal volume results in
closure of previously ventilated alveoli. (A shunt can be thought of as the
opposite of dead space in the lung: an area that is perfused, but not
ventilated.)
Choice E is incorrect because alveolar ventilation decreases (see above).



A change is made in her ventilator settings. The next arterial blood gas is: P02
60, PCO2 52, pH 7.30. Hemoglobin concentration is constant, as is the patient's
temperature. What has happened to the total oxygen content of the blood and
the oxygen saturation?
/ A. Arterial blood oxygen content and oxygen saturation has stayed the same
/ B. Arterial blood oxygen content has increased, but oxygen saturation has
decreased
/ C. Arterial blood oxygen content has increased, and oxygen saturation has
increased
/ D. Arterial blood oxygen content has decreased, and oxygen saturation has
decreased
/ E. Arterial blood oxygen content has decreased, but oxygen saturation has
increased



Explanation - Q: 1.4 Close

The correct answer is D. Arterial blood oxygen content has declined
because of the rightward shift of the oxyhemoglobin dissociation curve.
Arterial blood oxygen content is a function of hemoglobin concentration, of
partial pressure of oxygen in the blood, and of the affinity of hemoglobin for
oxygen. In this example, partial pressure of oxygen and hemoglobin
concentrations are constant. Because of the lower pH and the higher CO
2
,
however, the oxyhemoglobin dissociation curve has shifted to the right. That
is, for a given partial pressure of oxygen, the hemoglobin saturation is lower.
At higher concentrations of H
+
ions, PCO
2
, temperatures, and concentrations
of 2,3-DPG, hemoglobin has less affinity for oxygen. "A simple way to
remember these shifts is that exercising muscle is acid, hypercarbic, and hot,
and it benefits from increased unloading of O
2
from its capillaries." (West,
Respiratory Physiology, Chapter 6.)
Choices B and E are incorrect because oxygen saturation is the principal
determinant of oxygen content. That is, the majority of the oxygen in the
blood is carried bound to hemoglobin. As saturation declines, content
declines.




One week after admission, the patient develops fever to 39.4 C (103 F). A new
dense infiltrate is seen in her right upper Iobe, and purulent secretions are
suctioned from her endotracheal tube. Which of the following organisms is most
likely causing a new ventilator-associated pneumonia?


Explanation - Q: 1.5 Close

The correct answer is E. Staphylococcus aureus is a gram-positive coccus
that is a common cause of ventilator-associated pneumonia. It is a colonist of
the skin and the nasopharynx, and is a common cause, not only of
pneumonia (particularly ventilator-associated and post-influenza), but of
endocarditis, superficial skin infections, surgical wound infections, and
bacteremia (especially associated with indwelling catheters).
Borrelia burgdorferi(choice A) is a tick-transmitted spirochete that is
responsible for Lyme disease. This disorder is characterized by erythema
migrans in its early stage, followed by a variable presentation that can
include malaise and fatigue accompanied by arthralgias, carditis, migratory
musculoskeletal pain, meningitis, neuritis, chronic oligoarticular arthritis, and
skin and neurological abnormalities.
Candida albicans(choice B) is a common yeast, which occasionally
colonizes the upper airways of debilitated patients, but rarely is a cause of
pneumonia. It is a common cause of vaginitis and can cause thrush,
stomatitis, and esophagitis in immunosuppressed patients, and can cause
bloodstream infections in patients with indwelling vascular appliances.
Chlamydia pneumoniae(choice C) is a common cause of community-
acquired pneumonia as well as upper respiratory tract infection. Like all
Chlamydia, it is an obligate intracellular parasitic bacterium.
Pneumocystis carinii(choice D) is an opportunistic pathogen (closely related
to fungi and to protozoa), which causes pneumonia in hosts with
compromised cellular immunity due to AIDS, corticosteroids, cancer
chemotherapeutic agents, or primary defects of cellular immunity.




Vignette 2 of 5

A 7-year-old child is taken to the emergency department because he is feeling
short of breath. The episode began about an hour previously while the child was
playing sports, when he abruptly developed paroxysms of wheezing and
coughing.
When the physician enters the room, he notes that the child is sitting leaning
forward and is using his accessory respiratory muscles. Physical examination
demonstrates tachypnea and tachycardia. On auscultation, a prolonged
expiratory phase with relatively high-pitched wheezes through much of the
respiratory cycle are heard. No fine crackles are heard


Which of the following is the most likely diagnosis?
/ A. Asthma
/ B. Bronchiectasis
/ C. Pneumonia
/ D. Pulmonary edema
/ E. Pulmonary embolus



Explanation - Q: 2.1 Close

The correct answer is A. This patient most likely has asthma based on the
initial presentation. Asthma is characterized by reversible airway obstruction,
airway inflammation, and bronchospasm of the airways in response to a
variety of stimuli. These stimuli may include exposure to known allergens,
viral infections, exercise, cold air, crying, screaming, and hard laughing. It is
conventional to subdivide asthma into extrinsic (allergy-related) and intrinsic
(not related to allergy) subtypes, although this subclassification has been
challenged because many patients have overlapping features. The physical
features illustrated in this case are typical of a severe asthma attack.
Bronchiectasis (choice B) would not produce episodes of reversible airway
obstruction, and would present with fever, cough, and moist crackles.
Pneumonia (choice C) and pulmonary edema (choice D) would develop
more slowly, and would be likely to show fine crackles on auscultation.
Pulmonary embolus (choice E) would be very unusual in a young child.


If pulmonary studies were performed, which of the following would be most likely
to be markedly decreased?

/ A. Forced expiratory volume in the first second
/ B. Functional residual capacity
/ C. Pulmonary blood flow
/ D. Residual volume
/ E. Total lung capacity



Explanation - Q: 2.2 Close

The correct answer is A. The forced expiratory volume in the first second
(FEV1) is a commonly used pulmonary function test that evaluates the
degree of obstruction present that limits expiration. The degree to which it is
reduced is a measure of the severity of the asthmatic attack. In this case, in
which the child is in obvious severe respiratory distress, the FEV1 should be
markedly decreased.
In many asthmatic patients, functional residual capacity (choice B), residual
volume (choice D), and total lung capacity (choice E) are increased.
Pulmonary blood volume (choice C) is not usually measured in asthma, but
would be expected to be normal or increased (if PO
2
drops).


The diagram below shows spirographic tracings of forced expirations from a
healthy child (trace X) and from the 7-year-old patient (trace Z).



Which of the following is the FEV1/FVC ratio of the normal child and the patient?

Normal Patient
/ A. 1.0 0.2
/ B. 0.8 0.5
/ C. 0.7 0.3
/ D. 0.5 0.8
/ E. 0.2 1.0



Explanation - Q: 2.3 Close

The correct answer is B. A forced expiration is the simplest test of lung
function. The individual breathes in as much air as the lungs can hold and
then expels the air as rapidly and as far as possible. The forced vital capacity
(FVC) is the vital capacity measured with a forced expiration (FVC = 3 L for
patient Z). The forced expiratory volume in one second (FEV
1
) is the amount
of air that can be expelled from the lungs during the first second of a forced
expiration (FVC @ 1.5 L for patient Z). FEV
1
/FVC therefore is a function of
airway resistance. Airway resistance is often increased during an asthma
attack, which causes FEV
1
/FVC to decrease. FEV
1
/FVC is 0.5 in patient Z
(1.5/3.0) and 0.8 in the healthy child represented by trace X (4/5).



Which of the following medications will have the fastest onset if the 7-year-old
child has an acute attack of his condition?
/ A. AIbuterol inhalation
/ B. Beclomethasone inhalation
/ C. Ephedrine oral
/ D. Salmeterol inhalation
/ E. Theophylline oral tablets






Explanation - Q: 2.4 Close

The correct answer is A. This question is assessing your ability to
understand the onset of action for each of the answer choices. We have
provided a summary table of the sympathomimetic bronchodilators and
selected pharmacokinetic properties.

Based on the chart above you can see that albuterol is indicated for use in
the treatment of acute signs and symptoms of asthma since its onset is in
less than 5 minutes.
Beclomethasone (choice B) is a glucocorticoid agent, and not a
sympathomimetic. It would not have the rapid onset of action required in this
case.
Although dosage forms are generally not found on USMLE Step 1, you
should immediately recognize that any medication administered orally must
be absorbed from the intestinal tract and then reach the site of action.
Therefore, you should be able to conclude that oral ephedrine (choice C)
and oral theophylline tablets (choice E) will not begin to work for at least 1/2
hour after administration.
Salmeterol inhalation (choice D) has an onset of action of around 20
minutes and has a duration of action of approximately 12 hours. Salmeterol
is indicated for the "chronic" treatment/prevention of asthma signs and
symptoms.






Approximately an hour after the acute management began, the child began to
bring up tenacious, rubbery, white sputum.
The sputum is examined in the laboratory and Charcot-Leyden crystals are
found. These are composed of which of the following?

/ A. Bilirubin
/ B. Calcium phosphate
/ C. Cystine
/ D. Protein
/ E. Uric acid



Explanation - Q: 2.5 Close

The correct answer is D. Charcot-Leyden crystals are distinctive hexagonal
bipyramidal crystals composed of a protein (thought to function as a
lysophospholipase) produced by eosinophils. They can be seen in settings in
which tissue eosinophil counts are very high, including allergic conditions,
asthma, and parasitic diseases. The sputum in asthma patients also typically
shows large number of eosinophils, and may contain Curschmann spirals
(mucus casts of smaller airways).
Bilirubin (choice A) can be found in gallstones.
Calcium phosphate (choice B), cystine (choice C), and uric acid (choice E)
can be found in urinary tract stones; uric acid can also be found in crystals in
tissues in patients with gout.


When the child is discharged, he is placed on both albuterol and flunisolide
inhalation preparations. The physician should instruct the child to use flunisolide
and albuterol in which of the following ways?

/ A. These agents should not be used together
/ B. Use albuterol several minutes before the flunisolide
/ C. Use albuterol several minutes after the flunisolide
/ D. Use albuterol several hours after the flunisolide
/ E. Wash out his mouth before each use



Explanation - Q: 2.6 Close

The correct answer is B. This item deals with the basic science principles
underlying a common instruction given by physicians and pharmacists.
Albuterol is a sympathomimetic bronchodilator with a rapid onset of action.
Therefore, this medication is generally used several minutes before
corticosteroids because it dilates the bronchioles and permits the passage of
the corticosteroid deep into the lungs (choices A, C, and D are incorrect
statements).
Flunisolide is an inhaled corticosteroid indicated for treatment of bronchial
asthma when asthma is not controlled with bronchodilators and other non-
steroidal medications. Oral fungal infections have occurred with continued
use; therefore, patients should rinse out their mouth after each use (choice
E is an incorrect statement). These agents are not to be used in treatment of
acute asthma as single agents; children may experience HPA axis
suppression with prolonged usage. Systemic effects include Cushing's
syndrome, hyperglycemia, and glycosuria, and these agents may also cause
burning, erythema, and oral dryness.



Which of the following drugs is a leukotriene modifier indicated for the
prophylaxis and treatment of asthma in this child?

/ A. Bitolterol
/ B. Cromolyn sodium
/ C. Ipratropium
/ D. Montelukast
/ E. Theophylline



Explanation - Q: 2.7 Close

The correct answer is D. Montelukast is a selective and competitive
leukotriene receptor antagonist that inhibits the cysteinyl leukotriene
(CYSLT1) receptor. The CYSLT leukotrienes (LTC4, LTD4, and LTE4) are
arachidonic acid derivatives that are released from a variety of cells,
including mast cells and eosinophils. These leukotrienes bind to the CYSLT
receptors in the airways. When these receptors are activated there is a
strong correlation with the development of the signs and symptoms of
asthma, including airway edema, smooth muscle contraction, and airway
inflammation. Blockade of airway CYSLT receptors prevents this. It is
indicated for the prophylactic and chronic treatment of asthma in adults and
children > 6 years of age.
Bitolterol (choice A) is a beta2 agonist, and is able to relieve reversible
bronchospasm by relaxing the smooth muscles of the bronchioles. It is
indicated for the treatment for an acute asthma attack.
Cromolyn sodium (choice B) acts as an antiasthmatic and an antiallergic
mast cell stabilizer. By inhibiting the degranulation of mast cells, this agent
prevents the release of histamine and SRS-A (composed of leukotrienes).
Asthma induced by inhalation of antigens can be inhibited by varying
degrees with cromolyn pretreatments. This agent has no bronchodilator,
antihistaminic, anticholinergic, or anti-inflammatory activity. It is indicated for
prophylactic management of severe bronchial asthma, prevention of exercise
induced bronchospasm, and prevention of allergic rhinitis.
Ipratropium (choice C) is an antimuscarinic agent that is structurally related
to atropine. This agent is a quaternary amine (therefore, positively charged)
and there is little systemic absorption. It is indicated for bronchospasm
associated with COPD and rhinorrhea.
Theophylline (choice E) is a xanthine derivative medication that relaxes
smooth muscle. The mechanism of action may be related to its ability to
block adenosine receptors or to inhibit phosphodiesterase. Theophylline is
indicated for the symptomatic relief/prevention of bronchial asthma (acute,
childhood, nocturnal) as well as reversible bronchospasm associated with
chronic bronchitis or emphysema.



Later in life, the patient develops hypertension. Assuming that his respiratory
condition is still present, which of the following agents would be the most
appropriate pharmacotherapy?

/ A. Atenolol
/ B. Nadolol
/ C. Propranolol
/ D. Sotalol
/ E. Timolol



Explanation - Q: 2.8 Close

The correct answer is A. Patients with disease of the small airways are
generally not prescribed beta-receptor blocking agents since these agents
can block the bronchodilation produced by endogenous and exogenous
catecholamine stimulation of the beta2 receptors. However, relatively low
doses of selective beta1 receptor blocking agents, such as atenolol and
metoprolol, are relatively well tolerated.
All of the other answer choices are non-selective beta-receptor blocking
agents and should not be used in this patient since they are likely to
exacerbate the patient's condition.











Vignette 3 of 5


A 3-year-old boy is brought to the emergency department with shortness of
breath. His parents report that he has had several episodes in which he breathes
heavily and turns blue. During these episodes, he is often found squatting, which
appears to relieve his symptoms. His parents brought him in today, because the
boy lost consciousness. On examination, he is a poorly developed, thin boy who
is in acute distress. His skin appears blue, and he has labored breathing with
chest retractions A systolic ejection murmur is auscultated at the left third
intercostal space. A chest x-ray film shows a smalI, "boot shaped" cardiac
silhouette. The boy is admitted to the hospitaI.


Which of the following is the most likely diagnosis?

/ A. Coarctation of the aorta
/ B. Ebstein's anomaly
/ C. Patent ductus arteriosus
/ D. Tetralogy of Fallot
/ E. Transposition of the great vessels



Explanation - Q: 3.1 Close

The correct answer is D. Tetralogy of Fallot is a congenital heart anomaly
that presents as respiratory distress, cyanosis, clubbing, syncope and
sudden death. Historical findings often include reports of "Tet spells," in
which the patient is found squatting to relieve respiratory symptoms.
Examination findings include a systolic ejection murmur, clubbing, and
cyanosis. A characteristic small "boot-shaped" cardiac silhouette is seen on
chest x-ray. The anatomic defects of this tetralogy are pulmonary stenosis,
overriding aorta, right ventricular hypertrophy, and ventricular septal defect.
This anatomic tetralogy results in a right-to-left shunt, and thus a marked
decrease in pulmonary blood flow. The severity of symptoms depends on the
degree of decrease in pulmonary blood flow. The squatting increases
systemic vascular resistance, and decreases the right-left shunt.
Coarctation of the aorta (choice A) is a narrowing of the thoracic aorta.
Patients may have headache, epistaxis, and lower extremity claudication. On
examination, patients have diminished lower extremity pulses, and a systolic
or continuous murmur. This defect may be associated with CHF, aortic
dissection, intracranial aneurysmal rupture, and bacterial endocarditis.
Ebstein's anomaly (choice B) is an abnormal tricuspid valve placement that
creates an abnormally large right atrium and small right ventricle. The
disorder may present with cyanosis, but the clinical presentation may vary.
You should suspect Ebstein's anomaly when there is a history of maternal
lithium ingestion.
Patient ductus arteriosus (choice C) involves a right to left shunt that may
present as acyanotic respiratory distress. It will have a continuous
"machinery" murmur.
Transposition of the great vessels (choice E) typically presents as cyanosis
and CHF in the neonatal period. An "egg shaped" cardiac contour is seen on
chest x-ray films.



A complete blood count is performed, and the hemoglobin is markedly increased.
Which of the following substances triggers this erythrocytosis?

/ A. AIdosterone
/ B. Angiotensin ll
/ C. Erythropoietin
/ D. Interleukin 1
/ E. Renin



Explanation - Q: 3.2 Close

The correct answer is C. Erythropoietin is released from the kidney in
response to renal hypoxia. This circulates to the red marrow and stimulates
erythropoiesis, a process by which erythropoietic stem cells differentiate into
red blood cells.
Aldosterone (choice A) is a hormone released from the adrenal cortex in
response to angiotensin II. It triggers sodium retention in the renal collecting
ducts.
Angiotensin II (choice B) is a hormone created by a series of enzymatic
steps. These steps are carried out when the kidney releases renin in
response to decreased flow in the area of the renal tubule known as the
macula densa.
Interleukin 1 (choice D) is an inflammatory mediator. It has no effect on
erythropoiesis.
Renin (choice E) is an enzyme released by the kidney in response to
decreased flow in the renal tubule. It initiates a cascade of enzymatic steps
to create angiotensin II, which has several vascular, renal, and endocrine
effects.



While this patient is in the hospital he begins to have fever and headache. On
examination, he has numbness of the right side of his face, but no nuchal rigidity.
A head CT shows a focal lesion with a hypodense center surrounded by a ring of
enhancement. Which of the following is the most likely diagnosis?

/ A. Brain abscess
/ B. Guillain-Barr syndrome
/ C. Meningitis
/ D. Stroke
/ E. Tuberculosis



Explanation - Q: 3.3 Close

The correct answer is A. Patients with cyanotic heart disease (most
commonly tetralogy of Fallot ) may develop a brain abscess. In chronically
hypoxic patients, polycythemia with increased blood viscosity leads to poor
cerebral capillary flow and reduced tissue oxygenation. This poorly perfused
segment acts as a nidus for infection, and as a result, patients with tetralogy
of Fallot suffer from brain abscesses. This is a typical presentation for a brain
abscess, which may include fever, headache, seizures, nuchal rigidity,
papilledema, and focal neurologic defects. The CT findings are also classic
for brain abscess.
Guillain-Barre syndrome (choice B) is an acute inflammatory demyelinating
polyneuropathy. It typically presents as ascending weakness usually
beginning in the legs. It does not present as focal CNS findings.
Meningitis (choice C) would present as headache and fever, but focal
findings on neurologic examination and on CT would be unusual. Nuchal
rigidity is often present.
Stroke (choice D) would be unusual in this age group. In addition, the focal
lesion seen on CT does not describe the findings of stroke. CT findings for
stroke vary with the etiology of the stroke, but are typically less well
circumscribed and do not display ring enhancement.
Tuberculosis (choice E) could produce a meningitis, which afflicts the very
old and very young. It has a long, protracted course as symptoms of fever,
headache and nuchal rigidity tend to develop over a two-week course. A
focal lesion would not be seen on CT scan.



A brain biopsy from this patient grows gram-positive cocci in chains. Which of the
following is the most appropriate treatment?


/ A. Cefazolin
/ B. Ceftriaxone
/ C. FIuconazole
/ D. Metronidazole
/ E. Penicillin



Explanation - Q: 3.4 Close

The correct answer is B. The finding of gram-positive cocci in chains
indicates the abscess contains streptococci. An antibiotic must not only be
effective against these organisms, it must also penetrate the blood-brain
barrier. Ceftriaxone is a third generation cephalosporin that penetrates the
BBB. While the effectiveness against gram-positive organisms decreases as
one moves from first to second to third generation cephalosporins, unless the
medication arrives at the proper site, it cannot be effective.
Cefazolin (choice A) is a first generation cephalosporin. It would kill
streptococci, but it would not access the CNS.
Fluconazole (choice C) is an antifungal agent. The culture indicates
streptococci. Fungal brain abscess would be unusual in an
immunocompetent patient.
Metronidazole (choice D) is an antiparasitic that also has activity against
gram-negative organisms. It would be appropriate to use in conjunction with
ceftriaxone in an empiric setting. Once the definitive cause is known,
discontinuing extraneous antibiotics is appropriate.
Penicillin (choice E) would kill streptococci, but it could not access the CNS
in the presence of an intact blood-brain barrier.



Which of the following sets of changes depict the oxygen partial pressures in the
aorta, Ieft ventricle, right atrium, and vena cava of this patient during resting
conditions, compared to a healthy individuaI?




Explanation - Q: 3.5 Close

The correct answer is C. In Tetralogy of Fallot, the aorta originates from the
right ventricle or overrides the septum, and therefore receives blood from
both ventricles. This decreases the oxygen tension of aortic blood. Blood
flowing through the lungs is still oxygenated normally, causing the oxygen
tension of blood in the left ventricle to be normal. Because the oxygen
content of the arterial blood is lower than normal, the oxygen tension of the
venous blood (and therefore blood in the right atrium) is lower than normal.




Vignette 4 of 5

A 55-year-old man presents to the emergency department complaining of the
acute onset of severe shortness of breath.
His only associated symptom is a dry cough productive of scant frothy sputum.
He has a 70 pack-year smoking history and has had two similar episodes in the
past two years. On examination, he is afebrile, tachypneic, and distressed, but
without cyanosis. He is thin, and his accessory muscles contract with each
breath. He exhales through pursed lips. His chest examination reveals
diminished breath sounds with hyperresonance to percussion.


Which of the following is the most likely diagnosis?

/ A. Chronic bronchitis
/ B. Cystic fibrosis
/ C. Emphysema
/ D. Myocardial infarction
/ E. Pneumonia



Explanation - Q: 4.1 Close

The correct answer is C. Patients with COPD may be clinically classified as
"pink puffers" or "blue bloaters" based on several characteristics. This patient
is exhibiting the classic presentation for an emphysematous "pink puffer"; the
patient has dyspnea but is not cyanotic. "Pink puffers" maintain their
oxygenation until they decompensate precipitously. In contrast, the "blue
bloater" counterparts with chronic bronchitis (choice A) have a chronic
productive cough with cyanosis.
Cystic fibrosis (choice B) is an inherited disease involving a defective
chloride channel, and tends to affect patients at a younger age. The
defective chloride channel leads to viscous mucus that is difficult to clear. As
a result, patients develop chronic lung infections and fibrotic lung disease.
Myocardial infarction (choice D) classically presents as crushing substernal
chest pain, shortness of breath, and diaphoresis. Pain may radiate to the jaw
and be accompanied by nausea. If congestive heart failure is present,
patients may have dry cough, scant sputum, and rales on exam.
Pulmonary infections may precipitate COPD exacerbations, but this patient's
presentation is not consistent with pneumonia (choice E) . Pneumonia
presents as dyspnea, fever, and productive cough.


In this disorder, the mechanism for decreased FEV1 (forced expiratory volume in
1 second) is which of the following?

/ A. Airway collapse due to loss of elastic recoil
/ B. Airway constriction due to bronchospasm
/ C. BIockage of airways by increased mucus production
/ D. Decreased lung compliance due to pulmonary fibrosis
/ E. Lung collapse due to air in the pleural space



Explanation - Q: 4.2 Close

The correct answer is A. Patients with emphysema, a form of chronic
obstructive lung disease, suffer from a loss of elastic recoil in the lung. As a
patient exhales, the airways collapse, trapping air distal to the bronchiole. As
a result of airway collapse, patients with emphysema have increased
resistance to outflow, and a decreased FEV1.
Asthma, another form of COPD, results from the obstruction of small airways
due to bronchoconstriction (choice B).
Blockage of airways due to mucus production (choice C) characterizes
chronic bronchitis. While this obstruction produces COPD, it is not consistent
with emphysema.
Pulmonary fibrosis (choice D) would increase bronchiolar elasticity and
therefore would increase FEV1.
Lung collapse due to air in the pleural space (choice E) describes
pneumothorax. This does not explain the decrease in FEV1 in emphysema.





Which of the following drugs might prove helpful in treating this patient?

/ A. Acetazolamide
/ B. AIbuterol
/ C. Metoprolol
/ D. Metronidazole
/ E. Propranolol


Explanation - Q: 4.3 Close

The correct answer is B. Albuterol is a beta-adrenergic agonist and, as
such, promotes bronchial smooth muscle relaxation, and thus
bronchodilation. Bronchodilation may relieve symptoms in COPD patients
with acute exacerbation. Methylxanthines and/or anti-cholinergics may
provide an adjunct to beta agonists.
Acetazolamide (choice A) is a carbonic anhydrase inhibitor used as a
diuretic. There is nothing in this patient to suggest that he is volume
overloaded, and diuretics would thus be inappropriate.
Propanolol and metoprolol (choices C and E) are beta blockers and may
actually exacerbate this patient's symptoms through bronchoconstriction.
Metronidazole (choice D) is an antimicrobial agent. While antibiotics are
often used to treat infections that may exacerbate COPD, metronidazole is
not an appropriate agent to treat the pathogens that typically infect people
with COPD: Haemophilus influenzae, Strep. pneumoniae, and Moraxella
catarrhalis.



Which of the following sets of changes depict the mean arterial pressure (MAP),
Ieft ventricular peak systolic pressure (LVPSP), pulmonary wedge pressure
(PWP), and left atrial pressure (LAP) in this patient, compared to a healthy
individuaI?





Explanation - Q: 4.4 Close

The correct answer is A. The elastic recoil of the lungs is decreased in
emphysema. This causes the chest wall to expand sufficiently to create a
new balance between the elastic recoil of the lungs and chest wall, which
increases the functional residual capacity. Because the lungs are expanded
to higher than normal levels, both the total lung capacity and residual volume
are chronically increased. The vital capacity of the lungs (which is the
difference in volume between the total lung capacity and residual volume) is
decreased in emphysema because the patient cannot expel normal amounts
of air from the lungs.



An inherited form of this disease can result from which of the following?
/ A. A deficiency of alveolar dipalmitoyl phosphatidylcholine
/ B. A deficiency of serum alpha-1-antitrypsin
/ C. A deficiency of serum angiotensin converting enzyme.
/ D. An elevated alveolar dipalmitoyl phosphatidylcholine
/ E. An elevated serum alpha-1-antitrypsin
/ F. An elevated serum angiotensin converting enzyme



Explanation - Q: 4.5 Close

The correct answer is B. Alpha 1-antitrypsin deficiency is associated with
familial emphysema. Patients may have decreased (heterozygote) or absent
(homozygote) amounts of the protease inhibitor alpha 1-antitrypsin. The
precise mechanism in producing emphysema is unclear, but patients with
this deficiency typically have severe disease, often with an early age of
onset. Choice E therefore cannot be correct.
Dipalmitoyl phosphatidylcholine (choices A and D) is a component of
alveolar surfactant, which reduces alveolar surface tension. Not only does
this surfactant reduce the surface tension, it changes surface tension with
changing diameter. This prevents atelectasis by allowing interconnected
alveoli with different diameters to remain open at the same alveolar
pressures. (Without surfactant, the Law of LaPlace dictates that it takes
greater alveolar pressures to open a smaller alveolus.)
Elevated or decreased angiotensin converting enzyme (ACE; choices C and
F) would not produce emphysema. ACE converts angiotensin I to
angiotensin II, which has many effects on vascular and renal physiology, but
is not associated with emphysema.



Which of the following nerves provides innervation to the diaphragm?

/ A. Accessory nerve
/ B. Intercostal nerve
/ C. Phrenic nerve
/ D. Splanchnic nerve
/ E. Vagus nerve



Explanation - Q: 4.6 Close

The correct answer is C. The phrenic nerve originates from cervical roots 3,
4, and 5 to provide innervation to the diaphragm. This is significant because
spinal cord injury above this level renders the patient unable to breathe.
The accessory nerve (choice A), provides motor innervation to the trapezius
and the sternocleidomastoid muscles.
The intercostal nerves (choice B) innervate the intercostal muscles
responsible for chest wall expansion and retraction.
The splanchnic nerve (choice D) provides motor and autonomic fibers to the
gut.
The vagus nerve (choice E) provides motor innervation to the vocal cords,
heart, bronchus and GI tract. In addition it provides sensory innervation to
the bronchus, heart, GI tract and larynx.





Vignette 5 of 5

A 20-year-old woman is brought to the emergency department after a severe
traffic collision. Initial assessment reveals a tachypneic, tachycardic, hypotensive
woman in acute distress. On examination, she has multiple contusions on the left
Iateral chest wall and jugular venous distention (JVD). Her chest is
hyperresonant to percussion and she has diminished breath sounds on the left.
Her trachea deviates to the right. A chest x-ray film reveals diminished vascular
markings on the Ieft. An ECG shows sinus tachycardia.


Which of the following is the most likely diagnosis?

/ A. Cardiac contusion
/ B. Hypovolemic shock
/ C. Pericardial tamponade
/ D. Pulmonary contusion
/ E. Tension pneumothorax



Explanation - Q: 5.1 Close

The correct answer is E. This patient has the classic findings for tension
pneumothorax. She has a history of severe trauma, and the findings of
dyspnea, tachypnea, tachycardia, tracheal deviation, unilaterally decreased
breath sounds, and hyperresonance to percussion. This is a life-threatening
condition that must be remedied immediately.
Cardiac contusion (choice A) would not present with these findings. Patients
will be hemodynamically unstable, often showing various arrhythmias on
ECG. They will not have tracheal deviation, JVD, or hyperresonance to
percussion.
Hypovolemic shock (choice B) could not account for the patient's jugular
venous distention, hyperresonance to percussion, diminished breath sounds,
tracheal, or chest x-ray findings.
Pericardial tamponade (choice C) is a life-threatening condition often seen in
the trauma setting. Beck's triad characterizes pericardial tamponade:
decreased heart sounds, jugular venous distention, and hypotension.
Patients may also be tachycardic with pulsus paradoxus.
Pulmonary contusion (choice D) may produce respiratory distress
immediately after trauma occurs but usually complicates the chronic course.
It would not produce tracheal deviation, or hyperresonance to percussion. On
x-ray films it would appear as an ill-defined opacity.



Which of the following is the most likely mechanism of this patient's shock?

/ A. FIuid in the pericardial space prevents diastolic filling
/ B. Hypoxia from rapid loss of pulmonary capacity impedes cardiac function
/ C. Malpositioning of the great vessels has impeded venous return
/ D. Massive bleeding into the pleural space has led to hypovolemia
/ E. Myocardial dyskinesia has led to cardiac insufficiency



Explanation - Q: 5.2 Close

The correct answer is C. As discussed above, this patient is suffering from
tension pneumothorax, and the mechanism of shock in this condition is
mechanical. As air becomes trapped in the pathologic side of the chest, the
ipsilateral chest cavity expands. The enlarging cavity begins to encroach
upon the mediastinal and contralateral chest contents, and as the contents
herniate contralaterally, the great vessels become kinked and compressed.
This inhibits venous return and thus diastolic filling. This results in shock.
Immediate decompression of the chest will restore venous return.
As discussed above, this patient does not have pericardial tamponade
(choice A). In pericardial tamponade, fluid in the pericardial sac surrounding
the heart prevents diastolic filling, resulting in shock. Immediate
decompression of the pericardial space is indicated.
While hypoxia (choice B) may result from pneumothorax, this is not the
primary reason for the patient's shock.
Massive bleeding into the pleural space (hemothorax; choice D) may cause
shock in a fashion similar to pneumothorax. It is unlikely that this patient has
a hemothorax, however, as evidenced by the hyperresonance to percussion.
Choice E is incorrect. Severely contused myocardium would likely show
ECG abnormalities. A normal ECG makes this diagnosis less likely, and thus
not a likely reason for this patient's shock.



Which of the following is the most appropriate immediate management?

/ A. FIuid bolus
/ B. Needle thoracostomy
/ C. Open thoracotomy
/ D. Pacemaker placement
/ E. Pericardiocentesis




Explanation - Q: 5.3 Close

The correct answer is B. Since the tension within the chest cavity creates
the danger, relieving it is the mainstay of treatment. Continued tension will
continue to push the chest contents to the side contralateral to injury. This
will "kink" the venous return to the heart leading to diminished end-diastolic
volume with imminent cardiac collapse. In addition, the uninjured lung will
have limited inspiratory capacity. Thus to accomplish immediate
decompression, a needle is inserted in the second intercostal space at the
mid-clavicular line.
This is followed by placement of a chest tube to manage the pneumothorax
more chronically. This patient may have hypovolemia, as bleeding may be
present, but her hemodynamic instability may be entirely due to the tension
pneumothorax. Thus, fluid (choice A) may be given later, but treating the
pneumothorax is a higher priority.
Open thoracotomy (choice C) would treat the pneumothorax, but would take
longer than needle decompression, and may be more than is needed.
This case has given no justification for pacing the heart (choice D).
Pericardiocentesis (choice E) is used to treat pericardial tamponade, not
pneumothorax.



When instrumenting the chest, instruments are inserted into the chest along the
superior surface of the rib to avoid which of the following structures?

/ A. External oblique muscle
/ B. Intercostal artery
/ C. Parietal pleura
/ D. Phrenic nerve
/ E. Visceral pleura



Explanation - Q: 5.4 Close

The correct answer is B. The intercostal vein, artery, and nerve traverse the
inferior surface of the ribs in the intercostal groove. When placing
instruments into the chest, these structures must be avoided. Damaging
these structures can result in bleeding into the chest and may cause
significant hemothorax. Thus, instruments are inserted along the superior
surface of the rib.
The external oblique muscle (choice A) runs between the ribs from superior
to inferior and laterally to medially. (Like the fingers of a hand placed in pants
pockets.) This structure in invariably pierced when instrumenting the chest.
The parietal pleura (choice C) is the layer of pleura that adheres to the chest
wall. It is also pierced when instrumenting the chest.
The phrenic nerve (choice D) innervates the diaphragm from spinal
segments C3-5. It travels in the mediastinum and would not be at risk here.
The visceral pleura (choice E) is the thin layer of pleura that envelops the
lung. It would be at risk for damage if the instruments were inserted into the
chest too far. Placing the instrument in the proper orientation to the rib is of
no consequence


During a routine check up of a two-year-old boy, the patient's mother mentions
she's noticed intermittent and recurrent rashes on her son in the
past year. The frequency of the rash increases with cold, dry weather. The rash
does not appear to be tender, but is associated with moderate-
to-severe pruritus. The patient has no significant birth history or past medical
history. Physical examination reveals a healthy, welI-nourished
two-year-old boy, in no acute distress. Cutaneous examination shows lichenified,
dried plaques with scaling covering the plaque surface,
Iocated on the antecubital and popliteal fossae, eyelids, face and the neck. There
are multiple excoriated papules scattered within the plaques.
Question 1 of 5
Which of the following history is important in ascertaining the correct diagnosis?
/ A. Family cat with fungal infection
/ B. Family history of asthma, hay fever, or eczema
/ C. Family history of psoriasis
/ D. Perinatal history of cradle cap
/ E. Recent travel history

Explanation - Q: 1.1 Close

The correct answer is B. This patient has the hallmark presentation of
childhood atopic dermatitis, also loosely termed eczema. These lesions
typically have a lichenified, papular, slightly scaly appearance. The classic
locations are the antecubital and popliteal fossa, flexor wrist, eyelids, face,
and around the folds of the neck. Often there are intermingled excoriated
papules, suggesting the pruritic nature of these lesions. Patients with atopic
dermatitis often also have asthma or hay fever, and a family history of
asthma, hay fever, and eczema.
A family cat with a fungal infection (choice A) would suggest a diagnosis of
tinea, which usually presents on both cheeks of children after rubbing their
face against pets.
Psoriasis (choice C), though it tends to run in families, does not result in an
atopic tendency leading to dry, sensitive skin.
A perinatal history of cradle cap (choice D) is referring to seborrheic
dermatitis (dandruff) of the scalp. Again, this has not been shown to be
associated with atopic dermatitis in later childhood.
A recent travel history (choice E) would be consistent with an infectious
origin. Because of the intermittent occurrence of the lesions, in addition to
lack of other systemic signs of infection (such as fever, night sweats,
malaise), a systemic infectious process is unlikely.


Question 2 of 5
After further questioning, the patient's mother reports similar lesions in the
patient's father during his childhood. Which of the following is the
most likely diagnosis?
/ A. Bacterial infection
/ B. Eczema
/ C. Fungal infection
/ D. Psoriasis
/ E. Spider bites


Explanation - Q: 1.2 Close

The correct answer is B. Patients with atopic dermatitis tend to have a
positive family history of asthma, hay fever, and eczema.
Bacterial infection (choice A), when widespread, usually is associated with
systemic symptoms as well as cutaneous findings, such as fever, increased
irritability, and malaise. Also, the lesions tend to be impetiginized (yellowish
exudate and crust overlying an erythematous base).
Fungal infection (choice C) usually appears on faces and scalps of children,
especially in daycare settings. The morphology of tinea tend to be annular,
erythematous plaques with central clearing and a rim of scale on the
peripheral border, thus also known as "ring worm."
Psoriasis (choice D) tends to occur over extensor surfaces of the
extremities, scalp, and intergluteal fold. In children, lesions can also be seen
in diaper areas, resulting in misdiagnosis of diaper rash. Typically, lesions
tend to be covered by thick, silvery scales.
Spider bites (choice E), as well as other insect bites, tend to occur on the
lower extremities. With widespread involvement, one should see bites in
different stages ranging from erythematous and urticarial to bulla formation
and necrotic centers


Question 3 of 5
Which of the following tests would most likely confirm the probable diagnosis?
/ A. Bacterial culture
/ B. Erythrocyte sedimentation rate (ESR)
/ C. Fungal culture
/ D. HLA-B27
/ E. Immunoglobulin E (IgE) Ievel

Explanation - Q: 1.3 Close

The correct answer is E. Patients with atopy can often have an elevated
IgE level, correlating with increased sensitivity, as well as with the pruritic
nature of the lesions. However, a normal level of IgE does not rule out
eczema, which is mostly a clinical diagnosis.
Bacterial culture (choice A) in eczematous lesions will occasionally grow out
gram-positive staphylococcal organisms, but does not confirm a diagnosis of
atopic dermatitis.
Elevated erythrocyte sedimentation rate (choice B) is commonly seen in
inflammatory and infectious processes.
Fungal culture (choice C) would be the appropriate test to confirm tinea
lesions.
Human leukocyte antigen (HLA) B27 (choice D) is seen in different
syndromes and diseases including psoriatic arthritis and Reiter syndrome.


Question 4 of 5
Which of the following is the most appropriate first-Iine therapy for this patient?
/ A. Oral antibiotics
/ B. Oral antifungals
/ C. Topical antifungals
/ D. Topical steroids
/ E. UItraviolet light

Explanation - Q: 1.4 Close

The correct answer is D. Topical steroids are effective in the treatment of
atopic dermatitis.
Oral antibiotics (choice A) can occasionally be effective in atopic dermatitis
patients, especially those superinfected with gram-positive organisms, but
are not considered first-line therapy.
Oral antifungals (choice B) are used in kids with tinea infection, particularly
tinea capitis (fungal infection of the scalp).
Topical antifungals (choice C) are used in Candida and other fungal
infections.
Ultraviolet light (choice E) treatment has been found to be effective in the
treatment of atopic dermatitis by modulating the immune response in the
skin. However, the risk of future skin cancer secondary to ultraviolet
exposure makes this therapy undesirable in children.
However, the risk of future skin cancer secondary to ultraviolet exposure
makes this therapy undesirable in children.


Question 5 of 5
On his one month follow up visit, the patient's mother complains that despite
frequent bathing and prescribed therapy, her son is not improved.
What other factors should be addressed prior to changing therapy?
/ A. Add prophylactic oral antibiotic therapy
/ B. Avoid excessive external irritation
/ C. Check bacterial culture for sensitivity and specificity
/ D. Refer patient for arthritis workup
/ E. Take the family cat to the veterinarian for proper treatment

Explanation - Q: 1.5 Close

The correct answer is B. Any external irritation may precipitate an attack of
eczema. These include excessive bathing, vigorous rubbing, or chafing.
Local infection, irritating secretions, medicated baby oils, and soap and water
may aggravate the disease.
Prophylactic oral antibiotic therapy (choice A) is not necessary unless the
patient has had repeated episodes of superinfection in the past.
Bacterial culture for sensitivity and specificity (choice B) is applicable if
patient has impetiginized lesions that appear to be infected and has failed a
course of oral antibiotics.
Referral for arthritis workup (choice D) should be considered in patients with
severe psoriasis and joint complaints.
Taking the family cat to the veterinarian (choice E) is important in treating
young children with tinea (fungal infection).


A 35-year-old woman consults a dermatologist because of a persistent facial
rash. Physical examination demonstrates an erythematous rash,
without blistering or ulceration, involving both cheeks and the nose. The
nasolabial folds are relatively spared. The dermatologist also notes
scattered erythematous, firm, maculopapular lesions elsewhere on the woman's
face and on exposed areas of the neck, upper chest, and
elbows. Careful examination of the scalp demonstrates a few areas of focal
alopecia. On questioning, the woman reports that she has had the
rash more or less continuously for about six months, and that it gets much worse
when she is exposed to sunlight.
Question 1 of 5
Which of the following autoimmune diseases would most likely produce this
patient's skin problems?
/ A. Dermatomyositis
/ B. Progressive systemic sclerosis
/ C. Rheumatoid arthritis
/ D. Sjgren syndrome
/ E. Systemic lupus erythematosus


Explanation - Q: 2.1 Close

The correct answer is E. Systemic lupus erythematosus (SLE) is a systemic
autoimmune disorder in which tissues and cells are damaged by pathogenic
autoantibodies (especially antinuclear antibodies) and immune complexes.
Approximately 90% of systemic lupus erythematosus occurs in women. SLE
is associated with a wide variety of manifestations, including characteristic
skin rashes, arthritis, anemia, seizures, psychiatric illness, and may affect the
kidneys, lungs, and heart. The rash involving the nose and cheeks is the
characteristic malar "butterfly" rash of SLE; significant features include the
rash distribution, fixed erythema, sparing of the nasolabial folds, and
exacerbation with sunlight exposure. The rash is not the only cutaneous
manifestation of lupus; alopecia and the maculopapular rash described in the
case may also be seen.
The rash of dermatomyositis (choice A) is characteristically described as a
lilac or heliotrope discoloration of the upper eyelids accompanied by
periorbital edema.
Progressive systemic sclerosis (choice B), or scleroderma, produces thick,
dense skin secondary to dermal fibrosis.
Rheumatoid arthritis (choice C) characteristically produces subcutaneous
rheumatoid nodules.
Sjgren syndrome (choice D) does not have a characteristic skin
manifestation.


Question 2 of 5
The dermatologist biopsies the rash and sends the sample for pathologic
examination. The biopsy shows liquefactive degeneration of the
basal layer of the epidermis accompanied by edema at the dermal junction.
Immunofluorescence microscopy would most likely demonstrate
which of the following?
/ A. Granular deposition of immunoglobulin and complement along the
dermoepidermal junction
/ B. Granular deposits of lgA selectively localized to the tips of dermal papillae
/ C. Lacy network of lgG deposits in the intercellular spaces lining the
keratinocytes
/ D. Linear basement membrane depositions of immunoglobulin and
complement
/ E. No deposits of immunoglobulin or complement


Explanation - Q: 2.2 Close

The correct answer is A. While the etiology of systemic lupus
erythematosus is still unproven, it is suspected to involve alterations in
immunologic control that predispose for the formation of a wide variety of
autoantibodies. Both systemic lupus erythematosus and discoid lupus
erythematosus characteristically show granular deposition of immunoglobulin
and complement along the dermal epidermal junction; in discoid lupus these
deposits are confined to the area of rash, while in systemic lupus they can
also be seen in clinically "normal" skin.
Choice B is seen in dermatitis herpetiformis.
Choice C is seen in pemphigus vulgaris.
Choice D is seen in bullous pemphigoid.


*** Commercial version is infinite. Order at http://www.structurise.com/kleptomania ***
Question 3 of 5
The patient is referred to an internal medicine specialist for further evaluation.
The initial immunologic work-up demonstrates a high titer
positive ANA test, which is followed by the demonstration of other more specific
antibodies. High titers of which of the following are
considered to be highly specific for this patient's disease?
/ A. Anti dsDNA
/ B. Anti Jo-1
/ C. Anti SS-A
/ D. Anti SS-B
/ E. Anti-U1RNP

Explanation - Q: 2.3 Close

The correct answer is A. A wide variety of autoantibodies can be seen in
systemic lupus erythematosus, including all of those listed. However, high
titers of anti-double-stranded DNA and anti-Sm (not listed above) are
considered to be the most specific for systemic lupus erythematosus.
Anti-Jo-1 (choice B) is associated with inflammatory myopathies.
Anti SS-A (choice C) and anti-SS-B (choice D) are associated with Sjgren
syndrome.
Anti-U1RNP (choice E) is associated with SLE, but is not very specific.


Question 4 of 5
Urinalysis in this patient demonstrates severe proteinuria and microscopic
hematuria. Renal biopsy shows marked glomerular changes with a
marked increase in cellularity primarily related to the proliferation of endothelial
and mesangial cells. Fibrinoid necrosis and hyaline thrombi
are seen in the glomerular capillary beds. Some of the glomeruli have epithelial
crescents that filI Bowman's space. These changes are most
consistent with which of the following?
/ A. Diffuse proliferative glomerulonephritis
/ B. Focal proliferative glomerulonephritis
/ C. Membranous glomerulonephritis
/ D. Mesangial lupus glomerulonephritis
/ E. Tubulointerstitial nephritis

Explanation - Q: 2.4 Close

The correct answer is A. Systemic lupus erythematosus can present in a
wide variety of ways, and can have an even wider variety of complications,
making diagnosis difficult. SLE affects many organs in the body, but the
kidneys are a particular target. Choices A to D above can all be seen in
different lupus patients, and sometimes in the same patient at different times
or different biopsy sites. The specific set of lesions described in the question
are those of diffuse proliferative glomerulonephritis, which is the most serious
of the renal lesions of lupus, and occurs in 35-40% of patients who are
biopsied.
Tubulointerstitial nephritis (choice E) is not usually described in lupus
patients.








Question 5 of 5

With modern therapeutic management, the 10-year survival for patients with this
woman's disease in most developed countries is now
approximately which of the following?

/ A. Less than 5%
/ B. 20%
/ C. 50%
/ D. 80%
/ E. More than 95%



Explanation - Q: 2.5 Close

The correct answer is E. Modern management of systemic lupus
erythematosus has markedly improved the prognosis of systemic lupus
erythematosus, and the key appears to be good control of the initial acute
phase. Treatment for SLE depends on which clinical problems are present
and on whether the disease is active at that time. NSAIDs are helpful for mild
inflammation; corticosteroids are the mainstay of treatment for SLE, but
should be used judiciously. If used, bone protection is important.
Hydroxychloroquine and other anti-malarials reduce SLE activity and help
with symptoms involving the joints and skin. Severe SLE requires
immunosuppressive agents such as cyclophosphamide and azathioprine.



A 55-year-old man presents to the emergency department with a 2-week history
of worsening "sores and blisters." He states that the lesions
initially started as sores in his mouth. Over the course of two weeks, he's noted
progression, with new blister formation on his face and scalp.
One day prior to presentation, he developed similar lesions on his genital area as
well as his axillae. He is concerned about a possible
correlation with his recent unprotected sexual activity with an acquaintance. On
examination, he appears welI-nourished and is afebrile.
Cutaneous examination reveals lesions in various stages of healing. Over the
intertriginous areas, there are several flaccid, easily ruptured,
bullae that appear on erythematous bases. Some of the bullae have ruptured,
forming superficial erosions. OIder lesions are covered with
crusts that have little or no tendency to heaI. Some of the erosions appear to
enlarge by coalescence. Similar lesions are found on the scalp,
face, neck, axillae, and the genital area, as well as the oral mucosa. These
lesions are moderately tender to palpation.
Question 1 of 6
Which of the following is the most likely diagnosis?
/ A. Bullous pemphigoid
/ B. Gonococcal dermatitis
/ C. Herpes simplex infection
/ D. Pemphigus vulgaris
/ E. Syphilis

Explanation - Q: 3.1 Close

The correct answer is D. Pemphigus vulgaris is an autoimmune blistering
disorder characterized by formation of thin-walled, relatively flaccid, easily
ruptured, bullae that appear on either apparently normal skin and mucous
membranes or on erythematous bases. The bullae usually rupture to form
erosions with raw surfaces that ooze and bleed easily. The denuded areas
soon become partially covered with crusts that enlarge by confluence. The
mouth lesions appear first in 60% of cases. The short-lived bullae quickly
rupture to involve most of the mucosa with painful erosions. Cutaneous
lesions are commonly seen in the groin, scalp, face, neck, and axillae.
Bullous pemphigoid (choice A) is an autoimmune disease characterized by
large, tense, subepidermal bullae with a predilection for the groin, axillae,
and flexor surfaces. Involvement of the pharynx, mucosa and eye is rare.
After the bullae rupture, large denuded areas are seen, but these do not
materially increase in size as they do in pemphigus vulgaris. Instead, the
denuded areas show a tendency to heal spontaneously.
Gonococcal dermatitis (choice B) in gonococcemia is characterized by a
hemorrhagic, vesiculopustular eruption, bouts of fever, and arthralgia of one
or several joints. The skin lesions begin as tiny erythematous macules that
evolve into vesicopustules on a deeply erythematous base.
Herpes simplex virus (HSV, choice C) is one of the most common infections
worldwide. HSV 1 infection is the cause of most cases of orolabial herpes
simplex. Infected patients with HSV-2 are either completely asymptomatic, or
have recurrent genital herpes that presents as painful ulcers and erosions.
Syphilis (choice E) is a sexually transmitted disease. Primary syphilis
generally presents with a nontender, clean-based ulcer on the genitals.


Question 2 of 6

During the examination, the physician notes that slight pressure on the intact
bullae causes peripheral spreading of the lesion. Which of the
following most accurately describes this phenomenon?
/ A. Cicatricial pemphigoid
/ B. Epidermolysis bullosa acquisita
/ C. Nikolsky sign
/ D. Split papule
/ E. Target sign

Explanation - Q: 3.2 Close

The correct answer is C. The Nikolsky sign is a result of lack of cohesion in
the epidermis, so the upper layers can easily be made to slip laterally by
slight pressure or rubbing.
Cicatricial pemphigoid (choice A) is actually a variant of bullous pemphigoid
resulting in scarring, primarily of the mucosa.
Epidermolysis bullosa acquisita (choice B) is another bullous disorder. It is
not hereditary, and is characterized by mechanobullous eruption with
noninflammatory scarring that occurs in the elderly population.
Split papule (choice D) is one of the cutaneous manifestations of secondary
syphilis.
Target sign (choice E) refers to the targetoid lesions seen in erythema
multiforme.



Question 3 of 6
The etiology of this disorder is best characterized by which of the following?
/ A. Autoimmune blistering
/ B. Bullous impetigo
/ C. Exfoliative toxin-induced
/ D. Mechanobullous
/ E. Superantigen phenomenon

Explanation - Q: 3.3 Close

The correct answer is A. Pemphigus vulgaris is an autoimmune blistering
disease mediated by intercellular antibodies. These are demonstrable
throughout the epidermis and the oral epithelium, and circulating intercellular
antibodies are present. Direct immunofluorescence is of great value in the
early diagnosis of pemphigus vulgaris.
Bullous impetigo (choice B) is characterized by large, fragile bullae,
suggestive of pemphigus. When these rupture, they leave circinate, weepy,
or crusted lesions. The majority of these are caused by phage 71 coagulase-
positive Staph. aureus.
Exfoliative toxin-induced (choice C) would apply to skin erythema and
desquamation caused by phage type 71.
A mechanobullous (choice D) mechanism is seen in epidermolysis bullosa
acquisita.
The superantigen phenomenon (choice E) refers to (usually) bacterial
antigens that activate all T-lymphocytes that have a T-cell receptor with a
particular V sequence. As a consequence, large numbers of T-cells are
activated and systemic reactions such as shock are induced.



Question 4 of 6
Which of the following tests would be most useful for confirmation of the likely
diagnosis?
/ A. Bacterial culture
/ B. Complete blood count
/ C. Fungal culture
/ D. Rapid plasma reagin (RPR)
/ E. Tissue biopsy with direct immunofluorescence

Explanation - Q: 3.4 Close

The correct answer is E. Direct immunofluorescence (DIF) is of great value
in the early diagnosis of pemphigus vulgaris. DIF shows intercellular IgG
throughout the epidermis or the oral epithelium. IgG is found in both involved
and clinically normal skin in nearly all patients with pemphigus. In
acantholytic areas, C3 deposition is also reliably found. DIF results remain
positive for a long time and may still be positive many years after clinical
remission.
Bacterial culture (choice A) is helpful in superinfected lesions, but does not
help in confirming a diagnosis of pemphigus.
Complete blood count (choice B) is helpful in infectious processes.
Fungal culture (choice C) is used in tinea lesions.
The rapid plasma reagin (choice D) is a serum test used in diagnosing
syphilis.





Question 5 of 6
A biopsy of affected skin would show which of the following?
/ A. Acantholysis
/ B. Balloon degeneration
/ C. Reticular changes
/ D. Spongiosis
/ E. Subcorneal pustulosis

Explanation - Q: 3.5 Close

The correct answer is A. The pathologic changes in pemphigus vulgaris are
acantholysis, clefts, and blister formation in the intraepidermal areas.
Acantholysis is the separation of keratinocytes from one another. The loss of
cohesion or contact between cells begins with the detachment of
tonofilaments from desmosomes. Evidence indicates that an IgG
autoantibody actually induces these changes.
Balloon degeneration (choice B) and reticular changes (choice C) are both
histologic markers of herpes simplex viral-induced lesions.
Spongiosis (choice D) is a general term referring to serous exudates
between cells of the epidermis, with an inflammatory infiltrate in the dermis.
This is frequently seen in dermatitic lesions.
Subcorneal pustulosis (choice E) describes accumulation of neutrophils
underneath the stratum corneum. This can be seen in a number of different
disorders, including fungal infection, staphylococcal scalded-skin syndrome,
and psoriasis.


Question 6 of 6
Which of the following would be the most appropriate pharmacotherapy for this
disorder?
/ A. Antifungal agent
/ B. Gold salts
/ C. Oral antibiotic
/ D. Penicillamine
/ E. Systemic corticosteroid

Explanation - Q: 3.6 Close

The correct answer is E. Corticosteroids are the standard therapy for
pemphigus vulgaris. Generally, prednisone is given alone, or in combination
with one or several immunosuppressants. In addition, pemphigus antibody
titers are performed every 4 weeks, watching for a fall in titer. Medication is
continued until clinical disease is suppressed and pemphigus antibody
disappears from the serum.
Antifungal therapy (choice A) has no role in the treatment of pemphigus
vulgaris. Oral antifungals are often employed in treatment of onychomycosis.
Gold therapy (choice B) has been reported to be effective in the treatment of
pemphigus, but is infrequently used. Bone marrow suppression and
nephrotoxicity may occur.
Oral antibiotics (choice C), such as tetracycline, in combination with
nicotinamide, have had some success in controlling pemphigus, but are not
considered the gold standard.
Penicillamine (choice D) is used in Wilson syndrome; it has no role in
pemphigus therapy.





A 37-year-old white man presents with a chief complaint of rash for 2-3 years. He
says the rash initially started on the intergluteal fold, but
progressed to involve areas on his extremities and trunk. He admits to
occasionaI, moderate-to-severe pruritus with the rash. On cutaneous
examination, there are large erythematous plaques with thick micaceous, silvery
scales located on extensor surface of his elbows and knees
bilaterally. Similar lesions are seen on his lower back and intergluteal fold. In
addition, there are thick, welI-defined scaly plaques on his scalp,
extending beyond the hairline.



Question 1 of 5
Which of the following is the most likely diagnosis?
/ A. Drug eruption
/ B. Eczema
/ C. Psoriasis
/ D. Reiter syndrome
/ E. Zinc deficiency

Explanation - Q: 4.1 Close

The correct answer is C. Psoriasis is a common skin disorder characterized
by round, circumscribed, erythematous, dry, scaling plaques of various sizes,
covered by grayish white, imbricated, and silvery scales. The lesions have a
predilection for the scalp, extensor surfaces of the limbs, elbows, knees, and
sacral region. The eruption is usually symmetrical and may vary from solitary
to more than 100 lesions. The eruption usually develops slowly. Subjective
symptoms, such as itching or burning, may be present and may cause
extreme discomfort.
Drug eruptions (choice A) usually present as morbilliform erythematous
lesions, initially involving the trunk and extending to the extremities. The
lesions tend to be papular, coalescing into plaques without well-defined
borders.
Eczema (choice B) is a common inflammatory skin disorder; patients often
have a strong family history for the disorder. However, eczematous lesions
are lichenified and do not have thick, micaceous scales overlying the lesions.
Also, the typical location for eczema in young adult would be flexural surface
areas, such as the popliteal and antecubital fossas.
Reiter syndrome (choice D) is a characteristic clinical triad consisting of
urethritis, conjunctivitis, and arthritis. It may also involve the skin, mucous
membranes, gastrointestinal tract, and cardiovascular system.
Zinc deficiency (choice E) presents most commonly in infants or patients on
long term total parenteral nutritional supplements. The dermatitis found in all
forms of zinc deficiency is pustular and bullous.


Question 2 of 5
A biopsy from one of the lesions would most likely show which of the following?
/ A. Acanthosis with intracorneal pustule formation
/ B. Basal cell liquefaction with band-Iike chronic inflammatory infiltrate in the
papillary dermis
/ C. Hyperkeratosis with intra-epidermal clefts containing "corps ronds"
/ D. Large intraepidermal cleft in the suprabasal region
/ E. Large subepidermal blister with sparse inflammatory infiltrate

Explanation - Q: 4.2 Close

The correct answer is A. Microscopically, psoriasis characteristically shows
epidermal hyperplasia (acanthosis), often with fusion of rete ridges. Another
distinctive feature in psoriasis is the presence of subcorneal and intracorneal
pustules (sometimes called microabscesses of Munro) containing
aggregated neutrophils.
Choice B suggests lichen planus and other lichenoid lesions.
Choice C suggests Darier's disease.
Choice D suggests pemphigus vulgaris.
Choice E suggests bullous pemphigoid.


Question 3 of 5
Which of the following is the most appropriate next step in therapy?
/ A. Antibiotic
/ B. Antifungal
/ C. Corticosteroid
/ D. Stopping all of his oral medications
/ E. Vitamin supplement

Explanation - Q: 4.3 Close

The correct answer is C. Topical application of corticosteroids in creams,
ointments, lotions, and sprays is the mainstay of therapy for psoriasis.
Antibiotics (choice A) are used in patients with eczema to augment
superinfection with gram-positive organisms.
Antifungals (choice B) are used in tinea infection or seborrheic dermatitis,
but has no role in psoriatic therapy.
Certain classes of medications can exacerbate psoriatic lesions, such as
beta-blockers and lithium, but stopping all of his medications (choice D)
would be a drastic step.
Vitamin supplementation (choice E) does not play a role in psoriasis
therapy.



Question 4 of 5
Therapy is initiated and the patient returns to the clinic 4 months later with a
complaint of nail changes. What other extracutaneous
manifestation is commonly associated with this new symptom?
/ A. Arthritis
/ B. Asthma
/ C. Diarrhea
/ D. Hepatitis
/ E. Renal failure

Explanation - Q: 4.4 Close

The correct answer is A. Psoriatic arthritis is commonly seen in patient with
cutaneous and/or nail findings. It usually presents as asymmetrical distal
interphalangeal joint involvement with nail damage.
Asthma (choice B) is often associated with eczema or atopic dermatitis and
hay fever, the triad of atopy.
Diarrhea (choice C) is associated with zinc deficiency.
Hepatitis (choice D) and renal failure (choice E) can be complications of
therapy for psoriasis




Question 5 of 5
The most likely prognosis for this patient is which of the following?
/ A. 100% of patients are cured with proper management
/ B. Once disease free for one year, the likelihood of recurrence is very low
/ C. Only 10% cure rate with available therapy
/ D. Only recurs with upper respiratory infections
/ E. Persistent and recurrent disease

Explanation - Q: 4.5 Close

The correct answer is E. The course of psoriasis is unpredictable. It usually
begins on the scalp or elbows and may remain localized in the original region
for an indefinite period, or completely disappear, recur, or spread to other
parts. Two of the chief features of psoriasis are its tendency to recur and its
persistence.
Occasionally, patients remain disease free for years only to have lesions
recur (choice B).
The mechanism of psoriatic lesions has not been clarified, but underlying
immune and HLA-linkage have been implicated. At this time, cure of the
disease has not been achieved with therapy available (choices A and C).
A variant of psoriasis, known as guttate psoriasis, can be exacerbated by
upper respiratory infections (choice D).


An anxious mother brings in her 14-month-old daughter because of a rash for the
past 3 days. She states that approximately 5 days prior to
the onset of rash, her daughter had fever, cough, sneezing, and nasal congestion
associated with malaise. At that time, she was seen by her
regular pediatrician with a diagnosis of viral upper respiratory infection. After
treatment with acetaminophen and decongestants, the fever
continued and rash appeared initially on her face. By the third day, the rash
extended down to the trunk and extremities. On examination, the
patient seems irritable. Cutaneous examination reveals erythematous,
morbilliform papules coalescing into plaques involving the face, trunk,
and extremities. 1 mm white papules on an erythematous base are seen on the
buccal mucosa. Neurologic examination is normaI.

Question 1 of 6

Which of the following is the most likely diagnosis?
/ A. Chicken pox
/ B. German measles
/ C. Measles
/ D. Molluscum contagiosum
/ E. Roseola

Explanation - Q: 5.1 Close

The correct answer is C. Measles, also known as rubeola and morbilli, is a
worldwide disease that most commonly affects children under 15 months of
age. It is spread by respiratory droplets and has an incubation period of 9-12
days. The prodrome consists of fever, malaise, conjunctivitis, and prominent
upper respiratory symptoms (nasal congestion, sneezing, coryza, and
cough). After 1-7 days, the exanthem appears, usually as macular or
maculopapular lesions on the scalp line and behind the ears. The rash
spreads quickly over the face, then by the second or third day (unlike the
more rapid spread of rubella) extends down the trunk to the extremities.
Chicken pox (choice A) also known as varicella, represents primary infection
with the varicella zoster virus. Transmission is by direct contact with the
lesions and by the respiratory route. There is an initial viremia between days
4 and 6, seeding the liver, spleen, lung, and perhaps other organs. A
secondary viremia occurs from day 11 to 20, resulting in infection of the
epidermis. Varicella is characterized by a vesicular eruption consisting of
delicate "teardrop" vesicles on an erythematous base.
German measles (choice B) is also known as rubella. It has a prodrome of
1-5 days, consisting of fever, malaise, sore throat, eye pain, headache, and
red eyes. Pain on lateral and upward eye movement is characteristic. The
exanthem also begins on the face, spreads to cover the entire body within 24
hours, and resolves by the third day.
Molluscum contagiosum (choice D) results in smooth-surfaced, firm, dome-
shaped, pearly papules, averaging 3-5 mm in diameter. A central
umbilication is characteristic. Irritated lesions may become crusted and even
pustular. Lesions tend to be on the face, trunk, and extremities.
Roseola (choice E) is a common cause of sudden, unexplained high fever in
young children between 6 and 36 months of age. Prodromal fever is usually
high. Suddenly on about the 4th day, the fever drops. Coincident with the
drop in temperature, a morbilliform erythema consisting of rose-colored
discrete macules appears on the neck, trunk, and buttocks. Complete
resolution of the eruption occurs in 1 to 2 days.



Question 2 of 6
The white spots on the buccal mucosa are also known as which of the following?
/ A. Forscheimer's sign
/ B. Hand-foot-mouth disease
/ C. Herpangina
/ D. Koplik's spots
/ E. Target sign

Explanation - Q: 5.2 Close

The correct answer is D. Koplik's spots are pathognomonic for measles and
appear during the prodrome. They initially appear on the buccal mucosa
nearest to the lower molars as 1 mm white papules on an erythematous
base. After 6 to 7 days, the exanthem clears, with simultaneous subsidence
of the fever.
Forscheimer's sign (choice A) is an enanthem of pinhead-sized red macules
or petechiae on the soft palate and uvuvla in patients with rubella.
Hand-foot-mouth disease (choice B) is most frequently caused by
coxsackieviruses. Small, rapidly ulcerating vesicles surrounded by a red
areola are found on the buccal mucosa, tongue, soft palate, and gingiva.
Herpangina (choice C) is caused by multiple types of coxsackieviruses and
echoviruses. The characteristic finding is one or more yellowish white,
slightly raised 2-mm vesicles in the throat, usually surrounded by an intense
areola. The lesions are most frequently found on the anterior faucial pillars,
tonsils, uvula, or soft palate.
The target sign (choice E) is the typical cutaneous lesion seen in erythema
multiforme. These begin as sharply marginated, erythematous macules,
which become raised, edematous papules over 24 to 48 hours. Typically, a
ring of erythema forms around the periphery, and centrally, the lesions
become flatter, more purpuric, and dusky.

Question 3 of 6
Which of the following treatments has been shown to reduce the morbidity or
mortality of hospitalized children younger than 24 months of age
with this disease?
/ A. Antibiotics
/ B. Antiviral medication
/ C. High doses of vitamin A
/ D. Immune serum globulin
/ E. Supportive care

Explanation - Q: 5.3 Close

The correct answer is C. Administration of high doses of vitamin A will
reduce the morbidity and mortality of hospitalized children with measles. Two
doses of retinyl palmitate 200,000 IU, 24 hours apart, are recommended for
all children from 6 months to 24 months of age, for immunodeficient children,
children with malnutrition or evidence of vitamin A deficiency, and in recent
immigrants from areas of high measles mortality.
Antibiotics (choice A) are generally used only if the patient is thought to
have a bacterial superinfection.
Antiviral medication (choice B) is not available at this time for measles. A
live virus vaccine is recommended at 15 months with a booster at 5 years.
When given up to 5 days after exposure, vaccination may prevent infection.
Immune serum globulin (choice D) is given to children less than one year of
age who are exposed.
Symptomatic treatment with supportive care (choice E), bed rest,
analgesics, and antipyretics is the treatment of choice in non-complicated,
non-hospitalized cases.



Question 4 of 6
Which of the following is a possible complication of this disorder?
/ A. Acute renal failure
/ B. Arthralgia and arthritis
/ C. Dysphagia
/ D. Photophobia
/ E. Pneumonia and encephalitis



Explanation - Q: 5.4 Close

The correct answer is E. Complications of measles include otitis media,
pneumonia, encephalitis, and thrombocytopenic purpura. These
complications are more common in the malnourished and in children with T
cell deficiencies.
Acute renal failure (choice A) has not been reported in children with
measles.
Arthritis and arthralgia (choice B) are common complications, especially in
adult women with rubella (German measles).
Dysphagia (choice C) is one of the presenting signs for herpangina along
with fever, headache, sore throat, and stiff neck.
Photophobia (choice D), painful urination, and extensive respiratory and
alimentary tract involvement, can be complications of Stevens-Johnson
syndrome.




Question 5 of 6
Which of the following is a rare long-term neurologic complication of this
disease?
/ A. AIzheimer's disease
/ B. Creutzfeldt-Jacob disease
/ C. Friedreich's ataxia
/ D. Pick's disease
/ E. Subacute sclerosing panencephalitis

Explanation - Q: 5.5 Close

The correct answer is E. Subacute sclerosing panencephalitis is a
fortunately rare but devastating brain disorder that is thought to represent a
persistent measles virus. It occurs months to years after the initial measles
infection and is characterized by progressive intellectual deterioration that is
accompanied by seizures, loss of vision, and motor abnormalities. Death
almost always occurs within 1 to 3 years, with aspiration pneumonia being a
common immediate cause of death. To date, only supportive care and anti-
convulsants have offered any (transient) hope. All of the other conditions
listed also cause devastating progressive neurologic deterioration, but none
of them is related to measles infection. Prions have been implicated in
Creutzfeldt-Jacob disease (choice B), while the other diseases listed are not
known to have an infectious component.



Question 6 of 6
The girl has a 3-month-old healthy sibling that lives with her. Her mother is
concerned about the contagious nature of this disease. What is the
most appropriate action at this time?
/ A. Administer immune serum globulin
/ B. Administer live viral vaccination
/ C. Admit the child to hospital for close monitoring
/ D. Reassurance; wait for development of symptoms
/ E. Tell the mother that this is not a contagious condition

Explanation - Q: 5.6 Close

The correct answer is A. Children less than one year of age who are
exposed to measles should be treated with immune serum globulin.
Administering live viral vaccination (choice B) is recommended as a
preventive measure for children older than 15 months of age.
Admitting a healthy child to the hospital (choice C) is unnecessary at this
time.
Reassurance (choice D) without preventive measures in exposed children is
not considered standard therapy.
Measles is transmitted by respiratory droplets, often resulting in urban
outbreaks, demonstrating its contagious nature (choice E).



A 37-year-old woman presents to a walk-in clinic with a chief complaint of an
"unsightly" mole on her leg that she would like to have removed.
She first noted the mole approximately one year ago while shaving. Initially, there
was no associated symptom with the mole. However, in the
past 6 months, she has noted the mole had doubled in size with changing color,
as well as pruritus and occasional tenderness. She denies any
family history of skin cancer but admits to several blistering sunburns while
growing up in Los Angeles. On examination, she has blond hair,
blue eyes, and light skin with numerous freckles in the sun-exposed areas. Skin
examination reveals an 8 x 6 mm asymmetric brown macule
with an irregular, scalloped border and variegated black, brown and red
pigmentation located on the right shin. No lymphadenopathy was
appreciated in the popliteal and inguinal region bilaterally.
Question 1 of 6
Which of the following is the most likely diagnosis?
/ A. Basal cell carcinoma
/ B. Congenital nevus
/ C. Malignant melanoma
/ D. Seborrheic keratosis
/ E. Solar keratosis

Explanation - Q: 6.1 Close

The correct answer is C. Melanomas usually have a prolonged,
noninvasive, horizontally oriented growth phase in which the lesion enlarges
asymmetrically. Superficial spreading melanoma is the commonest type. It
affects adults of all ages and has no sex predilection. The upper back of both
sexes and the shins in women are the most common sites. There is a
tendency to multicoloration, not just with different shades of tan, but also
black, red, brown, and white spectrum. The ABCD criteria have been
developed to educate the general public. The letters stand for asymmetry,
border irregularity, color variegation, and diameter larger than 6 mm.
Basal cell carcinoma (choice A) is a common skin cancer composed of one
or a few small, waxy, semitranslucent nodules forming around a central
depression that may or may not be ulcerated, crusted, and bleeding. The
edge of larger lesions has a characteristic rolled border. Telangiectasias
course through the lesion. Bleeding on slight injury is a common sign.
Congenital nevus (choice B) is nevocytic lesion found in newborns. These
are categorized into giant, medium and small, depending on the size.
Seborrheic keratoses (choice D) are multiple, oval, slightly raised, light
brown to black, sharply demarcated papules or plaques, located on the chest
and back.
Solar (actinic) keratoses (choice E) are found chiefly on the chronically sun
exposed surfaces of the face, ears, and backs of hands and forearms. They
are usually multiple, discrete, flat or elevated, verrucous or keratotic, red,
pigmented or skin colored.


***
Question 2 of 6
Epidemiologic studies have shown that which of the following is the present
lifetime risk of developing this patient's lesion in Americans?
/ A. 1 in 3
/ B. 1 in 11
/ C. 1 in 72
/ D. 1 in 250
/ E. 1 in 1200

Explanation - Q: 6.2 Close

The correct answer is C. Epidemiologic studies have shown a steady
increase in incidence of melanoma over the last several decades. This
increase has been attributed to several factors including thinning of the
ozone layer and increased public and physician awareness leading to
increased diagnosis rate. Last statistic analysis approximates one in 72
Americans will have one melanoma. The prevalence of melanoma is
increased in individuals with light complexions, light eyes, blond or red hair,
and tendency to tan poorly and sunburn easily. There is also a familial
tendency to develop of melanoma, especially in patients with multiple
congenital nevi. Sunlight also plays a role in the etiologic factors of
melanoma, though a less essential and direct one than that of nonmelanoma
skin cancer.


Question 3 of 6
What is the most appropriate next step?
/ A. Check complete blood count and liver function tests
/ B. Chest x-ray film
/ C. Reassure patient that this is a benign lesion
/ D. Referral to hematology and oncology for initiation of treatment
/ E. Tissue biopsy to confirm diagnosis

Explanation - Q: 6.3 Close

The correct answer is E. Although this patient's history, clinical examination
and location of the lesion suggest the diagnosis of malignant melanoma, it is
most appropriate to obtain a tissue biopsy to confirm the diagnosis.
Complete blood count and liver function test (choice A) as well as a chest x-
ray film (choice B) should be part of the melanoma workup after tissue
confirmation of the diagnosis.
Reassuring the patient of its benign nature (choice C) is inappropriate
management of most pigmented lesions. Melanoma may clinically mimic
other benign lesions, but because of its aggressive nature and this patient's
history of size and color changes, tissue confirmation is needed.
Referral to hematology and oncology for treatment (choice D) is often
needed in management of malignant melanoma patients, but is not the first
step in managing a new patient.


Question 4 of 6
Which of the following would a biopsy of the lesion most likely show?
/ A. Basaloid lobules extending from basal layer of the epidermis
/ B. Hyperkeratosis and keratin horns
/ C. Mitosis and atypical melanocytes throughout epidermis
/ D. Nests of nevoid cells
/ E. Squamous proliferation with atypia and invasion into the dermis


Explanation - Q: 6.4 Close

The correct answer is C. Diagnostic criteria of melanoma include atypical
melanocytes scattered singly and in irregular nests, presence of mitoses,
inflammatory reaction, and the absence of dermal stroma.
Basaloid lobules (choice A) extending from the basal layer, showing clefting
and palisading border is characteristic of basal cell carcinoma.
Hyperkeratosis and keratin horns (choice B) are characteristic of seborrheic
keratosis.
Nests of nevoid cells (choice D) are seen in normal melanocytic nevi.
Squamous proliferation with atypia and invasion into the dermis (choice E) is
typical of squamous cell carcinoma.


Question 5 of 6

Which of the following is the most common subtype of this patient's lesion?
/ A. Acral lentiginous
/ B. Amelanotic
/ C. Lentigo maligna
/ D. Nodular
/ E. Superficial spreading


Explanation - Q: 6.5 Close

The correct answer is E. Melanoma has a number of subclassifications, the
members of which vary in prognosis and clinical setting. The most common
subtype is superficial spreading melanoma, which occurs in approximately
70% of patients and is most commonly found on the trunk and legs. This type
of melanoma presents as a flat or nearly flat brown lesion with variable black,
blue, or pink discoloration and typically a size greater than 6 mm.
Acral lentiginous melanoma (choice A) is an uncommon subtype of
melanoma that is often found in dark-skinned individuals and occurs on the
palms, soles, or beneath the nail plate.
Amelanotic melanoma (choice B) is a rare, nonpigmented form of melanoma
that appears pink or flesh-colored. Amelanotic melanoma is more frequently
found in metastases than in primary lesions, and is presumed to represent a
deterioration in the cancer cells' genetic structure that leads to an impaired
ability to synthesize melanin.
Lentigo maligna melanoma (choice C) occurs in up to 15% of cases, grows
slowly over a period of years to decades, and is usually found as part of a
large (often more than 3 cm) precursor lesion on sun damaged skin of the
head, neck, and arms.
Nodular melanoma (choice D) occurs in 15-30% of patients and, like
superficial spreading melanoma, is most often seen on the legs and trunks. It
tends to grow rapidly (weeks to months) and causes a darkly colored papule
or nodule that may bleed with minor trauma.


Question 6 of 6
Which of the following is the most important determinant of prognosis?
/ A. Presence of lymphocytes
/ B. Presence of mitoses
/ C. Presence of ulceration
/ D. Tumor surface diameter
/ E. Tumor thickness


Explanation - Q: 6.6 Close

The correct answer is E. The most important determinant of prognosis in
melanoma is the tumor thickness. This can be measured as the Breslow
depth, which is measured vertically in millimeters from the top of the granular
layer (or base of superficial ulceration) to the deepest site of tumor
involvement. Thicker tumors have a greater risk of metastasis, and, since
chemotherapy is problematic in melanoma (less than 20% response rate),
increased risk of poor outcome. Thin primary tumors less than 1 mm thick
and without metastases at time of diagnosis have a 5 year survival rate of
over 90%; while melanomas more than 4 mm thick without known
metastases have a 5-year survival rate of 45-70%. If regional lymph node
metastases are present, the 5-year survival rate is in different studies has
ranged from about 15 to 70%. If distant metastases are present, the 5-year
survival rate drops to less than 20%, with a median survival of only 6-9
months. The presence of ulceration (choice C) is thought to be the next
most important risk factor. The other features noted in the choices are
usually included in pathology reports, but have lesser prognostic value.


A 54-year-old man complains of daytime fatigue and sleepiness whenever he sits
down. The man states that he gets at least 6-7 hours of
sleep every night and does not have any problems falling asleep. His wife reports
that he is a loud snorer and has on occasion, gasped for air.
He also complains of nonspecific headaches with no particular pattern, as well as
a recent weight gain of 20 Ib in the last 6 months. Past
medical history is noncontributory, and the patient denies taking any medications.
Social habits include 2 shots of whiskey prior to bedtime
each night. Physical examination reveals a height of 173 cm (68 in) and weight of
103 kg (225 Ib). His blood pressure is 148/90 mm Hg and
his pulse is 85/min. His oropharynx is within normal limits, with no deviation or
swelling of the uvula. The rest of the physical examination is
unremarkable. Laboratory studies, including T3, T4, TSH, complete blood count,
electrolytes, BUN, creatinine, and glucose are within normal
Iimits.
Question 1 of 3
Which of the following studies would most likely lead to the correct diagnosis?
/ A. Arterial blood gases
/ B. Echocardiogram
/ C. Pulmonary function tests
/ D. Radioiodine uptake (RAIU)
/ E. SIeep study

Explanation - Q: 1.1 Close

The correct answer is E. The patient is presenting with symptoms
suggestive of sleep apnea (daytime somnolence, snoring, recent weight
gain, and mild hypertension); the diagnostic test is a sleep study
(polysomnography). Obstructive sleep apnea is characterized by repetitive
cessation of airflow during sleep from the collapse of the pharyngeal airway
despite continuing ventilatory effort. The pharyngeal airway is anatomically
small secondary to obesity, skeletofacial abnormalities, tonsillar/uvular
hypertrophy, or a posteriorly positioned jaw. During waking hours, airway
patency is maintained by activating the neuromuscular reflexes with
increased activity of the pharyngeal dilator muscles. During sleep, this reflex
is diminished or lost, so muscle activity decreases and the airway collapses.
The ensuring apnea decreases PO
2
and increases PCO
2
, causing increased
ventilatory effort and arousal from sleep with resumption of breathing. The
result is repetitive cycles of apnea and hyperpnea.
Arterial blood gases (choice A) are more useful to aid the diagnosis of
pulmonary processes causing hypoxia, however they can be used to
differentiate Pickwickian syndrome from other causes of obstructive sleep
apnea.
When sleep apnea becomes severe, resulting in pulmonary hypertension or
left heart failure, an echocardiogram (choice B) can be used to assess
cardiac status; it is seldom useful in the initial assessment for sleep apnea.
Pulmonary function tests (choice C) are useful in establishing the different
causes for dyspnea; it can be used to differentiate the different types of
central sleep apnea. Patients with neuromuscular disorders associated with
central sleep apnea can have abnormal pulmonary function tests with a
diminished FEV1.
Hypothyroidism can aggravate sleep apnea; a TSH level is often used to
exclude primary hypothyroidism. An RAIU (choice D) is only useful in
patients with hyperthyroidism.


A 54-year-old man complains of daytime fatigue and sleepiness whenever he sits
down. The man states that he gets at least 6-7 hours of
sleep every night and does not have any problems falling asleep. His wife reports
that he is a loud snorer and has on occasion, gasped for air.
He also complains of nonspecific headaches with no particular pattern, as well as
a recent weight gain of 20 Ib in the last 6 months. Past
medical history is noncontributory, and the patient denies taking any medications.
Social habits include 2 shots of whiskey prior to bedtime
each night. Physical examination reveals a height of 173 cm (68 in) and weight of
103 kg (225 Ib). His blood pressure is 148/90 mm Hg and
his pulse is 85/min. His oropharynx is within normal limits, with no deviation or
swelling of the uvula. The rest of the physical examination is
unremarkable. Laboratory studies, including T3, T4, TSH, complete blood count,
electrolytes, BUN, creatinine, and glucose are within normal
Iimits.
Question 2 of 3
Which of the following best distinguishes Pickwickian syndrome from this
patient's condition?
/ A. Daytime hypersomnolence
/ B. Hypoventilation while awake
/ C. Increased PCO2 during sleep
/ D. Obesity
/ E. Presence of apneic episodes during sleep


Explanation - Q: 1.2 Close

The correct answer is B. Dr. Osler first described the Pickwickian syndrome
after a character by Charles Dickens in the Pickwick Papers. A patient with
Pickwickian syndrome is obese, with daytime sleepiness and obstructive
sleep apnea. However, the distinguishing trait is that the patient must have
awake hypoventilation (high PCO
2
); it is considered a severe form of obesity-
hypoventilation syndrome.
Daytime hypersomnolence (choice A) and increased PCO
2
during sleep
(choice C) from the apneic episodes (choice E) characterize Pickwickian
syndrome as well as other causes of obstructive sleep apnea.
Not all obstructive sleep apnea patients are obese (choice D) and not all
patients who are obese have Pickwickian symptoms.


Question 3 of 3
Which of the following is the most appropriate next step in therapy for this
patient?
/ A. Acetazolamide
/ B. AIcohol avoidance and weight loss
/ C. Nasal oxygen
/ D. Surgery and continuous positive airway pressure (CPAP)
/ E. Thyroid supplementation

Explanation - Q: 1.3 Close

The correct answer is B. For mild to moderate obstructive sleep apnea
such as this patient has, the patient can benefit from alcohol avoidance,
weight loss, and not sleeping in the supine positive. For more severe cases,
continuous positive airway pressure (CPAP) can be very useful. Patients who
develop Cheyne-Stokes respiration with sleep apnea are generally in heart
failure. There is evidence that nasal CPAP can reduce left ventricular
afterload for these patients, improve cardiac function, and regulate ventilation
during sleep for better sleep quality. Decisions to use CPAP versus
noninvasive measures is based on the severity of the patient's symptoms
and the apnea/hypopnea index.
Acetazolamide (choice A) can be used in central sleep apnea of idiopathic
origin. It causes metabolic acidosis, which stimulates a central compensatory
response to increase ventilatory efforts.
Nasal oxygen (choice C) can be used for central sleep apnea, but is not
useful in obstructive sleep apnea.
Surgery (uvulopalatopharyngoplasty; choice D) can eliminate snoring,
especially in patients with an enlarged uvula or pharyngeal tissues; it only
cures 50% of the obstructive sleep apnea. It can be used in severe sleep
apnea to decrease the amount of CPAP required.
Thyroid supplementation (choice E) can help sleep apnea if the patient is
hypothyroid, but in the absence of hypothyroidism, is not useful.


A 54-year-old man is having trouble sleeping alone after his wife dies. He
discusses his problems with his friends at a card game. One of the
players suggest that he try some of his "sleeping pills," and later brings over a
bottle of benzodiazepines that were prescribed for him. The
man begins taking the medication to help him sleep.
Question 1 of 3
The man later presents to a psychiatrist and admits to self-medicating with
benzodiazepines. Which of the following would be a likely side
effect of this medication?
/ A. Decreased REM sleep
/ B. Night terrors
/ C. Nocturnal myoclonus
/ D. SIeep walking
/ E. Vivid dreams

Explanation - Q: 2.1 Close

The correct answer is A. Stage 4 is the deepest non-REM sleep and occurs
prior to REM sleep. Benzodiazepines shorten stage 4 and suppress REM
sleep. REM sleep is also decreased by barbiturates, alcohol, phenothiazines,
and MAO inhibitors. Stage 4 sleep is decreased by imipramine and
benzodiazepines.
Night terrors (choice B) and sleep walking (choice D) are parasomnias
related to intrusion of waking behavior (speaking or walking) into non-REM
sleep, during stage 4. Benzodiazepines shorten stage 4, and are used to
treat night terrors and sleepwalking. Imipramine is used to treat enuresis,
also associated with stage 4.
Nocturnal myoclonus (choice C) is a dyssomnia characterized by brief
repetitive leg jerks that occur in regular 20 to 40 second intervals. It is
thought to be secondary to loss of inhibition of at the level of the spinal cord.
It is normal in infants. The movements lead to transient arousal from sleep
and result in sleep fragmentation, and a sense of insufficient sleep.
Benzodiazepines and L-Dopa are the mainstays of treatment.
Vivid dreams (choice E) and nightmares occur during REM sleep (unlike
sleep terrors). Nightmares are not accompanied by movement unless the
person wakens. In contrast, persons with night terrors are not awake, but are
moving. Reports of nightmares occur when REM activity is increased, such
as after discontinuation of REM-suppressing substances (barbiturates,
alcohol, phenothiazines, and MAO Inhibitors).

Question 2 of 3
The patient stops using benzodiazepines. Six months later, he reports continued
daytime somnolence, and has begun waking at 4 am with
difficulty returning to sleep. He no longer goes to card games or has any interest
in his gardening club. He reports "feeling blue" with poor
appetite, and poor energy. This disorder is associated with which of the following
sleep disturbances?
/ A. Decreased REM Iatency and reduced proportion of REM sleep
/ B. Increased sleep latency and little or no REM sleep
/ C. Rapid cycling through sleep stages and increased proportion of slow wave
sleep
/ D. Stereotyped forceful teeth grinding or clenching
/ E. Increased proportion of REM sleep and nightmares


Explanation - Q: 2.2 Close

The correct answer is A. The patient meets criteria for depression.
Depressed patients rapidly enter REM upon falling asleep, but overall, have
a decrease in the proportion of REM sleep.
Increased REM latency and very little REM sleep (choice B) is associated
with mania. Manic patients may be awake for days (greatly increased sleep
latency) and may have very little REM when they do sleep.
Rapid cycling through sleep stages and increased proportion of slow wave
sleep (choice C) is associated with alcohol use. Alcohol also decreases the
amount of REM.
Stereotyped forceful teeth grinding or clenching (choice D) (bruxism) is
associated with dementia, mental retardation, and Parkinson disease.
Increased proportion of REM sleep and nightmares (choice E) (REM
rebound) occurs after discontinuation of REM-suppressing substances (e.g.,
barbiturates, alcohol, phenothiazines, and MAO inhibitors).

Question 3 of 3
One morning, the patient develops chest pain and is later confirmed to have had
a myocardial infarction. This event likely occurred during
which of the following stages of sleep?
/ A. Stage 1
/ B. Stage 2
/ C. Stage 3
/ D. Stage 4
/ E. REM sleep

Explanation - Q: 2.3 Close

The correct answer is E. REM sleep has been termed paradoxical sleep
because many physiological parameters are similar to the awake state.
During REM, brain oxygen use increases and this stage is associated with
near total paralysis of the skeletal system and penile erections. Pulse,
respiration, and blood pressure are high and variable, during REM sleep,
much higher than during non-REM sleep, and often higher than during
waking.
Stages 1, 2, 3, and 4 (choices A, B, C, and D) are non-REM sleep. Stage 1
is associated with low voltage theta waves. Stage 2 is associated with sleep
spindles and K complexes. Stages 3 and 4 are associated with delta waves
(slow wave sleep). REM sleep is low-voltage with random, fast, sawtooth
waves.


A 24-year-old graduate student is studying for examinations in the library. She
realizes that she has lost her notes, and becomes very upset.
Suddenly, according to library patrons, her head falls forward, and she slumps
over her books, striking her forehead against the table. Within
minutes, she appears to recover, sits up, and is helped to the student health
service by her roommate, who was present in the library. At the
clinic, the patient is confused about what happened to her, but is alert and
cooperative. She has no significant past medical history, and
denies any previous events similar to the one today. Review of systems is mostly
noncontributory, except for excessive sleepiness, and
frequently waking up and feeling that she is unable to move for a few moments in
the morning. She says that she has been sleeping well lately
(7-8 hours/night), despite a busy study schedule. Her roommate states that she
has never seen the patient sleepwalking, and that she does not
snore. Her vital signs are normaI, and physical examination, including a
neurologic examination, is unrevealing. A small laceration is present
on her forehead, where she hit her head against the table.
Question 1 of 3
Which of the following is the most likely diagnosis?
/ A. Chronic sleep restriction
/ B. Hypoglycemia
/ C. Mitral valve prolapse
/ D. Narcolepsy
/ E. SIeep apnea

Explanation - Q: 3.1 Close

The correct answer is D. This patient may be developing narcolepsy.
Narcolepsy is a sleep disorder characterized by excessive daytime
somnolence, in addition to cataplexy, hypnagogic or hypnopompic
hallucinations, and sleep paralysis. This patient had a sudden loss of
muscular tone elicited by emotion (cataplexy), and reports a common, but
nonspecific symptom of narcolepsy, sleep paralysis, a subjective feeling of
being unable to move immediately upon awakening.
Chronic sleep restriction (choice A), also known as sleep deprivation, is
unlikely, given her recent history of sleeping well.
Hypoglycemia (choice B) would likely have produced some degree of
autonomic changes, and would likely not have resolved on its own.
Mitral valve prolapse (choice C) is unlikely, given the lack of pertinent
physical examination findings (e.g., midsystolic click).
Sleep apnea (choice E) is unlikely in a 24-year-old woman who does not
snore.


Question 2 of 3
The patient's scalp laceration is treated, and she is sent home. She later returns
to the clinic to have her stitches removed. She reports that
during the intervening time, she had a recurrence of symptoms, falling to the floor
in the laundry room of her apartment building. AIso, her
daytime sleepiness has continued, and she reports she "needs to take naps a
lot." If an electroencephalogram were performed during the first
30 minutes of one of her naps, which of the following would most likely be
observed?
/ A. 3 Hz spike and wave discharge
/ B. 4-7 Hz rhythm, sleep spindles, and K-complexes
/ C. Low-amplitude, 8-13 Hz rhythm
/ D. Low-amplitude, high frequency, irregular pattern
/ E. Paroxysmal diffuse outbursts of high voltage, fast rhythm with many spikes

Explanation - Q: 3.2 Close

The correct answer is D. A low-amplitude, high frequency pattern is
characteristic of beta activity during alertness, or rapid eye movement (REM)
sleep. In narcolepsy, the latency to REM sleep is much shortened. In some
cases, affected individuals enter REM nearly instantaneously upon falling
asleep. In normal individuals, REM sleep does not begin for approximately
90 minutes after falling asleep, so in many cases, a short nap will not show
any REM.
A 3 Hz spike and wave discharge (choice A) is characteristic of petit mal
epilepsy (absence seizures).
4-7 Hz (theta) rhythm, sleep spindles and K complexes (choice B) are
characteristic of stage 2 of slow wave sleep.
An 8-13 Hz, low-amplitude rhythm (choice C) is called an alpha rhythm. It is
most prominent over the parieto-occipital cortex with the eyes closed and is
indicative of relaxed wakefulness in most normal individuals.
A characteristic fast rhythm with many spikes and paroxysmal diffuse
outbursts of high voltage (choice E) is seen in generalized tonic-clonic
seizures (grand mal seizures). During the clonic phase, slow waves interrupt
the high amplitude spikes.


Question 3 of 3
Which of the following could be used to treat this patient's excessive daytime
somnolence?
/ A. Amitriptyline
/ B. Dextroamphetamine
/ C. FIuoxetine
/ D. Imipramine
/ E. Triazolam

Explanation - Q: 3.3 Close

The correct answer is B. Therapy of narcolepsy is directed toward
reduction in daytime somnolence and suppression of REM phenomena (such
as cataplexy, hypnagogic hallucinations, and sleep paralysis). Reduction in
daytime somnolence is accomplished with CNS stimulants, such as
dextroamphetamine and methylphenidate, although a new drug, modafinil,
may promote wakefulness without the addictive potential of amphetamines.
Suppression of REM phenomena can be achieved via the use of tricyclic
antidepressants (e.g., protriptyline and clomipramine) and selective serotonin
reuptake inhibitors (SSRIs; e.g., fluoxetine, choice C).
Amitriptyline (choice A) is a tricyclic antidepressant, and suppresses REM,
but is very sedating, so would not be used to treat daytime somnolence.
Imipramine (choice D) is used to suppress REM phenomena, not to improve
alertness.
Triazolam (choice E) is a benzodiazepine, and would tend to worsen the
patient's daytime somnolence.


A 9-year-old boy is brought to the clinic with "sleep problems." The patient's
mother reports episodes of her son sitting up in bed, screaming
and crying. He appears frightened and is breathing fast and sweating. She tries
to wake him up, but without success. She says that the
episodes are probably more frightening for her than for her son because he does
not seem to recall anything about them.
Question 1 of 3
Which of the following is the most likely diagnosis?
/ A. Enuresis
/ B. Night terror disorder
/ C. Nightmare disorder
/ D. Rapid eye movement sleep behavior disorder
/ E. Somnambulism

Explanation - Q: 4.1 Close

The correct answer is B. Night terrors are characterized by a sudden
arousal from non-REM (NREM) (usually slow-wave) sleep with behavioral
and autonomic manifestations of fear. During these episodes, the patients
often scream or cry out, and also have a significant amount of autonomic
activity (e.g., tachycardia, tachypnea, sweating) and increased muscle tone.
They usually do not respond to external stimuli, and when they awaken, they
are often confused, but amnestic about the episode.
Enuresis (choice A) is bed-wetting. It is not directly related to night terrors.
Patients with nightmare disorder (choice C) describe frightening dreams.
Arousal during the dream frequently occurs. The fact that there is a dream
differentiates this disorder from night terror disorder.
In rapid eye movement sleep behavior disorder (choice D), the patient
actually acts out their dreams, and have been known to kick, punch, jump,
and run. They can injure their bed partner. These patients can awaken
rapidly and become rapidly oriented. They generally have a vivid recollection
of their dreams, as this disorder occurs during REM sleep. Despite this, they
rarely develop excessive daytime sleepiness.
Somnambulism (choice E), or sleepwalking, can present in a variety of
ways. Patients may just sit up, or walk, or exhibit complex behaviors. This
generally occurs during slow-wave NREM sleep. During the episode, they
often have a blank, staring face and are unresponsive to others.






Question 2 of 3
Which of the following medications could be used to treat this condition?
/ A. Bupropion
/ B. Chlorpromazine
/ C. Diazepam
/ D. Diphenhydramine
/ E. Sertraline

Explanation - Q: 4.2 Close

The correct answer is C. Medications are frequently not necessary for this
disorder, but in severe cases, the patient may be prescribed a
benzodiazepine, such as diazepam. Benzodiazepines suppress stage 3 and
stage 4 sleep (when night terrors occur). This should only be used short-
term, especially in children, because deep sleep is the period during which
growth hormone is primarily secreted.
Bupropion (choice A) and sertraline (choice E) are antidepressants and
may improve sleep, but do not treat disorders of slow wave NREM sleep.
Chlorpromazine (choice B) is an antipsychotic medication, and is not
indicated for this disorder.
Diphenhydramine (choice D) is an antihistamine sometimes used as a sleep
aid. Central histamine blockade results in sedation. Diphenhydramine may
help with sleep latency, but does not suppress slow wave sleep. Hence, it
would not treat night terrors.

Question 3 of 3
Which of the following is the mechanism of action of the drug that might be
prescribed to this patient?
/ A. BIocks dopamine and norepinephrine reuptake
/ B. BIocks dopamine receptors
/ C. BIocks histamine receptors
/ D. BIocks serotonin reuptake
/ E. Potentiates GABA effect

Explanation - Q: 4.3 Close

The correct answer is E. Benzodiazepines potentiate GABA at the GABA-A
receptor, by increasing the frequency of GABA-mediated chloride channel
opening.
Bupropion blocks the reuptake of dopamine and norepinephrine (choice A).
Chlorpromazine blocks dopamine receptors (choice B).
Diphenhydramine blocks histamine receptors (choice C).
Sertraline blocks serotonin reuptake (choice D).

A 57-year-old woman presents to her family physician complaining of numbness
and tingling in her right thumb, index and long finger for the
past four weeks. She reports that she wakes up in the middle of the night with
these symptoms and needs to shake her hands to "wake" them
up. She denies numbness or tingling in her other hand or either leg. She has no
neck or upper arm pain. On physical examination, her
symptoms are reproduced by Tinel testing (tapping the affected area). Her
symptoms are also exacerbated by hyperflexion of the wrist. There
is decreased sensation over the palmar aspects of the thumb, index and middle
fingers. There is no apparent motor weakness.


Question 1 of 6
Which of the following is the most likely diagnosis?
/ A. C5 cervical nerve root compression
/ B. Carpal tunnel syndrome
/ C. Cubital tunnel syndrome
/ D. Radial tunnel syndrome
/ E. Tarsal tunnel syndrome


Explanation - Q: 1.1 Close

The correct answer is B. The patient's symptoms are classic for carpal
tunnel syndrome. Carpal tunnel syndrome is the most common compressive
neuropathy in the upper extremity. It is caused by compression of the median
nerve in the carpal tunnel. The median nerve provides sensation to the
palmar side of the thumb, index finger, long finger and radial half of the ring
finger. The floor of the tunnel is formed by the carpal bones; the roof is
formed by the transverse carpal ligament.
A patient with a C5 nerve cervical nerve root compression (choice A) would
have weakness in their deltoids and biceps with sensory changes in the
lateral arm.
Cubital tunnel syndrome (choice C) is compression of the ulnar nerve at the
elbow. Symptoms include pain and paresthesias over the medial forearm
and hand as well as weakness in the ulnar nerve distribution.
Radial tunnel syndrome (choice D) is compression of a branch of the radial
nerve at the forearm. It is a pain-only problem without motor or sensory
changes.
Tarsal tunnel syndrome (choice E) is compression of the tibial nerve at the
medial ankle.


Question 2 of 6
Which of the following structures is responsible for the patient's symptoms?
/ A. Median nerve
/ B. Posterior interosseous nerve
/ C. Radial nerve
/ D. Sural nerve
/ E. UInar nerve


Explanation - Q: 1.2 Close

The correct answer is A. The median nerve is responsible for the patient's
symptoms. The median nerve provides sensation to the palmar side of the
thumb, index finger, long finger, and radial half of the ring finger.
The posterior interosseous nerve (choice B) provides no sensory innervation
to the hand.
The radial nerve (choice C) provides sensation to the dorsum of the hand
over the radial two and one-half digits over the proximal phalanx.
The sural nerve (choice D) supplies the lateral side of the foot.
The ulnar nerve (choice E) innervates the palmar small finger and ulnar one-
half of the ring finger


Question 3 of 6
Which of the following muscles is innervated by the affected nerve?
/ A. Abductor digiti minimi
/ B. Abductor pollicis brevis
/ C. Abductor pollicis longus
/ D. Adductor pollicis
/ E. Opponens digiti minimi


Explanation - Q: 1.3 Close

The correct answer is B. The symptoms above can be attributed to carpal
tunnel syndrome or compression of the median nerve. Of the muscles listed,
the median nerve only innervates the abductor pollicis brevis.
The abductor digiti minimi (choice A) and adductor pollicis (choice D) are
innervated by the ulnar nerve.
The abductor pollicis longus (choice C) is innervated by the posterior
interosseous branch of the radial nerve.
The opponens digiti minimi (choice E) is innervated by the ulnar nerve.


Question 4 of 6
The patient is given an injection of a corticosteroid into the affected area. What is
the mechanism of action of steroids in decreasing
inflammation?
/ A. Decreases the movement of granulocytes to the affected area
/ B. Inhibits the enzyme phospholipase A2
/ C. Interacts with opioid receptors
/ D. Irreversibly inhibits cyclooxygenase
/ E. Suppresses phagocytosis and lysosomal enzyme activity

Explanation - Q: 1.4 Close

The correct answer is B. Glucocorticoids decrease inflammation because of
their ability to lower peripheral lymphocyte levels and inhibit the enzyme
phospholipase A2. Phospholipase A2 is the enzyme that releases
arachidonic acid, the precursor of prostaglandins.
Colchicine is used to decrease the movement of granulocytes to the affected
area (choice A) in the treatment of gout.
Morphine and other opioids interact with opioid receptors (choice C).
Aspirin works by irreversibly acetylating and thus inhibiting cyclooxygenase
(choice D), which ultimately inhibits prostaglandin synthesis.
Suppressing phagocytosis and lysosomal enzyme activity (choice E) is the
mechanism of action of gold salts in decreasing inflammation.



Question 5 of 6
The patient does not respond to conservative methods of treatment and decides
to undergo an operative intervention. Preoperatively she is
given cefazolin, a first generation cephalosporin, for antibiotic prophylaxis. Which
of the following is the mechanism of action of cefazolin?
/ A. Inhibits bacterial cell wall synthesis
/ B. Inhibits bacterial protein synthesis by binding the 30s subunit
/ C. Inhibits bacterial protein synthesis by binding the 50s subunit
/ D. Inhibits bacteriaI RNA synthesis
/ E. Inhibits DNA gyrase

Explanation - Q: 1.5 Close

The correct answer is A. Cefazolin is a first generation cephalosporin.
Cephalosporins inhibit bacterial peptidoglycan synthesis and thus interfere
with the last step of bacterial cell wall synthesis.
Many antibiotics work by interfering with protein synthesis in many ways.
Aminoglycosides bind the 30s subunit of the bacterial ribosome and interfere
with protein synthesis (choice B). Erythromycin inhibits bacterial protein
synthesis by binding the 50s subunit (choice C).
Rifampin inhibits RNA synthesis in bacteria (choice D).
Quinolones inhibit DNA gyrase (choice E).


Question 6 of 6
Hypothyroidism is part of the differential diagnosis in patients with the condition
described above. Which of the following laboratory values
would be consistent with the diagnosis of primary hypothyroidism?
/ A. Decreased free T4 and decreased TSH
/ B. Decreased free T4 and increased TSH
/ C. Increased free T4 and decreased TSH
/ D. Increased free T4 and increased TSH
/ E. Increased free T4 and normaI TSH


Explanation - Q: 1.6 Close

The correct answer is B. Decreased levels of T4 and increased TSH are
signs of primary hypothyroidism. The TSH is high because the pituitary is
working to overcome the hypothyroid state by increasing the stimulation of
the thyroid gland.
Decreased free T4 and decreased TSH (choice A) are seen in patients with
secondary or tertiary hypothyroidism.
Increased free T4 and decreased TSH (choice C) are seen in patients with
hyperthyroidism.
Increased free T4 and increased TSH (choice D) may be seen in patients
with a pituitary TSH-secreting tumor.
Increased free T4 and normal TSH (choice E) is rare and may be seen in
patients with a peripheral unresponsiveness type of thyroid disease.


A 67-year-old woman has a 3 month history of increasing right shoulder pain.
She is now unable to lift her arm to brush her hair or to take a can
off a shelf. She denies any numbness or tingling or radiation of the pain down her
arm. On physical examination, she exhibits weakness in
abduction and external rotation or her right arm. Her shoulder shows a normal
passive range of motion. When asked to hold her arms
abducted to 90 degrees, she is unable to do so on the right. There is no motor
weakness in her forearm or hand. Her right upper extremity has
normal pulses and normal sensation.
Question 1 of 6
Which of the following is the most likely diagnosis?
/ A. Brachial plexus injury
/ B. GIenohumeral osteoarthritis
/ C. Proximal humerus fracture
/ D. Rotator cuff tear
/ E. Shoulder dislocation


Explanation - Q: 2.1 Close

The correct answer is D. The above history describes a rotator cuff tear.
Patients classically have difficulty doing overhead activities. On physical
examination, she has weakness with abduction and external rotation of her
shoulder. The supraspinatus is responsible for abduction and the
infraspinatus is responsible for external rotation of the shoulder. These
muscles, along with the teres minor and subscapularis, make up the rotator
cuff.
Brachial plexus injuries (choice A) commonly present with some type of
numbness or tingling in the upper extremity.
Glenohumeral arthritis (choice B) is unlikely because she has a full passive
range of motion, and shoulder arthritis is commonly associated with a
decreased range of motion. Also this history is relatively short term.
Proximal humerus fracture (choice C) and shoulder dislocation (choice E)
are unlikely because this process has been present for 3 months. These
injuries need more acute treatment. Also she has a full passive range of
motion, which is not common with fractures or dislocations.

Question 2 of 6
Which of the following muscles make up the injured structure?
/ A. Supraspinatus, infraspinatus, teres major, deltoid
/ B. Supraspinatus, infraspinatus, teres major, subscapularis
/ C. Supraspinatus, infraspinatus, teres minor, deltoid
/ D. Supraspinatus, infraspinatus, teres minor, subscapularis
/ E. Supraspinatus, infraspinatus, teres minor, trapezius


Explanation - Q: 2.1 Close

The correct answer is D. The above history describes a rotator cuff tear.
Patients classically have difficulty doing overhead activities. On physical
examination, she has weakness with abduction and external rotation of her
shoulder. The supraspinatus is responsible for abduction and the
infraspinatus is responsible for external rotation of the shoulder. These
muscles, along with the teres minor and subscapularis, make up the rotator
cuff.
Brachial plexus injuries (choice A) commonly present with some type of
numbness or tingling in the upper extremity.
Glenohumeral arthritis (choice B) is unlikely because she has a full passive
range of motion, and shoulder arthritis is commonly associated with a
decreased range of motion. Also this history is relatively short term.
Proximal humerus fracture (choice C) and shoulder dislocation (choice E)
are unlikely because this process has been present for 3 months. These
injuries need more acute treatment. Also she has a full passive range of
motion, which is not common with fractures or dislocations.

Question 2 of 6
Which of the following muscles make up the injured structure?
/ A. Supraspinatus, infraspinatus, teres major, deltoid
/ B. Supraspinatus, infraspinatus, teres major, subscapularis
/ C. Supraspinatus, infraspinatus, teres minor, deltoid
/ D. Supraspinatus, infraspinatus, teres minor, subscapularis
/ E. Supraspinatus, infraspinatus, teres minor, trapezius


Explanation - Q: 2.2 Close

The correct answer is D. The muscles that make up the rotator cuff are:
supraspinatus, infraspinatus, teres minor, subscapularis. An easy way to
remember these are the acronym SITS (Supraspinatus, Infraspinatus, Teres
minor, Subscapularis).
The deltoid (choices A and C), teres major (choices A and B), and
trapezius (choice E) are not considered part of the rotator cuff.



Question 3 of 6
Which of the following nerves innervates the deltoid muscle?
/ A. Axillary
/ B. Dorsal scapular
/ C. Long thoracic
/ D. Suprascapular
/ E. Thoracodorsal

Explanation - Q: 2.3 Close

The correct answer is A. The axillary nerve innervates the deltoid muscle,
as well as provides sensation over the lateral part of the arm.
The dorsal scapular nerve (choice B) innervates the rhomboids and levator
scapulae muscles.
The long thoracic nerve (choice C) innervates the serratus anterior. Injury to
this results in winging of the scapula.
The suprascapular nerve (choice D) innervates the supraspinatus and
infraspinatus muscles.
The thoracodorsal nerve (choice E) innervates the latissimus dorsi muscle.


Question 4 of 6
Which of the following structures are found in the quadrangular space?
/ A. Circumflex scapular vessels and the radial nerve
/ B. Posterior humeral circumflex artery and axillary nerve
/ C. Posterior humeral circumflex artery and the radial nerve
/ D. Profunda brachii artery and the axillary nerve
/ E. Profunda brachii artery and the radial nerve


Explanation - Q: 2.4 Close

The correct answer is B. The quadrangular space is bounded superiorly by
the teres minor and subscapularis muscles, medially by the long head of the
triceps, laterally by the surgical neck of the humerus, and inferiorly by the
teres major muscle. It transmits the posterior humeral circumflex vessels and
the axillary nerve.
The triangular space, which is bounded by the teres minor, teres major and
the long head of the triceps, transmits the circumflex scapular vessels
(choice A).
The posterior humeral circumflex artery and radial nerve (choice C) do not
run in a named interval or space.
The profunda brachii artery and the axillary nerve (choice D) do not run in a
named interval or space.
The triangular interval, which is bound by the teres major, long head of the
triceps and the medial head of the triceps, transmits the radial nerve and the
profunda brachii artery (choice E).


Question 5 of 6
Which of the following cervical nerve roots provides sensation to the lateral arm
and innervates the deltoid muscle?
/ A. C4
/ B. C5
/ C. C6
/ D. C7
/ E. C8



Explanation - Q: 2.5 Close

The correct answer is B. The C5 nerve root innervates the deltoid and
provides sensation to the lateral arm.
C4 (choice A) innervates the scapular muscles and provides sensation to
the lateral neck and shoulder.
C6 (choice C) innervates the wrist extensors, biceps and triceps, and
provides sensation to the radial forearm.
C7 (choice D) innervates the triceps and wrist flexors and provides
sensation to the middle finger.
C8 (choice E) innervates the finger flexors and interossei and provides
sensation to the ulnar hand.

Question 6 of 6
The patient has failed treatment by conservative measures for the above
condition and decides to undergo operative intervention.
Postoperatively, she is oversedated with morphine, which depresses her
respiratory drive. Which of the following drugs may be used to
reverse the morphine by acting as a competitive antagonist?
/ A. Acetylcysteine
/ B. Deferoxamine
/ C. FIumazenil
/ D. Naloxone
/ E. Physostigmine


Explanation - Q: 2.6 Close

The correct answer is D. Naloxone is a opioid antagonist. It can reverse all
aspects of morphine overdose, e.g., respiratory depression, level of
consciousness, bowel activity, and pupil size. Keep in mind that naloxone
has a short half-life, so repeated dosing may be necessary.
Acetylcysteine (choice A) is used to treat acetaminophen toxicity.
Deferoxamine (choice B) is used to treat iron toxicity.
Flumazenil (choice C) is used to reverse benzodiazepines.
Physostigmine (choice E) is used to treat anticholinergic toxicity.


A 6-week-old boy is brought to his family physician. His parents report that he
has not had significant use of his right arm since birth. Birth
history is significant for a prolonged labor with difficult breech delivery. On
physical examination, his arm hangs at his side and is in a medially
rotated position with the forearm in pronation. He will actively use his left arm, but
does not move his affected right arm or hand.


Question 1 of 6
Which of the following is the most likely diagnosis?
/ A. CIavicle fracture
/ B. Erb-Duchenne paralysis
/ C. Intracranial hemorrhage
/ D. KIumpke's paralysis
/ E. Spinal cord injury


Explanation - Q: 3.1 Close

The correct answer is B. This is a classic history for a birth related Erb-
Duchenne paralysis (also called Erb's Palsy). There is a history of a difficult
delivery. The child's arm rests at his side, is medially rotated and the forearm
is pronated. This is due to traction injury to the superior trunk of the brachial
plexus. The C5 and C6 nerve roots are most commonly affected. Erb-
Duchenne paralysis is the classic "waiter's tip" position.
Clavicle fractures (choice A) occur as a result of birth trauma, however, by 6
weeks of age the fracture usually is healed and the baby is asymptomatic.
Intracranial hemorrhage (choice C) is a potential birth-related injury, but its
presentation is variable and usually does not result in a isolated unilateral
upper extremity injury.
Klumpke's paralysis (choice D) is a birth palsy that results in a claw hand
due to injury of the C8 and T1 nerve roots.
Spinal cord injuries (choice E) do occur after traumatic births but are not
isolated to a single upper extremity finding.



Question 2 of 6
Injury to which of the following cervical nerve roots account for this patient's
posture?
/ A. C4 and C5
/ B. C5 and C6
/ C. C6 and C7
/ D. C7 and C8
/ E. C8 and T1

Explanation - Q: 3.2 Close

The correct answer is B. This patient has an Erb-Duchenne palsy, which is
the result of an injury to the superior trunks of the C5 and C6 nerve roots.
The C8 and T1 (choice E) nerve roots are injured in a Klumpke's paralysis.
The other combinations (choices A, C, D) all may be injured as a result of
birth palsy, but do not have distinct syndromes associated with them.


Question 3 of 6
This condition is associated with injury to the phrenic nerve in approximately 5%
of cases. Which of the following cervical nerve roots form the
phrenic nerve?
/ A. C2 through C4
/ B. C3 through C5
/ C. C4 through C6
/ D. C5 through C7
/ E. C8 through C8


Explanation - Q: 3.3 Close

The correct answer B. Phrenic nerve injury is seen in 5% of Erb-Duchenne
palsies. This nerve arises from the third through fifth cervical nerves (C3-C5).
The C5 through C7 (choice D) nerves give rise to the long thoracic nerve,
which innervates the serratus anterior.
The remainder of the choices do not make up the phrenic nerve.

Question 4 of 6
Paralysis of which of the following muscles results in the pronated position of this
patients forearm?
/ A. Biceps
/ B. Deltoid
/ C. Pronator quadratus
/ D. Pronator teres
/ E. Triceps


Explanation - Q: 3.4 Close

The correct answer is A. The biceps muscle is innervated by the C6 nerve,
and is a supinator of the forearm as well as a flexor of the elbow. Paralysis of
the biceps will result in overpull of the unaffected pronator muscles of the
forearm.
The deltoid (choice B) is innervated by the C5 nerve root. It is an abductor of
the arm. Paralysis will result in the patient's arm hanging at his or her side.
The pronator quadratus (choice C) and teres (choice D) are pronators of the
forearm. Paralysis will result in less pronation of the forearm, and they are
not involved in a Erb's palsy.
The triceps (choice E) extends the elbow and is not responsible for this
patient's pronated position.


Question 5 of 6
This child's father then states that he had a nerve palsy in his left arm. He
sustained this injury when he was 21 years old. He reports that he
was out with his friends drinking alcohol and woke up the next morning unable to
extend his fingers or wrist. His symptoms improved 1 year
after the injury. Which of the following nerves was most likely injured?
/ A. Axillary nerve
/ B. Median nerve
/ C. Musculocutaneous nerve
/ D. Radial nerve
/ E. UInar nerve


Explanation - Q: 3.5 Close

The correct answer is D. The child's father most likely had a radial nerve
palsy, sometimes called a Saturday night palsy. The radial nerve injury will
result in an inability to extend the elbow and extend the wrist or fingers.
Injury to the axillary nerve (choice A) is usually caused by proximal humerus
fractures and dislocations. It will result in weakness of lateral rotation and
abduction of the arm.
Injury to the median nerve (choice B), which may be a result of distal
humerus fracture, will result in loss of pronation, opposition of the thumb, and
flexion of the thumb and index finger interphalangeal joints.
Injury to the musculocutaneous nerve (choice C) results in weakness of
elbow flexion (biceps and brachialis) and supination (biceps).
Injury to the ulnar nerve (choice E) results in a claw hand (ring and little
fingers are hyperextended at the metacarpophalangeal joints and flexed at
the interphalangeal joints).



Question 6 of 6
The child's mother also reports that she had a nerve injury in the past. She states
that she was involved in a crush injury to her left leg as a
teenager. After the injury, she had a foot drop (she was unable to dorsiflex her
left foot at the ankle). Which of the following nerves was most
Iikely injured?
/ A. Deep peroneal nerve
/ B. Femoral nerve
/ C. Superficial peroneal nerve
/ D. Sural nerve
/ E. Tibial nerve






Explanation - Q: 3.6 Close

The correct answer is A. This patient most likely had an injury to her deep
peroneal nerve. It is a branch of the common peroneal nerve. The deep
peroneal nerve runs along the anterior surface of the interosseous
membrane and supplies the muscles of the anterior compartment of the leg
(tibialis anterior, extensor hallucis longus, extensor digitorum longus, and
peroneus tertius). The tibialis anterior is responsible for dorsiflexion of the
foot at the ankle. Injury to this nerve would result in a foot drop.
The femoral nerve (choice B) innervates the flexors of the hip (iliacus,
psoas, pectineus, rectus femoris and sartorius) as well as the extensors of
the knee (quadriceps muscles).
The superficial peroneal nerve (choice C) innervates the lateral
compartment of the leg (peroneus longus and peroneus brevis).
The sural nerve (choice D) provides sensation only to the lateral aspect of
the foot.
The tibial nerve (choice E) innervates the muscles of the superficial posterior
(gastrocnemius, soleus, plantaris) and the deep posterior (popliteus, flexor
hallucis longus, flexor digitorum longus, and tibialis posterior) compartments
of the leg. Injury to this nerve would result in a loss of plantar flexion.

A 23-year-old man is involved in a motor vehicle accident and is brought to the
emergency department with a displaced fracture of the distal
third of his left humeral shaft. On his right side he has a displaced fracture of the
surgical neck of his humerus as well as a fracture of the
medial epicondyle of his distal humerus. He complains of pain in his both arms
as well as the inability to move part of his hand. On physical
examination, his arm is swollen with a deformity at the corresponding parts of his
humerus. His motor examination is abnormaI.
Question 1 of 6
Which of the following nerves is most commonly damaged with this type of
injury?
/ A. Axillary
/ B. Median
/ C. Musculocutaneous
/ D. Radial
/ E. UInar

Explanation - Q: 4.1 Close

The correct answer is D. The radial nerve is most commonly injured in
fractures to the mid shaft or distal third of the humerus. The radial nerve is a
continuation of the posterior cord of the brachial plexus. It runs along the
posterior wall of the axilla and passes through the triangular space. It lies in
the spiral groove on the posterior aspect of the humerus and at the middle to
distal third of the humerus, it crosses the back of the humerus and pierces
the lateral intermuscular septum.
The axillary nerve (choice A) is commonly injured in proximal humerus
fractures and dislocations.
The median nerve (choice B) is commonly injured in supracondylar fractures
of the humerus.
The musculocutaneous nerve (choice C) is not commonly injured in
humerus fractures. This nerve runs more proximal in the arm and would not
be injured in a mid shaft or distal humerus fracture. It is a branch of the
lateral cord of the brachial plexus and supplies the coracobrachialis, biceps
brachii, and brachialis muscle. It terminates in the lateral cutaneous nerve of
the forearm.
The ulnar nerve (choice E) is commonly injured in medial epicondyle
fractures of the distal humerus.


Question 2 of 6
Which of the following muscles will this patient most likely have trouble using
secondary to the nerve injury sustained in his accident?
/ A. Biceps
/ B. Extensor carpi radialis longus
/ C. FIexor carpi radialis
/ D. FIexor carpi ulnaris
/ E. Pronator teres

Explanation - Q: 4.2 Close

The correct answer is B. The radial nerve innervates the triceps,
brachioradialis, wrist and finger extensors, and supinator. The extensor carpi
radialis longus is a wrist extensor and is innervated by the radial nerve.
The biceps muscles (choice A) are innervated by the musculocutaneous
nerve.
The flexor carpi radialis (choice C) is innervated by the median nerve.
The flexor carpi ulnaris (choice D) is innervated by the ulnar nerve.
The pronator teres (choice E) is innervated by the median nerve.


Question 3 of 6
Which of the following arteries may be injured with the fracture of his right
humerus?
/ A. Axillary artery
/ B. Brachial artery
/ C. Profunda brachii artery
/ D. Radial artery
/ E. UInar artery


Explanation - Q: 4.3 Close

The correct answer is C. The profunda brachii (deep brachial) artery is a
branch of the brachial artery, which runs posteriorly in the arm with the radial
artery and may be injured with humeral shaft fractures.
The axillary artery (choice A) extends from the first rib to the inferior border
of the teres major muscle where it becomes the brachial artery. This location
is too proximal for the fracture described.
The brachial artery (choice B) extends from the inferior border of the teres
major muscle to the cubital fossa in the elbow where it divides into the radial
and ulnar arteries.
The radial artery (choice D) is the lateral branch of the brachial artery. It
would not be injured in this injury since it originates distal to the fracture site.
The ulnar artery (choice E) is the medial branch of the brachial artery. It too
would not be injured in this injury since it originates distal to the fracture site.


Question 4 of 6
His left shoulder is fractured at the surgical neck of the humerus. If he had a
nerve injury, which of the following muscles would most likely be
affected?
/ A. Axillary
/ B. Median
/ C. Musculocutaneous
/ D. Radial
/ E. UInar


Explanation - Q: 4.4 Close

The correct answer is A. The axillary nerve is commonly injured in proximal
humerus fractures and dislocations. The axillary nerve is a branch of the
posterior cord of the brachial plexus. It runs along the posterior wall of the
axilla and then runs through the quadrangular space where it touches the
surgical neck of the humerus. At this point it may be injured in fractures or
dislocations of the proximal humerus.
The median nerve (choice B) is commonly injured in supracondylar fractures
of the humerus.
The musculocutaneous nerve (choice C) is not commonly injured in
humerus fractures.
The radial nerve (choice D) is most commonly injured in fractures to the mid
shaft or distal third of the humerus
The ulnar nerve (choice E) is commonly injured in medial epicondyle
fractures of the distal humerus.


Question 5 of 6
If his medial epicondyle fracture resulted in a nerve injury, which of the following
muscles would most likely be affected?
/ A. Extensor digiti minimi
/ B. FIexor carpi radialis
/ C. FIexor carpi ulnaris
/ D. FIexor digitorum superficialis
/ E. FIexor pollicis longus


Explanation - Q: 4.5 Close

The correct answer is C. One must first figure out that the ulnar nerve is
injured in a fracture to the medial epicondyle. The flexor carpi ulnaris is the
only muscle listed above innervated by the ulnar nerve.
The extensor digiti minimi (choice A) is innervated by the posterior
interosseous branch of the radial nerve.
The flexor carpi radialis (choice B), flexor digitorum superficialis (choice D),
and flexor pollicis longus (choice E) are innervated by the median nerve.

Question 6 of 6
Peripheral nerves are divided into different layers. Which of the following
describes the perineurium?
/ A. Connective tissue gaps between Schwann cells
/ B. Connective tissue that covers fascicles
/ C. Connective tissue that covers muscle bundles
/ D. Connective tissue that covers a single nerve fiber
/ E. Connective tissue that surrounds a entire nerve


Explanation - Q: 4.6 Close

The correct answer is B. Connective tissue that covers fascicles is termed
the perineurium. The perineurium actually has two layers, an outer
connective tissue layer and an inner epithelial layer. This inner layer has tight
junctions and forms part of the blood-nerve barrier.
Gaps between Schwann cells (choice A) are the nodes of Ranvier.
Connective tissue that surrounds muscle bundles (choice C) is epimysium
Connective tissue that covers a single nerve fiber (choice D) is called the
endoneurium.
Connective tissue that surrounds a entire nerve (choice E ) is called the
epineurium.



A 35-year-old woman consults a physician because of increasing discomfort in
her hands over the past year. Her symptoms have developed
insidiously and are often worst for the first hour after rising. She has also been
experiencing vague symptoms of fatigue and malaise,
particularly in the early afternoon. Physical examination of her hands is notable
for tenderness and synovial thickening of most of the small
hand joints of both hands, particularly the proximal interphalangeal and
metacarpophalangeal joints. A number of joints elsewhere, including in
the feet, elbows, and ankles, are also symmetrically involved. An autoantibody
directed against lgG is detected in serum.
Question 1 of 6
Which of the following is the most likely diagnosis?
/ A. Gout
/ B. Osteoarthritis
/ C. Osteomyelitis
/ D. Osteoporosis
/ E. Rheumatoid arthritis

Explanation - Q: 5.1 Close

The correct answer is E. This woman most likely has rheumatoid arthritis
(RA). This chronic syndrome is characterized by inflammation of peripheral
joints, and may or may not be accompanied by systemic manifestations. The
American Rheumatism Association has put out revised (1987) criteria for the
diagnosis of rheumatoid arthritis that require at least 4 of the following for
diagnosis: morning stiffness for one hour or longer, arthritis of three or more
joints, arthritis of hand joints, symmetric arthritis, rheumatoid nodules, serum
rheumatoid factor (RF; anti-IgG) present, and characteristic radiographic
changes.
In gout (choice A), the hands are usually spared, with common sites of
involvement including the foot (typically the great toe), knee, wrist, and
elbow.
Osteoarthritis (choice B) usually produces only a short period of morning
stiffness and commonly involves the distal as well as proximal
interphalangeal joints, vertebral column, hip, and knee.
Osteomyelitis (choice C) may involve vertebrae, feet (usually in diabetics), or
the metaphyses of the tibia or femur (more common in children).
Osteoporosis (choice D) causes bony loss, and is usually painless unless a
fracture occurs.


Question 2 of 6
Arthroscopic examination and biopsy of a joint would be most likely to show
which of the following?
/ A. Large numbers of bacteria
/ B. Roughly irregular and pitted cartilage surface
/ C. Small clusters of irregularly shaped, weakly birefringent crystals
/ D. Small clusters of needle-shaped, negatively birefringent crystals
/ E. Thickened synovium with large numbers of lymphocytes and plasma cells


Explanation - Q: 5.2 Close

The correct answer is E. The synovial membrane in rheumatoid arthritis is
attacked by lymphocytes and plasma cells and characteristically becomes
thickened, forming a pannus that can erode the underlying cartilage.
Large numbers of bacteria (choice A) suggests infectious arthritis.
Roughly irregular and pitted cartilage surface (choice B) suggests
osteoarthritis.
Small clusters of irregularly shaped, weakly birefringent crystals (choice C)
suggests pseudogout (calcium pyrophosphate crystal deposition).
Small clusters of needle-shaped, negatively birefringent crystals (choice D)
suggests gout.



Question 3 of 6
The patient also has a 2 cm, firm, non-tender, oval subcutaneous nodule near
her right elbow. Biopsy of this nodule would most likely show
which of the following?
/ A. Amorphous calcified material surrounded by fibrotic subcutaneous tissue
/ B. Central fibrinoid necrosis surrounded by epithelioid histiocytes
/ C. Cystic space filled with keratin and lined by squamous epithelium
/ D. Interwoven neural fibers
/ E. Membrane bound mass composed of mature adipocytes

Explanation - Q: 5.3 Close

The correct answer is B. The lesion is most likely a rheumatoid nodule,
which is histologically composed of an area of central fibrinoid necrosis,
surrounded by palisading epithelioid histiocytes admixed with plasma cells
and lymphocytes. None of the lesions seen in the alternative choices have
increased incidence in rheumatoid arthritis.
Amorphous calcified material surrounded by fibrotic subcutaneous tissue
(choice A) would be typical of dystrophic calcification.
Cystic space filled with keratin and lined by squamous epithelium (choice C)
would be typical of an epidermal inclusion cyst.
Interwoven neural fibers (choice D) would be typical of a neurofibroma.
Membrane bound mass composed of mature adipocytes (choice E) would
be typical of a lipoma.


Question 4 of 6
The patient is prescribed naproxen for treatment of her rheumatoid arthritis.
Which of the following occurs with naproxen administration?
/ A. Binding to opioid receptors
/ B. BIocking voltage-dependent sodium channels
/ C. Changes in platelet function
/ D. Production of prostaglandins
/ E. Stimulation of cyclooxygenase

Explanation - Q: 5.4 Close

The correct answer is C. Naproxen is a nonsteroidal anti-inflammatory drug
(NSAID) that exerts its action by inhibiting cyclooxygenase (compare with
choice E) leading to decreased production of prostaglandins (compare with
choice D). This agent is indicated for treatment of mild-moderate pain (acute
and chronic) caused by inflammation. Naproxen affects platelet function,
inhibiting platelet aggregation.
The most common side effects of naproxen are gastrointestinal, such as
nausea, vomiting, diarrhea, abdominal distress, flatulence, anorexia, and
possible bleeding or ulceration. When this medication is administered with
aspirin, and alcohol is ingested, there can be a dramatic increase in
gastrointestinal side effects. Since naproxen can be directly irritating to the
stomach mucosa, the concomitant administration of food or antacids may be
necessary. There is also a risk of renal damage with naproxen, as with many
of the NSAIDs, particularly in those with preexisting renal disease.
Codeine and morphine are examples of drugs that bind to opioid receptors
(choice A).
Local anesthetics (e.g., procaine, bupivacaine, lidocaine) prevent pain by
blocking nerve conduction. They block voltage-dependent sodium channels
(choice B), thereby blocking depolarization and action potential conduction.



Question 5 of 6
If this patient also has a past medical history significant for psoriasis, which of the
following medications is contraindicated for the treatment of
rheumatoid arthritis, since it is likely to precipitate an attack of psoriasis?
/ A. Codeine
/ B. Hydroxychloroquine
/ C. Ibuprofen
/ D. Methotrexate
/ E. Penicillamine


Explanation - Q: 5.5 Close

The correct answer is B. Hydroxychloroquine is an antimalarial agent that is
used in the treatment of moderate to severe RA that is unresponsive to
conventional therapy. The clinical benefits are generally seen 4-12 weeks
after therapy is initiated; up to 6 months may be required to achieve a clinical
response. The drug concentrates in the liver, spleen, kidney, heart, lung and
brain. Warnings and precautions include precipitation of a psoriasis attack in
patients with psoriasis, ophthalmic effects (irreversible retinal damage),
muscle weakness, bleaching of hair, alopecia, mucosal pigmentation, and
skin eruptions. The drug should not be used during pregnancy or in infants.
Codeine (choice A) is an opioid analgesic that exerts its action at mu,
kappa, and sigma opioid receptors. Mu receptors mediate analgesia,
euphoria, and respiratory depression; kappa receptors mediate sedation and
miosis, and sigma receptors are associated with dysphoria and
hallucinations.
Ibuprofen (choice C) is a phenylpropionic acid derivative with analgesic,
antipyretic, and anti-inflammatory effects. The anti-inflammatory and
analgesic effects are due to inhibition of prostaglandin synthetase enzyme
complex. It is indicated for treatment of mild to moderate pain, fever (except
in children with chickenpox or flu symptoms) and inflammatory conditions
(RA, rheumatic fever and osteoarthritis). The most common adverse effects
are gastrointestinal, such as nausea, vomiting, diarrhea, abdominal distress,
flatulence, anorexia, and possible bleeding and ulceration.
Methotrexate (choice D) is indicated for the treatment of severe, active,
classical or definite RA in adults who have an insufficient response with
conventional therapies. It is also used as an antineoplastic therapy. The most
common adverse reactions are nausea, vomiting, stomatitis, alopecia,
leukopenia, and pancytopenia.
Penicillamine (choice E) acts by suppression of RA disease activity. It is
indicated for severe active RA that is unresponsive to conventional therapy.



Question 6 of 6
Several years later, the patient develops dry eyes and mouth. Buccal biopsy
demonstrates a lymphocytic infiltrate around fibrotic minor salivary
gland tissue. This patient has most likely developed which of the following other
diseases?
/ A. Felty syndrome
/ B. Lyme disease
/ C. Reiter syndrome
/ D. Sjgren syndrome
/ E. Still disease

Explanation - Q: 5.6 Close

The correct answer is D. The patient has developed Sjgren syndrome,
which is characterized by an autoimmune attack on tear gland and salivary
gland tissue. Rheumatoid arthritis may have multiple extra-articular
manifestations, which may include rheumatoid nodules, vasculitis,
lymphadenopathy, Sjgren syndrome, Felty syndrome (rheumatoid arthritis
with neutropenia and splenomegaly, choice A), and episcleritis.
Lyme disease (choice B) is due to a spirochete infection and produces an
arthritis that can mimic rheumatoid arthritis.
Reiter syndrome (choice C) is associated with a reactive arthritis that follows
an antecedent infection such as urethritis or diarrhea.
Still disease (choice E) is a form of rheumatoid arthritis that lacks
rheumatoid factor.


A 68-year-old man presents to his primary care physician with increasing
deformity and disability in his right hand. He reports a five-year
history of symptoms, beginning with nodules over the palmar aspect of his ring
and small fingers of his hand. These have become larger and
more painfuI. He now comes in because he feels "cords" in these two fingers,
and he is unable to fully extend the fingers. He reports that his
father had a similar problem in the past. On physical examination, there are
nodules over the metacarpals of his ring and small fingers.
Additionally, there is a flexion contracture of 30 degrees, i.e., the range of motion
at the metacarpophalangeal joint of both these fingers is
from 30 to 85 degrees.
Question 1 of 3
Which of the following is the most likely diagnosis?
/ A. Dupuytren disease
/ B. Ledderhose disease
/ C. Osgood-Schlatter disease
/ D. Peyronie disease
/ E. Sever disease


Explanation - Q: 6.1 Close

The correct answer is A. This patient has classic symptoms of Dupuytren
disease, which is a palmar fibromatosis. The palmar fascia develops nodules
and cords. It is seen in 1-2% of the population, with 50% of patients having
bilateral disease. 10% of patients have a similar condition in other locations.
It has variable familial inheritance, is seen most commonly in people of Celtic
origin, and males are affected twice as commonly as females.
Ledderhose disease (choice B) is a similar condition affecting the plantar
surfaces of the feet (plantar fibromatosis).
Osgood-Schlatter disease (choice C) is a osteochondrosis that occurs at the
tibial tuberosity in children 11-15 years of age. It is a traction apophysitis
from the stress of the extensor mechanism in a growing child. It is usually
self-limited.
Peyronie disease (choice D) is a similar condition to Dupuytren disease, but
affects the penis.
Sever disease (choice E) is another osteochondrosis that affects the
apophysis of the calcaneus in children 9-11 years old.



Question 2 of 3
What anatomic structure is most likely involved is this disease?
/ A. Carpal tunnel
/ B. FIexor digitorum profundus
/ C. FIexor digitorum superficialis
/ D. Median nerve
/ E. Palmar fascia

Explanation - Q: 6.2 Close

The correct answer is E. The palmar fascia is the anatomic structure
involved in Dupuytren disease.
The carpal tunnel (choice A) is a space through which the median nerve and
flexor tendons run. It is not involved in Dupuytren disease.
The flexor digitorum profundus (choice B) and superficialis (choice C) are
flexors of the fingers, and are not involved in Dupuytren disease. The flexion
contracture is a result of the palmar fibrosis, not the flexor tendons.
The median nerve (choice D) is involved in carpal tunnel syndrome. It
supplies sensation to the radial three and a half fingers, and provides motor
innervation for opposition of the thumb.


*** Commercial version is infinite. Order at http://www.structurise.com/kleptomania ***
Question 3 of 3
After some questioning, this man reports that he also has a painful band on the
dorsal aspect of his penis, but he was too embarrassed to tell
anyone about this. Which of the following is the most likely diagnosis?
/ A. Dupuytren disease
/ B. Ledderhose disease
/ C. Osgood-Schlatter disease
/ D. Peyronie disease
/ E. Sever disease

Explanation - Q: 6.3 Close

The correct answer is D. Peyronie disease is a similar condition to
Dupuytren disease, which affects the penis. It is characterized by fibrosis of
the sinusoidal spaces of the corpora cavernosa.
This patient has classic symptoms of Dupuytren disease (choice A) which is
a palmar fibromatosis. The palmar fascia develops nodules and cords It is
seen in 1-2% of the population with 50% of patients having bilateral disease.
10% of patients have a similar condition in other locations. It has variable
familial inheritance, is seen most commonly in people of Celtic origin, and
males are affected twice as commonly as females.
Ledderhose disease (choice B) is a similar condition affecting the plantar
surfaces of the feet.
Osgood-Schlatter disease (choice C) is an osteochondrosis that occurs at
the tibial tuberosity in children 11-15 years of age. It is a traction apophysitis
from the stress of the extensor mechanism in a growing child. It is usually
self-limited.
Sever disease (choice E) is another osteochondrosis that affects the
apophysis of the calcaneus in children 9-11 years old.

Approximately 4 days after a romantic encounter with a new partner at a ski
lodge, a 32-year-old man develops mild urethral discomfort
followed, a few hours later, by painful urination accompanied by a purulent
discharge. He consults a physician the following day. Physical
examination demonstrates red, swollen urethral lips and a purulent, yellowish-
green urethral discharge. Gram's stain of the discharge
performed in the doctor's office demonstrates neutrophils packed with gram-
negative kidney bean-shaped diplococci.




Explanation - Q: 1.1 Close

The correct answer is B. The only medically important gram-negative cocci
are in the genera Neisseria and Moraxella. Neisseria meningitidis is the
etiologic agent of meningococcal meningitis and meningococcemia and
Neisseria gonorrhoeae is the etiologic agent of gonorrhea. Moraxella
catarrhalis is a cause of pharyngitis, otitis media, and sinusitis in children.
Chlamydia(choice A) is an intracellular organism that would not be visible
with the Gram's stain.
Staphylococcus(choice C) and Streptococcus(choice D) are gram-positive
cocci and would not be associated with this set of symptoms.
Treponema(choice E) is a spirochete that would not be visible on Gram's
stain.




Explanation - Q: 1.2 Close

The correct answer is C. This patient has a classic presentation for
gonorrhea. In men, the infection typically produces urethritis with a purulent
urethral discharge 2 to 14 days after sexual contact. In women, gonorrhea
may be nearly or completely asymptomatic, or may cause dysuria, increased
frequency of urination, and vaginal discharge. Rectal infection in men or
women may be asymptomatic or cause perianal discomfort and rectal
discharge.
AIDS (choice A) is a viral infection that would not cause these symptoms,
and would not be diagnosed with a Gram's stain.
Chlamydia urethritis (choice B) would not cause a purulent discharge
because Chlamydia trachomatis is an intracellular parasite of mucosal cells,
and it would not be visible on a Gram's stain.
Lymphogranuloma venereum (choice D) is also caused by Chlamydia
trachomatis, but by different serotypes than those that typically cause
chlamydial STD in the U.S. Because Chlamydia is an intracellular pathogen,
it would not cause a purulent discharge.
Syphilis (choice E) characteristically causes production of a chancre at the
site of inoculation rather than an urethritis, and the causative organism is a
narrow spirochete that can only be visualized by darkfield microscopy or
direct fluorescence.



Question 3 of 5
Following culture of the causative organism, which of the following biochemical
attributes would be most definitive in determining the causative
species?
/ A. Catalase production
/ B. Nitrate reduction
/ C. Production of acid from glucose but no other sugars
/ D. Production of acid from maltose
/ E. Production of cytochrome c oxidase

Explanation - Q: 1.3 Close

The correct answer is C. Neisseria gonorrhoeae is distinguished from all
other members of the genus by its utilization of glucose, but not maltose.
The entire genus Neisseria produces catalase (choice A); it is therefore not
a characteristic that distinguishes the species, but only the genus.
Nitrate reduction (choice B) is not a useful criterion, as it is absent in both
Neisseria meningitidis and N. gonorrhoeae.
Production of acid from maltose (choice D) is an attribute of Neisseria
meningitidis, but not of N. gonorrhoeae.
Production of cytochrome c oxidase (choice E) is an attribute of the entire
genus Neisseria, as well as several gram-negative bacilli, such as
Campylobacter, Helicobacter, and Vibrio.


Question 4 of 5
Genetic coding found for resistance to beta-Iactam antibiotics resides in which of
the following locations?
/ A. Chromosome
/ B. Conjugative plasmids
/ C. Cytoplasmic membrane
/ D. Non-conjugative plasmids
/ E. Phage genome



Explanation - Q: 1.4 Close

The correct answer is D. In Neisseria gonorrhoeae, the plasmids that code
for the production of beta lactamases possess the ori T, but not the tra
operon. Other plasmids in the cell mediate production of the conjugal bridge
by their possession of the tra operon, and the plasmids containing the drug
resistance genes are mobilized across the preformed conjugal bridge along
with the fertility factor plasmid.
The chromosome (choice A) is the site of coding of genes for penicillin-
binding proteins, but most enzyme-mediated drug resistances are coded for
on plasmids.
Conjugative plasmids (choice B) is not correct. Although in most cases of
enzyme-mediated drug resistance, the coding exists on plasmids, in the
specific case of Neisseria gonorrhoeae, the genes exist on plasmids that do
not have the tra operon and are thus, non-conjugative plasmids.
The cytoplasmic membrane (choice C) is not correct because although this
is the location of penicillin-binding proteins, and mutations in these proteins
are responsible for low-level penicillin resistance in some species, there is no
"genetic coding" i.e., DNA in this location. Furthermore, this is not the
mechanism of drug resistance in Neisseria at the present time.
The phage genome (choice E) is not correct because although phage may
impart other important pathogenic features to bacteria, they are not generally
associated with drug resistances.



Question 5 of 5
If this man had not sought treatment and had infected his pregnant wife, his baby
would be at most risk for developing which of the following
unless prophylactic measures were taken?
/ A. Condylomata lata
/ B. Ecthyma gangrenosum
/ C. Granuloma inguinale
/ D. Granulomatosis infantiseptica
/ E. Ophthalmia neonatorum

Explanation - Q: 1.5 Close

The correct answer is E. The Centers for Disease Control (CDC)
recommends routine use of 1% silver nitrate, erythromycin, or tetracycline
ophthalmic ointments or drops instilled in each eye after the delivery of an
infant to prevent ophthalmia, which might be caused by Neisseria,
Chlamydia, or Treponema.
Condylomata lata (choice A) is a manifestation of the secondary stage of
syphilis, when flat, wart like growths appear on the mucosa.
Ecthyma gangrenosum (choice B) is the characteristic skin lesion
associated with Pseudomonas infections.
Granuloma inguinale (choice C) is a disease of the genitalia caused by
Calymmatobacterium (Klebsiella) granulomatis.
Granulomatosis infantiseptica (choice D) is an infection of the fetus in utero
with Listeria monocytogenes.



A 19-year-old man presents to his college health clinic complaining of a painless
sore on his penis. The patient states that 4 weeks prior to
presentation, he had unprotected sexual intercourse with a new partner. About
two weeks after this encounter, he developed a red spot on the
glans of his penis. It has always remained small and now has created a "crater"
on the tip of his penis. He denies dysuria, fevers, chills, meatal
discharge, or any similar previous episodes. Three months ago he had an HIV
test, which was negative. Upon examination, the physician
elicits bilateral inguinal adenopathy that is firm but not tender to palpation. There
is no discharge elicited from the urethral meatus. On the right
side of the glans penis is a small chancre with indurated edges. The base of the
lesion is clean, and no fluid can be expressed upon applying
pressure. Rectal examination shows normal sphincter tone with a firm,
appropriately sized, non-tender prostate. Urine dipstick is negative for
any sign of infection.

Explanation - Q: 2.1 Close

The correct answer is E. This patient has primary syphilis. Syphilis, a
sexually transmitted disease, may present 2-4 weeks after exposure and
begins as a hyperemic or erythematous spot. This painless papule or
pustule, develops on the glans, corona, foreskin, shaft, suprapubic area, or
scrotum. It may break down to form an indurated, punched-out lesion. The
syphilitic (hard) chancre is relatively deep, has indurated edges and a clean
base, and is not tender on pressure. The lesion may be so small and
transient that it is missed. Without treatment, the lesion will heal
spontaneously and slowly. Inguinal adenopathy may be tender or nontender
and is typically firm and "rubbery."
The ulcer associated with chancroid (choice A) is painful, and is deep, with
an undermined border and a friable base that bleeds easily. The adenopathy
is painful and, with chronic infection, may cause lymphatic obstruction.
Incubation is one to four days.
Chlamydia(choice B) causes urethritis, and not genital ulcers. It typically
presents 7-21 days after exposure, with dysuria and mild-to-moderate whitish
or clear urethral discharge.
Genital herpes (choice C) typically presents as penile lesions of grouped
vesicles on an erythematous base that do not follow a neural distribution.
The lesions are tender to touch and the associated adenopathy is bilateral,
mildly tender, non-fixed, and slightly firm. The primary episode is more
severe than recurrent attacks and the incubation period is 2 -10 days.
Granuloma inguinale (choice E) initially presents as a papule that ultimately
forms a non-tender, erythematous ulcer with hemorrhagic secretions. There
is inguinal swelling or pseudobubo, which is actually a subcutaneous
granulomatous process rather than a true lymphadenopathy. Untreated, it
enlarges by direct extension, or may erode through the skin.





Explanation - Q: 2.2 Close

The correct answer is E. Syphilis is caused by the spirochete Treponema
pallidum. It gains access through intact or abraded skin or mucous
membranes.
Calymmatobacterium granulomatis(choice A) is the causative agent of
granuloma inguinale.
Chlamydia trachomatis(choice B) is the bacteria responsible for non-
gonococcal urethritis.
Chancroid is caused by Haemophilus ducreyi(choice C).
The herpes simplex virus (choice D) is associated with genital herpes.
Eighty percent of the genital lesions are caused by the type II virus. These
lesions are painful, and associated with systemic symptoms such as malaise,
anorexia, and fever.


Question 3 of 6
The treatment of choice for this infection is an intramuscular injection of penicillin
G. Which of the following antibiotics has antimicrobial activity
that is similar to that of the penicillins?
/ A. Aminoglycosides
/ B. Cephalosporins
/ C. Erythromycin
/ D. FIuoroquinolones
/ E. Sulfonamides


Explanation - Q: 2.3 Close

The correct answer is B. The penicillins are classified as beta-lactam drugs.
This is because of their core structure, which contains a thiazolidine ring
attached to a beta-lactam ring that carries a secondary amino group. The
mechanism of action of these agents involves damage to the bacterial cell
wall. The steps involved in this are (1) attachment to specific penicillin-
binding proteins that serve as drug receptors on bacteria, (2) inhibition of cell
wall synthesis by blocking transpeptidation of peptidoglycan, and (3)
activation of autolytic enzymes in the cell wall, which results in lesions that
cause bacterial death. The cephalosporins also have a similar basic structure
that incorporates the beta-lactam ring and therefore, their mechanism of
action is similar to the penicillins.
The aminoglycosides (choice A) are bactericidal by virtue of irreversible
inhibition of protein synthesis. They penetrate the bacteria's cell wall and
then bind to the 30S subunit of the bacterial ribosome to inhibit ribosomal
protein synthesis.
Erythromycin (choice C) is both inhibitory and bactericidal. It works by
binding to the 50S subunit (specifically onto the 23S rRNA) of the ribosome.
Protein synthesis is inhibited as aminoacyl translocation reactions and the
formation of initiation complexes are blocked.
The quinolones and fluoroquinolones (choice D) are potent inhibitors of
nucleic acid synthesis. They block the action of DNA gyrase (topoisomerase
II), the enzyme responsible for packing and unpacking supercoiled DNA.
Sulfonamides (choice E) are bacteriostatic, and work by competitive
inhibition. These medications compete with p-aminobenzoic acid (PABA) for
the enzyme dihydropteroate synthetase to block a step in the pathway of the
formation of purines, and therefore, ultimately, nucleic acids.



Question 4 of 6
If the patient were not treated, and instead developed advanced disease, which
of the following most accurately describes the likely findings
on physical examination?
/ A. Normal pupils
/ B. The pupils accommodate but do not react to light
/ C. The pupils accommodate and react to light
/ D. The pupils do not accommodate and do not react to light
/ E. The pupils do not accommodate but react to light


Explanation - Q: 2.4 Close

The correct answer is B. Tertiary syphilis can affect the central nervous
system (this is termed neurosyphilis) and spirochetes may be found in the
cerebrospinal fluid. This may lead to tabes dorsalis, which is a neurological
deficit caused by the destruction of the dorsal columns and dorsal roots of
the spinal cord. Patients may also develop general paresis, due to invasion
and destruction of brain parenchyma. Frequently, those patients with
neurosyphilis will exhibit Argyll-Robertson pupils. This pathological finding
consists of pupils that will accommodate but have an absent pupillary reflex
to light. With accommodation, the eyes move medially and the pupils
constrict when focusing on a close object. However, the pupils fail to
constrict in response to a bright light, i.e., the pupillary reflex is absent.



Question 5 of 6
If his disease were left untreated, which of the following regions of the spinal cord
would most likely be affected in this patient?
/ A. Anterior white commissure and ventral horns
/ B. Dorsal columns and dorsal roots
/ C. Dorsal columns, Iateral corticospinal tracts, spinocerebellar tracts
/ D. Lateral corticospinal tracts and ventral horns
/ E. Ventral horns

Explanation - Q: 2.5 Close

The correct answer is B. Tabes dorsalis, seen in patients with
neurosyphilis, is a slowly progressive degenerative disease that involves the
dorsal columns and dorsal roots. Demyelination of the dorsal columns leads
to a loss of tactile discrimination and position and vibration sensations. Pain
and paresthesias can occur with irritative involvement of the dorsal roots.
These patients can present with a positive Romberg sign.
Lesions of the anterior white commissure and ventral horns (choice A)
occurs in syringomyelia.
Lesions of the dorsal columns, lateral corticospinal tracts, and
spinocerebellar tracts (choice C) occurs with subacute combined
degeneration and Friedreich's ataxia.
Lesions of the lateral corticospinal tracts and ventral horns (choice D) occurs
in amyotrophic lateral sclerosis (ALS).
Lesions of the ventral horns (choice E) occur in poliomyelitis.



Explanation - Q: 2.6 Close

The correct answer is C. In order to evaluate for the presence of syphilis,
two strategies may be employed. One is the microscopic visualization of the
spirochete from the skin lesion and the other is a serologic (blood) test to
identify evidence of the body's reaction to syphilis. Fluorescent treponemal
antibody-absorption is a treponemal-specific serological test that will remain
positive for life. With this test, the patient's serum is filtered to separate out
any treponemal antibodies that are not specific to T. pallidum. The patient's
serum is poured onto a slide covered with T. pallidum antigens, then the
slide is washed, leaving the anti-treponemal antibody-antigen complexes on
the slide. Fluorescent antibodies that react with human immunoglobulins are
then added, and bind to the antibody-antigen complexes on the slide. The
slide is then examined with an ultraviolet microscope, any fluorescence
indicates a positive test.
Darkfield microscopy (choice A) and the direct fluorescent antibody T.
pallidum test, or DFA-TP (choice B) are both techniques that utilize fluid
from either the chancre or the maculopapular rash (if secondary syphilis).
Darkfield microscopy uses direct visualization of the organism. DFA-TP
consists of mixing anti-T. pallidum antibodies conjugated to fluorescein with a
sample of fluid. If the sample contains the spirochete, the antibodies will bind
and allow visualization of the spirochete with an ultraviolet microscope. Once
the lesion heals, there will be no spirochetes to see.
VDRL (choice E) and RPR (choice D) are non-treponemal serological tests
that are used for screening and to monitor the success of treatment. They
employ an antigen that is a mixture of cardiolipin, cholesterol, and lecithin.
Their titers rise following infection, and parallel disease activity. As the
disease improves, RPR and VDRL levels will decrease, therefore, titers are
followed to monitor treatment. Nontreponemal serological tests are
nonspecific, and other conditions may give a positive result.


A 27 year-old man visits his primary care physician because of testicular pain. He
states that over the last 3 weeks he has been feeling a
vague and heavy sensation in his right testicle. He denies any dysuria, urethral
discharge, testicular trauma, prior testicular surgery, fevers, or
chills. His genital examination is normal except for scrotal examination. His right
testicle is enlarged with an irregular, non-tender mass that
appears to be arising from and obliterating the normal testicular architecture. The
mass is not reducible, does not transilluminate, and does not
change with Valsalva maneuver. The spermatic cord can be palpated superiorly
to the mass and is normaI. Urinalysis is normaI.
Question 1 of 5
Which of the following is the most likely diagnosis?
/ A. Acute epididymitis
/ B. Hydrocele
/ C. Inguinal hernia
/ D. Testicular cancer
/ E. Varicocele


Explanation - Q: 3.1 Close

The correct answer is D. This patient has the classic signs and symptoms
of testicular cancer. The most common symptom of testicular cancer is
painless enlargement of the testis. Enlargement is usually gradual, and a
sensation of testicular heaviness is not unusual. The mass is typically firm
and nontender, and the epididymis should be easily separable from it.
Acute epididymitis (choice A) is an infection of the epididymis acquired by
the retrograde spread of organisms down the vas from the urethra. Patients
present with heaviness and a dull, aching discomfort in the affected
hemiscrotum that can radiate up to the ipsilateral flank. On examination, the
epididymis will be markedly swollen and exquisitely tender to touch,
eventually becoming a warm, red, enlarged, scrotal mass. Fevers and chills
may develop.
A hydrocele (choice B) is a fluid collection within the tunica vaginalis
surrounding the testis. It presents as a painless swelling of the scrotum that
transilluminates. This transillumination is often necessary to differentiate a
hydrocele from a testicular carcinoma.
An inguinal hernia (choice C) is the result of a weakness in the floor of the
inguinal canal. It may be an incidental finding on physical examination or
may present with pain in the groin with Valsalva maneuver. If a scrotal mass
is present, it may contain a loop of bowel protruding through the weakness in
the inguinal canal. A mass associated with an inguinal hernia can usually be
differentiated from the testicle itself.
A varicocele (choice E) is an abnormal dilatation of the veins of the
pampiniform plexus and the internal spermatic vein of the spermatic cord.
Left-sided varicoceles are most common, occurring in approximately 15% of
normal adult males. The dilated veins are best palpated with the patient in
the standing position and aided by a Valsalva maneuver. The vessels are
palpated superior to the testicle and are described as feeling like "a bag of
worms."


Question 2 of 5
Prior to removal of the testicle (radical orchiectomy), what two blood tests must
be performed on this patient?
/ A. AIdosterone and beta hCG (human chorionic gonadotropin)
/ B. AIpha-fetoprotein (AFP) and beta hCG
/ C. Complete blood count (CBC) and calcium
/ D. Prostate-specific antigen and alpha-fetoprotein
/ E. Testosterone and alpha-fetoprotein

Explanation - Q: 3.2 Close

The correct answer is B. Many germ cell tumors produce specific oncofetal
protein markers, either AFP or hCG, that can be detected in patients' serum
or tissue. Ninety percent of patients with nonseminomatous testis tumors will
have elevations of one or both markers and 5-10% with pure seminomas will
demonstrate elevations of hCG only. The amount of tumor burden is
proportional to the degree of marker elevation. It is important to draw these
levels prior to surgery. After removal, the tumor markers are monitored to
determine if there is residual disease (i.e., progression to retroperitoneal
lymph nodes).
Aldosterone (choice A) is produced within the outer cortical zona
glomerulosa of the adrenal cortex. Production is under the influence of
angiotensin II and the renin-angiotensin system. Aldosterone acts at the
distal tubule of the nephrons to cause sodium retention and potassium
secretion. It plays no role in the evaluation or monitoring of disease
progression for testicular cancer.
A complete blood count (choice C) measures the patient's white blood cells,
hemoglobin, hematocrit, and platelet counts. These values are not normally
altered in patients with testicular cancer. Calcium levels may be abnormal in
patients with malignancy and some paraneoplastic syndromes. Testicular
tumors are not generally associated with paraneoplastic syndromes.
Prostate-specific antigen (PSA) (choice D) is a tumor marker for prostate
cancer. It is secreted by the prostate and may be elevated in patients with
prostate cancer. It has no role in the work-up of testicular cancer.
Testosterone levels (choice E) are not altered with testicular cancer. There
is no need to measure this substance in these patients.



Question 3 of 5
Radical orchiectomy requires an inguinal incision and removal of the testicle and
spermatic cord. Which of the following nerves runs parallel to
the spermatic cord within the inguinal canaI?
/ A. Femoral
/ B. IIioinguinal
/ C. Lateral femoral cutaneous
/ D. Obturator
/ E. Pudendal

Explanation - Q: 3.3 Close

The correct answer is B. The inguinal canal is an oblique passage through
the inferior part of the anterior abdominal wall. The chief protection of the
inguinal canal is muscular. Its main constituent is the spermatic cord in the
male and the round ligament of the uterus in females. It contains the
ilioinguinal nerve in both sexes. This nerve is derived from the L1 segment,
enters the abdomen posterior to the medial arcuate ligament, and passes
inferolaterally, anterior to the quadratus lumborum muscle. The nerve pierces
the transversus abdominis muscle near the superior iliac spine, travels within
the inguinal canal, and passes through the superficial (external) inguinal ring
to supply the skin of the groin and scrotum or labium majora.
The femoral nerve (choice A) is the largest branch of the lumbar plexus. It
forms in the abdomen within the substance of the psoas major muscle and
descends posterolaterally through the pelvis to the midpoint of the inguinal
ligament. It then passes lateral to the femoral vessels, outside the femoral
sheath enclosing them. At no point in its course does it enter the inguinal
canal.
The lateral femoral cutaneous nerve (choice C) originates from L2 and L3
and is a direct branch of the lumbar plexus. It enters the thigh deep to the
lateral end of the inguinal ligament, near the anterior superior iliac spine. It
supplies the skin on the anterior and lateral aspects of the thigh.
The obturator nerve (choice D), originating from L2, L3, and L4 of the lumbar
plexus, is the nerve of the adductor muscles of the thigh. This nerve
descends through the psoas major muscle, leaving its medial border at the
brim of the pelvis. It pierces the psoas fascia, crosses the sacroiliac joint,
passes lateral to the internal iliac vessels and ureter, and enters the pelvis
minor. It leaves the pelvis through the obturator foramen and enters the
thigh. The obturator nerve supplies motor innervation to the obturator
externus, which is responsible for laterally rotating the thigh. This nerve also
provides a small cutaneous branch, which is responsible for sensation to the
medial aspect of the thigh.
The pudendal nerve (choice E) arises from the sacral plexus by separate
branches of the ventral rami of S2, S3, and S4. It accompanies the internal
pudendal artery and leaves the pelvis between the piriformis and coccygeus
muscles. It hooks around the sacrospinous ligament to enter the perineum
through the lesser sciatic foramen. Here, it supplies the muscles of the
perineum, including the external anal sphincter, and ends as the dorsal nerve
of the penis or clitoris.


Question 4 of 5
Patients with advanced stages of this condition may be treated with bleomycin.
Which organ system toxicity is unique to bleomycin?
/ A. Bone marrow
/ B. Cardiac
/ C. Neurologic
/ D. Pulmonary
/ E. Renal

Explanation - Q: 3.4 Close

The correct answer is D. Bleomycin, a chemotherapeutic agent, works by
binding to and then breaking DNA strands. Toxicities include pneumonitis
and pulmonary fibrosis. Patients should have pulmonary function testing
(PFT) prior to the administration of this medication.
Bone marrow toxicity (choice A), i.e., myelosuppression, is seen with many
different chemotherapeutic agents and is not unique to bleomycin.
Cardiac toxicity (choice B) is a rare complication of chemotherapy. It is
associated with the administration of doxorubicin.
Toxicity to the nervous system (choice C) is not usually encountered with
bleomycin.
Platinum-based chemotherapeutic regimens are most effective against
testicular tumors. Cisplatin is a known nephrotoxic agent and its use is
limited in patients with renal insufficiency. Carboplatin is another platinum-
based medication not associated with renal toxicity (choice E).



Question 5 of 5
A biopsy taken from an infant with this condition would most likely reveal which of
the following?
/ A. Lymphoma
/ B. Mixed cell type
/ C. Seminoma
/ D. Teratoma
/ E. Yolk sac tumor

Explanation - Q: 3.5 Close

The correct answer is E. Testicular tumors are classified as either germ cell
or non-germ cell tumors. Germ cell tumors comprise 95% of all testicular
tumors. The two major divisions are seminomas and nonseminomas. The
nonseminoma division includes yolk sac, embryonal carcinoma, teratoma,
choriocarcinoma, and mixed tumors. Yolk sac tumors, also called
endodermal sinus tumors or orchioblastomas, are nonseminomatous germ
cell tumors that have a peak incidence in the infant and childhood age group.
Grossly, these tumors appear yellow. Histologically, they contain Schiller-
Duval bodies that resemble 1-2 week old embryos, i.e., a cavity surrounded
by syncytiotrophoblasts and cytotrophoblasts. Yolk sac tumors metastasize
hematogenously, compared to other germ cell tumors, which spread via the
lymphatics.
Lymphoma (choice A) is the most common metastatic (secondary) tumor of
the testis and the most common testis tumor in men > 50 years old.
Mixed cell type (choice B) refers to tumors that contain a combination of
nonseminoma and/or seminoma. These account for up to 40% of testicular
germ cell tumors. If a tumor contains both seminoma and nonseminoma, it is
treated as a nonseminoma.
Seminoma (choice C) accounts for approximately 35% of germ cell tumors.
There are three histological subtypes, classic seminoma, anaplastic
seminoma, and spermatocytic seminoma. The classic variant accounts for
85% of all seminomas and is most common in the fourth decade of life.
Teratomas (choice D) contain derivatives of all three cell layers: ectoderm,
endoderm, and mesoderm. Microscopically, they appear as clear or
mucinous cystic areas interspersed with solid tissue including bone, muscle,
or cartilage. The peak incidence is in the 25-35 year old age group.




An 18-year-old girl is involved in a traffic accident resulting in head trauma and
intracerebral hemorrhage. She undergoes extensive
rehabilitation and her neurological and mental status improve. At present, she
presents with complaints of polydipsia, polyuria and nocturia,
and frequent daytime napping. On examination, no signs or symptoms of
significant dehydration are found. Her glucose tolerance test and
urine analysis are normaI. Serum osmolality is 316 mOsmoI/kg, and her urine
osmolality is 136 mOsmoI/kg.
Question 1 of 5
Which of the following is the most likely cause of polyuria in this patient?
/ A. ADH deficiency
/ B. Compulsive water drinking (psychogenic polydipsia)
/ C. Iatrogenic polydipsia
/ D. Increased metabolism of ADH
/ E. Lithium therapy

Explanation - Q: 1.1 Close

The correct answer is A. Polyuria is defined as urine production more than
3 L/day. There are two main causes of polyuria: water diuresis (osmolarity
less than 250 mosmol/L) and osmotic (solute) diuresis (more than 300
mosmol/L). Water diuresis can be caused by diabetes insipidus (DI) or
primary polydipsia with an increased water intake and consequent polyuria.
Decreased secretion (central diabetes insipidus) or action (nephrogenic DI)
of arginine vasopressin (AVP, ADH) is characterized by the production of
abnormally large volumes of urine (more than 50 mL/kg/d) with low
osmolarity (less than 200 mosmol/L). The polyuria leads to the symptoms of
urinary frequency and nocturia, which may disturb sleep, causing daytime
somnolence. Central DI can be primary (familial, idiopathic) or secondary
(posttraumatic, infection, tumors, sarcoidosis, histiocytosis X). ADH is
synthesized within supraoptic and paraventricular nuclei of the
hypothalamus, but the posterior pituitary lobe is the major site of AVP
storage and release. Injury may damage the axons of the pituitary stalk and
impair the transport and release of AVP. Administration of ADH in normal
people and those with primary polydipsia, does not increase urine osmolality
more than 9%. In central DI, the rise in urine osmolality is greater than 9%. In
nephrogenic DI, there is no change in urine osmolality with either
dehydration or with ADH administration. Plasma ADH is low in central DI. In
primary polydipsia, plasma sodium is low-normal, while in DI, it is normal to
high.
Emotionally disturbed patients can drink up to 6-7 L of fluid per day
(psychogenic, primary polydipsia, choice B). They usually do not have
nocturia. The polydipsia leads to the suppression of endogenous ADH and
resultant polyuria. Patients usually remain normonatremic, despite the large
fluid intake, although plasma osmolality may be normal or slightly reduced.
Psychotherapy is required for most patients with compulsive water drinking.
Thioridazine and lithium should be avoided since they cause polyuria.
Iatrogenic polydipsia (choice C) results from recommendations of popular
media or some health professionals to increase fluid intake for its presumed
preventive or therapeutic benefits for other diseases. This form can often be
corrected by patient counseling.
Vasopressinase, the enzyme that metabolizes AVP, increases during
pregnancy from the 4th to the 38th week by 1000-fold. As a result,
vasopressinase-induced DI (choice D) may be seen in the late pregnancy or
puerperium. A circulating enzyme destroys native AVP, but synthetic
(DDAVP) is unaffected, and therefore may be used to control the symptoms.
This form is usually referred to as gestational DI.
Vasopressin-resistant (nephrogenic) DI may be caused by the congenitally
defective expression of V2 receptors or the aquaporin-2 (AQP2) protein, or
may be acquired in association with certain diseases (pyelonephritis,
myeloma, Sjgren syndrome, sickle cell anemia) and/or drug treatment
(demeclocycline, lithium, methicillin). In this case, the polyuria is
unresponsive to vasopressin, and the hormone secretion is normal. Lithium-
induced DI (choice E) may respond to amiloride even if lithium treatment is
continued.



Question 2 of 5
Which of the following would produce polyuria via a mechanism of action most
similar to the mechanism of the disease process in this
patient?
/ A. Ethanol
/ B. Hyperosmolality
/ C. Hypotension
/ D. Lithium
/ E. Nausea

Explanation - Q: 1.2 Close

The correct answer is A. Ethanol inhibits vasopressin release from the
posterior pituitary, thereby inducing diuresis. Ethanol also inhibits oxytocin
secretion and has tocolytic activity due to both direct action on uterine
muscle and inhibition of oxytocin release. Phenytoin, haloperidol, clonidine,
and alpha adrenergic agonists also inhibit vasopressin secretion.
When plasma osmolality rises above the "threshold" of 280 mOsmol/kg,
specialized cells known as osmoreceptors, located in the anteromedial
hypothalamus near the supraoptic nucleus, react and stimulate AVP
secretion, which leads to antidiuresis that is maximal when plasma osmolality
reaches 295 mOsmol/kg (choice B).
When blood pressure falls (choice C), stretch receptors in the aorta and
carotid arteries are stimulated. They project via the vagus and
glossopharyngeal nerves to the nucleus tractus solitarius, and from there to
the supraoptic and paraventricular nuclei, thereby increasing the output of
ADH.
Lithium (choice D) does not affect ADH secretion. The primary effect of
lithium is to block ADH-mediated water reabsorption by inhibiting ADH-
sensitive adenylate cyclase production in the collecting duct cells, thereby
inducing ADH-resistance and causing polyuria.
Emetic stimuli (choice E) are very potent since they typically induce an
immediate 50- to 100-fold increase in plasma vasopressin. These stimuli act
via the emetic center in the medulla oblongata and can be completely
blocked with antiemetics, e.g. fluphenazine.



Question 3 of 5
Which of the following vasopressin receptors most likely mediates vasopressin's
antidiuretic action?
/ A. AQP6
/ B. Oxytocin receptor
/ C. V1a
/ D. V1b
/ E. V2

Explanation - Q: 1.3 Close

The correct answer is E. The vasopressin receptor family is unique among
all classes of peptide receptors, since its members couple to different
subsets of G proteins. Vasopressin binds to four distinct types of receptor
subtypes V1a, V1b, V2 and the oxytocin receptor. The V2 receptors are
located on the basolateral membrane of collecting duct cells. This receptor
belongs to the seven-transmembrane domain, G-protein-coupled receptor
superfamily. Stimulation of the V2 receptor by vasopressin leads to the
activation of a G protein (Gs), resulting in the activation of adenylyl cyclase,
the production of cAMP, and the stimulation of protein kinase A (PKA). PKA
stimulates aquaporin 2, a water channel protein, which undergoes vesicular
trafficking, inserting into the apical membrane of the duct cells, thereby
increasing the water permeability of that membrane. At maximal ADH levels,
less than 1% of the filtered water is excreted, with the minimum urine volume
of 500 mL/day and osmolality of 1200 mosmol/L. Long term regulation by
vasopressin involves the effects of cAMP on the cAMP-response elements in
the 5'-flanking region of the AQP2 gene, which increases the abundance of
aquaporin 2 protein in the target cells. The human V2 receptor gene, AVPR2,
is located in chromosome region Xq28. The genetic form of nephrogenic
diabetes insipidus is caused by mutations in the coding region of the V2
receptor gene.
AQP6 (choice A) is a water channel protein that is localized exclusively in
intracellular membranes in renal epithelia.
Vasopressin has low affinity for oxytocin receptors. The oxytocin receptor
(choice B) is a typical class I G protein-coupled receptor, primarily
stimulating phospholipase C beta activity.
The V1a receptor (choice C) mediates smooth muscle contraction
(vasoconstriction in splanchnic, coronary, cerebral, peripheral, pulmonary,
and intrahepatic vessels), platelet aggregation, hepatic glycogenolysis, and
coagulation factor release by vasopressin. It is also proposed that brain V1a
vasopressin receptors may mediate the effect of ADH on short-term memory
and learning, antipyresis, cardiovascular responsivity, and brain
development.
V1b (or V3) receptors (choice D) are expressed in the anterior pituitary, and
their activation leads to the potentiation of ACTH secretion from pituitary
corticotrophs. The binding of vasopressin to V1b and V1a receptors results in
activation of phospholipase C mediated by a specific pertussis toxin-
insensitive receptor-coupled G protein (Gq). This activation induces the
production of diacylglycerol (DAG), which activates protein kinase C, and
inositol trisphosphate (IP3) that mobilizes intracellular calcium.


Question 4 of 5
The principal target of vasopressin actions in the kidney is water permeability of
the collecting ducts. Which of the following water channel
proteins is the mediator of this vasopressin effect?
/ A. Aquaporin 0 (AQP0)
/ B. AQP1
/ C. AQP2
/ D. AQP3
/ E. AQP5


Explanation - Q: 1.4 Close

The correct answer is C. The aquaporins (AQPs) represent a family of
water-selective membrane channels that play a critical role in controlling the
water content of cells. To date, ten AQPs have been identified and cloned in
humans, from AQP0-AQP9. They are integral membrane proteins and
belong to the major intrinsic protein family. Aquaporin 2 (AQP2) is expressed
exclusively in the apical membrane and subapical vesicles in inner medullary
collecting duct principal cells. It is diffusely distributed in the cytoplasm in the
euhydrated state, whereas apical localization is seen in the dehydrated
conditions or after vasopressin administration. The binding of vasopressin
(ADH) to its receptor in the cells causes the translocation of the intracellular
pool of AQP2, and its exocytic insertion into the apical membrane (the shuttle
mechanism), and also stimulates transcription of the AQP2 gene. The cycle
of insertion of water channels into, then removal from the luminal membrane
is referred to as vesicular trafficking. AQP2 forms a tetrameric complex that
spans the membrane, creating a channel, which allows rapid water
movement in response to an osmolar gradient. When open, the channel
allows 3 billion molecules of water to pass each second. The net rate of
water flux across the cell is determined by the number of AQP2 channels in
the apical membrane and the strength of the osmotic gradient between
tubular fluid and the renal medulla. Tight junctions on the lateral surface
serve to prevent unregulated water flow. This water channels facilitate, in
some cases, the transport of other small solutes, e.g., urea and glycerol.
They are, however, completely impermeable to charged particles, such as
protons, which is critical for the conservation of membrane electrochemical
potential. Mercurial diuretics bind to a specific site on AQP2 and block water
reabsorption.
AQP0 (choice A), a major intrinsic protein of the ocular lens fiber membrane,
is the prototype molecule of the water channel protein family.
AQP1 (choice B, also known as CHIP, channel-forming integral protein of 28
kDa) was the first protein shown to function as a molecular water channel. It
is expressed in the proximal tubule, the descending thin limbs of Henle
loops, and in vasa recta, contributing to the urinary concentrating
mechanism.
AQP3 (choice D) is localized at the basolateral membranes of collecting duct
cells in the kidney. The permeability of the basolateral membrane is not
regulated by vasopressin, and the presence of AQP3 (together with AQP4)
means the basolateral membrane has continuous water permeability.
AQP5 (choice E) is implicated in the generation of saliva, tears, and
pulmonary secretions.



Question 5 of 5
Which of the following is the most appropriate pharmacotherapy?
/ A. Carbamazepine
/ B. Chlorpropamide
/ C. Desmopressin (DDAVP)
/ D. Hydrochlorothiazide
/ E. Indomethacin (NSAIDS)

Explanation - Q: 1.5 Close

The correct answer is C. DDAVP (desmopressin acetate, 1-deamino-8-D-
arginine vasopressin), a synthetic analog of arginine vasopressin, has
prolonged antidiuretic (but not vasopressor) activity lasting for 12 to 24h, and
may be administered intranasally, sc, or IV. DDAVP produces a slight
increase in total body water (1-3%) and a commensurate decrease in plasma
osmolality and sodium concentration that rapidly eliminates thirst and
polydipsia. DDAVP acts selectively at V2 vasopressin receptors to increase
urine concentration and decrease urine flow in a dose-dependent manner.
Carbamazepine (choice A), an anticonvulsant, may be beneficial because of
its ADH-releasing properties, but is not the most appropriate choice.
Chlorpropamide (choice B), a hypoglycemic agent, has an antidiuretic
action, and also promotes a renal response to ADH. At doses similar to those
used in the treatment of DM, chlorpropamide increases urine concentration,
decreases urine flow, thirst and polydipsia in a manner similar to DDAVP, but
with higher doses there is a risk of hypoglycemia.
Hydrochlorothiazide (choice D), a diuretic, can paradoxically decrease
polyuria to some extent with low salt intake, in the absence of ADH. Urine
volumes may fall 25-50% during the daily administration of 15-25 mg/kg of a
thiazide diuretic.
Inhibition of prostaglandin synthesis reduces delivery of solute to distal
tubules. This effect would reduce urine volume and increase urine osmolality.
Indomethacin (choice E) combined with restriction of sodium intake and
thiazide diuretics would be the most efficient therapy for nephrogenic DI, but
in this case of vasopressin deficiency, antidiuretic hormone replacement is
most appropriate.





A child is taken to a pediatrician because his mother notices that his eyes appear
very puffy. The mother notes that the boy's eyes appeared
normal two days ago, and part of what caused her concern was that her child
seemed to be rapidly becoming ilI. On physical examination, the
boy is noted to have generalized edema. No hypertension or jaundice is noted.
BIood urea nitrogen and serum creatinine are within normal
Iimits. A urine sample is collected, and the nurse notices that the top of the urine
has a small amount of foam at the top. Urinalysis is negative
for glucose, red cells, white cells, casts, crystals, and bacteria. A 24-hr-urine
specimen is collected, which demonstrates proteinuria of 55
mg/h/m2.
Question 1 of 6
Which of the following is the most likely diagnosis?
/ A. Acute renal failure
/ B. Chronic renal failure
/ C. Lower urinary tract disease
/ D. Nephritic syndrome
/ E. Nephrotic syndrome


Explanation - Q: 2.1 Close

The correct answer is E. Proteinuria greater than 40 mg/h/m2 in a child or
3.5 g/d/1.73 m
2
in an adult produces nephrotic syndrome, which is
characterized by generalized edema, often most noticeable in the face. The
condition develops when large amounts of protein are spilled through the
glomeruli into the duct system of the kidneys. In contrast, nephritic syndrome
(choice D) is accompanied by lower levels of proteinuria, together with
microscopic or macroscopic hematuria (red cells and blood in urine).
Acute (choice A) and chronic (choice B) renal failure are characterized by
rising serum levels of BUN and creatinine.
The findings seen do not suggest lower urinary tract disease (choice C),
which is typically due to a lower urinary tract infection (which would show
bacteria and white cells), calculi (crystals would likely be present), or tumor
(which would show abnormal bladder epithelial cells).


Question 2 of 6
The edema seen in this patient is the result of which of the following processes?
/ A. Lowered hydrostatic pressure in capillaries
/ B. Lowered hydrostatic pressure in the extracellular interstitial space
/ C. Lowered oncotic pressure in capillaries
/ D. Lowered oncotic pressure in the extracellular interstitial space
/ E. Occlusion of lymphatic channels
Explanation - Q: 2.2 Close

The correct answer is C. Edema occurs when there is a net shift of fluid into
the extracellular interstitial space. In the case of the edema produced by
nephrotic syndrome, the urinary protein losses lead to a decreased serum
concentration of albumin, which, in turn, lowers the oncotic pressure within
the capillary bed. This favors a shift of fluid from the vascular space to the
extracellular interstitial space.
Edema formation can also be favored by raising the hydrostatic pressure in
the capillary bed (opposite of choice A), lowering the hydrostatic pressure in
the extracellular interstitial space (choice B, as happens when a person
wearing support hose takes them off and develops ankle edema), or
occluding the lymphatic channels (choice E, as can happen in filarial
elephantiasis or following axillary lymph node dissection for breast cancer).
In contrast, lowering the oncotic pressure in the extracellular interstitial space
(choice D), as can happen when there is resolution of a previous tissue
injury with resorption of proteins released by cell injury, tends to shift fluid
back into the vascular space, and reduce edema.


Question 3 of 6
Which of the following is the most common cause of this patient's symptom
pattern in children?
/ A. Membranoproliferative glomerulonephritis
/ B. Membranous glomerulonephritis
/ C. Minimal change disease
/ D. Post-infectious acute glomerulonephritis
/ E. Proliferative glomerulonephritis as a complication of systemic lupus
erythematosus


Explanation - Q: 2.3 Close

The correct answer is C. Minimal change disease accounts for roughly 90%
of cases of nephrotic syndrome in children. This condition also occurs less
commonly in adults.
Membranoproliferative glomerulonephritis (choice A) can also cause
nephrotic syndrome in children, but is less common and may produce both
nephrotic syndrome and microscopic hematuria.
Membranous glomerulonephritis (choice B) is the most frequent cause of
nephrotic syndrome in adults greater than 40 years of age.
Post-infectious acute glomerulonephritis (choice D) is relatively common in
children, but usually causes nephritic syndrome, rather than nephrotic
syndrome.
Proliferative glomerulonephritis related to lupus (choice E) is a common
secondary cause of nephrotic syndrome, but most of the patients are adults.



Question 4 of 6
The peak age at which this patient's specific disease occurs is which of the
following?
/ A. 2 years
/ B. 5 years
/ C. 8 years
/ D. 12 years
/ E. 15 years

Explanation - Q: 2.4 Close

The correct answer is A. The peak age of incidence in children with minimal
change disease is 2 years, and over 80% of cases occur in children 6 years
of age or less. Adults who develop minimal change disease have mean age
of onset of 40 years.



Question 5 of 6
If a renal biopsy were performed, which of the following findings on electron
microscopy is most characteristic of this patient's specific
disease?
/ A. Effacement of podocyte foot processes
/ B. Expansion of mesangial cell population
/ C. "Spikes" of basement membrane material
/ D. Subendothelial deposits
/ E. SubepitheliaI "humps" deposits

Explanation - Q: 2.5 Close

The correct answer is A. Minimal change disease is so named because of
its very bland light microscopic appearance on renal biopsy. With electron
microscopy, widespread fusion is seen of the foot processes of the
podocytes that form the outer layer of the glomerular loops. You should be
aware that this finding, which is classically associated with minimal change
disease in test questions, is actually not completely specific for minimal
change disease and is instead found as well in other glomerular diseases
associated with high proteinuria. However, in these other diseases, the
glomeruli will show other changes (electron dense deposits and/or cellular
proliferations) as well.
"Spikes" (choice C) of basement membrane material separating
epimembranous electron-dense deposits are characteristic of membranous
glomerulonephritis.
Subendothelial electron-dense deposits (choice D) and the expansion of the
mesangial cell population (choice B) can be seen in a variety of glomerular
diseases, including proliferative glomerulonephritis and
membranoproliferative glomerulonephritis.
Electron-dense "humps" (choice E) are characteristic of post-infectious
acute glomerulonephritis.



Question 6 of 6
A drug from which of the following classes would be most likely to correct this
patient's proteinuria?
/ A. Antibiotic
/ B. Corticosteroid
/ C. Potassium-sparing diuretic
/ D. Sex steroid
/ E. Thiazide diuretic

Explanation - Q: 2.6 Close

The correct answer is B. Approximately 90% of children with minimal
change disease respond within two weeks to corticosteroid therapy with a
decrease in proteinuria. This rapid response, together with the high incidence
of minimal change disease compared to other causes of nephrotic syndrome
in children, has led to a common practice among pediatricians of reserving
renal biopsy for those cases in which the response to steroids was either
non-existent or much less pronounced. Of the children who respond, most
will undergo a complete remission, although in roughly half of cases
recurrent episodes of minimal change disease may later occur. Adults and
adolescents will also often respond to steroid therapy, although the time to
remission may be up to 16 weeks.
Antibiotics (choice A) are used in patient with minimal change disease to
control intercurrent infections that develop, in part, because of the
pharmacologic immunosuppression.
Potassium-sparing diuretics (choice C) and sex steroids (choice D) are not
usually used in this population.
Thiazide diuretics (choice E) are occasionally used in those patients with
very severe edema, particularly if accompanied by respiratory symptoms.



A child is brought to an emergency department after his mother notes brown-
discolored urine in the toilet. The child has periorbital edema,
although generalized edema is not seen. Urinalysis demonstrates 2+ proteinuria
and hematuria. A few neutrophils and renal epithelial cells are
also seen in the urinalysis sample.
Question 1 of 8
AIso noted on urinalysis are numerous large, elongated, stick-Iike, translucent
and colorless structures with low refractive index that are difficult
to discern on microscopy. These are most likely which of the following?
/ A. Ammonium biurate crystals
/ B. Calcium oxalate crystals
/ C. Hyaline casts
/ D. Triple phosphate crystals
/ E. Urothelial cells

Explanation - Q: 3.1 Close

The correct answer is C. The structures are hyaline casts.
Ammonium biurate crystals (choice A) have an unusual "crab-like" structure
and a yellow-brown color.
Calcium oxalate crystals (choice B) form colorless, birefringent,
octahedrons.
Triple phosphate crystals (choice D) form colorless, birefringent crystal with
a "coffin-lid" shape.
Urothelial cells (choice E) are clear and round, with a visible clear, ovoid
central nucleus.


Question 2 of 8
The significance of the elongated, colorless structures seen on urinalysis is which
of the following?
/ A. They indicate that desquamation of the urothelial mucosa is occurring
/ B. They indicate that infection is present
/ C. They indicate that stone formation is likely
/ D. They indicate that the kidney is involved
/ E. They indicate that the person may have an increased risk of developing
gout

Explanation - Q: 3.2 Close

The correct answer is D. Casts form within the renal tubular system, and
specifically indicate that the disease process involves the kidneys.
The presence of urothelial cells in urine would indicate that desquamation of
urothelial mucosa was occurring (choice A), as happens normally to a small
degree.
The presence of bacteria and neutrophils would suggest infection (choice
B).
The presence of some types of crystals in the urine (choice C) may suggest
that a predisposition for stone formation is present.
While urate crystals in tissues are a feature of gout, their presence in urine is
not considered a risk factor for developing gout (choice E).



Question 3 of 8
Which of the following is the most likely diagnosis?
/ A. Acute nephritis
/ B. Acute pyelonephritis
/ C. Chronic pyelonephritis
/ D. Minimal change disease
/ E. Transitional cell carcinoma of the bladder

Explanation - Q: 3.3 Close

The correct answer is A. This child has an acute nephritis, as evidenced by
the combination of hematuria, evidence of renal involvement (e.g., the
epithelial cells and casts), proteinuria, and lack of bacteria or significant
inflammatory cell response in the urine. Acute nephritis is clinically
subdivided into cases related to previous infection and those related to other
conditions (e.g., Henoch-Schnlein purpura, Berger disease, systemic lupus
erythematosus, familial forms, shunt nephritis).
Acute pyelonephritis (choice B) would show numerous bacteria and
neutrophils in the urine.
Chronic pyelonephritis (choice C) typically is associated with an
unremarkable urine, although scanty casts may be found.
Minimal change disease (choice D) would be characterized by severe
proteinuria without hematuria.
Transitional cell carcinoma of the bladder (choice E) would be very unusual
in a child, and might cause hematuria and abnormal urothelial cells in urine,
but would not cause casts in the urine.


Question 4 of 8
If this child's renal disease is related to a prior infection, the child most likely has
a history of which of the following in the preceding 7-21 days?
/ A. Cystitis
/ B. Diarrhea
/ C. Gastritis
/ D. Pharyngitis
/ E. Urethritis

Explanation - Q: 3.4 Close

The correct answer is D. Postinfectious glomerulonephritis most commonly
follows a sore throat, but can also follow streptococcal skin infections. The
peak incidence of poststreptococcal glomerulonephritis is 5-6 years, with
most cases occurring between the ages of 4 and 12. Cases have been
reported, however in children as young as 1 year of age and up to adults as
old as 90 years. Patients may exhibit a range of symptoms from one extreme
of an asymptomatic child with urinary abnormalities picked up incidentally to
the opposite extreme of a child with severe oliguria, edema, hypertension,
and azotemia. Statistically, most patients have disease toward the mild end
of the spectrum.
Poststreptococcal glomerulonephritis does not usually follow lower urinary
tract infections (choices A and E) or gastrointestinal disease (choices B
and C).


*** Commercial version is infinite. Order at http://www.structurise.com/kleptomania ***
Question 5 of 8
Which of the following is the most common causative organism implicated in the
infection preceding the child's urinary symptoms?



Explanation - Q: 3.5 Close

The correct answer is A. Nephritogenic strains of Group A beta-hemolytic
streptococci (e.g., type 12, associated with pharyngitis and type 49,
associated with impetigo) are most often implicated in postinfectious
glomerulonephritis.
Much less commonly, Group C Streptococcus(choice C), Staphylococcus
aureus(choice D), and Staphylococcus epidermidis(choice E) have been
implicated.
Group B Streptococcus(choice B) has, to date, not yet been implicated.



Question 6 of 8
Which of the following is also commonly present in patients with this child's
disease?
/ A. Arthritis
/ B. Hypertension
/ C. Hypothyroidism
/ D. Middle ear infection
/ E. Psoriasis


Explanation - Q: 3.6 Close

The correct answer is B. Hypertension is reported in 50-90% of children
who are hospitalized with acute poststreptococcal glomerulonephritis. The
degree of increase in blood pressure is variable, but may reach systolic
pressures greater than 200 mm Hg and diastolic pressures greater than 120
mm Hg. Children with severe hypertension may have headache, vomiting,
altered mentation, visual disturbances, aphasia, coma, and convulsions. The
other answers are distracters.


Question 7 of 8
While renal biopsy is no longer common in this child's condition, it has been
historically important in defining the pathophysiology of the
disease. Which of the following findings on electron microscopic examination is
the most specific for this disease?
/ A. Crescent formation in Bowman's space
/ B. Eosinophils in interstitium
/ C. Fusion of podocyte foot processes
/ D. KimmelstieI-Wilson nodules
/ E. SubepitheliaI "humps"

Explanation - Q: 3.7 Close

The correct answer is E. Subepithelial electron dense "humps," composed
of IgG and C3, are the most characteristic finding in acute postinfectious
glomerulonephritis. It is unclear whether these form in situ, or are circulating
complexes that show a specific tendency to lodge in this site. Serum C3
levels are often depressed in these patients, secondary to consumption in
the humps.
Crescent formation in Bowman's space (choice A) is also common in
postinfectious glomerulonephritis, but this finding is not specific. It does
suggest that severe disease is present.
Eosinophils in the interstitium (choice B) can be seen in drug- or allergy-
related tubulointerstitial disease.
Fusion of podocyte foot processes (choice C) is classically associated with
minimal change disease, although it may also be seen in some other renal
diseases.
Kimmelstiel-Wilson nodules (choice D) are a feature of diabetic renal
disease.


Question 8 of 8
Which of the following is the most likely outcome for this patient?
/ A. Complete resolution
/ B. Persistence of hematuria for more than 3 years
/ C. Progression to complete renal failure in less than 1 year
/ D. Progression to complete renal failure in more than 5 years
/ E. Prolonged relapsing and remitting course lasting for a decade or more


Explanation - Q: 3.8 Close

The correct answer is A. Poststreptococcal glomerulonephritis has a
favorable prognosis in more than 95% of cases, with most patients
experiencing complete resolution. Typically, the edema resolves within 5-10
days. The blood pressure usually drops to normal within several weeks (up
to 6 weeks), and serum C3 concentration returns to normal within a few
months. Microscopic hematuria and proteinuria may last up to 1 or 2 years
(rarely 3 or more years, still with eventual complete resolution). Medications
used to treat patients with postinfectious glomerulonephritis include
antihypertensives, furosemide, and antibiotics (to eradicate any residual
streptococci).


A 77-year-old man in an intensive care unit because of severe gram-negative
pneumonia and sepsis develops oliguria, with a urinary output of
only 25 cc/hr. A urine sample sent for chemical studies demonstrates a sodium
concentration of 15 mEq/L, urine creatinine of 41 mg/dL, and
an osmolality of 570 mOsm/kg. A serum sample taken at the same time shows
sodium of 142 mEq/L and creatinine of 2.2 mg/dL. Two days
previously, the patient's serum creatinine had been within normal limits.
Question 1 of 7
The major role of creatine in muscle is which of the following?
/ A. Bind phosphate to form a storage compound for energy
/ B. Bind to glucose in the pathway that forms galactose
/ C. Catalyze formation of water molecules in mitochondria
/ D. Combine with ammonia in the beginning of the urea cycle
/ E. Serves as a coenzyme in ketone body synthesis


Explanation - Q: 4.1 Close

The correct answer is A. Phosphocreatine, which is a phosphorylated form
of creatine, is a major storage form of chemical energy used by muscle. In
the resting muscle state, levels of phosphocreatine are high, and then as the
muscle contracts, the levels drop as the chemical energy is used. Some of
the phosphocreatine will also spontaneous lose inorganic phosphate,
producing creatinine. The rate at which this occurs in any individual tends to
be relatively constant (although individuals with high muscle mass, e.g.
athletes, will have higher production rates than those with low muscle mass).
Thus, the day to day changes in serum creatinine in an individual tend to
reflect day to day variations in renal function.
Choices B, C, D, and E refer respectively to uridine diphosphate,
cytochromes a and a3, CO
2
, and coenzyme A.





Question 2 of 7
What is this patient's fractional excretion of sodium [Fex Na (%)]?
/ A. 0.12%
/ B. 0.37%
/ C. 0.56%
/ D. 1.23%
/ E. 3.57%

Explanation - Q: 4.2 Close

The correct answer is C. The fractional excretion of sodium has become
popular as a means of clinically differentiating prerenal azotemia from other
forms of acute renal failure, and in particular acute tubular necrosis. It is
calculated with the following formula:

The fractional excretion of sodium is almost always less than 1% in prerenal
failure (due to underperfusion), and it is almost always greater than 1.5% in
acute tubular necrosis, which can occur in similar clinical settings but should
be treated very differently medically.



Question 3 of 7
What is this patient's renal failure index (RFI)?
/ A. 0.5 mEq/L
/ B. 0.8 mEq/L
/ C. 1.5 mEq/L
/ D. 1.8 mEq/L
/ E. 3.2 mEq/L

Explanation - Q: 4.3 Close

The correct answer is B. Like the fractional excretion of sodium, the renal
failure index is a popular index used to distinguish prerenal failure from other
forms of acute renal failure. It is very similar to the F
ex
Na, but assumes that
the serum sodium is constant:

With prerenal failure, the RFI is usually less than 1 mEq/L, while in most
other established forms of renal failure, it is usually greater than 3 mEq/L.


Question 4 of 7
Which of the following processes would be most likely to produce this patient's
rising serum creatinine?
/ A. Benign prostatic hyperplasia
/ B. Drug toxicity causing acute tubular necrosis
/ C. Hypotension
/ D. Post-infectious glomerulonephritis
/ E. Stone impacted in ureter

Explanation - Q: 4.4 Close

The correct answer is C. Both the fractional excretion of sodium and the
renal failure index suggest that this patient has prerenal failure. Prerenal
failure is due to inadequate perfusion of the kidney, and can be seen in
settings in which the blood pressure drops to the point of impairing blood flow
to individual glomeruli. The tubular network in this setting is still usually
functional, and actively reabsorbs sodium and water, producing relatively low
values of urine sodium and high osmolarity. Acute tubular necrosis (choice
B) and glomerulonephritis (choice D) are causes of intrarenal failure; while
stone impaction (choice E, usually bilateral unless only one kidney is
functional) and severe benign prostatic hyperplasia (choice A) can cause
lower urinary tract obstruction with post-renal failure.



Question 5 of 7
This patient's low urine sodium suggests that he is still actively reabsorbing
sodium. Which of the following hormones acts directly on the renal
tubular system to increase sodium reabsorption?
/ A. AIdosterone
/ B. Angiotensin l
/ C. Angiotensin ll
/ D. Erythropoietin
/ E. Renin


Explanation - Q: 4.5 Close

The correct answer is A. This question asks you to think about the specifics
of the function of the renin-angiotensin-aldosterone system. In this patient's
case, decreased blood pressure in the renal afferent arteriole stimulates the
baroreceptors in the juxtaglomerular cells, which secrete the enzyme renin
(choice E). Renin acts on circulating angiotensinogen made by the liver to
produce the prohormone angiotensin I (choice B). Angiotensin I is then
converted by angiotensin converting enzyme (ACE) in the lung to
angiotensin II (choice C). The angiotensin II acts on the adrenal zona
glomerulosa to trigger the secretion of aldosterone (choice A). It is the
aldosterone that acts on the renal tubular system to increase the absorption
of Na
+
and water.
Erythropoietin (choice D) is produced by the kidney and stimulates the
growth of bone marrow erythroid elements.




Question 6 of 7

The hormone that acts directly on the renal tubular system to increase sodium
reabsorption has which of the following effects?

/ A. Exchanges Na+ for HCO3- in the distal collecting duct
/ B. Exchanges Na+ for HCO3- in the loop of Henle
/ C. Exchanges Na+ for HCO3- in the proximal tubule
/ D. Exchanges Na+ for K+ in the collecting ducts
/ E. Exchanges Na+ for K+ in the loop of Henle



Explanation - Q: 4.6 Close

The correct answer is D. Aldosterone promotes the activity of a Na/K-
ATPase-dependent pump located in the epithelial cells lining the distal
tubules and collecting ducts. This pump moves Na
+
into the renal
extracellular fluid in exchange for K
+
. The other choices are distracters.




Question 7 of 7

Which of the following bacterial components is the most common exogenous
mediator of sepsis in gram-negative bacterial infections?
/ A. Double stranded RNA
/ B. Endotoxin
/ C. FIagellin
/ D. Lipoteichoic Acid
/ E. Peptidoglycan

Explanation - Q: 4.7 Close

The correct answer is B. Endotoxin, also known as lipopolysaccharide, is
located in the outer membrane of almost all gram-negative bacteria.
Endotoxin is composed of three parts: lipid A, core sugars, and long repeats
of oligosaccharide side chains. The lipid A moiety is buried in the membrane.
It is the component of endotoxin that elicits the release of mediators that can
cause fever, shock and death. The mortality rate when septic shock occurs is
approximately 45%. Most patients who die have multi-organ failure. Up to
one quarter of septic shock patients develop acute renal failure. Other major
complications include adult respiratory distress syndrome, central nervous
system dysfunction, liver failure, and disseminated intravascular coagulation.
All of the other answers are bacterial components that have been implicated
in mediating septic shock. It is worthwhile to note that peptidoglycan (choice
E) is thought to be the main mediator of sepsis in gram-positive bacterial
infections.


A 50-year-old obese man complains of being tired all the time. He is always
thirsty, and has increased frequency of urination. His blood
pressure is 145/92 mm Hg, his pulse is 75/min, and his respirations are 15/min.
Laboratory studies show:
UrinaI sis:
GIucose ++
Ketones - negative
Creatinine - 40 mg/dL
Urine fIow _3 mL/min
BIood:
GIucose _ 374 mg/dL
Creatinine _1.0 mg/dL
Question 1 of 6
Which of the following is the best estimate of glomerular filtration rate in this
individuaI?
/ A. 40 mL/min
/ B. 60 mL/min
/ C. 80 mL/min
/ D. 100 mL/min
/ E. 120 mL/min


Explanation - Q: 5.1 Close

The correct answer is E. Although not a perfect glomerular filtration marker,
the clearance of creatinine can be used to give a good estimate of
glomerular filtration rate (GFR). Creatinine has the advantage of being
produced endogenously, and does not need to be infused like inulin. GFR =
40 mg/dL x 3 mL/min 1.0 mg/dL = 120 mL/min. The patient has a normal
GFR, which is not uncommon in type 2 diabetes mellitus. On the other hand,
approximately 50% of patients with type 1 diabetes have increased GFR
(hyperfiltration) within the first year of the disease.





Question 2 of 6

Subcutaneous fat is obtained from the patient and from a normal individuaI. The
tissue is homogenized and solubilized with detergent. Total
protein is separated by polyacrylamide gel electrophoresis and then transferred
to nitrocellulose for subsequent Western blot. BIots are
probed for the insulin receptor (both and subunits), the phosphorylated form
of the insulin receptor, insulin receptor substrate-1 (IRS-1), and
the phosphorylated form of IRS-1. The following data are obtained:





Which of the following is the most likely primary defect in insulin signaling in the
patient?
/ A. Decreased IRS-1 activity
/ B. Decreased tyrosine kinase activity of the insulin receptor
/ C. Down regulation of the insulin receptor -subunit
/ D. Down regulation of the insulin receptor -subunit
/ E. Down regulation of IRS-1


Explanation - Q: 5.2 Close

The correct answer is B. The Western blot shows that the phosphorylated
form of the insulin receptor is decreased in the patient compared to normal.
When insulin binds to the -subunit of its receptor it causes a conformational
change in the -subunit that activates its tyrosine kinase activity. This leads
to autophosphorylation of tyrosine residues in the cytoplasmic domain of the
-subunit. There is actually cross-phosphorylation, where the tyrosine kinase
on one -subunit phosphorylates tyrosine residues on the other -subunit. A
diminished activity of the receptor tyrosine kinase activity could explain the
decrease in the phosphorylated form of the receptor. IRS-1 activity is also
likely to be decreased (choice A), given its decreased relative
phosphorylation in the patient. But because phosphorylation of IRS-1 occurs
after the insulin receptor is phosphorylated, its decreased activity would be a
secondary rather than primary defect.
The relative amount of insulin receptor protein (both - and -subunits) is the
same in both individuals, making downregulation of the receptor (choices C
and D) an unlikely cause of the insulin resistance.
Likewise, the Western blot shows that the relative amount of the IRS-1
protein is the same in both individuals, making it unlikely that decreased IRS-
1 (choice E) is the problem in the patient.


Question 3 of 6
A reduced calorie diet and regular exercise program are prescribed. The patient
is also started on glyburide (5 mg per day). Which of the
following best describes the effect of glyburide?
/ A. Decreases glucose absorption by the small intestine
/ B. Hyperpolarizes the membrane potential of beta cells
/ C. Increases ATP formation within beta cells
/ D. Increases GLUT 4 expression in fat and muscle
/ E. Increases insulin secretion from beta cells

Explanation - Q: 5.3 Close

The correct answer is E. Sulfonylureas act primarily by increasing insulin
secretion from cells. The drug apparently binds to or near ATP-sensitive K
+

channels in the cell membrane and closes them. The resultant decrease in
K
+
efflux depolarizes the membrane resting potential and causes the opening
of voltage-gated Ca
++
channels. The resultant influx of Ca
++
stimulates the
exocytosis of insulin from secretory granules.
Postprandial hyperglycemia can be diminished by drugs, like acarbose, that
decrease glucose absorption (choice A). Acarbose is a competitive inhibitor
of intestinal brush border -glucosidases. The digestion and absorption of
dietary sucrose and other carbohydrates is diminished. Sulfonylureas do not
affect carbohydrate absorption.
Hyperpolarization of the membrane of the cell (choice B) would make the
voltage-gated Ca
++
channels less likely to open, and would decrease insulin
secretion.
Glucose stimulates insulin secretion by entering cells and being
metabolized. The resultant increase in intracellular ATP closes the same
ATP-sensitive K
+
channels that are affected by the sulfonylureas. The
subsequent opening of voltage-gated Ca
++
channels and influx of calcium
stimulates insulin secretion. The sulfonylureas do not affect intracellular
concentration of ATP (choice C).
There is some in vitro evidence that sulfonylureas increase the action of
insulin at its target cells. However, there is no evidence that the drug works
by increasing the expression of the GLUT 4 transporter (choice D). Clinical
studies show that sulfonylureas do not have any therapeutic benefit on long-
term glycemic control when added to insulin therapy, suggesting that extra
cell effects of the drug have little clinical significance.


Question 4 of 6
Three years later, the patient is still overweight. Fasting plasma glucose is 300
mg/dL and HbA1c is 12% (normaI, 5-8%). He complains of
impotence, that has worsened over the past year. Nocturnal erections are
absent. Injection of papaverine (vasodilator) into the corpus
cavernosum produces an erection that lasts for an hour. Which of the following is
the most likely explanation for the erectile dysfunction?
/ A. Autonomic neuropathy
/ B. BIood vessel occlusion
/ C. Diabetic amyotrophy
/ D. Peripheral sensory neuropathy
/ E. Psychological


Explanation - Q: 5.4 Close

The correct answer is A. Impotence is a long-term complication of diabetes.
It is due to dysfunction of the autonomic nerves that mediate the erection. An
erection is produced when the parasympathetic nerves that innervate the
penile arterioles release acetylcholine, vasoactive intestinal peptide (VIP),
and nitric oxide (NO). These neurotransmitters produce vasodilation with
increased blood flow into the erectile tissues of the penis. As the penis
becomes engorged with blood, the penile veins become compressed, so that
outflow is blocked. Diabetes can be complicated by autonomic neuropathy.
The exact cause of the neuropathy is unknown, but may include direct toxic
effects of high extracellular glucose, accumulation of sorbitol within the
nerves, or poor blood supply to the nerves because of decreased NO release
by blood vessel endothelial cells.
Blood vessel occlusion (choice B) is a potential cause of erectile
dysfunction. It is sometimes responsible for the impotence in diabetes
because of vasculopathy. However, the fact that papaverine (vasodilator)
injection into the penis evoked a normal erection suggests that blood supply
to the penis is not compromised.
Peripheral motor neuropathy is also part of the long-term complications of
diabetes. Sometimes the patient with motor neuropathy experiences acute
pain and weakness in the thigh muscles bilaterally, accompanied by muscle
wasting. This has been termed diabetic amyotrophy (choice C). Peripheral
motor neuropathy would not affect erectile function.
Peripheral sensory neuropathy (choice D) is also a complication of diabetes.
It is characterized by years of paresthesias and pain that progress to
numbness and decreased tactile sensation. It is usually bilateral and
symmetrical. The causes of the sensory neuropathy are probably the same
as that for motor and autonomic neuropathy.
A psychological (choice E) cause of the impotence is unlikely, given the fact
that nocturnal erections do not occur.











Question 5 of 6
After another three years, Iater the patient is still struggling with poor glycemic
controI. Urine output is decreasing, plasma creatinine is 3.0
mg/dL (normaI 0.7 -1.2 mg/dL), and proteinuria is greater than 4 g/day. A biopsy
of renal tissue is likely to reveal which of the following
abnormalities?
/ A. Acute tubular necrosis
/ B. Immune complex glomerulonephritis
/ C. Membranous nephropathy
/ D. Minimal change disease
/ E. Nodular glomerulosclerosis


Explanation - Q: 5.5 Close

The correct answer is E. The patient is undoubtedly progressing toward
diabetic renal failure. Urine output is falling. In diabetic renal failure, unlike
other renal diseases, large amounts of protein continue to be excreted, even
in the face of decreased GFR and urine flow. Glomerular disease is
characterized by thickened capillary walls that can obliterate the vessels.
There is also deposition of basement membrane matrix in the mesangium. In
50% of the cases, the glomerulosclerosis is nodular (Kimmelstiel-Wilson
nodules).
Acute tubular necrosis (choice A) is an intrarenal cause of acute renal
failure. It is characterized by proximal tubular or thick ascending limb of the
loop of Henle necrosis due to either renal ischemia (e.g., septic shock or
hypotension during surgery) or toxic agents (e.g., aminoglycoside
antibiotics). Given the chronic nature of the patient's renal disease, acute
tubular necrosis is unlikely.
Immune complex glomerulonephritis (choice B) is unlikely. The glomerulus
is highly susceptible to the entrapment of immune complexes. For example,
immune complexes related to systemic lupus erythematosus can form
subepithelial deposits outside glomerular capillaries. The presence of the
immune complexes can be visualized at the light microscope level using
fluorescein-tagged antibodies against human IgG. The resulting injury
causes retraction of epithelial cell foot processes. The result is the clinical
picture of nephrotic syndrome. There is no inflammatory response leading to
an active urine sediment (with casts or red cells), because the immune
complexes are sequestered from blood-borne inflammatory cells. This sort of
subepithelial cell damage is also called membranous nephropathy (choice
C).
Minimal change disease (choice D) is a common cause of nephrotic
syndrome in children, but is less common in adults. Electron micrographs
show diffuse foot process fusion. The cause is unknown, but likely involves
the immune system. Glucocorticoids usually are effective in reversing the
disorder.



Question 6 of 6
Several weeks later, the patient is brought to the emergency department by
EMS. His wife was unable to wake him from an afternoon nap. He
is comatose. One week prior to the present episode, he came down with a case
of the flu. He has been extremely thirsty since. BIood is drawn
and sent to the laboratory. Which of the following findings is most likely to be
present in this patient?
/ A. Bicarbonate - 10 mEq/L (normaI, 22-28 mEq/L)
/ B. pH - 7.1 (normaI 7.35-7.45)
/ C. PIasma glucose - 45 mg/dL (normaI, 70-110 mg/dL)
/ D. Serum ketones - 45 mg/dL (normaI, < 3 mg/dL)
/ E. Serum osmolarity - 340 mosm/L (normaI, 280-295 mosm/L)

Explanation - Q: 5.6 Close

The correct answer is E. The patient most likely is suffering from
hyperosmolar hyperglycemic coma. Lethargy and confusion develop as
serum osmolarity exceeds 300 mosm/L, and coma can occur if osmolarity
exceeds 330 mosm/L. The most common precipitating factor is infection.
Although serum sodium values are not given, the blood glucose
concentration can be estimated using the following formula:
Effective serum osmolarity = 2 x ([Na
+
] in mEq/L) + ([glucose] in mg/dL)/18.
Assuming a normal sodium concentration of 140 mEq/L, blood glucose
would be 1080 mg/dL (340 mosm/L = 2 X 140 + [glucose]/18). Serum sodium
is likely to be on the low side, however, because of the extra water caused
by the osmotic effect of the glucose. With a low serum sodium, the glucose
would be even higher.
Diabetic ketoacidosis (choices A, B, and D) is rare in patients with type 2
diabetes. The insulin deficiency must be severe to produce ketoacidosis, and
type 2 diabetics have enough residual insulin effect to prevent excess ketone
body production. Blood pH is usually > 7.3 in type 2 diabetics with
hyperosmolar hyperglycemic coma. Because H
+
production is not increased,
serum bicarbonate remains within normal limits. Low serum bicarbonate
would be expected if metabolic acidosis were present.
Hypoglycemia (choice C) can cause coma, but blood levels need to be
below 30 mg/dL. At 45 mg/dL, there is impaired cognition, confusion, blurred
vision, weakness, and lethargy. In this patient, with uncontrolled type 2
diabetes, hypoglycemia is not likely.

A 32-year-old woman presents to her family doctor complaining of 3 days of
lower abdominal pain and dysuria. She also complains of urinary
urgency and frequency. She is married and in a monogamous relationship with
her husband. She has no other significant past medical history.
Her temperature is 37.2 C (99.0 F), blood pressure is 122/64 mm Hg, pulse is
88/min, and respirations are 16/min. Her cardiac and lung
examinations are unremarkable. Abdominal examination is unremarkable except
for mild suprapubic tenderness to palpation. She has no
costovertebral angle tenderness. Her pelvic examination is also normaI.
Question 1 of 5
Which of the following is the most likely diagnosis?
/ A. Appendicitis
/ B. Cystitis
/ C. Gastroenteritis
/ D. Pelvic inflammatory disease
/ E. Urolithiasis


Explanation - Q: 6.1 Close

The correct answer is B. Suprapubic tenderness as well as dysuria,
frequency, and urgency are classic signs and symptoms of cystitis, which is
an infection of the urinary bladder. It is most often due to enteric organisms.
Women are more prone to cystitis, due to the shorter urethra.
Appendicitis (choice A) is due to bacterial proliferation and invasion of the
appendiceal mucosa. It is thought to be caused by the obstruction of the
appendiceal lumen, most often by a fecalith. Patients present with severe
abdominal pain that may initially be periumbilical, then localized to the right
lower quadrant. There are often peritoneal signs, such as involuntary
guarding and rebound on physical examination.
Gastroenteritis (choice C) is an inflammation of the stomach and intestinal
lining due to an infectious organism, which can be viral or bacterial. It is
associated with abdominal pain, and either vomiting or diarrhea.
Pelvic inflammatory disease (choice D) generally presents with severe lower
abdominal pain and fever, and is associated with a purulent cervical
discharge and cervical motion tenderness.
Urolithiasis (choice E) is due to a stone in the ureter. It often causes severe
pain due to acute distension of the ureter. Patients often present with severe
flank pain radiating to the groin. It is generally not associated with dysuria,
unless there is a concomitant urinary tract infection.


Question 2 of 5
A urine culture reveals multiple colonies of gram-negative rods that ferment
lactose. Which of the following is the most likely pathogen?




Explanation - Q: 6.2 Close

The correct answer is A. Escherichia coli is a common enteric organism
that causes urinary tract infections. It ferments lactose. Other enteric
organisms that ferment lactose include Enterobacter and Klebsiella. Proteus
mirabilis(choice C) and Pseudomonas aeruginosa(choice D) are also
enteric organisms that can cause urinary tract infections, but they do not
ferment lactose.
Neisseria gonorrhoeae(choice B) is a gram-negative coccus. It is associated
with pelvic inflammatory disease and sexually transmitted diseases.
Staphylococcus aureus(choice E) is a gram-positive coccus that is involved
in many different types of infection, including scalded skin syndrome, toxic
shock syndrome, abscess formation, endocarditis, and food poisoning.


Question 3 of 5
The patient is prescribed trimethoprim-sulfamethoxazole. She returns one week
later complaining of severe right-sided back pain and fever in
addition to her previous complaints. She did not fill her prescription for the
antibiotics because she read that the medication can cause a skin
reaction. To which of the following reactions is she most likely referring?
/ A. Eczematous dermatitis
/ B. Mottling of tooth enamel
/ C. Psoriasis
/ D. Stevens-Johnson syndrome
/ E. Verruca vulgaris
Explanation - Q: 6.3 Close

The correct answer is D. Stevens-Johnson syndrome is a generalized
hypersensitivity reaction usually due to a drug, in this case,
sulfamethoxazole. It initially presents with skin and mucous membrane
manifestations but can involve multiple organ systems. Patients often have a
sudden onset of fever as well as a progressive pleomorphic rash which can
include petechiae, vesicles, and bullae. Sulfonamides, penicillins, and
anticonvulsants are common culprits. Trimethoprim is not generally
associated with Stevens-Johnson syndrome.
Eczematous dermatitis (choice A) is a pruritic inflammatory disorder. It is
associated with atopic individuals as well as infection and chemical irritant
contact.
Mottling of tooth enamel (choice B) is a side-effect of tetracyclines, when
used in children under 8 years of age or in pregnant women (effects on the
fetus).
Psoriasis (choice C) is a chronic inflammatory skin process characterized by
erythematous papules and plaques. They have a characteristic silver scale,
and the lesions are sharply demarcated.
Verruca vulgaris (choice E) is also known as the common wart. It is a benign
papilloma caused by certain strains of human papilloma virus.




Question 4 of 5
On examination, the patient now has costovertebral angle tenderness and has a
temperature of 38.9 C (102 F). A urinalysis reveals white cell
casts. Which of the following is the most likely diagnosis?
/ A. Diffuse cortical necrosis
/ B. GIomerulonephritis
/ C. Pelvic inflammatory disease
/ D. Pyelonephritis
/ E. Urolithiasis

Explanation - Q: 6.4 Close

The correct answer is D. The patient previously had an uncomplicated
cystitis. Since the infection was not treated, it migrated up her urinary system
and caused an ascending pyelonephritis. The presence of white cell casts in
the urine is highly suggestive of acute pyelonephritis. A descending
pyelonephritis is caused by bacterial dissemination to the kidney by a
hematogenous route, such as in the case of endocarditis.
Diffuse cortical necrosis (choice A) is an acute generalized ischemic
infarction of both kidney cortices. It is most often associated with obstetric
catastrophes, septic shock, and other causes of vascular collapse.
Glomerulonephritis (choice B) would produce hematuria and red cell casts,
oliguria, azotemia, and some degree of hypertension.
Pelvic inflammatory disease (choice D), as noted above, generally presents
with severe lower abdominal pain and fever, and is associated with a
purulent cervical discharge and cervical motion tenderness. It is not
associated with urinary symptoms.
Urolithiasis (choice E), as noted above, often causes severe pain due to
acute distension of the ureter. Patients often present with severe flank pain
radiating to the groin. It is generally not associated with dysuria, fever, or
costovertebral angle tenderness unless there is a concomitant urinary tract
infection affecting the kidney.



Question 5 of 5
The patient is treated with a fluoroquinolone. Which of the following medications
was used?
/ A. Cefazolin
/ B. Ceftriaxone
/ C. Levofloxacin
/ D. Nafcillin
/ E. Oxacillin

Explanation - Q: 6.5 Close

The correct answer is C. Levofloxacin is a fluoroquinolone, which is a
bactericidal agent that inhibits DNA gyrase and topoisomerase IV. Examples
of other fluoroquinolones include ciprofloxacin and gatifloxacin.
Cefazolin (choice A) and ceftriaxone (choice B) are cephalosporins. They
are beta-lactam antibiotics that inhibit cell wall synthesis. They are less
susceptible to penicillinases. Cefazolin is a first generation cephalosporin,
and ceftriaxone is a third generation cephalosporin.
Nafcillin (choice D) and oxacillin (choice E) are both penicillinase-resistant
penicillins. They are bactericidal agents that bind penicillin-binding proteins
and block transpeptidase cross-linking of the cell wall. They also activate
autolytic enzymes within the bacteria.

Vignette 1 of 6


A 26-year-old woman complains to her physician of discomfort during
intercourse. Pelvic examination demonstrates a frothy, yellow-green vaginal
discharge with a strong odor. SmalI, red, ulcerations of the vaginal wall are also
seen. A wet mount preparation demonstrates motile, flagellated protozoa.


Which of the following is the most likely causative organism?



Explanation - Q: 1.1 Close

The correct answer is E. Trichomonas vaginalis, the causative organism of
trichomoniasis, is a single-celled protozoan parasite with a predilection for
infecting the vagina. The organism can be identified with intra-office wet
mount preparations, cultured, or recognized on Pap smears (although this
should be confirmed by culture, as the Pap smear is not a particularly reliable
method).
Balantidium coli(choice A) is a ciliated protozoan that can affect the gut.
Candida albicans(choice B) is a spore- and hyphae-forming fungus that can
affect the vagina, often producing a discharge with a white, "cheesy"
appearance.
Entamoeba histolytica(choice C) is an amoeba that can infect the gut.
Giardia lamblia(choice D) is a flagellated parasite that can infect the gut.


Most cases of infection with this organism are acquired by which of the following
routes?

/ A. Contaminated blood products
/ B. Contaminated fomites
/ C. FecaI-oral route
/ D. Sexual transmission
/ E. Spread from skin colonization



Explanation - Q: 1.2 Close

The correct answer is D. Trichomoniasis is commonly spread by sexual
transmission, with both heterosexual spread and female homosexual spread
being effective routes of transmission. Infected males are commonly
asymptomatic; infected females may either be asymptomatic, or have the
symptoms noted in the question stem. The other methods of spread noted in
the choices do not usually occur.


Which of the following medications is most often used to treat this woman's
condition?

/ A. Ampicillin
/ B. Cephalosporins
/ C. Erythromycin
/ D. Metronidazole
/ E. Tetracycline



Explanation - Q: 1.3 Close

The correct answer is D. Metronidazole is the only agent known to be
effective against trichomoniasis. Often, symptomatic women (and their
asymptomatic partners as well) can be treated with a single oral dose, but
highly resistant strains are also emerging that require treatment with high
dose IV or topical metronidazole. All the agents in the other choices are
ineffective.

If this woman had been pregnant and had not been treated, she and/or her baby
would be at most significantly increased risk of which of the following?

/ A. Congenital cardiac malformation
/ B. Congenital nervous system malformation
/ C. Hydronephrosis
/ D. Preeclampsia
/ E. Premature rupture of membranes



Explanation - Q: 1.4 Close

The correct answer is E. Pregnant women with trichomoniasis can be
safely treated with metronidazole, and this should be done to prevent the
increased risk of premature rupture of membranes and preterm delivery.
Other complications of trichomoniasis not limited to pregnant women are
uncommon, but may include cuff cellulitis following hysterectomy and
emphysematous blebs in the vaginal wall.
Trichomoniasis does not predispose for congenital malformations (choices A
and B) or the maternal complications of pregnancy of hydronephrosis
(choice C) and preeclampsia (choice D).
A woman diagnosed with this disease should also be evaluated for which of the
following?

/ A. Functional bowel dysfunction
/ B. Hypertension
/ C. Other venereal disease
/ D. Ovarian malfunction
/ E. Pyelonephritis



Explanation - Q: 1.5 Close

The correct answer is C. The same sexual encounters that spread
trichomoniasis can also spread a wide variety of other sexually transmitted
disease, including herpes, human papilloma virus, syphilis, gonorrhea,
chlamydia, and AIDS. Additionally, the lesions produced by the
trichomoniasis particularly facilitate the spread of the AIDS virus, since the
normal mucosal barrier is breached. The woman's chance of transmitting
AIDS to her partner is also increased by the presence of the trichomoniasis
open sores. The other choices list distracters unrelated to trichomoniasis.




Vignette 2 of 6

A 32-year-old woman comes to the physician because of a vaginal discharge and
itching and discomfort in her genital area. She states that the symptoms started
about 3 days ago. Since that time, she has noted a progressive worsening.
She also complains of dyspareunia and dysuria. She has no significant past
medical history. Her past surgical history is significant for an appendectomy at
the age of 17. She takes no medications and is allergic to penicillin. Pelvic
examination demonstrates marked erythema and mild edema of the vulva with a
few excoriations of the vulva. A vaginal discharge is seen, which is white, thick,
and clumpy with a cottage cheese appearance. The vaginal pH is 4.5. A sample
of the vaginal discharge is placed on a slide and treated with 10% potassium
hydroxide. Microscopy reveals lysis of normal cellular elements with branching
pseudohyphae and buds.


Which of the following is the most likely diagnosis?

/ A. Bacterial vaginosis
/ B. Candidiasis
/ C. Genital herpes
/ D. Lichen sclerosis
/ E. Trichomoniasis


Explanation - Q: 2.1 Close

The correct answer is B. This patient's presentation is most consistent with
the diagnosis of candidiasis. Candidiasis is most commonly caused by the
organism Candida albicans; although up to nine different fungi have been
shown to cause mycotic vulvitis. Itching and irritation are the most common
presenting symptoms of patients with candidiasis. The irritation can be
worsened by urination or with intercourse, with resulting dysuria and
dyspareunia. Patients with candidiasis also frequently complain of a vaginal
discharge that is often described as "cottage cheesy" in its appearance.
Examination typically reveals erythema of the vulva or vagina, often with
some edema. Excoriations from the patient's scratching are also often seen.
The discharge is usually thick, white, and clumpy, although occasionally it
can be thin and watery. The clinical diagnosis can be confirmed by
performing 10% potassium hydroxide (KOH) microscopy, which
demonstrates the branching pseudohyphae and buds of the Candida. The
KOH causes lysis of normal cellular elements, which allows the buds and
pseudohyphae to be more easily visualized.
Bacterial vaginosis (choice A) is characterized by a malodorous vaginal
discharge. Examination shows a discharge coating the vaginal walls, clue
cells on microscopy, and a vaginal pH greater than 4.5.
Genital herpes (choice C) is characterized by painful lesions that begin as
fluid-filled papules or vesicles and progress to coalescent, shallow-based
ulcers. An initial herpes infection is associated with systemic symptoms such
as fever, myalgias, and malaise. Recurrent infections usually occur without
systemic symptoms.
Lichen sclerosis (choice D) is a vulvar dystrophy in which the vulva develops
an atrophic appearance that is often described as cigarette-paper or
parchment-like. It usually occurs in postmenopausal women, though it can
sometimes be seen in the pediatric age group.
Trichomoniasis (choice E) is characterized by a malodorous vaginal
discharge that is usually described as being greenish or yellow-green.
Microscopic evaluation demonstrates the motile organism, Trichomonas
vaginalis.


Which of the following is the most likely pathogen?



Explanation - Q: 2.2 Close

The correct answer is A. Candida albicans belongs to the family of
opportunistic mycoses and it is a common cause of vulvovaginitis in women.
It is a normal inhabitant of the GI tract, the upper respiratory tract, and the
female genital tract. However, when conditions are "right," candidal
overgrowth can result. Patients that are particularly predisposed to candidal
infections are those who are immunosuppressed and diabetics. Other
affected groups include patients on antibiotics, and certain women who are
prone to the disorder for unclear reasons. Along with vulvovaginitis, Candida
albicans is also associated with thrush, intertrigo, and esophagitis.
Occasionally, the organism can cause pneumonia, meningitis, and enteritis,
and it is also associated with endocarditis in IV drug abusers.
Gardnerella vaginalis(choice B) is a facultatively anaerobic, oxidase- and
catalase-negative, gram-variable rod. It is a normal inhabitant of the human
vagina and can be found in up to 70% of asymptomatic women. Overgrowth
of the organism leads to the condition known as bacterial vaginosis.
Herpes simplex virus 2 (choice C) is a double-stranded DNA virus that is a
member of the herpesviruses (which also includes HSV-1, varicella-zoster
virus, cytomegalovirus, Epstein-Barr virus, Human Herpesvirus 6 and 8.) It
most commonly causes genital infection in women. This patient's symptoms
and clinical exam findings are not consistent with genital herpes.
The postmenopausal estrogen decrease (choice D) that occurs in women
can cause a condition known as atrophic vaginitis. This condition is
characterized by dryness and irritation of the vagina and is a result of the
decline in estrogen that occurs after the menopause. This patient is
premenopausal and has candidiasis.
Trichomonas vaginalis(choice E) is a parasite that causes trichomoniasis in
women. A wet mount of vaginal discharge from a patient with this infection
will show motile, flagellated trophozoites.


This patient is started on miconazole. This medication works via which of the
following mechanisms?

/ A. Binding to bacterial ribosomes
/ B. Inhibition of DNA synthesis after conversion of the drug
/ C. Inhibition of mycolic acid transfer to the cell wall
/ D. Inhibition of the synthesis of ergosterol
/ E. Inhibition of the synthesis of mycolic acids



Explanation - Q: 2.3 Close
The correct answer is D. Miconazole is an antifungal agent that inhibits
ergosterol synthesis, thus disrupting cell wall synthesis. It is used as a topical
agent for infections with Candida and infections with mixed fungi. Miconazole
can also be used intravenously to treat systemic fungal infections. When
miconazole is used topically, the main toxicity is localized irritation and
burning. When used intravenously, miconazole can cause thrombophlebitis
and, rarely, cardiorespiratory arrest.
Numerous antibiotics function by binding to bacterial ribosomes (choice A).
For example, the aminoglycosides (e.g., gentamicin), tetracyclines (e.g.,
doxycycline), and macrolide antibiotics (e.g., erythromycin) all bind to
bacterial ribosomes. Miconazole does not function via this mechanism.
Acyclovir brings about inhibition of DNA synthesis after conversion of the
drug (choice B). Viral thymidine kinase converts acyclovir to a nucleoside
analog that then inhibits DNA synthesis.
Ethambutol is an antitubercular agent that functions through inhibition of
mycolic acid transfer to the cell wall (choice C).
Isoniazid (INH) blocks the formation of the mycobacterial cell wall through
inhibition of the synthesis of mycolic acids (choice E). It is used in the
treatment of tuberculosis, though resistance of the organism Mycobacterium
tuberculosis to INH is increasing.

The KOH wet preparation is falsely negative in 25% of cases of vulvovaginal
candidiasis. Which of the following represents the sensitivity of the KOH wet
preparation for identifying candidiasis?

/ A. 10%
/ B. 25%
/ C. 50%
/ D. 75%
/ E. 99%


Explanation - Q: 2.4 Close

The correct answer is D. The sensitivity of a test represents the percentage
of individuals with a disease who test positive with the diagnostic test. If the
potassium hydroxide (KOH) wet preparation is falsely negative in 25% of
cases, this means that 25 out of every 100 patients who have candidiasis will
have a negative wet preparation. This means that 75 of every 100 patients
with candidiasis will test positive by the wet preparation. And this 75 per 100
or 75% represents the sensitivity of the test. It is important to realize that the
sensitivity and specificity of a test are qualities of the test and are not
changed by the nature of the population being tested. Positive and negative
predictive values, on the other hand, depend on the prevalence of the
disease in the population being tested.
Vignette 3 of 6


A 28-year-old woman comes to the physician because of an increased vaginal
discharge that has an unpleasant odor.
She states that the symptoms started 2 days ago and have been worsening
since. Other than the malodorous discharge, she has no other complaints. Past
medical history is significant for occasional migraine headaches. Past surgical
history is significant for tonsillectomy as a child. She takes no medications
regularly and has no known drug allergies. Speculum examination demonstrates
a homogeneous, grayish-white discharge, partially adherent to the vaginal walls.
There is no edema or erythema of the cervix, vagina, or vulva. A drop of the
vaginal discharge is tested for pH, which is 4.9. When potassium hydroxide
(KOH) is added to the discharge, there is an intense amine odor. A normal saline
wet preparation of the discharge reveals a predominance of clue cells.


Which of the following is the most likely diagnosis?

/ A. Bacterial vaginosis
/ B. Candidiasis
/ C. Chlamydia
/ D. Gonorrhea
/ E. Trichomoniasis



Explanation - Q: 3.1 Close

The correct answer is A. Bacterial vaginosis has been referred to by many
other designations over the years including Gardnerella-associated vaginitis
and nonspecific vaginitis. It is the number one cause of vaginitis in
premenopausal women. Patients with this condition most commonly
complain of an increased vaginal discharge that is malodorous. The odor will
often be described as musty or fishy. However, up to 50% of patients with
bacterial vaginosis will be asymptomatic. Examination of a patient with
bacterial vaginosis will demonstrate a homogeneous vaginal discharge that
is white or grayish-white. The vaginal pH will be greater than 4.5, which is
more basic than the usual vaginal pH. Clue cells, which are epithelial cells
covered by Gardnerella vaginalis, will be predominant on the normal saline
wet preparation. Finally, the discharge will have a fishy odor, either before or
after the application of 10% KOH to the sample. This is known as a positive
"whiff test." Treatment is with metronidazole or clindamycin. Metronidazole
should be avoided in the first trimester of pregnancy.
Patients with candidiasis (choice B) most commonly complain of a thick,
"cottage-cheese" vaginal discharge. They also note itching and redness in
the vulvovaginal area and may complain of dyspareunia and dysuria.
Examination demonstrates the thick, white discharge, with erythema of the
vulva and vagina, and pseudohyphae on the KOH microscopic preparation.
Patients with Chlamydia(choice C) are often asymptomatic or have mild
symptoms that go unrecognized. A cervicitis is sometimes seen on physical
examination, and diagnosis is made most often with the use of DNA probe
systems.
Patients with gonorrhea (choice D) also are often asymptomatic or have only
mild symptoms. Endocervical gonorrhea is associated with a purulent vaginal
discharge. Diagnosis is usually made by culture of the organism or with DNA
probe systems.
Trichomoniasis (choice E) is diagnosed when the motile organism is
visualized on the normal saline wet preparation. This patient has a
predominance of clue cells on the wet preparation, which is consistent with
bacterial vaginosis and not trichomoniasis.


Which of the following is the most likely pathogen?



Explanation - Q: 3.2 Close

The correct answer is C. Gardnerella vaginalis is a facultatively anaerobic,
oxidase- and catalase-negative, gram-variable rod; that is, it has a cell wall
or a laminated cell wall typical of neither gram-positive nor gram-negative
bacteria. However, there is much uncertainty surrounding the exact
pathophysiology that underlies the development of bacterial vaginosis in
women. Currently, it is thought that the infection results from the synergism
between a number of bacteria including Gardnerella vaginalis, Mycoplasma
hominis, Ureaplasma urealyticum, Peptostreptococcus species, and
anaerobic gram-negative rods. When a critical concentration of certain
vaginal bacteria is present, particularly the anaerobes, Gardnerella vaginalis
is able to "overgrow" in the vagina, replace the native Lactobacillus species,
and cause symptoms. On the normal saline wet preparation, Gardnerella
vaginalis can be seen adherent to the epithelial cells.
Candida albicans(choice A) is a fungus that is the most common cause of
candidiasis. It normally colonizes the human gastrointestinal tract and
vagina, but overgrowth can lead to significant symptoms and severe
infection. Along with causing candidiasis, it can also cause endocarditis,
pneumonitis, and meningitis.
Chlamydia trachomatis(choice B) is an obligate intracellular organism that is
most commonly associated with cervicitis in women. It is also contributory in
many cases of pelvic inflammatory disease. It can also cause pharyngitis,
pneumonitis, and conjunctivitis.
Neisseria gonorrhoeae(choice D) is also known as the gonococcus. It
appears on Gram's staining as gram-negative diplococci. It can cause
cervicitis, urethritis, meningitis, pharyngitis, arthritis, endocarditis, and
ophthalmia. It is not implicated in bacterial vaginosis.
Trichomonas vaginalis(choice E) is a protozoan that moves via flagellae.
Diagnosis is made when motile trophozoites are seen on microscopic
examination of a vaginal discharge. The treatment of choice is
metronidazole.



The patient is started on metronidazole. This medication works via which of the
following mechanisms?

/ A. Binds 50s subunit of bacterial ribosome and inhibits translocation of the
peptide chain from the A site to the P site
/ B. BIocks cross-Iinking of the N-acetyl muramic acid/N-acetyl glucosamine
backbone by binding to D-ala-D-ala
/ C. Inhibits DNA-dependent RNA polymerase by binding to the subunit and
inhibiting initiation
/ D. Interferes with DNA gyrase activity and prevents the winding of DNA helix
into the supercoiled form
/ E. Is reduced to a substance that inhibits cellular DNA synthesis



Explanation - Q: 3.3 Close

The correct answer is E. Metronidazole is reduced to a substance that
inhibits cellular DNA synthesis. Metronidazole is commonly known as Flagyl
and is used to treat infections with anaerobic bacteria other than
Actinomyces. It is also used to treat anaerobic protozoa including Giardia
lamblia, Trichomonas vaginalis, and Entamoeba histolytica. It can be
administered both intravenously and orally, and has excellent bioavailability
when given orally. Metronidazole can increase the anticoagulant effect of
drugs in the coumarin class, and can cause a disulfiram-like reaction in
patients ingesting alcohol. Patients started on metronidazole should be
cautioned not to drink. Another common side effect is a metallic taste in the
mouth. Peripheral neuropathy, seizures, and ataxia have been seen with
prolonged use.
Binding to the 50s subunit of bacterial ribosome and inhibiting translocation
of peptide chain from A site to P site (choice A) describes the action of
chloramphenicol.
Blocking the cross-linking of the N-acetyl muramic acid-N-acetyl glucosamine
backbone by binding to D-ala-D-ala (choice B) describes the mechanism of
action of vancomycin.
Inhibiting the DNA-dependent RNA polymerase by binding to the subunit
and inhibiting initiation (choice C) describes the mechanism of action of the
rifamycins, such as rifampin.
Interfering with DNA gyrase activity and preventing the winding of the DNA
helix into the supercoiled form (choice D) describes the mechanism of action
of the quinolones.


Which of the following is the most commonly reported adverse response to
metronidazole?

/ A. Discoloration of teeth in children
/ B. Gastrointestinal distress
/ C. Hemolytic anemia in patients with G6PD deficiency
/ D. Pseudomembranous colitis
/ E. "Red man" syndrome



Explanation - Q: 3.4 Close

The correct answer is B. Metronidazole is associated with a large number
of side effects and adverse reactions. The most commonly reported side
effect is gastrointestinal distress. Other adverse reactions that can occur
include convulsions, insomnia, peripheral neuropathy, and thrombophlebitis.
Perhaps the most "interesting" side effect (or most frequently discussed) is
the disulfiram-like reaction that alcohol consumption can lead to in patients
taking the drug. Metronidazole has also been shown, in therapeutic doses, to
be carcinogenic in mice and rats.
Discoloration of teeth in children (choice A) is an adverse effect that is
associated with tetracycline use. This drug should, therefore, not be used in
pregnant women or children less than age 8.
Hemolytic anemia in patients with G6PD deficiency (choice C) is an adverse
effect associated with primaquine. Primaquine is a drug that is used to treat
malaria.
Pseudomembranous colitis (choice D) has been reported in association with
amoxicillin and cephalosporins, but it is most often associated with
clindamycin use. In the setting of these antibiotics, Clostridium difficile
overgrows and causes the colitis.
"Red man" syndrome (choice E) is associated with vancomycin and occurs
when the medication is given too rapidly intravenously. Vancomycin can also
cause ototoxicity.

To study the efficacy of metronidazole for the treatment of this disease, studies
are conducted in which the subjects are divided into 2 groups. One group
receives metronidazole and the other group receives a placebo. Two weeks later
the subjects are tested for bacterial vaginosis. Which of the following terms best
describes these studies?

/ A. Case-control studies
/ B. Case series
/ C. Cohort studies
/ D. Cross-sectional studies
/ E. Randomized controlled trials



Explanation - Q: 3.5 Close

The correct answer is E. A randomized controlled trial is an experimental
clinical trial in which a group is randomly divided into two separate groups.
One group is given the experimental treatment (in this case metronidazole)
and the other group (the control group) is given a placebo or "standard"
medication. These trials can be single-blinded so that the subjects do not
know which medication they are receiving, or double-blinded so that neither
the patients, nor the researchers know. These studies are effective at
reducing bias, but they are often difficult to coordinate.
Case-control studies (choice A) are performed by looking at a group of
patients with a disease (the cases) and a group without the disease (the
controls) and comparing exposures that the two groups had. These studies
are usually easier to perform than experimental studies but can be
confounded by numerous biases.
Case series (choice B) are reports on a set of patients. For example, a case
series involving metronidazole and bacterial vaginosis might report on 10
consecutive patients with HIV and bacterial vaginosis treated with
metronidazole and describe the outcomes.
Cohort studies (choice C) are prospective observational studies that follow a
population over time to look at outcomes and exposures. Cohort studies
allow a relative risk to be calculated among different populations.
Cross-sectional studies (choice D) look at a group of subjects at one point in
time. They are often easier to perform than prospective studies, but often
provide less information.
Vignette 4 of 6


A 55-year-old woman complains to a physician because she has been having
chronic pelvic pressure. On further questioning, she reports chronic lower back
pain, constipation, difficulty with walking, and impaired coitus. Pelvic examination
demonstrates that the uterine cervix lies low within the vaginal canaI, but does
not protrude through the introitus.


Which of the following is the most likely diagnosis?

/ A. Cystocele
/ B. Femoral hernia
/ C. Rectocele
/ D. Sarcoma botyroides
/ E. Uterine prolapse



Explanation - Q: 4.1 Close

The correct answer E. Uterine prolapse is the term used for when the
uterus descends from its normal position to one lower in the body. The
prolapse is graded based on the level of descent. This woman has first
degree prolapse, with descent of the cervix into the vagina. Second degree
prolapse is characterized by protrusion of the cervix through the introitus.
Third degree prolapse is characterized by complete eversion of the vagina.
Uterine prolapse, rectocele, and cystocele are usually due to problems with
the strength of the muscles forming the pelvic floor, and the lesions are
classified as pelvic relaxation disorders. A history of trauma (notably related
to multiple pregnancies) and hormonal changes (e.g., menopause) is
frequently elicited.
Cystocele (choice A) refers to the descent of the bladder to a lower than
normal position. It is usually due to problems with the strength of the muscles
forming the pelvic floor, and the lesions are classified as pelvic relaxation
disorders. A history of trauma (notably related to multiple pregnancies) and
hormonal changes (e.g., menopause) is frequently elicited.
A femoral hernia (choice B) would present with a mass in the upper part of
the thigh, adjacent to the groin.
Rectocele (choice C) refers to descent of the rectum to a lower than normal
position.
Sarcoma botyroides (choice D) is a form of embryonal rhabdomyosarcoma
that can present with polypoid masses protruding from the vagina, usually in
children under 5 years of age.

Uterine prolapse, rectocele, and cystocele are all usually due to which of the
following?

/ A. CNS dysfunction
/ B. Infection
/ C. Neoplasia
/ D. Pelvic inflammatory disease
/ E. Pelvic relaxation disorder



Explanation - Q: 4.2 Close

The correct answer is E. All three of lesions are usually due to problems
with the strength of the muscles forming the pelvic floor, and the lesions are
classified as pelvic relaxation disorders. A history of trauma (notably related
to multiple pregnancies) and hormonal changes (e.g., menopause) is
frequently elicited. Only rarely is the change in organ position related to other
disease (choices A, B, C, and D).


Which of the following is the largest and most important muscle in the pelvic
floor?

/ A. Coccygeus
/ B. External anal sphincter
/ C. Levator ani
/ D. Obturator internus
/ E. Piriformis muscles



Explanation - Q: 4.3 Close

The correct answer is C. The pelvic floor is formed by the large levator ani
(with parts including the pubococcygeus, puborectalis, and iliococcygeus)
and the much smaller coccygeus (choice A).
The small external anal sphincter (choice B) lies below the levator ani and is
not technically part of the pelvic floor.
The obturator internus (choice D) is in the lateral wall of the pelvis, and the
piriformis muscles (choice E) are in the posterior wall of the pelvis.

During childbirth, which of the following muscles is most often injured by a tear of
the perineum?
/ A. Coccygeus
/ B. IIiococcygeus
/ C. Pubococcygeus
/ D. Puborectalis
/ E. Superficial transverse perineal

Explanation - Q: 4.4 Close

The correct answer is C. The fetal head is supported during childbirth by
the pelvic floor. If injury occurs during the passage of the child through the
dilated uterine cervix, this injury most frequently involves a tear of the
perineum between the vagina and anus. This tear usually involves the
pubococcygeus, which is the main part of the levator ani. The
pubococcygeus runs from the pubic bone anteriorly to the coccyx posteriorly,
and contains an anterior opening for the urethra and vagina and a posterior
opening for the rectum. Tears to this muscle that heal poorly may
consequently predispose for uterine prolapse, cystocele, and rectocele.
The coccygeus (choice A) is a relatively small muscle of the pelvic floor that
runs from the ischial spine to the inferior end of the sacrum.
The iliococcygeus (choice B) is another part of the levator ani, and runs from
the iliac bone to the coccyx, forming part of the more lateral aspects of the
pelvic floor.
The puborectalis (choice D) is the third part of the levator ani, and runs from
pubic bone to pubic bone, forming a sling-like structure around the wall of the
anal canal.
The superficial transverse perineal muscle (choice E) is a usually very small
muscle that runs laterally to medially from the ischial tuberosity to the
perineal body (between the vagina and anus).


The muscle that is most often injured by a tear of the perineum is innervated by
which of the following?

/ A. Inferior gluteal nerve
/ B. Pelvic splanchnic nerve
/ C. Posterior femoral cutaneous nerve
/ D. Pudendal nerve
/ E. Superior gluteal nerve



Explanation - Q: 4.5 Close

The correct answer is D. The pudendal nerve (from S2-S4) is the principal
nerve to innervate structures of the perineum, including sensory innervation
to the genitalia, and motor innervation to muscles of the perineum, the
external urethral sphincter, and the external anal sphincter. This innervation
may have clinical significance, as babies can also have uterine prolapse,
which can be due either to congenital weakness in the pelvic musculature or
to defects in innervation.
The inferior gluteal nerve (choice A) supplies the gluteus maximus.
The pelvic splanchnic nerve (choice B) supplies the pelvic viscera via the
inferior hypogastric and pelvic plexuses.
The posterior femoral cutaneous nerve (choice C) supplies the skin of the
buttock and upper portions of the medial and posterior aspects of the thigh.
The superior gluteal nerve (choice E) supplies the gluteus medius and
gluteus minimus muscles.


For minor degrees of this patient's disorder, Kegel exercises are sometimes
prescribed. These can be performed by asking the patient to do which of the
following?

/ A. CIose the glottis and increase abdominal pressure
/ B. Contract muscles to interrupt a stream of flowing urine
/ C. Press the knees together
/ D. Tense the buttocks
/ E. When in a sitting position, press the knees upward against the hands



Explanation - Q: 4.6 Close

The correct answer is B. The Kegel exercises are specifically designed to
contract the pubococcygeus muscle, and can be performed by interrupting
the flow of urine. Patients with minor degrees of prolapse may also be
treated with a pessary, which is a small object placed in the vagina that
serves as a mechanical support for the uterus. Estrogen therapy is also of
some help in some patients. Severe degrees of prolapse may require either
hysterectomy or surgical strengthening of the pelvic floor. Choice A
describes the Valsalva maneuver, which these patients should not do,
because it increases intra-abdominal pressure and may worsen the prolapse.
(They should also avoid lifting heavy objects for the same reason.) The other
answers are distracters.




Vignette 5 of 6


A 15-year-old girl is seen by gynecologist and a pelvic examination is performed.
The physician begins by doing a manual palpation of the pelvic organs with one
hand while fingers of the other hand are in the vagina. During the pelvic
examination, a speculum is inserted into the vagina and the cervix is observed.
An irregular, roughly circular line is noted to be surrounding the cervical os. This
corresponds to the histological junction between which of the following?

/ A. The pseudo-stratified columnar epithelium of the ectocervix and the
stratified squamous epithelium of the endocervix
/ B. The simple columnar epithelium of the ectocervix and the simple squamous
epithelium of the endocervix
/ C. The simple squamous epithelium of the ectocervix and the pseudo-stratified
epithelium of the endocervix
/ D. The stratified squamous epithelium of the ectocervix and the simple
columnar epithelium of the endocervix
/ E. The stratified squamous epithelium of the ectocervix and the simple
squamous epithelium of the endocervix




Explanation - Q: 5.1 Close

The correct answer is D. The line in question is called the Z-line, and marks
the boundary between the stratified squamous epithelium of the ectocervix
and the simple columnar epithelium of the endocervix. This is also
sometimes called the transition or transformation zone, and is clinically
significant because this junctional area is a common site for development of
squamous cell carcinoma. The Z line is not fixed for life, but tends to start out
in teenage girls clearly visible on the ectocervix, and then migrate upward
into the (sometimes deep) endocervix as the girls age. For this reason,
women in their 30's and 40's may have high grade dysplasia and even
cervical cancer without any lesion being apparent on vaginal examination.


The physician takes a sample for Pap smear analysis. Review of the Pap smear
demonstrates the presence of koilocytes, hich are approximately the same size
as an intermediate celI, with enlarged hyperchromatic nuclei and perinuclear
halos.
If no other significantly abnormal cells are present, this Pap smear would be
classified as which of the following?

/ A. Carcinoma in situ
/ B. High-grade squamous intraepithelial lesion (CIN II-III)
/ C. Invasive squamous cell carcinoma
/ D. Low-grade squamous intraepithelial lesion (CIN I)
/ E. Reactive change






Explanation - Q: 5.2 Close

The correct answer is D. The cells described are typical for low-grade
squamous intra-epithelial lesion (LSIL), which corresponds in the tissue
section diagnosis of cervical intra-epithelial lesion grade I (CIN I).
Carcinoma in situ (choice A) and the higher grade range of high-grade
squamous intraepithelial lesions (HSIL; corresponding to CIN III) are
approximately the same diagnosis.
High-grade squamous intraepithelial lesions (HSIL, choice B) have smaller
cells with hyperchromatic nuclei, and correspond to the tissue section
diagnoses of CIN II and CIN III.
Descriptions of invasive squamous cell carcinoma (choice C) should include
references to both obviously malignant cells, and to the presence of a "tumor
background" with blood, cellular debris, and heavy inflammation.
Reactive change (choice E) is characterized by inflammatory cells and
epithelial cells with large, vesicular nuclei that are not hyperchromatic.


The abnormal cells observed in the Pap smear specimen are infected by which
of the following viruses?

/ A. AIDS virus
/ B. Cytomegalovirus
/ C. Herpes simplex l
/ D. Herpes simplex ll
/ E. Human papilloma virus



Explanation - Q: 5.3 Close

The correct answer is E. Various strains of the human papilloma virus
(HPV, notably types 16, 18, 31, 33, 35, and 39) have been implicated as
causing a variety of genital lesions, including genital warts, cervical
dysplasia, cervical cancer, vaginal dysplasia, vaginal cancer, and penile
cancer. (Strains of human papilloma virus have also been implicated in the
common wart found often found on fingers and in laryngeal warts.) The
koilocyte is the term used to describe the characteristic nuclear alterations
and perinuclear halo seen with HPV infection in the cervical squamous
epithelial cells.
Infection with the AIDS virus (choice A) cannot be recognized by cytologic
changes.
Cytomegalovirus (choice B) infection causes nuclei to be large, eosinophilic,
and "smudged"; cytomegalovirus is uncommonly seen in Pap smears.
Herpes simplex I and II (choice C and D) infections cannot be distinguished
cytologically, as both produce multinucleated cells whose nuclei develop a
characteristic "ground-glass" appearance.


The gynecologist decides to simply monitor the patient; however, she is soon lost
to follow-up. The patient does not seek gynecological attention again until the
age of 26. At that time, she goes to an emergency department because she has
been feeling chronic pelvic "heaviness" for several months, and has had more or
less continuous vaginal spotting. Pelvic examination demonstrates a 2.5 cm
ulcerated area involving the region of the cervix near the cervical os. Pap smear
taken from this area shows small neoplastic cells with enlarged hyperchromatic
nuclei (that are occasionally very large and bizarre) and brightly eosinophilic
cytoplasm. AIso noted on the smear are the presence of abundant blood,
neutrophils, and cellular debris. The cells that are seen are most likely derived
from which of the following?

/ A. Endocervical adenocarcinoma
/ B. Endometrial adenocarcinoma
/ C. Leiomyoma
/ D. Leiomyosarcoma
/ E. Squamous cell carcinoma



Explanation - Q: 5.4 Close

The correct answer is E. The description of the Pap smear is typical for
invasive squamous cell carcinoma. We are unfortunately seeing potentially
fatal cervical cancer in young women, who have typically begun having
sexual intercourse at a young age. The time between the initial HPV infection
and the development of cancer may be as short as 8 years, which means
that HPV infection may, in extreme cases, be more rapidly fatal than HIV
infection. On a related note, medical educators tend to teach students that
"cancer isn't contagious," which is true in general, but in the case of the HPV-
related genital cancers, the underlying infection leading to the cancer
certainly is contagious.
Adenocarcinomas of either the endocervix or endometrium (choices A and
B) shed cells that have clear, rather than orange, cytoplasm and resemble
the cells of normal columnar epithelium.
Benign leiomyomas (choice C) and malignant leiomyosarcomas (choice D)
are tumors that usually form in the muscle of the uterine wall, and do not
usually shed cells that can be reached with a Pap smear.



Further studies demonstrate that this young woman's cancer extends beyond the
cervix but not to the pelvic wall and involves the upper but not lower portion of
the vagina. This tumor has which of the following stages?

/ A. Stage 0
/ B. Stage l
/ C. Stage ll
/ D. Stage lll
/ E. Stage IV



Explanation - Q: 5.5 Close

The correct answer is C. This patient's tumor is in clinical stage II. Stage 0
(choice A) is used for carcinoma in situ. Stage I (choice B) is carcinoma
confined to the cervix. Stage II is carcinoma extending beyond the cervix but
not to the pelvic wall; and carcinoma involving the vagina but not the lower
third of the vagina. Stage III (choice D) is carcinoma extending to the pelvic
wall, with cancer-free rectum, and with tumor involving the lower 1/3 of the
vagina. Stage IV (choice E) is cancer extending beyond the true pelvis or
involving the mucosa of the bladder or rectum. The prognosis of cervical
cancer is strongly dependent on the stage, with 90% of women with Stage I
cancer having an at least 5 year survival, but only 20% or fewer of women
with Stage IV cancer having a 5 year survival. Treatment depends on stage,
and may involve surgery, radiation, or both.



Vignette 6 of 6

A 28-year-old woman presents to her family physician complaining that she has a
"growth" in her genital area. She states that she first noticed it 3 weeks ago and it
seems to have grown somewhat since that time. It does not bother her except for
its presence. She has hypothyroidism, for which she takes thyroid hormone
replacement. She has no other medical problems and has never had surgery.
She is allergic to penicillin. She has had 3 sexual partners over her lifetime and is
currently involved in a monogamous relationship. She has never had a sexually
transmitted disease in the past. She smokes 1 pack of cigarettes per day.
Examination demonstrates two nontender 6 mm welI-circumscribed, flesh-
colored, apillated, oval lesions on the labia majora. There is no ulceration,
erythema, purulence, or inguinal lymphadenopathy.

Which of the following is the most likely diagnosis?

/ A. Chancre
/ B. Condyloma
/ C. Herpes
/ D. Squamous cell carcinoma
/ E. Syphilitic gummas

Explanation - Q: 6.1 Close

The correct answer is B. This patient, who presents with asymptomatic,
well-circumscribed, benign-appearing, flesh-colored lesions in the genital
area most likely has condyloma acuminatum, or genital warts. These lesions
result from contact with infected partners and represent a sexually
transmitted disease. Most of the time these lesions are benign, without
carcinogenic potential except if they are associated with the cervix, where
they are an important etiologic agent for cervical carcinoma.
Chancre (choice A) is characterized as a painless ulcer and is the primary
genital lesion of syphilis. This patient did not have an ulceration.
Herpes simplex lesions (choice C) are characterized by grouped vesicles on
an erythematous base. These lesions are usually symptomatic with a
prodrome of burning or irritation. This patient had an asymptomatic lesion
without vesicles, ulcerations, erosions, or erythema.
Squamous cell carcinoma (choice D) can present in this location, but usually
is characterized with erythema, erosion, or ulceration.
Syphilitic gummas (choice E) are manifestations of tertiary syphilis. They are
nontender, pink to dusky red nodules or plaques that vary in size from
millimeters to several centimeters, and favor sites of previous trauma, but
may appear anywhere. They are usually firm, but can develop a "gum"-like
consistency. This patient had no history of syphilis, and the clinical
presentation is not consistent with these destructive lesions.



Histological examination of diagnostic tissue from this patient would most likely
reveal which of the following?

/ A. Architectural atypia
/ B. Cytologic atypia
/ C. Dysplastic cells
/ D. Epidermal necrosis
/ E. Koilocytes



Explanation - Q: 6.2 Close

The correct answer is E. Koilocytes are large keratinocytes with an
eccentric, pyknotic nucleus surrounded by a perinuclear halo. They are
characteristic of human papilloma virus (HPV) infection. The koilocytotic cells
are usually abundant in condylomata. Anogenital warts also are
characterized by acanthosis and parakeratosis, and they lack a granular
layer.
Architectural atypia (choice A) is a histologic description suggesting
malignancy, which this patient does not have.
Cytologic atypia (choice B) is also a histologic description suggesting
malignancy, which this patient does not have.
Dysplastic cells (choice C) is a term used to describe atypical pigmented
cells, such as melanoma and dysplastic nevi. They usually have a large
nucleus and/or large size.
Epidermal necrosis (choice D) occurs in skin conditions such as toxic
epidermal necrolysis (TEN). This patient, with condyloma, would have an
intact epidermis without necrosis.

Which of the following is the most likely pathogen?

/ A. Epstein-Barr virus
/ B. Herpes simplex virus
/ C. Human papilloma virus
/ D. Treponema pallidum
/ E. Varicella zoster virus


Explanation - Q: 6.3 Close

The correct answer is C. Human papilloma virus (HPV) is associated with
condyloma acuminatum. HPV is a double-stranded DNA virus found in
human and other species. For the most part, these viruses induce subclinical
lesions that are very slow-growing and thus, may take many years before
obvious lesions become apparent. There are at least 60 different types of
HPV. Anogenital warts are mostly associated with types 6, 11, 16, 18, 31,
33-35, 39-40, and 51-60. Common warts and flat warts are associated with
different types of HPV.
Epstein Barr virus (choice A) is associated with infections mononucleosis
and Burkitt's lymphoma, among other diseases. The Monospot test is used to
diagnose this virus.
Herpes simplex virus (choice B) is associated with oral and anogenital
lesions. There are two types (I and II) that can be seen in either location.
These lesions are characterized as painful clustered vesicular eruptions. This
patient does not have the characteristic eruption of genital herpes.
Treponema pallidum(choice D) is the spirochete that causes syphilis. This
patient does not have the characteristic presentation for primary, secondary,
or tertiary syphilis.
Varicella zoster virus (choice E) is associated with chicken pox and shingles.
This patient does not have any signs of the pruritic vesicular rash associated
with this virus.
This patient is treated with imiquimod. This medication works via which of the
following mechanisms?

/ A. As a bacteriostatic agent
/ B. As a chemotherapeutic agent
/ C. As an antiviral agent
/ D. Through physical destruction of infected cells
/ E. Through stimulation of release of proinflammatory cytokines



Explanation - Q: 6.4 Close

The correct answer is E. Imiquimod is a potent stimulator of
proinflammatory cytokines. It thus functions as an immunostimulant. It is a
relatively recently approved drug for the treatment of external warts. It has
been demonstrated in prospective, double-blind, placebo-controlled trials to
be safe and effective in the treatment of patients with external anogenital
warts. It is used as 5% cream, and can be used to 16 weeks depending upon
the individual patient's response to treatment.
A drug that functions as a bacteriostatic agent (choice A) would not be used
in the treatment of condyloma acuminatum, as this disease does not have a
bacterial etiology.
Drugs that function as chemotherapeutic agents (choice B) are widely used
for the treatment of condyloma. However, imiquimod is not a
chemotherapeutic agent. The most commonly used chemotherapeutic
agents used in the treatment of anogenital warts are podophyllin and
bleomycin.
Antiviral agents (choice C) such as acyclovir, are not effective in eradicating
the human papillomavirus that causes condyloma acuminatum.
Several modalities are used to treat condyloma through physical destruction
of infected cells (choice D). The most commonly used method of physical
destruction is with cryotherapy (liquid nitrogen.)



Vignette 1 of 3


A 13-year-old girl is evaluated by a pediatrician because she has been having
fainting and near-fainting spells at schooI, articularly during physical education
classes. These episodes are typically accompanied by dizziness and shortness
of breath, and are sometimes accompanied by chest pain. She is found to have a
number of abnormal findings on cardiac examination.

The first finding to catch the pediatrician's attention is a systolic ejection
crescendo-decrescendo murmur that is heard best between the apex and the left
sternal border. This murmur is most likely due to which of the following?

/ A. BIood flowing out of the left atrium and into the left ventricle
/ B. BIood flowing out of the left ventricle and into the aorta
/ C. BIood flowing out of the right atrium and into the right ventricle
/ D. BIood flowing out of the right ventricle and into the pulmonary artery
/ E. BIood flowing out of the right ventricle and into the right atrium



Explanation - Q: 1.1 Close

The correct answer is B. It can be helpful both clinically, and on
examinations, to get to the point that you can analyze the significance of a
cardiac murmur. In this case, the fact that the murmur is a systolic ejection
murmur indicates that it probably involves blood flow out of one of the
ventricles. This allows you to exclude choices A and C, which involve blood
flow out of the atria, which would occur during diastole. The fact that the
murmur is heard best between the apex and left sternal border indicates that
it involves the left ventricle, thereby excluding choices D and E, which
involve the right ventricle, and would be heard best along the right sternal
border. The murmur described involves the left ventricular outflow tract into
the aorta.


After hearing the murmur, the physician asks the patient to perform a Valsalva
maneuver. This maneuver causes the intensity of the systolic ejection murmur to
diminish. Which of the following describes the Valsalva maneuver?

/ A. Forced expiratory effort against a closed airway
/ B. Maintenance of the larynx briefly at the highest position in the neck by
voluntary muscular contraction
/ C. Mouth closed and nose held followed by swallowing
/ D. Pressure applied to cricoid cartilage
/ E. Thrusting a fist inward between the navel and costal margin




Explanation - Q: 1.2 Close

The correct answer is A. The Valsalva maneuver is a forced expiratory
effort against a closed airway. This increases the intrathoracic pressure, and
consequently increases the preload into the left side of the heart. This, in
turn, tends to diminish the pressure gradient across the left ventricular
outflow tract, and consequently diminishes the intensity of the ejection
murmur.
Choice B refers to the Mendelsohn maneuver, which is used as a
therapeutic technique for swallowing disorders.
Choice C refers to the Toynbee maneuver, which is used to open the
Eustachian tube.
Choice D refers to Selleck's maneuver, which is used to prevent
regurgitation during tracheal intubation.
Choice E refers to the Heimlich maneuver, which is used to expel a food
bolus from the airways.


Another murmur heard in this patient is a holosystolic murmur heard best at the
apex and in the left axilla. This would be most likely to be caused by which of the
following?

/ A. Combined mitral and tricuspid stenosis
/ B. Mitral regurgitation only
/ C. Mitral stenosis only
/ D. Tricuspid regurgitation only
/ E. Tricuspid stenosis only



Explanation - Q: 1.3 Close

The correct answer is B. The holosystolic murmur suggests that
regurgitation is occurring from a ventricle to an atrium during ventricular
contraction. This lets you exclude choices A, C, and E. The locations at
which the murmur is heard best favor the mitral valve over the tricuspid valve
(choice D), which would have been heard best at the lower left sternal
border. This murmur is due to systolic anterior motion of the mitral valve and
is related to a significant left ventricular outflow gradient.


Other findings that the physician notes include a double apical impulse, a double
carotid arterial pulse, and a jugular venous pulse that has a prominent "a" wave.
The "a" wave change is most immediately related to which of the following?

/ A. Diminished left ventricular compliance
/ B. Diminished right ventricular compliance
/ C. Increased compliance of both the right and left ventricles
/ D. Increased left ventricular compliance
/ E. Increased right ventricular compliance



Explanation - Q: 1.4 Close

The correct answer is B. The jugular venous pulse is best appreciated
when the patient is supine, with head slightly raised. The normal jugular
venous pulse has three recognizable upward waves, "a", "c", and "v". The "a"
wave is the first of these waves and is due to transmitted right atrial pressure
to the jugular veins during right atrial systole. It occurs just prior to LV
ejection (which can be recognized by simultaneous palpation of the carotid
pulse upstroke). An increased "a" wave can be due to decreased compliance
of the right ventricle, which inhibits right ventricular filling during atrial systole.
The state of the left ventricle's compliance is not directly measured with the
"a" wave, although in cases like this patient's, septal hypertrophy may affect
the compliance of both the right and left ventricles.


Two-dimensional echocardiography reveals systolic anterior motion of the
anterior mitral valve (which appears morphologically normaI) and asymmetric
septal hypertrophy. Which of the following is the most likely diagnosis?

/ A. Bacterial endocarditis
/ B. Congenital heart disease
/ C. Congestive heart failure
/ D. Hypertrophic cardiomyopathy
/ E. Rheumatic heart disease



Explanation - Q: 1.5 Close

The correct answer is D. This patient has hypertrophic cardiomyopathy,
which has been formerly known by a variety of names, including idiopathic
hypertrophic subaortic stenosis, and asymmetric septal hypertrophy. These
old terms have been replaced by the slightly broader term hypertrophic
cardiomyopathy, since it was discovered that the hypertrophy is not always
limited to the left ventricular outflow tract and septum. This patient's
presentation has many typical features, and the murmurs heard are often
present in these patients. Other presenting symptoms can include sudden
cardiac death, angina, palpitations, and congestive heart failure.
Hypertrophic cardiomyopathy may be either obstructive, as in this case, or
nonobstructive. In obstructive cases, anterior mitral valve movement toward
the septum during systole contributes to the pathophysiology by increasing
the level of obstruction over that present in diastole. This, in turn, causes
several of the cardiac findings noted earlier in this case, including the double
apical impulse and double carotid pulse (due to a "doubled" forceful ejection
of blood during systole before and after the obstruction increases by mitral
valve movement), the mitral regurgitation murmur, and the high flow systolic
ejection murmur in the left ventricular outlet.
Bacterial endocarditis (choice A) and rheumatic heart disease (choice E)
are primarily valvular diseases, and marked valvular abnormalities would be
seen on echocardiography.
Congenital heart disease of various types (choice B) would be obvious on
echocardiography, and features such as atrial or ventricular septal defects,
abnormally formed valves, or aberrant major blood vessel connections would
be easily apparent.
Congestive heart failure (choice C) can complicate many conditions,
including hypertrophic cardiomyopathy, but none of the findings illustrated in
the case specifically suggest that it is present.

In approximately 50% of cases, this patient's disease has a genetic basis, which
is of which of the following types?

/ A. Autosomal dominant
/ B. Autosomal recessive
/ C. Numerical abnormality of autosomes
/ D. Numerical abnormality of sex chromosomes
/ E. X-Iinked recessive



Explanation - Q: 1.6 Close

The correct answer is A. The genetics of hypertrophic cardiomyopathy is
typically autosomal dominant with variable penetrance and variable
expressivity. It is thought that many of the isolated cases may be new
autosomal dominant mutations. The autosomal dominant pattern of
inheritance is important, because it leads to the clinical suggestion that
immediate relatives of the patient be screened for hypertrophic
cardiomyopathy. This is particularly important now that medical (e.g.,
antiarrhythmia medication) and surgical (left ventricular myomectomy,
pacemaker implantation, implantable cardioverter defibrillator) methods of
treating this disease exist. The other choices are distracters.

Which of the following is the leading cause of sudden cardiac death in both
preadolescent and adolescent children?

/ A. Bacterial endocarditis
/ B. Congenital heart disease
/ C. Congestive heart failure
/ D. Hypertrophic cardiomyopathy
/ E. Rheumatic heart disease

Explanation - Q: 1.7 Close

The correct answer is D. Sudden death is death that occurs over a few
minute period and was medically unexpected. Hypertrophic cardiomyopathy
has a mortality rate of 4% per year overall, with a higher mortality rate of
sudden death in children of 6% per year. This makes this condition the
leading cause of sudden cardiac death in both preadolescent and adolescent
children. The death is often unexpected and is often associated with sports
or vigorous exertion. Sudden death can even be the presenting manifestation
of hypertrophic cardiomyopathy. It is thought that in most of these cases,
ventricular fibrillation is the arrhythmia that causes the death. Patients with
hypertrophic cardiomyopathy that does not cause ventricular outflow
obstruction may also die of arrhythmias. The other conditions listed in the
choices may also cause death, but it is usually not sudden.




Vignette 2 of 3


A 32-year-old woman consults a physician because she has been having
episodes of a sensation of motion accompanied by a ringing in her left ear. She
can tell when she is going to have an episode, roughly an hour before it happens
because she develops a sense of fullness in the ear. Once the symptoms of
motion become severe, she also experiences nausea, vomiting, diarrhea, pallor,
and sweating. The physician asks the patient to sit at the end of the examining
table. She is then asked to lay back quickly while the physician supports her
head and carefully hyperextends her neck. The physician then turns the head
from one shoulder to the other and then back again.


When the maneuver described is performed, the patient initially feels all right, but
after a few seconds delay, she experiences symptoms characteristics of her
vertigo spells. Which of the following is the most likely diagnosis?

/ A. Aspirin toxicity
/ B. Herpes zoster encephalitis
/ C. Meniere disease
/ D. Migraine headache
/ E. Transient ischemic attack



Explanation - Q: 2.1 Close

The correct answer is C. This patient most likely has Meniere disease. This
condition, which is also sometimes known as endolymphatic hydrops,
typically presents as illustrated in this case. The underlying problem is an
increase in volume and pressure of the fluid in both the vestibular and
cochlear apparatus. This dysfunction, in turn, causes nerve damage to both
the cochlear and vestibular branches of cranial nerve VIII, producing tinnitus
with hearing loss and vertigo, respectively. The other diseases listed in the
question choices are in the differential diagnosis of Meniere disease.
Aspirin toxicity (choice A) can cause tinnitus and hearing loss, but does not
usually cause vertigo.
Herpes zoster encephalitis (choice B), migraine (choice D), and transient
ischemic attacks (choice E) can all cause vertigo but do not usually cause
tinnitus.


Which of the following agents would be most likely to relieve her vertiginous
symptoms?

/ A. Apomorphine
/ B. Meclizine
/ C. Ondansetron
/ D. Pilocarpine
/ E. Promethazine



Explanation - Q: 2.2 Close

The correct answer is B. Meclizine (Antivert), an H1 antihistamine, has
antivertiginous properties and can be useful to patients with this disorder.
Other medications that can also help with the vertigo are scopolamine,
dimenhydrinate (Dramamine), and diazepam (Valium).
Apomorphine (choice A) is a dopamine agonist, which is used to induce
emesis in some poisoning cases.
Ondansetron (choice C) is a 5-HT3 antagonist, which is used as an
antiemetic. It would not be useful for vertigo.
Pilocarpine (choice D) is a muscarinic agonist and would likely exacerbate
this patient's vertigo.
Promethazine (choice E) is a dopamine antagonist (phenothiazine), which is
used as an antiemetic. It would not be useful for vertigo.




Which of the following is the most common morbidity associated with this
disease?

/ A. Chronic otitis externa
/ B. Chronic otitis media
/ C. Permanent hearing loss
/ D. Secondary development of a craniopharyngioma
/ E. Secondary development of an acoustic neuroma



Explanation - Q: 2.3 Close

The correct answer is C. Patients with Meniere disease often have
transient hearing loss in the affected ear(s) during the vertigo episodes. With
repeated episodes, the vertigo often improves, but the hearing loss can
become permanent. Patients with Meniere disease can usually be treated
with medication to reduce their vertigo (antihistamines such as meclizine,
anticholinergics such as scopolamine, benzodiazepines such as diazepam);
some cases with incapacitating vertigo are treated with surgery or ototoxic
drugs (such as high dose streptomycin).
Meniere disease is unrelated to middle or outer ear infection (choices A and
B).
Meniere disease is apparently unrelated to tumor (choices D and E,
although occasionally tumors involving the middle ear can mimic Meniere
disease).


Which of the following structures contains endolymph?

/ A. Helicotrema
/ B. Scala media
/ C. Scala tympani
/ D. Scala vestibuli
/ E. Spiral ganglion



Explanation - Q: 2.4 Close

The correct answer is B. The cochlea of the inner ear is a coiled structure
filled with endolymph and perilymph. Within it are three longitudinal
compartments. The membranous labyrinth within the cochlea is called the
scala media, or cochlear duct. This compartment contains the hair receptors
and is filled with endolymph that is secreted by the stria vascularis. The
composition of endolymph resembles intracellular fluid (K
+
> Na
+
).
The two perilymph-filled compartments, the scala tympani (choice C) and
the scala vestibuli (choice D) surround the cochlear duct. This space is
continuous with subarachnoid space, and therefore the perilymph closely
resembles cerebrospinal fluid. The helicotrema (choice A) is the apical
connection between the scala tympani and scala vestibuli.
The spiral ganglion (choice E) is located in the modiolus (the cochlear part
of the body labyrinth), and contains the cell bodies of the bipolar cells that
comprise the cochlear division of the eighth nerve.




Vignette 3 of 3

A fifty-year-old man presents to his primary care physician complaining that he
often feels as if the room is spinning when he gets up from a recumbent position
or turns his head. He has not lost consciousness and has had no chest pain. He
has no cardiac history and a recent treadmill test showed no abnormalities. On
examination, the sensation can be produced by rapidly turning the head. It can
be reproduced many times, but it eventually ceases. Nystagmus is elicited.
Hearing is normaI.


Which of the following is the most likely diagnosis?

/ A. Benign paroxysmal positional vertigo
/ B. Brain stem tumor
/ C. Meniere disease
/ D. Syncope
/ E. Vertebrobasilar TIA



Explanation - Q: 3.1 Close

The correct is answer A. Benign paroxysmal positional vertigo (BPPV) is
vertigo that is precipitated by head position. It can be precipitated by trauma,
but often no precipitating factor is identified. It generally abates after weeks
to months. On exam, patients display nystagmus and the symptoms can be
reproduced by head movement. In addition, the symptoms show latency,
fatigability and habituation.
Brain stem tumor (choice B) and vertebrobasilar TIA (choice E) are causes
of central causes of vertigo, but would often be associated with other
neurologic findings.
Meniere disease (choice C) displays the classic triad of unilateral tinnitus,
unilateral deafness, and paroxysmal vertigo. Since the patient has normal
hearing, this is unlikely to be Meniere syndrome, although hearing loss may
not manifest until later stages of the disease.
Syncope (choice D) is defined as transient loss of consciousness of
cardiovascular origin. Thus since the patient has a lack of other
cardiovascular symptoms and has not lost consciousness, this is not
syncope.


Which of the following is the most likely mechanism for this patient's symptoms?

/ A. Aberrant stimulation of hair cells
/ B. Hair cell death in the semicircular canals
/ C. Insufficient cardiac output
/ D. Insufficient cerebral perfusion
/ E. Massive infection in the middle ear



Explanation - Q: 3.2 Close

The correct answer is A. In benign paroxysmal positional vertigo, calcium
carbonate crystals called otoliths, which are usually fixed in a gelatinous
otolithic membrane, float freely in the endolymph. Usually, movement is
sensed by the movement of this heavy membrane as it stimulates hair-like
projections on sensory hair cells fixed to the membrane in the inner ear. In
BPPV, the loose crystals aberrantly stimulate the hair cells with certain head
movements. Thus the sensation of vertigo is produced.
Hair cell death (choice B) does not cause vertigo.
Insufficient cardiac output (choice C) and cerebral blood flow (choice D)
lead to syncope, not vertigo.
Massive infection in the middle ear (choice E) will produce ear pain and
fever. Only occasionally will it produce vestibular symptoms, as the vestibular
apparatus is in the inner ear.


Which of the following agents will help alleviate this patient's symptoms?

/ A. Amitriptyline
/ B. CIonidine
/ C. Digitalis
/ D. FIumazenil
/ E. Scopolamine



Explanation - Q: 3.3 Close

The correct answer is E. Scopolamine alleviates the symptoms of all types
of vertigo. It acts by antagonizing acetylcholine receptors.
Amitriptyline (choice A) is a tricyclic antidepressant and would not be used
to treat vertigo.
Clonidine (choice B) is an alpha 2 agonist used to treat hypertension. It
would not be used for vertigo.
Digitalis (choice C) is used as an inotrope or an antiarrhythmic, but the
patient's symptoms are not referable to cardiac disease. Thus this would not
be an appropriate agent for this patient.
Flumazenil (choice D) is a benzodiazepine antagonist used in
benzodiazepine overdose.


Which of the following structures is responsible for detecting rotational
acceleration?

/ A. Cochlea
/ B. Fovea centralis
/ C. Saccule
/ D. Semicircular canals
/ E. Utricle



Explanation - Q: 3.4 Close

The correct answer is D. The semicircular canals are responsible for
detecting rotational acceleration. As the head is rotated, fluid currents in the
endolymph cause the deflection of the cupula and stimulate its hair cells. The
magnitude of the fluid currents is proportional to the rotational acceleration,
and the direction is related to the direction of the acceleration.
The cochlea (choice A) is the inner ear structure in which the mechanical
energy of sound waves is transduced into neural impulses.
The fovea centralis (choice B) is the location on the retina responsible for
most color vision.
The saccule (choice C) and utricle (choice E) are inner ear structures that
detect linear acceleration.



Which of the following nerves transmits impulses originating from the vestibular
apparatus?

/ A. Cranial nerve ll
/ B. Cranial nerve VII
/ C. Cranial nerve VIII
/ D. Cranial nerve XI
/ E. Cranial nerve XII



Explanation - Q: 3.5 Close

The correct answer is C. The vestibular portion of cranial nerve VIII
(vestibulocochlear nerve) carries impulses from the vestibular apparatus to
the vestibular ganglion, which then conveys the impulses to the vestibular
nuclei of the brainstem.
Cranial nerve II (optic nerve, choice A) carries impulses generated by the
retina.
Cranial nerve VII (facial nerve, choice B) carries efferent motor impulses to
the face and afferent sensory impulses to the anterior two thirds of the
tongue.
Cranial nerve XI (accessory nerve, choice C) carries motor innervation to the
trapezius and sternocleidomastoid muscles.
Cranial nerve XII (hypoglossal nerve, choice E) carries motor innervation to
the tongue and strap muscles.


Vignette 1 of 4


A 35-year-old woman presents with complaints of weakness and spasticity in her
right leg. She also admits to difficulties with balance and malaise. The patient
states that about 6 months ago, while taking a shower one morning, she noticed
that she could not discriminate between hot and cold water on her right leg. She
began to feel very tired, and had some difficulties passing urine. Several months
ago she had noticed that the vision deteriorated in her right eye, although it
recovered over a period of two months. On examination, abnormalities of the left
eye and optic disc are present.
Neurologic examination is significant for weakness, Iack of coordination,
abnormally brisk reflexes in both legs, and difficulty walking were registered.


Which of the following is the pathological hallmark of this disease?

/ A. Ballooned cells and argyrophilic bodies
/ B. Concentrated, noninflammatory demyelination within the central pons
/ C. Demyelinated plaques with perivascular infiltration
/ D. GIiosis, foamy macrophages, Cowdry-A bodies
/ E. Noncaseating epithelioid granulomas



Explanation - Q: 1.1 Close

The correct answer is C. Multiple sclerosis (MS) is an inflammatory,
demyelinating disease of the central nervous system (CNS). MS is primarily
a disease of individuals living in temperate climates. The pathological
hallmark of MS is the demyelinated plaque, which consists of a well-
demarcated hypocellular area with loss of myelin, relative preservations of
axons, and the formation of astrocytic scars (chronic plaques). Plaques vary
from 1 mm to several centimeters in diameter. The lesions have a propensity
for the optic nerves, periventricular white matter, brain stem, cerebellum, and
spinal white matter. The visual problems the patient had were most likely due
to optic neuritis, in association with demyelination of the optic nerve. Active
plaques show perivascular infiltration of lymphocytes (predominantly T cells)
and macrophages, microscopically. These lesions appear as indurated areas
in gross preparations, thus the term "sclerosis." At sites of inflammation, the
blood-brain barrier is disrupted, but the vessel wall is preserved. In some
inflammatory lesions, dissolution of the multilayered myelin sheath with
reconstitution as a lattice-like network of myelin membrane fragments may
be seen (vesicular demyelination). It seems that interleukin-12, a potent
inflammatory substance is expressed at high levels in early lesions.
Cerebrospinal fluid abnormalities include abnormally increased levels of
newly synthesized IgG, oligoclonal banding, and mononuclear pleocytosis.
The argyrophilic intraneuronal inclusions (Pick bodies) and ballooned
neurons (choice A) are characteristic pathological findings in Pick disease.
Central pontine myelinolysis is characterized by concentrated, symmetric,
noninflammatory demyelination of the central parts of the pons (choice B).
Some patients may have extrapontine lesions in the thalamus, basal ganglia,
and cerebellum.
Subacute sclerosing encephalitis (SLE) develops following the reactivation of
latent measles virus. The gross pathology includes cortical atrophy, loss of
white matter, and ventricular enlargement. Intense gliosis, foamy
macrophages in the white matter, and Cowdry-A inclusions (choice D)
consisting of a central eosinophilic nuclear mass surrounded by a halo (also
common in herpes simplex encephalitis) are found in patients with SLE.
Noncaseating epithelioid granulomas (choice E) are associated with
sarcoidosis.



Which of the major histocompatibility antigens is associated with an increased
risk of this disease?

/ A. A3
/ B. B27
/ C. DR2
/ D. DR3
/ E. DR4



Explanation - Q: 1.2 Close

The correct answer is C. Histocompatibility antigens are glycoproteins that
are expressed at the surface of all vertebrate cells. Human HLA antigens are
present on the surface of all nucleated cells. Major histocompatibility
antigens are major antigens responsible for tissue recognition, and stimulate
the acute, rapid type of graft rejection. Each person has unique HLA
antigens. The presence of some HLA antigens have been correlated with the
certain autoimmune diseases. Susceptibility to MS is linked to the HLA-DR2
haplotype.
The severity of iron overload in hemochromatosis is determined
predominantly by genetic factors, namely the expression of HLA-A3 (choice
A) region of the major histocompatibility complex.
85% of patients with Reiter syndrome will have HLA-B27 (choice B) antigen
present on leukocytes.
There is a strong association between type 1 diabetes mellitus and specific
HLA-D phenotypes, particularly HLA-DR3 (choice D).
Approximately 35% of Caucasians are HLA-DR4-positive (choice E). HLA-
DR4-positive patients are genetically predisposed to autoimmune diseases
such as rheumatoid arthritis and type 1 diabetes.


Demyelination is the major feature of this disease. Which of the following cells
forms myelin in the central nervous system?

/ A. Astrocytes
/ B. Ependymal cells
/ C. Microglia
/ D. OIigodendrocytes
/ E. Schwann cells



Explanation - Q: 1.3 Close

The correct answer is D. The myelin sheath is formed by oligodendrocytes
in CNS and by Schwann cells (choice E) in the peripheral nervous system
(PNS). The gaps formed between myelin sheath cells along the axons are
called nodes of Ranvier. Since the lipid structure of myelin serves as a good
insulator, the myelin sheaths increase the rate of propagation and efficacy of
transmission of the impulse along the axon. The electrical impulse jumps
from one node to the next at the rate as fast as 120 m/s. This rapid type of
conduction is called saltatory conduction. Demyelination can occur early in
life as consequence of congenital metabolic disorders. Demyelination later in
life can be repaired with glia, which explains the frequent exacerbations and
remissions in MS. It is believed that in MS, multiple mechanisms of immune
injury of myelin coexist: cytokine-mediated injury of oligodendrocytes and
myelin, digestion of surface myelin antigens by macrophages, complement-
mediated injury, and direct injury by CD4+ and CD8+ T cells. This type of
injury causes the loss of saltatory conduction in nerve fibers. The exposed
axon is susceptible to further injury, resulting in irreversible axonal damage.
Tumor necrosis factor-alpha and IFN gamma may contribute directly to the
damage by injuring oligodendrocytes or the myelin membrane.
Glial cells do not carry action potentials, but they have many important
functions. There are several types of glia cells: astrocytes (choice A), which
transport nutrients to neurons, hold neurons in place, digest parts of dead
neurons, and regulate the composition of extracellular space, and microglia
(choice C), which possess phagocytic function, clean up debris, and protect
the brain from microorganisms.
Ependymal cells (choice B) form the epithelial lining of the ventricles.
Which of the following interferon (IFN) classes would most likely be used in this
patient?

/ A. IFN alpha-2a
/ B. IFN alpha-2b
/ C. IFN alpha-n3
/ D. IFN beta
/ E. IFN gamma



Explanation - Q: 1.4 Close

The correct answer is D. Interferons (IFNs) are produced by the body in
response to viral infections and tumors, and for the regulation of immunity.
They belong to cytokine family of proteins. Three types of natural IFNs are
produced: alpha, beta, and gamma. Alpha and beta types are classified as
Class I since they bind to IFN cell surface receptors type 1. IFN gamma is
class II, since it binds only to type 2 receptors. The interferons reduce the
proliferation of T cells and the production of tumor necrosis factor-alpha,
decrease antigen presentation, alter cytokine production to favor ones by
type 2 helper T cells, increase the secretion of interleukin-10, and reduce the
passage of immune cells across the blood-brain barrier.
Beta interferons are made by fibroblasts. IFN beta-1a, a non-glycosylated
molecule, with the same amino acid composition as natural human IFN beta,
reduces the frequency of relapses of MS by 40%. It has an additional benefit
of slowing and preventing the development of MS-related brain atrophy. IFN
beta-1b, a glycosylated molecule, reduces the relapse rate and slows
disease progression.
Another agent, glatiramer acetate, is a mixture of synthetic polypeptides. It
may promote proliferation of Th2 cytokines, and competes with myelin basic
protein for presentation on Class II MHC molecules, thereby inhibiting
antigen-specific T cell activation. The drug also alters the function of
macrophages and induces antigen-specific suppressor T cells. Mitoxantrone
is a FDA-approved agent that reduces neurologic disability and the
frequency of relapses in patients with progressive MS. Treatment of MS may
also include azathioprine, methotrexate, cyclophosphamide,
methylprednisolone, and 2-chlorodeoxyadenosine, depending on indications.
Leukocytes produce the interferon alpha group. Interferons alpha-2a (choice
A) and alpha-2b are made by recombinant DNA technology using genetically
engineered E. coli. Interferon alpha-2a is an effective antineoplastic,
immunomodulator and antiviral agent. It is used to treat hairy cell leukemia,
AIDS-related Kaposi sarcoma, chronic myelogenic leukemia, and chronic
hepatitis C.
IFN alpha-2b (choice B) is a highly purified protein of 165 AA, and is
indicated in AIDS-related Kaposi sarcoma, hairy cell leukemia, and chronic
hepatitis C. It is also used in condylomata acuminata and malignant
melanoma.
IFN alpha-n3 (choice C) is made from human leukocytes induced by murine
virus. It is only used in the treatment of condylomata acuminata.
IFN gamma (choice E) is type II, since it binds only type 2 receptors. Natural
killer cells (NK) and activated T cells produce this class of interferons. Type II
interferons have modest antiviral activity, are more potent
immunomodulators, and have a different range of immune functions,
including macrophage activation.




Vignette 2 of 4

A 55-year-old African-American woman presents to her physician complaining of
acute pain and redness in her right eye, with nausea and vomiting. She reports
seeing halos around lights. On examination, Iacrimation, Iid edema, conjunctival
injection, a steamy cornea, and a fixed mid-dilated pupil are seen. She has
markedly elevated intraocular pressure (IOP), and a visual acuity of 20/200 in her
right eye. She has no family history of eye diseases. She denies using cocaine or
taking paroxetine.


Which of the following is the most likely diagnosis?

/ A. Acute closed-angle glaucoma
/ B. Corneal laceration
/ C. Ocular hypertension without glaucoma
/ D. Open angle glaucoma
/ E. Retinal detachment



Explanation - Q: 2.1 Close

The correct answer is A. This patient has acute closed-angle glaucoma, as
evidenced by the pain and redness of her eye, the steamy cornea, her
description of halos around lights, and her markedly elevated intraocular
pressure. The fixed mid-dilated pupil indicates ischemia to the iris. Unlike
open angle glaucoma, closed-angle glaucoma (also called narrow-angle or
angle-closure glaucoma) is a medical emergency, and requires immediate
diagnosis and treatment to prevent permanent visual impairment.
The anterior and posterior chambers of the eye are filled with aqueous
humor, which flows from the posterior chamber, through the pupil, into the
anterior chamber, into the trabecular network, and then exits the eye via the
canal of Schlemm. Individuals with a shallow or narrow anterior chamber or
thickened lens may be predisposed to this type of glaucoma because their
iris is in close opposition to the chamber angle and cornea (narrow angle). If
the aqueous humor is produced at a greater rate than it can be drained,
intraocular pressure rises. Any stimulus that causes pupillary dilation (e.g.,
anticholinergic medications, sympathomimetics, emotional upset, dim
lighting) can precipitate an attack.
The cornea may look hazy because of edema, but corneal laceration (choice
B) is produced by trauma. There is no history of trauma in this patient and
the symptoms exhibited by this patient strongly suggest acute closed-angle
glaucoma
Intraocular hypertension (choice C) is a condition in which IOP is higher than
normal (greater than 21 mm Hg), but there is no damage to the optic nerve
or visual loss. About 90% of people with elevated IOP never develop
glaucoma. There is no associated pain or visual symptoms associated with
this.
Known as the "sneak thief of sight," open-angle glaucoma (choice D) has no
early warning signs or symptoms, and accounts for 90% of all glaucoma
cases. It is a slow progressive condition, usually affecting both eyes, in which
there is an increase in resistance to the outflow of aqueous humor. This
resistance is associated with a normal rate of production of the fluid and
normal anterior chamber angle. Elevated IOP is the most common finding
and age-related changes in the trabecular region are the most likely
explanation of this disease. Blurred vision, appearance of colored halos
around lights, problems with dark adaptation, and visual difficulties that new
eye prescriptions do not help are characteristic for this disease. Risk factors
for developing glaucoma are family history, age (increased risk with age),
medical conditions (diabetes, myopia, hypertension), and an abnormally high
IOP. African-Americans are three to four times more likely than Caucasians
to develop glaucoma. Also, Asians and Eskimos are more likely to develop
glaucoma than Caucasians.
Retinal detachment (choice E) represents separation of the inner layers of
retina from the retinal pigment epithelium, most commonly caused by a break
in the retina. Symptoms include the sensation of flashing lights (photopsia),
decreased visual acuity, wavy distortion of objects, floaters, and visual field
defects.



Which of the following medications might have precipitated this patient's
condition?

/ A. AIpha2-selective agonist
/ B. Beta adrenergic antagonist
/ C. Muscarinic antagonist
/ D. Parasympathomimetic
/ E. Prostaglandin analog



Explanation - Q: 2.2 Close

The correct answer is C. Anticholinergics (parasympathetic antagonists)
are a group of drugs that include alkaloids derived from the plant Atropa
belladonna (atropine, hyoscyamine, scopolamine) and related, synthetic
products (propantheline, dicyclomine, homatropine and eucatropine). They
compete reversibly with acetylcholine at the level of muscarinic receptors.
These agents block the cholinergic response of the ciliary muscle and the
sphincter pupillae muscle, causing cycloplegia and mydriasis. Pupillary
dilation causes an increased pupillary block that can precipitate acute
closed-angle glaucoma in patients with occludable angles. Therefore,
anticholinergics and other drugs with substantial anticholinergic activity
should not be used in cases of suspected or overt glaucoma without previous
gonioscopic angle evaluation.
Alpha
2
agonists (choice A) are indicated as second- or third-line agents in
the therapy of glaucoma. Both alpha
2
-selective adrenergic agonists
(apraclonidine and brimonidine) lower IOP effectively. But, because of high
rates of allergic reactions and tachyphylaxis, apraclonidine is most useful for
the short-term pressure control. Brimonidine (0.2%) is more lipophilic and
alpha
2
-selective than apraclonidine, and is used for chronic treatment of
glaucoma.
Topical beta blockers (timolol, betaxolol, levobunol, choice B) can be used
together with carbonic anhydrase inhibitors in the treatment of the acute
attack. Systemic side effects of these drugs include bronchospasm,
shortness of breath, depression, fatigue, confusion, impotence, bradycardia,
but these effects are minimized by using topical preparations of these
agents.
Parasympathomimetics (choice D) such as pilocarpine help open the
drainage canals and increase the flow of aqueous humor out of the eye. In
addition to miosis-induced increase in the outflow, contraction of the
sphincter pupillae ms. causes increased tension on the scleral spur, thereby
further reducing outflow resistance.
Latanoprost is a selective prostanoid F
2
alpha receptor agonist (choice E)
that reduces the intraocular pressure by increasing the outflow of aqueous
humor. Studies showed that the mechanism of this topical medication
(0.005%) includes an increased uveoscleral outflow. A single daily
application will effectively lower IOP for 24 hours. It has very few systemic
side effects. The major ocular side effect is increased pigmentation of the iris
and eyelid, and increased pigmentation and growth of the eyelashes.


Which of the following drugs should be included in this patient's treatment?

/ A. Atropine
/ B. Hydrochlorothiazide
/ C. Pilocarpine
/ D. Pirenzepine
/ E. Propranolol



Explanation - Q: 2.3 Close

The correct answer is C. The goal in the treatment of acute narrow angle
glaucoma is to lower the intraocular pressure. This can be done in three
different ways: blocking aqueous humor production (beta-blockers, carbonic
anhydrase inhibitors, alpha-2 agonists), reducing vitreous volume
(hyperosmotic agents), and increasing outflow of aqueous humor
(parasympathomimetics, prostaglandin analogs). Pilocarpine is a muscarinic
agonist, which causes pupillary miosis and constriction of the ciliary muscle.
This leads to an increase in aqueous humor outflow via the canal of
Schlemm, thus decreasing the intraocular pressure. Because this is an
emergency situation, the goal is to lower intraocular pressure as quickly as
possible. Therefore, several of these drugs may be used in combination.
Atropine (choice A) is a cholinergic antagonist, which will cause mydriasis;
this can precipitate or worsen acute narrow angle glaucoma.
Hydrochlorothiazide (choice B) is a thiazide diuretic. It is not used in
glaucoma. Carbonic anhydrase inhibitors are used instead.
Pirenzepine (choice D) is a muscarinic (M
1
) antagonist. M
3
receptors are
present on the ciliary muscle and pupillary sphincter muscle, and this drug
should theoretically have no effect. If it had any ability to block M1 receptors,
it would make this patient's situation worse.
Propranolol (choice E) is a non-selective beta blocker, which is not used in
glaucoma. However, beta blockers such as timolol and betaxolol are used.


Additional treatment might consist of which of the following agents?

/ A. Furosemide
/ B. Latanoprost
/ C. Neostigmine
/ D. Succinylcholine
/ E. Terbutaline



Explanation - Q: 2.4 Close

The correct answer is B. Latanoprost is a prostaglandin F2 analog that
acts by increasing the outflow of aqueous humor; it is used topically. This
drug and other similar drugs (unoprostone, travoprost, bimatoprost)
permanently increases the brown pigment of the iris and may increase
eyelash growth.
Furosemide (choice A), a loop diuretic, is not used for glaucoma; however,
carbonic anhydrase inhibitors such as dorzolamide (topical) and
acetazolamide (parenteral) could be used for this purpose. These agents
decrease aqueous humor production. Hyperosmotic agents such as mannitol
and isosorbide are also used and act by creating an osmotic gradient
between the ocular fluids and plasma.
Neostigmine (choice C) is an acetylcholinesterase inhibitor. In theory, it
could be helpful, however, it is positively charged and would not be able to
gain access to the eye. It is useful in myasthenia gravis, which is a disease
of the periphery, because it does not cross the blood-brain barrier.
Physostigmine would be a better choice.
Succinylcholine (choice D) is a depolarizing skeletal muscle relaxant and
has no role in glaucoma therapy.
Terbutaline (choice E) is a beta-2 agonist. This could worsen the situation by
increasing aqueous humor production. Instead, beta antagonists such as
timolol or betaxolol could be used to decrease aqueous humor production.


The patient is also given timolol eye drops and immediately becomes short of
breath. This patient likely has which of the following?

/ A. Asthma
/ B. Bronchitis
/ C. Cardiac arrhythmia
/ D. Myocardial infarction
/ E. Pericarditis



Explanation - Q: 2.5 Close

The correct answer is A. Even topical blockers can lead to fatal
bronchoconstriction in asthmatics. blockers are contraindicated in
asthmatics.
Bronchitis (choice B) is an inflammation of the bronchi from exogenous
irritation or infection. There would have to be coexisting asthma to cause
frank bronchoconstriction with the administration of a blocker.
blockers are used as cardiac protectants in patients with arrhythmia
(choice C) and myocardial infarction (choice D). blockers should be
avoided however in patients with a history of heart block
Pericarditis (choice E) is inflammation of the pericardium from infection,
inflammatory conditions or trauma. There is no real connection between
pericarditis and blocker administration.




Vignette 3 of 4


A 69-year-old man presents to the emergency department with painless vision
loss of his right eye. He describes the visual loss as a gradual progression from
blurry to total blackout over the past two hours. He has no history of prior visual
problems. Past medical history is significant for a myocardial infarction three
years ago. The patient takes 81 mg of aspirin daily. Vital signs are normaI.
Physical examination reveals 20/20 vision of the left eye but no vision in the right
eye.Extraocular muscles are intact. The neurologic examination is normaI. The
cardiac examination reveals an S4 heart sound.


The most likely cause of this unilateral blindness is which of the following?

/ A. Carotid artery dissection
/ B. Cavernous sinus thrombosis
/ C. Posterior cerebral artery occlusion
/ D. Posterior inferior cerebellar artery occlusion
/ E. Retinal artery occlusion



Explanation - Q: 3.1 Close

The correct answer is E. This patient has a sudden onset of complete,
unilateral, painless loss of vision with intact extraocular muscle movement.
This is consistent with occlusion of the retinal artery. The retina derives its
blood supply from the internal carotid artery via the ophthalmic artery, from
which branch the central retinal artery and ciliary arteries. The central retinal
artery supplies the inner layers of retina, while ciliary arteries supply blood to
the choriocapillaris and the outer retinal layers. Occlusion of small retinal
arterioles produces "cotton wool" spots while total occlusion of the
ophthalmic artery leads to total ischemia and blindness. This usually occurs
secondary to atherosclerotic disease. The key here is recognizing that the
loss of vision is an isolated finding with no other neurologic symptoms. Other
deficits would be consistent with a stroke of a larger vascular territory such
as the internal carotid artery.
Carotid artery dissection (choice A) is less likely than retinal artery occlusion
because there would likely be multiple associated neurologic findings in the
case of dissection, as the internal carotid artery supplies a large portion of
the anterior circulation of the brain. The most likely symptoms among a large
cadre of possibilities is face and arm weakness contralateral to the
dissection.
Cavernous sinus thrombosis (choice B) would lead to facial pain and
possible cranial nerve deficits. The optic nerve (cranial nerve II) extends
anteriorly from the optic chiasm through the superior orbital fissure.
Posterior cerebral artery occlusion (choice C) is less likely than retinal artery
occlusion because there would likely be multiple associated neurologic
findings. Also, the visual field loss resulting from a unilateral posterior
cerebral artery occlusion would more likely be a homonymous hemianopia, it
would not cause complete blindness in one eye. Other changes that may
occur from posterior cerebral artery stroke are complex findings, such as the
inability of recognize faces (prosopagnosia), failure to connect visual
perception with conscious thought, color vision loss, and difficulty with
reading (dyslexia or alexia). Memory problems and motor impairment are
also common associated findings.
Posterior inferior cerebellar artery occlusion (choice D) produces cerebellar
symptoms such as ipsilateral clumsiness. The posterior inferior cerebellar
artery supplies the cerebellar vermis and the ipsilateral cerebellar tonsil.


At the molecular leveI, which of the following components is essential for the first
step of the visual cascade?

/ A. 11-cis-retinal
/ B. AII-cis-retinal
/ C. AII-trans-retinal
/ D. Meta-rhodopsin ll
/ E. Rhodopsin



Explanation - Q: 3.2 Close

The correct answer is A. The visual cascade: 11-cis-retinal + opsin ->
rhodopsin + light -> meta-rhodopsin II. Meta-rhodopsin II dissociates after
light exposure to form all-trans-retinal. 11-cis retinal and opsin are essential
first steps in generating the photochemical visual cascade.
All-cis-retinal (choice B) is not a part of the visual cascade.
All-trans-retinal (choice C), meta-rhodopsin II (choice D), rhodopsin (choice
E) is a later part of the visual cascade: 11-cis-retinal + opsin -> rhodopsin +
light -> meta-rhodopsin II. Meta-rhodopsin II dissociates after light exposure
to form all-trans-retinal. 11-cis retinal and opsin are essential first steps in
generating the photochemical visual cascade.


This patient's condition affects both the rods and cones of the retina. Which of
the following characteristics distinguishes rods from cones?

/ A. Rods are not located within the fovea
/ B. Rods detect color
/ C. Rods have an on-center, off-surround organization of receptor fields
/ D. Rods provide high visual acuity
/ E. Rods recover sensitivity more rapidly after exposure to light



Explanation - Q: 3.3 Close

The correct answer is A. The fovea only contains cones and no rods are
present.
The cones are responsible for color vision (choice B). There are three types
of cone receptors: red, green, and blue.
Cones are smaller than rods and are organized into on-center, off-surround
receptor fields (choice C).
Cones are responsible for providing high visual acuity (choice D).
Cones are more sensitive to light and recover their sensitivity about 5 times
faster than rods after exposure to light (choice E). However, rods respond to
lower light levels better than do cones.




Vignette 4 of 4


A 72-year-old woman with a history of hypertension and obesity presents to her
primary care physician with a 3 day history of difficulty reading. She states that
she is able to read the words on the left side of a page without difficulty, but has
to turn her head to read the words on the right side of the page. Physical
examination reveals that she has a right homonymous hemianopia. Otherwise
she is without any other neurologic deficit. She is referred for a formal visual field
evaluation, which confirms that she has a right homonymous hemianopia with
macular sparing. A CT scan confirms that she had an ischemic stroke.

Occlusion of which of the following blood vessels would most likely result in this
patient's presentation?

/ A. Left middle cerebral artery
/ B. Left posterior cerebral artery
/ C. Left posterior communicating artery
/ D. Right anterior cerebral artery
/ E. Right ophthalmic artery



Explanation - Q: 4.1 Close

The correct answer is B. The posterior cerebral arteries are formed from
the bifurcation of the basilar artery. They supply the midbrain, the posterior
thalamus, and the occipital lobe, including the visual cortex. An occlusion of
the posterior cerebral artery results in a contralateral homonymous
hemianopia with macular sparing since the occipital pole subserving the
macula can receive collateral blood supply from the middle cerebral artery.
The left middle cerebral artery (choice A) arises from the internal carotid
artery and makes up part of the anterior circle of Willis. The middle cerebral
artery supplies the caudate nucleus, the putamen, the globus pallidus, and
the internal capsule through the lateral lenticulostriate arteries. As the middle
cerebral artery travels distally it supplies the lateral convexity of the
hemisphere and underlying insula. A left middle cerebral artery occlusion
would result in a stroke that would result in a stroke presenting with a global
aphasia and a contralateral hemiparesis.
The left posterior communicating artery (choice C) arises from the carotid
siphon and travels posteriorly to join the posterior cerebral artery. It supplies
the optic chiasm and tract, the hypothalamus, the subthalamus, and a portion
of the anterior thalamus. Any visual field deficit caused by a posterior
communicating artery occlusion would most likely be accompanied by a
contralateral motor deficit as well.
The right anterior cerebral artery (choice D) arises from the internal carotid
artery and makes up part of the anterior circle of Willis. The proximal anterior
cerebral artery supplies the superior surface of the optic nerve, the optic
chiasm, the anterior hypothalamus, the anterior commissure, and fornix via
the medial lenticulostriate arteries. Just distal to the anterior communicating
artery the medial distal striate artery (the recurrent artery of Heubner) arises
to supply the head of the caudate, the anterior limb of the internal capsule,
the anterior putamen and globus pallidus, and the inferior frontal lobe. The
distal anterior cerebral artery is the usual location of occlusions by thrombotic
emboli. This segment supplies the anterior 2/3 of the medial cortex.
Occlusion of the distal right anterior cerebral artery would result in weakness
in the left lower extremity because the leg is represented medially on the
motor homunculus.
The right ophthalmic artery (choice E) is the first intradural branch off the
internal carotid artery. It travels with the optic nerve to supply the globe by
way of the central retinal artery and the ciliary arteries. Occlusion of the right
ophthalmic artery would result in a right monocular field deficit. If this is
transient, it is called amaurosis fugax and is usually the result of a small fibrin
embolus.



What is the most likely location of this patient's lesion?

/ A. Banks of the calcarine sulcus
/ B. Banks of the central sulcus
/ C. Cingulate gyrus
/ D. Middle frontal gyrus
/ E. Superior temporal gyrus



Explanation - Q: 4.2 Close

The correct answer is A. The primary visual cortex or striate cortex (area
17) receives its input from the geniculocalcarine fibers of the lateral
geniculate body. The primary visual cortex is located in the banks of the
calcarine fissure. The cuneus is the upper bank and the lingual gyrus is the
lower bank. A lesion in this area results in contralateral hemianopia with
macular sparing.
The banks of the central sulcus (choice B) contain the precentral and
postcentral gyrus. The precentral gyrus contains the primary motor cortex,
while the postcentral gyrus contains the primary somatosensory cortex.
The cingulate gyrus (choice C) contains a portion of the limbic association
cortex and is involved in emotions.
The middle frontal gyrus (choice D) contains the frontal eye fields (area 8).
The frontal eye fields control the initiation of saccades. Stimulation of the
frontal eye fields causes contralateral eye deviation.
The superior temporal gyrus (choice E) contains Wernicke's speech area in
the dominant hemisphere, as well as containing the primary and secondary
auditory cortex.


If the patient instead presented with an inability of the left eye to perceive images
in the left half of her visual field and of her right eye to perceive images in the
right side of her right visual field, what would be the most likely location of her
lesion?


/ A. Lateral geniculate body
/ B. Optic chiasm
/ C. Optic nerve
/ D. Optic tract
/ E. Parietal geniculocalcarine radiations
/ F. Primary visual cortex
/ G. Temporal geniculocalcarine radiations



Explanation - Q: 4.3 Close

The correct answer is B. This patient has bitemporal hemianopia, which
occurs with lesions of the middle part of the optic chiasm. The decussating
fibers of the optic nerves, which carry the visual impulses from the nasal
halves of the retina, are blocked. Conditions such as pituitary adenoma and
craniopharyngioma can cause this. (These patients would also present with
other symptoms, e.g., endocrine symptoms).
A lateral geniculate body (choice A) would produce a contralateral
homonymous hemianopia.
An optic nerve lesion (choice C) can result in ipsilateral blindness.
An optic tract lesion (choice D) could produce a contralateral homonymous
hemianopia.
A lesion of the parietal geniculocalcarine radiations (choice E), which project
from the lateral geniculate body to the lower bank of the calcarine sulcus
(lingual gyrus), could result in a contralateral lower quadrantanopia.
A lesion of the primary visual cortex (choice F) could result in a contralateral
hemianopia with macular sparing.
A lesion of the temporal geniculocalcarine radiations (choice G), which
project from the lateral geniculate body to the upper bank of the calcarine
sulcus (lingual gyrus), could result in a contralateral upper quadrantanopia
("pie in the sky").


Vignette 1 of 5


A 5 year-old girl and her mother are referred for evaluation of bed-wetting. The
mother states that her daughter has never fully gained control of her bladder. It
took her an extended period of time to grow out of diapers and the girl has finally
stopped having "accidents" during the day. However, she continues to wet her
bed at night. The patient has developed normally in all other aspects and will be
starting kindergarten in three months. Both her mother and father are extremely
frustrated and have been losing sleep, as the girl wakes up at night 4 to 5 times
per week. Physical examination is unremarkable.

Which of the following will most likely be the result of further investigation?

/ A. Large capacity bladder
/ B. Learning disability
/ C. NormaI "work-up"
/ D. Urinary tract infection
/ E. Urological cancer


Explanation - Q: 1.1 Close

The correct answer is C. Enuresis was originally denoted as incontinence
of urine, but now, it is a term restricted to those children who are over the
age of 3 who wet the bed. Most children have achieved normal bladder
control by that time, girls earlier than boys. At age six years, 10% have
enuresis. It is important to rule out other possible causes of enuresis, but
usually, investigation will reveal only normal results. It is believed that up to
50% of cases are caused by delayed maturation of the nervous system or
intrinsic myoneurogenic bladder dysfunction. 30% are of psychologic origin
and 20% are secondary to more obvious organic disease.
The bladder capacity (choice A) of bed wetters and non-bed wetters is the
same. The difference is that the non-bed wetters are able to hold their urine
in their bladders throughout the night.
When compared with normal children, the majority of children with bed-
wetting problems score just as well on tests of intelligence and school
achievement. These children are no more likely to suffer from permanent
neurological abnormalities, poor school achievement, low IQs, or severe
learning disability (choice B).
It is true that bed-wetting children are more likely to get urinary tract
infections (choice D) if their personal hygiene is poor; it is not true that such
infections are the main cause of bed-wetting.
Malignancy (choice E) of the urinary tract is extremely rare in children and is
unlikely to be the cause of this child's problem.
The condition persists, and the patient is placed on desmopressin acetate
(DDAVP). This medication works via which of the following mechanisms?

/ A. Decrease detrusor muscle tone
/ B. Improves alertness of the patient during the sleep cycle
/ C. Increase external sphincter contraction
/ D. Increase salt reabsorption in the collecting tubules
/ E. Increase water permeability and reabsorption in collecting tubules



Explanation - Q: 1.2 Close

The correct answer is E. Desmopressin, or DDAVP, can be used
intranasally in patients suffering from enuresis. This medication is a long-
acting synthetic analogue of vasopressin. The medication acts at V2
receptors, which are found on renal tubule cells, and mediates an anti-
diuretic effect by increasing water permeability and water reabsorption in the
collecting tubules, leading to decreased urine output. It is effective in 70% of
patients with increased nocturnal urine output.
Detrusor muscle tone (choice A) is under control of the parasympathetic
system. DDAVP does not affect this system.
Some sympathomimetic drugs may cause enough wakefulness so that the
child perceives the urge to void (choice B). Dextroamphetamine sulfate is
one such medication.
The external sphincter (choice C) is under somatic nervous control. This
medication does not contribute to this system.
Desmopressin works by increasing water permeability, not salt permeability
in the collecting tubules (choice D).


Which of the following would be appropriate management of a child with this
condition?

/ A. Do not let the child spend the night away from home
/ B. Have the child practice holding her urine during the day
/ C. Let the child sleep through the night
/ D. Punish the child for each accident
/ E. Put night-Iights in the bedroom, bathroom, and hallway



Explanation - Q: 1.3 Close

The correct answer is E. By putting lights in the bedroom, hallway, and
bathroom, the child may feel safer getting up and walking to the bathroom
during the night.
You should make it as easy as possible for your child to spend the night
away from home (choice A). If he or she goes to a slumber party, hide a
disposable diaper in the bottom of the sleeping bag, and have the child slip it
on under his or her pajamas.
Having the child hold their urine during the day (choice B) i.e., create urinary
retention, adds nothing to the effectiveness of treatment. Many of these
children can already hold urine for a full school day, and often avoid going to
the school bathroom because they are too shy or are uncomfortable going in
the school toilet. This technique may be dangerous as it can result in reflux
of urine towards the kidneys.
The child must learn to wake up when he or she needs to urinate (choice C).
If the child gets up for any reason, have him or her get out of bed and try to
use the toilet.
Never punish a child for wetting the bed (choice D). He or she cannot control
it. Punishment can sometimes make bed-wetting worse by increasing the
sense of shame and embarrassment the child feels.



If a biopsy of this patient's bladder were obtained, which histological cell type
would be found in the superficial layer?

/ A. Multiple nephrons
/ B. Pseudostratified mucous membrane
/ C. Smooth muscle fibers
/ D. Squamous cells
/ E. Transitional cells



Explanation - Q: 1.4 Close

The correct answer is E. The mucosa of the bladder is composed of
transitional epithelium. Beneath this is a well-developed submucosal layer
formed largely of connective tissue and elastic tissue. The detrusor muscle
lies external to the submucosa. It is made up of a mixture of smooth muscle
fibers (choice C) arranged at random in a longitudinal, circular, and spiral
manner.
The nephron (choice A) is the functioning unit of the kidney. It is composed
of the glomerulus, which projects into Bowman's capsule, which, in turn, is
continuous with the epithelium of the proximal convoluted tubule. This tubule
then continues as the loop of Henle, distal convoluted tubule, and then the
collecting duct. Together, these make up a nephron. Nephrons are not found
in normal bladder epithelium.
The seminal vesicles are made up of a mucous membrane that is
pseudostratified (choice B).
Squamous epithelium can be found lining that portion of the urethra that is
contained within the glans penis. If squamous cells (choice D) are found in
the bladder, it is considered an abnormality.




Vignette 2 of 5


A healthy 52 year-old man is evaluated during a routine physical examination. On
rectal examination, he is found to have a nodule on the left apex of his prostate.
His prostate specific antigen (PSA) is 6.7 ng/dL. He denies any nocturia, dysuria,
hematuria, or urinary hesitancy. The patient undergoes a transrectal ultrasound
with biopsy of the prostate gland. The pathology report is consistent with cancer
of the prostate.


Which of the following is the most common histological variant of prostate
cancer?

/ A. Adenocarcinoma
/ B. Sarcoma
/ C. Seminoma
/ D. Squamous cell carcinoma
/ E. Transitional cell carcinoma



Explanation - Q: 2.1 Close

The correct answer is A. Prostate cancers are adenocarcinomas that arise
from the prostatic acinar cells. In eighty-five percent of the cases, the
adenocarcinoma is multifocal. Prostate cancers are graded according to their
dysplasia, utilizing the Gleason scoring system.
Sarcoma of the prostate (choice B) accounts for 0.1% of all malignant
prostatic tumors. Rhabdomyosarcoma is the most frequent mesenchymal
tumor within the prostate, and is seen almost exclusively in childhood.
Seminomas (choice C) are not seen within the prostate. Seminoma is a
germ cell tumor of the testis.
Primary squamous carcinoma of the prostate (choice D) is rare and is
associated with a poor survival. More commonly, squamous differentiation
occurs in the primary and metastatic deposits of adenocarcinomas after
estrogen therapy.
Transitional cell carcinoma (choice E) is the primary malignancy of the
urothelial tract. While there is potential for the prostatic urethra to develop
transitional cell carcinoma, it is unlikely that this will be diagnosed via a
transrectal biopsy


Which of the following cell type(s) secretes prostate specific antigen (PSA)?

/ A. Benign prostate epithelial cells only
/ B. Malignant prostate epithelial cells only
/ C. Benign and malignant prostate epithelial cells
/ D. Prostate stromal cells
/ E. Seminal vesicle epithelium



Explanation - Q: 2.2 Close

The correct answer is C. Prostate-specific antigen (PSA), a glycoprotein
with a MW of 34,000, is a biological marker used in the detection of prostate
cancer. It is secreted from prostatic epithelial cells and is responsible for
aiding in the liquefaction of semen. This marker, which is evaluated with a
blood test, can be produced by both benign and malignant epithelial cells.
The prostate stromal cells (choice D) do not produce PSA.
The seminal vesicles (choice E) are responsible for providing a majority of
semen volume, however they do not produce PSA.


This patient's cancer is most likely located in which of the following areas of the
prostate?

/ A. Apex of the prostate
/ B. Base of the prostate
/ C. Central zone
/ D. Peripheral zone
/ E. Transitional zone



Explanation - Q: 2.3 Close

The correct answer is D. Approximately 60-70% of prostate cancers occur
in the peripheral zone of the prostate. Fortunately, this is the portion of the
prostate that is palpated during digital rectal examination.
The prostate is often described as having an apex, middle area, and base.
The apex is the most distal aspect and the area of narrowest diameter. The
base of the prostate, the widest area, is the portion of the prostate adjacent
to the bladder. On rectal examination, the apex is encountered initially and
the base is the area located in further into the anal canal. These are
descriptive anatomic terms and the frequency of prostate cancer is not
differentiated between the two (choice A and B).
5-10% of prostate cancers occur in the central zone (choice C). This is the
portion of the prostate that surrounds the ejaculatory ducts.
10-20% of prostate carcinomas develop in the transitional zone (choice E) of
the prostate. This area is located just outside the urethra at the
verumontanum and is composed of periurethral glands, which are
responsible for benign prostatic hyperplasia (BPH).


The physician recommends surgery for removal of the prostate and associated
lymph nodes. During this procedure, the obturator nerve is at risk of transection.
If this occurs, the patient may suffer which of the following?

/ A. Loss of erectile function
/ B. Loss of sensation over the lateral aspect of the thigh
/ C. Loss of sensation over the medial aspect of the thigh
/ D. Loss of sensation over the posterior aspect of the thigh
/ E. Weakness with lower extremity extension



Explanation - Q: 2.4 Close

The correct answer is C. The obturator nerve, originating from L2, L3, and
L4 of the lumbar plexus, is the nerve of the adductor muscles of the thigh.
This nerve descends through the psoas major muscle, leaving its medial
border at the brim of the pelvis. It pierces the psoas fascia, crosses the
sacroiliac joint, passes lateral to the internal iliac vessels and ureter, and
enters the pelvis minor. It leaves the pelvis through the obturator foramen
and enters the thigh. The obturator nerve supplies motor innervation to the
obturator externus, which is responsible for laterally rotating the thigh. This
nerve also provides a small cutaneous branch, which is responsible for
sensation to the medial aspect of the thigh.
The neural innervation responsible for erectile function (choice A) is
complex, but the obturator nerve plays no role. The neurovascular bundles
that contribute to erections travel on the lateral aspect of the prostate and
may also be injured during prostatectomy. These bundles arise from the
pelvic plexus, which is formed by parasympathetic visceral efferent
preganglionic fibers that arise from the sacral center (S2-S4) and
sympathetic fibers from the thoracolumbar center (T11-L2).
The lateral femoral cutaneous nerve, originating from L2 and L3 of the
lumbar plexus, is responsible for sensation to the lateral aspect of the thigh
(choice B).
The majority of the posterior aspect of the thigh (choice D) is innervated by
branches from the posterior femoral cutaneous nerves, which originate from
the sacral plexus.
Extension of the thigh (choice E) is performed by many muscles. The
anterior thigh muscles play a major role in this function. The muscles are
innervated by the femoral nerve (originating from L2, L3, and L4).


The physician is concerned that this patient has metastatic prostate cancer. What
is the imaging modality that is most Iikely to confirm his diagnosis of metastatic
prostate cancer?

/ A. Bone scan
/ B. CT scan of the abdomen
/ C. CT scan of the chest
/ D. CT scan of the pelvis
/ E. MRI of the brain



Explanation - Q: 2.5 Close

The correct answer is A. The most frequent site of metastatic prostate
carcinoma is the lymphatic system. This is followed in frequency by bone
metastases. Therefore, the imaging modality that is used to evaluate for
metastatic prostate cancer must evaluate the skeletal system. Bone
scintigraphy, a radionuclide bone scan (choice A), provides the most
sensitive method for detecting bony metastases.
Abdominal and pelvic imaging (choice B and D) may evaluate for local
disease extension but are not routinely useful because of low sensitivity.
These are very poor tests for evaluating for enlarged lymph nodes.
Lung metastases may occur, however, however almost all cases have bone
involvement as well. Therefore, bone scan will provide more information that
a CT scan of the chest (choice C).
It is rare for prostate cancer to metastasize to the brain, therefore an MRI of
the brain (choice E) has little value.


If this patient ultimately develops metastatic prostate cancer, he may be treated
with an agent such as leuprolide. Such medications work by which of the
following mechanisms?

/ A. BIocking cytochrome P450
/ B. BIocking testicular androgen production
/ C. BIocking testosterone at the target tissue level
/ D. Directly blocking the adrenal gland synthesis of androgens
/ E. Indirectly suppressing LH and FSH secretion



Explanation - Q: 2.6 Close

The correct answer is E. These medications, known as GnRH agonists
(e.g., leuprolide, nafarelin, goserelin, buserelin), work by activating the GnRH
receptors in the pituitary gland. This creates an initial increase in the release
of LH and FSH, causing an initial increase in testosterone ("flare
phenomenon"). With time, there is down-regulation of the pulsatile release of
LH and FSH, and ultimately, suppression of LH and FSH due to inhibition of
the hypothalamic-pituitary axis. This leads to a decrease in testosterone
levels. Castrate levels of testosterone are reached within 30 days.
Cytochrome P-450 (choice A) plays a role in the adrenal and gonadal
synthesis of androgens. This can be inhibited by ketoconazole, which can
produce castrate levels of testosterone within 8 hours.
Directly inhibiting testicular androgen production (choice B) can be
accomplished via bilateral orchiectomy.
The activity of testosterone can be blocked at the tissue level by inhibiting
the binding of testosterone and dihydrotestosterone to the intracellular
androgen receptors (choice C). Medications that work via this mechanism
are classified as nonsteroidal antiandrogens, e.g., flutamide and bicalutimide.
The only way to directly block the production of androgens from the adrenal
gland (choice D) is bilateral adrenalectomy. This is no longer necessary
because pharmacological androgen deprivation is now available.



Vignette 3 of 5

A 61-year-old white man complains of painless hematuria. The patient states that
he noticed some blood in his urine approximately two months prior to
presentation. However, the bleeding cleared on its own and he did not seek
evaluation at that time. But, over the last 72 hours, the hematuria returned and
has persisted, and now he is concerned. The patient denies dysuria, urethral
discharge, fevers, chills, sensation of incomplete emptying, or weight loss. His
history is significant for diet-controlled diabetes and hypertension. He has never
had surgery. His only medication is amlodipine for hypertension. He admits to a
75 pack-year history of cigarette use. On examination, his vital signs are normaI.
There is no palpable lymphadenopathy or abdominal masses. His urological
examination, including digital rectal examination is normaI.

Which of the following is the most likely diagnosis?

/ A. Carcinoma of the bladder
/ B. Carcinoma of the penis
/ C. Carcinoma of the prostate
/ D. Renal stone
/ E. Urinary tract infection



Explanation - Q: 3.1 Close

The correct answer is A. This patient has painless hematuria. The
possibility of bladder cancer is very high, especially with his extensive
smoking history. The possibility of cancer within the urinary tract must be
considered in any patient who complains of painless, gross hematuria. The
timing of the hematuria is very important. Initial hematuria usually arises from
the urethra, typically the prostatic urethra. Total hematuria is most common
and indicates that the bleeding is most likely coming from the bladder or
upper urinary tracts. Terminal hematuria occurs at the end of micturition and
is usually secondary to pathology in the area of the bladder neck.
Carcinoma of the penis (choice B) is usually of squamous cell origin. The
most common locations for this cancer are the glans penis and prepuce. It
almost always arises in uncircumcised men. In order for penile cancer to
cause hematuria it must obstruct the urethral meatus or invade the urethra.
Either scenario should be easily seen on physical examination.
Hematuria is not a common complaint in patients with prostate cancer
(choice C). Carcinoma of the prostate usually originates in the peripheral
zone of the prostate, which does not communicate with the urethra. If a
patient with prostate cancer complains of hematuria, they most likely have
locally advanced or metastatic disease. Either of these should be
appreciated on physical examination.
Patients with renal stones (choice D) may have gross hematuria. However,
they should have other symptoms as well: i.e., fever, flank pain, abdominal
pain, dysuria, or urinary frequency. On examination, they may have
costovertebral angle tenderness, and possibly an elevated body temperature
if the stone obstructs the drainage of urine.
If a patient with a urinary tract infection (choice E) has gross hematuria, they
should have other complaints such as fever, chills, urinary frequency, or
dysuria. It will be important to obtain a urine culture from this patient, but the
history of tobacco use must make the physician concerned about a bladder
tumor.
Which of the following risk factors is associated with this patient's diagnosis?

/ A. AIcohol abuse
/ B. EIevated serum calcium levels
/ C. Ingestion of artificial sweeteners
/ D. Rubber tire factory workers
/ E. Unprotected sexual intercourse



Explanation - Q: 3.2 Close

The correct answer is D. Aromatic amines and azo dyes are known
carcinogens. Beta-naphthylamine is one of the agents associated with an
increased risk of cancer of the urothelium. People who work with dyes,
petroleum, leather, printing, and the rubber industry are exposed to this
carcinogen. After absorption, it is believed to be hydroxylated into an active
form and then is detoxified by conjugation with glucuronic acid. When
excreted in the urine, the nontoxic conjugate is split by the urinary enzyme
glucuronidase to release the electrophilic reactant again, thus inducing
bladder cancer.
Alcohol (choice A) has been implicated in many different disease processes,
but it has not been shown to be a risk factor for bladder cancer. Cigarette
smoking is the agent most often implicated as a cause of bladder cancer.
Elevated serum calcium (choice B) levels play a role in the formation of
renal stones, not bladder cancer.
Artificial sweeteners (choice C) have been proposed as risk factors for
bladder cancer, however multiple studies have failed to confirm this
association.
Unprotected sexual intercourse (choice E) puts a person at risk for a
sexually transmitted disease such as gonorrhea. Gonorrhea is the cause of
gonococcal urethritis.


During removal of this patient's bladder, the urachus is identified and ligated.
Which of the following most appropriately describes the urachus?

/ A. It connects the bladder to the rectum in early fetal life
/ B. It is responsible for development of the renal parenchyma
/ C. It is the remnant of the embryological allantois
/ D. It is the remnant of the embryological cloaca
/ E. It is the remnant of the embryological urogenital sinus



Explanation - Q: 3.3 Close

The correct answer is C. Embryologically, the allantois connects the
urogenital sinus with the umbilicus. Normally, the allantois is obliterated at
birth and is represented by a fibrous cord, the urachus, which extends from
the dome of the bladder to the navel. Urachal formation is related to bladder
descent. Lack of descent is associated with a patent urachus. If the entire
tract remains patent, then urine will constantly drain from the umbilicus. This
will be identified in the first few days of life.
Very early in fetal development, the blind end of the hindgut, caudal to the
point of origin of the allantois, expands to form the cloaca, which is separated
from the outside by a thin plate of tissue called the cloacal membrane.
During the seventh week of development, the cloaca divides into a ventral
and dorsal portion. The ventral aspect is renamed the urogenital sinus and
the dorsal portion becomes the rectum (choices D and E).
During the period of fetal life prior to division of the cloaca, the urinary and
intestinal systems are connected. Although the allantois drains the cloaca, it
is not responsible for keeping the two connected. When the cloaca is divided
into the urogenital sinus and rectum, the allantois drains the urogenital sinus
only (choice A).
The renal parenchyma is formed from the metanephros. The metanephros is
the final phase of development of the nephric system and originates from
both the intermediate mesoderm and the mesonephric duct. The ureteral bud
is an outgrowth of the mesonephric duct, which grows cephalad and acquires
a metanephric cap as it moves. During this move, the metanephric cap
becomes progressively larger, and rapid internal differentiation takes place.
Ultimately, this metanephric tissue will become the renal parenchyma
(choice B).


Which of the following infectious agents is associated with another form of this
patient's likely condition?



Explanation - Q: 3.4 Close

The correct answer is C. In Egypt, parts of Africa, and the Middle East,
Schistosoma haematobium is the cause of bilharzial infection, which is
associated with squamous cell cancer of the bladder. This type of bladder
cancer is found in 60% of the bladder carcinomas in these parts of the world.
In the US, chronic infection, vesical calculi, or chronic catheter use are
associated with squamous cell cancer of the bladder. Schistosoma
hematobium have a terminal spine and travel through the inferior mesenteric
veins to settle in the veins of the pelvic organs, particularly the bladder. They
may cause hematuria, dysuria and, an increased risk of bladder cancer.
All of the parasites listed in the answer options are classified as trematodes,
or flukes. The life cycle of the trematodes include snails as the intermediate
host and humans as the definitive hosts. The Clonorchis and the
Paragonimus are hermaphrodites and therefore have a second host, fish,
and crabs, respectively.
Clonorchis sinensis(choice A), also called the Oriental liver fluke, settles in
the biliary ducts, where they grow into adults. Symptoms of patients with
large worm loads include abdominal pain and weight loss.
Paragonimus westermani(choice B) settles in the lung and causes
symptoms such as a cough with bloody sputum.
Schistosoma japonicum(choice D) has a very small lateral spine on its eggs.
They also live in the mesenteric veins and may cause hepatosplenomegaly.
They are found in Japan and China.
Schistosomes can be identified by their size and location of the spine on their
eggs. Schistosoma mansoni(choice E) with a prominent lateral spine, affects
the GI tract by living in the mesenteric veins and may cause massive
hepatosplenomegaly.


Which of the following chemotherapeutic agents causes hematuria secondary to
hemorrhagic cystitis?

/ A. BIeomycin
/ B. Cyclophosphamide
/ C. Doxorubicin (Adriamycin)
/ D. Hydroxyurea
/ E. Vinblastine



Explanation - Q: 3.5 Close

The correct answer is B. Cyclophosphamide is broken down to the
metabolite acrolein. It is this metabolite that is responsible for the
hemorrhagic cystitis that these patients develop. Patients given
cyclophosphamide for treatment of other cancers, are also at an increased
risk for developing bladder cancer. Pretreatment with MESNA (2-
mercaptoethanesulfonic acid) reduces the risk of hemorrhagic cystitis and
bladder cancer.
Bleomycin (choice A) is associated with pulmonary fibrosis, and patients
should have pulmonary function tests prior to its use.
Doxorubicin (choice C) may be cardiotoxic, but does not affect the bladder.
Hydroxyurea (choice D) is associated with bone marrow depression.
Vinblastine (choice E) is associated with bone marrow suppression and
alopecia.




Vignette 4 of 5

A 61-year-old man presents to his family physician complaining that his urinary
stream has become weaker over the past few months, and he finds himself
waking up three to four times each night just to urinate. It is difficult for him to
generate a urine stream and he often feels that the bladder is still full even after
he has completely urinated. He denies any burning or blood with urination and
there is no urethral discharge or history of fevers. There has been no change in
his bowel habits.
He has never experienced anything similar in the past. His vital signs are normaI.
Digital rectal examination reveals an enlarged prostate that is smooth, nontender,
and without nodules. His prostate specific antigen (PSA) is 2.1 ng/dL (normal <4
ng/dL). Urinalysis is normal and urine culture shows no growth after 24 hours of
incubation.


Which of the following is the most likely diagnosis?

/ A. Benign prostatic hyperplasia
/ B. Carcinoma of the prostate
/ C. Prostatitis
/ D. Urethritis
/ E. Urinary tract infection



Explanation - Q: 4.1 Close

The correct answer is A. A patient with these complaints who has an
enlarged prostate, but normal laboratory values, is most likely suffering from
benign prostatic hyperplasia. This benign neoplasm of the prostate may take
years to become evident. The first signs of benign prostatic hyperplasia
(BPH) begin at about age 35 and involve the growth of benign tissue within
the transition zone of the prostate. The changes that occur within the urinary
tract are slow and insidious. The symptoms are classified as either
obstructive or irritative. Obstructive symptoms consist of a decrease in force
and caliber of the urine stream. There is also urinary hesitancy and
intermittency, with terminal dribbling and incomplete emptying. Irritative
symptoms include nocturia and frequency.
Carcinoma of the prostate (choice B) is rarely associated with urinary
symptoms. Although not absolute, patients will generally have either an
abnormality on rectal examination or an elevated PSA level. If the patient
does have prostate cancer and signs of urinary outflow obstruction, the
cancer is most likely locally advanced.
Patients with prostatitis (choice C) usually have a more rapid onset of
symptoms, which may be similar to this patient's complaints. However,
prostatitis is associated with a tender, boggy prostate and an abnormal
urinalysis, i.e., presence of white blood cells and/or leukocyte esterase or
nitrites in urine, and positive urine cultures.
Urethritis (choice D) is accompanied by dysuria and urethral discharge, and
most often represents a sexually transmitted disease.
Patients suffering from a urinary tract infection (choice E) may have
symptoms similar to those of this patient. However, they may have other
signs consistent with infection, i.e., fevers or chills. Urinalysis and urine
culture would be abnormal with a urinary tract infection.


Histologic examination of diagnostic tissue from this patient would most likely
reveal which of the following?

/ A. Adenocarcinoma
/ B. GIandular hyperplasia only
/ C. Stromal and glandular hyperplasia
/ D. Stromal hyperplasia only
/ E. Transitional cell hyperplasia



Explanation - Q: 4.2 Close

The correct answer is C. BPH produces nodular hyperplasia of the prostate
that begins in the periurethral tissue (the transition zone). These nodular
enlargements encroach on the lateral walls of the urethra to compress it to a
slit-like orifice. Microscopically, the nodularity is due to glandular proliferation
or dilatation as well as fibrous or muscular proliferation of the stroma
(choices B and D). The glands are often large with infoldings of acini
composed of tall columnar cells. The nuclei of the glands, however, show no
changes of malignancy.
BPH and adenocarcinoma (choice A) are two completely separate entities,
and, in general, do not occur simultaneously.
Transitional cells (choice E) line the urothelial tract. These cells are not seen
within the prostate.


Which of the following hormones is most likely responsible for this pathologic
process?

/ A. AIdosterone
/ B. Cortisol
/ C. Dihydrotestosterone
/ D. Estrogen
/ E. Testosterone



Explanation - Q: 4.3 Close

The correct answer is C. Dihydrotestosterone (DHT) is formed by the
conversion of testosterone to dihydrotestosterone by the enzyme 5-alpha
reductase, a nuclear membrane-bound enzyme found within the prostate.
Once formed, dihydrotestosterone is a more potent androgen than
testosterone due to its higher affinity for the prostatic androgen receptor. The
importance of DHT in prostate growth is dramatically demonstrated in
patients with congenital deficiency of 5-alpha reductase. Affected males have
ambiguous genitalia at birth, but at puberty, rising testosterone levels cause
normal virilization, functional erections, and ejaculation. The low levels of
serum DHT, however, result in a vestigial prostate that never develops BPH.
Administration of DHT results in prostatic growth.
Aldosterone (choice A) is a mineralocorticoid produced within the zona
glomerulosa of the adrenal cortex.
Cortisol (choice B) is the major glucocorticoid produced within the adrenal
cortex. Neither aldosterone or cortisol play a role in the hormonal
dependency of BPH.
Estrogens (choice D) are produced by men, and serum levels of estrogens
increase with age. However, their contribution to the development of BPH is
unclear at this time.
As previously stated, it is the conversion of testosterone to
dihydrotestosterone that has been implicated as the major contributor to
BPH, and not the testosterone itself (choice E).




This patient is started on terazosin to relieve his bladder outlet obstruction. This
medication works via which of the following mechanisms?

/ A. AIpha receptor blockade
/ B. Androgen receptor blockade
/ C. Anti-inflammatory action
/ D. Detrusor inhibition
/ E. Inhibition of 5-alpha reductase



Explanation - Q: 4.4 Close

The correct answer is A. Terazosin and other alpha-blockers are utilized as
first line management of BPH. The rationale for alpha-blockers in the
treatment of BPH is based on the hypothesis that clinical BPH is, in part, due
to prostate smooth muscle-mediated bladder outlet obstruction. By
preventing the stimulation of these smooth muscles within the prostate, the
prostate gland is able to "relax" and therefore it becomes less obstructive to
urinary outflow.
Androgen receptor blockers (choice B) (e.g., flutamide, bicalutamide) play a
role in the medical management of metastatic prostate cancer. They are not
utilized in men with BPH.
Anti-inflammatory agents (choice C) may be used in patients with prostatitis,
but are not useful for BPH.
Detrusor inhibitors (choice D), or antispasmodics, prevent uninhibited
detrusor contractions. Terazosin does not have the ability to do this.
Inhibiting the 5-alpha reductase enzyme (choice E) with finasteride will
decrease the amount of dihydrotestosterone available to the prostate for
growth stimulation. Terazosin has no ability to inhibit this enzyme.


This patient does not respond to medical management of his disorder and his
physician recommends transurethral resection of the prostate. Which of the
following best describes the anatomic location of the prostate with respect to the
urethra?

/ A. The ejaculatory ducts empty into the urethra distal to the prostate
/ B. The membranous urethra is distal to the prostatic urethra
/ C. The prostate does not surround the urethra
/ D. The prostate lies distal to the fossa navicularis
/ E. The prostate lies distal to the pendulous urethra



Explanation - Q: 4.5 Close

The correct answer is B. The male urethra is divided into four anatomic
parts. The easiest way to understand the relationship of these parts is to
follow the passage of urine as it leaves the bladder (the most proximal
location) and travels out the urethral meatus (the most distal location). From
the bladder, the urine is emptied into the prostatic urethra. This section is
completely surrounded by the prostate (compare with choice C) and also
contains the verumontanum, the site where the ejaculatory ducts enter the
urethra (compare with choice A). After traveling through the prostatic
urethra, the membranous urethra is encountered in its location distal to the
prostate (choice B). This portion of the urethra traverses the urogenital
diaphragm and contains most of the voluntary urinary sphincter.
The remainder of the distal portion of the urethra is considered the anterior
urethra. This portion contains the bulbar urethra, which is just distal to the
membranous urethra, as well as the pendulous urethra. The pendulous
urethra (choice E) is that portion of the urethra traveling within the portion of
the penis that can be palpated on routine exam.
The fossa navicularis (choice D) is a dilated portion of the urethra at the
distal aspect of the pendulous urethra.





Vignette 5 of 5

A 56-year-old, moderately obese man with long-standing type 2 diabetes
develops recurrent urinary tract infections (UTIs) over the past six months. He is
treated each time with oral antibiotics as an outpatient. The only symptoms that
he has are Iow-grade fevers and foul smelling, concentrated urine. There is no
back pain, dysuria, hematuria, urinary frequency, or incontinence. On physical
examination, his abdomen is soft, non-tender, non-distended, and there is no
suprapubic distention or discomfort. Pelvic examination is normaI. Urinalysis in
the office is consistent with another UTI.


What is the most likely cause of his recurrent urinary tract infections?

/ A. BIadder cancer
/ B. Neurogenic bladder
/ C. Small renal stone
/ D. Urethral diverticulum
/ E. Vesicoureteral reflux




Explanation - Q: 5.1 Close

The correct answer is B. Diabetes mellitus is the most common cause of
peripheral neuropathy in Europe and North America and the bladder does
not go unaffected. The exact incidence of symptomatic voiding dysfunction is
anywhere from 5% to 50% of those specifically questioned. The classic
description of the bladder neuropathy and its lower urinary tract effects is of a
primarily sensory neuropathy. The cycle is described is one of insidious
onset of impaired bladder sensation. This leads to a gradual increase in the
time interval between voiding and may progress to the point at which the
patient voids only once or twice a day without ever sensing any real urgency.
Investigation of these "diabetic bladders" will reveal impaired bladder
sensation, increased bladder capacity, decreased bladder contractility,
impaired urine flow, and increased residual urine. This residual urine is
retained in the bladder and increases the rate of urinary tract infection.
Although these patients live with an enlarged bladder that constantly
contains urine, their sensory abnormalities prevent them from realizing this.
Additionally, in an obese adult, it may be difficult to palpate the distended
bladder.
Cancer of the bladder (choice A) usually presents with hematuria, gross or
microscopic. Those patients with carcinoma in situ (CIS) may have urinary
urgency and frequency. The risk of UTI is not increased with bladder cancer.
Small renal stones (choice C) usually present with pain. Infection will occur if
the urinary drainage system is blocked. However, these patients are typically
much sicker and complain of back and/or abdominal pain. Large struvite
renal stones may cause recurrent UTI.
Urethral diverticulum (choice D) is uncommon. Most cases are probably
secondary to obstetric urethral trauma or severe urethral infection. While
these are associated with recurrent cystitis, they are usually palpable as a
rounded cystic mass in the anterior wall of the vagina that leaks from the
urethral orifice when pressure is applied.
Vesicoureteral reflux (choice E) is a common cause of urinary tract
infections in children. This occurs when urine backs up the ureters and into
the kidneys during voiding. The cause is an abnormal submucosal tunnel of
the distal ureters as they traverse the bladder wall. This is not commonly
seen in adults.


The patient's urine culture grows >100,000 colony forming units of Klebsiella
pneumoniae. Which of the following most accurately describes this bacterium?

/ A. Gram-negative diplococcus, oxidase-positive
/ B. Gram-negative rod, motile
/ C. Gram-negative rod with a capsule
/ D. Gram-positive coccus, catalase-positive
/ E. Gram-positive rod, spore forming

Explanation - Q: 5.2 Close

The correct answer is C. Klebsiella pneumoniae is a gram-negative rod that
has a capsule that helps to inhibit phagocytosis. This bacteria is also a
glucose and lactose fermenter and is nonmotile.
Neisseria meningitidis is a gram-negative diplococcus that is oxidase-positive
(choice A).
Proteus is a gram-negative rod that is very motile (choice B).
Staphylococcus aureus is a gram-positive coccus that is catalase-positive
(choice D).
Bacillus and Clostridia are both gram-positive rods that produce spores
(choice E).

Which of the following medications would be contraindicated in this patient?

/ A. Doxazosin
/ B. Finasteride
/ C. Oxybutynin chloride
/ D. Sildenafil
/ E. Tamsulosin



Explanation - Q: 5.3 Close

The correct answer is C. Oxybutynin chloride (trade name Ditropan) has a
direct antispasmodic effect on smooth muscle and blocks cholinergic
receptors that are on the bladder. This effect will inhibit detrusor muscle
contraction. This will lead to an increase in bladder capacity, something that
is already a problem with this patient. This patient's bladder needs all the
help in can get when it comes time to contract. You do not want to give any
medication which will inhibit these contractions.
Doxazosin and tamsulosin (choices A and E), are alpha-1 blocking agents,
that relax smooth muscle fibers of the prostate and bladder neck. Because of
their alpha-1 blocking ability, they may also cause hypotension by blocking
these receptors in the peripheral vascular system. These medications are not
contraindicated in this patient. Tamsulosin is a newer generation of alpha-1
receptor blocking agents. Compared to doxazosin and the other alpha-
blockers used in the treatment of benign prostatic hyperplasia, it is more
highly selective for the alpha receptors of the prostate. For this reason its risk
of hypotension is lower than the older generation of alpha-blockers.
Finasteride (choice B) prevents the enzyme 5-alpha reductase from
converting testosterone to dihydrotestosterone within the prostate.
Decreased levels of dihydrotestosterone in the prostate help stop the growth
of the prostate due to benign prostatic hyperplasia.
Sildenafil (choice D) inhibits the enzyme type 5 cyclic GMP
phosphodiesterase, which is primarily located within the cavernosal smooth
muscle of the penis. This leads to increased levels of cyclic GMP, causing
decreased intracellular calcium, which leads to relaxation of cavernosal
smooth muscle and therefore increased blood flow to the penis and an
erection. It has no effect on the detrusor muscle.

If this patient continues to have high residuals of urinary volume, the pressure in
the ureters will increase and exert back-pressure on structures, which in the
adult, form from which of the following embryological structures?

/ A. Mesonephric duct
/ B. Mesonephros
/ C. Metanephric blastema
/ D. Nephrotomes
/ E. Urogenital sinus


Explanation - Q: 5.4 Close

The correct answer is C. The back-pressure will be exerted on the kidneys.
The development of the kidneys is as follows: The intermediate mesoderm
on either side of the dorsal body wall gives rise to three successive nephric
structures of increasingly advanced design. The first is a small group of
transitory, nonfunctional, segmental nephrotomes (choice D), which develop
in the cervical region. These structures represent a vestige of the pronephroi
or primitive kidneys, which develop in some lower vertebrates.
As the cranial nephrotomes regress in the fourth week, they are succeeded
by a pair of elongated mesonephros (choice B), which develop in the
thoracic and lumbar regions. These mesonephros are functional, having
complete although simple nephrons. The mesonephros are drained by a pair
of mesonephric (Wolffian) ducts (choice A), which grow caudally to open into
the posterior wall of the primitive urogenital sinus. By the fifth week, a pair of
ureteric buds sprout from the distal mesonephric ducts and induce the
overlying sacral intermediate mesoderm to develop into the metanephric
blastema (or definitive kidneys) (choice C). The ureters and the collecting
duct system of the kidneys differentiate from the ureteric bud, whereas the
nephrons differentiate from the metanephric blastema.
The cloacal expansion of the hindgut is partitioned by the urorectal septum
into an anterior primitive urogenital sinus (choice E) and a posterior rectum.
This structure does not contribute to the definitive kidneys

A 16-year-old girl is brought to a pediatrician because her mother is concerned
about her weight loss over the last year. When the pediatrician
attempts to interview the girI, she is clearly hostile, denies that she has any
medical problem, and states that she is just trying to "Iook good."
The girI's height is 175 cm (5'9") and her weight is 41.4 kg (91 pounds). AIthough
the girl is unwilling to discuss her weight with the physician,
her mother reports that her daughter is convinced that she is too fat. The mother
states that the girl is always finding excuses to not eat, or at
the most, to eat only a few bites of fruit, even though she is still interested in
cooking elaborate meals for her family. On one occasion, she
discovered her daughter vomiting after a meaI, and believes that the vomiting
was self-induced.
Question 1 of 6
Which of the following is the normal range for body mass index (BMI)?
/ A. Below 16
/ B. 16-19
/ C. 19-25
/ D. 25-35
/ E. Over 35

Explanation - Q: 1.1 Close

The correct answer is C. Body mass index is coming into increasing use
when evaluating weight problems, because it provides a simple calculation
that allows individuals of a variety of heights and weights to be classified as
underweight, overweight, or in the normal range. The normal range for BMI is
19 to 25.
Individuals with BMI below 16 (choice A) are considered anorexic and those
with BMI 16-19 (choice B) are considered underweight.
Individuals with BMI 25-35 (choice D) are considered mildly to moderately
obese and those with BMI greater than 35 (choice E) are considered
severely obese.



Question 2 of 6
This girI's body mass index is which of the following?
/ A. 10.5
/ B. 13.5
/ C. 16.5
/ D. 21.5
/ E. 27.5


Explanation - Q: 1.2 Close

The correct answer is B. Body mass index was originally a European
concept, and is calculated as the body weight in kilograms divided by the
square of the height in meters. In this country, we need to convert from
pounds to kilograms and from feet and inches to meters in order to perform
the calculation (a calculator helps). This girl's body weight is 91 pounds,
which is divided by the conversion factor 2.2, yielding 41.4 kilograms. Her
height is 5 x 12 + 9 = 69 inches, which is divided by the conversion factor
39.4 to get the height in meters, in this case 1.75 meters. Now calculate the
BMI as 41.4 kilograms/ (1.75 meters x 1.75 meters) = 13.5. This girl has a
BMI in the anorexic range.


Question 3 of 6
Which of the following is the most likely diagnosis?
/ A. Anorexia nervosa
/ B. Binge eating disorder
/ C. Body dysmorphic disorder
/ D. Bulimia nervosa
/ E. Normal adolescent developmental issues
/ F. Orthorexia nervosa


Explanation - Q: 1.3 Close

The correct answer is A. It is not uncommon for individuals with severe
eating disorders to be very unwilling to seek help, as in this case. This
markedly underweight girl, who is still convinced that she is fat and is dieting
compulsively, most likely has anorexia nervosa. Anorexia nervosa is best
understood as an eating disorder with a large psychological component in
which an individual engages in the relentless pursuit of thinness. As many as
50% of anorexics also have binge and purge behavior. On a practical basis,
the condition is usually diagnosed when the weight is 85% or less than what
is expected for age and height. The psychological component usually
includes a perception of being overweight despite evidence to the contrary, a
terror of gaining weight, and a denial of the dangers of very low weight. Many
theorists cite our society's preoccupation with slenderness as the premier
standard for female beauty as a major contributing factor to this disease.
While over 90% of patients with anorexia are young women who develop it in
their teens to early 20's (peak age 16), the condition can also be seen in
children, middle-aged adults, boys, and men.
Binge eating disorder (choice B) refers to binge eating not followed by
vomiting, and is often accompanied by obesity.
Body dysmorphic disorder (choice C) is separated from anorexia nervosa by
psychiatrists because it is based on a preoccupation with "ugliness" rather
than "fat"; patients with this condition are likely to make numerous
statements like "my nose is terrible." In body dysmorphic disorder, the focus
is generally on a part of the body (face, buttocks) rather than the body image
as a whole.
Bulimia nervosa (choice D) is characterized by episodes of binge eating
followed by vomiting or purging with laxatives. There is some overlap with
anorexia nervosa, but many patients have normal weight to mild obesity.
Normal adolescent developmental issues (choice E) often involve concern
for weight, self-image and sexual attractiveness. However, the case steps
outside the normal range. The girl's overall weight, the hostility regarding
discussing these issues, and the self-induced vomiting are all indications of
something outside the range of normal.
Orthorexia nervosa (choice F), while not a formally accepted medical
classification, is a term in common use in the discussion of eating disorders,
and refers to a pathologic fixation on eating "pure" or "superior" food.

Question 4 of 6
This girl would most likely also have which of the following menstrual disorders
as a complication of her condition?
/ A. Amenorrhea
/ B. Dysfunctional uterine bleeding
/ C. Premenstrual syndrome
/ D. Primary dysmenorrhea
/ E. Secondary dysmenorrhea


Explanation - Q: 1.4 Close

The correct answer is A. Women with anorexia nervosa do not usually
have menstrual periods. Many medical specialists require the presence of
amenorrhea before making a diagnosis of anorexia nervosa in women.
Depending upon the age at which the anorexia begins, they may either never
have periods (primary amenorrhea) or may have periods that cease when
they lose weight (secondary amenorrhea). The other conditions listed are not
specifically related to anorexia nervosa.
Dysfunctional uterine bleeding (choice B) is abnormal uterine bleeding
(prolonged flow, irregular flow, or profuse flow) not associated with tumor,
inflammation, or pregnancy.
Premenstrual syndrome (choice C) refers to the presence of nervousness,
irritability, emotional instability, anxiety, headaches, edema, or mastalgia
during the 7 to 10 days before the onset of menstruation.
Dysmenorrhea is menstrual pain, and may be either primary (choice D) and
unrelated to demonstrable reproductive pathology or secondary (choice E)
to other reproductive disease.


Question 5 of 6
The most common cause of death in this condition is arrhythmia, which is usually
related to which of the following electrolyte abnormalities?
/ A. High serum calcium
/ B. High serum iron
/ C. Low serum magnesium
/ D. Low serum potassium
/ E. Low serum sodium


Explanation - Q: 1.5 Close

The correct answer is D. While many fluid and electrolyte abnormalities can
be seen in patients with anorexia nervosa, low serum potassium is
particularly common, and may cause a fatal arrhythmia. The low serum
potassium is usually initially due to very low intake, but may be exacerbated
by processes that increase body potassium losses, such as with vomiting,
diuretic use, or heavy laxative use. Large numbers of mild cases of anorexia
nervosa are never diagnosed and may not develop severe medical
problems. Mortality rates among diagnosed cases are surprisingly high, and
may be up to 10-20% among the most severe cases without effective
intervention. In addition to arrhythmias, anorexics may develop potentially
fatal liver or renal disease, teeth problems, esophageal rupture (in bulimics
compulsively vomiting), a weakened immune system, anemia, malnutrition,
impaired mentation, and salivary duct stones. Severe anorexics are initially
treated with hospitalization to prevent death, suicide, and medical crises. It is
of particular concern to the medical community serving these patients that
our present system of health maintenance organizations often limits the
hospitalization of these individuals to less than one month, when four or
more months may be required to bring them away from the edge of death.
Psychiatric help is usually only effective after a near normal weight has been
medically achieved. The other answers listed in the choices are distracters.








Question 6 of 6
Over the next several years, in addition to the treatment of her current condition,
this girl should also be monitored for the possible onset of
which of the following?
/ A. Acute stress disorder
/ B. Agoraphobia
/ C. Conversion disorder
/ D. Depression
/ E. Panic disorder
/ F. Substance abuse

Explanation - Q: 1.6 Close

The correct answer is D. Between 50% to 80% of patients diagnosed with
anorexia nervosa also suffer from depression at some point in their lives.
Acute stress disorder (choice A) is defined by mental reexperiencing of a
traumatic event, avoidance of stimuli that remind the person of the event and
a host of secondary symptoms including sleep disruption or excess,
withdrawal from the world, impulsive behavior, headaches and somatic
complaints. Symptoms must resolve in less than one month. If they do not,
we use the diagnosis of posttraumatic stress disorder. There is no
association between anorexia and acute stress disorder.
Agoraphobia (choice B) suggests a fear of being open, vulnerable and
exposed when out and about in the community. There is no link between this
anxiety disorder and anorexia.
Conversion disorder (choice C) is defined by the loss of physical functioning
due to psychological etiology. Generally, the symptom is focused on the loss
of the use of a limb or one of the primary senses. There is no correlation of
this disorder with anorexia.
Panic disorder (choice E) is characterized by the occurrence of three panic
attacks in a three-week period. A panic attack is an overwhelming, sudden
physiological arousal featuring tachycardia, profuse sweating,
hyperventilation, and feelings of doom and dread. There is no association
between anorexia and panic disorders.
Substance abuse (choice F) is relatively uncommon with anorexic patients.
They seem obsessively concerned with which types of substances they are
willing to put into their bodies.


A 30-year-old woman has noticed a gradual increase in weight over the past
eighteen months. The fat deposition has occurred mostly in the
face, neck, trunk, and abdomen. She has also noticed an increase in black facial
hairs that has been accompanied by acne. Physical
examination reveals the presence of purple abdominal striae, but generalized
hyperpigmentation is absent. A poorly-healed laceration is
observed on the right shin. BIood pressure is 155/100 mm Hg. The physician
suspects Cushing disease.


Question 1 of 5
Which of the following sets of hormone levels would be most consistent with this
diagnosis?



Explanation - Q: 2.1 Close

The correct answer is D. Pituitary ACTH-dependent Cushing syndrome
(Cushing disease) is the most frequent cause of Cushing syndrome. It is
responsible for 70% of reported cases. It is characterized by increased
plasma ACTH and increased serum cortisol. Another hallmark of Cushing
disease is the absence of the normal diurnal rhythm in plasma cortisol. This
rules out choice C, which shows a prominent decrease in PM cortisol levels.
Plasma ACTH is typically only modestly increased. Hyperpigmentation is
rare.
Choice A is incorrect because serum cortisol levels are low. The increased
plasma ACTH is consistent with primary adrenal insufficiency (Addison
disease).
Choice B is incorrect because serum cortisol levels are low. The lower-than-
normal plasma ACTH is consistent with secondary (pituitary) adrenal
insufficiency.
Choice E shows increased serum cortisol and absence of diurnal rhythm, but
plasma ACTH is below normal. These laboratory values would be consistent
with Cushing syndrome caused by a cortisol-secreting adrenal adenoma
(primary hypercortisolism).


Question 2 of 5
ACTH measured in blood drawn from which of the following vessels would give
the best indication of direct anterior pituitary secretion?
/ A. Inferior petrosal sinus
/ B. Internal jugular vein
/ C. Sigmoid sinus
/ D. Sphenoid sinus
/ E. Transverse sinus

Explanation - Q: 2.2 Close

The correct answer is A. This question may seem difficult, but the key is the
fact that the pituitary is located near the cavernous sinuses. Venous drainage
from the anterior pituitary, although variable, can be expected to flow, via the
nearby cavernous sinuses, posteriorly into the superior and inferior petrosal
sinuses before draining, via the sigmoid sinuses, into the internal jugular
veins. A skilled invasive radiologist can sample blood bilaterally from the
inferior petrosal sinuses. If the ACTH concentration in the sinus blood is
greater than twice the concentration in a peripheral vein, Cushing disease is
likely. Bilateral sampling of blood from the inferior petrosal sinuses can also
locate the tumor to one side or the other.
The internal jugular vein (choice B) receives blood from a number of other
veins in addition to the petrosal sinuses. Hence, the pituitary ACTH is diluted
considerably by the time it reaches the jugular vein.
The sigmoid sinus (choice C) receives drainage via the petrosal sinuses
before flowing into the internal jugular vein, but also receives flow from the
larger transverse sinuses (choice E), which dilutes the ACTH concentration.
The transverse sinus receives drainage from a number of cerebral and
cerebellar locations, which does not include flow from the anterior pituitary.
The sphenoid sinus (choice D) is part of the nasal system. It is an air-filled
cavity and does not carry blood.


Question 3 of 5
Which of the following is considered the regulated step in steroid hormone
synthesis in the zona fasciculata?
/ A. Cholesterol to pregnenolone
/ B. Corticosterone to aldosterone
/ C. 11-Deoxycortisol to cortisol
/ D. Pregnenolone to progesterone
/ E. Progesterone to 17- -hydroxyprogesterone


Explanation - Q: 2.3 Close

The correct answer is A. The conversion of cholesterol to pregnenolone is
considered the rate-limiting and regulated step in steroid biosynthesis in the
zona fasciculata, which secretes the glucocorticoid, cortisol. ACTH binds to a
membrane-associated receptor and increases cAMP formation. The
subsequent activation of protein kinases and phosphorylation results in
increased expression of LDL receptors and activation of cholesterol
esterase. The increase in LDL receptors increases the delivery of cholesterol
to the zona fasciculata. The activation of cholesterol esterase increases the
liberation of intracellular cholesterol from cholesterol esters. ACTH also
induces a mitochondrial protein called steroidogenic acute regulatory (StAR)
protein. StAR functions to shuttle cholesterol from the outer mitochondrial
membrane to the inner mitochondrial membrane, where the cholesterol side
chain cleavage enzyme converts it to pregnenolone.
Corticosterone is converted to aldosterone (choice B) in the zona
glomerulosa, but not in the zona fasciculata. This is also a regulated step.
Angiotensin II acts via a membrane-associated receptor to increase the
activity of aldosterone synthase (a combination of two enzymes with 18-
hydroxylase activity) and, hence, increase aldosterone synthesis and
secretion.
11-deoxycortisol is converted to cortisol (choice C), primarily in the zona
fasciculata, but also in the zona reticularis. The enzyme responsible is 11-
hydroxylase, a mitochondrial enzyme. 11-hydroxylase is not regulated by
ACTH.
Pregnenolone is converted to progesterone (choice D) in all steroid
secreting tissues. The enzymes responsible are 3-ol dehydrogenase and
D4,5-isomerase. These enzymes are cytoplasmic, and are not regulated by
ACTH.
The conversion of progesterone to 17-progesterone (choice E) is catalyzed
by 17-hydroxylase, which is cytoplasmic. This enzyme is not found in the
zona glomerulosa, which explains why this layer of the adrenal cortex does
not secrete cortisol or sex steroids. 17-hydroxylase is not regulated by
ACTH.




Question 4 of 5
MRI of the pituitary with gadolinium enhancement shows an 8 mm mass which is
subsequently removed using microsurgery. Pathologic
examination of the mass would most likely show which of the following?
/ A. Benign adenoma
/ B. Craniopharyngioma
/ C. Diffuse hyperplasia of corticotroph cells
/ D. Malignant adenocarcinoma
/ E. Metastatic breast carcinoma



Explanation - Q: 2.4 Close

The correct answer is A. A benign pituitary adenoma is present 90% of the
time in patients with Cushing disease. The tumor is usually less than 10 mm
in diameter. These microadenomas are not encapsulated, but are
surrounded by a rim of compressed normal anterior pituitary cells. The
microadenomas also typically exhibit bundles of perinuclear microfilaments
surrounding the nucleus (Crooke's changes). Electron microscopy shows the
presence of secretory granules (200-700 nm in diameter) within the
cytoplasm of the adenoma cells. It should be noted that 10% of adults
between 20 and 50 years of age have incidental pituitary tumors and no
apparent symptoms.
Craniopharyngioma (choice B) is the most common tumor that impairs
hypothalamic-pituitary function in children and young adults. It may also
occur in adults. These intracranial tumors are usually benign, but may
become large before being detected. Endocrine abnormalities are common,
but may be discovered only after other symptoms related to increased
intracranial pressure become manifest. Growth hormone deficiency is the
most common endocrine abnormality, but any of the anterior pituitary
hormones may be diminished.
Diffuse hyperplasia of corticotroph cells (choice C) of the anterior pituitary is
rare. When it does occur, it is presumably due to excessive stimulation of the
pituitary by corticotropin-releasing hormone (CRH) secreted by a benign
hypothalamic gangliocytoma.
Malignant adenocarcinoma (choice D) is rare in the anterior pituitary. These
tumors are not functional and do not secrete ACTH.
Breast tumor cells (choice E) have been known to metastasize to the
anterior pituitary. These are space-filling growths that compromise pituitary
function. They do not secrete ACTH.
Question 5 of 5
Following the pituitary microsurgery, the patient complains of being confused,
fatigued and nauseated. She also has hypoglycemia. Cold
intolerance is not present. Treatment with which of the following is most likely
indicated?
/ A. Estrogen
/ B. FIudrocortisone (mineralocorticoid)
/ C. Human growth hormone
/ D. Hydrocortisone (glucocorticoid)
/ E. Thyroxine


Explanation - Q: 2.5 Close

The correct answer is D. The prolonged increase in cortisol due to the
ACTH-secreting adenoma will have suppressed ACTH secretion from the
uninvolved corticotrophs of the anterior pituitary. Following removal of the
adenoma, the pituitary will not secrete enough ACTH at first, leading to
adrenal insufficiency. The symptoms of the patient are all consistent with low
blood levels of cortisol. Glucocorticoid replacement is indicated, and may be
needed for up to a year before normal function of the pituitary returns.
Depending on the extent of the pituitary microsurgery, a significant number of
gonadotrophs might also be removed and lead to hypogonadotropic
hypogonadism. In this case, estrogen (choice A) replacement may be
necessary. However, the symptoms of this patient are not consistent with
estrogen deficiency.
Since aldosterone secretion from the zona glomerulosa of the adrenal cortex
is primarily regulated by angiotensin II, removal of an ACTH-secreting
pituitary adenoma has little effect on its blood level. Hence, mineralocorticoid
(choice B) replacement is rarely required.
Pituitary microsurgery also has the potential to remove somatotrophs.
However, growth hormone deficiency is not likely, given the fact that about
50% of the cells in the anterior pituitary are somatotrophs. Furthermore,
somatotrophs are located primarily in the lateral wings of the pituitary, while
ACTH-secreting corticotrophs tend to be more medial. Hence, growth
hormone (choice C) replacement is less likely to be required than cortisol
replacement.
Removal of thyrotrophs is also possible with pituitary microsurgery. However,
the patients symptoms are not consistent with hypothyroidism, and thyroxine
(choice E) replacement is not required.



A 35-year-old man presents to his physician with a 40-pound weight loss over the
preceding 4 months. The man has not been dieting, but has
been having abdominal pain, nausea, and diarrhea. He has also had a chronic
cough. Sputum studies demonstrate Pneumocystis carinii, and
esophagogastroduodenoscopy with biopsy demonstrates Candida and Herpes in
the esophagus.

Question 1 of 9

The pathogen which causes this patient's underlying disease is a member of
which of the following groups of viruses?
/ A. Caliciviridae
/ B. Coronaviridae
/ C. FIaviviridae
/ D. Picornaviridae
/ E. Retroviridae

Explanation - Q: 3.1 Close

The correct answer is E. This man with multiple infections probably has
AIDS, in which "wasting" due to infection and weight loss is common. All
three of this patient's secondary infections (Pneumocystis pneumonia,
Candida esophagitis, and Herpes esophagitis) are considered to be "AIDS-
defining illnesses," that indicate that the patient's HIV infection has
progressed to the point of producing profound immunosuppression. HIV is a
member of the Lentivirus subgroup of retroviruses. This viral family is
characterized by prolonged infections, often with a latent period. These
viruses are usually inadequately opposed by the host immune response, and
often can infect the nervous system. The Retroviridae also include the
oncovirus group that includes the human T-cell Leukemia/Lymphotropic
virus.
The Caliciviridae (choice A) includes the Norwalk virus and the virus that
causes Hepatitis E.
Coronaviruses (choice B) cause upper respiratory illnesses.
Flaviviruses (choice C) include the Hepatitis C virus, yellow fever virus,
dengue virus, and several encephalitis viruses.
Picornaviruses (choice D) include poliovirus, coxsackieviruses, echoviruses,
hepatitis A virus, and rhinoviruses.






Question 2 of 9
Approximately how many people worldwide are infected with the virus that
causes this patient's underlying disease?
/ A. 430,000
/ B. 750,000
/ C. 1 million
/ D. 16 million
/ E. 33 million

Explanation - Q: 3.2 Close

The correct answer is E. Currently, about 33 million people worldwide have
HIV infection, and there have been 16 million (choice D) deaths. The vast
majority of these cases are in developing countries in Africa and Southeast
Asia. In the United States, we presently have approximately 1-2 million
(choice C) individuals with asymptomatic HIV infection and nearly 750,000
individuals (choice B) with AIDS. Over 430,000 (choice A) of US patients
have died of AIDS.



Question 3 of 9
Which of the following is the most frequent means of transmission worldwide for
the virus that causes this patient's underlying disease?
/ A. Accidental needle exposures by health care workers
/ B. Infected blood products
/ C. Perinatal transmission
/ D. Sexual contact
/ E. Shared needles by drug abusers




Explanation - Q: 3.3 Close

The correct answer is D. Over 70% of HIV infection is acquired via sexual
transmission. You may not be aware that worldwide HIV infection is more
common in heterosexual men and women than in homosexual men. Most
HIV cases in the US were initially in male homosexuals. However, more
recently there has been a shift in the demographics and the majority of new
cases of HIV infection are in the heterosexual population.
Infection of health care workers through accidental needle exposures
(choice A) is now rare in the United States because of the widespread use
of universal blood precautions.
In this country, contaminated blood products (choice B) only rarely transmit
the HIV virus. In contrast, in developing countries, contaminated blood
products are an important problem, as many of these countries have high
HIV rates and do not have the financial resources to adequately screen the
blood.
Most children who develop AIDS acquire the infection perinatally (choice C).
Most of the individuals in the United States who acquire AIDS via a
parenteral route are intravenous drug abusers who share needles (choice
E).


Question 4 of 9
This virus has a particularly high affinity for which of the following?
/ A. CD4 T Iymphocytes and B cells
/ B. CD4 T Iymphocytes and monocytes.
/ C. CD8 T Iymphocytes and B cells
/ D. CD8 T Iymphocytes and CD4 T Iymphocytes
/ E. CD8 T Iymphocytes and monocytes

Explanation - Q: 3.4 Close

The correct answer is B. HIV has a particular tendency to bind to CD4 T
lymphocytes (helper T lymphocytes) and monocytes and then become
internalized. Most other cells, include CD8 T lymphocytes (suppressor T
cells) and B cells, are not particularly susceptible to the HIV virus.



Question 5 of 9
This virus uses which of the following enzymes to produce a DNA copy of itself?
/ A. Integrase
/ B. gp120
/ C. p24
/ D. Protease
/ E. Reverse transcriptase

Explanation - Q: 3.5 Close

The correct answer is E. The viral enzyme reverse transcriptase is used to
generate a DNA copy from RNA. This viral DNA is then inserted into the host
DNA, which allows the virus to "hide" in the host DNA, and also permits more
copies to be made.
Reverse transcriptase is a product of the gene Pol, which also codes for
integrase (choice A, which integrates proviral dsDNA into host DNA) and
protease (choice D, which cleaves polyprotein).
The Env gene codes for gp120 (choice B, a surface protein that binds CD4
on the host cell and is responsible for tropism) and gp41 (a transmembrane
protein for cell fusion).
The Gag gene codes for group-specific antigens including p24 (choice C, a
capsid protein), p7p9 (core nucleocapsid proteins), and p17 (matrix proteins
that stabilize the envelope).
The HIV genome also has several regulatory genes including LTR
(integration and virus gene expression), Tat (transactivator of transcription
that functions in upregulation), Rev (upregulates transport of transcripts to
cytoplasm), and Nef (decreases major histocompatibility complex type I
expression on infected T cells, thereby protecting the infected cells from the
immune system).



Question 6 of 9
Which of the following is the typical period of latency between initial infection with
this virus and development of clinically evident disease?
/ A. 1 year
/ B. 6 years
/ C. 11 years
/ D. 16 years
/ E. 21 years

Explanation - Q: 3.6 Close

The correct answer is C. HIV infection remains a mostly fatal illness, since
it still eventually causes a profound immunosuppression, which leaves
individuals vulnerable to a variety of other infections. While there is some
variation from case to case (dependent upon both the size of the initial
inoculation and host resistance factors), it typically takes 11 years in
untreated cases for HIV infection to progress to clinical AIDS. The
introduction of regimens of multiple antiretroviral drugs and prophylaxis
against opportunistic pathogens may be changing this pattern, but it is still
too early to know to what degree.



Question 7 of 9
The patient is prescribed a non-nucleoside reverse transcriptase inhibitor. Which
of the following agents was most likely prescribed?
/ A. Indinavir
/ B. Lamivudine
/ C. Lopinavir
/ D. Nevirapine
/ E. Zidovudine


Explanation - Q: 3.7 Close

The correct answer is D. As we have developed a wider variety of methods
of treating AIDS, the task of remembering the different drugs that can be
used has become more complex. Most of the useful drugs with direct activity
against the HIV virus block either reverse transcriptase or protease. Of the
drugs listed in the choices, only nevirapine is classified as a non-nucleoside
reverse transcriptase inhibitor.
Indinavir (choice A) and lopinavir (choice C) and are classified as protease
inhibitors.
Lamivudine (choice B) and zidovudine (choice E) are thymidine analogues
that block reverse transcriptase.



/ A. Amphotericin B
/ B. CIotrimazole
/ C. Ganciclovir
/ D. Pyrimethamine
/ E. Trimethoprim/sulfamethoxazole

Explanation - Q: 3.8 Close

The correct answer is E. The choices of drugs for the therapy of the wide
variety of secondary infections AIDS patients develop is also of considerable
concern to medical personnel. Pneumocystis pneumonia is most often
treated with trimethoprim/sulfamethoxazole (Bactrim), with an alternative
drug for patients allergic to trimethoprim/sulfamethoxazole being
pentamidine. Early steroid therapy may also be helpful in hospitalized
patients with Pneumocystis pneumonia who have a PaO2 less than 70 mm
Hg.
Amphotericin B (choice A) is a systemic antifungal agent typically used in
AIDS patients to treat cryptococcal meningitis.
Clotrimazole (choice B) is used in AIDS patients to treat oral candidiasis.
Ganciclovir (choice C) is used in AIDS patients to treat CMV infections.
Pyrimethamine (choice D) is used in AIDS patients to treat toxoplasmosis.


Question 9 of 9
Individuals with this patient's underlying disease also have an increased chance
of developing which of the following cancers?
/ A. BIadder cancer
/ B. Breast cancer
/ C. Cervical cancer
/ D. Prostate cancer
/ E. Testicular cancer

Explanation - Q: 3.9 Close

The correct answer is C. Individuals with AIDS also have an increased
cancer rate of tumors known to have viral associations, including Kaposi's
sarcoma (related to infection with Human Herpes Virus 8); Burkitt's,
immunoblastic, and primary CNS lymphomas (related to Ebstein-Barr virus
infection); and invasive cervical cancer (related to HPV infection).
Cancers of the bladder, breast, prostate, and testes (choices A, B, D, and
E) are not related to any known viral infection.


A 3-year-old boy is evaluated by a pediatric endocrinologist because of
excessive weight gain. The boy has had problems since birth. He was
noted to have neonatal central hypotonia, which has improved somewhat since 1
year of age. He was also noted as a baby to have an
unusually long head (dolichocephaly) with almond-shaped eyes and a small
mouth. His testes were undescended and he had a small penis.
Initially, he had feeding problems with poor weight gain in infancy, but since 1
year of age he has shown excessive weight gain. As a baby, he
was lethargic and had a weak cry. His motor milestones and speech
development have been delayed.






Question 1 of 5

Which of the following is the most likely diagnosis?
/ A. Angelman syndrome
/ B. Down syndrome
/ C. Prader-Willi syndrome
/ D. Triple X syndrome
/ E. Turner syndrome

Explanation - Q: 4.1 Close

The correct answer is C. In general, when you encounter a child who has
had problems since infancy and who has a distinctive facial appearance, you
should consider the possibility that the child has a genetic abnormality (often
that involves all or a significant portion of a chromosome). Excessive weight
gain is a specific cue to think of Prader-Willi syndrome, although you should
be aware that, as illustrated in this case, this may not develop until 1 to 6
years of age. The other conditions listed in the answer options do not have a
marked tendency to develop obesity.
Angelman syndrome (choice A) is often considered together with Prader-
Willi syndrome for genetic reasons (see question 3 below), but has a very
different clinical presentation with severe mental retardation, gait
irregularities, and a tendency to uncontrolled laughter ("happy puppet"
syndrome).
Down syndrome (choice B) is characterized by mental retardation, abnormal
facies (with a flat hypoplastic face with short nose and small low set ears), a
tendency to develop lenticular opacities, heart disease, leukemia, and
Alzheimer disease.
Triple X syndrome (choice D) produces phenotypic females that are usually
apparently normal.
Turner syndrome (choice E) produces phenotypic females with short stature,
hypogonadism, amenorrhea, webbed neck, and a predisposition for
coarctation of the aorta.





Question 2 of 5
This patient's probable genetic disorder most likely involves which of the
following chromosomes?
/ A. 15
/ B. 18
/ C. 21
/ D. X
/ E. Y


Explanation - Q: 4.2 Close

The correct answer is A. Prader-Willi syndrome is due to a defect in the
long arm of chromosome 15 in the 15q11-q13 region.
Associate chromosome 18 (choice B) with Edwards syndrome (trisomy 18).
Associate chromosome 21 (choice C) with Down syndrome (trisomy 21).
Associate the X chromosome (choice D) with Turner syndrome (XO), triple X
syndrome (XXX), and Klinefelter syndrome (XXY).
Associate the Y chromosome (choice E) with Klinefelter syndrome (XXY).



Question 3 of 5
This patient's probable genetic disorder is often cited as an example of which of
the following genetic mechanisms?
/ A. Genetic imprinting
/ B. Monosomy
/ C. Robertsonian translocation
/ D. Trinucleotide repeats
/ E. Trisomy

Explanation - Q: 4.3 Close

The correct answer is A. Genetic imprinting refers to the finding that certain
genes must come from a specific parent in order to function. The molecular
basis of the phenomenon appears to be that male and female germ cells
apparently add methyl groups to different areas of DNA as a post-
translational modification. These areas of methylation then affect when and
how the affected genes are transcribed. (This is part of a broader
phenomenon that determines how cells differentiate, based on which genes
are "turned on" or turned off.") There has been a great deal of research
interest in both Prader-Willi syndrome and Angelman syndrome (see brief
description in the discussion of question 1) since it was discovered that these
two very different conditions both seemed to be due to a deletion in the same
area of chromosome 15. The initial understanding was that if the deletion
was in the father's chromosome, then Prader-Willi syndrome occurred, while
if it was in the mother's, Angelman syndrome occurred. In practice, for the
USMLE, this is probably the level of detail you will need to remember. (What
actually turns out to be the case is that in this region of the genome there are
at least two genes that require paternal imprinting and whose absence
causes Prader-Willi syndrome, and there is at least one gene requiring
maternal imprinting, whose absence causes Angelman syndrome.)
With respect to the other choices listed in the question, associate monosomy
X (choice B) with Turner syndrome (monosomies involving autosomal
chromosomes are fatal in utero).
Robertsonian translocations (choice C) are a specialized form of
translocation in which the centromeres of two acrocentric chromosomes
appear to have fused to make a new chromosome containing both long arms
of the original chromosomes; this can which behave like a trisomy, and
accounts for some cases of Down syndrome.
Trinucleotide repeats (choice D) are highly repeated short segments of DNA
that tend to be seen in autosomal dominant diseases that show progression
in severity with succeeding generations, such as Fragile X syndrome and
Huntington disease.
Common trisomies (choice E) include Down syndrome, Edwards syndrome,
and Patau syndrome.


Question 4 of 5
If this child goes on to develop morbid obesity, which of the following
complications would be most likely to occur?
/ A. Aortic dissection
/ B. Crohn disease
/ C. Diabetes mellitus
/ D. Pheochromocytoma
/ E. Zollinger-EIIison syndrome

Explanation - Q: 4.4 Close

The correct answer is C. The Prader-Willi patients have markedly increased
appetite secondary to hypothalamic dysfunction and a much less than
normal caloric need (secondary primarily to low muscle mass, which may be
somewhat ameliorated by growth hormone supplementation during childhood
and adolescence). To maintain a normal weight, they require a daily caloric
intake about half of those of normal individuals, only around 1000 calories
per day. If they do not have this severe of a dietary restriction, they will
eventually develop morbid obesity, which is often complicated by diabetes
mellitus type II. Other complications to which they are particularly vulnerable
include osteoporosis often with scoliosis, hypertension, right-sided heart
failure, a tendency to pick at skin lesions, dental problems, and a tendency to
have an exaggerated response to sedatives and anesthetics. The other
choices listed in the question are distracters and are not associated with
Prader-Willi syndrome.


Question 5 of 5
Over the next decade, this child has progression of his disease. When he is seen
as a teenager, he is noted to be mildly mentally retarded,
and shows multiple behaviors distressing to those around him. He spends a
great deal of time ravenous and being pre-occupied with food to
the point of breaking windows to get into the locked kitchen and storeroom. He
almost always sleeps poorly, and has temper tantrums, violent
outbursts, obsessive-compulsive behavior, a tendency to be argumentative, and
he steals (most often food or things to get money for food)
and lies (most often about food). He is late going through puberty. The
physiologic basis of the feeding and sleep problems that dominate this
clinical picture is thought to primarily involve dysfunction of which of the following
parts of the brain?
/ A. Cerebellum
/ B. Cerebral cortex
/ C. Hypothalamus
/ D. Medulla
/ E. Pons

Explanation - Q: 4.5 Close

The correct answer is C. The genetics of Prader-Willi syndrome are
fascinating, and the disease is often taught to medical students by
geneticists. While this can leave a medical student with a great appreciation
of how far we have progressed in our ability to sort through the genetic basis
of disease, it has the disadvantage that the clinical side of the picture is often
comparatively neglected. It tends to be forgotten that Prader-Willi syndrome
is a disease with a fairly high incidence of 1:12,000-15,000 (affecting both
sexes and all races), which may have a devastating impact on the patient
and family. This question illustrates some of the problems that are seen.
Severe hypothalamic dysfunction leading to chronic insatiable hunger and
chronic sleep deprivation (which may be aggravated by obesity-related sleep
apnea) are thought to be a major part of the underlying cause of most of
these patients' sometimes striking behavior problems, and are believed to be
related to severe dysfunction of the hypothalamus. Added to their
unhappiness are the problems that come with the irritability this produces,
particularly when it affects interpersonal relationships with those around
them. The families of these patients are also usually very unhappy as well,
secondary to concern about the patient, constant emotional turmoil, the
financial pressures of chronic illness, and often nearly irreconcilable
differences among family members about how to fairly allocate financial and
time resources between the affected child and other family members. (Often
what evolves is that one parent becomes the affected child's principal
advocate and caregiver, while the other parent becomes the advocate and
caregiver for the remainder of the family.) The other choices listed in the
question are distracters.


Normal Lab Values Close

* = Included in the Biochemical Profile (SMA-12)

REFERENCE RANGE
SI REFERENCE
INTERVALS
BLOOD, PLASMA, SERUM
* Alanine aminotransferase (ALT,
GPT at 30C)
8-20 U/L 8-20 U/L
Amylase, serum 25-125 U/L 25-125 U/L
* Aspartate aminotransferase (AST,
GOT at 30C)
8-20 U/L 8-20 U/L
Bilirubin, serum (adult) Total //
Direct
0.1-1.0 mg/dL // 0.0-0.3
mg/dL
2-17 mol/L // 0-5
mol/L
* Calcium, serum (Total) 8.4-10.2 mg/dL 2.1-2.8 mmol/L
Cholesterol, serum < 200 mg/dL < 5.2 mmol/L

Cortisol, serum
0800 h: 5-23 g/dL // 1600
h: 3-15 g/dL
138-635 nmol/L // 82-
413 nmol/L

2000 h: 50% of 0800 h
Fraction of 0800 h:
0.50
Creatine kinase, serum (at 30C)
ambulatory
Male: 25-90 U/L 25-90 U/L
Female: 10-70 U/L 10-70 U/L
* Creatinine, serum 0.6-1.2 mg/dL 53-106 mol/L
Electrolytes, serum
Sodium 135-145 mEq/L 136-145 mmol/L
Chloride 95-105 mEq/L 95-105 mmol/L
* Potassium 3.5-5.0 mEq/L 3.5-5.0 mmol/L
Bicarbonate 22-28 mEq/L 22-28 mmol/L
Magnesium (mg 2+) 1.5-2.0 mEq/L 1.5-2.0 mmol/L
Estriol (E3) total, serum (in pregnancy)

24-28 wks // 32-36 wks
30-170 ng/mL // 60-280
ng/mL
104-590 // 208-970
nmol/L

28-32 wks // 36-40 wks
40-220 ng/mL // 80-350
ng/mL
140-760 // 280-1210
nmol/L
Ferritin, serum Male: 15-200 ng/mL 15-200 g/L
Female: 12-150 ng/mL 12-150 g/L
Follicle-stimulating hormone,
serum/plasma
Male: 4-25 mIU/mL 4-25 U/L


Female: -- premenopause 4-
30 mIU/mL
4-30 U/L


-- midcycle peak 10-90
mIU/mL
10-90 U/L


--postmenopause 40-250
mIU/mL
40-250 U/L
Gases, arterial blood (room air)
pH 7.35-7.45 [H
+
] 36-44 nmol/L
PCO
2
33-45 mm Hg 4.4-5.9 kPa
PO
2
75-105 mm Hg 10.0-14.0 kPa
Glucose, serum Fasting: 70-110 mg/dL 3.8-6.1 mmol / L


2-h postprandial: < 120
mg/dL
< 6.6 mmol/L
Growth hormone - arginine
stimulation
Fasting: < 5 ng/mL < 5 g/L
provocative stimuli: 7 ng/mL > 7 g/L
Immunoglobulins, serum
IgA 76-390 mg/dL 0.76-3.90 g/L
IgE 0-380 IU/mL 0-380 kIU/mL
IgG 650-1500 mg/dL 6.5-15 g/L
IgM 40-345 mg/dL 0.4-3.45 g/L
Iron 50-170 g/dL 9-30 mol/L
Lactate dehydrogenase, serum 45-90 U/L 45-90 U/L
Luteinizing hormone, serum/plasma Male: 6-23 mIU/mL 6-23 U/L


Female: --follicular phase 5-
30 mIU/mL
5-30 U/L
--midcycle 75-150 mIU/mL 75-150 U/L


--postmenopause 30-200
mIU/mL
30-200 U/L
Osmolality, serum 275-295 mOsmol/kg 275-295 mOsmol/kg
Parathyroid hormone, serum, N-
terminal
230-630 pg/mL 230-630 ng/L
Phosphatase (alkaline), serum (p-
NPP at 30C)
20-70 U/L 20-70 U/L
Phosphorus (inorganic), serum 3.0-4.5 mg/dL 1.0-1.5 mmol/L
Prolactin, serum (hPRL) < 20 ng/mL < 20 g/L
Proteins, serum
Total (recumbent) 6.0-7.8 g/dL 60-78 g/L
Albumin 3.5-5.5 g/dL 35-55 g/L
Globulins 2.3-3.5 g/dL 23-35 g/L
Thyroid-stimulating hormone, serum
or plasma
0.5-5.0 U/mL 0.5-5.0 mU/L

Thyroidal iodine (
123
I) uptake
8-30% of administered
dose/24 h
0.08-0.30/24 h
Thyroxine (T
4
), serum 5-12 g/dL 64-155 nmol/L
Triglycerides, serum 35-160 mg/dL 0.4-1.81 mmol/L
Triiodothyronine (T
3
), serum (RIA) 115-190 ng/dL 1.8-2.9 nmol/L
Triiodothyronine (T
3
) resin uptake 25-35% 0.25-0.35
Urea nitrogen, serum (BUN) 7-18 mg/dL 1.2-3.0 mmol urea/L
Uric acid, serum 3.0-8.2 mg/dL 0.18-0.48 mmol/L

CEREBROSPINAL FLUID
Cell count 0-5 cells/mm
3
0-5 x 10
6
/L
Chloride 118-132 mEq/L 118-132 mmol/L
Gamma globulin 3-12% total proteins 0.03-0.12
Glucose 40-70 mg/dL 2.2-3.9 mmol/L
Pressure 70-180 mm H
2
O 70-180 mm H
2
O
Proteins, total < 40 mg/dL < 0.40 g/L

HEMATOLOGIC
Bleeding time (template) 2-7 minutes 2-7 minutes
Erythrocyte count Male: 4.3-5.9 million/mm
3
4.3-5.9 x 10
12
/L
Female: 3.5-5.5 million/mm
3
3.5-5.5 x 10
12
/L
Erythrocyte sedimentation rate
(Westergren)
Male: 0-15 mm/h 0-15 mm/h
Female: 0-20 mm/h 0-20 mm/h
Hematocrit Male: 41-53% 0.41-0.53
Female: 36-46% 0.36-0.46
Hemoglobin A
1C
6% 0.06%
Hemoglobin, blood Male: 13.5-17.5 g/dL 2.09-2.71 mmol/L
Female: 12.0-16.0 g/dL 1.86-2.48 mmol/L
Hemoglobin, plasma 1-4 mg/dL 0.16-0.62 mol/L
Leukocyte count and differential
Leukocyte count 4500-11,000/mm
3
4.5-11.0 x 10
9
/L
Segmented neutrophils 54-62% 0.54-0.62
Band forms 3-5% 0.03-0.05
Eosinophils 1-3% 0.01-0.03
Basophils 0-0.75% 0-0.0075
Lymphocytes 25-33% 0.25-0.33
Monocytes 3-7% 0.03-0.07
Mean corpuscular hemoglobin 25.4-34.6 pg/cell 0.39-0.54 fmol/cell
Mean corpuscular hemoglobin
concentration
31-36% Hb/cell 4.81-5.58 mmol Hb/L
Mean corpuscular volume .80-100 m
3
80-100 fl
Partial thromboplastin time
(activated)
25-40 seconds 25-40 seconds
Platelet count 150,000-400,000/mm
3
150-400 x 10
9
/L
Prothrombin time 11-15 seconds 11-15 seconds
Reticulocyte count 0.5-1.5% of red cells 0.005-0.015

Thrombin time
< 2 seconds deviation from
control
< 2 seconds deviation
from control
Volume
Plasma Male: 25-43 mL/kg 0.025-0.043 L/kg


Female: 28-45 mL/kg 0.028-0.045 L/kg
Red cell Male: 20-36 mL/kg 0.020-0.036 L/kg


Female: 19-31 mL/kg 0.019-0.031 L/kg

SWEAT
Chloride 0-35 mmol/L 0-35 mmol/L

URINE
Calcium 100-300 mg/24 h 2.5-7.5 mmol/24 h
Chloride Varies with intake Varies with intake
Creatinine clearance Male: 97-137 mL/min
Female: 88-128 mL/min
Estriol, total (in pregnancy)
30 wks 6-18 mg/24 h 21-62 mol/24 h
35 wks 9-28 mg/24 h 31-97 mol/24 h
40 wks 13-42 mg/24 h 45-146 mol/24 h
17-Hydroxycorticosteroids Male: 3.0-10.0 mg/24 h 8.2-27.6 mol/24 h
Female: 2.0-8.0 mg/24 h 5.5-22.0 mol/24 h
17-Ketosteroids, total Male: 8-20 mg/24 h 28-70 mol/24 h
Female: 6-15 mg/24 h 21-52 mol/24 h
Osmolality 50-1400 mOsmol/kg
Oxalate 8-40 g/mL 90-445 mol/L
Potassium Varies with diet Varies with diet
Proteins, total < 150 mg/24 h < 0.15 g/24 h
Sodium Varies with diet Varies with diet
Uric acid Varies with diet Varies with diet

You might also like